You are on page 1of 426

INSTRUCTIONS

1. This Test Booklet contains 100 items (questions). Each item comprises four responses (answers). You will select
the response which you want to mark on the Answer sheet. In case, you feel that there is more than one correct
response, mark the response which you consider the best. In any case, choose ONLY ONE response for each item.
2. All items carry equal marks.
3. Penalty for wrong answers:
THERE WILL BE PENALTY FOR WRONG ANSWERS MARKED BY THE CANDIDATE FOR WRONG ANSWER.
(i) There are four alternatives for the answer to every question. For each question which has a penalty for which a
wrong answer has been given by the candidate, one-third of the marks assigned to that question will be deducted
as penalty.
(ii) If a candidate gives more than one answer, it will be treated as a wrong answer even if one of the given answers
happens to be correct and there will be same penalty as above to that question.
(iii) If a question is left blank, i.e., no answer is given by the candidate, there will be no penalty for that question.

1. With reference to the Cabinet Mission, which of the following 1. A very strong centralized political authority and a very
statements is/are correct? weak provincial or local political authority
1. It recommended government a federal government. 2. Emergence of administrative structure based on control
2. It enlarged the powers of the Indian Courts. and possession of land
3. It provided for more Indians in the ICS. 3. Creation of lord-vassal relationship between the feudal
Select the correct answer using the code given below. lord and his overlord
(a) 1 only (b) 2 and 3 Select the correct answer using the code given below.
(c) 1 and 3 (d) None (a) 1 and 2 only (b) 2 and 3 only
2. Which one of the following National Parks has a climate that (c) 3 only (d) 1, 2 and 3
varies from tropical to subtropical, temperate and arctic? 6. 'BioCarbon Fund Initiative for Sustain able Forest
(a) Khangchendzonga National Park Landscapes' is managed by the
(b) Nandadevi National Park (a) Asian Development Bank
(b) International Monetary Fund
(c) Neora Valley National Park
(c) United Nations Environment Programme
(d) Namdapha National Park
(d) World Bank
3. Amnesty International is
7. India is a member of which among the following?
(a) an agency of the United Nations to help refugees of
1. Asia-Pacific Economic Cooperation
civil wars
2. Association of South-East Asian Nations
(b) a global Human Rights Movement
3. East Asia Summit
(c) a non-governmental organization to help people
Select the correct answer using the code given below.
voluntary very poor people
(a) 1 and 2 only
(d) an inter-governmental agency to cater to medical
(b) 3 only
emergencies in war-ravaged regions
(c) 1, 2 and 3
4. With reference to the art and archaeo-logical history of India,
(d) India is a member of none of them
which one among the following was made earliest?
8. In India, the steel production industry requires the import
(a) Lingaraja Temple at Bhubneshwar of
(b) Rock-cut Elephant at Dhauli (a) saltpetre (b) rock phosphate
(c) Rock-cut Monuments at Mahabalipuram (c) coking coal (d) All of the above
(d) varaha Image at Udayagiri 9. The provisions in Fifth Schedule and Sixth Schedule in the
5. With reference to Indian history, which of the following is/ Constitution of India are made in order to
are the essential elementy elements of the feudal system? (a) protect the interests of Scheduled Tribes
(b) determine the boundaries between States and economic stress and improve risk management
(c) determine the powers, authority and responsibilities (c) reduce the greenhouse gas emissions but places a
of Panchayats heavier burden on developed countries
(d) protect the interests of all the border States (d) transfer technology from developed Countries to poor
10. With reference to the Union Government, consider the countries to enable them to replace the use of
following statements : chlorofluorocarbons in refrigeration with harmless
chemicals
1. The Department of Revenue is responsible for the
16. Consider the following statements
preparation of Union Budget that is presented to the
1. The winds which blow between 30° N and 60° S
Parliament.
latitudes throughout the year are known as westerlies.
2. No amount can be withdrawn from the Consolidated 2. The moist air masses that cause winter rains in North-
Fund of India without the authorization from the Western region of India are part of westerlies.
Parliament of India. Which of the statements given above is/are correct?
3. All the disbursements made from Public Account also (a) 1 only (b) 2 only
need the authorization from the Parliament of India. (c) Both 1 and 2 (d) Neither 1 nor 2
Which of the statements given above is / are correct? 17. With reference to 'Indian Ocean Rim Association for Regional
(a) 1 and 2 only Cooperation (IOR-ARC)', consider the following statements:
(b) 2 and 3 only 1. It was established very recently in response to incidents
(c) 2 only of piracy and accidents of oil spills.
(d) 1, 2 and 3 2. It is an alliance meant for maritime security only.
11. Who /Which of the following is the custodian of the Which of the statements given above is / are correct?
Constitution of India? (a) 1 only (b) 2 only
(a) The President of India (c) Both 1 and 2 (d) Neither 1 nor 2
(b) The Prime Minister of India 18. Which one of the following movements has contributed to
(c) The Lok Sabha Secretariat a split in the Indian National Congress resulting in the
(d) The Supreme court of India emergence of 'moderates' and 'extremists'?
12. Consider the following statements: (a) Swadeshi Movement
1. The Accelerated Irrigation Benefits Programme was (b) Quit India Movement
launched during 1996-97 to provide loan assistance to (c) Non-Cooperation Movement
poor fanners.
(d) Civil Disobedience Movement
2. The Command Area Development Programme was
19. In a particular region in India, the local people train the
launched in 1974-75 for the development of water-use
roots of living trees into robust bridges across the streams.
efficiency.
As the time passes, these bridges become stronger. These
Which of the statements given above is/are correct?
unique 'living root bridges' are found in
(a) 1 only (b) 2 only
(a) Meghalaya (b) Himachal Pradesh
(c) Both 1 and 2 (d) Neither 1 nor 2
(c) Jharkhand (d) Tamil Nadu
13. The Genetic Engineering Appraisal Committee is constituted
under the 20. Tides occur in the oceans and seas due to which among the
(a) Food Safety and Standards Act, 2006 following?
(b) Geographical Indications of Goods (Registration and 1. Gravitational force of the Sun
Protection) Act, 1999 2. Gravitational force of the Moon
(c) Environment (Protection) Act, 1986 3. Centrifugal force of the Earth
(d) Wildlife (Protection) Act, 1972 Select the correct answer using the code given below.
14. In the Mekong-Ganga Cooperation, an initiative of six (a) 1 only (b) 2 and 3 only
countries, which of the following is/are not a participant/ (c) 1 and 3 only (d) 1, 2 and 3
participants? 21. In which of the following activities are Indian Remote
1. Bangladesh Sensing (IRS) satellites used?
2. Cambodia 1. Assessment of crop productivity
3. China 2. Locating groundwater resources
4. Myanmar 3. Mineral exploration
5. Thailand 4. Telecommunications
Select the correct answer using the code given below. 5. Traffic studies
(a) 1 only (b) 2, 3 and 4 Select the correct answer using the code given below.
(c) 1 and 3 (d) 1, 2 and 5 (a) 1, 2 and 3 only (b) 4 and 5 only
15. 'Basel III Accord' or simply 'Basel III', often seen in the news, (c) 1 and 2 only (d) 1, 2, 3, 4 and 5
seeks to 22. Consider the following States
(a) develop national strategies for the conservation and 1. Arunachal Pradesh
sustainable use of biological diversity 2. Himachal Pradesh
(b) improve banking sector's ability to deal with financial
3. Mizoram the news, is
In which of the above States do 'Tropical Wet Evergreen (a) a strategy to tackle the regional terrorism, an outcome
Forests' occur? of a meeting of the Shanghai Cooperation Organization
(a) 1 only (b) 2 and 3 only (b) a plan of action for sustainable economic growth in the
(c) 1 and 3 only (d) 1, 2 and 3 Asia-Pacific Region, an outcome of the deliberations
23. The term 'IndARC', sometimes seen in the news, is the name of the Asia-Pacific Economic Forum
of (c) an agenda for women's empowerment, an outcome of a
(a) an indigenously developed radar system inducted into World Conference convened by the United Nations
Indian Defence (d) a strategy to combat wildlife trafficking, a declaration
(b) India's satellite to provide services to the countries of of the East Asia Summit
Indian Ocean Rim 30. "Each day is more or less the same, the morning is clear and
(c) a scientific establishment set up by India in Antarctic bright with a sea breeze; as the Sun climbs high in the sky,
region heat mounts up, dark clouds form, then rain comes with
(d) India's underwater observatory to scientifically study thunder and lightning. But rain is soon over." Which of the
the Arctic region following regions is described in the above passage?
24. With reference to 'Forest Carbon Partnership Facility', which (a) Savannah (b) Equatorial
of the following statements is/ are correct? (c) Monsoon (d) Mediterranean
1. It is a global partnership of governments, businesses, 31. With reference to Indian economy, consider the following
civil society and indigenous peoples. statements:
2. It provides financial aid to universities, individual 1. The rate of growth of Real Gross Domestic Product
scientists and institutions involved in scientific forestry has steadily increased in the last decade.
research to develop eco-friendly and climate adaptation 2. The Gross Domestic Product at market prices (in
technologies for sustainable forest management. rupees) has steadily increased in the last decade.
3. It assists the countries in their 'REDD+ (Reducing Which of the statements given above is/are correct?
Emissions from Deforestation and Forest
(a) 1 only (b) 2 only
Degradation+)' efforts by providing them with financial
(c) Both 1 and 2 (d) Neither 1 nor 2
and technical assistance.
32. Consider the following statements :
Select the correct answer using the code given below.
1 The Legislative Council of a State in India can be larger
(a) 1 only (b) 2 and 3 only
(c) 1 and 3 only (d) 1, 2 and 3 in size than half of the Legislative Assembly of that
25. Which one of the following was given classical language particular State
status recently? 2. The Governor of a State nominates the Chairman of
(a) Odia (b) Konkani Legislative Council of that particular State.
(c) Bhojpuri (d) Assamese Which of the statements given above is/are correct?
26. With reference to an organization known as 'Birdlife (a) 1 only (b) 2 only
International', which of the following statements is/are (c) Both 1 and 2 (d) Neither 1 nor 2
correct? 33. "To uphold and protect the Sovereignty, Unity and Integrity
1. It is a Global Partnership of Conservation of India" is a provision made in the
Organizations. (a) Preamble of the Constitution
2. The concept of 'biodiversity hotspots' originated from (b) Directive Principles of State Policy
this organization. (c) Fundamental Rights
3. It identifies the sites known/referred to as 'Important (d) Fundamental Duties
Bird and Biodiversity Areas'. 34. Which one of the following is the best description of the
Select the correct answer using the code given below. term 'ecosystem'?
(a) 1 only (b) 2 and 3 only (a) A community of organisms interacting with one another.
(c) 1 and 3 only (d) 1, 2 and 3 (b) That part of the Earth which is inhabited by living
27. Which one of the following countries of South-West Asia organisms.
does not open out to the Mediterranean Sea? (c) A community of organisms together with the
(a) Syria (b) Jordan environment in which they live
(c) Lebanon (d) lsrael
(d) The flora and fauna of a geographical area
28. In India, in which one of the following types of forests is
35. The fundamental object of Panchayati Raj system is to
teak a dominant tree species?
ensure which among the following?
(a) Tropical moist deciduous forest
1. People's participation in development
(b) Tropical rain forest
(c) Tropical thorn scrub forest 2. Political accountability
(d) Temperate forest with grasslands 3. Democratic decentralization
29. 'Beijing Declaration and Platform for Action', often seen in 4. Financial mobilization
Select the correct answer using the code given below. (a) AIDS (b) Bird flu
(a) 1, 2 and 3 only (b) 2 and 4 only (c) Dengue (d) Swine flu
(c) 1 and 3 only (d) 1, 2, 3 and 4 43. With reference to bio-toilets used by the Indian Railways,
36. With reference to Indian economy, consider the following consider the following statements:
1. Bank rate 1. The decomposition of human waste in the bio-toilets
2. Open market operations is initiated by a fungal inoculum.
3. Public debt 2. Ammonia and water vapour are the only end products
4. Public revenue in this decomposition which are released into the
Which of the above is/are component/ components of atmosphere.
Monetary Policy? Which of the statements given above is/are correct?
(a) 1 only (b) 2, 3 and 4 (a) 1 only (b) 2 only
(c) 1 and 2 (d) 1, 3 and 4 (c) Both 1 and 2 (d) Neither 1 nor 2
37. With reference to inflation in India, which of the following 44. The problem of international liquidity is related to the non-
statements is correct? availability of
(a) Controlling the inflation in India is the responsibility (a) goods and services
of the Government of India only (b) gold and silver
(b) The Reserve Bank of India has no role in controlling (e) dollars and other hard currencies
the inflation (d) exportable surplus
(c) Decreased money circulation helps in controlling the 45. With reference to 'fuel cells' in which hydrogen-rich fuel
inflation and oxygen are used to generate electricity. Consider the
(d) Increased money circulation helps in controlling the following statements :
inflation 1. If pure hydrogen is used as a fuel, the fuel cell emits
38. Consider the following countries heat and water as by-products.
1. China 2. Fuel cells can be used for powering buildings and not
for small devices like laptop computers.
2. France
3. Fuel cells produce electricity in the form of Alternating
3. India
Current (AC).
4. Israel
Which of the statements given above is / are correct?
5. Pakistan
(a) 1 only (b) 2 and 3 only
Which among the above are Nuclear Weapons States as (c) 1 and 3 only (d) 1, 2 and 3
recognized by the Treaty on the Non-Proliferation of Nuclear 46. Kalamkari painting refers to
Weapons, commonly known as Nuclear Non-Proliferation (a) a hand-painted cotton textile in South India
Treaty (NPT)? (b) a handmade drawing on bamboo handicrafts in North-
(a) 1 and 2 only (b) 1, 3, 4 and 5 only East India
(c) 2, 4 and 5 only (d) 1, 2, 3, 4 and 5 (e) a block-painted woollen cloth in Western Himalayan
39. The ideal of Welfare State' in the Indian Constitution is region of India
enshrined in its (d) a hand-painted decorative silk cloth in North-Western
(a) Preamble India
(b) Directive Principles of State Policy 47. Which one of the following best describes the main objective
(c) Fundamental Rights of 'Seed Village Concept'?
(d) Seventh Schedule (a) Encouraging the farmers to use their own farm seeds
40. The substitution of steel for wooden ploughs in agricultural and discouraging them to buy the seeds from others
production is an example of (b) Involving the farmers for training in quality seed pro
(a) labour-augmenting technological progress duction and thereby to make available quality seeds to
(b) capital-augmenting technological progress others at appropriate time and affordable cost
(c) capital-reducing technological progress (c) Earmarking some villages exclusively for the production
(d) None of the above of certified seeds
41. There is a Parliamentary System of Government in India (d) Identifying the entrepreneurs in village and providing
because the them technology and finance to set up seed companies
48. There has been a persistent deficit budget year after year.
(a) Lok Sabha is elected directly by the people
Which of the following actions can be taken by the
(b) Parliament can amend the Constitution
government to reduce the deficit?
(c) Rajya Sabha cannot be dissolved
1. Reducing revenue expenditure
(d) Council of Ministers is responsible to the Lok Sabha
2. Introducing new welfare schemes
42. H1N1 virus is sometimes mentioned in the news with 3. Rationalizing subsidies
reference to which one of the following diseases? 4. Expanding industries
Select the correct answer using the code given below. (c) Assam and Gujarat
(a) 1 and 3 only (b) 2 and 3 only (d) Arunachal Pradesh and Gujarat
(c) 1 only (d) 1,2,3 and 4 57. Consider the following statements regarding the Directive
49. Which of the following has/have been accorded Principles of State Policy:
'Geographical Indication' status? 1. The Principles spell out the socio-economic democracy
1. Banaras Brocades and Sarees in the country.
2. Rajasthani Daal-Bati-Churma 2. The provisions contained in these Principles are not
3. Tirupathi Laddu enforceable by any court.
Select the correct answer using the code given below. Which of the statements given above is / are correct?
(a) 1 only (b) 2 and 3 only (a) 1 only (b) 2 only
(c) 1 only 3 only (d) 1, 2 and 3 (c) Both 1 and 2 (d) Neither 1 nor 2
50. With reference to the Indian Renewable Energy Development 58. In the 'Index of Eight Core Industries', which one of the
Agency Limited (IREDA), which of the following statements following is given the highest weight?
is/are correct? (a) Coal production
1. It is a Public Limited Government Company. (b) Electricity generation
2. It is a Non - Banking Financial Company. (c) Fertilizer production
Select the correct answer using the code given below. (d) Steel production
(a) 1 only (b) 2 only 59. Which of the following National Parks is unique in being a
(c) Both 1 and 2 (d) Neither 1 or 2 swamp with floating vegetation that supports a rich
51. 'Pradhan Mantri Jan-Dhan Yojana' has been launched for biodiversity?
(a) providing housing loan to poor people at cheaper (a) Bhitarkanika National Park
interest rates
(b) Keibul Lamjao National Park
(b) promoting women's Self-Help Groups in backward areas
(c) Keoladeo Ghana National Park
(c) promoting financial inclusion in the country
(d) Sultanpur National Park
(d) providing financial help to the marginalized
communities 60. Which of the following statements is/are correct regarding
52. With reference to the Fourteenth Finance Commission, which National Innovation Foundation-India (NIF)?
of the following statements is/ are correct? 1. NIF is an autonomous body of the Department of
1. It has increased the share of States in the central Science and Technology under the Central Government.
divisible pool from 32 percent to 42 percent. 2. NIF is an initiative to strengthen the highly advanced
2. It has made recommendations concerning sector- scientific research in India's premier scientific
specific grants. institutions in collaboration with highly advanced
Select the correct answer using the code given below. foreign scientific institutions.
(a) 1 only (b) 2 only Select the correct answer using the code given below.
(c) Both 1 and 2 (d) Neither 1 nor 2 (a) 1 only (b) 2 only
53. The 'Fortaleza Declaration', recently in the news, is related (c) Both 1 and 2 (d) Neither 1 nor 2
to the affairs of 61. What can be the impact of excessive / in appropriate use of
(a) ASEAN (b) BRICS nitrogenous fertilizers in agriculture?
(c) OECD (d) WTO 1. Proliferation of nitrogen-fixing microorganisms in soil
54. A decrease in tax to GDP ratio of a country indicates which
can occur.
of the following?
2. Increase in the acidity of soil can take place.
1. Slowing economic growth rate
3. Leaching of nitrate to the ground-water can occur.
2. Less equitable distribution of national income
Select the correct answer using the code given below. Select the correct answer using the code given below.
(a) 1 only (b) 2 only (a) 1 and 3 only (b) 2 only
(c) Both 1 and 2 (d) Neither 1 nor 2 (c) 2 and 3 only (d) 1, 2 and 3
55. In the South Atlantic and South-Eastern Pacific regions in 62. With reference to the International Union for Conservation of
tropical latitudes, cyclone does not originate. What is the Nature and Natural Resources (IUCN) and the Convention
reason? on International Trade in Endangered Species of Wild Fauna
(a) Sea surface temperatures are low and Flora (CITES), which of the following statements is/are
(b) Inter-Tropical Convergence Zone seldom occurs correct?
(c) Coriolis force is too weak 1. IUCN is an organ of the United Nations and CITES is
(d) Absence of land in those regions an international agreement between governments.
56. Which one of the following pairs of States of India indicates 2. IUCN runs thousands of field projects around the world
the eastern most and western most State? to better manage natural environments.
(a) Assam and Rajasthan 3. CITES is legally binding on the States that have joined
(b) Arunachal Pradesh and Rajasthan it, but this Convention does not take the place of
national laws. 1. It is a herbivorous marine animal.
Select the correct answer using the code given below. 2. It is found along the entire coast of India.
(a) 1 only (b) 2 and 3 only 3. It is given legal protection under Schedule I of the
(c) 1 and 3 only (d) 1, 2 and 3 Wildlife (Protection) Act, 1972.
63. The Fair and Remunerative Price (FRP) of sugarcane is Select the correct answer using the code given below.
approved by the (a) 1 and 2 (b) 2 only
(a) Cabinet Committee on Economic Affairs (c) 1 and 3 (d) 3 only
(b) Commission for Agricultural Costs and Prices 70. Who of the following was/were economic critic/ critics of
(c) Directorate of Marketing and Inspection, Ministry of colonialism in India?
Agriculture 1. Dadabhai Naoroji
2. G. Subramania Iyer
(d) Agricultural Produce Market Committee
3. R. C. Dutt
64. What explains the eastward flow of the equatorial counter-
Select the correct answer using the code given below.
current?
(a) 1 only (b) 1 and 2 only
(a) The Earth's rotation on its axis
(c) 2 and 3 only (d) 1, 2 and 3
(b) Convergence of the two equatorial currents
71. Which one of the following issues the 'Global Economic
(c) Difference in salinity of water
Prospects' report periodically?
(d) Occurrence of the belt of calm near the equator
(a) The Asian Development Bank
65. Consider the following pairs:
(b) The European Bank for Reconstruction and
Place of Pilgrimage Location
Development
1. Srisailam : Nallamala Hills
(c) The US Federal Reserve Bank
2. Omkareshwar : Satmala Hills
(d) The World Bank
3. Pushkar : Mahadeo Hills
72. When the Reserve Bank of India reduces the Statutory
Which of the above pairs is/are correctly matched?
Liquidity Ratio by 50 basis points, which of the following is
(a) 1 only (b) 2 and 3 only
likely to happen?
(c) 1 and 3 only (d) 1, 2 and 3
(a) India's GDP growth rate increases drastically
66. With reference to Rowlatt Satyagraha, which of the following
(b) Foreign Institutional Investors may bring more capital
statements is/ are correct?
into our country
1. The Rowlatt Act was based on the recommendations
(c) Scheduled Commercial Banks may cut their lending
of the 'Sedition Committee'.
rates
2. In Rowlatt Satyagraha, Gandhiji tried to utilize the Home
(d) It may drastically reduce the liquidity to the banking
Rule League.
system
3. Demonstrations against the arrival of Simon
73. With reference to the use of nano-technology in health sector,
Commission coincided with Rowlatt Satyagraha.
which of the following statements is/are correct?
Select the correct answer using the code given below.
1. Targeted drug delivery is made possible by
(a) 1 only (b) 1 and 2 only
nanotechnology.
(c) 2 and 3 only (d) 1, 2 and 3
2. Nanotechnology can largely contribute to gene therapy.
67. Among the following, which were frequently mentioned in
Select the correct answer using the code given below.
the news for the outbreak of Ebola virus recently?
(a) 1 only (b) 2 Only
(a) Syria and Jordan
(c) Both 1 and 2 (d) Neither 1 nor 2
(b) Guinea, Sierra Leone and Liberia
74. In India, markets in agricultural products are regulated under
(c) Philippines and Papua New Guinea
the
(d) Jamaica, Haiti and Surinam
(a) Essential Commodities Act, 1955
68. With reference to 'fly ash' produced by the power plants
(b) Agricultural Produce Market Committee Act enacted
using coal as fuel, which of the following statements is/are
by States
correct?
(c) Agricultural Produce (Grading and Marking) Act, 1937
1. Fly ash can be used in the production of bricks for
(d) Food Products Order, 1956 and Meat and Food
building construction.
Products Order, 1973
2. Fly ash can be used as a replacement for some of the
75. Which one of the following is the national aquatic animal of
Portland cement concrete.
India?
3. Fly ash is made up of silicon dioxide and calcium oxide
(a) Saltwater crocodile (b) Olive ridley turtle
only, and does not contain any toxic elements.
(c) Gangetic dolphin (d) Gharial
Select the correct answer using the code given below.
76. With reference to Congress Socialist Party, consider the
(a) 1 and 2 (b) 2 only
following statements:
(c) 1 and 3 (d) 3 only
1. It advocated the boycott of British goods and evasion
69. With reference to 'dugong', a mammal found in India, which
of taxes.
of the following statements is/an; correct?
2. It wanted to establish the dictatorship of proletariat.
3. It advocated separate electorate for minorities and (c) 1 and 3 only (d) 1, 2 and 3
oppressed classes. 83. The area known as 'Golan Heights' sometimes appears in
Which of the statements given above is / are correct? the news in the context of the events related to
(a) 1 and 2 only (b) 3 only (a) Central Asia (b) Middle East
(e) 1, 2 and 3 (d) None (c) South-East Asia (d) Central Africa
77. Consider the following statements: 84. Convertibility of rupee implies
1. The Rajya Sabha has no power either to reject or to (a) being able to convert rupee notes into gold
amend a Money Bill. (b) allowing the value of rupee to be fixed by market forces
2. The Rajya Sabha cannot vote on the Demands for (c) freely permitting the conversion of rupee to other
Grants. currencies and vice versa
3. The Rajya Sabha cannot discuss the Annual Financial (d) developing an international market for currencies in
Statement. India
Which of the statements given above is / are correct? 85. Consider the following pairs:
(a) 1 only (b) 1 and 2 only Medieval Indian State Present Region
(c) 2 and 3 only (d) 1, 2 and 3 1. Champaka : Central India
78. The Government of India Act of 1919 clearly defined 2. Durgara : Jammu
(a) the separation of power between the judiciary and the 3. Kuluta : Malabar
legislature Which of the above pairs is / are correctly matched?
(b) the jurisdiction of the central and provincial (a) 1 and 2 (b) 2 only
governments (c) 1 and 3 (d) 1 and 3 Only
(c) the powers of the Secretary of State for India and the 86. Consider the following rivers:
Viceroy 1. Vamsadhara
(d) None of the above 2. Indravati
79. Which of the following brings out the 'Consumer Price Index
3. Pranahita
Number for Industrial Workers'?
4. Pennar
(a) The Reserve Bank of India
(b) The Department of Economic Affairs Which of the above are tributaries of Godavari?
(c) The Labour Bureau (a) 1, 2 and 3 (b) 2, 3 and 4
(d) The Department of Personnel and Training (c) 1, 2 and4 (d) 2 and 3 only
80. In the context of modern scientific research, consider the 87. When a bill is referred to a joint sitting of both the Houses
following statements about 'IceCube', a particle detector of the Parliament, it has to be passed by
located at South Pole, which was recently in the news: (a) a simple majority of members present and voting
1. It is the world's largest neutrino detector, encompassing (b) three-fourths majority of members present and voting
a cubic kilometre of ice. (c) two-thirds majority of the Houses
2. It is a powerful telescope to search for dark matter. (d) absolute majority of the Houses
3. It is buried deep in the ice. 88. Which one of the following regions of India has a
Which of the statements given above is/are correct? combination of mangrove forest, evergreen forest and
(a) 1 only (b) 2 and 3 only
deciduous forest?
(c) 1 and 3 only (d) 1, 2 and 3
81. The terms 'Agreement on Agriculture', 'Agreement on the (a) North Coastal Andhra Pradesh
Application of Sanitary and Phytosanitary Measures' and (b) South-West Bengal
'Peace Clause' appear in the news frequently in the context (c) Southern Saurashtra
of the affairs of the (d) Andaman and Nicobar Islands
(a) Food and Agriculture Organization 89. Which of the following kingdoms were associated with the
(b) United Nations Framework Conference on Climate life of the Buddha?
Change 1. Avanti
(c) World Trade Organization 2. Gandhara
(d) United Nations Environment Programme 3. KosalAa
82. With reference to 'Near Field Communication (NFC) 4. Magadha
Technology', which of the following statements is/are Select the correct answer using the code given below.
correct? (a) 1, 2 and 3 (b) 2 and 3 only
1. It is a contactless communication technology that uses
(c) 1, 3 and 4 (d) 3 and 4 only
electromagnetic radio fields.
2. NFC is designed for use by devices which can be at a 90. Which one of the following is associated with the issue of
distance of even a metre from each other control and phasing out of the use of ozone-depleting
3. NFC can use encryption when sending sensitive substances?
information. (a) Bretton Woods Conference
Select the correct answer using the code given below. (b) Montreal Protocol
(a) 1 and 2 only (b) 3 Only
(c) Kyoto Protocol (b) regions inside the Earth where shale gas is available
(d) Nagoya Protocol (c) search for the Earth-like planets in outer space
91. Consider the following: (d) search for meteorites containing precious metals
The arrival of Babur into India led to the 96. Who of the following organized a march on the Tanjore
1. introduction of gunpowder in the subcontinent coast to break the Salt Law in April 1930?
2. introduction of the arch and dome in the region's (a) V. O. Chidambaram Pillai
architecture (b) C. Rajagopalachari
3. establishment of Timurid dynasty in the region (c) K. Kamaraj
Select the correct answer using the code given below. (d) Annie Besant
(a) 1 and 2 only (b) 3 only 97. Who of the following founded a new city on the south bank
of a tributary to river Krishna and undertook to rule his new
(c) 1 and 3 only (d) 1, 2 and 3
kingdom as the agent of a deity to whom all the land south
92. The Government of India has established NITI Aayog to
of the river Krishna was supposed to belong?
replace the
(a) Amoghavarsha I (b) Ballala II
(a) Human Rights Commission
(c) Harihara I (d) Prataparudra II
(b) Finance Commission
98. Consider the following statements
(c) Law Commission
1. The first woman President of the Indian National
(d) Planning Commission
Congress was Sarojini Naidu.
93. What is Rio+20 Conference, often mentioned in the news?
2. The first Muslim President of the Indian National
(a) It is the United Nations Conference on Sustainable
Congress was Badruddin Tyabji.
Development
Which of the statements given above is / are correct?
(b) It is a Ministerial Meeting of the World Trade
(a) 1 only (b) 2 only
Organization
(c) Both 1 and 2 (d) Neither 1 nor 2
(c) It is a Conference of the Inter-governmental Panel on
99. Which of the following statements regarding 'Green Climate
Climate Change
Fund' is/are correct?
(d) It is a Conference of the Member Countries of the
1. It is intended to assist the developing countries in
Convention on Biological Diversity
adaptation and mitigation practices to counter climate
94. Consider the following statements
change.
1. The Executive Power of the Union of India is vested in
2. It is founded under the aegis of UNEP, OECD, Asian
the Prime Minister.
Development Bank and World Bank.
2. The Prime Minister is the ex officio Chairman of the
Select the correct answer using the code given below.
Civil Services Board.
(a) 1 only (b) 2 Only
Which of the statements given above is / are correct?
(c) Both 1 and 2 (d) Neither 1 nor 2
(a) 1 only (b) 2 only
100. Indira Gandhi Prize for Peace, Disarmament and Development
(c) Both 1 and 2 (d) Neither 1 nor 2
for 2014 was given to which one of the following?
95. The term 'Goldilocks Zone' is often seen in the news in the
(a) Bhabha Atomic Research Centre
context of
(b) Indian Institute of Science
(a) the limits of habitable zone above the surface of the
(c) Indian Space Research Organization
Earth
(d) Tata Institute of Fundamental research
ANSWERS AND EXPLANATIONS
1. (a) Britain's Prime Minister Clement Attlee was announced receipts flowing to the Government in connection with
the dispatch of the Mission of three Cabinet Ministers the conduct of Government business i.e. Non-Tax
Pethic Lawrence, Sir Stafford Cripps, and A.V. Revenues are credited into the Consolidated Fund
Alexander. According to the proposals of the Cabinet constituted under Article 266 (1) of the Constitution of
Mission announced, the country could have a Federal India. No amount can be withdrawn from the Fund
government consisting of the Indian provinces and without authorization from the Parliament.
princely states. 11. (d) The Supreme Court of India is the apex court in India.
2. (d) Namdapha National Park is located in Arunachal As stated by the Indian Constitution, the function of
Pradesh. The climate of this area varies from tropical to the Supreme Court of India is that of a custodian of the
subtropical, temperate and arctic. It is tropical and Constitution, a court established by the authority of a
subtropical in southern regions and arctic type found federal government, and the uppermost court of appeal.
in northern part of the park. 12. (b) The Command Area development programme was
3. (b) Amnesty International is a non-governmental launched in 1974-75 as a centrally sponsored scheme
organisation focused on human rights. It was founded and its main objective was to improve the utilization of
in London in 1961 by the lawyer Peter Benenson. It the created irrigation potential and optimizing the
draws attention to human rights abuses and campaigns agricultural production and productivity from the
for compliance with international laws and standards. irrigated lands. The Accelerated Irrigation Benefit
4. (b) Dhauligiri located on the River Daya banks, at Programme (AIBP) was launched during 1996- 1997 to
Bhubaneswar in Odisha. It is believed that Dhauli hill give loan assistance to the States to help them complete
was the battlefield of the Kalinga War. The rock-cut some of the incomplete major/medium irrigation projects
elephant is the oldest Buddhist sculptures located here. which were at an advanced stage of completion and to
5. (b) A person became a vassal by pledging political create additional irrigation potential in the country.
allegiance and providing military, political, and financial 13. (c) The Genetic Engineering Appraisal Committee (GEAC)
service to a lord. A lord possessed complete sovereignty is the apex body constituted in the Ministry of
over land or acted in the service of another sovereign, Envir onment and Forests under 'Rules for
usually a king. If a lord acted in the service of a king, Manufacture, Use, Import, Export and Storage of
the lord was considered a vassal of the king. As part of Hazardous Microorganisms/Genetically Engineered
the feudal agreement, the lord promised to protect the Organisms or Cells 1989', under the Environment
vassal and provided the vassal with a plot of land. Protection Act, 1986. The Rules of 1989 also define five
This land could be passed on to the vassal's heirs, competent authorities i.e. the Institutional Biosafety
giving the vassal tenure over the land. Committees (IBSC), Review Committee of Genetic
6. (d) Housed within the Carbon Finance Unit of the World Manipulation (RCGM), Genetic Engineering Approval
Bank, the BioCarbon Fund is a public-private sector Committee (GEAC), State Biotechnology Coordination
initiative mobilizing financing to help for development Committee (SBCC) and District Level Committee (DLC)
of projects that sequester or conserve carbon in forest for handling of various aspects of the rules.
and agro-ecosystems. It was created in 2004. 14. (c) The Mekong-Ganga Cooperation (MGC) was
7. (b) At present, India is not a member of APEC and ASEAN. established on November 10, 2000 at Vientiane at the
The East Asia Summit (EAS) is a forum held annually First MGC Ministerial Meeting. It comprises six member
by leaders of 16 countries in the East Asian region. countries, namely India, Thailand, Myanmar, Cambodia,
The first summit was held in Kuala Lumpur (Malaysia) Laos and Vietnam. They emphasised four areas of
on 14 December 2005. India is the member of this forum. cooperation, which are tourism, culture, education, and
8. (c) The coal found in India is mainly of non-coking quality transportation linkage in order to be solid foundation
and hence coking coal has to be imported. 70% of the for future trade and investment cooperation in the
steel produced today uses coal. Coking coal is a vital region.
ingredient in the steel making process. 15. (b) Basel III is a comprehensive set of reform measures
9. (a) The Fifth Schedule of the Constitution of India deals which was developed by the Basel Committee on
with administration and control of scheduled areas and Banking Supervision and to strengthen the regulation,
scheduled tribes in these areas. The Sixth Schedule to supervision and risk management of the banking sector.
the Constitution of India contains provisions These measures aim to:
concerning the administration of tribal areas in the improve the banking sector's ability to absorb shocks
States of Assam, Meghalaya, Tripura and Mizoram. arising from financial and economic stress, whatever
10. (c) All revenues received by the Government by way of the source; improve risk management and governance;
taxes like Income Tax, Central Excise, Customs and other strengthen banks' transparency and disclosures.
16. (b) The Westerlies are prevailing winds from the west the eastern and western slopes of the Western Ghats
toward the east in the middle latitudes between 30 and in such states as Tamil Nadu, Karnataka, Kerala and
60 degrees latitude. They originate from the high- Maharashtra.
pressure areas in the horse latitudes and tend towards 23. (d) IndARC stands for India's underwater observatory to
the poles and steer extra tropical cyclones in this scientifically study the Arctic region. This is the
general manner. The moist air masses that cause winter country's first underwater moored observatory in the
rains in North-Western region of India are part of Kongsfjorden fjord, half way between Norway and the
westerlies. North Pole. It represents a major milestone in India's
17. (d) IOR-ARC is a regional cooperation initiative of the scientific endeavours in the Arctic region. It is
Indian Ocean Rim countries which was established in developed by Earth System Science Organisation
Mauritius in March 1997 with the aim of promoting (ESSO), National Centre for Antarctic and Ocean
economic and technical cooperation. IOR-ARC is the Research (NCAOR), National Institute of Ocean
only pan-Indian ocean grouping. It brings together Technology (NIOT) and Indian National Centre for
countries from three continents having different sizes, Ocean Information Services (INCOIS). IndARC was
economic strengths, and a wide diversity of languages, deployed from RV Lance which is a research vessel
cultures. It aims to create a platform for trade, socio- belonging to the Norwegian Polar Institute.
economic and cultural cooperation in the Indian Ocean 24. (c) The Forest Carbon Partnership Facility is a global
rim area, which constitutes a population of about two partnership of governments, businesses, civil society,
billion people. Presently it has 19 members- Australia, and Indigenous Peoples focused on reducing
Bangladesh, India, Indonesia, Iran, Kenya, Malaysia, emissions from deforestation and forest degradation,
Madagascar, Mauritius, Mozambique, Oman, forest carbon stock conservation, the sustainable
Seychelles, Singapore, South Africa, Sri Lanka, management of forests, and the enhancement of forest
Tanzania, Thailand, UAE and Yemen. carbon stocks in developing countries (activities
Six priority areas were identified to take forward the commonly referred to as REDD+).
cooperation under IOR-ARC: (i) Maritime Safety and 25. (a) Odia (Odiya) language on 20 February 2014 became
Security; (ii) Trade and Investment Facilitation; (iii) the sixth language of the country to get classical
Fisheries Management; (iv) Disaster Risk Reduction; language' status after the Union Cabinet approved it.
(v) Academic and S&T Cooperation; and (vi) Tourism Odia is billed as the first language from the Indo-Aryan
Promotion and Cultural Exchanges. linguistic group and the case for making it a classical
18. (a) Surat split is an important event in the modern history language was also premised on the fact that it has no
of India. It took place in 1907 when the moderates parted resemblance to Hindi, Sanskrit, Bengali and Telugu.
company with the Extremists. The split in the congress With this, Odia came into the same league as Sanskrit,
was due to many reasons. The Moderates had Tamil, Telugu, Kannada and Malayalam, which have
controlled the congress from its very beginning and already been conferred the classical language status.
had their own ways of thinking and doing which were 26. (c) BirdLife international is a global partnership of
not acceptable to the younger generations who were conservation organisations that strives to conserve
impatient with the speed at which the moderates were birds, their habitats and global biodiversity. It is
moving and leading the nation. This was the time of working with people towards sustainability in the use
Swadeshi Movement. of natural resources. It is the World's largest partnership
19. (a) In the present-day Meghalaya state of northeast India of conservation organisations, with over 120 partner
is a form of tree shaping, which creates these organizations. An Important Bird and Biodiversity Area
suspension bridges, they are handmade from the aerial (IBA) is an area recognized as being globally important
roots of living banyan fig trees. habitat for the conservation of bird populations. The
20. (d) Tides are the rise and fall of sea levels caused by the program was developed and sites are identified by
combined effects of gravitational forces exerted by the BirdLife International.
Moon, Sun, and rotation of the Earth. 27. (b) Jordan does not open out to the Mediterranean Sea.
21. (a) Indian Remote Sensing (IRS) satellites are used in 28. (a) The tropical moist deciduous forests are found in
Assessmen t of crop productivity, Locating Sahyadris, the north-eastern parts of the peninsula and
groundwater resources and Mineral exploration. This along the foothills of the Himalayas. Teak and sal are
system was launched in 1979 and 1981. This system is found in these forests.
used in agriculture, water resources, forestry and 29. (c) The Beijing Declaration and Platform for Action was
ecology, geology, marine fisheries and coastal adopted by the Fourth World Conference on Women
management. It is the largest constellation of the remote in 1995. The Platform for Action reaffirms the
sensing satellites. fundamental principle that the rights of women and
22. (c) Tropical Wet Evergreen Forests are found in Assam, girls are an "inalienable, integral and indivisible part of
Arunachal Pradesh, Meghalaya, Nagaland, Tripura, universal human rights." The Platform for Action also
West Bengal and Andaman and Nicobar Island and on calls upon governments to take action to address
several critical areas of concern, among them violence as long as they enjoy the support and confidence of
against women. the Lok Sabha.
30. (b) The passage points out equatorial region. 42. (d) H1N1 flu is also known as swine flu. The cause of
31. (c) Both statements are correct. swine flu is an influenza A virus type designated as
32. (d) The maximum strength of the legislative council is fixed H1N1.
at one third of the total strength of the legislative 43. (d) The bio-toilets are fitted underneath the lavatories and
assembly and the minimum strength is fixed at 40. The the human waste discharged into them is acted upon
chairman of the legislative council is elected by the by a particular kind of bacteria that converts it into
council itself from amongst its members. non-corrosive neutral water. The new-age green toilets
33. (d) The idea behind incorporation of fundamental duties have been designed by Railways along with Defence
was to remind the citizens of the country that they Research and Development Organisation (DRDO)
have certain obligations towards the country and keeping in mind the requirements of Indian trains.
society. The fundamental duties were added to the 44. (c) Dollars and hard currencies are used for payments in
constitution on the recommendations of the Swaran international trade, investment and liquidation of
Singh Committee. There were ten fundamental duties international debt. So if there is shortage of liquidity,
at the time of incorporation but the eleventh was this means scarcity of Dollars and hard currencies to
inserted by the 86th Amendment in 2002. To uphold carry on the above transactions. That is why even SDR
and protect the sovereignty, unity and integrity of India was introduced by IMF.
is one of them. 45. (a) When pure hydrogen is used as the fuel, the only by-
34. (a) An ecosystem includes all of the living things (plants, products generated from the fuel cell are pure water
animals and organisms) in a given area, interacting with and heat. Similar to a battery, a fuel cell with a supply of
each other, and also with their non-living environments hydrogen and oxygen can be used to power devices
(weather, earth, sun, soil, climate, atmosphere). that use electricity.
35. (a) The fundamental object of Panchayati Raj system is to 46. (a) Kalamkari painting is a type of hand-painted or block-
ensure people's participation in development, political printed cotton textile which are produced in parts of
accountability, and democratic decentralization. India and in Iran. The Machilipatnam Kalamkari craft
36. (c) The RBI implements the monetary policy through open made at Pedana nearby Machilipatnam in Krishna
market operations, bank rate policy, reserve system, district of Andhra Pradesh. It evolved with patronage
credit control policy, moral persuasion and through of the Mughals and the Golconda sultanate.
many other instruments. 47. (b) Seed village concept is to promote the quality seed
37. (c) When inflation becomes very high, the RBI decreases production of foundation and certified seed classes.
supply of money (to check inflation) by adopting light The area which is suitable for raising a particular crop
monetary policy. will be selected, and raised with single variety of a
38. (a) NPT designated nuclear weapon states are China, kind. Suitable area for seed production will be identified
France, Russia, United Kingdom and the United States. by the Scientists. The foundation/ certified seeds or
The NPT is a landmark international treaty whose University labelled seeds will be supplied by the
objective is to prevent the spread of nuclear weapons University through Krishi Vigyan Kendras (KVKs) and
and weapons technology and to promote cooperation Research Stations at 50% subsidy cost to the identified
in the peaceful uses of nuclear energy and to further farmers in the area. The farmers will use these quality
the goal of achieving nuclear disarmament and seeds and take up their own seed production in a small
complete disarmament. It is opened for signature in area (1 acre) for their own use. The crops are Rice,
1968 and the Treaty entered into force in 1970. Pulses and Oilseeds.
39. (b) DPSPs aim to create social and economic conditions 48. (a) Government can go for petroleum products
under which the citizens can lead a good life. They deregulation. Rationalizing subsidies is also the
also aim to establish social and economic democracy solution.
through a welfare state. The Directive Principles of State 49. (a) Banaras Brocades and Sarees (Logo) has been
Policy is guidelines/principles given to the central and accorded 'Geographical Indication' status. A
state governments of India, to be kept in mind while geographical indication (GI) is a name or sign used on
framing laws and policies. certain products which corresponds to a specific
40. (b) Capital-augmenting technological progress results in geographical location or origin (e.g. a town, region, or
the more productive use of existing capital goods; for country). India, as a member of the World Trade
example, the substitution of steel from wooden ploughs Organization (WTO), enacted the Geographical
in agricultural production. Indications of Goods (Registration and Protection) Act,
41. (d) The executive in a Parliamentary system is responsible 1999 which has come into force from 15 September 2003.
to the legislature for all its actions. The ministers are 50. (c) IREDA is a Public Limited Government Company which
answerable to the parliament and responsible to the was established as a Non-Banking Financial Institution
Lok Sabha. The Council of Ministers remains in office in 1987 under the administrative control of MNRE to
promote, develop and extend financial assistance for 63. (a) The Fair and Remunerative Price (FRP) of sugarcane is
renewable energy and energy efficiency /conservation approved by Cabinet Committee on Economic Affairs.
projects with the motto: "ENERGY FOR EVER" 64. (a) The Earth's rotation on its axis explains the eastward
51. (c) Pradhan Mantri Jan-Dhan Yojana (PMJDY) is National flow of the equatorial counter-current. The earth would
Mission for Financial Inclusion to ensure access to have been rotating east to west, the piled up water
financial services, namely, Banking/ Savings & Deposit would have come down on the west side. Therefore,
Accounts, Remittance, Credit, Insurance, Pension in essentially it's the earth's rotation that explains the
an affordable manner. eastward flow of equatorial counter current.
52. (a) Union Government accepted the 14th Finance 65. (a) The town of Srisailam is reputed for the shrine of Lord
Commission's recommendation to devolve an Mallikarjuna on the flat top of Nallamala Hills.
unprecedented 42 per cent of the divisible pool to states Omkareshwar is one of the Dwadasa jyotirlingas
during 20015-16 to 2019-20, against 32 per cent situated on the Mandhata hills in Vindhya Mountains
suggested by the previous commission. of Madhya Pradesh. Pushkar is located along Ratnagiri
53. (b) Fortaleza declaration was part of 6th BRICS summit Hills in Rajasthan.
which resulted in the official inauguration of the New 66. (b) The Rowlatt committee was a Sedition Committee
Development Bank. appointed in 1918 by the British Indian Government
54. (b) The tax-GDP ratio is calculated on nominal size of the with Mr Justice Rowlatt, an English judge, as its
economy. So if the economy grows by around 9 per president. The purpose of the committee was to
cent and inflation by 10 per cent, nominal GDP would evaluate political terrorism in India, especially Bengal
be 19 per cent, making it difficult to raise the tax-GDP and Punjab. In organizing his satyagraha Gandhi tried
ratio. A low tax to GDP ratio indicates lower economic to utilize three types of political networks - The Home
development and less equitable distribution of wealth. Rule Leagues, Certain pan islamist groups and a
55. (a) In the South Atlantic and South-Eastern Pacific regions satyagraha sabha which he himself started in Bombay
in tropical latitudes, cyclone does not originate because on 24 February.
sea surface temperatures are low. 67. (b) Guinea, Sierra Leone and Liberia were in news for the
56. (d) Arunachal Pradesh and Gujarat are the easternmost outbreak of Ebola virus recently.
and westernmost States. 68. (a) Fly ash brick (FAB) is a building material, specifically
57. (c) The directive principles ensure that the State shall strive masonry units, containing class C fly ash and water.
to promote the welfare of the people by promoting a All fly ash includes substantial amounts of silicon
social order in which social, economic and political dioxide (SiO2) aluminum oxide (Al2O3) and calcium
justice is informed in all institutions of life. The oxide (CaO), the main mineral compunds in coal-bearing
provisions of The Directive Principles of State Policy rock strata. Fly ash can be used as a replacement for
are not enforceable by any court, but the principles some of the Portland cement contents of concrete.
laid down therein are considered fundamental in the 69. (c) The dugong is a medium-sized marine mammal. Dugong
governance of the country, making it the duty of the is listed under schedule 1 of India Wildlife Protection
State to apply these principles in making laws to Act, 1972. In 2008, a MoU was signed between the
establish a just society in the count. Ministry of Environment and Forests and the
58. (b) Electricity generation (weight: 10.32%) increased by Government of India, in order to conserve dugongs. In
3.5 % in July, 2015. Its cumulative index during April to fact the highest level of legal protection is accorded to
July, 2015-16 increased by 2.0 % over the corresponding dugongs in India.
period of previous year. 70. (d) Dadabhai Naoroji, R. C. Dutt, Ranade, Gokhale, G.
59. (b) The Keibul Lamjao National Park is a national park Subramania Iyer, were among those who grounded
located in Manipur. It is 40 km in area and the only Indian nationalism firmly on the foundation of anti-
floating park in the world which is located in North imperialism by fashioning the world's first economic
East India, and an integral part of Loktak Lake. critique of colonialism, before Hobson and Lenin.
60. (a) National Innovation Foundation India (NIF) is an 71. (d) The World Bank issues the 'Global Economic Prospects'
autonomous body under the Department of Science report periodically.
and Technology , Government of India. It was set up in 72. (c) When the Reserve Bank of India reduces the Statutory
February 2000 at Ahmadabad, Gujarat, India to provide Liquidity Ratio by 50 basis points; the Scheduled
institutional support for scouting, spawning, Commercial Banks may cut their lending rates.
sustaining and scaling up the grassroots innovations 73. (c) Targeted drug delivery is made possible by
across the country. nanotechnology. Medical nanotechnology can largely
61. (c) Excessive/ inappropriate use of nitrogenous fertilizers contribute to genetic therapy and improvement.
increase the acidity of soil and Leaching of nitrate to Diseases can be easily treated if approached at the
the ground water. genetic level. So instead of treating diseases based on
62. (b) IUCN is not an organ of UN. It has observer and the symptoms, nanotechnology will help medical
consultative status at the United Nations. practitioners treat the problem by looking at the root
cause. 84. (c) Convertibility of rupee implies freely permitting the
74. (b) Agricultural Produce Market Committee Acts of conversion of rupee to other currencies and vice versa.
respective states are responsible for markets in Currency Convertibility is the ease with which a
agricultural products. country's currency can be converted into gold or
75. (c) The Gangetic dolphins have been declared as the another currency.
National Aquatic Animal of India .River Dolphin is the 85. (b) States such as Champaka (Chamba), Durgara (Jammu),
National Aquatic Animal of India. The Ministry of Trigarta (Jalandhar), Kuluta (Kulu), Kumaon and
Environment and Forests notified the Ganges River Garhwal managed to remain outside the main areas of
Dolphin as the National Aquatic Animal on 18th May conflict in the northern plains.
2010. This mammal is also said to represent the purity 86. (d) Major tributaries of the river include the Purna (South),
of the holy Ganga as it can only survive in pure and Pravara, Indravati, Manjira River, Bindusara River,
fresh water. Sabari River, Wainganga, and Wardha River. Pranahita
76. (d) All statements are wrong. is the name given to the combined flow of the rivers
77. (b) A Money Bill cannot be introduced in Rajya Sabha. Wardha and Wainganga.
Rajya Sabha has no power either to reject or amend a 87. (a) In India, if a bill has been rejected by any house of the
Money Bill. It can only make recommendations on the parliament and if more than six months have elapsed,
Money Bill. It has no power to vote on the Demands the President may summon a joint session for purpose
for Grants. of passing the bill. The bill is passed by a simple majority
78. (b) The Montagu-Chelmsford Reforms which became the of a joint sitting.
Government of India Act in 1919 clearly defined the 88. (d) Andaman and Nicobar Islands is rich in biodiversity.
jurisdiction of the central and provincial governments. Different types of forests as Tropical Evergreen forests,
79. (c) The labour Bureau brings out consumer price index Moist Deciduous forests, Mangrove forests, Littoral
numbers. forests are found here.
80. (d) IceCube is an enormous particle detector located deep 89. (c) Four great monarchies in the time of the Buddha are
in the ice at the South Pole. IceCube is the world's Avanti, Magadha, Kosala and Vamsa (or Vatsa)
largest neutrino detector, encompassing a cubic 90. (b) The Montreal Protocol is an international treaty
kilometer of ice. The IceCube telescope is a powerful designed to protect the ozone layer by phasing out the
tool to search for dark matter and could reveal the production of numerous substances that ar e
physical processes associated with the enigmatic origin responsible for ozone depletion.
of the highest energy particles in nature. Buried deep 91. (b) In the fourteenth and fifteenth century gun powder
within the East Antarctic ice sheet at the South Pole, a was used for military purpose in India and even artillery
giant high-energy neutrino observatory due for in its rudimentary form was known. The Babur's first
completion in 2009 could provide scientists, including real expedition took place in India in 1519 when he
from Europe, with an unprecedented window to the captured Bhera. The Delhi Sultanate's greatest
Universe, as well as a means to answer some of the contribution to Indian fine arts was the introduction of
most fundamental questions of astrophysics and Islamic architectural features, including true domes and
cosmology. arches, and the integration of Indian and Islamic styles
81. (c) 'Agreement on Agriculture', 'Agreement on the of architecture. The Timurid dynasty was a Sunni
Application of Sanitary and Phytosanitary Measures' Muslim dynasty. Members of the Timurid dynasty were
and Peace Clause' are related to World Trade strongly influenced by the Persian culture and had
Organization. established two well-known empires in history, namely
82. (c) Near field communication utilizes electromagnetic radio the Timurid Empire (1370-1507) in Persia and Central
fields while technologies such as Bluetooth and Wi-Fi Asia and the Mughal Empire (1526-1857) in the Indian
focus on radio transmissions instead. Near Field subcontinent.
Communication is an offshoot of Radio-Frequency 92. (d) The Government of India has established NITI Aayog
Identification (RFID) with the exception that NFC is to replace the Planning Commission. The Union
designed for use by devices within close proximity to Government of India announced formation of NITI
each other. When a secure channel is established, the Aayog on 1 January 2015 and the first meeting of NITI
information is encrypted and only an authorized device Aayog was held on 8 February 2015.
can decode it. NFC users should ensure that the 93. (a) The United Nations Conference on Environment and
companies do their business with the use of secure Development (UNCED), also known as the Rio Summit
channels. or Earth Summit. It was a major United Nations
83. (b) Golan Heights are located in Syria. However, the two- conference held in Rio de Janeiro from 3 to 14 June
thirds of the western region are currently occupied by 1992. In 2012, the United Nations Conference on
Israel, whereas the one-third of eastern region is Sustainable Development was also held in Rio, and is
controlled by Syria. also commonly called Rio+20 or Rio Earth Summit 2012.
94. (d) The Executive powers of the Union of India is vested 99. (a) The Green Climate Fund (GCF) is a fund within the
in the President. The Cabinet Secretary (and not the framework of the UNFCCC founded as a mechanism to
Prime Minister) is the ex-officio head of the Civil redistribute money from the developed to the
Services Board. developing world, in order to assist the developing
95. (a) "Goldilocks Zone," is the region around a star that has countries in adaptation and mitigation practices to
just the right conditions to find liquid water on a planet's counter climate change. The Fund is governed by the
surface. And liquid water is a key ingredient in the GCF Board. The assets of the GCF will be administered
search for life. by a trustee only for the purpose of, and in accordance
96. (b) C. Rajagopalachari organized a march from Trichinopoly with, the relevant decisions of the GCF Board. The
to Vedaranniyam on the Tanjore coast to break the salt World Bank was invited by the COP to serve as the
law in April 1930. interim trustee of the GCF, subject to a review three
97. (c) Vijayanagara or "city of victory" was the name of both years after operation of the Fund.
a city and an empire. The empire was founded in the 100. (c) Indian space agency ISRO was conferred the Indira
fourteenth century. In its heyday it stretched from the Gandhi Prize for Peace, Disarmament and Development
river Krishna in the north to the extreme south of the for 2014 in recognition of its contribution in
peninsula, (Krishna-Tungabhadra doab). It was strengthening international cooperation in peaceful use
founded by Harihara 1 and Bukka, in 1336. of outer space.
98. (b) Annie Besant was the first woman President of Indian
National Congress. She presided over the 1917 Calcutta
session of the Indian National Congress. Badruddin
Tyabji was the "First Muslim" to become the "President
of Indian National Congress".
INSTRUCTIONS
1. This Test Booklet contains 100 items (questions). Each item comprises four responses (answers). You will select
the response which you want to mark on the Answer sheet. In case, you feel that there is more than one correct
response, mark the response which you consider the best. In any case, choose ONLY ONE response for each item.
2. All items carry equal marks.
3. Penalty for wrong answers:
THERE WILL BE PENALTY FOR WRONG ANSWERS MARKED BY THE CANDIDATE FOR WRONG ANSWER.
(i) There are four alternatives for the answer to every question. For each question which has a penalty for which a
wrong answer has been given by the candidate, one-third of the marks assigned to that question will be deducted
as penalty.
(ii) If a candidate gives more than one answer, it will be treated as a wrong answer even if one of the given answers
happens to be correct and there will be same penalty as above to that question.
(iii) If a question is left blank, i.e., no answer is given by the candidate, there will be no penalty for that question.

DIRECTIONS for the following 8 (eight) items : Read the Manak remarried in Guleri's absence. Manak, who acts as a
following seven passages and answer the items that follow. Your reluctant but passive spectator, is meanwhile, informed by a friend
answers to these items should be based on the passages only. that Guleri, on hearing about her husband's second marriage,
poured kerosene on her clothes and set fire to them. Manak is
PASSAGE - 1 heartbroken and begins to live as if he were a dead man. When his
The richer States have a responsibility to cut down carbon second wife delivers a son, Manak stares at the child for a long
emissions and promote clean energy investments. These are the time and blurts out, "Take him away ! He stinks of kerosene."
States that got electricity, grew faster and now have high per 2. This is a sensitive issue-based story which tries to sensitise
capita income, making them capable of sharing India's burden of the readers about
becoming eco-friendly. Delhi, for example, can help by generating (a) Male chauvinism and infidelity
its own clean electricity using solar rooftop panels or even help (b) Love and betrayal
poor States finance their clean energy projects. It is no secret that (c) Lack of legal safeguards for women
State Electricity Boards, which control 95% of the distribution (d) Influence of patriarchal mindset
network, are neck-deep in losses. These losses further discourage PASSAGE - 3
State utilities from adopting renewable energy as it is more The ultimate aim of government is not to rule or control by fear,
expensive than fossil fuels. nor to demand obedience, but conversely, to free every man from
1. Which among the following is the most logical and rational fear, that. he may live in all possible security. In other words, to
assumption that can be made from the above passage?
strengthen his natural right to exist and work without injury to
(a) The richer States must lead in the production and
himself or others. The object of government is not to change men
adoption of renewable energy.
from rational beings into beasts or puppets. It should enable them
(b) The poor States always have to depend on rich States
to develop their minds and bodies in security, and to employ their
for electricity.
reason unshackled.
(c) The State Electricity Boards can improve their finances
3. Which among the following is the most logical and rational
by undertaking clean energy projects.
inference that can be made from the above passage?
(d) The high economic disparity between the rich and poor
(a) The true aim of government is to secure the citizens
States is the major cause of high carbon emissions in
their social and political freedom.
India.
(b) The primary concern of government is to provide
PASSAGE - 2 absolute social security to all its citizens.
Set against a rural backdrop, 'Stench of kerosene' is the story of a (c) The best government is the one that allows the citizens
couple, Guleri and Manak, who have been happily married for to enjoy absolute liberty in all matters of life.
several years but do not have a child. Manak's mother is desperate (d) The best government is the one that provides absolute
to have a grandchild to carry on the family name. Hence, she gets physical security to the people of the country
PASSAGE - 4 6. What is the most logical, rational and crucial message that
Our municipal corporations are understaffed. The issue of skills is implied in the above passage?
and competencies of the staff poses an even greater challenge. (a) MGNREGS should be extended only to those who have
Urban services delivery and infrastructure are complex to plan a bank account.
and execute. (b) The paper-based system of payments is more efficient
than electronic payment in the present scenario.
They require a high degree of specialization and
(c) The goal of electronic wage payments was not to
professionalism. The current framework within which municipal
eliminate mediation by village leaders.
employees, including senior management, are recruited does not
(d) It is essential to provide financial literacy to the rural
adequately factor in the technical and managerial competencies
poor.
required. Cadre and recruitment rules only specify the bare
minimum in academic qualifications. There is no mention of PASSAGE - 7
managerial or technical competencies, or of relevant work Individuals, groups and leaders who promote human development
experience. This is the case with most municipal corporations. operate under strong institutional, structural and political
They also suffer from weak organisation design and structure. constraints that affect policy options. But experience suggests
4. Which among the following in the most logical and rational broad principles for shaping an appropriate agenda for human
assumption that can be made from the above passage? development. One important finding from several decades of
(a) The task of providing urban services is a complex issue human development experience is that focusing exclusively on
which requires the organisational expansion of economic growth is problematic. While we have good knowledge
municipal bodies all over the country, about how to advance health and education, the causes of growth
(b) Our cities can provide better quality of life if our local are much less certain and growth is often elusive. Further, an
government bodies have adequate staff with required unbalanced emphasis on growth is often associated with negative
skills and competencies. environmental consequences and adverse distributional effects.
(c) Lack of skilled staff is due to the absence of institutions The experience of China, with its impressive growth record, reflects
which offer the requisite skills in city management. these broader concerns and underlines the importance of balanced
(d) Our country is not taking advantage of the approaches that emphasize investments in the non-income aspects
demographic dividend to manage the problems of human development.
associated with rapid urbanization 7. With reference to the above passage, consider the following
PASSAGE - 5 statements:
Flamingos in large flocks in the wild are social extremely loyal. 1. In developing countries, a strong institutional
They perform group mating dances. Parents are very fond of their framework is the only requirement for human
chicks, gathering them into crèches for protection while both males development and policy options
and females fly off to search for food. 2. Human development and economic growth are not
5. Which among the following is the most logical corollary to always positively inter-related.
the above passage? 3. Focusing only on human development should be the
(a) Mass nesting in all species of birds is essential to goal of economic growth
ensure complete survival of their off spring. Which of the above statements is/are correct ?
(b) Only birds have the capacity to develop social (a) 1 only (b) 2 and 3 only
behaviour and thus can do mass nesting to raise their (c) 2 only (d) 1,2 and 3
chicks in safety. 8. With reference to the above passage, the following
(c) Social behaviour in some species of birds increases assumptions have been made:
the odds of' survival in an unsafe world. 1. Higher economic growth is essential to ensure
(d) All species of birds set up creches for their chicks to reduction in economic disparity.
teach them social behaviour and loyalty. 2. Environmental degradation is sometimes a
PASSAGE - 6 consequence of economic growth
Vast numbers of Indian citizens without bank accounts live in Which of the above is/are valid assumption/assumptions?
rural areas, are financially and functionally illiterate, and have (a) 1 only (b) 2 only
little experience with technology. A research study was conducted (c) Both 1 and 2 (d) Neither 1 nor 2
in a particular area in which electronic wage payments in Mahatma 9. If A runs less fast than B, and B runs as fast but not faster
Gandhi National Rural Employment Guarantee Scheme than C; then, as compared to A, C runs
(MGNREGS) are meant to go directly to the poor. It was observed (a) slower than A
that recipients often assume that the village leader needs to mediate (b) faster than A
the process, as was the case under the previous paper-based (c) with same speed as A
system. Among households under this research study area who (d) Given data is not sufficient to determine
claimed to have at least one bank account, over a third reported 10. Each of A, B, C and D has ` 100. A pays ` 20 to B, who pays
still receiving MGNREGS wages in cash directly from a village ` 10 to C, who gets ` 30 from D. In this context, which one of
leader. the following statements is not correct?
(a) C is the richest
(b) D is the poorest. (four-wheelers) and scooters/motorbikes (two-wheelers) is
(c) C has more than what A and D have together. 100 more than twice the number of parked vehicles. The
(d) B is richer than D number of cars parked is
11. In a town, 45% population read magazine A, 55% read (a) 35 (b) 45
magazine B, 40% read magazine C, 30% read magazines A (c) 50 (d) 55
and B, 15% read magazines B and C, 25% read magazines A 19. The mangroves can shed tons of leaves per acre every year;
and C; and 10% read all the three magazines. What fungi and bacteria break down this leaf litter and consume
percentage do not read any magazine? it, they then are consumed by tiny worms and crustaceans,
(a) 10% (b) 15% which in turn feed small fish, which feed larger fish and
(c) 20% (d) 25% birds and crocodiles.
12. Examine the following statements: Which among the following is the most logical inference of
1. Lady's finger is tastier than cabbage. the above statement?
2. Cauliflower is tastier than lady's finger. (a) Coastal areas cannot have food chains without
3. Cabbage is not tastier than peas. mangroves.
The conclusion that can be drawn from these statements is (b) Mangroves are an essential component of all marine
that ecosystems.
(a) peas are as tasty as lady's finger. (c) Mangroves have a crucial role in some of the coastal
(b) peas are as tasty as cauliflower and lady's finger. food chains.
(c) cabbage is the least tasty of the four vegetables. (d) The composition of marine flora and fauna is largely
(d) cauliflower is tastier than cabbage. determined by mangroves.
13. Shahid and Rohit start from the same point in opposite 20. "By liberty I mean the eager maintenance of that atmosphere
directions. After each 1 km, Shahid always turns left and in which men have the opportunity to be their best selves."
Rohit always turns right. Which of the following statements Which one of the following expresses the view implied in
is correct ? the above statement?
(a) After both have travelled 2 km, the distance between (a) Liberty is the absence of restraint on human action.
them is 4 km. (b) Liberty is what law permits people to perform.
(b) They meet after each has travelled 3km. (c) Liberty is the ability to do what one desires.
(c) They meet for the first time after each has travelled 4 (d) Liberty is the maintenance of conditions for the growth
km. of human personality.
(d) They go on without ever meeting again.
14. In a 500 metres race, B starts 45 metres ahead of A, but A DIRECTIONS for the following 8 (eight) items : Read the
wins the race while B is still 35 metres behind. What is the following six passages and answer the items that follow. Your
ratio of the speeds of A to B assuming that both start at the answers to these items should be based on the passages only.
same time? PASSAGE - 1
(a) 25: 21 (b) 25: 20
Human history abounds in claims and theories confining the right
(c) 5:3 (d) 5:7
of governing to a few select citizens. Exclusion of the many is
15. Two equal glasses of same type are respectively 1/3 and 1/
justified on the ground that human beings may be rightfully
4 full of milk. They are then filled up with water and the
segregated for the good of society and viability of the political
contents are mixed in a pot. What is the ratio of milk and
process.
water in the pot?
21. Which one of the following statements is least essential as
(a) 7 : 17 (b) 1 : 3
a part of the argument in the above passage?
(c) 9 : 21 (d) 11 : 23
(a) Man seeks control over external things affecting him.
16. Out of 130 students appearing in an examination, 62 failed in
(b) In society, there are 'super' and 'sub' human beings.
English, 52 failed in Mathematics, whereas 24 failed in both
(c) Exceptions to universal citizen participation are
English and Mathematics. The number of students who
conducive to systemic efficacy.
passed finally is
(d) Governing implies recognition of disparities in
(a) 40 (b) 50
individual capacities.
(c) 55 (d) 60
17. In a group of persons travelling in a bus, 6 persons can PASSAGE - 2
speak Tamil, 15 can speak Hindi and 6 can speak Gujarati. In By 2050, the Earth's population will likely have swelled from seven
that group none can speak any other language. If 2 persons to nine billion people. To fill all those stomachs - while accounting
in the group can speak two languages only and one person for shifting consumption patterns, climate change, and a finite
can speak all the three languages, then how many persons amount of arable land and potable water – some experts say food
are there in the group? production will have to double. How can we make the numbers
(a) 21 (b) 22 add up? Experts say higher yielding crop varieties and more
(c) 23 (d) 24 efficient farming methods will be crucial. So will waste reduction.
18. In a parking area, the total number of wheels of all the cars Experts urge cities to reclaim nutrients and water from waste streams
and preserve farmland. Poor countries, they say, can improve crop of rain forests, reduced nutritional content, increased demand for
storage and packaging and rich nations could cut back on resource- preservation and packaging. Food insecurity also increases as
intensive foods like meat. the produce comes from regions that are not feeding their own
22. Which one of the following statements best sums up the population properly.
above passage? 25. With reference to the above passage, which of the following
(a) The population of the world is growing very fast. statements is/are true?
(b) Food security is a perennial problem only in developing 1. Consuming regionally grown food and not depending
countries. on long travelled food is a part of eco-friendly
(c) The world does not have enough resources to meet behaviour.
the impending food scarcity. 2. Food processing industry puts a burden on our natural
(d) Food security is increasingly a collective challenge. resources.
PASSAGE - 3 Select the correct answer using the code given below:
(a) 1 only (b) 2 only
Many people in India feel that if we cut our defence expenditure
(c) Both 1 and 2 (d) Neither 1 nor 2
on weapon-building, we can create a climate of peace with our
neighbours, subsequently reducing the conflict or creating a no- PASSAGE - 6
war situation. People who proclaim such ideas are either the I must say that, beyond occasionally exposing me to laughter, my
victims of war or the propagators of false argument. constitutional shyness has been of no disadvantage whatever. In
23. With reference to the above passage, which of the following fact I can see that, on the contrary, it has been all to my advantage.
is the most valid assumption? My hesitancy in speech, which was once an annoyance, is now a
(a) Building of weapons systems by us has instigated our pleasure. Its greatest benefit has been that it has taught me the
neighbours to wage wars against us. economy of words. I have naturally formed the habit of restraining
(b) The greater spending on weapon-building by us would my thoughts. And I can now give myself the certificate that a
lessen the possibility of armed conflict with our thoughtless word hardly ever escapes my tongue or pen. I do not
neighbours. recollect ever having had to regret anything in my speech or writing.
(c) It is necessary to have state of the art weapons systems I have thus been spared many a mishap and waste of time.
Experience has taught me that silence is part of the spiritual
for national security.
discipline of a votary of truth. Proneness to exaggerate, to suppress
(d) Many people in India believe that we are wasting our
or modify the truth, wittingly or unwittingly, is a natural weakness
resources on weapon-building.
of man, and silence is necessary in order to surmount it. A man of
PASSAGE - 4 few words will rarely be thoughtless in his speech; he will measure
India accounts for nearly a fifth of the world's child deaths. In every word. We find so many people impatient to talk. There is no
terms of numbers, it is the highest in the world - nearly 16 lakhs chairman of a meeting who is not pestered with notes for
every year. Of these, more than half die in the first month of life. permission to speak. And whenever the permission is given the
Officials believe that the reason for this is the absence of steps to speaker generally exceeds the time-limit, asks for more time, and
propagate basic health practices relating to breast feeding and keeps on talking without permission. All this talking can hardly be
immunisation. Also the large reproductive population of 2.6 crore said to be of any benefit to the world. It is so much waste of time.
remains bereft of care during the critical phases of pregnancy and My shyness has been in reality my shield and buckler. It has
allowed me to grow. It has helped me in my discernment of truth.
post-delivery. Added to this is the prevalence of child marriages,
26. The author says that a thoughtless word hardly ever escapes
anaemia among young women and lack of focus on adolescent
his tongue or pen. Which one of the following is not a valid
sanitation, all of which impact child death rates.
reason for this?
24. Which is the critical inference that can be made from the (a) He has no intention to waste his time.
above passage? (b) He believes in the economy of words.
(a) A lot of Indians are illiterate and hence do not recognize (c) He believes in restraining his thoughts.
the value of basic health practices. (d) He has hesitancy in his speech.
(b) India has a very huge population and the government 27. The most appropriate reason for the author to be spared
alone cannot manage public health services. many a mishap is that
(c) Universalization and integration of maternal health and (a) he hardly utters or writes a thoughtless word.
child health services can effectively address the (b) he is a man of immense patience.
problem. (c) he believes that he is a spiritual person.
(d) The nutrition of women in child bearing age does not (d) he is a votary of truth.
affect child mortality rate. 28. For the author, silence is necessary in order to surmount
PASSAGE - 5 (a) constitutional shyness.
(b) hesitancy in speech.
Foods travel more than the people who eat them. Grocery stores (c) suppression of thoughts.
and supermarkets are loaded with preserved and processed foods. (d) tendency to overstate.
This, however, often leads to environmental threats, such as 29. Twelve people form a club. By picking lots, one of them will
pollution generated by long distance food transportation and host a dinner for all once in a month. The number of dinners
wastage of food during processing and transportation, destruction a particular member has to host in one year is
(a) One (b) Zero 35. In a test, a candidate attempted only 8 questions and secured
(c) Three (d) Cannot be predicted 50% marks in each of the questions. If he obtained a total of
30. An automobiles owner reduced his monthly petrol 40% in the test and all questions in the test carried equal
consumption when the prices went up. The price- marks, how many questions were there in the test?
consumption relationship is as follows: (a) 8 (b) 10
Price (in ` Per litre) 40 50 60 75
(c) 15 (d) 16
Monthly consumption (in litres) 60 48 40 30
If the price goes up to ` 80 per litre, his expected consumption 36. A father is nine times as old as his son and the mother is
(in litres) will be eight times as old as the son. The sum of the father's and the
(a) 30 (b) 28 mother's age is 51 years. What is the age of the son ?
(c) 26 (d) 24 (a) 7 years (b) 5 years
31. Consider the figures given below: (d) 4 years (d) 3 years
37. Four persons A, B, C and D consisting of two married couples
are in a group. Both the women are shorter than their
respective husbands. A is the tallest among the four. C is
taller than B. D is B's brother. In this context, which one of
the following statements is not correct ?
To fit the question mark, the correct answer is
(a) All four have family ties.
(b) B is the shortest among the four.
(c) C is taller than D.
(a) (b)
(d) A is B's husband.
38. Consider the following statements :
1. A man had a wife, two sons and daughters in his family
(c) (d) 2. The daughters were invited to a feast and the male
members of the family went out to take part in a pienic
32. Consider the following matrix: 3. The man's father did not return from his work.
3 8 10 2 ? 1 Which of the following statements is true?
(a) Only the man's wife was left at home.
6 56 90 2 20 0
(b) It is likely that the man's wife was left at home.
What is the missing number at '?' in the matrix?
(a) 5 (b) 0 (c) None was left at home
(c) 7 (d) 3 (d) More than one person was left at home.
33. What is the missing number 'X' of the series 7, X, 21, 31, 43? 39. Geeta: Naresh has become a better boxer since he started
(a) 11 (b) 12 meditation.
(c) 13 (d) 14 Radha: Impossible, A boxer's most important asset is his
34. Four cardboard pieces of specific shapes are shown in the aggressiveness.
following figure: Radha's statement reflects her belief that
(a) meditation tends to make a person less aggressive.
(b) meditation has little or no effect on the person who
practises it.
(c) Naresh was a poor boxer earlier because he was not
aggressive enough.
(d) Naresh would not have taken to meditation as he was
Which one of the following figures given can be formed by a boxer.
joining these pieces together? 40. All good athletes want to win and all athletes who want to
win eat a well-balanced diet; therefore all athletes who do
not cat a well-balanced diet are bad athletes.
(a) (b) The best conclusion from this statement is that
(a) no bad athlete wants to win.
(b) no athlete who does not eat a well-balanced diet is a
good athlete.
(c) every athlete who eats a well-balanced diet is a good
(c) (d)
athlete.
(d) all athletes who want to win are good athletes.
DIRECTIONS for the following 7 (seven) items : Read the the following statements?
following four passages and answer the items that follow. Your (a) The conflict between the claims of State and individual
answers to these items should be based on the passages only. remains unresolved.
(b) Anarchy and chaos are the obvious results of
PASSAGE - 1 democratic traditions.
India has suffered from persistent high inflation. Increase in (c) Old values, ideas and traditions persist despite the
administered prices, demand and supply imbalances, imported dynamic nature of human society.
inflation aggravated by rupee depreciation, and speculation - have (d) Constitutional guarantee of freedom of speech is not
combined to keep high inflation going. If there is an element in the interest of society.
common to all of them, it is that many of them are the outcomes of PASSAGE - 4
economic reforms. India's vulnerability to the effects of changes Climate change is a complex policy issue with major implications
in international prices has increased with trade liberalisation. The in terms of finance. All actions to address climate change ultimately
effort to reduce subsidies has resulted in a continuous increase in involve costs. Funding is vital for countries like India to design
the prices of commodities that are administered. and implement adaptation and mitigation plans and projects. Lack
41. What is the most logical, rational and crucial message that of funding is a large impediment to implementing adaptation plans.
is implied in the above passage? The scale and magnitude of the financial support required by
(a) Under the present circumstances, India should developing countries to enhance their domestic mitigation and
completely avoid all trade liberalisation policies and all adaptation actions are a matter of intense debate in the multilateral
subsidies. negotiations under the United Nations Framework Convention
(b) Due to its peculiar socio-economic situation, India is on Climate Change (UNFCCC). The Convention squarely puts
not yet ready for trade liberalisation process. the responsibility for provision of financial support on the
(c) There is no solution in sight for the problems of developed countries, taking into account their contribution to the
continuing poverty and inflation in India in the near stock of greenhouse gases (GHGs) in the atmosphere. Given the
future. magnitude of the task and the funds required, domestic finances
(d) Economic reforms can often create a high inflation are likely to fall short of the current and projected needs of the
economy. developing countries. Global funding through the multilateral
PASSAGE - 2 mechanism of the Convention will enhance their domestic capacity
No Right is absolute, exclusive or inviolable. The Right of personal to finance the mitigation efforts.
property, similarly, has to be perceived in the larger context of its 44. According to the passage, which of the following is/are a
matter of intense debate in the multilateral negotiations under
assumed legitimacy. The Right of personal property should unite
UNFCCC regarding the role of developing countries in
the principle of liberty with that of equality, and both with the
climate change?
principle of cooperation.
1. The scale and size of required financial support.
42. In the light of the argument in the above passage, which
2. The crop loss due to climate change in the developing
one of the following statements is the most convincing
countries.
explanation?
3. To enhance the mitigation and adaptation actions in
(a) The Right of personal property is a Natural Right duly
the developing countries.
supported by statutes and scriptures.
Select the correct answer using the code given below:
(b) Personal property is a theft and an instrument of (a) 1 only (b) 2 and 3 only
exploitation. The Right of personal property is therefore (c) 1 and 3 only (d) 1, 2 and 3
violative of economic justice. 45. In this passage, the Convention puts the responsibility for
(c) The Right of personal property is violative of the provision of financial support on the developed
distributive justice and negates the principle of countries because of
cooperation. 1. their higher level of per capita incomes.
(d) The comprehensive idea of economic justice demands 2. their large quantum of GDP.
that the Right of each person to acquisition of property 3. their large contribution to the stock of GRGs in the
has to be reconciled with that of others. atmosphere.
PASSAGE - 3 Select the correct answer using the code given below:
The conflict between man and State is as old as State history. (a) 1 only (b) 1 and 2 only
Although attempts have been made for centuries to bring about a (c) 3 only (d) 1, 2 and 3
proper adjustment between the competing claims of State and the 46. With regards to developing countries, it can be inferred
individual, the solution seems to be' still far off. This is primarily from the passage that climate change is likely to have
because of the dynamic nature of human society where old values implications on their
and ideas constantly yield place to new ones. It is obvious that if 1. domestic finances.
individuals are allowed to have absolute freedom of speech and 2. capacity for multilateral trade.
action, the result would be chaos, ruin and anarchy. Select the correct answer using the code given below:
43. The author's viewpoint can be best summed up in which of (a) 1 only
(b) 2 only
(c) Both 1 and 2 (a) 39 (b) 30
(d) Neither 1 nor 2 (c) 21 (d) 20
47. Which one of the following is essentially discussed in the 55. Two men, Anil and David, and two women, Shabnam and
passage? Rekha are in a sales group. Only two speak Tamil. The other
(a) Conflict between developed and developing countries
two speak Marathi. Only one man and one woman can drive
regarding support for mitigation
a car. Shabnam speaks Marathi. Anil speaks Tamil. Both
(b) Occurrence of climate change due to excessive
Rekha and David can drive.
exploitation of natural resources by the developed
countries Which of the following statements is true?
(c) Lack of political will on the part of all the countries to (a) Both the Tamil speakers can drive a car.
implement adaptation plans. (b) Both the Marathi speakers can drive a car.
(d) Governance problems of developing countries as a (c) Both of those who can drive a car speak Marathi.
result of climate change (d) One of those who can drive a car speaks Tamil.
48. Between 6 PM and 7 PM the minute hand of a clock will be
56. In a plane, line X is perpendicular to line Y and parallel to
ahead of the hour hand by 3 minutes at
line Z; line U is perpendicular to both lines V and W; line X
(a) 6: 15 PM (b) 6: 18 PM
(c) 6: 36 PM (d) 6: 48 PM is perpendicular to line V.
49. There are 5 tasks and 5 persons. Task-l cannot be assigned Which one of the following statements is correct?
to either person-l or person-2. Task-2 must be assigned to (a) Z, U and W are parallel.
either person-3 or person-4. Every person is to be assigned (b) X, V and Y are parallel.
one task. In how many ways can the assignment be done? (c) Z, V and U are all perpendicular to W.
(a) 6 (b) 12 (d) Y, V and W are parallel.
(c) 24 (d) 144 57. A cow costs more than 4 goats but less than 5 goats. If a
50. The monthly incomes of Peter and Paul are in the ratio of 4 goat costs between ` 600 and ` 800, which of the following
: 3. Their expenses are in the ratio of 3 : 2. If each saves ` is a most valid conclusion?
6,000 at the end of the month, their monthly incomes
(a) A cow costs more than ` 2,500.
respectively are (in `)
(a) 24,000 and 18,000 (b) 28,000 and 21,000 (b) A cow costs less than ` 3,600.
(c) 32,000 and 24,000 (d) 34,000 and 26,000 (c) A cow costs between ` 2,600 and ` 3,800.
51. Two cities A and B are 360 km apart. A car goes from A to B (d) A cow costs between ` 2,400 and ` 4,000.
with a speed of 40 km/hr and returns to A with a speed of 60 58. A society consists of only two types of people fighters and
km/hr. What is the average speed of the car? cowards. Two cowards are always friends. A fighter and a
(a) 45 km/hr (b) 48 km/hr coward are always enemies. Fighters are indifferent to one
(c) 50 km/hr (d) 55 km/hr another. If A and B are enemies, C and D are friends, E and F
are indifferent to each other, A and E are not enemies, while
DIRECTIONS for the following 2 (two) items : Read the following Band F are enemies.
passage and answer the 2 (two) items that follow:
Which of the following statements is correct?
A, B, C, D, E and F are cousins. No two cousins are of the same (a) B, C and F are cowards.
age, but all have birthdays on the same day of the same month. (b) A, E and F are fighters.
The youngest is 17 years old and the oldest E is 22 years old. F is (c) B and E are in the same category.
somewhere between B and D in age. A is older than B. C is older
(d) A and F are in different categories.
than D. A is one year older than C.
59. In a box of marbles, there are three less white marbles than
52. Which one of the following is possible?
the red ones and five more white marbles than the green
(a) D is 20 years old
ones. If there are a total of 10 white marbles, how many
(b) F is 18 years old
marbles are there in the box?
(c) F is 19 years old
(d) F is 20 years old (a) 26 (b) 28
53. What is the number of logically possible orders of all six (c) 32 (d) 36
cousins in terms of increasing age? 60. Candidates in a competitive examination consisted of 60%
(a) 1 (b) 2 men and 40% women. 70% men and 75% women cleared the
(c) 3 (d) 4 qualifying test and entered the final test where 80% men
54. In a society it is customary for friends of the same sex to and 70% women were successful.
hug and for friends of opposite sex to shake hands when Which of the following statements is correct?
they meet. A group of friends met in a party and there were (a) Success rate is higher for women.
24 handshakes. (b) Overall success rate is below 50%.
Which one among the following numbers indicates the (c) More men cleared the examination than women.
possible number of hugs? (d) Both (a) and (b) above are correct.
DIRECTIONS for the following 7 (seven) items : Read the malnourished. Lakhs of them die from preventable conditions.
following six passages and answer the items that follow. Your Diarrhoea leaves Indians' bodies smaller on average than those of
answers to these items should be based on the passages only. people in some poorer countries where people eat fewer calories.
Underweight mothers produce stunted babies prone to sickness
PASSAGE - 1 who may fail to develop their full cognitive potential. The germs
Climate change is already making many people hungry all over released into environment harm rich and poor alike, even those
the world, by disrupting crop yields and pushing up prices. And who use latrines.
it is not just food but nutrients that are becoming scarcer as the 63. Which among the following is the most critical inference
climate changes. It is the poorest communities that will suffer the that can be made from the above passage?
(a) The Central and State governments in India do not
worst effects of climate change, including increased hunger and
have enough resources to afford a latrine for each
malnutrition as crop production and livelihoods are threatened.
household.
On the other hand, poverty is a driver of climate change, as
(b) Open defecation is the most important public health
desperate communities resort to unsustainable use of resources problem of India.
to meet current needs. (c) Open defecation reduces the human capital of India's
61. Which among the following is the most logical corollary to workforce.
the above passage? (d) Open defecation is a public health problem in all
(a) Government should allocate more funds to poverty developing countries.
alleviation programmes and increase food subsidies to PASSAGE - 4
the poor communities.
We generally talk about democracy but when it comes to any
(b) Poverty and climate impacts reinforce each other and
particular thing, we prefer a belonging to our caste or community
therefore we have to re-imagine our food systems. or religion. So long as we have this kind of temptation, our
(c) All the countries of the world must unite in fighting democracy will remain a phoney kind of democracy. We must be
poverty malnutrition and treat poverty as a global in a position to respect a man as a man and to extend opportunities
problem. for development to those who deserve them and not to those who
(d) We must stop unsustainable agricultural practices happen to belong to our community or race. This fact of
immediately and control food prices. favouritism has been responsible for much discontent and ill-will
PASSAGE - 2 in our country.
The Global Financial Stability Report finds that the share of 64. Which one of the following statements best sums up the
above passage?
portfolio investments from advanced economies in the total debt
(a) Our country has a lot of diversity with its many castes,
and equity investments in emerging economies has doubled in
communities and religions.
the past decade to 12 percent. The phenomenon has implications
(b) True democracy could be established by providing
for Indian policy makers as foreign portfolio investments in the equal opportunities to all.
debt and equity markets have been on the rise. The phenomenon (c) So far none of us have actually understood the meaning
is also flagged as a threat that could compromise global financial of democracy.
stability in a chain reaction, in the event of United States Federal (d) It will never be possible for us to establish truly
Reserve's imminent reversal of its "Quantitative Easing" policy. democratic governance in our country.
62. Which among the following is the most rational and critical PASSAGE - 5
inference that can be made from the above passage?
The existence/establishment of formal financial institutions that
(a) Foregin portfolio investments are not good for emerging
offer safe, reliable and alternative financial instruments is
economies. fundamental in mobilising savings. To save, individuals need
(b) Advanced economies undermine the global financial access to safe and reliable financial institutions, such as banks,
stability. and to appropriate financial instruments and reasonable financial
(c) India should desist from accepting foreign portfolio incentives. Such access is not always available to all people in
investments in the future. developing countries like India and more so, in rural areas. Savings
(d) Emerging economies are at a risk of shock from help poor households manage volatility in cash flow, smoothen
advanced economies. consumption, and build working capital. Poor households without
PASSAGE - 3 access to a formal savings mechanism encourage immediate
spending temptations.
Open defecation is disastrous when practised in very densely
65. With reference to the above passage, consider the following
populated areas, where it is impossible to keep away human faeces
statements:
from crops, wells, food and children's hands. Groundwater is also
1. Indian financial institutions do not offer any financial
contaminated by open defecation. Many ingested germs and
instruments to rural households to mobilise their
worms spread diseases. They prevent the body from absorbing
savings.
calories and nutrients. Nearly one-half of India's children remain
2. Poor households tend to spend their earnings/savings
due to lack of access to appropriate financial
instruments. 71. The number of persons who read magazine X only is thrice
Which of the statements given above is/are correct? the number of persons who read magazine Y. The number of
(a) 1 only (b) 2 only. persons who read magazine Y only is thrice the number of
(c) Both 1 and 2 (d) Neither 1 nor 2 persons who read magazine X. Then, which of the following
66. What is the crucial message conveyed in the passage? conclusions can be drawn?
(a) Establish more banks. 1. The number of persons who read both the magazines
is twice the number of persons who read only magazine
(b) Increase the Gross Domestic Product (GDP) growth
X.
rate
2. The total number of persons who read either one
(c) Increase the interest rate of bank deposits magazine or both the magazines is twice the number of
(d) Promote financial inclusion persons who read both the magazines.
PASSAGE - 6 Select the correct answer using the code given below:
Governments may have to take steps which would otherwise be (a) 1 only (b) 2 only
an infringement on the Fundamental Rights of individuals, such (c) Both 1 and 2 (d) Neither 1 nor 2
72. The graph below depicts the earnings of A and B over the
as acquiring a person's land against his will, or refusing permission
period 2000 to 2010:
for putting up a building, but the larger public interest for which
these are done must be authorized by the people (Parliament).
Discretionary powers to the administration can be done away
with. It is becoming more and more difficult to keep this power
within limits as the government has many number of tasks to
perform. Where discretion has to be used, there must be rules and
safeguards to prevent misuse of that power. Systems have to be
devised which minimise, if not prevent, the abuse of discretionary
power. Government work must be conducted within a framework
of recognised rules and principles, and decisions should be similar
and predictable.
67. Which among the following is the most logical assumption
From the graph, which one of the following can be
that can be made from the above passage?
concluded?
(a) Government should always be given wide discretionary
(a) On the average A earned more than B during this period.
power in all matters of administration.
(b) The supremacy of rules and safeguards should prevail (b) On the average B earned more than A during this period.
as opposed to the influence of exclusive discretion of (c) The earnings of A and B were equal during this period.
authority. (d) The earnings of A were less as compared to B during
(c) Parliamentary democracy is possible only if the this period.
Government has wider discretionary power. 73. Two pipes A and B can independently fill a tank completely
(d) None of the above statements is a logical assumption in 20 and 30 minutes respectively. If both the pipes are
that can be made from this passage. opened simultaneously, how much time will they take to fill
68. A selection is to be made for one post of Principal and two the tank completely?
posts of Vice-Principal. Amongst the six candidates called (a) 10 minutes (b) 12 minutes
for the interview, only two are eligible for the post of Principal (c) 15 minutes (d) 25 minutes
while they all are eligible for the post of Vice-Principal. The 74. Each of the six different faces of a cube has been coated
number of possible combinations of selectees is with a different colour i.e., V, I, B, G, Y and O. Following
(a) 4 (b) 12 information is given:
(c) 18 (d) None of the above 1. Colours Y, O and B are on adjacent faces.
69. A student has to opt for 2 subjects out of 5 subjects for a 2. Colours I, G and Y are on adjacent faces.
course, namely, Commerce, Economics, Statistics, 3. Colours B, G and Y are on adjacent faces.
Mathematics I and Mathematics II. Mathematics II can be 4. Colours O, V and B are on adjacent faces.
offered only if Mathematics I is also opted. The number of Which is the colour of the face opposite to the face coloured
different combinations of two subjects which can be opted with O?
is (a) B (b) V
(a) 5 (b) 6 (c) G (d) I
(c) 7 (d) 8 75. Consider the following statements followed by two
70. A person ordered 5 pairs of black socks and some pairs of conclusions:
brown socks. The price of a black pair was thrice that of a Statements : Some men are great.
brown pair. While preparing the bill, the bill clerk interchanged Some men are wise.
the number of black and brown pairs by mistake which
Conclusion I : Men are either great or wise.
increased the bill by 100%. What was the number of pairs of
Conclusion II : Some men are neither great nor wise
brown socks in the original order?
Which one of the following is correct?
(a) 10 (b) 15
(c) 20 (d) 25 (a) Only conclusion I is valid
(b) Only conclusion II is valid
(c) Both the conclusions are valid 79. The proportion of expenditure on various items by two
(d) Neither of the conclusions is valid families A and B are represented in the following Bar Charts:
76. Consider the following statements:
1. Some claim to have seen UFOs (Unidentified Flying
Objects).
2. Life on other heavenly bodies is considered to be a
possibility.
3. Voyage to space is now an established fact.
From the above statements, it may be concluded that
(a) UFOs are heavenly bodies
(b) UFOs are sent from other heavenly bodies
(c) Some living species in other heavenly bodies are more
intelligent than man
(d) Nothing definite can be said about the UFOs
77. If ABC × DEED = ABCABC; where A, B, C, D and E are
different digits, what are the values of D and E? From these charts, we can conclude that
(a) D = 2, E = 0 (b) D = 0, E = 1 (a) Family A spent more money on food than Family B.
(c) D = 1, E = 0 (d) D = 1, E = 2 (b) Family B spent more money on food than Family A.
78. Year-wise variation of the price of a certain commodity is (c) Family A and Family B spent the same amount on food.
shown in the following graph: (d) The expenditure on food by Family A and Family B
cannot be compared.
80. Usha runs faster than Kamala, Priti runs slower than Swati,
Swati runs slower than Kamala. Who is the slowest runner?
(a) Kamala (b) Priti
(c) Swati (d) Usha

The price of the commodity in the year 1990


(a) must have been ` 10/-
(b) must have been ` 12/-
(c) must have been any where between ` 10/- and ` 20/-
(d) is higher than that in the year 1991
ANSWERS AND EXPLANATIONS
1. (a) (a) is correct as this assumption is inherent in the (d) is wrong as the demographic dividend and the
first part of the passage which syas that the richer problems associated with rapid urbanization are out of
states, having grown, have the capability to share India's context of the passage.
burden of becoming eco-friendly. 5. (c) (a), (b) and (d) arewrong as they talk about birds in
(b) is absurd as nowhere the passage talks about poor general. The passage talks specifically about
states being dependent on rich States for electricity. Flamingos.
(c) is wrong as clean energy projects are more (c) is correct asit talks about some species of birds.
expensive than the traditional fossil fuels. 6. (d) (a) is wrong as it is out of context of the passage.
(c) is wrong as the major cause of high carbon Further extended MGNREGS only to those who have a
emissions in India is development using cheaper bank account would go against the very essence of
energy sources which have higher carbon emissions. the scheme.
2. (d) (a) is wrong as the passage is not at all related to (b) is wrong in spite of the paper-based system of
Male chauvinism and infidelity payments being more efficient than electronic payment
(b) is wrong as Love and betrayal is not the issue the system. This is not the main message of the passage.
passage is trying to sentitise about. The passage is (c) is wrong as it negates the fact mentioned in the
more about a mothers' influence on her child. How she passage. Refer the second sentence which says,
governed his life? 'electronic wage payments in MGNREGS are meant to
(c) is wrong as the passage is not at all related to go directly to the poor.'
Lack of legal safeguards for women. Guleri didn't (d) is absolutely correct as the opening sentence
resorted for any such protection. clearly says that citizens living in rural areas, are
(d) is correct as discussed in (b). The real reason financially illiterate. So it is essential to provide financial
behind this situation was Manak's mother. He could literacy to the rural poor.
not realise her wife's love as he was under the influence 7. (c) 3 is wrong as the passage does not talk about focusing
of her mother's mindset. only on human development. It do talks about an unbalanced
3. (b) (a) is wrong as the passage does not talk about social emphasis on growth but that, in any case, does not mean
and political freedom rather it talks about social security. focusing only on human development.
(b) is correct asit talks about absolute social security 2 is absolutely correct as the passage clearly says an
to all the citizens which is the essence of the passage. unbalanced emphasis on growth is often associated with
Refer the 1st sentence, 'to free every man from fear, negative environmental consequences and adverse
that. he may live in all possible security.' distributional effects.
(c) is wrong as the passage does not talk about 1 is wrong as it is completely out of context of the
absolute liberty in all matters of life rather it calls to passage.
develop the citizen's minds and bodies in security. Refer 8. (b) 1 is wrong as the passage nowhere talks about economic
last sentence. disparity rather it talks about health, education or other non-
(d) is wrong as the passage talk about absolute income aspects of human development. The passage is
security and not just physical security as 'all possible essentially about drawing a balance between economic
security' mentioned in 1st sentence refers to physical growth and human development.
and mental (refer last sentence) security. 2 is the correct assumption as unbalanced growth
4. (b) (a) is wrong as the passage does not talk about sometimes lead to environmental degradation.
organisational expansion of municipal bodies all over 9. (b) A < B < C
the country. Now, compared to A, C runs faster than A.
(b) is correct as the passage clearly talks about lack 10. (c) According to question, A, B, C and D each carry `
of adequate staff with required skills and competencies 100.
in the municipal corporations or local government +20 -10
bodies and that these skills are very much required as A B C -30 D
the operations are quite complex. Hence the assumption 80 120 140 70
that our cities can provide better quality of life with
Here, option (c) is not correct. C has more than what A
quality staff is clearly marked out.
and D have together.
(c) is wrong as the passage does not talk about the
reasons for the lack of skilled staff.
11. (c) 15. (a) Let the capacity of each of the equal glass = x litre
Each glass containing x/3 and x/4 of milk.
The quantities of milk in first and the second glasses
45% x x
30% 55% were and respectively
Magazine 3 4
Magazine
A B 4x + 3x 7 x
x x
10% = + = =
3 4 12 12
25% 15%
So, the quantities of water in the first and the second
glasses were
40%
x x
Magazine = x- and x -
C 3 4
2x 3x 8x + 9x 17x
= + = =
Number of population who read the magazine 3 4 12 12
= (45 + 55 + 40) – (30 + 25 +15) + 10 7x 17x 7
= 140 – 70 + 10 Ratio of milk and water = : = or 7 : 17
12 12 17
= 80%
Number of population who do not read magazine 16. (a)
= 100 – 80 = 20%
12. (c) Cauliflower > Lady finger > Cabbage
62
But Peas > Cabbage 24 52
English
The conclusion that can be drawn from these Mathematics
statements is that cabbage is the least tasty among the
four vegetables.
13. (b) After both have travelled 3 km, their positions would
be on follows. Total students = 130
Starting point Total students failed = (62 + 52) – 24
S R = 114 – 24 = 90
Left
Right Number of students passed finally = 130 – 90 = 40
1 km 1 km 17. (c) Let a = Speaks one language
Let b = Speaks two language
Let c = Speaks three language
then
a + 2b + 3c = 6 +15 +6
Destination a + 4 + 3 = 27
point a = 20
Hence, (b) is correct. \ Total number of persons in the group
14. (a) Total distance = 500m = 20 + 2 + 1 = 23.
Distance covered by A = 500m 18. (c) Let number of cars be x
Distance covered by B = 500 – 45 – 35 Let number of scooters / motor bikes = y
= 500 – 80 \ Total number of parked vehicles = x + y
= 420 According to question
4x + 2y = 2 (x + y) + 100
distance of A 4x + 2y = 2 x + 2y + 100
speed of A time 2x = 100
speed of B = distance of B x = 50
time Hence, Number of cars parked is 50.
19. (c) The passage clearly talks about the role Mangroves
500 time
= ´ play in some of the coastal food chains. Other food
time 420 chains - with or without Mangroves - might exist but
500 they are not mentioned in the passage. So (c) is correct.
= The passage does not say that no food chain is
420
possible without Mangroves so (a) is wrong. (b) is
25 also wrong as the passage does not talk about
= = 25 : 21
21 Mangroves being an essential component of all marine
ecosystems. Further (d) is also wrong for the same impacts it produces - pollution, wastage of food and rain
reasoning as the passage does not talk about the forests, reduced nutritional content, increased demand for
composition of marine flora and fauna. preservation and packaging. All this leads to put a burden
20. (d) (a) is wrong because the sentence says that liberty on our natural resources. So 2 is true. 1 is not correct as the
means the eager maintenance of that atmosphere passage does not talk about the regionally grown food. It
which might require restrictions as well. talks about regions with good produce feeding other parts
(b) is wrong because if the people work only of the world leaving their own region hungry. So 1 is not
according to the law they might become good correct.
citizens but might not work to their best selves. 26. (a) The first six sentences of the passage clearly answers
(c) is wrong because if people do what they desire it the question. (b), (c) and (d) are clearly mentioned in these
might lead to materialist thinking which is not the sentences. It is not mentioned that the author has no
best self. intention to waste his time but what is mentioned is that his
(d) is correct because with the growth of human constitutional shyness saves or spares him a lot of time.
personalitythe people would be their best selves. 27. (a) The first half or the first 7-8 sentences of the passage
21. (a) (a) is not or least essential asa part of the argument clearly explains the shyness of the author or his hesitancy
because it is irrelevant in the context of the passage. in speech which has restrainedhis thoughts. As a result no
(b) is essential as a part of the argument as the 'super' thought less word is uttered from himwhile speaking or
and 'sub' human beings refers to the 'few select citizens' writing and hence he has never regrettedhis speech or
and the 'excluded ones'.. writing. This has spared him from many a mishap and waste
(c) is essential asa part of the argument as the second of time. Immense patience and spiritual person cannot be
sentence clearly says this. related to him at all. The author sarcastically mentions that
(d) is essential asthe recognition of disparities in 'silence is part of the spiritual discipline of a votary of
individual capacities leads to the exclusion policies. truth'.So (a) is correct answer.
22. (d) (a) is wrong as it states a fact mentioned in the 28. (d) The 10th sentence of the passage 'Proneness to
passage. It does not sum up the passage. exaggerate, to _______ in order to surmount it' clearly
(b) is wrong as the passage does not restrict the suggests 'tendency to overstate' as the answer. So (d) is
problem to the developing countries only. correct answer.
(c) is wrong as the passage only says that the food 29. (d) We cannot predict the number of dinners for a
production will have to double. It does not talk about particular member from the given data. It may be possible
food scarcity at all. Rather it talks about how to meet that by choosing members from picking lots, one may have
the required target. to host a dinner more than one times.
(d) correctly sums up the passage. Food security is 30. (a) The product of price (in `/litre) and the monthly
increasingly a collective challenge and the passage consumption (in litres) is constant. This constant is equal
suggests means to combat it. to 2400.
23. (b) (a) is wrong as the passage says that if we can cut Expected consumption when the price goes up to ` 80
our defence expenses we can find peace or Building of 2400
weapons systems by us will instigated our neighbours per litre = = 30litres
80
to wage wars against us is a false propagation.
31. (a) The ‘ ’ is shifting one corner to the next in the
(b) is correct as the passage supports weapon-
clockwise direction and each time it is getting inverted. Thus,
building by India. It can also be inferred from the the should appear at the top left corner pointing upward.
passage that India wants to establish peace with our
Rest of the elements are shifting positions in the counter
neighbours. So 'weapon-building would lessen the
clockwise direction.
possibility of armed conflict with our neighbours' is
32. (a) In each column, let the number at the top be ‘x’ and the
the most valid assumption.
one at the bottom is ‘(x – 1)’. The relation between columns
Although (c) is also an assumption but use of the
are given by
words'state of the art'and'national security' does not
(x) (x – 1) = 3(3 – 1) = 6
make it the most valid assumption.
Therefore, 5(5 – 1) = 20
(d) is an inference but not an assumption.
24. (c) The questions asks for the critical inference. (d) is a 33. (c) 13
wrong as it presents a contradictory logic as mentioned in 7 X 21 31 43
the passage. (a) and (b) are correct statements in the context
+6 +8 +10 +12
of the passage but they present only part logic of the
passage. Only (c) presents the most critical inference as it 34. (d) By combining the given fragment, we get figure (d).
highlights the problem and the solutions mentioned in the 35. (b) Let each question carry x marks.
passage. According to question
25. (b) The passage talks about the preserved and processed 50
foods or the Food processing industry and the various Þ 8x ´ = 40
100
40 (c) is wrong as the passage does not talk about poverty at
x= = 10 all.
4
42. (d)
100 (a) is wrong as it negates the basic premise of the passage.
Numbers of question in the test = = 10
10 According to the passage 'No Right is absolute,
36. (d) Let the son’s age = x exclusive or inviolable whereas (a) says that The Right
Age of Father = 9x of personal property is a Natural Right.
Age of Mother = 8x (b) is wrong as it presents another extreme stand on
Now, According to question Personal property by equating it to theft and an
8x + 9x = 51 instrument of exploitation. It misses out the assumed
legitimacy of the right and does not see it in the context.
Þ 17x = 51
(c) is wrong as it talks about The Right of personal property
x = 3 years
being violative of distributive justice and negating the
37. (c) Given, A is tallest among the four. Hence, A is male, D is
principle of cooperation. It does not explain the
also male, as D is the brother of B. Then, B and C are
argument.
females. D is the husband of C. D is the shorter than A
(d) is correct as it clearly explains the argument by
and is taller than B. These, B is the shorter from above
mentioning that the Right of each person to acquisition
only (c) is incorrect.
of property has to be reconciled with that of others.Refer
38. (b) the principle of cooperation mentioned in the last
Man Wife
sentence.
Note : Economic justice, which touches the individual
person as well as the social order, encompasses the
Son Son Wife
moral principles which guide us in designing our
All the male member of the family went out to take part economic institutions. These institutions determine
in a picnic. It is given that daughters were invited to a how each person earns a living, enters into contracts,
feast. exchanges goods and services with others and
Nothing is specified about men’s wife. Hence, the otherwise produces an independent material
statement that the man’s wife is likely to be left at home foundation for his or her economic sustenance. The
is true. ultimate purpose of economic justice is to free each
39. (a) Radha did not accept the argument that a person person to engage creatively in the unlimited work
becomes a better boxer after meditation. The reason beyond economics, that of the mind and the spirit.
for this is that a boxer, according to her, should be The Three Principles of Economic Justice: Like every
aggersive. This indicates that Radha believes that system, economic justice involves input, output, and
meditation make sa person less aggessive. feedback for restoring harmony or balance between
40. (b) The given statement can be represented as– input and output. Within the system of economic
justice as defined by Louis Kelso and Mortimer Adler,
Eat well there are three essential and interdependent principles:
Want to balance Participative Justice (the input principle), Distributive
win diet Justice (the out-take principle), and Social Justice (the
feedback principle). Like the legs of a three-legged
stool, if any of these principles is weakened or missing,
Good Athletes the system of economic justice will collapse.
It is stated that all good athletes want to win. It does
not mean that no one wants to win. Hence, (b) is true.
There can be people other than good athletes, who eat
well balanced diet.
41. (d) The passage clearly talks about the high inflation in
India and its inability to contain it due to the
innumerable external factors which are a result of the
economic reforms. So (d) is correct.
(a) is wrong as it talks about avoiding all trade liberalisation
policies and all subsidies which is more of a solution of
the problem of inflation but not a message of the
passage.
(b) is wrong as it talks about reasons for India's non-
readiness to take up the trade liberalisation process. It
is out of scope of the passage
43. (a) (a) is correct as the passage talks about the conflict Task 4 no. of ways = 3
between man and State. Moreover the passage confirms Task 5 no. of ways = 3
that this is a continuous tussle and is as old as the state Total no. of ways for condition = 3 + 3 + 3 + 2 + 1
history. The solution seems to be' still far off suggests that = 12
the conflict will remain unresolved. Condition II
(b) and (d) are wrong as they present only the idea When task T2 is given to be person 4
contained in the last line of the passage. Task
(c) is wrong as the passage clearly mentions that old
values and ideas constantly yield place to new ones which
1 2 3 4 5
is contrary to what is mentioned in (c). Moreover it is not
the author's viewpoint. No. of ways for Task T1 = 2
44. (c) The passage clearly talks about 1 and 3. Refer the 4th No. of ways for Task T2 = 1
sentence, 'The scale and magnitude of the financial support No. of ways for Task T3 = 3
______ multilateral negotiations under the United Nations No. of ways for Task T4 = 3
Framework Convention on Climate Change (UNFCCC)', No. of ways for Task T5 = 3
Total number of ways for condition II
which mentions 1 and 3 both. Crop failure, which is an effect
= 3 + 3 + 3 + 2 +1
of climate change, is not mentioned in the passage.
= 12
45. (c) Refer the 5th sentence, 'The Convention squarely puts Total number of ways for condition I and II= 12 + 12
the _______ stock of greenhouse gases (GHGs) in the = 24
atmosphere', which clearly mentions 3. 1 and 2 are not 50. (a) Let the Income of Peter and Paul are 4x and 3x
mentioned in the passage. Let the Expenses of Peter and Paul are 3y and 2y
46. (a) Refer the second last sentence, 'Given the magnitude So, According to question
of the task and the funds required, domestic finances are 4x – 3y = 6000 ... (i)
likely to fall short of the current and projected needs of the 3x – 2y = 6000 ... (ii)
developing countries', which clearly shows that climate Now, From equation (i) and equation (ii)
change is likely to have implications on theirdomestic 4x – 3y = 3x – 2y
finances. 2 is not mentioned in the passage. \ x = yÞ y= x
47. (a) The passage clearly depicts the tussle between Now, put the value of y in equation (i), we get
Þ x = 6000
developed and developing countries regarding support for
\ 4x = 24000
mitigation. The developing nations need financial support
3x = 18000
to adapt to the new measures and they want the developed Monthly Incomes of Peter and Paul are ` 24000 and `
nations to bear the expense of what they have caused. (b), 18000 respectively.
(c) and (d) are wrong because the passage does not mention 51. (b) Average speed of two cars at a speed V1 and V2 km/hr
either of exploitation of natural resources, Lack of political
will or Governance problems.
2V1V2 2 ´ 40 ´ 60
= V +V =
48. (c) Between 6 PM and 7 PM, the hour hand and the minute 1 2 60 + 40
hand coincide at 6 hr ‘m’ minutes.
2 ´ 40 ´ 60
30h ´ 2 30 ´ 6 ´ 2 8 = = 48 km/hr
i.e. m = = = 32 mins. 100
11 11 11
Sol. 52 & 53. E A C B/D F D/B
Hence, the hour hand and minute hand coincide at 6 hr
22 21 20 19 18 17
8
32 mins. 52. (b) F is 18 years old.
11 53. (b) Two orders is possible in increasing age.
After 3 minutes, i.e. at 6 : 36 PM the minute hand of a 54. (c) Let x be the number of women.
clock will be ahead of the hourhand by 3 minutes. Let y be the number of men.
49. (c) Here are five persons, and 5 tasks Total number of hand shakes = xy = 24
So, When T2 task is fixed for person 3 Then, the possible factors of x and y are x = 6 or 4, y =
Task 4 or 6
T2 Number of hugs = x C2 + yC2
1 2 3 4 5 6´5 4´3
= 6 C2 + 4 C 2 = +
For Task 1 no. of ways = 2 2 ´1 2
Task 2 no. of ways = 1 = 15 + 6 = 21
Task 3 no. of ways = 3
55. (d) It is clear that more men cleared the examination than
Shabnam Marathi No Driving women.
61. (b) A corollary is a proposition that follows from one
Anil Tamil No Driving
already proved. (b) is the most appropriate as the passage
Tamil/ clearly states that poverty and climate impacts reinforce
Rekha Marathi Driving
each other. Poverty leads to climate deterioration (due to
David Marathi/ unsustainable use of resources, refer last line) and climate
Driving
Tamil change leads to poverty.
From above table, it can be concluded that only of (a) and (c) are wrong as they talk about poverty
those who can drive car can speaks Tamil. alleviation, increase in food subsidies, etc. in alienation.
They have got nothing to do with climate change.
56. (d) X U Z (d) is wrong as it suggests solution to curb climate
U change (stop unsustainable agricultural practices) and
poverty (control food prices). It does not talk about their
W
inter-dependence.
V 62. (d) The passage asks for the most rational and critical
inference from the passage. (a) is wrong as it just portrays
Y one part of the passage. The portfolio investments have
From above diagram, it is clear that Y, V and W are both the good and bad effects. (b) is wrong as the passage
parallel. suggests threat from the portfolio investments. It cannot be
57. (d) Let ‘g’ be the cost of goats. stated from the passage that Advanced economies will
4g < Cow < 5g always undermine the global financial stability. There are a
Now, Given a goat price = ` 600 lot of other external factors working simultaneously. For
2400 < Cow < 3000 ... (i) similar reasons (c) is wrong. (d) is the most rational and
Given, a goat price = ` 800 critical inference as portfolio investments definitely impacts
3200 < Cow < 4000 ... (ii) emerging economies in certain cases so there is a risk.
Hence, it can be seen that from above equations that Note : Portfolio investments: Portfolio investments are
cow cost is between ` 2400 and ` 4000. passive investments, as they do not entail active
management or control of the issuing company. Rather,
58. (b) A Fighters the purpose of the investment is solely financial gain.
Enemy This is in contrast to foreign direct investment (FDI),
B Cowards which allows an investor to exercise a certain degree
of managerial control over a company. For international
C Cowards transactions, equity investments where the owner
Friends
Enemy holds less than 10% of a company's shares are
D Cowards
classified as portfolio investments. These transactions
E Fighters are also referred to as "portfolio flows" and are
recorded in the financial account of a country's balance
F Fighters of payments.Portfolio investments include
transactions in equity securities, such as common
From above table, it is clear that A, E and F are fighters. stock, and debt securities, such as banknotes, bonds,
59. (b) White Marbles Red Marbles and debentures.
10 13 Quantitative Easing: Quantitative easing (QE) is a type
White Marbles Green Marbles of monetary policy used by central banks to stimulate
10 . y 5 the economy when standard monetary policy has
Now, total number of Marbles = 5 + 10 + 13 = 28 become ineffective. A central bank implements
60. (c) Let initial population be100 quantitative easing by buying financial assets from
Men Women commercial banks and other financial institutions, thus
60% 40% raising the prices of those financial assets and lowering
60 40 their yield, while simultaneously increasing the money
70% 75% supply. This differs from the more usual policy of
42 30 buying or selling short-term government bonds to keep
80% 70% interbank interest rates at a specified target value.
33 21
Quantitative easing can help ensure that inflation does 68. (d) Number of ways to select Principal = 2C1
not fall below a target. Risks include the policy being Number of ways to select Vice Principal = 5C2
more effective than intended in acting against deflation Total number of ways = 2C1 + 5C2
(leading to higher inflation in the longer term, due to 5´4
= 2+ = 2 + 10
increased money supply), or not being effective 2 ´1
enough if banks do not lend out the additional Number of possible combinations of selectres
reserves. = 2 × 10 = 20
63. (b) (a) is out of context of the passage. The passage 69. (c) If mathematics I is not opted, then two subjects out of
does not talk about resources to afford a latrinefor each four subjects have to be opted for.
household. \ Number of ways in which two subjects can be opted
(c) is wrong as the passage does not talk about the 4´3
reduction of the human capital of India's workforce. for =6
2
(d) is wrong as the passage suggests open defecation
It mathematics II is opted, then it can be offered only if
as a public health problem in India. Refer last 3rd
mathematics I is also opted for Number of ways in
sentence, 'Diarrhoea leaves Indians' bodies smaller on
which two subjects can be opted for
average than those of people in some poorer countries
= 6 +1 = 7.
where people eat fewer calories.'
70. (d) Let number of a pairs of brown socks = y
(b) is the most critical inference as it has severe effects
Price of brown socks = x
- malnourishment, diarrhoea, underweight mothers -
Price of black socks = 3x
on a huge Indian population and needs immediate
According to question
attention.
Þ 5 ´ 3x + yx = 100 ... (i)
64. (b) (a) is wrong as it is a general statement and does
Now, clerk has interchanged socks pairs then price is
present the crux of the passage.
increased by 100%
(c) is wrong as the opening line of the passage says
that people talk about democracy (or understand (15 x + yx) ´ 100
democracy) but they have a weakness favouring their 3xy + 5x = (15x + yx) +
100
caste or community or religion. = 3xy + 5x = 30x + 2xy ... (ii)
(d) is wrong as it projects an extreme situation which Þ 30x + 2yx = 3xy + 5x
is out of the scope of the passage. Þ 25x = xy
(b) is the most appropriate statement as it talks about y = 25
providing equal opportunities to all irrespective of the \ So, number of brown socks = 25
caste or community or religion. That's what is a True 71. (*) Data Inconsistent.
democracy. 72. (a) From given years 2000 to 2007
65. (b) Clearly only 2 is correct. 1 is wrong as the passage Earnings of A > Earnings of B
says that 'Such access is not always available to all people So,
in developing countries like India and more so, in rural areas.' Average of A > Average of B
This means that they are sometimes available.2 is correct as Hence, It can be conducted that the average A earned
mentioned in the last sentence of the passage. more than B during this period.
66. (d) The crucial message conveyed in the passage is to
promote financial inclusion such that every citizen of the 30 ´ 20
73. (b) Time taken by both tanks =
country has access to bank accounts and can save money 30 + 20
so as to have better finances . Establishing more banks is a 30 ´ 20
route to financial inclusion. Increasing the interest rate of =
50
bank deposits would promote savings. So (d) is correct.
67. (b) (a) is clearly wrong as the passage nowhere talks = 12 min
about discretionpower in all matters of administration. 74. (c) From 2 and 3, I and B are on opposite faces. from 1 and
It talks about certain situations mentioned in the 1st 4, Y and V are adjacent to O.
sentence.
(b) is correct as the passage talks about the supremacy Y
of rules and safeguards, refer last sentence. Further I O B
the 3rd sentence, 'Where discretion has to be ................ V
misuse of that power'reinforces the prevalence of rules
and safeguards as opposed to the influence of exclusive G
discretion of authority.
(c) is wrong as it negates what the passage mean by Hence, G is the colour of the face opposite to the face
Parliamentary democracy - authorized by the people. coloured with O.
78. (c) From the graph commodity price are between ` 10 and
75. (d) Men Great ` 20.
Wise
50
79. (c) A spent on food = 20, 000 ´
100
= `10000
or
10
B spent on food = 1, 00, 000 ´
100
= `100, 00
Wise Men So, Both A and B spent same amount on the food.
80. (b) According to question,
Priti < Swati < Kamala < Usha
Great From the above, it can concluded that priti is the
slowest runner.

Conclusion I : False
Conclusion II : False
76. (d)
77. (c) ABC × DEED = ABCABC
We know that, when we write any Number in form of
ABCABC then that number must be divisible by 13 , 11
and 7 or ABCABC is multiple of 1001
So, 1001 = DEED
1001 = 13 × 11 × 7
ABC × DEED = ABC ABC
Þ ABC × 1001 = ABC ABC
On comparing
D = 1, E = 0
21

2016 SOLVED PAPER 2

This Paper is taken from our :

ISBN : 9789386146823
22

CSAT PAPER-2 2016


GENERAL COMPREHENSION PASSAGE-3
Understanding of the role of biodiversity in the
DIRECTIONS for the following 8 (eight) items : Read hydrological cycle enables better policy-making. The
the following eight passages and answer the item that term biodiversity refers to the variety of plants, animals,
follows each passage. Your answers to these items microorganisms, and the ecosystems in which they
should be based on the passages only. occur. Water and biodiversity are interdependent. In
PASSAGE-1 reality, the hydrological cycle decides how biodiversity
functions. In turn, vegetation and soil drive the
By killing transparency and competition, crony
movement of water. Every glass of water we drink has, at
capitalism is harmful to free enterprise, opportunity and
least in part, passed through fish, trees, bacteria, soil and
economic growth. Crony capitalism, where rich and the
other organisms. Passing through these ecosystems, it is
influential are alleged to have received land and natural
resources and various licences in return for payoffs to cleansed and made fit for consumption. The supply of
venal politicians, is now a major issue to be tackled. One water is a critical service that the environment provides.
of the greatest dangers to growth of developing 3. Which among the following is the most critical
economies like India is the middle-income trap where inference that can be made from the above passage ?
crony capitalism creates oligarchies that slow down the (a) Biodiversity sustains the ability of nature to
growth. recycle water
1. Which among the following is the most logical (b) We cannot get potable water without the
corollary to the above passage ? existence of living organisms
(a) Launching more welfare schemes and (c) Plants, animals and microorganisms
allocating more finances for the current continuously interact among themselves
schemes are urgently needed (d) Living organisms could not have come into
(b) Efforts should be made to push up economic existence without hydrological cycle
growth by other means and provide licences to PASSAGE-4
the poor In the last decade, the banking sector has been
(c) Greater transparency in the functioning of the restructured with a high degree of automation and
government and promoting the financial
products that mainly serve middle-class and upper
inclusion are needed at present
middle-class society. Today there is need for a new
(d) We should concentrate more on developing
agenda for the banking and non-banking financial
manufacturing sector than service sector
services that does not exclude the common man.
PASSAGE-2 4. Which one of the following is the message that is
Climate adaptation may be rendered ineffective if essentially implied in the above passage?
policies are not designed in the context of other (a) Need for more automation and more products
development concerns. For instance, a comprehensive of bank
strategy that seeks to improve food security in the (b) Need for a radical restructuring of our entire
context of climate change may include a set of public finance system
coordinated measures related to agricultural extension, (c) Need to integrate banking and non-banking
crop diversification, integrated water and pest institutions
management and agricultural information series. Some (d) Need to promote financial inclusion
of these measures may have to do with climate changes
PASSAGE-5
and others with economic development.
2. What is the most logical and rational inference that Safe and sustainable sanitation in slums has immeasurable
can be made from the above passage? benefits to women and girls in terms of their health,
(a) It is difficult to pursue climate adaptation in the safety, privacy and dignity. However, women do not
developing countries feature in most of the schemes and policies on urban
(b) Improving food security is a far more complex sanitation. The fact that even now the manual scavenging
issue than climate adaptation exists, only goes to show that not enough has been done
(c) Every developmental activity is directly or to promote pour-flush toilets and discontinue the use of
indirectly linked to climate adaptation dry latrines. A more sustained and rigorous campaign
(d) Climate adaptation should be examined in needs to be launched towards the right to sanitation on a
tandem with other economic development very large scale. This should primarily focus on the
options abolition of manual scavenging.
23
5. With reference to the above passage, consider the (a) Organic farming should immediately replace
following statements: mechanised and chemical dependant
1. Urban sanitation problems can be fully solved agricultural practices all over the world
by the abolition of manual scavenging only (b) It is imperative for us to modify our land use
2. There is a need to promote greater awareness practices in order to mitigate climate change.
on safe sanitation practices in urban areas. (c) There are no technological solutions to the
Which of the statements given above is/are problem of greenhouse gas emissions
correct? (d) Tropical areas are the chief sites of carbon
(a) 1 only (b) 2 only sequestration
(c) Both 1 and 2 (d) Neither 1 nor 2 DIRECTIONS for the following 8 (eight) items : Read
PASSAGE-6 the following five passages and answer the items that
follow each passage. Your answers to these items
To understand the nature and quantity of Government
should be based on the passages only.
proper for man, it is necessary to attend to his character.
As nature created him for social life, she fitted him for the PASSAGE-1
station she intended. In all cases she made his natural As we look to 2050, when we will need to feed two billion
wants greater than his individual powers. No one man is more people, the question of which diet is best has taken
capable, without the aid of society, of supplying his own on new urgency. The foods we choose to eat in the
wants; and those wants, acting upon every individual, coming decades will have dramatic ramifications for the
impel the whole of them into society. planet. Simply put, a diet that revolves around meat and
6. Which among the following is the most logical and dairy a way of eating that is on the rise throughout the
rational inference that can be made from the above developing world, will take a greater toll on the world’s
passage ? resources than one that revolves around unrefined
(a) Nature has created a great diversity in human grains, nuts, fruits and vegetables.
society 9. What is the critical message conveyed by the above
(b) Any given human society is always short of its passage?
wants (a) Our increasing demand for foods sourced from
(c) Social life is a specific characteristic of man animals puts a greater burden on our natural
(d) Diverse natural wants forced man towards resources
social system (b) Diets based on grains, nuts, fruits and
vegetables are best suited for health in
PASSAGE-7 developing countries
The nature of the legal imperatives in any given state (c) Human beings change their food habits from
corresponds to the effective demands that state time to time irrespective of the health concerns
encounters, and that these, in their turn, depend, in a (d) From a global perspective, we still do not know
general way, upon the manner in which economic power which type of diet is best for us
is distributed in the society which the state controls. PASSAGE-2
7. The statement refers to: All humans digest mother’s milk as infants, but until
(a) the antithesis of Politics and Economics cattle began being domesticated 10,000 years ago,
(b) the interrelationship of Politics and Economics children once weaned no longer needed to digest milk.
(c) the predominance of Economics over Politics As a result, they stopped making the enzyme lactase,
(d) the predominance of Politics over Economics which breaks down the sugar lactose into simple sugars.
PASSAGE-8 After humans began herding cattle, it became
tremendously advantageous to digest milk, and lactose
About 15 percent of global greenhouse gas emissions
tolerance evolved independently among cattle herders in
come from agricultural practices. This includes nitrous
Europe, the middle East and Africa. Groups not
oxide from fertilizers; methane from livestock, rice
dependant on cattle, such as the Chinese and Thai,
production, and manure storage; and carbon dioxide remain lactose intolerant.
(CO2) from burning biomass, but this excludes CO2 10. Which among the following is the most logical
emissions from soil management practices, sayannah assumption that can be made from the
burning and deforestation. Foresty and use, and land-use above passage?
change account for another 17 percent of greenhouse gas (a) About 10,000 years ago, the domestication of
emissions each year, three quarters of which come from animals took place in some parts of the world
tropical deforestation. The remainder is largely from (b) A permanent change in the food habits of a
draining and burning tropical peatland. About the same community can bring about a genetic change in
amount of carbon is stored in the world’s peatlands as is its members
stored in the Amazon rainforest. (c) Lactose tolerant people only are capable of
8. Which among the following is the most logical and getting simple sugars in their bodies
rational inference that can be made from the above (d) People who are not lactose tolerant cannot
passage? digest any dairy product
24
PASSAGE-3 is not so much and not always, of a constraint for an
“The conceptual difficulties in National Income open, and growing economy, which has adequate
comparisons between underdeveloped and industrialised exchange surplues to buy food abroad. For the world as a
countries are particularly serious because a part of the whole, Spply-demand balance is always an inescapable
national output in various underdeveloped countries is prerequisite for warding off hunger and starvation.
produced without passing through the commercial However, global availability of adequate supply does not
channels.” necessarily mean that food would automatically move
11. In the above statement, the author implies that: from countries of surplus to counteries of deficit if the
latter lack in purchasing power. The uneven distribution
(a) the entire national output produced and
of Hunger, starvation, under or malnourishment, etc., at
consumed in industrialized countries passes
the world-level, thus owes itself to the presence of empty-
through commercial channels
pock hungry mouths, overwhelmingly confined to the
(b) the existence of a non-commercialized sector in
underdeveloped economies. Inasmuch as ‘a two-square
different underdeveloped countries renders the
meal’ is of elemental significance to basic human
national income comparisons over countries
existence, the issue of worldwide supply` of food has
difficult
been gaining significance, in recent times, both because
(c) no part of national output should be produced the quantum and the composition of demand has been
and consumed without passing through commercial undergoing big changes, and because, in recent years, the
channels capabilities individual countries to generate
(d) a part of the national output being produced uninterrupted chain of food supplies have come under
and consumed without passing through commercial strain. Food production, marketing and prices, especially
channels is a sign of underdevelopment price-affordability by the poor in the developing world,
PASSAGE-4 have become global issues that need global thinking and
An increase in human-made carbon dioxide in the global solutions.
atmosphere could initiate a chain reaction between 13. According to the above passage, which of, the
plants and microorganisms that would unsettle one of following are the fundamental solutions for the
the largest carbon reservoirs on the planet–soil. In a world food security problem?
study, it was found that the soil, which contains twice the 1. Setting up more agro-based industries
amount of carbon present in all plants and Earth’s 2. Improving the price affordability by the poor
atmosphere combined, could become increasingly 3. Regulating the conditions of marketing
4. Providing food subsidy to one and all
volatile as people add more carbon dioxide to the
Select the correct answer using the code given
atmosphere. This is largely because of increased plant
below:
growth. Although a greenhouse gas and a pollutant,
(a) 1 and 2 (b) 2 and 3 only
carbon dioxide also supports plant growth. As trees and
(c) 1, 3 an 4 only (d) 1, 2, and 4
other vegetation flourish in a carbon dioxide-rich future,
14. According to the above passage, the
their roots could stimulate microbial activity in soil that
biggest challenge to world agriculture is:
may in turn accelerate the decomposition of soil carbon
(a) to find sufficient land for agriculture and to
and its release into the atmosphere as carbon dioxide.
expand food processing industries
12. Which among the following is the most logical (b) to eradicate hunger in underdeveloped
corollary to the above passage ? countries
(a) Carbon dioxide is essential for the survival of (c) to achieve a balance between the production of
microorganisms and plants food and non-food items
(b) Humans are solely responsible for the release of (d) to achieve a balance between demand for and
carbon dioxide into the atmosphere supply of food
(c) Microorganisms and soil carbon are mainly 15. According to the above passage, which of the
responsible for the increased plant growth following helps/help in reducing hunger and
(d) Increasing green cover could trigger the release starvation in the developing economies ?
of carbon trapped in soil 1. Balancing demand and supply of food
PASSAGE-5 2. Increasing imports of food
Historically, the biggest challenge to world agriculture 3. Increasing purchasing power of the poor
has been to achieve a balance between demand for and 4. Changing the food consumption patterns and
supply of food. At the level of individual countries, the practices
demand-supply balance can be a critical issue for a Select the correct answer using the code given
closed economy, especially if it is a populous economy below:
and its domestic agriculture is not growing sufficiently (a) 1 only (b) 2, 3 and 4 only
enough to ensure food supplies, on an enduring basis; it (c) 1 and 3 only (d) 1, 2, 3 and 4
25
16. The issue of worldwide supply of food has gained bring government and decision making closer to the
importance mainly because of: people also helps to enhance accountability.
1. overgrowth of the population worldwide 17. According to the passage, which of the following
2. sharp decline in the area of food production factor/factors led to the adverse consequences for
3. limitation in the capabilities for sustained governance/public administration?
supply of food 1. Inability of civil services to strike a balance
Select the correct answer using the code given between internal and external accountabilities
below: 2. Lack of sufficient professional training to the
(a) 1 and 2 only (b) 3 only officers of All India Services
(c) 2 and 3 only (d) 1, 2 and 3 3. Lack of proper service benefits in civil services
4. Lack of Constitutional provisions to define the
DIRECTIONS for the following 6 (six) items: Read the
respective roles of professional civil services
following two passages and answer the items that
vis-a-vis political executive in this context
follow each passage. Your answers to these items
Select the correct answer using the code given
should be based on the passages only.
below :
PASSAGE-1 (a) 1 only (b) 2 and 3 only
Accountability, or the lack of it, in governance generally, (c) 1 and 4 only (d) 2, 3 and 4
and civil services, in particular, is a major factor 18. With reference to the passage, the following
underlying the deficiencies in governance and public assumptions have been made :
administration. Designing an effective framework for 1. Political executive is an obstacle to the
accountability has been a key element of the reform accountability of the civil services to the
agenda. A fundamental issue is whether civil services society
should be accountable to the political executive of the 2. In the present framework of Indian polity, the
day or to society at large. In other words, how should political executive is no longer accountable to
internal and external accountability be reconciled? the society
Internal accountability is sought to be achieved by Which of these assumptions is/are valid?
internal performance monitoring, official supervision by (a) 1 only
bodies like the–Central–Vigilance Commission–and– (b) 2 only
Comptroller and Auditor– General, and judicial review (c) Both 1 and 2
of executive decisions. Articles 311 and 312 of the Indian (d) Neither 1 nor 2
Constitution provide job security and safeguards to the 19. Which one of the following is the essential message
civil services, especially the All India Services. The implied by this passage?
framers of the Constitution had envisaged that provision (a) Civil services are not accountable to the society
of these safeguards would result in a civil service that is they are serving
not totally subservient to the political executive but will (b) Educated and enlightened persons are not
have the strength to function in larger public interest. taking up political leadership
The need to balance internal and external accountability (c) The framers of the Constitution did not
is thus built into the Constitution. The issue is where to envisage the problems being encountered by
draw the line. Over the years, the emphasis seems to have the civil services
tilted in favour of greater internal accountability of the (d) There is a need and scope for reforms to
civil services to the political leaders of the day who in improve the accountability of civil services
turn are expected to be externally accountable to the 20. According to the passage, which one of the
society at large through the election process. This system following is not a means of enhancing internal
for seeking accountability to society has not worked out, accountability of civil services?
and has led to several adverse consequences for (a) Better job security and safeguards
governance. (b) Supervision by Central Vigilance Commission
Some special measures can be considered for (c) Judicial review of executive decisions
improving accountability in civil services. Provisions of (d) Seeking accountability through enhanced
articles 311 and 312 should be reviewed and laws and participation by people in decision making
regulations framed to ensure external accountability of process
civil services. The proposed Civil Services Bill seeks to PASSAGE-2
address some of these requirements. The respective roles In general, religious traditions stress our duty to god, or
of professional civil services and the political executive to some universal ethical principle. Our duties to one
should be defined so that professional managerial another derive from these. The religious concept of rights
functions and management of civil services are is primarily derived from our relationship to this divinity
depoliticized. For this purpose, effective statutory civil or principle and the implication it has on our other
service boards should be created at the centre and in the relationships. This correspondence between rights and
states. Decentralization and devolution of authority to duties is critical to any further understanding of justice.
26
But, for justice to be practiced; viture, rights and duties and storage, an unproven technology, to achieve negative
cannot remain formal abstractions. They must be emissions and to buy some time during the first half of
grounded in a community (common unity) bound the century.
together by a sense of common union (communion). Some liquid biofuels such as corn-based ethanol,
Even as a personal virtue, this solidarity is essential to mainly for transport, may aggravate rather than
the practice and understanding of justice. ameliorate carbon emissions on a life-cycle basis. Second
21. With reference to the passage, the following generation biofuels, based on ligno-cellulosic feedstocks
assumptions have been made : – such as straw, bagasse, grass and wood – hold the
1. Human relationships are derived from their promise of sustainable production that is high-yielding
religious traditions and emit low levels of greenhouse gases, but these are
2. Human beings can be duty bound only if they still in the R & D stage.
believe in god 23. What is/are the present constraint/constraints in
3. Religious traditions are essential to practice and using biomass as fuel for power generation?
understand justice 1. Lack of sustainable supply of biomass
Which of these assumption(s) is/are valid? 2. Biomass production competes with food
(a) 1 only (b) 2 and 3 only production
(c) 1 and 3 only (d) 1, 2 and 3 3. Bio-energy may not always be low carbon on a
22. Which one of the following is the crux of this life-cycle basis
Select the correct answer using the code given
passage?
below:
(a) Our duties to one another derive from our
(a) 1 and 2 only (b) 3 only
religious traditions
(c) 2 and 3 only (d) 1, 2 and 3
(b) Having relationship to the divine principle is a
24. Which of the following can lead to food security
great virtue
problem?
(c) Balance between rights and duties is crucial to
1. Using agricultural and forest residues as
the delivery of justice in a society
feedstock for power generation
(d) Religious concept of rights is primarily derived
2. Using biomass for carbon capture and storage
from our relationship to god
3. Promoting the cultivation of energy crops
DIRECTIONS for the following 5 (five) items : Read the Select the correct answer using the code given
following two passages and answer the items that below:
follow each passage. Your answers to these items (a) 1 and 2 only (b) 3 only
should be based on the passages only. (c) 2 and 3 only (d) 1, 2 and 3
25. In the context of using biomass, which of the
PASSAGE-1 following is/are the characteristic/characteristics of
Biomass as fuel for power, heat, and transport has the the sustainable production of biofuel?
highest mitigation potential of all renewable sources. It 1. Biomass as a fuel for power generation could
comes from agriculture and forest residues as well as meet all the primary energy requirements of the
from energy crops. The biggest challenge in using world by 2050
biomass residues is a long-term reliable supply delivered 2. Biomass as a fuel for power generation does not
to the power plant at reasonable costs; the key problems necessarily disrupt food and forest resources
are logistical constraints and the costs of fuel collection. 3. Biomass as a fuel for power generation could
Energy crops, if not managed properly, compete with help in achieving negative emissions, given
food production and may have undesirable impacts on certain nascent technologies
food prices. Biomass production is also sensitive to the Select the correct answer using the code given
physical impacts of a changing climate. below:
Projections of the future role of biomass are (a) 1 and 2 only (b) 3 only
probably overestimated, given the limits to the (c) 2 and 3 only (d) 1, 2 and 3
sustainable biomass supply, unless breakthrough 26. With reference to the passage, following
technologies substantially increase productivity. Climate assumptions have been mad :
-energy models project that biomass use could increase 1. Some climate-energy models suggest that the
nearly four-fold to around 150 – 200 exajoules, almost a use of biomass as a fuel for power generation
quarter of world primary energy in 2050. However the helps in mitigating greenhouse gas emissions
maximum sustainable technical potential of biomass 2. It is not possible to use biomass as a fuel for
resources (both residues and energy crops) without power generation without disrupting food and
disruption of food and forest resources ranges from 80 – forest resources
170 exajoules a year by 2050, and only part of this is Which of these assumptions is/are valid?
realistically and economically feasible. In addition, some (a) 1 only (b) 2 only
climate models rely on biomass-based carbon capture (c) Both 1 and 2 (d) Neither 1 nor 2
27
PASSAGE-2 5. Ribeiro does not like to wear the black colour
6. Each of the friends wore only one outer garment of
We are witnessing a dangerous dwindling of biodiversity
only one colour
in our food supply. The green revolution is a mixed
29. What is C’s surname ?
blessing. Over time farmers have come to rely heavily on
(a) Riberio (b) Kumar
broadly adapted, high yield crops to the exclusion of
(c) Singh (d) Cannot be determined
varieties adapted to the local conditions. Monocropping 30. What is the colour of the tie ?
vast fields with the same genetically uniform seeds helps (a) Black (b) Blue
boost yield and meet immediate hunger needs. Yet high- (c) White (d) Cannot be determined
yield varieties are also genetically weaker crops that 31. Who wore the sweater ?
require expensive chemical fertilizers and toxic (a) A (b) B
pesticides. In our focus on increasing the amount of food (c) C (d) Cannot be determined
we produce today, we have accidentally put ourselves at 32. A person walks 12 km due north, then 15 km due
risk for food shortages in future. east, after that 19 km due west and then 15 km due
27. Which among the following is the most logical and south. How far is he from the starting point?
critical inference that can be made from the above (a) 5 km (b) 9 km
passage? (c) 37 km (d) 61 km
(a) In our agricultural practices, we have become 33. A cube has all its faces painted with different
heavily dependent on expensive chemical colours. It is cut into smaller cubes of equal sizes
fertilizers and toxic pesticides only due to green such that the side of the small cube is one-fourth the
big cube. The number of small cubes with only one
revolution
of the sides painted is:
(b) Monocropping vast fields with high-yield
(a) 32 (b) 24
varieties is possible due to green revolution
(c) 16 (d) 8
(c) Monocropping with high-yield varieties is the
34. A military code writes SYSTEM as SYSMET and
only way to ensure food security to millions NEARER as AENRER. Using the same code,
(d) Green revolution can pose a threat to FRACTION can be written as:
biodiversity in food supply and food security in (a) CARFTION (b) FRACNOIT
the long run (c) NOITCARF (d) CARFNOIT
35. Four-digit numbers are to be formed using the digits
MENTAL, ABILITY, ANALYTICAL
1, 2, 3 and 4; and none of these four digits are
& LOGICAL REASONING repeated in any manner. Further,
28. A person climbs a hill in a straight path from point 1. 2 and 3 are not to immediately follow each
‘O’ on the ground in the direction of north-east and other
reaches a point ‘A’ after travelling a distance of 5 km. 2. 1 is not to be immediately followed by 3
Then, from the point ‘A’ he moves to point ‘B’ in the 3. 4 is not to appear at the last place
direction of north-west. Let the distance AB be 12 4. 1 is not to appear at the first place
km. Now, how far is the person away from the How many different numbers can be formed?
starting point ‘O’? (a) 6 (b) 8
(a) 7 km (b) 13 km (c) 9 (d) None of the above
(c) 17 km (d) 11 km 36. In a class of 60 students, where the number of girls is
twice that of boys, Kamal, a boy, ranked seventeenth
DIRECTIONS for the following 3 (three) items : from the top. If there are 9 girls ahead of Kamal, the
Consider the given information and answer the three number of boys in rank after him is:
items that follow. (a) 13 (b) 12
When three friends A, B and C met, it was found that (c) 7 (d) 3
each of them wore an outer garment of a different colour. 37. A person X was driving in a place where all roads
In random order, the garments are: jacket, sweater and ran either north-south or east-west, forming a grid.
tie; and the colours are: blue, white and black. Their Roads are at a distance of 1 km from each other in a
surnames in random order are : Ribeiro, Kumar and parallel. He started at the intersection of two roads,
Singh. drove 3 km north, 3 km west and 4 km south. Which
Further, we know that : further route could bring him back to his starting
1. neither B nor Ribeiro wore a white sweater point, if the same route is not repeated?
2. C wore a tie (a) 3 km east, then 2 km south
3. Singh’s garment was not white (b) 3 km east, then 1 km north
(c) 1 km north, then 2 km west
4. Kumar does not wear a jacket
(d) 3 km south, then 1 km north
28
38. Consider the following statement: DIRECTIONS for the following 5 (five) items: Consider
“We shall go either for a picnic or for trekking”. the following information and answer the five items
Which of the following, if true, would falsify this that follow:
claim?
(a) We go for a picnic but not for trekking There are five persons in a group — P, Q, R, S and T. The
(b) Activities such as picnic and trekking are group has one doctor, one lawyer and one artist. P and S
encouraged by the health authorities are unmarried students. T is a man married to one of the
(c) We go for trekking and not for picnic group members. Q is the brother of P and is neither
(d) We do not go either for picnic or for trekking doctor nor artist. R is not doctor.
39. There were 50 faculty members comprising 30 43. Who is the doctor?
males and the rest females. No male faculty member (a) T (b) P
knew music, but many of the female faculty (c) Q (d) R
members did. The Head of the institution invited six 44. Who is the artist?
faculty members to a tea party by draw of lots. At (a) P (b) Q
the party it was discovered that no member knew (c) R (d) T
music. The conclusion is that: 45. Who is the spouse of R?
(a) the party comprised male faculty members (a) P (b) T
only (c) Q (d) S
(b) the party comprised only those female faculty 46. Who is the lawyer?
members who could not give renderings in (a) P (b) Q
music (c) R (d) S
(c) the party comprised both male and female 47. Who of the following is definitely a man?
faculty members (a) P (b) S
(d) nothing can be said about the gender (b) Q (d) None of the above
composition of the party 48. There is an order of 19000 quantity of a particular
40. Five people A, B, C, D and E are seated about a product from a customer. The firm produces 1000
round table. Every chair is spaced equidistant from quantity of that product per day out of which 5% are
adjacent chairs. unfit for sale. In how many days will the order be
(i) C is seated next to A completed?
(ii) A is seated two seats from D (a) 18 (b) 19
(iii) B is not seated next to A (c) 20 (d) 22
On the basis of above information, which of the 49. Consider the following statements:
following must be true? 1. Either A and B are of the same age or A is older
1. D is seated next to B than B
2. E is seated next to A 2. Either C and D are of the same age or D is older
3. D and C are separated by two seats than C
Select the correct answer using the code given 3. B is older than C
below: Which of the following conclusions can be drawn
(a) 1 only (b) 1 and 2 only from the above statements?
(c) 3 only (d) Neither 1 nor 2 nor 3 (a) A is older than B
41. There are five hobby clubs in a college — (b) B and D are of the same age
(c) D is older than C
photography, yachting, chess, electronics and
(d) A is older than C
gardening. The gardening group meets every
second day, the electronics group meets every third DIRECTIONS for the following 3 (three) items:
day, the chess group meets every fourth day, the Consider the given-formation and answer the three
yachting group meets every fifth day and the items that follow.
photography group meets every sixth day. How
many times do all the five groups meet on the same Six boxes A, B, C, D, E and F have been painted with six
day within 180 days? different colours viz., violet, indigo, blue, green, yellow
(a) 5 (b) 18 and orange and arranged from left to right (not
(c) 10 (d) 3 necessarily either kept or painted with the colours in the
42. There are some nectar-filled flowers on a tree and same order). Each box contains a ball of any one of the
some bees are hovering on it. If one bee lands on following six games: cricket, hockey, tennis, golf, football
each flower, one bee will be left out. If two bees land and volleyball (not necessarily in the same order). The
on each flower, one flower will be left out. The golf ball is in violet box and is not in the box D. The box A
number of flowers and bees respectively are: which contains tennis ball is orange in colour and is at
(a) 2 and 4 (b) 3 and 2 the extreme right. The hockey ball is neither in box D nor
(c) 3 and 4 (d) 4 and 3 in box E. The box C having cricket ball is painted green.
29
The hockey ball is neither in the box painted blue nor in ARITHMETIC
the box painted yellow. The box C is fifth from right and
56. An agricultural field is in the form of a rectangle
next to box B. The box B contains volleyball. The box having length X1 meters and breadth X2 meters (X1
containing the hockey ball is between the boxes and X2 are variable). If X1 + X2 = 40 meters, then the
containing golf ball and volleyball. area of the agricultural field will not exceed which
50. Which one of the following boxes contains the golf one of the following values?
ball? (a) 400 sq m (b) 300 sq m
(c) 200 sq m (d) 80 sq m
(a) F (b) E
57. The sum of the ages of 5 members comprising a
(c) D (d) None of the above family, 3 years ago, was 80 years. The average age of
51. Which of the following statements is/are correct? the family today is the same as it was 3 years ago,
(a) D is painted yellow because of an addition of a baby during the
(b) F is painted indigo intervening period. How old is the baby ?
(c) B is painted blue (a) 6 months (b) 1 year
(c) 2 years (d) 2 years and 6 months
(d) All of the above
58. The total emoluments of two persons are the same,
52. The football is in the box of which colour? but one gets allowances to the extent of 65% of his
(a) Yellow basic pay and the other gets allowances to the extent
(b) Indigo of 80% of his basic pay. The ratio of the basic pay of
(c) Cannot be determined as data are inadequate the former to the basic pay of the latter is:
(d) Blue (a) 16 : 13 (b) 5 : 4
(c) 7 : 5 (d) 12 : 11
53. In a. question paper there are five questions to be
59. A person is standing on the first step from the
attempted and answer to each question has two bottom of a ladder. If he has to climb 4 more steps to
choices - True (T) or False (F). It is given that no two reach exactly the middle step, how many steps does
candidates have given the answers to the five the ladder have?
questions in an identical sequence. For this to (a) 8 (b) 9
happen the maximum number of candidates is: (c) 10 (d) 11
60. AB is a vertical trunk of a huge tree with A being the
(a) 10 (b) 18
point where the base of the trunk touches the
(c) 26 (d) 32 ground. Due to a cyclone, the trunk has been broken
54. A ate grapes and pineapple; B ate grapes and at C which is at a height of 12 meters, broken part is
oranges; C ate oranges, pineapple and apple; D ate partially attached to the vertical portion of the trunk
grapes, apple and pineapple. After taking fruits, B at C. If the end of the broken part B touches the
and C fell sick. In the light of the above facts, it can ground at D which is at a distance of 5 meters from
be said that the cause of sickness was: A, then the original height of the trunk is:
(a) 20 m (b) 25 m
(a) Apple (b) Pineapple
(c) 30 m (d) 35 m
(c) Grapes (d) Oranges 61. Ram and Shyam work on a job together for four
55. Consider the following statements. days and complete 60% of it. Ram takes leave then
1. The rate of population growth is increasing in and Shyam works for eight more days to complete
the country the job. How long would Ram take to complete the
2. The death rate is declining faster in the country entire job alone?
(a) 6 days (b) 8 days
compared to birth rate
(c) 10 days (d) 11 days
3. The birth rate is declining faster in the country 62. If R and S are different integers both divisible by 5,
compared to death rate then which of the following is not necessarily true?
4. Rural-urban migration is taking place regularly (a) R – S is divisible by 5
in the country (b) R + S is divisible by 10
Which one of the following conclusions may be true (c) R × S is divisible by 25
(d) R2 + S2 is divisible by 5
in the light of the above facts?
63. How many numbers are there between 100 and 300
(a) The rate of population growth is increasing due which either begin with or end with 2?
to rural-urban migration (a) 110 (b) 111
(b) The rate of population growth is increasing due (c) 112 (d) None of the above
to decline in death rate only 64. W can do 25% of a work-in 30 days, X can do 1/4 of
(c) The rate of population growth is increasing due the work in 10 days, Y can do 40% of the work in 40
to increase in birth rate only days and Z can do 1/3 of the work in 13 days. Who
will complete the work first?
(d) The rate of population growth is increasing due (a) W (b) X
to faster decline in death rate than in birth rate (c) Y (d) Z
30
65. The average monthly income of a person in a certain students in the class, what is the number of girls in
family of 5 is ` 10,000. What will be the average monthly the class?
income of a person in the same family if the income (a) 6 (b) 12
of one person increased by ` 1,20,000 per year? (c) 18 (d) 21
(a) ` 12,000 (b) ` 16,000 74. The monthly average salary paid to all the
(c) ` 20,000 (d) ` 34,000 employees of a company was ` 5000. The monthly
66. In a race, a competitor has to collect 6 apples which average salary paid to male and female employees
are kept in a straight line on a track and a bucket is was ` 5200 and ` 4200 respectively. Then the
placed at the beginning of the track which is a percentage of males employed in the company is :
starting point. The condition is that the competitor (a) 75% (b) 80%
can pick only one apple at a time, run back with it (c) 85% (d) 90%
and drop it in the bucket. If he has to drop all the 75. Two numbers X and Y are respectively 20% and 28%
apples in the bucket, how much total distance he less than a third number Z. By what percentage is
has to run if the bucket is 5 meters from the first the number Y less than the number X ?
apple and all other apples are placed 3 meters apart ? (a) 12% (b) 10%
(a) 40 m (b) 50 m (c) 9% (d) 8%
(c) 150 m (d) 75 m
67. A round archery target of diameter 1 m is marked MENSURATION
with four scoring regions from the centre outwards
as red, blue, yellow and white. The radius of the red 76. A cylindrical overhead tank of radius 2 m and
band is 0.20 m. The width of all the remaining bands height 7 m is to be filled from an underground tank
is equal. If archers throw arrows towards the target, of size 5.5 m × 4 m × 6 m. How much portion of the
what is the probability, that the arrows fall in the red underground tank is still filled with water after
region of the archery target? filling the overhead tank completely?
(a) 0.40 (b) 0.20 (a) 1/3 (b) 1/2
(c) 0.16 (d) 0.04 (c) 1/4 (d) 1/6
68. A person allows 10% discount for cash payment 77. A piece of tin is in the form of a rectangle having
from the marked price of a toy and still he makes a length 12 cm and width 8 cm. This is used to
10% gain. What is the cost price of the toy which is construct a closed cube. The side of the cube is:
marked ` 770? (a) 2 cm (b) 3 cm
(a) ` 610 (b) ` 620 (c) 4 cm (d) 7 cm
(c) ` 630 (d) ` 640
69. A class starts at 11:00 am and lasts till 2:27 pm. Four TIME, SPEED AND DISTANCE
periods of equal duration are held during this interval. 78. A and B walk around a circular park. They start at 8
After every period, a rest of 5 minutes is given to the a.m. from the same point in the opposite directions.
students. The exact duration of each period is: A and B walk at a speed of 2 rounds per hour and 3
(a) 48 minutes (b) 50 minutes rounds per hour respectively. How many times shall
(c) 51 minutes (d) 53 minutes
they cross each other after 8·00 a.m. and before 9.30.
70. 30g of sugar was mixed in 180 ml water in a vessel A,
a.m.?
40 g of sugar Was mixed in 280 ml of water in vessel
B and 20 g of sugar was mixed in 100 ml of water in (a) 7 (b) 6
vessel C. The solution in vessel B is : (c) 5 (d) 8
(a) sweeter than that in C 79. Four friends A, B, C and D need to cross a bridge. A
(b) sweeter than that in A maximum of two persons can cross it at a time. It is
(c) as sweet as that in C night and they just have one lamp. Persons that
(d) less sweet than that in C cross the bridge must carry the lamp to find the way.
71. In aid of charity, every student in a class contributes A pair must walk together at the speed of slower
as many rupees as the number of students in that person. After crossing the bridge, the person having
class. With the additional contribution of ` 2 by one faster speed in the pair will return with the lamp
student only, the total collection is ` 443. Then how each time to accompany another person in the
many students are there in the class? group. Finally, the lamp has to be returned at the
(a) 12 (b) 21 original place and the person who returns the lamp
(c) 43 (d) 45 has to cross the bridge again without lamp. To cross
72. Anita’s mathematics test had 70 problems carrying the bridge, the time taken by them is as follows :
equal marks i.e., 10 arithmetic, 30 algebra and 30 A : 1 minute, B : 2 minutes, C : 7 minutes and D : 10
geometry. Although she answered 70% of the minutes. What is the total minimum time required
arithmetic, 40% of the algebra and 60% of the by all the friends to cross the bridge?
geometry problems correctly, she did not pass the (a) 23 minutes (b) 22 minutes
test because she got less than 60% marks. The (c) 21 minutes (d) 20 minutes
number of more questions she would have to 80. A daily train is to be introduced between station A
answer correctly to earn a 60% passing marks is: and station B starting from each at 6 AM and the
(a) 1 (b) 5 journey is to be completed in 42 hours. What is the
(c) 7 (d) 9 number of trains needed in order to maintain the
73. In a class, there are 18 very tall boys. If these Shuttle Service?
constitute three-fourths of the boys and the total (a) 2 (b) 3
number of boys is two-thirds of the total number of (c) 4 (d) 7
31

HINTS & SOLUTIONS


1. (c) Greater transparency in the functioning of the 11. (d) According to the author, the national output has
government and promoting the financ ial to pass through the commercial channels, before
inclusion is mandatory because there have been consumption, lacking which would lead to loss
instances where the elite are receiving land and of income, leading to underdevelopment and
natural resources by bribing the politicians. The economic disparity.
middle class continues to struggle and is 12. (d) Due to the Greenhouse effect and soil pollution,
deprived of opportunity or ec onomic a chain reaction has led to excessive carbon
development. deposition and contamination. Afforestation
2. (d) Climatic adaptation is relative to many vital would lead to the carbon breakdown and release
factors. So while formulating policies, the other the trapped carbon back to the atmosphere.
parameters need to be simultaneously evaluated 13. (b) The passage suggests the biggest hurdle in the
with respect to the climatic adaptation. world agriculture is to maintain a demand -
3. (a) Biodiversity is an important aspect of our supply equilibrium. Based on the relevant
ecosystem. Given that water is an essential information provided in the passage, it can be
ascertained that regulating the pricing
component of our existence, it is subjected
component for ensuring affordability to the poor
through many organisms before we get to sip a
and a proper marketing mix would be beneficial.
glass. Thus, water is naturally recycled with the
14. (d) The biggest challenge that has always haunted
help of the biodiversity.
world agriculture is to achieve a balance the
4. (d) The biggest problem in our country is the
demand and supply for foods.
disparity in the financial status of the citizens.
15. (c) To reduce hunger and starvation, ensuring
For instance, all the wealth is mostly equilibrium between demand and supply of food
accumulated by the elite class and the upper - is mandatory, alongside ushering measures for
middle class. However, the poor continues to be Purchasing Power Parity, to help the poor
deprived and exploited. The banking sector consumers too.
should look forward to promote financial 16. (b) Since there is disparity in the sustainable
inclusion, for equal distribution of financial provision and supply for food worldwide, this
services. issue has gained significance.
5. (b) Manual scavenging needs to be addressed. 17. (c) The key responsibility areas of the civil service
However, campaigns and awareness offic ers need to be clearly defined and
programmes are needed to be conducted, standardized, based on the political agendas.
especially targeting the female counterparts, to Lack of such c larity would lead to
address issues related to sanitation hazards; misgovernance. Also, striking equilibrium
manual scavenging being a part of it. between external and internal accountabilities
6. (d) The various needs and demands of man is mandatory for effective management.
compelled him to creating a society, as these 18. (d) The passage does not provide relevant
needs could not have been achieved individually. information on either of the provided options.
7. (b) The word 'corresponds' indicates a strong bond 19. (d) Civil services, being an extremely reputed
or the interrelationship between politics and taskforce, should implement reforms to create
economics, for effective formulation of state benc hmark in servic e and improve
policies. accountability of the office bearers.
8. (a) As per the information provided in the passage, 20. (d) Seeking ac countability through increased
since chemical and mechanical agro - practices participation by individuals in the decision
contributes to environmental pollution, organic making process would not help increase internal
accountability.
farming should be implemented as an
21. (a) Religious traditions, regardless of felicitating the
alternative, for environmental conservation.
Almighty or the Supernatural, ushers belief,
9. (a) ethics and a specific code of conduct in the
10. (b) Genetic alteration arises from the introduction society. Man being a social animal derives the
of a new practice, and subsequent follow up of value set from the society, important for
the same. In this case, a particular modification maintaining relationships and goodwill. Thus,
in the food habit of the cattle herders resulted in human relationships can be assumed to have
a massive change in the genetic constitution. been derived from the religious traditions.
32
22. (c) Fundamental rights, combined with duties, are
mandatory for dispensing justice in a society. Riberio Sumar Singh Jacket Sweater Tie Blue White Black
A ´ ü ´ ´ ü ´ ´ ü ´
23. (d) As per the passage, all the three options pose
challenges to the utilization of biomass as fuel. B ´ ´ ü ü ´ ´ ´ ´ ü
Issues related to climatic variations, aggravation C ü ´ ´ ´ ´ ü ü ´ ´
of carbon emissions due to liquid bio-fuels and 29. (a) C’s surname is Riberio.
c ompetition between biomass and food 30. (b) The colour of the tie is Blue.
production, have adverse effects. 31. (a) A wore the sweater.
24. (b) It is stated that unscrupulous cultivation of 32. (a) The direction diagram of a person is as follows:
energy c rops will lead to an unhealthy 19 km
c ompetition with food c rops, thereby
B
contributing to inflation and price hike for food D
4 km 15 km
C
crops.
12 km
25. (b) Technological intervention, with proper O
monitoring, could assist in using biomass for 15 km A
power generation and ac hieving negative
emissions.
26. (a) As per the information provided in the 2nd E
X
paragraph, some energy models (unproven
N
technology) might negate carbon emissions,
thereby mitigating environmental pollution due NW NE

to the greenhouse effect.


27. (d) Green Revolution has its disadvantages.
W E
Biodiversity is already at stake. Crops exposed
to exc essive c hemic als or the genetic ally
modified foods lack in nutritional content, in
SE
spite of boosting the yield. However, considering SW
S
the long term aspect, green revolution could risk
quality and health. The required distance is,
In DOAE
28. (b) According to the given information, the OA = DB = 4 km
direction diagram of a person is as shown OE = AX = 3 km
below: AE2 = OA2 + OE2
AE2 = 42 + 32
N
B AE2 = 25
12 km
NW NE AE = 5 km
33. (d) The number of small cubes with only one of the
A sides painted is 8.
W E
34. (d) As S Y S T E M and N E A R E R

5 km
SYSMET AENRER
SE
O
SW Similarly,
S
FRAC TION
\ Required distance,
BO2 = BA2 + OA2 CARFNOIT
BO2 = 122 + 52 35. (a)
36. (b) Let the number of boys be x, then numbers of girls
BO2 = 144 + 25
= 2x
BO2 = 169
\ x + 2x = 60; or 3x = 60; or x = 20
BO = 13 km
So numbers of boys = 20 and numbers of girls = 40
So, he is 13 km far away from the starting Numbers of student behind Kamal in rank
point O. = (60 – 17) = 43
Sol. (29-31) : Numbers of girls ahead of Kamal in rank = 9
On the basis of given information, the arrangement is as \ Numbers of boys behind Kamal in rank
following: = (43 – 31) = 12
33
37. (b) North We can say that the 5 groups meet on the same
day on every 60th day.
C 3km
B \ Number of times all the 5 groups meet on
X’s 180
4km 3km
starting the same day within 180 days = =3
point 60
1km A
West East 42. (c) No. of bee > Flower. So, checking by option there
1km 1km are 3 and 4.
Sol. (43-47) :
D 1km E
According to the given information, the
arrangement of five persons in a group is as
X’s returning point
South following :
· X starts from point A, passes through point
B and C and reaches point D. Doctor Lawyer Artist Gender
P
· X returns from point D, passes through point
E and reaches point A back. Q ü Male
38. (d) Option (d) is the just contradiction of given R ü Female
statement, if it is true, it would falsify the given S
statement.
T ü Male
39. (d) Question does not say anything about the gender
composition of the group invited for party.
couple Brother
Further, we donot know which specific female T ¬¾¾¾ ® R , Q ¬¾¾¾
® P
(+) (- ) (unmarried)
faculty know music. So either all in party were (+)
males, or some males and some females who did S is also unmarried.
not know music were present in the party. So 43. (a) T is the Doctor.
(d) is definitely correct. 44. (c) R is the Artist.
40. (b) The arrangement is in circular table 45. (b) T is the spouse of R.
· C is seated next to A 46. (b) Q is the Lawyer
Conclusion : AC – or CA 47. (c) Q is definitely a man.
48. (c) Quantity of product to be completed = 19000
A C Firm production per day = 1000 – 5% of 1000
= 950
C A Firm production in 19 days = 19000 – 5% of 1900
or
5
= 19000 – ´ 19000
100
· C is seated two seats from D
= 19000 – 950 = 18050
Conclusion : A – D or D – A So, firm needs one extra day other than 19 days
· B is not seated next to A to complete the order.
Conclusion : E must be next to A \ Firm will complete the order in 20 days.
Combining, A C D B E 49. (d) From statement 1,
A³B
E E
From statement 2,
C£D
A A and from statement 3
B D
OR B>C
By combining the above three statements, we get
D
C C A³B>C£D
B
Here, A > B > C < D
(There can be many arrangements)
or A = B > C = D
So, only statement 1 and 2 are correct. A>C
41. (d) Here, we need to take the L.C.M. of 2, 3, 4, 5 and So, A is older than C.
6 to find out the number of days after which all Sol. (50-52) :
these 5 groups meet on the same day. According to the given information, the arrangement
So, L.C.M. (2, 3, 4, 5, 6) = 60 of six boxes is as following:
34
57. (b) Let x1, x2, x3, x4 and x5 be the present ages of the
Boxes G am es Colours family comprising of 5 members
A Ten nis Oran ge 3 years ago,
B Volleyball Yellow/Blue (x1 + x2 + x3 + x4 + x5) – 3 × 5 = 80
Þ (x1 + x2 + x3 + x4 + x5) – 15 = 80 ...(i)
C Cricket Green
Now, let the age of new born baby be x6.
D Football Yellow/Blue
Given,
E Golfball Violet Average age of family today = Average age of
F Hockey Indigo family 3 years ago
(x1 + x 2 + x 3 + x 4 + x5 ) - 15
Cricket Tennis Þ
5
B C B A (Orange) x1 + x 2 + x 3 + x 4 + x 5 + x 6
(Green) =
6
50. (b) E boxes contains the golfball.
80 15 + 80 + x 6
51. (b) F is painted indigo is the correct statement. Þ = [from (i)]
5 6
52. (c) We cannot determined that the football is in the
Þ x6 = 1
box of which colour.
\ Baby’s age = 1 year
53. (d) Since, there are 5 questions to be attempted and
58. (d) Let the basic pay of the first person be ` x and
each questions has two choices– True or False.
that of second person be ` y.
Therefore, the maximum number of candidates
According to the question,
so that none of them gives the answers to the
five questions in an identical sequence is an Emolument of both the persons are same.
arrangement of 2 different options at 5 places. So, x + 0.65x = y + 0.8y
x 1.8
2 2 2 2 2 Þ x(1 + 0.65) = y(1 + 0.8) Þ =
y 1.65
Above figure shows that each question can be
filled with 2 choices. x 12
Þ = or 12 : 11
So, required numbered of candidates = 2 × 2 × 2 × 2 y 11
× 2 = 32 59. (b) A person is standing on the first step from the
54. (d) According to given information: bottom of a ladder.
Now, he has to climb 4 more steps to reach
Fruits Eaten exactly the middle step,
Persons Grapes Pineapple Oranges Apple 9
A ü ü 8
7
B ü ü
6
C ü ü ü Middle
5 4
D ü ü ü more steps
4
3
B and C both ate Oranges, So, it can be said that
2
the cause of sickness was eating of oranges.
Step 1
55. (d) The death rate is declining more faster than in
birth rate. This is the prominent reason for So, it is clear that the ladder have 9 steps.
increament in the rate of population growth. 60. (b) B C

56. (a) Given, length = x1m and breadth = x2m


Also, x1 + x2 = 40 where x1 and x2 are variables
We know that, of all the rectangles, a square has
12m
the largest area.
For the given rectangle to be a square x1 = x2
So, x1 + x2 = 40 Þ x1 = x2 = 20 m
\ Maximum area of the given rectangle = x1x2 A
A D
5m
= (20 × 20)m2 = 400 m2 Fig. Vertical Trunk Fig. Broken Trunk
35
According to pythagoras theorem,
1
CD2 = AD2 + AC2 X can do of the work in 10 days
4
Þ CD2 = 52 + 122 Þ CD2 = 25 + 144 \ X can do the complete work in 40 days.
Þ CD = 169 = 13 m Y can do 40% of the work in 40 days.
Þ Y can do 100% of the work in 100 days
Now, AB = AC + CD ( Q CD is the broken part of
the trunk AB touching ground at D) \ Y can do the complete work in 100 days.
Þ AB = (12 + 13) m = 25 m 1
Z can do of the work in 13 days.
61. (c) Let the work done by Ram and Shyam be R and 3
S respectively.
\ Z can do the complete work in 39 days.
60 Hence, it is clear from above results that Z will
Given, (R+S)¢s 4 days work = of work complete the work first.
100
65. (a) Average monthly income of a person in family
60 1 15 of 5 = ` 10,000
Þ (R + S)¢s 1 day work = ´ of work = of
100 4 100 \ Monthly income of family = ` 10,000 × 5 = `
work 50,000
After Ram takes leave : After Increament
æ 60 ö Increase in monthly income of one person
40
S’s 8 days work = çè1 - ÷ of work = of work
100 ø 100 1, 20, 000
= = ` 10, 000
12
40 1 5
Þ S¢s 1 day work ´ of work = of work Now, the average monthly income of a person
100 8 100
50, 000 + 10, 000
So, Ram’s 1 day work = (R + S)¢s 1 day work – S¢s in family = = ` 12,000
5
1 day work

æ 15 5 ö 66. (c)
= çè - ÷ of work
100 100 ø

1
= of work] 5m 3m 3m 3m 3m 3m
10
\ Ram will take 10 days to complete the entire job Distances covered by the competitor to collect
alone. the apples in the bucket are as follows:
1st apple : 2(5) = 10 m
62. (b) By looking at all the options, we observe that
option (b) is not necessarily true. 2nd apple : 2(5 + 3) = 16 m
We know that, a number is divisible by 10 iff it 3rd apple : 2(5 + 2 × 3) = 22 m
has 0 at the unit’s place. 4th apple : 2(5 + 3 × 3) = 28 m
(R + S) may or may not have 0 at the unit’s place. 5th apple : 2(5 + 4 × 3) = 34 m
Therefore, it may or may not be divisible by 10. 6th apple : 2(5 + 5 × 3) = 40 m
Thus, it is not necessarily true. Therefore, total distance covered = 10 + 16 + 22 +
63. (a) From 100 to 199, there are 10 numbers ending 28 + 34 + 40 = 150 m
with 2. They are 102, 112, 122, 132, 142, 152, 162,
172, 182, 192. 67. (c)
And from 200 to 300, there are 100 numbers
beginning with 2. They are 200, 201, 202...,299.
\ There are 110 numbers between 100 and 300 R B Y W
which either begin with or end with 2.
64. (d) W can do 25% of a work in 30 days.
1
Þ W can do of a work in 30 days
4
We know that, area of a circle = pr2
\ W can do the complete work in 120 days.
So, Red band area = p(0.2)2 = 0.04p
36
All the other than red are in the form of a ring. 70. (d) Vessel A : 30g sugar mixed in 180ml water.
So, So, the concentration of sugar in vessel A is
Blue band area = p((0.3)2 – (0.2)2)
30 1
= p((0.3 – 0.2) (0.3 + 0.2)) = 0.05 p g / ml i.e., g / ml .
180 6
Similarly, yellow band area = 0.07 p
Vessel B : 40g sugar mixed in 280ml water.
And white band area = 0.09 p So, the concentration of sugar in vessel B is
\ Required probability
40 1
g / ml i.e., g / ml .
0.04p 280 7
= = 0.16
( 0.04 + 0.05 + 0.07 + 0.09 ) p Vessel C : 20g sugar mixed in 100ml water.
Alternate Method: So, the concentration of sugar in vessel C is
Area of red circle = p(20)2 20 1
g / ml i.e. g / ml .
100 5
1
Radius of Archry board = m = .50 m More the concentration of sugar, more will be
2
the sweetness.
Area of Archry Board = p(.50)2
Therefore, in terms of sweetness : C > A > B
p(.20) 2 0.400 71. (b) Let the number of students in the class be x.
P(Î) = = = 0.16
p.(50) 2 0.2500 Total c ollec tion without considering the
additional contribution by one student = 443 – 2
68. (c) Marked price (M.P.) of toy = ` 770 = ` 441
10 So, x × x = 441
Discount = 10% of MP = ´ 770 = ` 77
100 Þ x2 = 441 Þ x = 441 = 21
Price after discount = ` (770 – 77) = ` 693 72. (b) Questions correctly answered by Anita are as
Let cost price (C.P.) of toy be ` x. follows:
According to Question, Arithmetic – 70% of 10 = 7
Algebra – 40% of 30 = 12
10
693 – x = of x Geometry – 60% of 30 = 18
100
\ Total questions correctly answered = 7 + 12
Þ 693 – x = 0.1 x + 18 = 37
Þ 1.1 x = 693 Þ x = 630 Anita had to attempt 60% of 70 questions i.e., 42
\ Cost price of the toy = ` 630 questions correctly to pass the test.
MP = 770 \ Number of more questions required to be
answered = 42 – 37 = 5
90
SP after discount = ´ 770 3
100 73. (b) Here, ´ x = 18
4
100 90 x = 24
CP getting 10% gain = ´ ´ 770 = 630
110 100
2
69. (a) Class start at 11:00 am Boys = 24 and ´ y = 24
3
Class ends at 02:27 pm
y = 36
So, duration of class = 3h 27min = 207 min Total students = 36
There will be 4 periods. The number of girls in the class = 36 – 24 =12.
After every period, there will be a 5 min break. 74. (b) Let the number of employees in the company be
Here, only 3 breaks will be taken as there are 100 and the number of male employees in the
only 4 periods and end of 4th period means end company be x.
of the class. 4 th break is not needed to be
So, the number of female employees = 100 – x.
considered.
According to the question,
So, duration of the periods = 207 – 3×5 = 192 min.
5200x + 4200 (100 - x)
192 = 5000
\ Time assigned to each period = = 48min . 100
4
37

Þ 52x + 42 (100 - x) = 5000 78. (a) Here, A and B are moving in opposite directions.
So, Relative speed = 2 + 3 = 5 rounds/hour So,
Þ 52x + 4200 - 42x = 5000 they cross each other 5 times in an hour and 2
times in half an hour.
Þ 10x = 800 Þ x = 80
Hence, they cross each other 7 times before 9 :
So, there are 80% male employees in the 30 a.m.
company. 79. (a) Here, A takes the shortest time to cross the bridge
75. (b) Let the third number Z = 100 i.e. 1 min. And, D takes 10 min, C takes 7 min
So, X = 80, Y = 72 and B takes 2 min to cross the bridge.
\ Percentage by whic h Y is less than X So, 4 friends can cross the bridge in minimum
time in the following ways.
80 - 72
= ´ 100 = 10% 1. A + B crossing and A returning : (2 + 1) min
80
2. A + C crossing and A returning : (7 + 1) min
76. (a) Volume of cylindrical overhead tank = pr2h =
3. A + D crossing and A returning : (10 + 1) min
22 4. A crossing the bridge for the last time : 1 min.
´ (2) 2 ´ 7 = 88 m3
7
Hence, total minimum time = 3 + 8 + 11 + 1 = 23
Volume of underground tank = (5.5 × 4 × 6)m 3 = min.
132m3
Portion of underground tank still filled with 80. (c)
A B
water after filling the overhead tank completely. st
6 am Fig. 1 day 6 am
Volume of underground tank – Volume of overhead tank
=
Volume of underground tank
Train 2 Train 1
A B
132 - 88 44 1
= = =
nd
6 am Fig. 2 day 6 am
132 32 3
77. (c) Area of rectangle = 12 × 8 cm2 = 96 cm2 Above given figures show that train 1 leaves
Rectangle is used to construct a closed cube. station A on 1st day and train 2 leaves station B
on 1st day. Both the trains don’t reach their
Surface area of cube formed = Area of given
destination even on the 2nd day after 24 hours as
rectangle.
the journey completes in 42 hours.
6 (side)2 = 96 cm 2
\ Two more trains need to be introduced at both
Þ side = 16 Þ side = 4 cm. the stations i.e., station A and B. So, 4 trains are
needed in order to maintain the shuttle service.
1

2016 SOLVED PAPER 1

This Paper is taken from our :

ISBN : 9789386146823
2

CSAT PAPER-1 2016


GEOGRAPHY 6. With reference to ‘Initiative for Nutritional Security
through Intensive Millets Promotion’, which of the
1. Which of the following is/are tributary tributaries of
following statements is/are correct?
Brahmaputra?
1. This initiative aims to demonstrate the
1. Dibang
2. Kameng improved production and post-harvest
3. Lohit technologies, and to demonstrate value
Select the correct answer using the code given addition techniques, in an integrated manner,
below. with cluster approach.
(a) 1 only (b) 2 and 3 only 2. Poor, small, marginal and tribal farmers have
(c) 1 and 3 only (d) 1, 2 and 3 larger stake in this scheme.
2. In which of the following regions of India are shale 3. An important objective of the scheme is to
gas resources found? encourage farmers of commercial crops to shift
1. Cambay Basin to millet cultivation by offering them free kits of
2. Cauvery Basin critical inputs of nutrients and micro irrigation
3. Krishna-Godavari Basin equipment.
Select the correct answer using the code given Select the correct answer using the code given
below. below.
(a) 1 and 2 only (b) 3 only (a) 1 only (b) 2 and 3 only
(c) 2 and 3 only (d) 1, 2 and 3 (c) 1 and 2 only (d) 1, 2 and 3
3. Recently, linking of which of the following rivers 7. Which of the following is/are the advantage /
was undertaken? advantages of practising drip irrigation?
(a) Cauvery and Tungabhadra
1. Reduction in weed
(b) Godavari and Krishna
2. Reduction in soil salinity
(c) Mahanadi and Sone
3. Reduction in soil erosion
(d) Narmada and Tapti
4. Recently, which of the following States has explored Select the correct answer using the code given
the possibility of constructing an artificial inland below.
port to be connected to sea by a long navigational (a) 1 and 2 only
channel? (b) 3 only
(a) Andhra Pradesh (b) Chhattisgarh (c) 1 and 3 only
(c) Karnataka (d) Rajasthan (d) None of the above is an advantage of practising
drip irrigation
AGRICULTURE 8. Why does the Government of India promote the use
5. The FAO accords the status of ‘Globally Important of Neem-coated Urea’ in agriculture?
Agricultural Heritage System (GIAHS)’ to (a) Release of Neem oil in the soil increases
traditional agricultural systems. What is the overall nitrogen fixation by the soil microorganisms
goal of this initiative? (b) Neem coating slows down the rate of
1. To provide modern technology, training in dissolution of urea in the soil
modern farming methods and financial (c) Nitrous oxide, which is a greenhouse gas, is not
support to local communities of identified at all released into atmosphere by crop fields
GIAHS so as to greatly enhance their (d) It is a combination of a weedicide and a
agricultural productivity
fertilizer for particular crops
2. To identify and safeguard eco-friendly
9. With reference to ‘Pradhan Mantri Fasal Bima
traditional farm practices and their associated
Yojana’, consider the following statements:
landscapers, agricultural biodiversity and
knowledge systems of the local communities 1. Under this scheme, farmers will have to pay a
3. To provide Geographical Indication status to all uniform premium of two percent for any crop
the varieties of agricultural produce in such they cultivate in any season of the year.
identified GIAHS 2. This scheme covers post-harvest losses arising
Select the correct answer using the code given out of cyclones and unseasonal rains.
below. Which of the statements given above is/are correct?
(a) 1 and 3 only (b) 2 only (a) 1 only (b) 2 only
(c) 2 and 3 only (d) 1, 2 and 3 (c) Both 1 and 2 (d) Neither 1 nor 2
3
ENVIRONMENT: INTERNATIONAL TREATIES 2. It originated in the World Summit on
Sustainable Development held in Johannesburg
10. Consider the following pairs : in 2002.
T erm s s o m etim es s een in th e n ew s T h eir origin Which of the statements given above is/are correct?
1 . A n n ex — I C o u n tries C a rta gen a P ro toco l (a) 1 only (b) 2 only
2 . C ertified E m is s io n s R ed u ction s N a g oya P rotoc ol (c) Both 1 and 2 (d) Neither 1 nor 2
3 . C lea n D ev elop m en t M ech a n is m Kyo to P rotoc ol 15. Consider the following statements:
Which of the pairs given above is/are correctly (1) The International Solar Alliance was launched
at the United Nations Climate Change
matched?
Conference in 2015.
(a) 1 and 2 only (b) 2 and 3 only
(2) The Alliance includes all the member countries
(c) 3 only (d) 1, 2 and 3
of the United Nations.
11. With reference to an initiative called ‘The Which of the statements given above is/are correct?
Economics of Ecosystems and Biodiversity (TEEB)’, (a) 1 only (b) 2 only
which of the following statements is/are correct? (c) Both 1 and 2 (d) Neither 1 nor 2
1. It is an initiative hosted by UNEP, IMF and 16. With reference to the Agreement at the UNFCCC
World Economic Forum. Meeting in Paris in 2015, which of the following
2. It is a global initiative that focuses on drawing statements is/are correct?
attention to the economic benefits of 1. The Agreement was signed by all the member
biodiversity. countries of the UN and it will go into effect in
3. It presents an approach that can help decision- 2017.
makers recognize, demonstrate and capture the 2. The Agreement aims to limit the greenhouse
value of ecosystems and biodiversity. gas emissions so that the rise in average global
Select the correct answer using the code given temperature by the end of this century does not
below. exceed 2 °C or even 1.5 °C above pre-industrial
(a) 1 and 2 only (b) 3 only levels.
(c) 2 and 3 only (d) 1, 2 and 3 3. Developed countries acknowledged their
12. Which of the following statements is/are correct? historical responsibility in global warming and
committed to donate $ 1000 billion a year from
Proper design and effective implementation of UN-
2020 to help developing countries to cope with
REDD+ Programme can significantly contribute to
climate change.
1. protection of biodiversity
Select the correct answer using the code given
2. resilience of forest ecosystems below.
3. poverty reduction (a) 1 and 3 only (b) 2 only
Select the correct answer using the code given (c) 2 and 3 only (d) 1, 2 and 3
below. 17. Consider the following statements:
(a) 1 and 2 only (b) 3 only 1. The Sustainable Development Goals were first
(c) 2 and 3 only (d) 1, 2 and 3 proposed in 1972 by a global think tank called
13. What is ‘Greenhouse Gas Protocol’? the ‘Club of Rome’.
(a) It is an international accounting tool for 2. The Sustainable Development Goals have to be
government and business leaders to achieved by 2030.
understand, quantify and manage greenhouse Which of the statements given above is/are correct?
gas emissions (a) 1 only (b) 2 only
(c) Both 1 and 2 (d) Neither 1 nor 2
(b) It is an initiative of the United Nations to offer
18. The term ‘Intended Nationally Determined
financial incentives to developing countries to
Contributions’ is sometimes seen in the news in the
reduce greenhouse gas emissions and to adopt
context of
eco-friendly technologies
(a) pledges made by the European countries to
(c) It is an inter-governmental agreement ratified rehabilitate refugees from the war-affected
by all the member countries of the United Middle East
Nations to reduce greenhouse gas emissions to (b) plan of action outlined by the countries of the
specified levels by the year 2022 world to combat climate change
(d) It is one of the multilateral REDD+ initiatives (c) capital contributed by the member countries in
hosted by the World Bank the establishment of Asian Infrastructure
14. With reference to ‘Agenda 21’, sometimes seen in Investment Bank
the news, consider the following statements : (d) plan of action outlined by the countries of the
1. It is a global action plan for sustainable world regarding Sustainable Development
development Goals
4
19. What is/are the importance/importance of the Select the correct answer using the code given
‘United Nations Convention to Combat below.
Desertification’? (a) 1 and 2 only (b) 3 only
1. It aims to promote effective action through (c) 2 and 3 only (d) 1, 2 and 3
innovative national programs and supportive 24. In the cities of our country, which among the
inter-national partnerships. following atmospheric gases are normally
2. It has a special/particular focus on South Asia considered in calculating the value of Air Quality
and North Africa regions, and its Secretariat Index?
facilitates the allocation of major portion of 1. Carbon dioxide 2. Carbon monoxide
financial resources to these regions. 3. Nitrogen dioxide 4. Sulfur dioxide
3. It is committed to bottom-up approach, 5. Methane
encouraging the participation of local people in Select the correct answer using the code given
combating the desertification. below.
Select the correct answer using the code given
(a) 1, 2 and 3 only (b) 2, 3 and 4 only
below.
(c) 1, 4 and 5 only (d) 1, 2, 3, 4 and 5
(a) 1 only (b) 2 and 3 only
25. Which of the following are the key features of
(c) 1 and 3 only (d) 1, 2 and 3
‘National Ganga River Basin Authority (NGRBA)’?
ENVIRONMENT: NATIONAL ORG, 1. River basin is the unit of planning and
SCHEMES, INITIATIVES management.
2. It spearheads the river conservation efforts at
20. Which of the following best describes/ describe the the national level.
aim of ‘Green India Mission’ of the Government of 3. One of the Chief Ministers of the States through
India? which the Ganga flows becomes the Chairman
1. Incorporating environment al benefits and of NGRBA on rotation basis.
costs into the Union and State Budgets thereby Select the correct answer using the code given
implementing the ‘green accounting’ Below.
2. Launching the second green revolution to (a) 1 and 2 only (b) 2 and 3 only
enhance agricultural output so as to ensure (c) 1 and 3 only (d) 1, 2 and 3
food security to one and all in the future
3. Restoring and enhancing forest cover and ENVIRONMENT: FLORA-FAUNA
responding to climate change by a combination 26. What is/are unique about ‘Kharai camel’, a breed
of adaptation and mitigation measures found in India?
Select the correct answer using the code given 1. It is capable of swimming up to three
below. kilometers in seawater.
(a) 1 only (b) 2 and 3 only 2. It survives by grazing on mangroves.
(c) 3 only (d) 1, 2 and 3 3. It lives in the wild and cannot be domesticated.
21. ‘Net metering’ is sometimes seen in the news in the Select the correct answer using the code given
context of promoting the below.
(a) production and use of solar energy by the (a) 1 and 2 only (b) 3 only
households/consumers (c) 1 and 3 only (d) 1, 2 and 3
(b) use of piped natural gas in the kitchens of 27. Recently, our scientists have discovered a new and
households distinct species of banana plant which attains a
(c) installation of CNG kits in motor-cars height of about 11 metres and has orange-coloured
(d) installation of water meters in urban fruit pulp. In which part of India has it been
households discovered?
22. ‘Gadgil Committee Report’ and ‘Kasturirangan (a) Andaman Islands
Committee Report’, sometimes seen in the news, are (b) Anaimalai Forests
related to (c) Maikala Hills
(a) constitutional reforms (d) Tropical rain forests of northeast
(b) Ganga Action Plan 28. With reference to ‘Red Sanders’, sometimes seen in
(c) linking of rivers the news, consider the following statements:
(d) protection of Western Ghats 1. It is a tree species found in a part of South India.
23. On which of the following can you find the Bureau 2. It is one of the most important trees in the
of Energy Efficiency Star Label? tropical rain forest areas of South India.
1. Ceiling fans Which of the statements given above is/are correct?
2. Electric geysers (a) 1 only (b) 2 only
3. Tubular fluorescent lamps (c) Both 1 and 2 (d) Neither 1 nor 2
5
29. Recently, for the first time in our country, which of 34. With reference to ‘Financial Stability and
the following States has declared a particular Development Council’, consider the following
butterfly as ‘State Butterfly’? statements :
(a) Arunachal Pradesh (b) Himachal Pradesh 1. It is an organ of NITI Aayog.
(c) Karnataka (d) Maharashtra 2. It is headed by the Union Finance Minister.
30. In which of the following regions of India are you 3. It monitors macroprudential supervision of the
most likely to come across the ‘Great Indian economy.
Hornbill’ in its natural habitat? Which of the statements given above is/are correct?
(a) Sand deserts of northwest India (a) 1 and 2 only (b) 3 only
(b) Higher Himalayas of Jammu and Kashmir (c) 2 and 3 only (d) 1, 2 and 3
(c) Salt marshes of western Gujarat 35. With reference to ‘Bitcoins’, sometimes seen in the
(d) Western Ghats news, which of the following statements is/are
correct?
BANKING-FINANCE
1. Bitcoins are tracked by the Central Banks of the
31. What is/are the purpose/purposes of the ‘Marginal countries.
Cost of Funds based Lending Rate (MCLR)’ 2. Anyone with a Bitcoin address can send and
announced by RBI? receive Bitcoins from anyone else with a Bitcoin
1. These guidelines help improve the address.
transparency in the methodology followed by 3. Online payments can be sent without either
banks for determining the interest rates on side knowing the identity of the other.
advances. Select the correct answer using the code given
2. These guidelines help ensure availability of below.
bank credit at interest rates which are fair to the (a) 1 and 2 only (b) 2 and 3 only
borrowers as well as the banks. (c) 3 only (d) 1, 2 and 3
Select the correct answer using the code given below. 36. Regarding ‘Atal Pension Yojana’, which of the
(a) 1 only (b) 2 only following statements is/are correct?
(c) Both 1 and 2 (d) Neither 1 nor 2 1. It is a minimum guaranteed pension scheme
32. The establishment of ‘Payment Banks’ is being mainly targeted at unorganized sector workers.
allowed in India to promote financial inclusion. 2. Only one member of a family can join the
Which of the following statements is/are correct in scheme.
this context? 3. Same amount of pension is guaranteed for the
1. Mobile telephone companies and supermarket spouse for life after subscriber’s death.
chains that are owned and controlled by
Select the correct answer using the code given
residents are eligible to be promoters of
below.
Payment Banks.
(a) 1 only (b) 2 and 3 only
2. Payment Banks can issue both credit cards and
(c) 1 and 3 only (d) 1, 2 and 3
debit cards.
37. The term ‘Base Erosion and Profit Shifting’ is
3. Payment Banks cannot undertake lending
sometimes seen in the news in the context of
activities.
Select the correct answer using the code given (a) mining operation by multinational companies
below. in resource-rich but backward areas
(a) 1 and 2 only (b) 1 and 3 only (b) curbing of the tax evasion by multinational
(c) 2 only (d) 1, 2 and 3 companies
33. The term ‘Core Banking Solutions’ is sometimes (c) exploitation of genetic resources of a country by
seen in the news. Which of the following statements multinational companies
best describes/describe this term? (d) lack of consideration of environmental costs in
1. It is a networking of a bank’s branches which the planning and implementation of
enables customers to operate their accounts developmental projects
from any branch of the bank on its network 38. There has been a persistent deficit budget year after
regardless of where they open their accounts. year. Which action/actions of the following can be
2. It is an effort to increase RBI’s control over taken by the Government to reduce the deficit?
commercial banks through computerization. 1. Reducing revenue expenditure
3. It is a detailed procedure by which a bank with 2. Introducing new welfare schemes
huge non-performing assets is taken over by 3. Rationalizing subsidies
another bank. 4. Reducing import duty
Select the correct answer using the code given Select the correct answer using the code given
below. below.
(a) 1 only (b) 2 and 3 only (a) 1 only (b) 2 and 3 only
(c) 1 and 3 only (d) 1, 2 and 3 (c) 1 and 3 only (d) 1, 2, 3 and 4
6
39. Which of the following is/are included in the capital 46. Consider the following statements :
budget of the Government of India? 1. New Development Bank has been set up by
1. Expenditure on acquisition of assets like roads, APEC.
buildings, machinery, etc. 2. The headquarters of New Development Bank is
2. Loans received from foreign governments in Shanghai.
3. Loans and advances granted to the States and Which of the statements given above is/are correct?
Union Territories (a) 1 only (b) 2 only
Select the correct answer using the code given (c) Both 1 and 2 (d) Neither 1 nor 2
below.
47. What is/are the purpose/purposes of Government’s
(a) 1 only (b) 2 and 3 only
‘Sovereign Gold Bond Scheme’ and ‘Gold
(c) 1 and 3 only (d) 1, 2 and 3
Monetization Scheme’?
40. With reference to the ‘Trans-Pacific Partnership’,
consider the following statements: 1. To bring the idle gold lying with Indian
1. It is an agreement among all the Pacific Rim households into the economy
countries except China and Russia. 2. To promote FDI in the gold and jewellery sector
2. It is a strategic alliance for the purpose of 3. To reduce India’s dependence on gold imports
maritime security only. Select the correct answer using the code given
Which of the statements given above is/are correct? below.
(a) 1 only (b) 2 only (a) 1 only (b) 2 and 3 only
(c) Both 1 and 2 (d) Neither 1 nor 2 (c) 1 and 3 only (d) 1, 2 and 3
41. ‘European Stability Mechanism’, sometimes seen in 48. ‘Global Financial Stability Report’ is prepared by the
the news, is an (a) European Central Bank
(a) agency created by EU to deal with the impact of (b) International Monetary Fund
millions of refugees arriving from Middle East (c) International Bank for Reconstruction and
(b) agency of EU that provides financial assistance Development
to eurozone countries (d) Organization for Economic Cooperation and
(c) agency of EU to deal with all the bilateral and Development
multilateral agreements on trade 49. With reference to the International Monetary and
(d) agency of EU to deal with the conflicts arising Financial Committee (IMFC), consider the following
among the member countries
statements :
42. The term ‘Regional Comprehensive Economic
1. IMFC discusses matters of concern affecting the
Partnership’ often appears in the news in the
context of the affairs of a group of countries known global economy, and advises the International
as Monetary Fund (IMF) on the direction of its
(a) G20 (b) ASEAN work.
(c) SCO (d) SAARC 2. The World Bank participates as observer in
43. In the context of which of the following do you IMFC’s meetings.
sometimes find the terms ‘amber box, blue box and Which of the statements given above is/are correct?
green box’ in the news? (a) 1 only (b) 2 only
(a) WTO affairs (c) Both 1 and 2 (d) Neither 1 nor 2
(b) SAARC affairs 50. Which of the following best describes the term
(c) UNFCCC affairs ‘import cover’, sometimes seen in the news?
(d) India-EU negotiations on FTA (a) It is the ratio of value of imports to the Gross
44. Recently, which one of the following currencies has Domestic Product of a country
been proposed to be added to the basket of IMF’s (b) It is the total value of imports of a country in a
SDR? year
(a) Ruble (b) Rand (c) It is the ratio between the value of exports and
(c) Indian Rupee (d) Renminbi that of imports between two countries
45. With reference to ‘IFC Masala Bonds’, sometimes (d) It is the number of months of imports that
seen in the news, which of the statements given could be paid for by a country’s international
below is/are correct? reserves
1. The International Finance Corporation, which 51. With reference to ‘Stand up India scheme’, which of
offers these bonds, is an arm of the World Bank. the following statement is/are correct?
2. They are the rupee-denominated bonds and are 1. Its purpose is to promote entrepreneurship
a source of debt financing for the public and among SC/ST and women entrepreneurs.
private sector. 2. It provides for refinance through SIDBI.
Select the correct answer using the code given Select the correct answer using the code given
below. below.
(a) 1 only (b) 2 only (a) 1 only (b) 2 only
(c) Both 1 and 2 (d) Neither 1 nor 2 (c) Both 1 and 2 (d) Neither 1 nor 2
7
52. Pradhan Mantri MUDRA Yojana is aimed at 58. Regarding DigiLocker, sometimes seen in the news,
(a) bringing the small entrepreneurs into formal which of the following statements is/are correct?
financial system 1. It is a digital locker system offered by the
(b) providing loans to poor farmers for cultivating Government under Digital India Programme.
particular crops 2. It allows you to access your e-documents
(c) providing pensions to old and destitute irrespective of your physical location.
persons Select the correct answer using the code given
(d) funding the voluntary organizations involved below.
in the promotion of skill development and (a) 1 only (b) 2 only
employment generation (c) Both 1 and 2 (d) Neither 1 nor 2
53. India’s ranking in the ‘Ease of Doing Business Index’ 59. Which of the following is/are the indicator/
is sometimes seen in the news. Which of the indicators used by IFPRI to compute the Global
following has declared that ranking? Hunger Index Report?
(a) Organization for Economic Cooperation and 1. Undernourishment
Development (OECD) 2. Child stunting
(b) World Economic Forum 3. Child mortality
(c) World Bank Select the correct answer using the code given
(d) World Trade Organization (WTO) below.
54. Recently, India’s first ‘National Investment and (a) 1 only (b) 2 and 3 only
Manufacturing Zone’ was proposed to be set up in (c) 1, 2 and 3 (d) 1 and 3 only
(a) Andhra Pradesh (b) Gujarat
SCIENCE-TECH: BIOLOGY,
(c) Maharashtra (d) Uttar Pradesh
MEDICINE, PUBLIC HEALTH
55. Which one of the following is a purpose of ‘UDAY’,
a scheme of the Government? 60. In the context of the developments in
(a) Providing technical and financial assistance to Bioinformatics, the term ‘transcriptome’,
start-up entrepreneurs in the field of renewable sometimes seen in the news, refers to
sources of energy (a) a range of enzymes used in genome editing
(b) Providing electricity to every household in the (b) the full range of mRNA molecules expressed by
country by 2018 an organism
(c) Replacing the coal-based power plants with (c) the description of the mechanism of gene
natural gas, nuclear, solar, wind and tidal expression
power plants over a period of time (d) a mechanism of genetic mutations taking place
(d) Providing for financial turnaround and revival in cells
of power distribution companies 61. ‘Mission Indradhanush’ launched by the
56. What is/are the purpose/purposes of ‘District Government of India pertains to
Mineral Foundations’ in India? (a) immunization of children and pregnant women
1. Promoting mineral exploration activities in (b) construction of smart cities across the country
mineral-rich districts (c) India’s own search for the Earth-like planets in
2. Protecting the interests of the persons affected outer space
by mining operations (d) New Educational Policy
3. Authorizing State Governments to issue 62. With reference to pre-packaged items in India, it is
licences for mineral exploration mandatory to the manufacturer to put which of the
Select the correct answer using the code given following information on the main label, as per the
below. Food Safety and Standards (Packaging and
(a) 1 and 2 only (b) 2 only Labelling) Regulations, 2011?
(c) 1 and 3 only (d) 1, 2 and 3 1. List of ingredients including additives
57. ‘SWAYAM’, an initiative of the Government of 2. Nutrition information
India, aims at 3. Recommendations, if any, made by the medical
(a) promoting the Self Help Groups in rural areas profession about the possibility of any allergic
(b) providing financial and technical assistance to reactions
young start-up entrepreneurs 4. Vegetarian/non-vegetarian
(c) promoting the education and health of Select the correct answer using the code given
adolescent girls below.
(d) providing affordable and quality education to (a) 1, 2 and 3 (b) 2, 3 and 4
the citizens for free (c) 1, 2 and 4 (d) 1 and 4 only
8
63. Which of the following statements is/are correct? 69. Consider the following statements:
Viruses can infect The Mangalyaan launched by ISRO
1. bacteria 1. is also called the Mars Orbiter Mission
2. fungi 2. made India the second country to have a
3. plants spacecraft orbit the Mars after USA
Select the correct answer using the code given 3. made India the only country to be successful in
below. making its spacecraft orbit the Mars in its very
(a) 1 and 2 only (b) 3 only first attempt
(c) 1 and 3 only (d) 1, 2 and 3 Which of the statements given above is/are correct?
(a) 1 only (b) 2 and 3 only
SCIENCE-TECH : ICT RELATED (c) 1 and 3 only (d) 1, 2 and 3
64. ‘Project Loon’, sometimes seen in the news, is
related to FREEDOM STRUGGLE
(a) waste management technology 70. The ‘Swadeshi’ and ‘Boycott’ were adopted as
(b) wireless communication technology methods of struggle for the first time during the
(c) solar power production technology (a) agitation against the Partition of Bengal
(d) water conservation technology
(b) Home Rule Movement
65. With reference to ‘Li-Fi’, recently in the news, which
(c) Non-Cooperation Movement
of the following statements is/are correct?
1. It uses light as the medium for high-speed data (d) visit of the Simon Commission to India
transmission. 71. Satya ShodhakSamaj organized
2. It is a wireless technology and is several times (a) a movement for upliftment of tribals in Bihar
faster than ‘WiFi’. (b) a temple-entry movement in Gujarat
Select the correct answer using the code given (c) an anti-caste movement in Maharashtra
below. (d) a peasant movement in Punjab
(a) 1 only (b) 2 only 72. The Montague-Chelmsford Proposals were related
(c) Both 1 and 2 (d) Neither 1 nor 2 to
SCIENCE-TECH: SPACE MISSIONS (a) social reforms
(b) educational reforms
66. What is ‘Greased Lightning-10 (GL-10)’, recently in (c) reforms in police administration
the news? (d) constitutional reforms
(a) Electric plane tested by NASA
73. Consider the following :
(b) Solar-powered two-seater aircraft designed by
1. Calcutta Unitarian Committee
Japan
2. Tabernacle of New Dispensation
(c) Space observatory launched by China
(d) Reusable rocket designed by ISRO 3. Indian Reform Association
67. With reference to ‘Astrosat’, the astronomical Keshab Chandra Sen is associated with the
observatory launched by India, which of the establishment of which of the above?
following statements is/are correct? (a) 1 and 3 only (b) 2 and 3 only
1. Other than USA and Russia, India is the only (c) 3 only (d) 1, 2 and 3
country to have launched a similar observatory 74. What was the main reason for the split in the Indian
into space. National Congress at Surat in 1907?
2. Astrosat is a 2000 kg satellite placed in an orbit (a) Introduction of communalism into Indian
at 1650 km above the surface of the Earth. politics by Lord Minto
Select the correct answer using the code given (b) Extremists’ lack of faith in the capacity of the
below. moderates to negotiate with the British
(a) 1 only (b) 2 only Government
(c) Both 1 and 2 (d) Neither 1 nor 2
(c) Foundation of Muslim League
68. India is an important member of the ‘International
(d) Aurobindo Ghosh’s inability to be elected as the
Thermonuclear Experimental Reactor’. If this
President of the Indian National Congress
experiment succeeds, what is the immediate
advantage for India? 75. The plan of Sir Stafford Cripps envisaged that after
(a) It can use thorium in place of uranium for the Second World War
power generation (a) India should be granted complete independence
(b) It can attain a global role in satellite navigation (b) India should be partitioned into two before
(c) It can drastically improve the efficiency of its granting independence
fission reactors in power generation (c) India should be made a republic with the
(d) It can build fusion reactors for power condition that she will join the Commonwealth
generation (d) India should be given Dominion status
9
CULTURE/ANCIENT/MEDIEVAL 83. With reference to the cultural history of medieval
India, consider the following statements :
76. Banjaras during the medieval period of Indian
1. Siddhas (Sittars) of Tamil region were
history were generally
monotheistic and condemned idolatry.
(a) agriculturists (b) warriors
(c) weavers (d) traders 2. Lingayats of Kannada region questioned the
77. Who of the following had first deciphered the edicts theory of rebirth and rejected the caste
hierarchy.
of Emperor Ashoka?
(a) Georg Bilhler (b) James Prinsep Which of the statements given above is/are correct?
(c) Max Muller (d) William Jones (a) 1 only (b) 2 only
78. With reference to the religious history of India, (c) Both 1 and 2 (d) Neither 1 nor 2
consider the following statements : 84. Regarding the taxation system of Krishna Deva, the
1. The concept of Bodhisattva is central to ruler of Vijayanagar, consider the following
Hinayana sect of Buddhism. statements:
2. Bodhisattva is a compassionate one on his way 1. The tax rate on land was fixed depending on the
to enlightenment. quality of the land.
3. Bodhisattva delays achieving his own salvation
2. Private owners of workshops paid an industries
to help all sentient beings on their path to it. tax.
Which of the statements given above is/are correct?
Which of the statements given above is/are correct?
(a) 1 only (b) 2 and 3 only
(c) 2 only (d) 1, 2 and 3 (a) 1 only (b) 2 only
79. What is/are common to the two historical places (c) Both 1 and 2 (d) Neither 1 nor 2
known as Ajanta and Mahabalipuram? 85. Which one of the following books of ancient India
1. Both were built in the same period. has the love story of the son of the founder of Sunga
2. Both belong to the same religious denomination. dynasty?
3. Both have rock-cut monuments. (a) Swapnavasavadatta (b) Malavikagnirnitra
Select the correct answer using the code given (c) Meghadoota (d) Ratnavali
below. 86. In the context of the history of India, consider the
(a) 1 and 2 only following pairs:
(b) 3 only
(c) 1 and 3 only  Te rm   D escription
(d) None of the statements given above is correct (1) Eripa tti Lan d, reven ue from w h ich w as
80. With reference to the economic history of medieval set a part for the main ¬ten an ce
India, the term Araghatta’ refers to of th e village ta n k
(a) bonded labour (2) Ta n iyurs Villa ges don ated to a single
(b) land grants made to military officers Bra h min or a group of
(c) waterwheel used in the irrigation of land Bra h min s
(d) wastel and converted to cultivated land (3) Gh atikas Colleges gen era lly a ttach ed to
81. With reference to the cultural history of India, the th e temples
memorizing of chronicles, dynastic histories and
Epic tales was the profession of who of the Which of the pairs given above is/are correctly
following? matched?
(a) Shramana (b) Parivraaj (a) 1 and 2 (b) 3 only
(c) Agrahaarika (d) Maagadha (c) 2 and 3 (d) 1 and 3
82. Consider the following pairs: POLITY
Famous place  Region 87. With reference to the ‘Gram Nyayalaya Act’, which
1. Bodhgaya  Baghelkhand of the following statements is/are correct?
2. Khajuraho     Bundelkhand 1. As per the Act, Gram Nyayalayas can hear only
3. Shirdi Vidarbha civil cases and not criminal cases.
4. Nasik (Nashik) Malwa 2. The Act allows local social activists as
mediators/reconciliators.
5. Tirupati  Rayalaseema
Select the correct answer using the code given
Which of the pairs given above are correctly
below.
matched?
(a) 1, 2 and 4 (b) 2, 3, 4 and 5 (a) 1 only (b) 2 only
(c) 2 and 5 only (d) 1, 3, 4 and 5 (c) Both 1 and 2 (d) Neither 1 nor 2
10
88. The Parliament of India acquires the power to Which of the statements given above is/are correct?
legislate on any item in the State List in the national (a) 1 only (b) 2 only
interest if a resolution to that effect is passed by the (c) Both 1 and 2 (d) Neither 1 nor 2
(a) Lok Sabha by a simple majority of its total 94. ‘Doctors Without Borders (Medecins Sans
membership Frontieres)’, often in the news, is
(b) Lok Sabha by a majority of not less than two- (a) a division of World Health Organization
thirds of its total membership (b) a non-governmental international organization
(c) Rajya Sabha by a simple majority of its total (c) an inter-governmental agency sponsored by
membership European Union
(d) Rajya Sabha by a majority of not less than two- (d) a specialized agency of the United Nations
thirds of its members present and voting 95. Which of the following is not a member of ‘Gulf
89. Rashtriya Garima Abhiyaan’ is a national campaign Cooperation Council’?
to (a) Iran (b) Saudi Arabia
(a) rehabilitate the homeless and destitute persons (c) Oman (d) Kuwait
and provide them with suitable sources of 96. ‘Belt and Road Initiative’ is sometimes mentioned in
livelihood the news in the context of the affairs of
(b) release the sex workers from their practice and (a) African Union (b) Brazil
provide them with alternative sources of (c) European Union (d) China
livelihood 97. Consider the following pairs :
(c) eradicate the practice of manual scavenging Community sometimes in the affairs of mentioned
and rehabilitate the manual scavengers in the news
(d) release the bonded labourers from their 1. Kurd Bangladesh
bondage and rehabilitate them
2. Madhesi Nepal
90. Consider the following statements:
1. The minimum age prescribed for any person to 3. Rohingya Myanmar
be a member of Panchayat is 25 years. Which of the pairs given above is/are correctly
2. A Panchayat reconstituted after premature matched?
dissolution continues only for the remainder (a) 1 and 2 (b) 2 only
period. (c) 2 and 3 (d) 3 only
Which of the statements given above is/are correct? 98. With reference to ‘Organization for the Prohibition
(a) 1 only (b) 2 only of Chemical Weapons (OPCW)’, consider the
(c) Both 1 and 2 (d) Neither 1 nor 2 following statements :
91. Which of the following statements is/are correct? 1. It is an organization of European Union in
1. A Bill pending in the Lok Sabha lapses on its working relation with NATO and WHO.
prorogation. 2. It monitors chemical industry to prevent new
2. A Bill pending in the Rajya Sabha, which has weapons from emerging.
not been passed by the Lok Sabha, shall not 3. It provides assistance and protection to States
lapse on dissolution of the Lok Sabha. (Parties) against chemical weapons threats.
Select the correct answer using the code given below. Which of the statements given above is/are correct?
(a) 1 only (b) 2 only (a) 1 only (b) 2 and 3 only
(c) Both 1 and 2 (d) Neither 1 nor 2 (c) 1 and 3 only (d) 1, 2 and 3
92. Consider the following statements :
1. The Chief Secretary in a State is appointed by
DEFENSE
the Governor of that State. 99. Which one of the following is the best description of
2. The Chief Secretary in a State has a fixed tenure. ‘INS Astradharini’, that was in the news recently?
Which of the statements given above is/are correct? (a) Amphibious warfare ship
(a) 1 only (b) 2 only (b) Nuclear-powered submarine
(c) Both 1 and 2 (d) Neither 1 nor 2 (c) Torpedo launch and recovery vessel
(d) Nuclear-powered aircraft carrier
INTERNATIONAL RELATIONS & DEFENSE
93. Consider the following statements :
PERSON IN NEWS
The India-Africa Summit 100. A recent movie titled ‘The Man Who Knew Infinity’
1. held in 2015 was the third such Summit is based on the biography of
2. was actually initiated by Jawaharlal Nehru in (a) S. Ramanujan (b) S. Chandrasekhar
1951 (c) S. N. Bose (d) C. V. Raman
11

HINTS & SOLUTIONS


1. (d) The Brahmaputra enters India in the state of conservation of Globally Important Agricultural
Arunachal Pradesh from its original source Heritage Systems (GIAHS). The GIAHS Initiative
Tibet, and is joined by the Dibang River and the promotes public understanding, awareness,
Lohit River at the head of the Assam Valley. It is national and international recognition of
joined in Sonitpur by the Kameng River (or Jia Agricultural Heritage systems. It aims to
Bhoreli). identify and ensure global recognition of the
Brahmaputra’s main left bank tributaries, viz., importance of unique traditional agricultural
Dibang or Sikang and Lohit. The important systems for food security and sustainable
right bank tributaries are the Subansiri, development. The GIAHS initiative explicitly
Kameng, Manas and Sankosh. recognises that change in “traditional”
Therefore, all 3 correct. political, social and economic processes is
Ref. NCERT Physical Geography Class11, Ch.3 inevitable; they cannot be frozen or re-created.
Drainage system Page, 26 (ii) There is no provision to give Geographical
2. (d) Research is under process for presence of shale indication to “ALL” varieties. Hence statement
gas in Cambay basin at Mehsana, Ahmedabad 3 is wrong. By elimination method, we are left
and Bharuch districts of Gujarat, Cauvery basin with answer B- only 2.
at Nagapattinam in Tamil Nadu and in KG 6. (c) (i) This initiative aims to demonstrate the
Basin at East and West Godavari districts of improved production and post-harvest
Andhra Pradesh. technologies in an integrated manner to
Ref: As per Economic Survey 2013, page 196 catalyze increased production of millets in the
3. (b) (i) India took a step forward in its ambitious but country. The scheme, through processing and
long-pending goal to interlink major rivers to value addition techniques, is expected to
form a national water grid. The Pattiseema generate consumer demand for millet based
project lifts flood water from the river Godavari food products. Poor, small, marginal and tribal
and pumps it into the Polavaram right canal farmers have larger stake in this scheme.
that empties into the river Krishna in (ii) 1st and 2nd statements are right. Policy aims
Vijayawada. to give free kits but “wanting to shift
(ii) Andhra Pradesh Government is interlinking commercial crop farmers to millet” doesn’t fit
Krishna-Godavari rivers through the logic. There is no such mention in the scheme,
Pattiseema lift irrigation scheme. nor any talk about giving micro-irrigation
Ref: The hindu/ HYDERABAD, July 19, 2016 equipment. Hence 3rd statement wrong.
http://www.thehindu.com/news/national/andhra- Ref:http://agricoop. nic.in/Admin_Agricoop/
pradesh/after-river-interlinking-focus-on-water- Uploaded_ File/INSIMP.pdf
tanks/article8867726.ece 7. (c) (i) Advantages of Drip Irrigation:
4. (d) (i) The Rajasthan government is exploring (a) Maximum use of available water.
ways to develop an artificial inland port in (b) No water being available to weeds.
Jalore by bringing in Arabian Sea water into (c) Maximum crop yield.
Rajasthan through Gujarat. If implemented, (d) High efficiency in the use of fertilizers.
Rajasthan will become accessible by water (e) Less weed growth and restricts population
transport and will be able to shed the tag of a of potential hosts.
landlocked state. (f) No soil erosion.
(ii) Rajasthan wants to develop artificial inland (ii) Drip irrigation wets less ground, hence less
shipping port at Jalore, it’ll be connected to the weed seeds germinate. And over-irrigation of
Arabian Sea by developing a channel along the crops by using techniques such as drip irrigation
Kutch Creek. can increase the soil salinity. Thus by elimination
Ref: According to TimesofIndia / Jaipur-Jul 24, 2015; method, we get answer (c) only 1 and 3.
http://timesofindia.indiatimes.com/city/jaipur/ Ref: Environment website Gov. of Australia ;http://
Rajasthan-government-mulls-inland-port-in-Jalore/ www.environment.nsw.gov.au/salinity/solutions/
articleshow/48195329.cms irrigation.htm
5. (b) (i) In order to safeguard and support the 8. (b) (i) Spraying urea with neem oil slows the
world’s agri-cultural heritage systems, in 2002 release of nitrogen, by about 10 to 15 per cent,
FAO started an initiative for the dynamic concomitantly reducing consumption of the
12
fertiliser. Many research studies in India have Change (UNFCCC) since 2005, with the
conclusively established that neem oil acts as an objective of mitigating climate change through
effective nitrification inhibitor if coated onto urea reducing net emissions of greenhouse gases
(ii) Spraying urea with neem oil slows the through enhanced forest management in
release of nitrogen, by about 10 to 15 per cent, developing countries.
concomitantly reducing consumption of the (ii) Under REDD+ Developing country will
fertiliser. Hence B is the closet answer. have to prove the ‘result’ they have fought
Ref: Busienss Standard Newspaper/ New Delhi deforestation without harming local communities
February 16, 2016; http://www.business-standard. or biological diversity. Only then, they’ll get the
com/article/economy-policy/neem-coated-truth- Money. Therefore, 1 and 2 are correct.
urea-policy-isn-t-a-game-changer-116021601168_1 Although REDD+ has galvanized significant
.html support internationally, among both developed
9. (b) (i) Pradhan Mantri Fasal Bima Yojana will also and developing countries, its implications for
seek to address a long standing demand of poverty alleviation at the local level remain
farmers and provide farm level assessment for unclear. hence 3rd statement is wrong.
localised calamities including hailstorms, Therefore Answer (a).
unseasonal rains, landslides and inundation. 13. (a) (i) The Greenhouse Gas Protocol (GHG
(ii) Post-harvest loss is included so #2 is right. Protocol) is the most widely used international
But Under the new scheme, farmers will have to accounting tool for government and business
pay a uniform premium of two per cent for all leaders to understand, quantify, and manage
kharif crops and 1.5 per cent for all rabi crops. greenhouse gas emissions. A decade-long
For annual commercial and horticultural crops, partnership between the World Resources
farmers will have to pay a premium of 5 per cent.
Institute (WRI) and the World Business Council
So, #1 is wrong. Therefore, answer (b) only 2.
for Sustainable Development (WBCSD), the
Ref: Indianexpress/ January 14, 2016; http://
GHG Protocol is working with businesses,
indianexpress.com/article/business/business-others/
governments, and
pradhan-mantri-fasal-bima-yojana-crop-insurance-
(ii) A is correct according to their "ABOUT US"
plan-to-entail-rs-8-8k-cr-outgo/
page
10. (c) (i) The Kyoto Protocol separates countries into
Ref : http://www.ghgprotocol.org/about-ghgp
two groups. Annex I includes developed
14. (a) Agenda 21 is a non-binding, voluntarily
nations, while Non-Annex I refers to
developing countries. implemented action plan of the United Nations
A CER is a certificate which is issued every time with regards to sustainable development. It is a
the United Nations prevents one tonne of CO2 product of the Earth Summit held in Rio de
equivalent being emitted through carbon Janeiro, Brazil, in 1992. Therefore, only first
projects registered with the Clean Development statement right.
Mechanism (CDM). 15. (a) (i) The COP-21 Climate Conference was held in
(ii) All three terms are associated with Kyoto Paris, France from 30 November to 12
Protocol. Therefore, answer “c” 3 only. December 2015. During this conference, India
11. (c) (i) The Economics of Ecosystems and and France have launched the International
Biodiversity (TEEB) is a study led by Pavan Solar Alliance (ISA). International Solar Alliance
Sukhdev. It is an international initiative to draw includes an alliance of 121 countries located
attention to the global economic benefits between Tropic of Cancer and Tropic of
of biodiversity. Its objective is to highlight the Capricorn.
growing cost of biodiversity loss and (ii) While ISA was launched on Sidelines of
ecosystem and to draw together expertise from Paris Summit, therefore first statement right
the fields of science, economics and policy to but it includes only the 121 countries between
enable practical actions. Capricorn and Cancer receiving sunlight for
(ii) As per the official page of the organization. 300 days or more. Hence 2nd statement wrong.
2 and 3 are correct, but there is no mention of 16. (b) (i) The agreement emphasized on urgent need
their association with UNEP, IMF. Hence 1st to address the significant gap between the
statement is wrong. Accordingly answer is “c” aggregate effect of Parties’ mitigation pledges in
12. (a) (i) Reducing emissions from deforestation and terms of global annual emissions of greenhouse
forest degradation (REDD) is a mechanism that gases by 2020 and aggregate emission pathways
has been under negotiation by the United consistent with holding the increase in the
Nations Framework Convention on Climate global average temperature to well below 2°C
13
above preindustrial levels and pursuing efforts quality of natural resources of the country. GIM
to limit the temperature increase to 1.5°C above puts the “greening” in the context of climate
preindustrial levels. change adaptation and mitigation, meant to
(ii) Paris Summit - not all nations have signed enhance ecosystem services like carbon
the agreement and there is no deadline to ratify sequestration and storage (in forests and other
it. hence, statement 1 is wrong. This eliminates ecosystems), hydrological services and
a and b. And developed countries committed to biodiversity; along with provisioning services
give $100 billion by 2020. But, for future, there is like fuel, fodder, small timber.
no mention of giving $1,000 billion dollars. (ii) Ref: India Yearbook 2016 page 338:
Therefore, third statement wrong. We are left Environment ministry has launched National
with Answer (b) only. Mission for a Green India through a consultative
process involving relevant stakeholders, aimed at
17. (b) At the United Nations Sustainable
both increasing the forest and tree cover by 5 million
Development Summit on 25 September 2015,
ha, as well as increasing the quality of the existing
world leaders adopted the 2030 Agenda for
forest cover in another 5 million ha. Hence only
Sustainable Development, which includes a set
statement 3 fits.
of 17 Sustainable Development Goals (SDGs) to
21. (a) (i) A Net metering is a billing mechanism that
end poverty, fight inequality and injustice, and
credits solar energy system owners for the
tackle climate change by 2030. The history of electricity they add to the grid. Net metering
the SDGs can be traced to 1972 when allows residential and commercial customers
governments met under the auspices of the who generate their own electricity from solar
United Nations Human and Environment power to feed electricity they do not use back
Conference. The ‘Club of Rome’ is a global into the grid.
think tank that deals with a range of (ii) Under Net-metering system, Customer
international political issues. installs a solar or windpower plant on his
Ref: UNDP & The Hindu premise, gets it grid-connected with the
http://www.undp.org/content/undp/en/home/…/ electricity distribution company (DISCOM)
post-2015-development-agenda.html Hence (a) is the apt choice.
18. (b) (i) Countries across the globe committed to 22. (d) (i) The Gadgil Commission was an
create a new international climate agreement environmental research commission appointed
by the conclusion of the U.N. Framework by the Ministry of Environment and Forests of
Convention on Climate Change (UNFCCC) India. The Commission submitted the report to
Conference of the Parties (COP21) in Paris in the Government of India on 31 August 2011.
December 2015. The Kasturirangan Committee Report has
(ii) INDC are associated with UNFCCC-Climate sought to balance the two concerns of
change. Hence b) is the apt choice. development and environment protection, by
19. (c) (i) United Nations Convention to Combat watering down the environmental regulation
Desertification (UNCCD) aims to promote effective regime proposed by the Western Ghats Ecology
action through innovative national programmes Experts Panel’s Gadgil report in 2012.
and supportive international partnerships. (ii) They’re associated with Western Ghat
UNCCD is committed to a bottom-up approach, conservation plan.
encouraging the participation of local people in 23. (d) (i) The Bureau of Energy Efficiency is an agency
combating desertification. of the Government of India, under the Ministry
(ii) As per of the convention, Statement 3 is of Power created in March 2002 under the
right. But UNCCCD focuses on Africa, Asia, provisions of the nation’s 2001 Energy
Latin America and the Caribbean, Northern Conservation Act. The agency’s function is to
Mediterranean and Central and Eastern develop programs which will increase the
Europe. Hence statement 2 is wrong. Therefore, conservation and efficient use of energy in
answer (c). India. The government has proposed to make it
Ref:http://www.unccd.int/en/regional-access/Pages/ mandatory for all appliances in India to have
default.aspx/ ratings by the BEE starting in January 2010.
20. (c) (i) The National Mission for a Green India was (ii) According to centre for science and
announced by the Prime Minister as one of the environment (CSE), both 1 and 3 are correct.
eight Missions under the National Action Plan There is only option (d) whether such
on Climate Change (NAPCC). It recognizes that combination is possible.
climate change phenomenon will seriously Ref:http://cseindia.org/content/energy-efficient-
affect and alter the distribution, type and appliances
14
24. (b) (i) The API level is based on the level of 6 27. (a) Scientists at the Botanical Survey of India (BSI)
atmospheric pollutants, namely sulfur dioxide have discovered a new species of banana from a
(SO2), nitrogen dioxide (NO2), suspended remote tropical Krishna Nalah rain forest on
particulates smaller than 10 µm in aerodynamic the Little Andaman islands. The new species is
diameter (PM10), suspended particulates smaller about 11 metres high, whereas as the usual
than 2.5 µm in aerodynamic diameter (PM2.5), banana species is about three to four meters
carbon monoxide (CO), and ozone (O3) high. The fruit pulp is orange in colour,
measured at the monitoring stations throughout. distinctive from the white and yellow color of
So by elimination we are left with option “b”. regular bananas. Hence (a) is the correct answer.
Ref: Thehindu newspaper/ April 9, 2015; Ref: TheHindu/October 11, 2015
http://www.thehindu.com/opinion/blogs/blog- http://www.thehindu.com/news/national/other-
datadelve/article7083985.ece states/andamans-yield-a-sweet-banana-with-orange-
25. (a) (i) National Ganga River Basin Authority pulp/article7750316.ece
(NGRBA) is a financing, planning, 28. (a) (i) Pterocarpus santalinus, with the common
implementing, monitoring and coordinating names red sanders, red sandalwood, and
authority for the Ganges River, functioning saunderswood, is a species of Pterocarpus
under the water resource ministry of India. The
endemic to the southern Eastern Ghats
mission of the organisation is to spearhead the
mountain range of South India. This tree is
river conservation efforts at the national level.
valued for the rich red color of its wood. The
(ii) In 2014, UPSC asked similar question,
wood is not aromatic.
where one of the statement was- “PM is
(ii) It was in national media during April 2015,
chairman of NGRBA”. That statement was
when Andhra Pradesh police killed 20 Red
right as per India Yearbook 2014. Therefore,
Sander smugglers in an encounter. So first
here statement 3 is wrong, CM can’t by the
statement is right.
chairman of NGRBA. By elimination, we reach
answer (a). (iii) Red Sanders is associated with Tropical dry
26. (a) These camels can swim up to three kilometers deciduous forest, as per environment ministry
into the sea in search of mangroves - So, 1 and 2 website, Hence 2nd statement wrong.
are correct.Most families in Jatt and Rabari 29. (d) In June 2015, Maharashtra government has
communities of Kachchh, are traditional declared the Blue Mormon (Papilio
rearers of Kharai camels. So, third statement polymnestor) as the State butterfly.
also correct. Ref: TheHindu newspaper(http://www.thehindu.com
Kharai camels found in the Kutch region are /news/national/other-states/maharashtra-gets-state-
very unique. They can can swim through deep butterfly/article7342955.ece MUMBAI, June 22, 2015)
sea waters and it feeds on mangroves and other 30. (d) Frontline/ENVIRONMENT "WILDLIFE
saline plants. The breeders have meticulously Print edition : February 6, 2015
followed the traditional grazing pattern in the http://www.frontline.in/environment/wild-life/cry-
coastal districts. They do not provide special from-the-ghats/article6805391.ece
housing or shelter to the camels.Given the 31. (c) (i) With effect from 1st April, 2016, all loans in
breed’s ability to survive both on land and sea, India shall be priced with reference to Marginal
the Kharai camel is one of the most preferred Cost of Funds based Lending Rates (MCLR)
choices of graziers in the arid coastal region of which will comprise of :- 1. Marginal Cost of
Kachchh. People consume its milk, while male Funds 2. Negative carry on account of CRR
calves are sold for economic returns (females 3. Operating Costs 4. Tenor of premium. Banks
are not sold because they are considered shall review and publish their MCLR every
sacred). A male calf fetches anywhere between month on a pre-announced date. Marginal Cost
Rs. 6,000 and Rs.14,000, says Ramesh Bhatti of of Funds will comprise of marginal cost of
Sahjeevan, an NGO working on livelihood borrowings and return on net-worth.
issues of graziers in Kachchh. It can be (ii) Both right. Verbatim lifted from RBI's press
domesticated also – so its not wild. They are statement first paragraph.
reared in four blocks of Abdasa, Bundra, Ref : https://rbi.org.in/SCRIPTs/BS_PressRelease
Lakhpat and Bachau of Gujarat. Display.aspx?prid=35749
Ref : DOWN TO EARTH (An important source for Env) 32. (b) (i) They can enable transfers and remittance
Rare Kharai camel species, found in Kutch district, is through mobile phones. The RBI guidelines say
under threat due to the onslaught of industrialisation, that the payments bank cannot undertake
post-2001 earthquake. lending activities.
15
(ii) Mobile phone companies and supermarket 38. (c) (i) In order to reduce the fiscal deficit, the
eligible. But, Payment banks can’t issue credit government needs to increase its earnings and
card or give loans. Go by eliminating all options reduce its expenditure. This is done by
with “2”, and you’re left with (b) 1 and 3 only. following methods: Cut down the subsidies;
33. (a) (i) Core Banking Solution (CBS) is networking reforms in tax structures; improve profit
of branches, which enables Customers to generated by Public Sector Enterprises and
operate their accounts, and avail banking austerity measures.
services from any branch of the Bank on CBS (ii) Just by applying logic i.e. to reduce deficit,
network, regardless of where he maintains his we’ve to increase income and reduce Expenditure.
account. The customer is no more the customer Does it Does it
of a Branch. He becomes the Bank’s Customer. Option increase reduce
Thus CBS is a step towards enhancing customer income Expenditure
convenience through Anywhere and Anytime 1. Reducing revenue expenditure – YES
Banking. 2. Introducing new welfare schemes – NO
(ii) 3 is definitely not the purpose, so by
3. Rationalizing subsidies – YES
elimination the answer is (a).
4. Reducing import duty NO –
34. (c) (i) Financial Stability and Development
Council is an apex-level body constituted by So, answer is only 1 and 3.
the government of India. The idea to create 39. (d) (i) The main items of capital budget are receipts
and expenditure for capital (Financial) gains. It
such a super regulatory body was first mooted
also includes loans raised by Government from
by the Raghuram Rajan Committee in 2008.
public, Reserve Bank and other parties, and
Finally in 2010, the then Finance Minister of
from foreign Governments and bodies. It also
India, Pranab Mukherjee, decided to set up
includes capital expenditure on acquisition of
such an autonomous body dealing with macro
assets like land, buildings, machinery,
prudential and financial regularities in the
equipment, etc and loans and advances granted
entire financial sector of India.
by Central Government to State and Union
(ii) It falls under Dep. Of economic affairs and
Territory Governments, Government companies,
FM is the chairman. So, 1 is wrong and 2 is right.
Corporations and other parties.
By elimination, we are left with answer “c”.
Ref: to the parts of Budget given in Disha’s Crack
35. (b) (i) Bitcoin are not maintained by the Central
CSAT Paper-1, page E-56 (3rd Ed.)
Bank. Bitcoins are created as a reward for
40. (d) (i) It involves 12 countries: the US, Japan,
payment processing work in which users offer
Malaysia, Vietnam, Singapore, Brunei, Australia,
their computing power to verify and record
New Zealand, Canada, Mexico, Chile and Peru.
payments into a public ledger. The pact aims to deepen economic ties between
(ii) They’re not tracked by any central bank or these nations, slashing tariffs and fostering
authority. Both 2 and 3 are right. trade to boost growth. It was signed on 4
36. (c) There is no restriction as such that only one February 2016 in Auckland, New Zealand.
member of a family can join the scheme. (ii) TPP is meant for free trade. So second
Ref: Given in Disha Publication’s Mega Yearbook statement is wrong. We’ve to cross check the
2016, page-84. list of Pacific rim countries against TPP nations.
37. (b) (i) Base Erosion and Profit Shifting (BEPS) is The Pacific Rim includes countries bordering
a tax avoidance strategy used by multinational the Pacific Ocean, but North Korea and
companies, wherein profits are shifted from Columbia also Pacific rim countries. But not in
jurisdictions that have high taxes (such as the TPP list of countries (https://ustr.gov/tpp/)
United States and many Western European similarly Columbia absent. Therefore, answer
countries) to jurisdictions that have low (or no) (d). neither 1 nor 2 correct.
taxes (so-called tax havens). The BEPS project is 41. (b) The European Stability Mechanism is a
said to be an “attempt by the world’s major European Union agency that provides financial
economies to try to rewrite the rules on assistance, in the form of loans, to eurozone
corporate taxation to address the widespread countries or as new capital to banks in
perception that the [corporations] don’t pay difficulty.
their fair share of taxes”. Ref: Indianexpress July 2015
(ii) BEPS aims to curb the tax evasion by http://indianexpress.com/article/opinion/editorials/
MNCs, hence B is the answer. its-a-deal-2/
16
42. (b) Regional Comprehensive Economic Partnership 47. (c) (i) The purpose of ‘Sovereign Gold Bond
(RCEP) is a proposed free trade agreement Scheme’ and ‘Gold Monetization Scheme’ is to
(FTA) between the ten member states of the reduce India’s gold imports and bring all the
Association of Southeast Asian Nations gold lying idle with individuals and households
(ASEAN) (Brunei, Burma (Myanmar), Cambodia, in India into the economy.
Indonesia, Laos, Malaysia, the Philippines, (ii) FDI is not the purpose - so all options
Singapore, Thailand, Vietnam) and the six involving 2 are wrong. 1 and 3 are right.
states with which ASEAN has existing FTAs Ref : Economic Survey Vol.2 Page 18 confirms.
(Australia, China, India, Japan, South Korea 48. (b) (i) The Global Financial Stability Report (GFSR)
and New Zealand). is a survey by the IMF staff published twice a
43. (a) In WTO terminology, subsidies in general are year, in the spring and fall. The report draws
identified by “Boxes” which are given the out the financial ramifications of economic
colours of traffic lights: green (permitted), issues highlighted in the IMF’s World Economic
amber (slow down — i.e. be reduced), red Outlook (WEO).
(forbidden). (ii) IMF prepares this report.
44. (d) The value of the SDR or XDR of IMF is based on Ref : thehindu:April 14, 2016 .
www.thehindu.com/business/Economy/world-
a basket of key international currencies
output-faces-risk -of-39-dro p-by-2021 /article
reviewed every five years. In the review
8472298.ece
conducted in November 2015, the IMF decided
49. (c) (i) The IMFC advises and reports to the IMF
that the Renminbi (Chinese Yuan) would be
Board of Governors on the supervision and
added to the basket effective October 1, 2016. management of the International Monetary and
From that date, the SDR or XDR basket will Financial System. It also considers proposals by
consist of the following five currencies: U.S. the Executive Board to amend the Articles of
dollar 41.73%, Euro 30.93%, Chinese Yuan 10.92%, Agreement and advises on any other matters
Japanese Yen 8.33%, British Pound 8.09%. that may be referred to it by the Board of
45. (c) (i) (a) The International Finance Corporation Governors. A number of international
(IFC), the investment arm of the World Bank, institutions, including the World Bank,
issued a ` 1,000 crore bond to fund infrastructure participate as observers in the IMFC’s meetings.
projects in India. (ii) Both statements are right as per the official
(b) These bonds were listed on the London IMF page: April 5, 2016.
Stock Exchange (LSE). Ref:http://www.imf.org/external/np/exr/facts/
(c) They are rupee-denominated bonds issued groups. htm#IC
to overseas buyers. 50. (d) (i) The stock of foreign exchange reserves in
(d) The purpose of the issue was to fund terms of months of retained imports of goods
infrastructure projects in India. IFC named as at end of year. It measures the number of
them ‘masala’ bonds to reflect the Indian angle months of money available in the national bank
to it. to cover the cost of imports.
Ref: TheEconomist; http://www.economist.com/
(ii) First statement is right and Second
node/16793524
statement initial part is also correct- they’re
51. (c) (i) Prime Minister launched the ‘Stand up
rupee denominated and ‘debt’ type of
India Scheme’ aimed at promoting
instrument. We’ve to check whether public entrepreneurship among Scheduled Castes/
sector firms also use it. answer is yes. Hence Scheduled Tribes and women by giving loans in
both are correct. the range of ` 10 lakh to ` 1 crore for setting up
Ref: http://indianexpress.com/article/business/ a new enterprise. There would be a ` 10,000
business-others/ifcs-rising-rupee-offerings-add- crore refinance window through Small Industries
spice-to-masala-bonds/ Development Bank of India (SIDBI) and the
46. (b) (i) The New Development Bank (NDB), National Credit Guarantee Trustee Company
formerly referred to as the BRICS Development Ltd (NCGTC) will create a corpus of ` 5,000
Bank, is a Multilateral Development Bank crore.
established by the BRICS states (Brazil, Russia, (ii) Both statements are correct as per : Press
India, China and South Africa). Its headquarter Information Bureau (PIB) Government of India
is in Shanghai, China. Ministry of Finance/03-April-2016.
(ii) First statement is wrong- its BRICS nations. Ref : http://pib.nic.in/newsitePrintRelease.aspx?relid
2nd is right - HQ is Shanghai. =138523
17
52. (a) (i) Pradhan Mantri Mudra Yojana under Ref: Press Information Bureau (PIB.) 2016,
the Micro Units Development and Refinance Government of India. Ministry of Human Resource
Agency (MUDRA) Bank is a new institution Development;
being set up by Government of India for http://pib.nic.in/newsite/PrintRelease.aspx?relid=
development and refinancing activities relating 148515.
to micro units. It was announced by the Finance 58. (c) DigiLocker is a “digital locker” service
Minister while presenting the Union Budget for launched by the Government of India in
FY 2016. The purpose of MUDRA is to provide February 2015 to provide a secure dedicated
funding to the non corporate small business personal electronic space for storing the
sector. documents of resident Indian citizens.
Ref: Given in Disha Mega Yearbook page-A-83. Ref: Indianexpress /November 17, 2015.
MUDRA Yojana aims to help small businessmen http://indianexpress.com/article/india/india-news-
who’re unable to get loans from formal financial india/digital-lockers-put-your-documents-in-a-safe-
system. Hence (a) is the answer. on-secure-cloud/
53. (c) The ease of doing business index is an index 59. (d) Four Indicators are undernourished population,
created by the World Bank Group. It is an wasting in Under 5 children; wasting is very
annual report on the state of health of low weight for height i.e very thin child;
economies based on detailed diagnostics not of Stunting in under 5 children; stunting is very
the relatively more visible features (such as low height for age i.e. very short child; Under 5
growth) and various macroeconomic parameters mortality rate.
(such as the public debt) but of underlying and Ref: As per the official site of IFPRI
embedded characteristics. 60. (b) A transcriptome is the full range of messenger
54. (a) In order to boost ‘Make in India’ campaign, first RNA, or mRNA, molecules expressed by an
national investment and manufacturing zone to organism. The term “transcriptome” can also
come up in Andhra Pradesh. The state assured be used to describe the array of mRNA
the Centre of availability of 10 sq km of land in transcripts produced in a particular cell or
tissue type.
one place in Prakasham district.
Ref: Textbook of Veterinary internal medicine.
Ref: Economic Times/ET Bureau Sep 21, 2015.
https://books.google.co.in/books?id=4Qzau1jag
http://articles.economictimes.indiatimes.com/2015-
OYC&pg=PA2118&dq=transcriptome+mRNA+
09-21/news/66760948_1_andhra-pradesh-nimz-
expressed+by+an+organism&hl=en&sa=X&ved=
manufacturing-zone
0ahUKEwiOn5Ug77OAhXEMY8KHVY3CtUQ6AEI
55. (d) Ujwal DISCOM Assurance Yojana (UDAY) is
KTAA#v=onepage&q=transcriptome%20mRNA%20
the financial turnaround and revival package
expressed% 20by %20an%20 organism&f=false
for electricity distribution companies of India
61. (a) (i) The Government of India launched
(DISCOMs) initiated by the Government of Mission Indradhanush in December 2014 with
India with the intent to find a permanent the ultimate goal to ensure that all children
solution to the financial mess that the power under the age of two years and pregnant
distribution is in. women are fully immunized with all available
Ref: Economic Survey Vol.2, page 125. vaccines.
56. (b) District Mineral Foundation (DMF) is a trust set (ii) This mission aims to achieve 100%
up as a non-profit body, in those districts vaccination of children and pregnant women
affected by the mining works, to work for the by 2020.
interest and benefit of persons and areas Ref: Disha Publication’s Mega Yearbook 2016, page
affected by mining related operations. It is B-484
funded through the contributions from miners. 62. (c) (i) Items 1, 2 and 4 are the mandatory
Its manner of operation comes under the information to the manufacturer to put these on
jurisdiction of the relevant State Government. the main label as per FSS(packaging & labeling)
Ref: Economic Survey Vol.2, page 141. regulations 2011.
57. (d) SWAYAM or Study Webs of Active–Learning (ii) According to official notification
for Young Aspiring Minds programme Ref:http://www.fssai.gov.in/Portals/0/Pdf/Food
of Ministry of Human Resource Development, %20Safety%20and%20standards%20(Packaging
Government of India. Professors of centrally %20and%20Labelling)%20regulation,%202011.
funded institutions like IITs, IIMs, central pdf.
universities will offer online courses to citizens Labels have to contains 1, 2 and 4 that much is
of India. All courses would be offered free of confirmed. And since UPSC has not given any
cost under this. option on “All correct”. Hence answer (c).
18
63. (d) (i) A virus is a small infectious agent that 68. (d) (i) The ITER (International Thermonuclear
replicates only inside the living cells of other Experimental Reactor) project aims to make the
organisms. Viruses can infect all types of life long-awaited transition from experimental
forms, from animals and plants to studies of plasma physics to full-scale
microorganisms, including bacteria and fungi. electricity-producing fusion power stations.
(ii) According to the type of the host they (ii) As such project started in 2006, to find
infect, viruses are classified mainly into the whether Nuclear fusion can be a source of
following four types: energy and electricity in future. Hence (d) is the
apt choice.
(a) Plant viruses including algal viruses-RNA/
Ref: Indianexpress/ June 28, 2016; http://
DNA
indianexpress.com/article/india/india-news-india/
(b) Animal viruses including human viruses-
india-on-schedule-with-deliveries-for-iter-fusion-
DNA/RNA reactor-official-2880396/
(c) Fungal viruses(Mycoviruses)-ds RNA 69. (c) (i) The Mars Orbiter Mission (MOM), also
(d) Bacterial viruses (Bacteriophages) including called Mangalyaan. It is India’s first interplanetary
cyanophages-DN mission and ISRO has become the fourth space
Hence answer “(d)”. agency to reach Mars, after the Soviet space
Ref: Class 11 science textbook of Tamilandu State program, NASA, and the European Space
board. Agency. It is the first Asian nation to reach Mars
64. (b) (i) Project loon is a network of balloons orbit, and the first nation in the world to do so
travelling on the edge of space, designed to in its first attempt.
extend internet connectivity to people in rural (ii) Statement 1 and 3 are right. Apart from
& remote areas worldwide. India and USA, Russia and European space
(ii) Under Project Loon, Google Inc. aims to agency also have it. European Space Agency’s
provide internet connectivity via using helium Mars Express has two components 1) Mars
balloons. Hence (b) is the apt choice. Orbiter 2) Beagle 2 Lander. Hence 2nd
65. (c) (i) This OWC technology uses light from light- statement is wrong. Hence by elimination we
emitting diodes (LEDs) as a medium to deliver are left with choice (c).
networked, mobile, high-speed communication 70. (a) (i) The British Government’s decision to partition
Bengal had been made public in December
in a similar manner to Wi-Fi. It is a
1903.
bidirectional, high-speed and fully networked
(ii) The Swadeshi and Boycott Movement was
wireless communication technology similar to
an effort by Indian people to oppose British
Wi-Fi. rule. It began when British officials separated
(ii) Li-fi gives data transfer rate of 10 gigabytes the Bengal province, dividing it by Bengali,
per second. So 1st statement is right. Li-Fi Hindi and Oriya languages and Muslim and
device circulates data via LEDs that emit an Hindu religions. Indians responded by
intermittent flicker at a speed imperceptible to boycotting British products and using only
the human eye. So, 2nd statement also right. Indian-made goods.
Ref: Feb 2016, theHindu ; http://www.thehindu.com (iii) Swadeshi and Boycott started against
/todays-paper/tp-international/lifi-data-transmission- Partition of Bengal in 1905.
through-light/article6218892.ece Ref: Disha’s Crack CSAT Paper-1, 2016(4th
66. (a) The ‘Greased Lightening-10’ (GL-10) is a 28 kg Edition) Page H-227.
with 10 engines that can take off and land like a 71. (c) Satyashodhak Samaj is a society established
helicopter and fly efficiently like an aircraft. It is by Jyotirao Phule on September 24, 1873. This
a battery-powered drone prototype plane was started as a group whose main aim was to
designed by NASA. liberate the social shudra and untouchable
67. (d) (i) India has become only the fifth country after castes from exploitation and oppression.
the US, Russia, Europian Union and Japan, to Ref:Class 12 NCERT History, Page 130.
have an eye scanning the exotic depths of the 72. (d) (i) The Montagu–Chelmsford Reforms were
reforms introduced by the British Government
universe after the ASTROSAT, the Indian Space
in India to introduce self-governing institutions
Research Organisation’s multi-wavelength space
gradually to India. The reforms were outlined in
observatory, was successfully placed in orbit.
the Montagu-Chelmsford Report prepared in
(ii) Astrosat weighs 1513 kgs. Hence 2nd 1918 and formed the basis of the Government of
statement also wrong. India Act 1919.
Ref: theHindu/ September 28, 2015; http:// (ii) They’re related to Constitutional reforms.
www.thehindu.com/sci-tech/science/isro-launches- Ref: Disha’s Crack CSAT Paper-1, 2016(4th
astrosat-first-space-observatory/article7697707.ece Edition) Page H-228.
19
73. (b) In 1881, Keshab Chandra Sen established Thus, Boddhisattva is central to Mahayan sect
NabaBidhan (New Dispensation) meaning new of Buddhism. Hence first statement is wrong.
universalist religion after having differences While 2nd and 3rd are correct.
within BrahmoSamaj. He was also part of Ref : As per Std 12th NCERT, Themes in India
Indian reform association to legalize Brahmo History, Part-I, Page 103.
marriage and to fix minimum age of marriage. 79. (b) First and second statements are wrong because
Calcutta Unitarian Committee was formed by Ajanta (Buddhist caves) was built from 2nd
Raja Rammohan Roy, Dwarkanath Tagore and Century BCE to 5th Century CE, while
William Adam, hence irrelevant to the Mahabalipuram (Hindu temple) was built
question. Hence Answer (b) only 2 and 3. during 7-8th Centuries by Pallava Kings. Third
74. (b) (i) Extremists and moderates both seemed to be statement is correct- both are rock cut
locking horns. The Extremists thought that the monuments.
people had been encouraged and the battle for Ref: Class-11 NCERT, Page 83.
freedom had begun. They felt the time had 80. (c) The Persian wheel is a mechanical water lifting
come for the big push to drive the British out device operated usually by draught animals
and considered the Moderates to be a like bullocks, buffaloes or camels. It is used to
stumbling block to the movement. lift water from water sources typically open
(ii) So, Surat split occurred because extremists wells. In Sanskrit the word Araghatta has been
were dissatisfied with Moderates’ capacity to used in the ancient texts to describe the Persian
negotiate with the British. Wheel. The ‘ara-ghatta’ comes from the
75. (d) (i) The main proposals of the plan of Sir combination of the words ‘ara’ meaning spoke
Stafford Cripps was that an Indian Union with and ‘ghatta’ meaning pot.
a dominion status would be set up; it would be Ref: 7thStd NCERT, Our Past II, Page 6-7. It
free to decide its relations with the discusses about the invention of water-wheel
Commonwealth and free to participate in the irrigation, known in Persian as “Agrahatta”.
United Nations and other international bodies.
81. (d) The Maagadha and the suutas were associated
(ii) Cripps proposed to give dominion status
with memorizing of chronicles, dynastic
to India after WW2.
histories and Epic tales.
Ref. Disha’s Crack CSAT Paper-1, 2016(4th
82. (c) Khajuraho is located in the Bundelkhand region
Edition) Page H-231.
of Madhya Pradesh. Tirupati is located in
76. (d) In India, Banjara people were transporters of
Rayalaseema region.
goods from one place to another and the goods
83. (c) (i) Siddhas means the refined monotheistic
they transported included salt, grains, firewood
creed existed. Sittar implies an expert in
and cattle.Thus, Banjaras were trader-nomads.
occultism, alchemy and so on with magic or
Ref: Class 7 NCERT Page 94-95.
superstitious powers.
77. (b) In 1837, British archeologist and historian
James Prinsep decoded the edicts of emperor (ii) The lingayats believed that on death the
Ashoka. Prinsep’s inscription proved to be a devotee will be reunited with Shiva and not
series of edicts issued by a king calling himself return to this world. Therefore instead of
“Beloved-of-the-Gods, King Piyadasi.” cremating the body, they ceremonially bury
Ref: Std 12th NCERT, Themes in India History, their dead.
Part-I, Page 28-29 (iii) They criticised the caste system and
78. (b) The concept of Bodhisattva is central to the questioned belief in the theory of rebirth.
Mahayana doctrines of Buddhism. The term (iv) Certain practices were encouraged by the
‘bodhisattva’ literally means ‘one who has lingayats such as post-puberty marriages and
enlightenment as his/her essence’. A widow remarriage. Knowledge of the
bodhisattva is a being who is destined for Virashaiva tradition has been derived from
enlightenment rather than one who has gained vachanas (sayings) composed in Kannada by
it already. A bodhisattva is also normally those who had joined
thought of as consciously working towards Ref: Std 12th NCERT, Themes in India History,
enlightenment: you can’t call someone a Part-II, Page 148
bodhisattva just because they might be 84. (c) (i) In reign of Krishna Deva, land revenue was
enlightened in the future if they haven’t started the main source of income of the government.
making an effort yet. For this reason, the earliest To assess the proper revenue entire land was
use of the term ‘bodhisattva’ refers to classified into four parts: the wet land, the dry
Siddhartha Gautama before he gained land, the orchards and the woods. The
enlightenment, and also in his previous lives. government also levied other taxes like the
20
grazing tax, the customs duty, tax on gardening 90. (b) This question can be directly solved from Disha
and industries taxes on manufacture of various publication’s Polity compendium, page P-121.
articles. • For PRI Bodies minimum age is 21 years, to
(ii) From the travelogue written by contest elections, so first statement is wrong.
Persianenvoy Abdur Razzaqin the court of • 2nd statement is correct- verbatim given in
Vijaynagar, we find both statements are correct. the last line of p-121.Hence answer only B)
85. (b) The Malavikagnimitra is a Sanskrit play by 91. (b) • prorogation doesn’t affect the bills of any
Kalidas. The play tells the love story of other business pending before the house so 1st
Agnimitra, the Shunga Emperor. He falls in statement is wrong.
love with the picture of an exiled servant girl Ref. Disha Polity Compendium Page P-54.
named Malavika. • A bill pending in the Rajya Sabha but not
86. (d) A special category of land, eripatti, or land, is passed by the Lok Sabha does not lapse if the
mentioned. Donated by individuals, the president has notified the holding of a joint
revenue from this land was set apart for the sitting before the dissolution of Lok Sabha. So,
maintenance of the village tank. statement 2 is right.
Ref: A History of India by Romila Thapar. 92. (a) 1st statement is right. Because
87. (b) • Gram Nyayalaya try criminal cases, civil • Under Article 168 of the Constitution of
suits, claims or disputes which are specified in India: All executive actions of the Governor of a
the First Schedule and the Second Schedule to State shall be expressed to be taken in the name
the Act.The Gram Nyayalaya are supposed to of Governor. (which include transfer, posting,
try to settle the disputes as far as possible by promotion of civil servants allotted to that state
bringing about conciliation between the parties cadre.) so, Yes, as such chief Secretary is
and for this purpose, it can make use of the "chosen" by Chief minister officially its
appointed conciliators. expressed that chief Secretary is 'appointed' by
• Reference- Page no. 601 of India year book the governor of the state.
2016, under heading Judiciary it says- Panchayat • Another way to look at it is- even state
Courts also function in some states under advocate general, university vice chancellors are
various names like Nyaya Panchayat, Panchayat 'chosen' by CM from among his favorite people-
but officially they're 'appointed' by the Governor.
Adalat, Gram Kachehri, etc., to decide civil and
Ref: Page 26.5- Indian Polity by M. Laxmikanth,
criminal disputes of petty and local nature.
4th Edition (Macgrawhill Publication)
That means first statement is wrong.
93. (a) • 1st statement is right: Third India-Africa
• Under this act, District court with
Forum Summit was held in New Delhi (India)
consultation of DM, prepares panel of social
in from from 26–30 October 2015 under the
workers to act as councilors. Hence 2nd
theme: “Partners in Progress: Towards a Dynamic
statement is right.
and Transformative Development Agenda”
Ref: Original Act Page no 7; http://doj.gov.in
• 2nd statement is wrong: because First
88. (d) Given verbatim in Disha Publications Crack CSAT
Summit was held in New Delhi in April 2008.
Paper-1, 2016(4th Edition)Page p-101
Therefore, it can’t be Nehru’s brainchild.
• When Rajya Sabha Passes a Resolution If
Ref: https://en.wikipedia.org/wiki/India%E2%80%
the Rajya Sabha declares that it is necessary in
93 Africa_ Forum_Summit
the national interest that Parliament should 94. (b) It is a non-governmental and non-military
make laws on a matter in the State List, then the organization established by group of French
Parliament becomes competent to make laws doctors in 1971 - with its Headquarter in
on that matter. Such a resolution must be Brussels, Belgium. They were in news during
supported by two-thirds of the members 2014-15 because of their relief work in Ebola
present and voting. affected countries.
89. (c) (i) It is a campaign for eradication of inhuman 95. (a) Iran is not the member of this middle eastern
practice of manual scavenging and comprehensive organization.
rehabilitation of manual scavengers in India. Gulf Cooperation Council (GCC) is a political
Ashif Shaikh is well known for his role in the and economic alliance of six Middle Eastern
campaign (Rashatriya Garima Abhiyan) through countries—Saudi Arabia, Kuwait, the United
various campaigns of Jan Sahas. Arab Emirates, Qatar, Bahrain, and Oman.
(ii) Rashtriya Garima Abhiyaan is an NGO-led 96. (d) The Belt and Road (abbreviated B&R), One
movement for manual scavengers. Belt, One Road (abbreviated OBOR) or the Belt
Ref: The Hindu newspaper and Road Initiative is a development strategy
21
and framework, proposed by Chinese (ii) It won Nobel Peace prize in 2013. There is
paramount leader Xi Jinping that focuses on no specific mention of its association with EU,
connectivity and cooperation among countries NATO or WTO on the official website of OPCW.
primarily between the People’s Republic of Therefore, statement 1 is wrong. By elimination
China and the rest of Eurasia. of all options with statement 1, we are left with
Ref: The Hindu and Govt Websites http:// answer (b).
english.gov.cn/beltAndRoad/ 99. (c) INS Astradharini is an indigenously designed
97. (c) The Kurds are an ethnic group in the Middle and built torpedo launch and recovery vessel
East Asia. The Madhesi , also referred to as built by Shoft Shipyard for Indian Navy. It was
Teraibasi Nepali are an indigenous ethnic commissioned by Indian Navy on 6 October
group of Nepalese people who are natives of 2015 at Naval Base in Visakhapatanam.
the Madhesh plains of Southern Nepal in Terai Ref: India Yearbook 2016 page 972
belt of South Asia. The Rohingya people, are 100. (a) (i) ‘The Man who knew infinite’ is a
biographical movie of Indian Mathematician
Muslim Indo-Aryan peoples from the Rakhine
Srinivas Ramanujan (played by Dev Patel) and
State, Myanmar.
his association with British mathematician GH
98. (b) (i) The OPCW Member States share the
Hardy. Ramanujan was a self-taught (autodidatic)
collective goal of monitoring chemical industry
Mathematician- born and died in Madras
to prevent new weapons from re-emerging; Presidency.
providing assistance and protection to States (ii) He, who after growing up poor in Madras,
Parties against chemical threats; and fostering India, earns admittance to Cambridge
international cooperation to strengthen University during World War I, where he
implementation of the Convention and promote became a pioneer in mathematical theories
the peaceful use of chemistry. with the guidance of his professor, G. H. Hardy.
INSTRUCTIONS
1. This Test Booklet contains 100 items (questions). Each item comprises four responses (answers). You will select
the response which you want to mark on the Answer sheet. In case, you feel that there is more than one correct
response, mark the response which you consider the best. In any case, choose ONLY ONE response for each item.
2. All items carry equal marks.
3. Penalty for wrong answers:
THERE WILL BE PENALTY FOR WRONG ANSWERS MARKED BY THE CANDIDATE FOR WRONG ANSWER.
(i) There are four alternatives for the answer to every question. For each question which has a penalty for which a
wrong answer has been given by the candidate, one-third of the marks assigned to that question will be deducted
as penalty.
(ii) If a candidate gives more than one answer, it will be treated as a wrong answer even if one of the given answers
happens to be correct and there will be same penalty as above to that question.
(iii) If a question is left blank, i.e., no answer is given by the candidate, there will be no penalty for that question.

1. If the interest rate is decreased in an economy, it will Which of the statements given above is/are correct?
(a) decrease the consumption expenditure in the economy (a) 1 and 2 only (b) 3 only
(b) increase the tax collection of the Government (c) 1 and 3 only (d) 1, 2 and 3
(c) increase the investment expenditure in the economy 5. Which one of the following is the process involved in
(d) increase the total savings in the economy photosynthesis?
2. Consider the following statements : (a) Potential energy is released to form free energy
1. The President shall make rules for the more convenient (b) Free energy is converted into potential energy and
transaction of the business of the Government of India, stored
and for the allocation among Ministers of the said (c) Food is oxidized to release carbon dioxide and water
business. (d) Oxygen is taken, and carbon dioxide and water vapour
2. All executive actions of the Government of India shall are given out
be expressed to be taken in the name of the Prime 6. In addition to fingerprint scanning, which of the following
Minister. can be used in the biometric identification of a person?
Which of the statements given above is/are correct? 1. Iris scanning
(a) 1 only (b) 2 only 2. Retinal scanning
(c) Both 1 and 2 (c) Neither 1 nor 2 3. Voice recognition
3. Consider the following statements regarding a No- Select the correct answer using the code given below.
Confidence Motion in India : (a) 1 only (b) 2 and 3 only
1. There is no mention of a No-Confidence Motion in (c) 1 and 3 only (d) 1, 2 and 3
the Constitution of India. 7. Which of the following statements is/are correct regarding
2. A Motion of No-Confidence can be introduced in the vegetative propagation of plants?
Lok Sabha only. 1. Vegetative propagation produces clonal population.
Which of the statements given above is/are correct?
2. Vegetative propagation helps in eliminating the virus.
(a) 1 only (b) 2 only
3. Vegetative propagation can be practiced most of the year.
(c) Both 1 and 2 (d) Neither 1 nor 2
Select the correct answer using the code given below.
4. With reference to Neem tree, consider the following
(a) 1 only (b) 2 and 3 only
statements :
(c) 1 and 3 only (d) 1, 2 and 3
1. Neem oil can be used as a pesticide to control the
8. Which of the following pairs is/are correctly matched?
proliferation of some species of insects and mites.
Spacecraft Purpose
2. Neem seeds are used in the manufacture of biofuels
and hospital detergents. 1. Cassini-Huygens : Orbiting the Venus and
3. Neem oil has applications in pharmaceutical industry. transmitting data to the Earth
2. Messenger : Mapping and investigating the . Which of the above are famous for the production of
Mercury traditional sarees/ fabric?
3. Voyager 1 and 2 : Exploring the outer solar (a) 1 and 2 only (b) 2 and 3 only
system (c) 1, 2 and 3 (d) 1,3 and 4.
Select the correct answer using the code given below. 17. Consider the following pairs :
(a) 1 only (b) 2 and 3 only National Highway Cities connected
(c) 1 and 3 only (d) 1, 2 and 3 1. NH 4 Chennai and Hyderabad
9. Consider the following pairs : 2. NH 6 Mumbai and Kolkata
Region Well-known for the 3. NH 15 Ahmedabad and Jodhpur
production of Which of the above pairs is/are correctly matched?
1. Kinnaur : Areca nut (a) 1 and 2 only (b) 3 only
2. Mewat : Mango (c) 1, 2 and 3 (d) None
3. Coromandel : Soya bean 18. Consider the following international agreements :
Which of the above pairs is/ are correctly matched? 1. The International Treaty on Plant Genetic Resources
(a) 1 and 2 only (b) 3 only for Food and Agriculture
(c) 1, 2 and 3 (d) None 2. The United Nations Convention to Combat
10. Which of the following is/are the example/ examples of Desertification
chemical change? 3. The World Heritage Convention
1. Crystallization of sodium chloride? Which of the above has/have a bearing on the biodiversity?
2. Melting of ice (a) 1 and 2 only (b) 3 only
3. Souring of milk (c) 1 and 3 only (d) 1, 2 and 3
Select the correct answer using the code given below. 19. Consider the following statements regarding ‘Earth Hour’ :
(a) 1 and 2 only (b) 3 only 1. It is an initiative of UNEP and UNESCO.
(c) 1, 2 and 3 (d) None 2. It is a movement in which the participants switch off
11. The power of the Supreme Court of India to decide disputes the lights for one hour on a certain day every year.
between the Centre and the States falls under its 3. It is a movement to raise the awareness about the climate
(a) advisory jurisdiction (b) appellate jurisdiction change and the need to save the planet.
(c) original jurisdiction (d) writ jurisdiction Which of the statements given above is/are correct?
12. Consider the following techniques phenomena : (a) 1 and 3 only (b) 2 only
1. Budding and grafting in fruit plants (c) 2 and 3 only (d) 1, 2 and 3
2. Cytoplasmic male sterility 20. Which one of the following is the correct sequence of a food
3. Gene silencing chain?
Which of the above is/are used to create transgenic crops? (a) Diatoms-Crustaceans-Herrings
(a) 1 only (b) 2 and 3 (b) Crustaceans-Diatoms-Herrings
(c) 1 and 3 (d) None (c) Diatoms-Herrings-Crustaceans
13. Consider the following statements : (d) Crustaceans-Herrings-Diatoms
1. Maize can be used for the production of starch. 21. What are the significances of a practical approach to sugarcane
2. Oil extracted from maize can be a feedstock for biodiesel. production known as ‘Sustainable Sugarcane Initiative’?
3. Alcoholic beverages can be produced by using maize. 1. Seed cost is very low in this compared to the
Which of the statements given above is/are correct? conventional method of cultivation.
(a) 1 only (b) 1 and 2 only 2. Drip irrigation can be practiced very effectively in this.
(c) 2 and 3 only (d) 1, 2 and 3 3. There is no application of chemical/inorganic fertilizers
14. Among the following organisms, which one does not at all in this.
belong to the class of other three? 4. The scope for intercropping is more in this compared
(a) Crab (b) Mite to the conventional method of cultivation.
(c) Scorpion (d) Spider Select the correct answer using the code given below.
15. The power to increase the number of judges in the Supreme (a) 1 and 3 only (b) 1, 2 and 4 only
Court of India is vested in (c) 2, 3 and 4 only (d) 1, 2, 3 and 4
(a) the President of India 22. If a wetland of international importance is brought under
(b) the Parliament the ‘Montreux Record’, what does it imply?
(c) the Chief Justice of India (a) Changes in ecological character have occurred, are
(d) the Law Commission occurring or are likely to occur in the wetland as a result
16. Consider the following towns of India: of human interference
1. Bhadrachalam 2. Chanderi (b) The country in which the wetland is located should
3. Kancheepuram 4. Karnal enact a law to prohibit any human activity within five
kilometres from the edge of the wetland
(c) The survival of the wetland depends on the cultural Which of the statements given above is/are correct?
practices and traditions of certain communities living (a) 1 only (b) 2 and 3 only
in its vicinity and therefore the cultural diversity therein (c) 1, 2 and 3 (d) None
should not be destroyed 28 Consider the following languages :
(d) It is given the status of ‘World Heritage Site’ 1. Gujarati
23. Which one of the following pairs of islands is separated 2. Kannada
from each other by the ‘Ten Degree Channel’?
3. Telugu
(a) Andaman and Nicobar
Which of the above has/have been declared as ‘Classical
(b) Nicobar and Sumatra
Language/ Languages’ by the Government?
(c) Maldives and Lakshadweep
(d) Sumatra and Java (a) 1 and 2 only (b) 3 only
24. Consider the following pairs : (c) 2 and 3 only (d) 1, 2 and 3
Programme/Project Ministry 29. Consider the following pairs :
1 . Drought-Prone Area : Ministry of Agriculture 1. Dampa Tiger : Mizoram
Programme Reserve
2. Desert Development : Ministry of Environment 2. Gumti Wildlife : Sikkim
Programme and Forests Sanctuary
3. National Watershed : Ministry of Rural 3. Saramati Peak : Nagaland
Development Project Development Which of the above pairs is/are correctly matched?
for Rainfed Areas (a) 1 only (b) 2 and 3 only
Which of the above pairs is/ are correctly matched? (c) 1 and 3 only (d) 1, 2 and 3
(a) 1 and 2 only (b) 3 only 30. With reference to a conservation organization called “Wetlands
(c) 1, 2 and 3 (d) None International’, which of the following statements is/ are
25. With reference to Bombay Natural History Society (BNHS), correct?
consider the following statements :
1. It is an intergovernmental organization formed by the
1. It is an autonomous organization under the Ministry of
countries which are signatories to Ramsar Convention.
Environment and Forests.
2. It works at the field level to develop and mobilize
2. It strives to conserve nature through action-based research,
knowledge, and use the practical experience to
education and public awareness.
advocate for better policies.
3. It organizes and conducts nature trails and camps for
Select the correct answer using the code given below.
the general public.
Which of the statements given above is/are correct? (a) 1 only (b) 2 only
(a) 1 and 3 only (b) 2 only (c) Both 1 and 2 (d) Neither 1 nor 2
(c) 2 and 3 only (d) 1, 2 and 3 31. With reference to a grouping of countries known as BRICS,
26. With reference to ‘Global Environment Facility'’, which of consider the following statements :
the following statements is/are correct? 1. The First Summit of BRICS was held in Rio de Janeiro
(a) It serves as financial mechanism for ‘Convention on in 2009.
Biological Diversity’ and ‘United Nations Framework 2. South Africa was the last to join the BRICS grouping.
Convention on Climate Change’ Which of the statements given above is/are correct?
(b) It undertakes scientific research on environmental (a) 1 only (b) 2 only
issues at global level (c) Both 1 and 2 (d) Neither 1 nor 2
(c) It is an agency under OECD to facilitate the transfer of 32. Consider the following diseases :
technology and funds to underdeveloped countries 1 . Diphtheria 2. Chickenpox
with specific aim to protect their environment
3. Smallpox
(d) Both (a) and (b)
Which of the above diseases has/have been eradicated in
27. With reference to technologies for solar power production,
India?
consider the following statements :
(a) 1 and 2 only (b) 3 only
1. ‘Photovoltaics’ is a technology that generates
electricity by direct conversion of light into electricity, (c) 1, 2 and 3 (d) None
while ‘Solar Thermal’ is a technology that utilizes the 33. Which of the following phenomena might have influenced
Sun’s rays to generate heat which is further used in the evolution of organisms?
electricity generation process. 1. Continental drift
2. Photovoltaics generates Alternating Current (AC), 2. Glacial cycles
while Solar Thermal generates Direct Current (DC). Select the correct answer using the code given below.
3. India has manufacturing base for Solar Thermal (a) 1 only (b) 2 only
technology, but not for Photovoltaics. (c) Both 1 and 2 (d) Neither 1 nor 2
34. Other than poaching, what are the possible reasons for the Select the correct answer using the code given below.
decline in the population of Ganges River Dolphins? (a) 1 and 2 (b) 2 only
1. Construction of dams and barrages on rivers (c) 2 and 3 (d) 3 only
2. Increase in the population of crocodiles in rivers 41. The Partition of Bengal made by Lord Curzon in 1905 lasted
3. Getting trapped in fishing nets accidentally until
4. Use of synthetic fertilizers and other agricultural (a) the First World War when Indian troops were needed
chemicals in crop-fields in the vicinity of rivers by the British and the partition was ended
Select the correct answer using the code given below. (b) King George V abrogated Curzon’s Act at the Royal
(a) 1 and 2 only (b) 2 and 3 only Durbar in Delhi in 1911
(c) 1, 3 and 4 only (d) 1, 2, 3 and 4 (c) Gandhiji launched his Civil Disobedience Movement
35. The Radcliffe Committee was appointed to (d) the Partition of India in 1947 when East Bengal became
(a) solve the problem of minorities in India East Pakistan
(b) give effect to the Independence Bill 42. The 1929 Session of Indian National Congress is of
(c) delimit the boundaries between India and Pakistan significance in the history of the Freedom Movement because
(d) enquire into the riots in East Bengal the
36. Brominated flame retardants are used in many household (a) attainment of Self-Government was declared as the
products like mattresses and upholstery. Why is there objective of the Congress
some concern about their use? (b) attainment of Poorna Swaraj was adopted as the goal
1. They are highly resistant to degradation in the of the Congress
environment. (c) Non-Cooperation Movement was launched
2. They are able to accumulate in humans and animals. (d) decision to participate in the Round Table Conference
Select the correct answer using the code given below. in London was taken
(a) 1 only (b) 2 only 43. With reference to the famous Sattriya dance, consider the
(c) Both 1 and 2 (d) Neither 1 nor 2 following statements :
37. Consider the following : 1. Sattriya is a combination of music, dance and drama.
1. Bats 2. It is a centuries-old living tradition of Vaishnavites
2. Bears of Assam.
3. Rodents 3. It is based on classical Ragas and Talas of devotional
The phenomenon of hibernation can be observed in which songs composed by Tulsidas, Kabir and Mirabai.
of the above kinds of animals? Which of the statements given above is/ are correct?
(a) 1 and 2 only (a) 1 only (b) 1 and 2 only
(b) 2 only (c) 2 and 3 only (d) 1, 2 and 3
(c) 1, 2 and 3 44. Chaitra 1 of the national calendar based on the Saka Era
(d) Hibernation cannot be observed in any of the above corresponds to which one of the following dates of the
38. Which one of the following is the largest Committee of Gregorian calendar in a normal year of 365 days?
the Parliament? (a) 22nd March (or 21st March)
(a) The Committee on Public Accounts (b) 15th May (or 16th May)
(b) The Committee on Estimates (c) 31st March (or 30th March)
(c) The Committee on Public Undertakings (d) 21st April (or 20th April)
(d) The Committee on Petitions 45. With reference to the Indian history of art and culture, consider
39. Which of the following adds/add carbon dioxide to the the following pairs :
carbon cycle on the planet Earth? Famous work of Sculpture Site
1. Volcanic action 1. A grand image of Buddha's : Ajanta
2. Respiration Mahaparinirvana with
3. Photosynthesis numerous celestial musicians
4. Decay of organic matter above and the sorrowful figures
Select the correct answer using the code given below. of his followers below
(a) 1 and 3 only (b) 2 only 2. A huge image of Varaha : Mount Abu
(c) 1, 2 and 4 only (d) 1, 2, 3 and 4 Avatar (boar incarnation)
40. If you walk through countryside, you are likely to see some of Vishnu, as he rescues
birds stalking alongside the cattle to seize the insects Goddess Earth from the deep
disturbed by their movement through grasses. and chaotic waters, sculpted
Which of the following is/are such bird/birds? on rock
1. Painted Stork 3. “Arjuna’s Penance”/ : Mamallapuram
2. Common Myna “Descent of Ganga” sculpted
3. Black-necked Crane on the surface of huge boulders
Which of the pairs given above is/are correctly matched? Select the correct answer using the code given below.
(a) 1 and 2 only (b) 3 only (a) 1 and 2 only (b) 2 only
(c) 1 and 3 only (d) 1, 2 and 3 (c) 1 and 3 only (d) 1, 2 and 3
46. The Ghadr (Ghadar) was a 53. Ibadat Khana at Fatehpur Sikri was
(a) revolutionary association of Indians with headquarters (a) the mosque for the use of Royal Family
at San Francisco (b) Akbar’s private prayer chamber
(b) nationalist organization operating from Singapore (c) the hall in which Akbar held discussions with scholars
(c) militant organization with headquarters at Berlin of various religions
(d) communist movement for India's freedom with head
(d) the room in which the nobles belonging to different
quarters at Tashkent
religions gathered to discuss religious affairs
47. With reference to India's culture and tradition, what is
54. In the context of food and nutritional security of India,
'Kalaripayattu?
enhancing the ‘Seed Replacement Rates’ of various crops
(a) It is an ancient Bhakti cult of Shaivism still prevalent in
helps in achieving the food production targets of the future.
some parts of South India
(b) It is an ancient style bronze and brasswork still found But what is/ are the constraint/ constraints in its wider/
in southern part of Coromandel area greater implementation?
(c) It is an ancient form of dance-drama and a living 1. There is no National Seeds Policy in place.
tradition in the northern part of Malabar 2. There is no participation of private sector seed
(d) It is an ancient martial art and a living tradition in some companies in the supply of quality seeds of vegetables
parts of South India and planting materials of horticultural crops.
48. Consider the following pairs : 3. There is a demand-supply gap regarding quality seeds
1. Garba Gujarat in case of low value and high volume crops.
2. Mohiniattam Odisha Select the correct answer using the code given below.
3. Yakshagana Karnataka (a) 1 and 2 (b) 3 only
Which of the pairs given above is/are correctly matched? (c) 2 and 3 (d) None
(a) 1 only (b) 2 and 3 only 55. With reference to ‘Eco-Sensitive Zones’, which of the
(c) 1 and 3 only (d) 1, 2 and 3 only following statements is/are correct?
49. With reference to Buddhist history, tradition and culture in 1. Eco-Sensitive Zones are the areas that are declared
India, consider the following pairs : under the Wildlife (Protection) Act, 1972.
Famous shrine Location 2. The purpose of the declaration of Eco-Sensitive Zones
1. Tabo monastery and : Spiti Valley is to prohibit all kinds of human activities in those zones
temple complex except agriculture.
2. Lhotsava Lhakhang : Zanskar Valley Select the correct answer using the code given below.
temple, Nako (a) 1 only (b) 2 only
3. Alchi temple complex : Ladakh
(c) Both 1 and 2 (d) Neither 1 nor 2
Which of the pairs given above is/are correctly matched?
56. Consider the following statements :
(a) 1 only (b) 2 and 3 only
(c) 1 and 3 only (d) 1, 2 and 3 1. Animal Welfare Board of India is established under
50. Consider the following statements : the Environment (Protection) Act, 1986.
1. ‘Bijak’ is a composition of the teachings of Saint Dadu 2. National Tiger Conservation Authority is a statutory
Dayal. body.
2. The Philosophy of Pushti Marg was propounded by 3. National Ganga River Basin Authority is chaired by
Madhvacharya. the Prime Minister.
Which of the statements given above is/are correct? Which of the statements given above is/are correct?
(a) 1 only (b) 2 only (a) 1 only (b) 2 and 3 only
(c) Both 1 and 2 (d) Neither 1 nor 2 (c) 2 only (d) 1, 2 and 3
51. A community of people called Manganiyars is well-known 57. Consider the following pairs :
for their Vitamin Deficiency disease
(a) martial arts in North-East India 1. Vitamin C : Scurvy
(b) musical tradition in North-West India 2. Vitamin D : Rickets
(c) classical vocal music in South India 3. Vitamin E : Night blindness
(d) pietra dura tradition in Central India Which of the pairs given above is/ are correctly matched?
52. What was/were the object/objects of Queen Victoria’s (a) 1 and 2 only (b) 3 only
Proclamation (1858)?
(c) 1, 2 and 3 (d) None
1. To disclaim any intention to annex Indian States
58. There is some concern regarding the nanoparticles of
2. To place the Indian administration under the British
some chemical elements that are used by the industry in
Crown
3. To regulate East India Company’s trade with India the manufacture of various products. Why?
1. They can accumulate in the environment, and 2. Keoladeo Ghana : Confluence of
contaminate water and soil. National Park Banas and Chambal
2. They can enter the food chains. 3. Kolleru Lake : Confluence of Musi and
3. They can trigger the production of free radicals. Krishna
Select the correct answer using the code given below. Which of the above pairs is/ are correctly matched?
(a) 1 and 2 only (b) 3 only (a) 1 only (b) 2 and 3 only
(c) 1 and 3 only (d) 1, 2 and 3 (c) 1 and 3 only (d) 1, 2 and 3
59. Which of the following organizations brings out the 67. Which one of the following pairs does not form part of the
publication known as ‘World Economic Outlook’? six systems of Indian Philosophy?
(a) The International Monetary Fund (a) Mimamsa and Vedanta (b) Nyaya and Vaisheshika
(b) The United Nations Development Programme (c) Lokayata and Kapalika (d) Sankhya and Yoga
(c) The World Economic Forum 68. Consider the following pairs :
(d) The World Bank Hills Region
60. With reference to Union Budget, which of the following 1. Cardamom Hills : Coromandel Coast
is/are covered under Non-Plan Expenditure? 2. Kaimur Hills : Konkan Coast
1. Defence expenditure 3. Mahadeo Hills : Central India
2. Interest payments 4. Mikir Hills : North-East India
3. Salaries and pensions
Which of the above pairs are correctly matched?
4. Subsidies
(a) 1 and 2 (b) 2 and 3
Select the correct answer using the code given below.
(c) 3 and 4 (d) 2 and 4
(a) 1 only (b) 2 and 3 only
69. Which one of the following Schedules of the Constitution
(c) 1, 2, 3 and 4 (d) None
of India contains provisions regarding anti-defection?
61. Which of the following have coral reefs?
1. Andaman and Nicobar Islands (a) Second Schedule (b) Fifth Schedule
2. Gulf of Kachchh (c) Eighth Schedule (d) Tenth Schedule
3. Gulf of Mannar 70. The most important strategy for the conservation of
4. Sunderbans biodiversity together with traditional human life is the
Select the correct answer using the code given below. establishment of
(a) 1, 2 and 3 only (b) 2 and 4 only (a) biosphere reserves (b) botanical gardens
(c) 1 and 3 only (d) 1, 2, 3 and 4 (c) national parks (d) wildlife sanctuaries
62. In India, the problem of soil erosion is associated with which 71. Turkey is located between
of the following? (a) Black Sea and Caspian Sea
1. Terrace cultivation (b) Black Sea and Mediterranean Sea
2. Deforestation (c) Gulf of Suez and Mediterranean Sea
3. Tropical climate (d) Gulf of Aqaba and Dead Sea
Select the correct answer using the code given below. 72. What is the correct sequence of occurrence of the following
(a) 1 and 2 only (b) 2 only cities in South-East Asia as one proceeds from south to
(c) 1 and 3 only (d) 1, 2 and 3 north?
63. The seasonal reversal of winds is the typical characteristic 1. Bangkok
of 2. Hanoi
(a) Equatorial climate (b) Mediterranean climate 3. Jakarta
(c) Monsoon climate (d) All of the above climates 4. Singapore
64. With reference to the cultural history of India, the term Select the correct answer using the code given below.
‘Panchayatan’ refers to (a) 4-2-1-3 (b) 3-2-4-1
(a) an assembly of village elders (c) 3-4-1-2 (d) 4-3-2-1
(b) a religious sect 73. The scientific view is that the increase in global
(c) a style of temple construction temperature should not exceed 2 °C above pre-industrial
(d) an administrative functionary level. If the global temperature increases beyond 3 °C above
65. Consider the following rivers : the pre-industrial level, what can be its possible impact/
1. Barak impacts on the world?
2. Lohit 1. Terrestrial biosphere tends toward a net carbon
3. Subansiri source.
Which of the above flows/flow through Arunachal 2. Widespread coral mortality will occur.
Pradesh? 3. All the global wetlands will permanently disappear.
(a) 1 only (b) 2 and 3 only 4. Cultivation of cereals will not be possible anywhere
(c) 1 and 3 only (d) 1, 2 and 3 in the world.
65. Consider the following pairs :
Select the correct answer using the code given below.
Wetlands Confluence of rivers
(a) 1 only (b) 1 and 2 only
1. Harike Wetlands : Confluence of Beas and
(c) 2, 3 and 4 only (d) 1, 2, 3 and 4
Satluj/Sutlej
74. The national motto of India, ‘Satyameva Jayate’ inscribed 81. In medieval India, the designations ‘Mahattara’ and ‘Pattakila’
below the Emblem of India is taken from were used for
(a) Katha Upanishad (a) military officers
(b) Chandogya Upanishad (b) village headmen
(c) Aitareya Upanishad (c) specialists in Vedic rituals
(d) Mundaka Upanishad (d) chiefs of craft guilds
75. In the Constitution of India, promotion of international 82. Lichens, which are capable of initiating ecological
peace and security is included in the succession even on a bare rock, are actually a symbiotic
(a) Preamble to the Constitution association of
(p) Directive Principles of State Policy (a) algae and bacteria (b) algae and fungi
(c) Fundamental Duties (c) bacteria and fungi (d) fungi and mosses
(d) Ninth Schedule 83. If you travel through the Himalayas, you are likely to see
76. What are the benefits of implementing the ‘Integrated which of the following plants naturally growing there?
Watershed Development Programme’?
1. Oak 2. Rhododendron
1. Prevention of soil runoff
3. Sandalwood
2. Linking the country’s perennial rivers with seasonal rivers
Select the correct answer using the code given below.
3. Rainwater harvesting and recharge of groundwater table
4. Regeneration of natural vegetation (a) 1 and 2 only (b) 3 only
Select the correct answer using the code given below. (c) 1 and 3 only (d) 1, 2 and 3
(a) 1 and 2 only (b) 2, 3 and 4 only 84. Which of the following are some important pollutants
(c) 1, 3 and 4 only (d) 1, 2, 3 and 4 released by steel industry in India?
77. Which of the following are associated with ‘Planning’ in 1. Oxides of sulphur 2. Oxides of nitrogen
India? 3. Carbon monoxide 4. Carbon dioxide
1. The Finance Commission Select the correct answer using the code given below.
2. The National Development Council (a) 1, 3 and 4 only (b) 1and 3 only
3. The Union Ministry of Rural Development (c) 1 and 4 only (d) 1, 2, 3 and 4
4. The Union Ministry of Urban Development 85. Which of the following Kingdoms were associated with the
5. The Parliament life of the Buddha?
Select the correct answer using the code given below. 1. Avanti 2. Gandhara
(a) 1, 2 and 5 only (b) 1, 3 and 4 only 3. Kosala 4. Magadha
(c) 2 and 5 only (d) 1, 2, 3, 4 and 5 Select the correct answer using the code given below.
78 Which of the following is/are the function/functions of the (a) 1, 2 and 3 (b) 2 and 4.
Cabinet Secretariat? (c) 3 and 4 only (d) 1, 3 and 4
1. Preparation of agenda for Cabinet Meetings 86. Every year, a monthlong ecologically important campaign/
2. Secretarial assistance to Cabinet Committees festival is held during which certain communities/ tribes plant
3. Allocation of financial resources to the Ministries saplings of fruit-bearing trees. Which of the following are
Select the correct answer using the code given below. such communities/tribes?
(a) 1 only (b) 2 and 3 only (a) Bhutia and Lepcha (b) Gond and Korku
(c) 1 and 2 only (d) 1, 2 and 3 (c) Irula and Toda (d) Sahariya and Agariya
79. Consider the following statements : 87. The sales tax you pay while purchasing a toothpaste is a
A Constitutional Government is one which (a) tax imposed by the Central Government
1. places effective restrictions on individual liberty in (b) tax imposed by the Central Government but collected
the interest of State Authority
by the State Government
2. places effective restrictions on the Authority of the
(c) tax imposed by the State Government but collected
State in the interest of individual liberty
by the Central Government
Which of the statements given above is/are correct?
(a) 1 only (b) 2 only (d) tax imposed and collected by the State Government
(c) Both 1 and 2 (d) Neither 1 nor 2 88. What does venture capital mean?
80. Which of the following are the discretionary powers (a) A short-term capital provided to industries
given to the Governor of a State? (b) A long-term start-up capital provided to new
1. Sending a report to the President of India for imposing entrepreneurs
the President’s rule (c) Funds provided to industries at times of incurring losses
2. Appointing the Ministers (d) Funds provided for replacement and renovation of
3. Reserving certain bills passed by the State Legislature industries
for consideration of the President of India 89. The main objective of the 12th Five-Year Plan is
4. Making the rules to conduct the business of the (a) inclusive growth and poverty reduction
State Government (b) inclusive and sustainable growth
Select the correct answer using the code given below. (c) sustainable and inclusive growth to reduce
(a) 1 and 2 only (b) 1 and 3 only unemployment
(c) 2, 3 and 4 only (d) 1, 2, 3 and 4 (d) faster, sustainable and more inclusive growth
90. With reference to Balance of Payments, which of the 96. Consider the following pairs :
following constitutes/constitute the Current Account? Region often in news Country
1. Balance of trade 1. Chechnya : Russian Federation
2. Foreign assets 2. Darfur : Mali
3. Balance of invisibles 3. Swat Valley : Iraq
4. Special Drawing Rights Which of the above pairs is/are correctly matched?
Select the correct answer using the code given below. (a) 1 only (b) 2 and 3 only
(a) 1 only (b) 2 and 3 (c) 1 and 3 only (d) 1, 2 and 3
(c) 1 and 3 (d) 1, 2 and 4 97. With reference to Agni-IV Missile, which of the following
91. The terms ‘Marginal Standing Facility Rate’ and ‘Net Demand statements is/are correct?
and Time Liabilities’, sometimes appearing in news, are used 1. It is a surface-to-surface missile.
in relation to 2. It is fuelled by liquid propellant only
(a) banking operations 3. It can deliver one-tonne nuclear warheads about 7500
(b) communication networking km away
(c) military strategies Select the correct answer using the code given below.
(d) supply and demand of agricul tural products (a) 1 only (b) 2 and 3 only
92. What is/are the facility/facilities the beneficiaries can get (c) 1 and 3 only (d) 1, 2 and 3
from the services of Business Correspondent (Bank Saathi) 98. With reference to two non-conventional energy sources called
in branchless areas? ‘coalbed methane’ and ‘shale gas’, consider the following
1. It enables the beneficiaries to draw their subsidies statements :
and social security benefits in their villages. 1. Coalbed methane is the pure methane gas extracted
2. It enables the beneficiaries in the rural areas to make from coal seams, while shale gas is a mixture of
deposits and withdrawals. propane and butane only that can be extracted from
Select the correct answer using the code given below. fine-grained sedimentary rocks.
(a) 1 only (b) 2 only 2. In India, abundant coalbed methane sources exist, but
(c) Both 1 and 2 (d) Neither 1 nor 2 so far no shale gas sources have been found.
93. In the context of Indian economy, which of the following is/are Which of the statements given above is/are correct?
the purpose/purposes of ‘Statutory Reserve Requirements’? (a) 1 only (b) 2 only
1. To enable the Central Bank to control the amount of (c) Both 1 and 2 (d) Neither 1 nor 2
advances the banks can create 99. With reference to ‘Changpa’ community of India, consider
2. To make the people’s deposits with banks safe and liquid the following statements :
3. To prevent the commercial banks from making 1. They live mainly in the State of Uttarakhand.
excessive profits 2. They rear the Pashmina goats that yield a fine wool.
4. To force the banks to have sufficient vault cash to 3. They are kept in the category of Scheduled Tribes.
meet their day-to-day requirements Which of the statements given above is/are correct?
Select the correct answer using the code given below. (a) 1 only (b) 2 and 3 only
(a) 1 only (b) 1 and 2 only (c) 3 only (d) 1, 2 and 3
(c) 2 and 3 only (d) 1, 2, 3 and 4 100. In India, cluster bean (Guar) is traditionally used as a
94. Recently, a series of uprisings of people referred to as vegetable or animal feed, but recently the cultivation of this
‘Arab Spring’ originally started from has assumed significance. Which one of the following
(a) Egypt (b) Lebanon statements is correct in this context?
(c) Syria (d) Tunisia (a) The oil extracted from seeds is used in the manufacture
95. Consider the following countries : of biodegradable plastics
1. Denmark (b) The gum made from its seeds is used in the extraction
2. Japan of shale gas
3. Russian Federation (c) The leaf extract of this plant has the properties of
4. United Kingdom antihistamines
5. United States of America (d) It is a source of high quality biodiesel
Which of the above are the members of the ‘Arctic Council’?
(a) 1, 2 and 3 (b) 2, 3 and 4
(c) 1, 4 and 5 (d) 1, 3 and 5
ANSWERS AND EXPLANATIONS
1. (c) When interest rates decreases then investment final answer B only 2 and 3. MESSENGER is a robotic
expenditure by businesses on capital goods like NASA spacecraft orbiting the planet Mercury. Voyager
factories and equipment increase in an economy. 1 (September 1977) and voyager 2 (Aug 1977) were
2. (a) Clause (3) of Article 77 “Conduct of Business of the launched to study the outer Solar System.
Government of India” of the Constitution of India lays 9. (d) Kinnaur (Himachal) is famous for apples. Areca nut
down as follows: “The President shall make rules for mostly confined to Karnataka, Kerala and Assam.
the more convenient transaction of the business of the Mewat in haryana is not famous for mango. UP is
Government of India, and for the allocation among famous for mango.
Ministers of the said business. Similarly, coromandel coast is not famous for soya
The Constitution of India mentions that All executive beans.Top two producers of soya are Madhya Pradesh
action of the Government of India shall be expressed to and Maharashtra which are outside coromandel coast.
be taken in the name of the President. Therefore, only Therefore, (d) is the correct option.
option (a) is correct. 10. (b) Chemical changes occur when a substance combines
3. (c) Article 75 says that the Council of Ministers shall be with another to form a new substance. Crystallization
collectively responsible to Loksabha. In other words, of sodium chloride is not a chemical as water of
Lok Sabha can remove them by passing a no- crystallization can be lost to get salt again . Similarly
confidence motion. But the word “no confidence melting of ice is reversible .However souring of milk is
motion” itself is not given in Constitution itself, it comes a chemical change as it is not reversible and a new
from Rule 198 of Lok Sabha Rules. Rajya Sabha cannot compound is formed.
pass or remove council of ministers by passing no- 11. (c) It is under original jurisdiction the supreme court
confidence motion. No confidence motion can be decides the disputes between centre and one or more
introduced, only in Lok Sabha. states.
4. (d) All 1,2 and 3 are true. 12. (d) Transgenic crops include transfer of DNA between two
5. (b) In photosynthesis food is oxidized to form energy , unrelated species. All the above methods does not
oxygen and water. Thus the free energy is converted involve transfer DNA.
into potential energy and stored . 13. (d) All three statements are correct as Maize can be used
6. (d) Biometrics identification is used in computer science for the production of starch.Oil extracted from maize
as a form of identification and access control. Biometric can be a feedstock for biodiesel. Alcoholic beverages
identifiers are the distinctive, measurable characteristics can be produced by using maize.
used to label individuals. Biometric identifiers are 14. (a) Mite, Spider and Scorpion belong to the category of
categorized as physiological versus behavioral Arachnids, while Crabs belong to the category of
characteristics. Physiological Biometric identifiers are crustaceans.
fingerprint, palm veins, face recognition, DNA, palm
15. (b) It is the Parliament which has the power to increase the
print, hand geometry, iris recognition, retina and odour/
number of judges in the Supreme Court of India.
scent. Behavioural identifiers are typing rhythm, gait
Parliament increased the number of judges from the
and voice .
original eight in 1950 to eleven in 1956, fourteen in 1960,
7. (c) Vegetative reproduction is a form of asexual eighteen in 1978, twenty-six in 1986 and thirty-one in
reproduction in plants. It is a process by which new 2008.
organisms arise without production of seeds .It helps
16. (b) Chanderi in Madhyapradesh and Kancheepuram in
in development of clones. Vegetative propagation
Tamil Nadu are famous for Silk sarees.
involves only mitosis, this ensures that the genetic
17. (d) NH 4 is a major National Highway in Western and
information in DNA of vegetative progeny (child) is
Southern India. It connects Mumbai with Chennai. NH
same as in the mother plant and can be practiced
6, runs through Gujarat, Maharashtra, Chhattisgarh,
throughout the year.
Odisha, Jharkhand and West Bengal state in India. It
However it does not helps in elimination of viruses.
connects Surat with Kolkata. NH 15 is a major National
Plant once systematically infected with a virus, usually
Highway in Western and Northwestern India. NH 15
remains infected for its lifetime. Thus any vegetative
connects Samakhiali in Gujarat with Pathankot in
parts taken for propagation remains infected.
Punjab.
8. (b) Cassini–Huygens is an unmanned spacecraft sent to
18. (d) The International Treaty on Plant Genetic Resources
the planet Saturn. Therefore, option 1 is wrong, this
for Food and Agriculture aims at guaranteeing food
eliminates option (a), (c) and (d). Now we are left with
security through the conservation, exchange and
sustainable use of the world’s plant genetic resources environmental issues at global level. It is not an agency
for food and agriculture. The United Nations under OECD.
Convention to Combat Desertification is a Convention 27. (a) Photovoltaics' is a technology that generates electricity
to combat desertification and mitigate the effects of by direct conversion of light into electricity, while 'Solar
drought. The World Heritage Convention is concerned Thermal' is a technology that utilizes the Sun's rays to
with the protection of the world cultural and natural generate heat which is further used in electricity
heritage.All three of them have a bearing on the generation process. Both Photovoltaic cells and solar
biodiversity. thermal generate direct current (DC). India has a
19. (c) Earth Hour is a worldwide movement for the planet manufacturing base for both.
organized by the World Wide Fund for Nature (WWF). 28. (c) The languages declared as 'Classical Language/
The event is held worldwide annually encouraging Languages' by the Government are
individuals, communities, households and businesses 1. Sanskrit, 2. Tamil, 3. Telugu, 4. Kannada, 5. Malayalam,
to turn off their non-essential lights for one hour, from 6. Odia.
8:30 to 9:30 p.m. to raise the awareness about the climate 29. (c) Dampa Tiger Reserve, the largest wildlife sanctuary in
change and the need to save the planet Mizoram. Saramati peak is in Nagaland. It is located
20. (a) Diatoms are autotrophs which prepare their own food. near Tuensang town with a height of 3,826 m. Gumti
Crustaceans are herbivorous animals which feed on Wildlife Sanctuary is famous wildlife reserve in Tripura.
diatoms. Herrings are carnivorous animals which feed 30. (b) This organization is in partnership but not formed by
on Crustaceans. This completes the food chain. Ramsar convention. The organization was formed in
21. (b) NPK (fertilizers) can be applied at the rate of 112 kg, 25 1954 and Ramsar convention was signed in 1971.
kg and 48 kg per acre, respectively through inorganic “Wetlands International”, It is an intergovernmental
or organic methods. Hence statement #3 is wrong, this organization formed by the countries which are
eliminates (a), (c) and (d). therefore, answer (b) 1,2,4 signatories to Ramsar Convention.It works at the field
only. level to develop and mobilize knowledge, and use the
22. (a) The Montreux Record is a register of wetland sites on practical experience to advocate for better policies.
the List of Wetlands of International Importance where 31. (b) BRICS is the acronym for an association of five major
changes in ecological character have occurred, are emerging national economies: Brazil, Russia, India,
occurring, or are likely to occur as a result of China, and South Africa. The grouping was originally
technological developments, pollution or other human known as “BRIC” before the inclusion of South Africa
interference. in 2010. The BRIC first formal summit held in
23. (a) The Ten Degree Channel is a channel that separates Yekaterinburg, commenced on 16 June 2009.
the Andaman Islands from the Nicobar Islands in the 32. (b)
Bay of Bengal. 33. (c) Continental Drift and Glacial Cycles have influenced
24. (d) The draught prone area programme is by ministry of the evolution of organisms. Continental drift is the
Rural development to minimise the adverse effects of movement of the Earth's continents relative to each
drought on production of crops and livestock and other by appearing to drift across the ocean bed. A
productivity of land, water and human resources. The glacial period is an interval of time within an ice age
desert development programme is also by ministry of that is marked by glacier advances.
Rural development to minimise the adverse effect of 34. (c) The Radcliffe Line is a boundary demarcation line
drought and control desertification. National between India and Pakistan upon the Partition of India.
Watershed Development Project for Rainfed Areas is The Radcliffe Line was named after its architect, Sir
by ministry of agriculture. Cyril Radcliffe.
25. (c) The Bombay Natural History Society is one of the 35. (c) Statement 1, 3 and 4 are correct as per WWF and
largest non-governmental organizations in India Environment Ministry.
engaged in conservation and biodiversity research. It
36. (c) Brominated flame retardants used in many household
strives to conserve nature through action-based
products are highly resistant to degradation in the
research, education and publicawareness. It organizes
environment and they are able to accumulate in humans
and conducts nature trails and camps for the general
and animals.
public.
37. (c) Hibernation is a state of inactivity and metabolic
26. (a) The Global Environment Facility (GEF) unites 183
depression in endothermsis characterized by low body
countries in partnership with international institutions,
temperature, slow breathing and heart rate, and low
civil society organizations (CSOs), and the private
metabolic rate.It is found in Bat ,bear and rodents etc
sector to address global environmental issues. It serves
38. (b) The largest Committee is Estimates, given its 30
as financial mechanism for 'Convention on Biological
members
Diversity' and 'United Nations Framework Convention
on Climate Change' It undertakes scientific research on
Committee on No. of members 49. (c) Tabo monastery and temple complex is in Spiti Valley
and Alchi temple complex is in Ladakh. Lhotsava
Pub lic A ccou nts 22
Lhakhang, Nako is in Himachal Pradesh . Zanskar Valley
Es timates 30 is in Jammu and Kashmir.
Pub lic Un dertakin gs 22 50. (d) Statement 1 is wrong because Kabir founded a
Petition s LS(15) , RS(10) community – Kabirpnathi, who consider Bijak, a
39. (c) Volcanic action, Respiration and decay of organic collection of his verses, as their scripture.Statement 2
matter adds/add carbon dioxide to the carbon cycle on is wrong because Vallabhacharya’s philosophy came
the planet Earth. However, photosynthesis uses Carbon to be known as Pushtimarga (the path of grace).
di oxide to produce food. 51. (b) Manganiyars–a tribal community from Rajasthan (North
40. (b) Common Myna are birds stalking alongside the cattle west) with a strong musical tradition.
to seize the insects disturbed by their movement 52. (a) The object/objects of Queen Victoria's Proclamation
through grasses. The common myna is readily (1858) were to disclaim any intention to annex Indian
identified by the brown body, black hooded head and States as the announcement reversed Lord Dalhousie's
the bare yellow patch behind the eye. pre-war policy of political unification through princely
41. (b) In 1911 King George V visited India. A durbarwas held state annexation.It was also to place the Indian
at Delhi and The capital of India was transferred from administration under the British Crown Therefore,
Calcutta to Delhi also Partition of Bengal was annulled. statement 1 & 2 are correct. However it was not to
regulate East India Company's trade with India so
42. (b) The 1929 Session of Indian, National Congress is of
statement 3 is wrong.
significance in the history of the Freedom Movement
because the attainment of Poorna Swaraj was adopted 53. (c) Ibadat Khana at Fatehpur Sikri was the hall in which
as the goal of the Congress. Akbar held discussions with scholars of various
religions on every Thursday to debate on religious
43. (d) Sattriya dance form was introduced in the 15th century
issues raised by Akbar.
A.D by the great Vaishnava saint and reformer of
Assam, Mahapurusha Sankaradeva as a powerful 54. (b) We have a National Seed policy made in 2002. So 1 is
medium for propagation of the Vaishnava faith. It is a not a constraint. The private sector produces high-
neo-Vaishnava treasure of Assamese music,dance and priced seeds but in lower volume. It supplies nearly the
drama .Music of Sattriya dance comprises of classical entire hybrid seeds required for vegetables. So 2 is not
ragas (melodies), talas (rhythms) and traditional songs. a constraint. In the case of low value and high volume
Tulsidas ,Kabir and Mira were Vaishnavite. crops such as wheat and rice, farmers tend to use their
own preserved seeds as there is demand and supply
44. (a) Chaitra 1 of the national calendar based on the Saka
gap. So 3 is a constraint.
Era corresponds to 22nd March of the Gregorian
calendar in a normal year of 355 days and on 21 March 55. (d) Eco-sensitive zones are the ecologically important areas
in leap year. designated to be protected from industrial pollution
and unregulated development under the Environment
45. (c) 1 is correctly matched because Parinirvana of the
Protection Act of 1986. Only environmentally hazardous
Buddha in Cave 17 of Ajanta, with numerous celestial
human activities are prohibited in those areas.
musicians above and the sorrowful figures of his
Therefore, both the statements are incorrect.
followers below, is one of the grandest and yet most
delicately expressive scenes ever made in stone. 2 is 56. (b) The Animal Welfare Board of India was established in
wrong because huge image of Varaha Avatar ofVishnu, 1962 under Section 4 of The Prevention of Cruelty to
as herescues Goddess Earth from the deepand chaotic Animals Act, 1960. The National Tiger Conservation
waters, sculpted on rock is found in Mamallapuram. 3 Authority is set up under the Chairmanship of the
is right because Arjuna's Penance “Descent of Ganga” Minister for Environment and Forests and is a statutory
sculpted on thesurface of huge boulders is found in body. National Ganga River Basin Authority was
Mamallapuram. established by the Central Government of India, on 20
February 2009.The Prime Minister is the chair of the
46. (a) The Ghadar Party was a revolutionary association
Authority.
founded by Punjabi Indians, in the United States and
Canada with the aim to gaining India's independence 57. (a) Deficiency of vitamin C causes Scurvy, disease of gums.
from British rule. Key members included Lala Har Dayal, Deficiency of vitamin D causes Rickets, disease of
Sohan Singh Bhakna, Kartar Singh Sarabha, and bones and Deficiency of Vitamin A causes night
Rashbehari Bose.It had its headquarters at San blindness, disease of eyes.
Francisco. 58. (d)
47. (d) Kalaripayattu is an ancient martial art and a living 59. (a) The World Economic Outlook (WEO) database
tradition in some parts of Kerala/ South India. contains selected macroeconomic data series from the
48. (c) Yakshgana is a theatre form of Karnataka.Mohiniattam statistical appendix of the World Economic Outlook
is from Kerala and Garba is a dance form from Gujarat. report prepared by IMF.
60. (c) Non-plan expenditure covers interest payments, larger part in Western Asia which is located between
subsidies (mainly on food and fertilisers), wage and Black Sea and Mediterranean
salary payments to government employees, grants to 72. (c) The correct sequence of occurrence of the following
States and Union Territories governments, pensions, cities in South-East Asia as one proceeds from south
police, economic services in various sectors, defence, to north is Jakarta-Singapore-Bangkok and Hanoi.
loans to public enterprises, loans to States, Union 73. (b) If the global temperature increases beyond 3°C above
Territories and foreign governments. the pre-industrial level then Terrestrial biosphere tends
61. (a) Coral reefs are diverse underwater ecosystems held toward a net carbon source and Widespread coral
together by calcium carbonate structures secreted by mortality will occur.
corals. Andaman & Nicobar,Gulf of Kachchh and Gulf 74. (d) The words Satyameva Jayate came from Mundaka
of Mannar have coral reefs. However Sunderbans do Upanishad, meaning ‘Truth Alone Triumphs’.
not have coral reef. 75. (b) Promotion of international peace and security is
62. (b) In India, the problem of soil erosion is associated with included in the Directive Principles of State as Article
deforestataion .Terace cultivation helps in less erosion 51 of constitution mentions to promote international
of soil. peace and security and maintain just an honorable
63. (c) Monsoon is the only climate which is having reversal relations between nations between nations; to foster
of wind between seasons. respect for international law and treaty obligations, and
64. (c) Panchayatana is a style of temple construction that to encourage settlements of international disputes by
has a central shrine surrounded by four subsidiary arbitration.
shrines. 76. (c) Integrated Watershed Management Programme
65. (b) Rivers Lohit and Subansiri flow through Arunachal (IWMP) was launched during 2009-10.It aims at
Pradesh. River Barak flows in south Assam. Prevention of soil runoff, Rainwater harvesting and
66. (a) Harike Wetlands is at Confluence of Beas and Satluj/ recharge of groundwater table and Regeneration of
Sutlej. The Keoladeo National Park formerly known as natural vegetation.
the Bharatpur Bird Sanctuary in Bharatpur is at the 77. (c) Finance commission is involved in distribution of taxes
confluence of two rivers, the Gambhir and Banganga. and grants. It is not involved in planning. So by just
Kolleru Lake is one of the largest freshwater lakes in eliminating all options involving statement 1, we get
India located in state of Andhra Pradesh. Kolleru is answer (c ) correct.
located between Krishna and Godavari delta. 78. (c) The functions of the Cabinet Secretariat are preparation
67. (c) Lokayata and Kapalika do not form of Six system of of agenda for Cabinet Meetings & Secretarial assistance
Indian philosophy to Cabinet Committees. However Allocation of financial
resources to the Ministries as per the provisions in
S ix s ys tem of Indian philos ophy are budget is prepared by finance ministry.
Samkhy a Prakriti an d Pu ru s h 79. (c) A constitutional Government needs to balance between
Yo ga Releas in g Pu ru s h from Parkriti individual liberty viz a viz State Authority.
Nyay a Log ical th inking 80. (b) The governor has Constitutional discretion in cases of
Vais hes hika Realis tic and o b jective ph ilo s o ph y Reservation of bill for consideration of the President
o f u nivers e and Recommendation of the imposition of President's
M imams a A nalyzin g Samhita and Brahmana rule.Therefore, statement 1 and 3 are definitely right.
p ortio n of Ved a. Moreover Governor only appoints those persons as
ministers who are recommended by the Chief Minister.
68. (c) The Cardamom Hills are southern hills of India and part So Governor doesn’t have “Discretion” in appointment
of the southern Western Ghats located in southeast of the minister.Therefore 2 is wrong .
Kerala and southwest Tamil Nadu. They are not in 81. (b) In medieval India, the designations ‘Mahattara’ and
coromandel coast. Kaimur Range is the eastern portion ‘Pattakila’ were used for village headmen.
of the Vindhya Range extending from Madhya Pradesh 82. (b) In organisms called lichens, a chlorophy ll-containing
to Bihar. They are not in konkan coast. The Mahadeo partner, which is an alga, and a fungus live together
Hills are in Madhya Pradesh, state of central India.Mikir fungus provides shelter, water and minerals to the alga
hills are in assam i.e. in North East India. and, in return, the alga provides food which it prepares
69. (d) The 52nd Amendment act of 1985, added 10th schedule by photosynthesis.
to the Constitution. This is often referred to as anti- 83. (a) Oak and Rhododendron is found in Himalayan region
defection law. while sandalwood is found in south India.
70. (a) Biosphere reserves are established under UNESCO’s 84. (d) In steel furnace coke reacts with iron to release
Man and Biosphere (MAB) Programme. pollutants like Oxides of sulphur, Oxides of nitrogen,
71. (b) Turkey's smaller part is in Southeastern Europe and its Carbon monoxide & Carbon dioxide.
85. (c) Buddh born in Lumbini, in Kosala kingdom.Buddh died 93. (b) Because Reserve requirements are designed as
in Kusinara, in Magadha kingdom. Avanti lay outside “precautionary measures” and not to stop banks from
the area visited by buddha, and was converted to his “excessive” profit. This eliminates (c) and (d) .
teaching by his disciple Mahakaccana. Gandhara is 94. (d) The Arab Spring is a revolutionary wave of
western part of Pakistan & Afghanistan and Buddha demonstrations, protests and civil wars in the Arab
never went to Pakistan. world that began on 18 December 2010 and spread
86. (b) Every year the Gond and Korku tribes celebrate a throughout the countries of the Arab League. It
month long ecologically important campaign/festival originally started from Tunisia.
in which they plant saplings of fruit-bearing trees. 95. (d) The Arctic Council is a high-level intergovernmental
87. (d) Taxes on tooth paste come under GST which is forum that addresses issues faced by the Arctic
administered by State government.Sales tax is paid to governments and the indigenous people of the Arctic.
sales tax authority in the state from where the goods It has eight member countries: Canada, Denmark,
are moved. Finland, Iceland, Norway, Russia, Sweden, and the
88. (b) Venture capital (VC) is a long term financial capital United States.
provided to early-stage, high-potential, growth startup 96. (a) The Chechen Republic is a federal subject (a republic)
companies or new companies. of Russia. Darfur is a region in western Sudan. Swat is
89. (d) 12th Five Year Plan of the Government of India (2012- a valley and an administrative district in Pakistan.
17) aims at faster, sustainable and moreinclusive 97. (a) Agni IV is a surface-to-surface missile. It is fuelled by
growth. solid propellant. It can deliver one-tonne nuclear
90. (c) The current account consists of the balance of trade warheads about 4000 km away.
and balance of invisibles. 98. (d) Shale gas contains methane. First statement says it
91. (a) Marginal Standing Facility rate is the rate at which contains ‘propane and butane only’ is wrong. Shale
banks borrow funds overnight from the Reserve Bank gas resources exist in India. Therefore, statement 2 is
of India (RBI) against approved government securities. also wrong.
Net Demand and time liability is the sum of demand 99. (b) The Changpa are a semi-nomadic Tibetan ethnic group
and time liability of Banks with public and other banks found mainly in Zanskar region of Jammu and Kashmir.
wherein assets with other banks is subtracted to get They rear the Pashmina goats that yield a fine wool.
net liability of other bank. They are kept inthe category of Scheduled Tribes.
92. (c) Besides giving access to banking, it also enables 100. (b) Guar gum is used in hydraulic fracturing technology
government subsidies and social security benefits to during shale gas extraction.
be directly credited to the accounts of the beneficiaries,
enabling them to draw the money from the bank saathi
or business correspondents in their village itself.
v

INSTRUCTIONS
1. This Test Booklet contains 100 items (questions). Each item comprises four responses (answers). You will select
the response which you want to mark on the Answer sheet. In case, you feel that there is more than one correct
response, mark the response which you consider the best. In any case, choose ONLY ONE response for each item.
2. All items carry equal marks.
3. Penalty for wrong answers:
THERE WILL BE PENALTY FOR WRONG ANSWERS MARKED BY THE CANDIDATE FOR WRONG ANSWER.
(i) There are four alternatives for the answer to every question. For each question which has a penalty for which a
wrong answer has been given by the candidate, one-third of the marks assigned to that question will be deducted
as penalty.
(ii) If a candidate gives more than one answer, it will be treated as a wrong answer even if one of the given answers
happens to be correct and there will be same penalty as above to that question.
(iii) If a question is left blank, i.e., no answer is given by the candidate, there will be no penalty for that question.

Directions for the following 7 (seven) items : 2. With reference to "ideological bias", the passage implies
Read the following two passages and answer the items that follow that
each passage. Your answers to these items should be based on (a) free market is fair but not competent.
the passages only. (b) free market is not fair
Passage - 1
11.but competent.
(c) free market is fair and competent.
Many nations now place their faith in capitalism and governments (d) free market is neither fair nor biased.
choose it as the strategy to create wealth for their people. The 3. The exercise of private greed will be in the larger public
spectacular economic growth seen in Brazil, China and India after interest" from the passage
the liberalisation of their economies is proof of its enormous 1. refers to the false ideology of capitalism.
potential and success. However, the global banking crisis and the
2. underlies the righteous claims of the free market.
economic recession have left many bewildered. The debates tend
to focus on free market operations and forces, their efficiency and 3. shows the benevolent face of capitalism.
their ability for self correction. Issues of justice, integrity and 4. ignores resultant gross inequity.
honesty are rarely elaborated to highlight the failure of the global Which of the statements given above is/are correct ?
banking system. The apologists of the system continue to justify (a) 1 only (b) 2 and 3
the success of capitalism and argue that the recent crisis was a (c) 1and 4 (d) 4 only
blip. Passage - 2
Their arguments betray an ideological bias/ with the Net profits are only 2.2% of their total assets for central public
assumptions that an unregulated market is fair and competent, sector undertakings, lower than for the private corporate sector.
and that the exercise of private greed will be in the larger . public While the public sector or the State-led entrepreneurship played
interest. an important role in triggering India's industrialization, our evolving
Few recognize the bidirectional relationship between capitalism development needs, comparatively less-than-satisfactory
and greed; that each reinforces the other. Surely, a more honest performance of the public sector enterprises, the maturing of our
conceptualisation of the conflicts of interest among the rich and private sector, a much larger social base now available for
powerful players who have benefited from the system, their biases expanding entrepreneurship and the growing institutional
and ideology is needed; the focus on the wealth creation should capabilities to enforce competition policies would suggest that
also highlight the resultant gross inequity. the time has come to review the role of public sector.
1. The apologists of the "Free Market System", according to What should the portfolio composition of the government be ?
the passage, believe in It should not remain static all times. The airline industry works
(a) market without control by government authorities. well as a purely private affair. At the opposite end, rural roads,
(b) market without protection by the government. whose sparse traffic makes tolling unviable, have to be on the
(c) ability of market to self correct. balance-sheet of the State. If the government did not own rural
(d) market for free goods and services.
roads, they would not exist. Similarly, public health capital in our 8. A question paper must have a question on one of the eight
towns and cities will need to come from the public sector. Equally, poets :A, B, C, D, E, F, G or H. The first four belong to the
preservation and improvement of forest cover will have to be a medieval period while the rest are considered modern poets.
new priority for the public sector assets. Generally, modern poets figure in the question paper in
Take the example of steel. With near-zero tariffs, India is a alternate years. Generally those who like H like G also; and
globally competitive market for the metal. Indian firms export steel those who like F like E also. The paper-setter does not like to
into the global market, which demonstrates there is no gap in ask about F as he has written a book on F, but he likes F. Last
technology. Indian companies are buying up global steel year, the paper contained a question on A. On the basis of
companies, which shows there is no gap in capital availability. the information given, this year's paper is most likely to
Under these conditions, private ownership works best. contain a question on
(a) C (b) E
Private ownership is clearly desirable in regulated industries,
(c) F (d) H
ranging from finance to infrastructure, where a government agency
9. In a group of six women there are four dancers, four vocal
performs the function of regulation and multiple competing firms
musicians, one actress and three violinists. Girija and Vanaja
are located in the private sector. Here, the simple and clean solution
are among the violinists while Jalaja and Shailaja do not
- government as the umpire and the private sector as the players
know how to play on the violin. Shailaja and Tanuja are
is what works best. Inmany of these industries, we have a legacy
among the dancers. Jalaja, Vanaja, Shailaja and Tanuja are all
of government ownership, where productivity tends to be lower,
vocal musicians and two of them are also violinists. If Pooja
fear of bankruptcy is absent, and the risk of asking for money
is an actress, who among the following is certainly a dancer
from the tax payer is ever present. There is also the conflict of and a violinist ?
interest between government as an owner and as the regulator. (a) Jalaja (b) Pooja
The formulation and implementation of competition policy will be (c) Shailaja (d) Tanuja
more vigorous and fair if government companies are out of action. 10. The letters L, M, N, O, P, Q, R, S and T in their order are
4. According to the passage, what is/are the reason/reasons substituted by nine integers 1 to 9 but not in that order. 4 is
for saying that the time has come to review the role of public assigned to P. The difference between P and T is 5. The
sector ? difference between N and T is 3. What is the integer assigned
1. Now public sector has lost its relevance in the to N?
industrialization process. (a) 7 (b) 5
2. Public sector does not perform satisfactorily. (c) 4 (d) 6
3. Entrepreneurship in private sector is expanding. 11. The number of deaths among the army personnel is 8 in
4. Effective competition policies are available now. 1000, but among the civilian population it is 20 per 1000.
Which of the statements given above is/are correct in the Which one of the following inferences can be drawn from
given context ? this statement ?
(a) 1and 3 only (b) 2 only (a) It is better to join the army.
(c) 2, 3 and 4 only (d) 1, 2, 3 and 4 (b) The relationship is fortuitous.
5. According to the passage, rural roads should be in the (c) Quality of Life Index is very high within the armed forces.
domain of public sector only. Why ? (d) The groups cann ot be compar ed due to th eir
heterogeneity.
(a) Rural development work is the domain of government
12. Given the statement :"Buses are the cause of more accidents
only.
than cars, and trucks cause fewer accidents than buses",
(b) Private sector cannot have monetary gains in this. which of the following conclusions can we draw ?
(c) Government takes money from tax payers and hence it (a) There are more buses on the road than trucks.
is the responsibility of government only. (b) Car drivers are more careful than bus drivers.
(d) Private sector need not have any social responsibility. (c) Truck drivers are more skilled than either car or bus
6. The portfolio composition of the government refers to drivers.
(a) Public sector assets quality. (d) None of the above
(b) Investment in liquid assets. 13. If political leadership fails to emerge, there is Q likelihood of
(c) Mix of government investment in different industrial military taking over power in developing countries. Radical
sectors. student groups or labour may try to raise revolution but
(d) Buying Return on Investment yielding capital assets. they are not likely to compete with the military. Military
7. The author prefers government as the umpire and private intervention, rule, and withdrawal from politics is closely
sector as players because related to a society's level of political development."
(a) Government prescribes norms for a fair play by the In the context of political development, the assumption in
the above passage is that
private sector.
(a) political leadership is not an effective instrument.
(b) Government is the ultimate in policy formulation.
(b) military fills in political vacuum.
(c) Government has no control over private sector players. (c) military intervention is inevitable for development.
(d) None of the above statements is correct in this context. (d) None of the above
14. Four persons, Alok, Bhupesh, Chander and Dinesh have a (c) they saw the whip in the old man's hand.
total of ` 100 among themselves. Alok and Bhupesh between (d) the road was uneven.
them have as much money as Chander and Dinesh between 20. The expression "a stray dog watched the procession
them, but Alok has more money than Bhupesh; and Chander philosophically" means that
has only half the money that Dinesh has. Alok has in fact ` (a) the dog was restless and ferocious.
5 more than Dinesh has. Who has the maximum amount of (b) the dog stood aloof, looking at the procession with
money ? seriousness.
(a) alok (b) Bhupesh (c) the dog looked at the procession with big,wondering
(c) Chander (d) Dinesh eyes.
15. Examine the following statements : (d) the dog stood there with his eyes closed.
1. George attends Music classes on Monday. Passage - 2
2. He attends Mathematics classes on Wednesday. Cynthia was a shy girl. She believed that she was plain and
3. His Literature classes are not on Friday. untalented. One day her teacher ordered the entire class to show
4. He attends History classes on the day following the up for audition for the school play. Cynthia nearly died of fright
day of his Mathematics when she was told that she would have to stand on stage in front
5. On Tuesday, he attends this Sports classes. of the entire class and deliver dialogues. The mere thought of it
If he attends just one subject in a day and his Sunday is free, made her feel sick. But a remarkable transformation occurred
then he is also free on during the audition. A thin, shy girl, her knees quaking, her
(a) Monday (b) Thursday stomach churning in terror, began to stun everyone with her
(c) Saturday (d) Friday excellent performance. Her bored classmates suddenly
16. Ina row 'A' is in the 11th position from the left and 'B' is in the stopped their noisy chat to stare at her slender figure on the
10th position from the right. If ‘A’ and ‘B’ interchange, then stage. At the end of her audition, the entire room erupted in
'A' becomes 18th from the left. How many persons are there thunderous applause.
in the row other than 'A' and 'B' ? 21. Cynthia was afraid to stand on stage because
(a) 27 (b) 26 (a) she felt her classmates may laugh at her.
(c) 25 (d) 24 (b) her stomach was churning.
17. Location of B is north of A and location of C is east of A. The (c) she lacked self-confidence.
distances AB and AC are 5 km and 12 km respectively. The (d) she did not like school plays.
shortest distance (in km) between the locations B and C is 22. Cynthia's classmates were chatting because
(a) 60 (b) 13 (a) it was their turn to act next.
(c) 17 (d) 7 (b) they were bored of the performances.
18. Two cars start towards each other, from two places A and B (c) Cynthia did not act well.
which are at a distance of 160 km. They start at the same time (d) the teacher had no control over them.
08 :10 AM. If the speeds of the cars are 50 km and 30 km per 23. Cynthia's knees were quaking because
hour respectively, they will meet each other at (a) she felt nervous and shy.
(a) 10 :10 AM (b) 10 :30 AM
(b) The teacher scolded her.
(c) 11:10 AM (d) 11:20 AM
(c) she was very thin and weak.
(d) she was afraid of her classmates.
Directions for the following 6 (six items) :
24. The transformation that occurred during the audition refers
The following six]items are based on two passages in to
English to test the comprehension of English language and
(a) the nervousness of Cynthia.
therefore these items do not have Hindi version. Read each passage
and answer the items that follow. (b) the eruption of the entire room in thunderous applause.
Passage-1 (c) the surprise on\ the faces of her classmates.
In front of us was walking\a bare-headed old man intattered (d) the stunning performance of Cynthia.
clothes. He was driving his beasts. They were all laden with heavy 25. If the 3rd day of a month is Monday, which one of the
loads of clay from the hills and looked tired. The man carried a following will be the fifth day from 21st of this month ?
long whip which perhaps he himself had made. As he walked (a) Monday (b) Tuesday
down the road he stopped now and then to eat the wild berries (c) Wednesday (d) Friday
that grew on bushes along the uneven road. When he threw away 26. For a charity show, the total tickets sold were 420. Half of
the seeds, the bold birds would fly to peck at them. Sometimes a these tickets were sold at the rate of ` 5 each, one-third at
stray dog watched the procession philosophically and then the rate of ` 3 each and the rest for ` 2 each. What was the
began to bark. When this happened, my two little sons would total amount received?
stand still holding my hands firmly. A dog can sometimes be (a) ` 900 (b) ` 1,540
dangerous indeed. (c) ` 1,610 (d) ` 2,000
19. The author's children held his hands firmly because Directions for the following 3 (three) items :
(a) they were scared of the barking dogs. Read the passage given below and answer the items follow.
(b) they wanted him to pluck berries. A, B, C, D, E, F are members of a family. They are engineer,
stenographer, doctor, draughtsman, lawyer and judge (not in (b) To make long term investment decisions for economic
order). A, the engineer is married to the lady stenographer. growth
The judge is married to the lawyer. F, the draughtsman is the son (c) To facilitate wider adaptability of crops
of B and brother of E. C, the lawyer is the daughter-in-law of D. E (d) To predict drought conditions and to recharge aquifers
is the unmarried doctor. D is the grandmother of F. There are two Passage -2
married couples in the family. It is essential that we mitigate the emissions of greenhouse
27. What is the profession of B ? gases and thus avoid some of the worst impacts of climate change
(a) Judge (b) Lawyer that would take place in coming years and decades. Mitigation
(c) Draughtsman (d) Cannot be determined would require a major shift in the way we produce and consume
28. Which of the following is/are a couple/couples ? energy. A shift away from overwhelming dependence on fossil
(a) AD only (b) BC only fuels is now long overdue, but unfortunately, technological
(c) Both AD and BC (d) Both AC and BD development has been slow and inadequate largely because
government policies have not promoted investments in research
29. What is the profession of D?
and development, myopically as a result of relatively low prices of
(a) Judge (b) Stenographer oil. It is now, therefore, imperative for a country like India treating
(c) Doctor (d) Cannot be determined the opportunity of harnessing renewable energy on a large scale
Directions for the following 6 (six) items : as a national imperative. This country is extremely well endowed
Read the following two passages and answer the items that follow with solar, wind and biomass sources of energy. Where we have
each passage. Your answers to these items should be based on lagged, unfortunately, is in our ability to develop and to create
the passages only. technological solutions for harnessing these resources.
Passage - 1 One particular trajectory for carryingout stringent mitigation
Climate change poses potentially devastating effects on India's of greenhouse gas emissions assessed by the Intergovernmental
agriculture. While the overall parameters of climate change are Panel on Climate Change (IPCC) clearly shows the need for
increasingly accepted - a 1°C average temperature increase over ensuring that global emissions of greenhouse gases peak no later
the next 30 years, sea level rise of less than 10 cm in the same than 2015 and reduce rapidly thereafter. The cost associated with
period, and regional monsoon variations and corresponding such a trajectory is truly modest and would amount, in the
droughts - the impacts in India are likely to be quite site and crop estimation of IPCC, to not more than 3 percent of the global GDP
specific. Some crops may respond favourably to the changing in 2030. In other words, the level of prosperity that the world
conditions, others may not. This emphasizes the need to promote would have reached without mitigation would at worst be
agricultural research and create maximum flexibility in the system postponed by a few months or a year at the most. This is clearly
to permit adaptations. not a very high price to pay for protecting hundreds of millions of
The key ingredient for “drought proofing” is the managed recharge people from the worst risks associated with climate change. Any
of aquifers. To ensure continued yields of important staple crops such effort, however, would require lifestyles to change
(e.g. wheat), it may also be necessary to shift the locations where appropriately also. Mitigation of greenhouse gas emissions is not
these crops are grown, in response to temperature changes as a mere technological fix, and clearly requires changes in lifestyles
well as to water availability. The latter will be a key factor in making and transformation of a country's economic structure, whereby
long term investment decisions. effective reduction in
For example, water runoff from the Himalayas is predicted to emissions is brought about, such as through the consumption of
increase over the next 30 years as glaciers melt, but then decline much lower quantities of animal protein. The Food and Agriculture
substantially thereafter. It will be critical to provide incentives to Organization (FAO) has determined that the emissions from the
plan for these large-scale shifts in agro-ecological conditions. livestock sector amount to 18 percent of the total. The reduction
India needs to make long term investment in research and of emissions from this source is entirely in the hands of human
development in agriculture. India is likely to experience changed beings, who have never questioned the impacts that their dietary
weather patterns in future. habits of consuming more and more animal protein are bringing
30. Consider the following statements : about. Mitigation overall has huge co-benefits, such as lower air
Climate change may force the shifting of locations of the pollution and health benefits, higher energy security and greater
existing crops due to employment.
1. melting of glaciers. 32. According to the passage, which of the ' following would
2. water availability and temperature suitability at other help in the mitigation of greenhouse gases ?
locations. 1. Reducing the consumption of meat
3. poor productivity of crops. 2. Rapid economic liberalization
3. Reducingthe consumerism
4. wider adaptability of crop plants.
4. Modern management practices of livestock below :
Which of the statements given above are correct ? Select the correct answer using the code given below :
(a) 1,2 and 3 (b) 2 and 3 only (a) 1,2 and 3 (b) 2, 3 and 4
(c) 1and 4 only (d) 1,2, 3 and 4 (c) 1 and 3 only (d) 2 and 4 only
31. According to the passage, why is it important to promote 33. Why do we continue to depend on the fossil fuels heavily ?
agricultural research in India? 1. Inadequate technological development
(a) To predict variations in monsoon patterns and to 2. Inadequate funds for research and development
manage water resources 3. Inadequate availability of alternative sources of energy
Select the correct answer using the code given below : 41. Out of a total of 120 musicians in a club, 5% can play all the
(a) 1only (b) 2 and 3 only three instruments, guitar, violin and flute. It so happens that
(c) 1and 3 only (d) 1,2 and 3 the number of musicians who can play any two and only
34. According to the passage, how does the mitigation of two of the above instruments is 30. The number of musicians
greenhouse gases help us ? who can play the guitar alone is 40. What is the total number
1. Reduces expenditure on public health of those who can play violin alone or flute alone ?
2. Reduces dependence on livestock (a) 45 (b) 44
3. Reduces energy requirements (c) 38 (d) 30
4. Reduces rate of global climate change 42. Six identical cards are placed on a table. Each card has
Select the correct answer using the code given below : number '1' marked on one side and number '2' marked on its
(a) 1,2 and 3 (b) 1, 3 and 4 other side. All the six cards are placed in such a manner that
(c) 2, 3 and 4 (d) 1and 4 only the number '1' is on the upper side. In one try, exactly four
35. What is the essential message of the passage ? (neither more nor less) cards are turned upside down. In
(a) We continue to depend on fossil fuels heavily how many least number of tries can the cards be turned
(b) Mitigation of the greenhouse gases is imperative upside down such that all the six cards show number '2' on
(c) We must invest in research and development the upper side ?
(d) People must change their lifestyle (a) 3 (b) 5
36. There are 50 students admitted to a nursery class. Some (c) 7 (d) This cannot be achieved
students can speak only English and some can speak only Directions for the following 5 (five) items :
Hindi. 10 students can speak both English and Hindi. If the Read the following two passages and answer the items that follow
number of students who can speak English is 21, then how each passage. Your answers to these items should be based on
many students can speak Hindi, how many can speak only the passages only.
Hindi and how many can speak only English? Passage - 1
(a) 21, 11and 29 respectively In recent times, India has grown fast not only compared to its own
(b) 28, 18 and 22 respectively past but also incomparison with other nations. But there cannot
(c) 37, 27 and 13 respectively be any room for complacency because it is possible for the Indian
(d) 39, 29 and 11respectively economy to develop even faster and also to spread the benefits of
37. A gardener increased the area of his rectangular garden by this growth more widely than has been done thus far. Before going
increasing its length by 40% and decreasing its width by into details of the kinds of micro-structural changes that we need
20%. The area of the new garden to conceptualize and then proceed to implement, it is worthwhile
(a) has increased by 20%. elaborating on the idea of inclusive growth that constitutes the
(b) has increased by 12%. defining concept behind this Government's various economic
(c) has increased by 8%. policies and decisions. A nation interested in inclusive growth
(d) is exactly the same as the old area. views the same growth differently depending on whether the gains
38. Six books are labelled A, B, C, D, E and F and are placed side of the growth are heaped primarily on a small segment or shared
by side. Books B, C, E and F have green covers while others widely by the population. The latter is cause for celebration but
have yellow covers. Books A, B and D are new while the rest not the former. In other words, growth must not be treated as an
are old volumes. Books A, B and C are law reports while the end in itself but as an instrument for spreading prosperity to all.
rest are medical extracts. Which two books are old medical India's own past experience and the experience of other nations
extracts and have green covers ? suggests that growth is necessary for eradicating poverty but it is
(a) B and C (b) E and F not a sufficient condition. In other words, policies for promoting
(c) C and E (d) C and F growth need to be complemented with policies to ensure that
39. A straight line segment is 36 cm long. Points are to be marked more and more people join in the growth process and, further, that
on the line from both the end points. From each end, the first there are mechanisms inplace to redistribute some of the gains to
point is at a distance of 1cm from the end, the second point those who are unable to partake in the market process and, hence,
is at a distance of 2 cm from the first point and the third get left behind.
point is at a distance of 3 cm from the second point and so A simple way of giving this idea of inclusive growth a sharper
on. If the points on the ends are not counted and the common form is to measure a nation's progress in terms of the progress of
points are counted as one, what is the number of points ? its poorest segment, for instance the bottom 20 per cent of the
(a) 10 (b) 12 population. One could measure the per capita income of the bottom
(c) 14 (d) 16 quintile of the population and also calculate the growth rate of
40. If Sohan, while selling two goats at the same price, makes a income; and evaluate our economic success in terms of these
profit of 10% on one goat and suffers a loss of 10% on the measures that pertain to the poorest segment. This approach is
other attractive because it does not ignore growth like some of the older
(a) he makes no profit and no loss. heterodox criteria did. It simply looks at the growth of income of
(b) he makes a profit of 1%. the poorest sections of the population. It also ensures that those
(c) he suffers a loss of.1%. who are outside of the bottom quintile do not get ignored. If that
(d) he suffers a loss of 2%. were done, then those people would in all likelihood drop down
into the bottom quintile and so would automatically become a
direct target of our policies. Hence the criterion being suggested 47. Consider the following statements :
here is a statistical summing up of the idea of inclusive growth, According to the passage, private oil companies re-enter
which, in turn, leads to two corollaries : to wish that India must the oil producing market if
strive to achieve high growth and that we must work to ensure 1. a transparent rule-based petrol pricing exists.
that the weakest segments benefit from the growth.
2. there is no government interference in the oil producing
43. The author's central focus is on. market.
(a) applauding India's economic growth not only against
3. subsidies are given by the government
its own past performance, but against other nations.
(b) emphasizing the need for economic growth which is 4. regulations of anti-trust are removed.
the sole determinant of a country's prosperity. Which of the statements given above are correct ?
(c) emphasizing inclusive growth where gains of growth (a) 1and 2 (b) 2 and 3
are shared widely by the population. (c) 3 and 4 (d) 2 and 4
(d) emphasizing high growth. 48. Five persons fire bullets at a target at an interval of 6, 7, 8, 9
44. The author supports policies which will help and 12 seconds respectively. The number of times they would
(a) develop economic growth. fire the bullets together at the target in an hour is
(b) better distribution of incomes irrespective of rate of (a) 6 (b) 7
growth. (c) 8 (d) 9
(c) develop economic growth and redistribute economic 49. A group of 630 children is seated in rows for a group photo
gains to those getting left behind. session. Each row contains three less children than the row
(d) put an emphasis on the development of the poorest in front of it.Which one of the following number of rows is
segments of society. not possible ?
45. Consider the following statements : (a) 3 (b) 4
According to the author, India's economy has grown but (c) 5 (c) 6
there is no room for complacency as 50. There are seven persons up on a ladder, A, B, C, D, E, F and
1. growth eradicates poverty. G (not in that order). A is further up than E but is lower than
2. growth has resulted in prosperity for all. C. B is in the middle. G is between A and B. E is between B
Which of the statements given above is/are correct ? and F. If F is between E and D, the person on the bottom step
(a) 1only (b) 2 only of the ladder will be
(c) Both 1and 2 (d) Neither 1nor 2 (a) B (b) F
Passage -2 (c) D (d) E
Itis easy for the government to control State-owned companies 51. Consider that :
through nods and winks. So what really needs to be done as a first
1. A is taller than B. 2. C is taller than A.
step is to put petrol pricing on a transparent formula - if the price
of crude is x and the exchange rate y, then every month or fortnight, 3. D is taller than C. 4. E is the tallest of all.
the government announces a maximum price of petrol, which If they are made to sit in the above order of their height, who
anybody can work out from the x and the y. The rule has to be will occupy the mid position ?
worked out to make sure that the oil-marketing companies can, in (a) A (b) B
general, cover their costs. This will mean that if one company can (c) C (d) D
innovate and cut costs, it will make greater profits. Hence, firms 52. Consider the following statements :
will be more prone to innovate and be efficient under this system. There are six villages A, B, C, D, E and F.
Once the rule is announced, there should be no interference by F is 1 km to the west of D.
the government. If this is done for a while, private companies will B is 1 km to the east of E.
re-enter this market. And once a sufficient number of them are in A is 2 km to the north of E.
the fray, we can remove the rule-based pricing and leave it truly to
C is 1 km to the east of A.
the market (subject to, of course, the usual regulations of anti-
trust and other competition laws). D is 1 km to the south of A.
46. Consider the following statements : Which three villages are in a line ?
According to the passage, an oil company can make greater (a) A, C, B (b) A, D, E
profits, if a transparent formula for petrol pricing is announced (c) C, B, F (d) E, B, D
every fortnight or month, by 53. Four children are sitting in a row. A is occupying the seat
1. promoting its sales. next to B but not next to C. If C is not sitting next to D, who
is/are occupying seat/seats adjacent to D ?
2. undertakinginnovation.
(a) B (b) A
3. cutting costs.
(c) B and A (d) Impossible to tell
4. selling its equity shares at higher prices.
54. Assume that
Which of the statements given above is/are correct ?
1. the hour and minute hands of a clock move without jerking.
(a) 1only (b) 2 and 3
2. the clock shows a time between 8 o'clock and 9 o'clock.
(c) 3 and 4 (d) 1, 2 and 4
3. the two hands of the clock are one above the other.
After how many minutes (nearest integer) will the two hands 57. What is the most important message conveyed by the
be again lying one above the other ? passage ?
(a) 60 (b) 62 (a) Endemism is a characteristic feature of Himalayan region.
(c) 65 (d) 67 (b) Conservation efforts should emphasize on biogeographic
Directions for the following 8 (eight) items : ranges rather than on some species or habitats.
Read the following two passages and answer the items that follow (c) Climate change has adverse impact on the Himalayan
each passage. Your answers to these items should be based on ecosystem.
the passages only. (d) Without Himalayan ecosystem, the life of the
Passage - 1 communities of uplands and downstreams will have no
The Himalayan ecosystem is highly vulnerable to damage, both sustenance.
due to geological reasons and on account of the stress caused by 58. With reference to the passage, the following assumptions
increased pressure of population, exploitation of natural resources have been made :
and other related challenges. These aspects may be exacerbated 1. To maintain natural ecosystems, exploitation of natural
due to the impact of climate change. It is possible that climate resources should be completely avoided.
change may adversely impact the Himalayan ecosystem through 2. Not only anthropogenic but also natural reasons can
increased temperature, altered precipitation patterns, episodes of adversely affect ecosystems.
drought and biotic influences. This would not only impact the 3. Loss of endemic diversity leads to the extinction of
very sustenance of the indigenous communities in uplands but ecosystems.
also the life of downstream dwellers across the country and Which of the above assumptions is/are correct ?
beyond. Therefore, there is an urgent need for giving special
(a) 1and 2 (b) 2 only
attention to sustain the Himalayan ecosystem. This would require
(c) 2 and 3 (d) 3 only
conscious efforts for conserving all the representative systems.
Further, it needs to be emphasized that the endemics with Passage -2
restricted distribution, and most often with specialized habitat It is often forgotten that globalization is not only about policies
requirements, are among the most vulnerable elements. In this on international economic relationships and transactions, but has
respect the Himalayan biodiversity hotspot, with rich endemic equally to do with domestic policies of a nation. Policy changes
diversity, is vulnerable to climate change. The threats include necessitated by meeting the internationally set conditions (by
possible loss of genetic resources and species, habitats and WTO etc.) of free trade and investment flows obviously affect
concomitantly a decrease in ecosystem services. Therefore, domestic producers and investors. But the basic philosophy
conservation of endemic elements inrepresentative ecosystems/ underlying globalization emphasizes absolute freedom to markets
habitats assumes a great significance while drawing conservation to determine prices and production and distribution patterns, and
plans for the region. view government interventions as processes that create distortions
Towards achieving the above, we will have to shift toward and bring in inefficiency. Thus, public enterprises have to be
contemporary conservation approaches, which include a paradigm privatized through disinvestments and sales; sectors and activities
of landscape level interconnectivity between protected area hitherto reserved for the public sector have to be opened to the
systems. The concept advocates a shift from the species-habitat private sector. This logic extends to the social services like
focus to an inclusive focus on expanding the biogeographic range education and health. Any restrictions on the adjustments in
so that natural adjustments to climate change can proceed without workforce by way of retrenchment of workers should also be
being restrictive. removed and exit should be made easier by removing any
55. Consider the following statements : restrictions on closures. Employment and wages should be
According to the passage, the adverse impact of climate governed by free play of market forces, as any measure to regulate
change on an ecosystem can be a them can discourage investment and also create inefficiency in
production. Above all, in line with the overall philosophy of
1. permanent disappearance of some of its flora and fauna.
reduction in the role of the State, fiscal reforms should be
2. permanent disappearance of ecosystem itself.
undertaken to have generally low levels of taxation and government
Which of the statements given above is/are correct ?
expenditure should be kept to the minimum to abide by the principle
(a) 1only (b) 2 only
of fiscal prudence. All these are policy actions on the domestic
(c) Both 1and 2 (d) Neither 1nor 2 front and are not directly related to the core items of the
56. Which one of the following statements best implies the need globalization agenda, namely free international flow of goods and
to shift toward contemporary conservation approach ? finance.
(a) Exploitation of natural resources causes a stress on 59. According to the passage, under the globalization,
the Himalayan ecosystem. government interventions are viewed as processes leading
(b) Climate change alters precipitation patterns, causes to
episodes of drought and biotic interference.
(a) distortions and inefficiency in the economy.
(c) The rich biodiversity, including endemic diversity,
(b) optimum use of resources.
makes the Himalayan region a biodiversity hotspot.
(d) The Himalayan biogeographic region should be enabled (c) more profitability to industries.
to adapt to climate change smoothly. (d) free play of market forces with regard to industries.
40. According to the passage, the basic philosophy of 67. The following table shows the marks obtained by two
globalization is to students in different subjects :
(a) give absolute freedom to producers to determine prices
and production. Student Maximum Student Maximum
(b) give freedom to producers to evolve distribution A Marks B Marks
patterns. English 60 100 80 150
(c) give absolute freedom to markets to determine prices, Paychology 70 100 70 100
production and employment.
(d) give freedom to producers to import and export. History 50 100 60 100
61. According to the passage, which of the following is/are Sanskrit 30 50 15 25
necessary for ensuring globalization ?
The difference in the mean aggregate percentage marks of
1. Privatization of public enterprises the students is
2. Expansionary policy of public expenditure
(a) 2.5 % (b) 13.75%
3. Free play of market forces to determine wages and
employment (c) 1.25% (d) Zero
4. Privatization of social services like education and health 68. Examine the following figure :
Select the correct answer using the code given below :
(a) 1only (b) 2 and 3 only
(c) 1, 3 and 4 (d) 2, 3 and 4
62. According to the passage, in the process of globalization
the State should have Which one of the following figures has the above figure
(a) expanding role. embedded in it?
(b) reducing role.
(c) statutory role.
(d) none of the above roles.
Directions for the following 4 (four) items :
The following graph shows the average profit of two fruit-sellers (a) (b) (c) (d)
A and B in thousands (`) per year from the year 1995 to 2000. 69. Consider the following matrix :
Consider the graph and answer the 4 (four) items that follow :

6
5
B
4
3 A
2
Which one of the following figures fits into the blank part of
1 the above matrix ?
0 X
1995 1996 1997 1998 1999 2000 (a) (b)
Year
63. In which year is the average profit of A and B same ?
(a) 1995 (b) 1996 (c) (d)
(c) 1997 (d) 1998
64. What is the difference between the average profit of B and A 70. The following table gives population and total income of a
in the year 1998 ? city for four years :
(a) – ` 100 (b) – `1,000
(c) + ` 600 (d) – ` 300 Year 1992 1993 1994 1995
65. How much more average profit did A make in the year 2000 Population in
than in the year 1999 ? 20 21 22 23
lakhs
(a) ` 200 (b) ` 1,000 Income in
(c) ` 1,500 (d) ` 2,000 1010 1111 1225 1345
crores (` )
66. What is the trend of the average profit of B from the year
1997 to the year 2000 ? Which one of the following statements is correct inrespect
Consider the following matrix : of the above data ?
(a) Non-increasing (b) Non-decreasing (a) Population increased by 5% or more every year.
(c) Steady (d) Fluctuating (b) Income increased by 10% or more every year.
(c) Per capita income was always above ` 5,000.
(d) Per capita income was highest in 1994.
71. Consider the table given below in which the numbers bear (a) (b)
certain relationship among themselves along the rows :

29 13 18 (c) (d)
33 X 19 75. A bell rings every 18 minutes. A second bell rings every 24
30 27 3 minutes. A third bell rings every 32 minutes. Ifall the three
bells ring at the same time at 8 o'clock in the morning, at what
Which one of the following numbers is the missing number other time will they all ring together ?
indicated above by X ? (a) 12 :40 hrs (b) 12 :48 hrs
(a) 19 (b) 15 (c) 12 :56 hrs (d) 13 : 04 hrs
76. Price is not the same thing as value. Suppose that on a day
(c) 14 (d) 8
the price of everything viz., coal, bread, postage stamps, a
72. Consider the following matrix with one empty block inthe day's labour, the rent of houses, etc. were to double. Prices
lower extreme corner : then would certainly rise, but values of all things except one
would not."
The writer wants to say that if prices of all things were
doubled
(a) the values of all things would remain constant.
(b) the values of the things sold would be doubled.
(c) the values of the things bought would be halved.
(d) the value of money only would be halved.
77. A and B decide to travel from place X to place Y by bus. A
has ? 10 with him and he finds that itis 80% of the bus fare
for two persons. Bfinds that he has ` 3 with him and hands
it over to A. In this context, which one of the following
statements is correct ?
(a) Now the money A has is just enough to buy two tickets.
Which of the following figures could fit in the empty block (b) A stillneeds ` 2 for buying the tickets.
and thus complete the matrix ? (c) After buying the two tickets A will be left with 50 paise.
(d) The money A now has is still not sufficient to buy two
tickets.
78. As per agreement with a bank, a businessman had to refund
a loan in some equal instalments without interest. After
paying 18 instalments he found that 60 percent of his loan
(a) (b) (c) (d) was refunded. How many instalments were there inthe
73. With reference to the figure given below, the number of agreement ?
different routes from S to T without retracing from U and/or (a) 22 (b) 24
V, is (c) 3 (d) 33
79. A worker reaches his factory 3 minutes late if his speed from
his house to the factory is 5 km/hr. If he walks at a speed of
6 km/hr, then he reaches the factory 7 minute early. The
distance of the factory from his house is
S T (a) 3 km (b) 4 km
U V (c) 5 km (d) 6 km
80. "Liberty, therefore, is never real unless the Government can
be called to account when it invades rights."
(a) 3 (b) 6 Which one of the following is the best justification of the
(c) 9 (d) 18 above statement ?
74. Consider the following figures : (a) In the realisation that the government can be brought
to book in a court of law
(b) Inidentifying a man as a political unit in a way which
? distinguishes him from other citizens
(c) In a decentralized society wherein the basic needs of
Change in positions of beads in the four figures above men can find satisfaction
follows a sequence. Following the same sequence, which of (d) In the understanding that liberty and restraints are
the figures below should appear as the fifth figure above ? complementary
ANSWERS AND EXPLANATIONS
1. (c) The apologists of the free market system believe in the 9. (d)
ability of the market to self correct. Their discussions
are generally about free market operations and forces, Profession
on the efficiency of such enterprises and their ability Women Dancers Musicians Actress Violinists
for self correction. Girija
2. (c) Ideological bias means an opinion that is of a partial Vanaja
nature. Here the idea expressed in the passage is that Jalaja ×
an unregulated market is free and competent.
Shailaja ×
3. (c ) It is clearly said that the arguments betray meaning
Tanuja
give away a biased opinion. So (1) referring to the false
ideology of capitalism is one of the options. It is also Pooja
mentioned that a more honest analysis would reflect
the resultant gross inequality. So (4) is also the correct 6 4 9
10. (d)
option. L M N O P Q R S T
4. (c) According to the passage net profits are only 2.2% as Difference between P & T is 5 i.e., T = 5 + 4 = 9
far as the central public sector undertakings are Difference between N & T is 3 i.e., N = 9 – 3 = 6
concerned. So option (2) is correct. The private sector So, integer assigned to N = 6
has grown. There is a larger base with more educated 11. (d) The groups can not be compared due to their
and moneyed people around. Hence option (3) is 12. (d)
correct too from the point of view of the paragraph. 13. (b)
The role of governmental institutions in encouraging 14. (a) a + b + c + d = 100
competition among the private sector enterprises Also a + b = c + d = 50
ensures the continuing growth of this sector. d d
c= , \ + d = 50 Þ d = 33.3
5. (b) The rural roads should be in the domain of the public 2 2
sector only because the sparse or the little traffic in \ d>c
these roads makes it impossible to make profits on the Also, a > b & between a and d, a = d + 5
road tax or tolls as they are called. \ a>d
6. (c) The government does invest in different industrial \ Alok (a) has the maximum money.
15. (d) Schedule of classes
sectors. This is what the paragraph says. Indeed there
is governmental investment in a variety of industries
Subject Day
from airlines, roads, steel, and finance to industries
where the private sector plays a prominent role. For in Music Monday
the latter it acts as the umpire or the regulating agency Maths Wednesday
so as to create a competitive atmosphere for the growth
of entrepreneurship. History Thursday
7. (a) The government acts as a regulating agency by Sports Tuesday
prescribing broad norms within which the private
Literature Saturday
enterprises can conduct their business in an
atmosphere of fair and fearless competition. That is Hence Friday is free.
why the government is made the umpire in a field of 16. (c)
business players.
A B
8. Medieval Period Poets = A, B, C, D Ist
11th 10th
Modern Period Poets = E, F, G, H arrangement
Option (a) is eliminated as it is a modern poet. Question on
10 students 9 students
modern poet will come in next year.
Option (c) is eliminated as the paper-seller does not like to B A
ask about F. 18th
Option (b) is appropriate as the paper-seller likes F and those
who life like (E) also. 17 students
No. of students between A & B in the 1st arrangement 28. (c) Couples are AD & BC
= 17 – 11 = 6 29. (b) Stenographer
\ Total no. of students except A & B is, 30. (a) According to the passage the melting of glaciers from
= 10 + 6 + 9 = 25 the Himalayas will lead to increased run offs or flow
17. (b)
of water in the next 30 years, after which it will decrease
B so option (1) is one of the factors which will force the
shifting of location of existing crops due to climatic
conditions. Temperatures will increase by 1degree
2 2
5 Shortest Distance, BC = 5 + 12 = 13 km centigrade over the same period all over the world, and
the sea level is expected to rise by 10cm, so obviously
very hot places cannot be suitable for certain crops. So
option (2) is also correct. Due to drought conditions
A C
12 and monsoon variations certain areas will give poor
crop productivity. So crop production will have to be
18. (a) 50 kmph 30 kmph shifted to more suitable areas to adjust with ecological
C imbalances. Hence option (3) has to be included in the
A B
answer too.
31. (c) As India is likely to experience changed weather
patterns in the future it is important to invest in research
160 km and development in agriculture so that it is possible to
Suppose the cars meet at point C after ‘t’ hrs. locate and indentify the areas which will be most suitable
\ AC = 50 t and BC = 30 t for different crops.
\ 50 t + 30 t = 160
32. (c) According to the passage there should be changes in
160 lifestyle; especially the overuse of technology should
t= = 2 hrs.
80 be stopped at all cost as stated in option (3). There
So, the cars will meet at 10 : 10 AM should be less consumption of animal protein that is
19. (a) 20. (c) meat. The FAQ or the Food Administration
21. (c) 22. (b) Organization claims that nearly 18% of greenhouse
23. (a) 24. (d)
gases are emitted from livestock. So option (1) is also
25. (b) Tuesday
3rd Þ Monday + 7 true.
10th Þ Monday + 7 33. (a) As Indians we continue to depend on fossil fuel heavily
17th Þ Monday + 7 because of the lack of investment in research and
24th Þ Monday development in other forms of energy which are so
Fifth day from 21st is 25th day. readily available. The lack of technological development
So, on 25th day is Tuesday. prevents us from harnessing the resources of wind,
26. (c) Total amount received is solar and biomass energy, which are readily available.
420 420 420 34. (d) According to the passage the mitigation of greenhouse
= ´5 + ´3 + ´2
2 3 6 gases cause lower air pollution, this will lead to better
= 210 × 5 + 140 × 3 + 70 × 2 health among the public, as a result there will be less
= 1050 + 420 + 140 = ` 1610 expenditure on public health as stated in option (1);
Solutions for (27 to 29) scientists have already predicted that the emission of
+
greenhouse gases will have disastrous impact on climate
Male
change in the next 30 years and worst still after that. So
– Female option (4) is also correct.
35. (b) The essential message conveyed by the passage is
that it is absolutely essential to reduce to a very great
A (Engineer) D (Stenographer)
(+) (–) extent the emission of greenhouse gases into the
atmosphere.

36. (d) English Hindi

B (Judge) C (Lawyer)
(+) (–) 11 10 29

E (Doctor) F (Draughtman) Students who can speak Hindi = 29 + 10 = 39


(?) (+)

27. (a) Judge


Students who can speak only Hindi = 50 – 21 = 29 Total S.P. = 2 × 110 = 220
Students who can speak only English = 50 – 29 + 10
2000 20
= 11 - 220 ´ 100
9
37. (b) Let initial dimensions be, & b
\ Loss % = = 9 ´ 100
\ Final length is 1.4 l 2000 2000
Final breadth is 0.8 b 9 9
\ Final area is = 1.4 l × 0.8 b = 1.12 lb = 1% loss
\ Area is increased by 12%. Shortcut method: Loss %

40 ´ ( -20 )
2
common gain and loss% 100
Shortcut Method : + 40 – 20 + = = = 1%
100 10 100
= 20 – 8 = 12%
41. (b) Guitar Violin
Therefore, the area of the new garden increased by
12%.
a b f
38. (b)
Books Covers Volume Law/Medical g
c e
A Yellow New Law
B Green New Law d
C Green Old Law
D Yellow New Medical Flute
E Green Old Medical a + b + c + d + e + f + g = 120
F Green Old Medical 5 ´ 120
g= =6
100
Therefore, E and F are old medical extracts and have g=6 ..(1)
green covers. c + b + e = 30 ...(2)
1 2 3 .......... n a = 40 ...(3)
39. (c) So, a + b + c + e + d + f + g = 120 ...(4)
From (1), (2), (3) & (4)
40 + 30 + d + f + 6 = 120
36 km
d + f = 120 – 76
\ 1 + 2 + 3 + ........ n = 36 d + f = 44
n ( n + 1) 42. (a)
= 36 3 Flip
2
n(n + 1) = 72 1 1 1 1 1 1
Þn=8 Flip
\ Leaving the end points the no. of points starting 1 2 2 2 2 1 1st tries
from A is 7.
Flip
Similarly starting from B, the no. of points will be 7. 1 2 1 1 1 2 2nd tries
And also no. of the points between A & B will coincide. 3rd tries
2 2 2 2 2 2
\ Total no. of points = 7 + 7 = 14
40. (c) Let C.P of 1st goat is Rs. 100. 43. (c) In this passage the author wants to convey that the
\ S.P. of 1st goat is Rs. 110 (10% profit) best way to ensure the prosperity and further economic
growth of the nation is by implementing inclusive
\ S.P. of 2nd goat is Rs. 110 (Same S.P.)
growth where the benefits of growth are shared by the
100 entire population and not by certain segments only.
\ C.P. of 2nd goat = ´ 110 (10% loss)
90 44. (c) To make the process of inclusive growth more effective
the author suggests that we should not only
1100 concentrate on the poorer sections of society, but also
=
9 ensure that the rich and middle class stand to gain
certain points. In this way it will be possible to
1100 2000
\ Total C.P. = 100 + = redistribute the economic gains made by these sections
9 9 of society to those getting left behind.
45. (d) According to the passage eradication of poverty is not 51. (c) Sequence according to height.
a sufficient condition for growth. So option (1) is ruled
out. Growth has to be treated as an instrument for E > D> C>A> B
ensuring prosperity for all. But this is yet to be attained.
So India has to strive on. So option (2) is also ruled out.
Mid Position
46. (b) If the price of crude oil is x and the exchange rate is y,
and if such a transparent formula is set in place by the Therefore, ‘C’ occupy the mid position.
government the oil companies can make profits by
innovating within the rules and regulations of anti- trust 52. (b) 1 km
A C
and other competition laws. To ensure greater profits
they will also have to find out ways to cut costs. So
options (2) and (3) have been selected. 1 km
47. (a) According to the passage private oil companies can 1 km
re-enter the oil producing market if a transparent rule F D
based petrol pricing exists because they will be able to
innovate, cut their costs and earn more profits which is 1 km
an attractive incentive for any business. 1 km
48. (b) Time gap after which they will first hit the target is E B
given by LCM of 6, 7, 8, 9, 12.
Hence, A, D, E in a line.
2 6, 7, 8, 9, 12
53. (b) C B A D
3 3, 7, 4, 9, 6
Hence, A occupy seats adjacent to D.
2 1, 7, 4, 3, 2
54. (c) Two hands of clock are on one above between 8 to 9.
1, 7, 2, 3, 1 12 to 8 Þ 40 min
LCM = (12 × 42) sec. 60 480
\ In 1 hr (= 3600 sec) no. of time they will hit together is 40 min Þ ´ 40 = min past 8.
55 11
3600 50 1
= = = 7 times Two hands of clock are on one above between 9 to 10.
12 ´ 42 7 7 12 to 9 Þ 45 min
= 7 times in an hour.
49. (d) Let no. of column = x, no. of rows = y 60 540
45 min Þ ´ 45 = min past 9.
\ xy = 630 – [3 × 1 + 3 × 2 + ......... + 3 × (y – 1)] 55 11
= 630 – 3 [1 + 2 + ........ + (y – 1)] So two hands be lying one above the other
3 ( y - 1) y
xy = 630 – 480 540 660 + 540 - 480 720
2 = 60 - + = = = 65.4
11 11 11 11
621 55 (d) According to the passage due to the adverse impact of
(a) If y = 3, then 3x = 630 – 9 Þ x = = 207
3 climatic change on the ecosystem can be a possible
loss of certain species of animals and their habitats
612
(b) If y = 4, then 4x = 630 – 18 Þ x = = 153 and also a decrease in the services of the ecosystem.
4 So neither (1) nor (2) are correct.
600 56. (b) The passage clearly states that the climatic change may
(c) If y = 5, then 5x = 630 – 30 Þ x = = 120 have an adverse affect on the Himalayan ecosystem,
5
by altering temperatures, changing precipitation
50. (c) patterns , leading to drought and consequently the
Top C death of several species of animals and plants including
A humans.
57. (b) The most important message conveyed in the passage
G is summed up in the last few lines. The idea is to shift
the attention from merely the species-habitat focus to
Middle B
the entire biographical range meaning all the plants
E and animals including humans so that climatic
adjustments can be taken care of more effectively.
F 58. (b) It is clear from the passage that not only human activity
Bottom D is causing environmental pollution, but also natural
reasons can adversely affect climate systems. This is
D is at the bottom step of the ladder. clearly stated in the first four lines of the passage.
59. (a) According to the passage government interference
leads to distortions and inefficiency in the economy in 68. (c)
the sense that there is room for corruption as well as a
lack of interest in investment on the part of the
entrepreneurs.
60. (c) The first paragraph states that the basic philosophy of
globalization is to ensure absolute freedom for the
markets, to set their prices, produce their goods, and
distribute them as per their own criterion.
61. (c) The passage clearly states that in accordance with the
conditions set by the WTO. Etc. for globalization, public 69. (b) 1st Column 3rd Column
sectors should be privatized. So option (1) is correct.
Employment and wages should be conditioned by the
free play of the market forces involved, otherwise it
might discourage investment as stated in option (3). 2nd arrow
Even social services like heath and education should Þ bend towards Þ
welcome private players as is correctly expressed in rights
option (4).
Þ 2nd arrow Þ
62. (b) The entire passage focuses on the fact that the state bend down
should play a reducing role in the process of
globalization. This is elaborated in the last few lines of
the passage with particular reference to India. 2nd Column
63. (b) From the given options, in 1996, the graphs of A & B
intersect, which shows, same average profit.
\ Ans - 1996
64. (c) From the graph, we observe that the difference between Second arrow bends
points A & B is more than half of 1000, but less than Þ towards left
1000.
So second arrow
Þ move up

4 B 70. (c) Rule out options:-


Option (a): - We clearly see that from 1993 to 94, increase
3 A
1
in pop is less than 5% ´100 5
21
Option (b): - from year 1994 to 1995
1345 - 1225
Increase in income = ´ 100
\ 500 < |Difference between A & B| < 1000. 1225
From the option, + 600 satisfies this condition.
120 480
65. (d) In 2000, Profit of A = 6000 = ´4 = 10
In 1999, profit of A = 4000 49 49
\ Difference = 2000 A Rs \ Not true
66. (d) Option (c): per capita income (p.c.i.)
67. (d) For A : Agg. marks = 60 + 70 + 50 + 30 = 210 Total Income in an year
Agg max.marks = 100 + 100 + 100 + 50 = 350 =
Total population in that year.
210
% Agg marks of A = ´ 100 = 60% 101000
350 In 1992 p.c.i = = 5050 > 5000
20
For B:
Agg marks = 80 + 70 + 60 + 15 = 225 111100
In 1993 p.c.i = 5000
Agg max marks = 150 + 100 + 100 + 25 = 375 21
225 122500
% Agg marks of B = ´ 100 = 60% In 1994 p.c.i = 5000
375 22
Difference in agg percentage = 0 134500
Note: Difference in agg percentage is same as difference In 1995 p.c.i = 5000
23
in mean agg percentage.
Option (c) is correct.
71. (d) Sum of numbers in 1st Row, 29 + 13 + 18 Þ 60 74. (b)
Sum of numbers in 3rd Row, 30 + 27 + 3 Þ 60 75. (b) LCM of 18, 24, 32
So, 2nd Row, 33 + X + 19 = 60 LCM of 9, 12, 16
X + 52 = 60 LCM = 3 × 4 × 3 × 4 = 144 min
X = 60 – 52 144 2
X=8 = = 2 hr = 2 hrs 24 min
60 5
72. (a) Remove one circle, a triangle is remove,
\ Bell will ring together again after 2 × (2 hrs 24 min)
So, = 4 hrs. 48 min
i.e., 12 : 48 hrs
76. (d) Due to inflation the value of money only would be
halved
77. (c) Let cost of 1 ticket is R. x.

after removing circle, triangle is also remove. 80


\ For A, 10 = ´ 2x
So, 100
50 25
Þ x= = = 6.25 Rs.
8 4
Now B gives Rs. 3 to A.
So, A has 13 Rs.
Cost of 2 tickets = 12.50 Rs., which is more than enough
A circle and a triangle is reducing by one.
to buy 2 tickets.
73. (d) 18
\ (a) Rules out
1 (b) is clearly ruled out
2 (c) is true, because after buying 2 tickets.
Form S T, 3 routes A is left with 13 – 12.50 = 50 paise
U V 3
78. (c) Let amount in each installment = x
Let total loan amount = l
1 60
V 2 \ 18x = l
T, 3 routes 100
S U 3
l 18 ´100
\ = = 30 = no. of installments.
x 60
1 79. (c) Let the distance be x km
2 As per question,
T, 3 routes
S U 3 x 3 x 7
- = +
5 60 6 60
x x 10 1
1 - = =
2 5 6 60 6
S T, 3 routes
3 x 1
U V =
30 6
x = 5 km
1 80. (a)
U 2
S T, 3 routes
V 3

Total routes = 3 + 3 + 3 + 3 + 3 + 3 = 18
From 1 to 2 - vertical interchange of half shaded circle.
INSTRUCTIONS
1. This Test Booklet contains 100 items (questions). Each item comprises four responses (answers). You will
select the response which you want to mark on the Answer sheet. In case, you feel that there is more than one
correct response, mark the response which you consider the best. In any case, choose ONLY ONE response for
each item.
2. All items carry equal marks.
3. Penalty for wrong answers:
THERE WILL BE PENALTY FOR WRONG ANSWERS MARKED BY THE CANDIDATE FOR WRONG
ANSWER.
(i) There are four alternatives for the answer to every question. For each question which has a penalty for which
a wrong answer has been given by the candidate, one-third of the marks assigned to that question will be
deducted as penalty.
(ii) If a candidate gives more than one answer, it will be treated as a wrong answer even if one of the given answers
happens to be correct and there will be same penalty as above to that question.
(iii) If a question is left blank, i.e., no answer is given by the candidate, there will be no penalty for that question.

1. Under the Scheduled Tribes and Other Traditional Forest Select the correct answer using the codes given below.
Dwellers (Recognition of Forest Rights) Act, 2006 who shall (a) 1, 3, 4, 6 and 7 only (b) 1, 2, 3, 5 and 6 only
be the authority to initiate the process for determining the (c) 2, 4, 5 and 7 only (d) 1, 2, 3, 4, 5, 6 and 7
nature and extent of individual or community forest rights or
5. Acid rain is caused by the pollution of environment by
both?
(a) carbon dioxide and nitrogen
(a) State Forest Department
(b) District Collector / Deputy Commissioner (b) carbon monoxide and carbon dioxide
(c) Tahsildar / Block Development Officer / Mandal (c) ozone and carbon dioxide
Revenue Officer (d) nitrous oxide and sulphur dioxide
(d) Gram Sabha 6. With reference to food chains in ecosystems, consider the
2. Improper handling and storage of cereal grains and oilseeds following statements:
result in the production of toxins known as aflatoxins which 1. A food chain illustrates the order in which a chain of
are not generally destroyed by normal cooking process. organisms feed upon each other.
Aflatoxins are produced by 2. Food chains are found within the populations of a
(a) bacteria (b) protozoa species.
(c) moulds (d) viruses 3. A food chain illustrates the numbers of each organism
3. ‘Economic Justice’ as one of the objectives of the Indian which are eaten by others.
Constitution has been provided in
Which of the statements given above is/are correct?
(a) the Preamble and the Fundamental Rights
(a) 1 only (b) 1 and 2 only
(b) the Preamble and the Directive Principles of State Policy
(c) the Fundamental Rights and the Directive Principles of (c) 1, 2 and 3 (d) None
State Policy 7. Consider the following pairs:
(d) None of the above National Park River flowing
4. Due to improper/ indiscriminate disposal of old and used through the Park
computers or their parts, which of the following are released 1. Corbett National Park: Ganga
into the environment as e-waste? 2. Kaziranga National Park: Manas
1. Beryllium 2. Cadmium 3. Silent Valley: National Park : Kaveri
3. Chromium 4. Heptachlor Which of the above pairs is/are correctly matched?
5. Mercury 6. Lead
(a) 1 and 2 (b) 3 only
7. Plutonium
(c) 1 and 3 (d) None of these
8. Consider the following organisms: 15. Quit India Movement was launched in response to
1. Agaricus (a) Cabinet Mission Plan
2. Nostoc (b) Cripps Proposals
3. Spirogyra (d) Simon Commission Report
Which of the above is/are used as biofertilizer / biofertilizers? (d) Wavell Plan
(a) 1 and 2 (b) 2 only
16. The balance of payments of a country is a systematic record
(c) 2 and 3 (d) 3 only of
9. Which of the following adds / add nitrogen to the soil?
(a) all import and export transactions of a country during a
1. Excretion of urea by animals given period of time, normally a year
2. Burning of coal by man
(b) good exported from a country during a year
3. Death of vegetation
(c) economic transaction between the government of one
Select the correct answer using the codes given below.
country to another
(a) 1 only (b) 2 and 3 only
(d) capital movements from one country to another.
(c) 1 and 3 only (d) 1, 2 and 3
17. The Reserve Bank of India regulates the commercial banks in
10. In which of the following States is lion-tailed macaque found
matters of
in its natural habitat?
1. Tamil Nadu 1. liquidity of assets
2. Kerala 2. branch expansion
3. Karnataka 3. merger of banks
4. Andhra Pradesh 4. winding-up of banks
Select the correct answer using the codes given below. Select the correct answer using the codes given below.
(a) 1, 2 and 3 only (b) 2 only (a) 1 and 4 only (b) 2, 3 and 4 only
(c) 1, 3 and 4 only (d) 1, 2, 3 and 4 (c) 1, 2 and 3 only (d) 1, 2, 3 and 4
11. Some Buddhist rock-cut caves are called Chaityas, while the 18. An increase in the Bank Rate generally indicates that the
others are called Viharas. What is the difference between the (a) market rate of interest is likely to fall
two? (b) Central Bank is no longer making loans to commercial
(a) Vihara is a place of worship, while Chaitya is the dwelling banks
place of the monks (c) Central Bank is following an easy money policy
(b) Chaitya is a place of worship, while Vihara is the dwelling (d) Central Bank is following a tight money policy
place of the monks 19. In India, deficit financing is used for raising resources for
(c) Chaitya is the stupa at the far end of the cave, while (a) economic development
Vihara is the hall axial to it.
(b) redemption of public debt
(d) There is no material difference between the two
(c) adjusting the balance of payments
12. Which one of the following describes best the concept of
(d) reducing the foreign debt
Nirvana in Buddhism?
(a) The extinction of the flame of desire 20. Which of the following characterizes / characterize the people
of Indus Civilization?
(b) The complete annihilation of self
(c) A state of bliss and rest 1. They possessed great palaces and temples.
(d) A mental stage beyond all comprehension. 2. They worshipped both male and female deities.
13. According to the Constitution of India, which of the following 3. They employed horse-drawn chariots in warfare.
are fundamental for the governance of the country? Select the correct statement/statements using the codes given
(a) Fundamental Rights below.
(b) Fundamental Duties (a) 1 and 2 only
(c) Directive Principles of State Policy (b) 2 only
(d) Fundamental Rights and Fundamental Duties (c) 1, 2 and 3
14. The people of India agitated against the arrival of Simon (d) None of the statements given above is correct
Commission because 21. Which of the following diseases can be transmitted from one
(a) Indians never wanted the review of the working of the person to another through tattooing?
Act of 1919 1. Chikungunya
(b) Simon Commission recommended the abolition of 2. Hepatitis B
Dyarchy (Diarchy) in the Provinces 3. HIV-AIDS
(c) there was no Indian member in the Simon Commission Select the correct answer using the codes given below.
(d) the Simon Commission suggested the partition of the (a) 1 only (b) 2 and 3 only
country.
(c) 1 and 3 only (d) 1, 2 and 3
22. Which of the following statements is/are applicable to Jain 29. With reference to the history of Indian rock-cut architecture,
doctrine? consider the following statements:
1. The surest way of annihilating Karma is to practice 1. The caves at Badami are the oldest surviving rock-cut
penance. caves in India.
2. Every object, even the smallest particle has a soul. 2. The Barabar rock-cut caves were originally made for
3. Karma is the bane of the soul and must be ended. Ajivikas by Emperor Chandragupta Maurya.
Select the correct answer using the codes given below. 3. At Ellora, caves were made for different faiths.
(a) 1 only (b) 2 and 3 only Which of the statements given above is/are correct?
(c) 1 and 3 only (d) 1, 2 and 3 (a) 1 only (b) 2 and 3 only
23. Which one of the following terms describes not only the (c) 3 only (d) 1, 2 and 3
physical space occupied by an organism, but also its 30. Recombinant DNA technology (Genetic Engineering) allows
functional role in the community of organisms? genes to be transferred
(a) Ecotone (b) Ecological niche 1. across different species of plants
(c) Habitat (d) Home range 2. from animals to plants
24. Photochemical smog is a resultant of the reaction among 3. from microorganisms to higher organisms
(a) NO2, O3 and peroxyacetyl nitrate in the presence of Select the correct answer using the codes given below.
sunlight (a) 1 only (b) 2 and 3 only
(b) CO, O2 and peroxyacetyl nitrate in the presence of (c) 1 and 3 only (d) 1, 2 and 3
sunlight 31. The Chinese traveller Yuan Chwang (Hiuen Tsang) who
(c) CO, CO2 and NO2 at low temperature visited India recorded the general conditions and culture of
(d) high concentration of NO2, O3 and CO in the evening India at that time. In this context, which of the following
25. Consider the following minerals: statements is/are correct?
1. Calcium 1. The roads and river-routes were completely immune
2. Iron from robbery.
3. Sodium 2. As regards punishment for offences, ordeals by fire,
Which of the minerals given above is/are required by human water and poison were the instruments for determining
body for the contraction of muscles? the innocence or guilt of a person.
(a) 1 only (b) 2 and 3 only 3. The tradesmen had to pay duties at ferries and barrier
(c) 1 and 3 only (d) 1, 2 and 3 stations.
26. What will follow if a Money Bill is substantially amended by Select the correct answer using the codes given below.
the Rajya Sabha? (a) 1 only (b) 2 and 3 only
(a) The Lok Sabha may still proceed with the Bill, accepting (c) 1 and 3 only (d) 1, 2 and 3
or not accepting the recommendations of the Rajya 32. Consider the following:
Sabha 1. Star tortoise
(b) The Lok Sabha cannot consider the bill further 2. Monitor lizard
(c) The Lok Sabha may send the Bill to the Rajya Sabha for 3. Pygmy hog
reconsideration 4. Spider monkey
(d) The President may call a joint sitting for passing the Which of the above are naturally found in India?
Bill (a) 1, 2 and 3 only (b) 2 and 3 only
27. Which one of the following statements is correct? (c) 1 and 4 only (d) 1, 2, 3 and 4
(a) In India, the same person cannot be appointed as 33. Which of the following can be found as pollutants in the
Governor for two or more States at the same time drinking water in some parts of India?
(b) The Judges of the High Court of the States in India are 1. Arsenic 2. Sorbitol
appointed by the Governor of the State just as the 3. Fluoride 4. Formaldehyde
Judges of Supreme Court are appointed by the President 5. Uranium
(c) No procedure has been laid down in the Constitution Select the correct answer using the codes given below.
of India for the removal of a Governor from his/her post (a) 1 and 3 only (b) 2, 4 and 5 only
(d) In the case of a Union Territory having a legislative (c) 1,3 and 5 only (d) 1, 2, 3, 4 and 5
setup, the Chief Minister is appointed by the Lt. 34. With reference to Indian History, the Members of the
Governor on the basis of majority support Constituent Assembly from the Provinces were
28. Which one of the following pairs is correctly matched? (a) directly elected by the people of those Provinces
Geographical Feature Region (b) nominated by the Indian National Congress and the
(a) Abyssinian Plateau : Arabia Muslim League
(b) Atlas Mountains : North-Western Africa (c) elected by the Provincial Legislative Assemblies
(c) Guiana Highlands : South-Western Africa (d) selected by the Government for their expertise in
(d) Okavango Basin : Patagonia constitutional matters
35. Consider the following animals: (a) is found in atmosphere as moisture and clouds
1. Sea cow (b) is found in freshwater lakes and rivers
2. Sea horse (c) exists as groundwater
3. Sea lion (d) exists as soil moisture
Which of the above is/are mammal/mammals? 42. Consider the following pairs:
(a) 1 only (b) 1 and 3 only 1. Nokrek Bio-sphere Reserve : Garo Hills
(c) 2 and 3 only (d) 1, 2 and 3 2. Logtak (Loktak) Lake : Barail Range
36. Consider the following statements: 3. Namdapha National Park : Dafla Hills
1. An amendment to the Constitution of India can be Which of the above pairs is/are correctly matched?
initiated by an introduction of a bill in the Lok Sabha
(a) 1 only (b) 2 and 3 only
only.
(c) 1, 2 and 3 (d) None
2. If such an amendment seeks to make changes in the
federal character of the Constitution, the amendment 43. Consider the following pairs:
also requires to be ratified by the legislature of all the 1. Electromagnetic radiation
States of India. 2. Geothermal energy
Which of the statements given above is/are correct? 3. Gravitational force
(a) 1 only (b) 2 only 4. Plate movements
(c) Both 1 and 2 (d) Neither 1 nor 2 5. Rotation of the earth
37. Consider the following statement : Attorney General of India 6. Revolution of the earth
can Which of the above are responsible for bringing dynamic
1. take part in the proceedings of the Lok Sabha changes on the surface of the earth?
2. be a member of a committee of the Lok Sabha (a) 1, 2, 3 and 4 only
3. speak in the Lok Sabha (b) 1, 3, 5 and 6 only
4. vote in the Lok Sabha (c) 2, 4, 5 and 6 only
Which of the statements given above is/are correct? (d) 1, 2, 3, 4, 5 and 6
(a) 1 only (b) 2 and 4 44. Which of the following bodies does not/do not find mention
(c) 1, 2 and 3 (d) 1 and 3 only in the Constitution?
38. With reference to the usefulness of the by-products of sugar 1. National Development Council
industry, which of the following statements is/are correct? 2. Planning Commission
1. Bagasse can be used as biomass fuel for the generation
3. Zonal Councils
of energy.
Select the correct answer using the codes given below.
2. Molasses can be used as one of the feedstocks for the
production of synthetic chemical fertilizers. (a) 1 and 2 only (b) 2 only
3. Molasses can be used for the production of ethanol. (c) 1 and 3 only (d) 1, 2 and 3
Select the correct answer using the codes given below. 45. The demand for the Tebhaga Peasant Movement in Bengal
(a) 1 only (b) 2 and 3 only was for
(c) 1 and 3 only (d) 1, 2 and 3 (a) the reduction of the share of the landlords from one-
39. Variations in the length of daytime and nighttime from season half of the crop to one-third
to season are due to (b) the grant of ownership of land to peasants as they
(a) the earth’s rotation on its axis were the actual cultivators of the land
(b) the earth’s revolution round the sun in an elliptical (c) the uprooting of Zamindari system and the end of
manner serfdom
(c) latitudinal position of the place (d) writing off all peasant debts
(d) revolution of the earth on a tilted axis. 46. The Parliament can make any law for whole or any part of
40. The Narmada river flows to the west, while most other large India for implementing international treaties
peninsular rivers flow to the east. Why? (a) with the consent of all the States
1. It occupies a linear rift valley. (b) with the consent of the majority of States
2. It flows between the Vindhyas and the Satpuras. (c) with the consent of the States concerned
3. The land slopes to the west from Central India. (d) without the consent of any State
Select the correct answer using the codes given below. 47. In the grasslands, trees do not replace the grasses as a part
(a) 1 only (b) 2 and 3 of an ecological succession because of
(c) 1 and 3 (d) None (a) insects and fungi
41. On the planet earth, most of the freshwater exists as ice caps (b) limited sunlight and paucity of nutrients
and glaciers. Out of the remaining freshwater, the largest (c) water limits and fire
proportion (d) None of the above
48. Which one of the following is the correct sequence of 56. The Ilbert Bill controversy was related to the
ecosystems in the order of decreasing productivity? (a) imposition of certain restrictions to carry arms by the
(a) Oceans, lakes, grasslands, mangroves Indians
(b) Mangroves, oceans, grasslands, lakes (b) imposition of restrictions on newspapers and magazines
(c) Mangroves, grasslands, lakes, oceans published in Indian languages
(d) Oceans, mangroves, lakes, grasslands (c) removal of disqualifications imposed on the Indian
49. Contour bunding is a method of soil conservation used in magistrates with regard to the trial of the Europeans
(a) desert margins, liable to strong wind action (d) removal of a duty on imported cotton cloth
57. A rise in general level of prices may be caused by
(b) low flat plains, close to stream courses, liable to flooding
1. an increase in the money supply
(c) scrublands, liable to spread of weed growth
2. a decrease in the aggregate level of output
(d) None of the above 3. an increase in the effective demand
50. The Government enacted the Panchayat Extension to Select the correct answer using the codes given below.
Scheduled Areas (PESA) Act in 1996. Which one of the (a) 1 only (b) 1 and 2 only
following is not identified as its objective? (c) 2 and 3 only (d) 1, 2 and 3
(a) To provide self-governance 58. Which one of the following groups of items is included in
(b) To recognize traditional rights India’s foreign-exchange reserves?
(c) To create autonomous regions in tribal areas (a) Foreign-currency assets, Special Drawing Rights
(d) To free tribal people from exploitation (SDRs) and loans from foreign countries
51. Priority Sector Lending by banks in India constitutes the (b) Foreign-currency assets, gold holdings of the RBI and
lending to SDR’s
(a) agriculture (c) Foreign-currency assets, loans from the World Bank
(b) micro and small enterprises and SDRs
(c) weaker sections (d) Foreign-currency assets, gold holdings of the RBI and
(d) All of the above loans from the World Bank
52. Which one among the following industries is the maximum 59. Which one of the following is likely to be the most inflationary
consumer of water in India? in its effect?
(a) Repayment of public debt
(a) Engineering (b) Paper and pulp
(b) Borrowing from the public to finance a budget deficit
(c) Textiles (d) Thermal power
(c) Borrowing from banks to finance a budget deficit
53. To obtain full benefits of demographic dividend, what should
(d) Creating new money to finance a budget deficit
India do?
60. Supply of money remaining the same when there is an
(a) Promoting skill development increase in demand for money, there will be
(b) Introducing more social security schemes (a) a fall in the level of prices
(c) Reducing infant mortality rate (b) an increase in the rate of interest
(d) Privatization of higher education (c) a decrease in the rate of interest
54. In the context of cultural history of India, a pose in dance and (d) an increase in the level of income and employment
dramatics called ‘Tribhanga’ has been a favourite of Indian 61. Fruits stored in a cold chamber exhibit longer storage life
artists from ancient times till today. Which one of the following because
statements best describes this pose? (a) exposure to sunlight is prevented
(a) One leg is bent and the body is slightly but oppositely (b) concentration of carbon dioxide in the environment is
curved at waist and neck increased
(b) Facial expressions, hand gestures and make-up are (c) rate of respiration is decreased
combined to symbolize certain epic or historic characters (d) there is an increase in humidity
(c) Movements of body, face and hands are used to express 62. Consider the following fauna of India:
oneself or to tell a story 1. Gharial 2. Leatherback turtle
(d) A little smile, slightly curved waist and certain hand 3. Swamp deer
gestures are emphasized to express the feelings of love Which of the above is/are endangered?
or eroticism. (a) 1 and 2 only (b) 3 only
55. Annie Besant was (c) 1, 2 and 3 (d) None
1. responsible for starting the Home Rule Movement 63. Ball bearings are used in bicycles, cars, etc., because
(a) the actual area of contact between the wheel and axle is
2. the founder of the Theosophical Society
increased
3. once the President of the Indian National Congress
(b) the effective area of contact between the wheel and
Select the correct statement/statements using the codes given axle is increased
below. (c) the effective area of contact between the wheel and
(a) 1 only (b) 2 and 3 only axle is reduced
(c) 1 and 3 only (d) 1, 2 and 3 (d) None of the above statements is correct
64. Consider the following phenomena: 71. The efforts to detect the existence of Higgs boson particle
1. Size of the sun at dusk. have become frequent news in the recent past. What is/are
2. Colour of the sun at dawn the importance / importances of discovering this particle?
3. Moon being visible at dawn 1. It will enable us to understand as to why elementary
4. Twinkle of stars in the sky particles have mass.
5. Polestar being visible in the sky 2. It will enable us in the near future to develop the
Which of the above are optical illusions? technology of transferring matter from one point to
(a) 1, 2 and 3 (b) 3, 4 and 5 another without traversing the physical space between
(c) 1, 2 and 4 (d) 2, 3 and 5 them.
65. Rainbow is produced when sunlight falls on drops of rain. 3. It will enable us to create better fuels for nuclear fission.
Which of the following physical phenomena are responsible Select the correct answer using the codes given below.
for this? (a) 1 only (b) 2 and 3 only
1. Dispersion 2. Refraction (c) 1 and 3 only (d) 1, 2 and 3
3. Internal reflection 72. Mycorrhizal biotechnology has been used in rehabilitating
Select the correct answer using the codes below. degraded sites because mycorrhiza enables the plants to
(a) 1 and 2 only (b) 2 and 3 only 1. resist drought and increase absorptive area
(c) 1 and 3 only (d) 1, 2 and 3 2. tolerate extremes of pH
66. Many transplanted seedling do not grow because 3. resist disease infestation
(a) the new soil does not contain favourable minerals Select the correct answer using the codes given below.
(b) most of the root hair grip the new soil too hard (a) 1 only (b) 2 and 3 only
(c) most of the root hairs are lost during transplantation (c) 1 and 3 only (d) 1, 2 and 3
(d) leaves get damaged during transplantation 73. Who among the following constitute the National
67. Economic growth in country X will necessarily have to occur Development Council?
if 1. The Prime Minister
(a) there is technical progress in the world economy 2. The Chairman, Finance Commission
(b) there is population growth in X 3. Ministers of the Union Cabinet
(c) there is capital formation of X 4. Chief Ministers of the States
(d) the volume of trade grows in the world economy Select the correct answer using the codes given below.
68. Which of the following statements is/are correct? (a) 1, 2 and 3 only (b) 1, 3 and 4 only
1. Viruses lack enzymes necessary for the generation of (c) 2 and 4 only (d) 1, 2, 3 and 4
energy. 74. The national income of a country for a given period is equal
2. Viruses can be cultured in any synthetic medium. to the
3. Viruses are transmitted from one organism to another (a) total value of goods and services produced by the
by biological vectors only. nationals
Select the correct answer using the codes given below. (b) sum of total consumption and investment expenditure
(a) 1 only (b) 2 and 3 only (c) sum of personal income of all individuals
(c) 1 and 3 only (d) 1, 2 and 3 (d) money value of final goods and services produced
69. Which of the following leaf modifications occurs/occur in 75. Which of the following grants / grant direct credit assistance
desert areas to inhibit water loss? to rural households?
1. Hard and waxy leaves 1. Regional Rural Banks
2. Tiny leaves or no leaves 2. National Bank for Agriculture and Rural Development
3. Thorns instead of leaves 3. Land Development Banks
Select the correct answer using the codes given below. Select the correct answer using the codes given below.
(a) 1 and 2 only (b) 2 only (a) 1 and 2 only (b) 2 only
(c) 1 and 3 only (d) 1, 2 and 3 (c) 1 and 3 only (d) 1, 2 and 3
70. The known forces of nature can be divided into four classes, 76. Consider the following statements:
viz., gravity, electromagnetism, weak nuclear force and strong The parliamentary Committee on public accounts
nuclear force. With reference to them, which one of the 1. consists of not more than 25 Members of the Lok Sabha
following statements is not correct? 2. scrutinizes appropriation and finance accounts of the
(a) Gravity is the strongest of the four Government
(b) Electromagnetism acts only on particles with an electric 3. examines the report of the Comptroller and Auditor
charge General of India.
(c) Weak nuclear force causes radioactivity Which of the statements given above is/are correct?
(d) Strong nuclear force holds protons and neutrons inside (a) 1 only (b) 2 and 3 only
the nucleus of an atom (c) 3 only (d) 1, 2 and 3
77. Consider the following Bhakti Saints: Which of the statements given above is/are correct?
1. Dadu Dayal (a) 1 only (b) 2 only
2. Guru Nanak (c) Both 1 and 2 (d) Neither 1 nor 2
3. Tyagaraja 84. In the context of India, which of the following principles is/
Who among the above was/were preaching when the Lodi are implied institutionally in the parliamentary government?
dynasty fell and Babur took over? 1. Members of the Cabinet are Members of the Parliament.
(a) 1 and 3 (b) 2 only 2. Ministers hold the office till they enjoy confidence in
(c) 2 and 3 (d) 1 and 2 the Parliament.
78. With reference to the food chains in ecosystems, which of 3. Cabinet is headed by the Head of the State.
the following kinds of organism is/are known as decomposer Select the correct answer using the codes given below.
organism/organisms? (a) 1 and 2 only (b) 3 only
1. Virus
(c) 2 and 3 only (d) 1, 2 and 3
2. Fungi
85. The annual range of temperature in the interior of the
3. Bacteria
continents is high as compared to coastal areas. What is/are
Select the correct answer using the codes given below.
the reason/reasons?
(a) 1 only (b) 2 and 3 only
1. Thermal difference between land and water
(c) 1 and 3 only (d) 1, 2 and 3
2. Variation in altitude between continents and oceans
79. The most important fishing grounds of the world are found
3. Presence of strong winds in the interior
in the regions where
(a) warm and cold atmospheric currents meet 4. Heavy rains in the interior as compared to coasts
(b) rivers drain out large amounts of freshwater into the Select the correct answer using the codes given below.
sea (a) 1 only (b) 1 and 2 only
(c) warm and cold oceanic currents meet (c) 2 and 3 only (d) 1, 2, 3 and 4
(d) continental shelf is undulating 86. Which of the following is/are the characteristic/characteristics
80. Which of the following is/are unique characteristic/ of Indian coal?
characteristics of equatorial forests? 1. High ash content
1. Presence of tall, closely set trees with crowns forming a 2. Low sulphur content
continuous canopy 3. Low ash fusion temperature
2. Coexistence of a large number of species Select the correct answer using the codes given below.
3. Presence of numerous varieties of epiphytes (a) 1 and 2 only (b) 2 only
Select the correct answer using the codes given below. (c) 1 and 3 only (d) 1, 2 and 3
(a) 1 only (b) 2 and 3 only 87. Which of the following statements regarding laterite soils of
(c) 1 and 3 only (d) 1, 2 and 3 India are correct?
81. Which of the following constitute Capital Account? 1. They are generally red in colour.
1. Foreign Loans 2. They are rich in nitrogen and potash.
2. Foreign Direct Investment 3. They are well-developed in Rajasthan and UP.
3. Private Remittances 4. Tapioca and cashew nuts grow well on these soils.
4. Portfolio Investment Select the correct answer using the codes given below.
Select the correct answer using the codes given below.
(a) 1, 2 and 3 (b) 2, 3 and 4
(a) 1, 2 and 3 (b) 1, 2 and 4
(c) 1 and 4 (d) 2 and 3 only
(c) 2, 3 and 4 (d) 1, 3 and 4
88. Consider the following statements:
82. Consider the following historical places:
1. Natural gas occurs in the Gondwana beds.
1. Ajanta Caves
2. Lepakshi Temple 2. Mica occurs in abundance in Kodarma.
3. Sanchi Stupa 3. Dharwars are famous for petroleum.
Which of the above places is/are also known for mural Which of the statements given above is/are correct?
paintings? (a) 1and 2 (b) 2 only
(a) 1 only (b) 1and 2 only (c) 2 and 3 (d) None
(c) 1, 2 and 3 (d) None 89. Consider the following crops:
83. With reference to the history of philosophical thought in 1. Cotton
India, consider the following statements regarding Sankhya 2. Groundnut
school: 3. Rice
1. Sankhya does not accept the theory of rebirth or 4. Wheat
transmigration of soul. Which of these are Kharif crops?
2. Sankhya holds that it is the self-knowledge that leads (a) 1and 4 (b) 2 and 3 only
to liberation and not any exterior influence or agent. (c) 1, 2 and 3 (d) 2, 3 and 4
90. “Climate is extreme, rainfall is scanty and the people used to Which of the statements given above is/are correct?
be nomadic herders.” (a) 1 only (b) 2 only
The above statement best describes which of the following (c) Both 1 and 2 (d) Neither 1 nor 2
regions?
96. With reference to National Legal Services Authority, consider
(a) African Savannah the following statements:
(b) Central Asian Steppe 1. Its objective is to provide free and competent legal
(c) North American Prairie services to the weaker sections of the society on the
(d) Siberian Tundra basis of equal opportunity.
2. It issues guidelines for the State Legal Services
91. Consider the following statements:
Authorities to implement the legal programmes and
1. Inflation benefits the debtors. schemes throughout the country.
2. Inflation benefits the bondholders. Which of the statements given above is/are correct?
Which of the statements given above is/are correct? (a) 1 only (b) 2 only
(a) 1 only (b) 2 only (c) Both 1 and 2 (d) Neither 1 nor 2
(c) Both 1 and 2 (d) Neither 1 nor 2 97. During a thunderstorm, the thunder in the skies is produced
92. Disguised unemployment generally means by the
(a) large number of people remain unemployed 1. meeting of cumulonimbus clouds in the sky
(b) alternative employment is not available 2. lightning that separates the nimbus clouds
(c) marginal productivity of labour is zero 3. violent upward movement of air and water particles
(d) productivity of workers is low Select the correct answer using the codes given below.
93. Consider the following statements: (a) 1 only
1. The Council of Ministers in the Centre shall be (b) 2 and 3
collectively responsible to the Parliament. (c) 1 and 3
2. The Union Ministers shall hold the office during the (d) None of the above produces the thunder
pleasure of the President of India. 98. Consider the following pairs:
3. The Prime Minister shall communicate to the President Tribe State
about the proposals for legislation. 1. Limboo (Limbu) : Sikkim
Which of the statements given above is/are correct? 2. Karbi : Himachal Pradesh
(a) 1 only (b) 2 and 3 only 3. Dongaria Kondh : Odisha
(c) 1 and 3 only (d) 1, 2 and 3 4. Bonda : Tamil Nadu
94. Consider the following statements: Which of the above pairs are correctly matched?
1. National Development Council is an organ of the (a) 1 and 3 only (b) 2 and 4 only
Planning Commission. (c) 1, 3 and 4 only (d) 1, 2, 3 and 4
2. The Economic and Social Planning is kept in the 99. Consider the following liquid assets:
Concurrent List in the Constitution of India. 1. Demand deposits with the banks
3. The Constitution of India prescribes that Panchayats 2. Time deposits with the banks
should be assigned the task of preparation of plans for 3. Saving deposits with the banks
economic development and social justice. 4. Currency
Which of the statements given above is/are correct? The correct sequence of these assets in the decreasing order
(a) 1 only (b) 2 and 3 only of liquidity is
(c) 1 and 3 only (d) 1, 2 and 3 (a) 1-4-3-2 (b) 4-3-2-1
95. Consider the following statements: (c) 2-3-1-4 (d) 4-1-3-2
1. The Chairman and the Deputy Chairman of the Rajya 100. In the context of Indian economy, ‘Open Market Operations’
Sabha are not the members of that House. refers to
2. While the nominated members of the two Houses of (a) borrowing by scheduled banks from the RBI
the Parliament have no voting right in the presidential (b) lending by commercial banks to industry and trade
election, they have the right to vote in the election of (c) purchase and sale of government securities by the RBI
the Vice President. (d) None of the above
ANSWERS AND EXPLANATIONS
1. (d) Section C of the Forest Dwellers Act provides a 10. (a) Lion-tailed Macaques are found in the mountain
transparent three step procedure for deciding on who forests scattered across three Indian states stated above.
gets rights. The lion-tailed Macaques are endangered as per IUCN.
Firstly, the Gram Sabha makes a recommendation- i.e, 11. (b) Chaityas refer to the halls enclosing the stupas. Chaityas
who has been cultivating land for how long, which were probably constructed to hold large numbers of
minor forest produce is collected ; etc. The gram sabha devotees for prayer. Viharas on the other hand are
plays this role because it is a public body where all constructions built in ancient India in order to provide
people participate and hence is fully democratic and resting places for the wandering Buddhist monks.
transparent. The Gram Sabha’s recommendation goes 12. (c) The concept of Nirvana was originally explained by
through two stages of screening committees- the Taluka Lord Buddha (566-486 BC). The word ‘Nirvana’ comes
and the District levels. from the root meaning ‘to blow out’ and refers to the
2. (c) Aflatoxins are produced by many species of extinguishing of the fires of greed, hatred, and delusion.
Aspergillus, a fungus. Aspergillus Flavus and When these emotional and psychological defilements
Aspergillus Parasiticus are weedy moulds that grow are destroyed by wisdom, the mind becomes free,
on a large number of substrates, in particular under radiant, and joyful and he who has realized the Truth
high moisture conditions. (Nirvana) is the happiest being in the world. He is free
3. (b) The Preamble to the Constitution of India in its from all complexes and obsessions. He does not repent
introductory statement says- “Justice- Social, Economic the past nor does he brood over the future. He lives
and Political” and the Directive Principles of state fully in present. He appreciates and enjoys things in
policies aim to create social and economic condition life in the purest sense without self projections. He is
under which the citizens can lead a good life. They also free from the thirst of becoming and the illusion of self.
aim to establish social and economic democracy Buddism explain Nivrana as a state of bliss or peace.
through a welfare state. 13. (c) Directive Principles of State Policy are guidelines to
4. (b) Electronic waste or E-waste has ferrous and non- the central and State government of India to be kept in
ferrous metals both. Non-ferrous metals like copper, mind while framing laws and policies. DPSPs aim to
aluminium, silver, gold, platinum, palladium etc. The create social and economic conditions under which the
presence of elements like lead, mercury arsenic, citizens can lead a good life. They also aim to establish
cadmium, selenium and hexavalent chromium are social and economic democracy through a welfare state.
classified as hazardous waste. They act as a check on the government. It is a yardstick
5. (d) Acid rain is caused by a chemical reaction that begins in the hands of the people to measure the performance
when compounds of sulphur dioxide and nitrogen of the government. It shall be the duty of the state to
oxide react with molecules in the atmosphere to produce apply these principles in making laws.
acids. 14. (c) In November 1927, the British govt. appointed the
6. (a) A food chain illustrates the order in which a chain of
Indian statutory commission, known popularly after the
organisms feed upon each other.
name of its chairman as the Simon Commission. All the
A food chain is the sequence of who eats whom in a
members of the commission were Englishmen. This
biological community to obtain nutrition.
announcement was greeted with chorus of protest from
Sample:- Grassland Biome
all Indians. What angered them most was the exclusion
GRASS > GRASS HOPPER > RAT> SNAKE > HAWK
of Indians from the commission and the basic notion
7. (d) Through Corbett National Park Ramganga flows
behind this exclusion was that foreigners would discuss
(not Ganga) which is a tributary of Ganges. Through
and decide upon India’s fitness for Self- government.
Silent Valley National Park river Bhavani flows which is
15. (b) The British Govt. sent a delegation to India under Sir
a tributary of Kaveri.
Stafford Cripps, to negotiate with the Indian National
Kaziranga and Manas are both national parks.
Congress a deal to obtain total cooperation during the
8. (b) Nostoc fix atmospheric nitrogen and are used as
war, in return of progressive devolution and distribution
inoculations for paddy crop.
of power from the Crown and the Viceroy to an elected
9. (c) Excretion of urea by animals adds nitrogen to the soil,
Indian Legislature. The talks failed, as they did not
as urea is the main nitrogen containing substance in
address the key demand of a time table of self
the urine of mammals, therefore urea is widely used in
government and of definition of the powers to be
fertilizers as the convenient source of nitrogen.
Death of vegetation adds up in soil organic matters relinquished, essentially making an offer of limited
including humus. As organic matters decompose dominion status that was wholly unacceptable to the
inorganic nitrogen is released into the soil. Indian movement.
16. (c) The Balance of Payments(BoPs) accounts are an “Swadhyay Paramam Tap” According to Jain doctrines
accounting record of all monetary transactions between Penance washes away all the blemishes and purges the
country and the rest of the world. These transactions soul of all karmic matter.
include payment for the country’s exports and imports Also, according to Jainism, Karma is the bane of the
of goods, services, financial capitals and financial soul. Karma not only encompasses the causality of
transfers. transmigration but it is also conceived as an extremely
17. (d) The Reserve Bank of India is the main monetary subtle matter which infiltrates the soul, obscuring its
authority of the country and beside that, in its capacity natural, transparent, pure qualities. Karma is thought
as the central bank, acts as the bank of the national and of as a kind of pollution that taints the soul with various
state governments. Some times it happens that some of colors. Based on its karma, a soul undergoes
the banks closedown due to non recovery of loans or transmigration and reincarnation in various states of
such other issues. In such conditions people have to
existence like heavens or hells or as humans or animals.
suffer as their money is with the bank then. For this
23. (b) Ecological Niche is a term that describes the way of life
reason there is provision for winding up of the banking
of a species. Each species is thought to have a separate
company under the Banking Regulation Act, 1949. The
power of winding up of Bank lies in the hand of Reserve unique niche. The ecological niche describes how an
Bank of India. organism or population responds to the distribution of
18. (d) A tight monetary policy is a course of action undertaken resources and competitors. A niche is the sum of the
by Central bank to constrict spending in an economy, habitat requirements that allow a species to persist and
or to curb inflation when it is rising too fast. The produce offspring.
increased bank rate increases the cost of borrowing 24. (a) The burning of fossil fuel can create another
and effectively reduces its attractiveness. atmospheric pollution problem known as Photochemical
19. (a) Deficit financing refers to the difference between Smog. Photochemical smog is a condition that develops
expenditure and receipts. In public finance, it means when primary pollutants like oxides of nitrogen, Volatile
the govt. is spending more than what it is earning. organic compounds created from fossil fuel combustion
Deficit financing is a necessary evil in a welfare state as interact under the influence of sunlight to produce
the states often fail to generate tax revenue which is secondary pollutants. The major chemical pollutants in
sufficient enough to take care of the expenditure of the Photochemical smog are NO and NO2, VOCs(volatile
state. The basic intention behind deficit financing is to organic compounds), Ozon e(O 3 ) and PAN
provide the necessary impetus to economic growth by (Peroxyacetyl Nitrate). NO2 decreases visibility due to
artificial means. yellowish colour. It also contributes to heart and lung
20. (b) Indus valley people did not possess great palaces and problem. Ozone(O 3) contributes to bronchial
temples rather the civilization was noted for its cities constriction, coughing and wheezing. PAN causes eye
built of brick, roadside drainage system and multistoried irritation, high toxicity to plants.
houses. 25. (c) Calcium is the most common and abundant mineral in
Indus valley people were peace loving. They were never the body. It is important for healthy bones and teeth,
engaged in any war. However speculations have been helps muscle relax and contract, important in nerve
rife that some tectonic forces destroyed the civilization. functioning, blood clotting etc. Sodium is needed for
Some historians are of the view that invasion of Aryans,
proper fluid balance, nerve transmission and muscle
sea level changes, earthquakes might have brought the
contraction.
civilization to its end therefore people employing horse
26. (a) When a money bill returns to the Loksabha with
drawn chariots in warfare is not true. Moreover, Indus
valley seals show swastika, animals which is suggestive amendments made by the Rajyasabha, it is open to
of their religious beliefs. In view of large number of Loksabha to accept or to reject any or all of the
figurines found in Indus valley, some scholars believe recommendations. When the Loksabha chooses to
they worshipped mother goddess symbolizing fertility. accept or decline the money bill with or without the
They worshipped a father God who might be a recommendation, the money bill is deemed passed in
progenitor of the race and was a probably a prototype both houses.
of Siva as the Lord of the Animals. 27. (d) A lieutenant Governor is in charge of a Union Territory
21. (b) Body piercing or getting one’s body tattooed may cost whereas a Governor is in charge of a State. The rank of
one a huge price- Infection of Hepatitis B and C virus is Lt.Governor is present only in the states of Delhi,
absolutely possible. The needle used in the act may Andaman and Nicobar Islands and Puducherry. So in
just be infected with the said virus causing liver disease, the case of a Union Territory specified where there is a
which in its ultimate stage often turns cancerous. This legislative setup, the Chief Minister is appointed by
is deemed to happen if the needle is not properly sterilized. the Lieutenant Governor.
22. (d) The surest way of annihilating Karma is to practice 28. (b) Atlas mountain ranges are situated in the north western
penance. Karma is the bane of the soul and must be part of Africa. They extend almost 2000 km. They pass
ended. through Morocco, Algeria and Tunisia.
29. (b) The Barbara rock cut caves are the oldest rock cut caves. 37. (c) The Attorney General of India has a post parallel to any
They were originally made for the Ajivikas during the minister in Parliament. He can take part in the
Mauryan period (322-185 BCE). The Ellora caves were proceedings of either house. He can be a member of
built between 5th century and 10th century. There were any committee of Parliament. He has the right to speak
34 caves out of which 12 were Buddhist caves, 17 were in the Parliament but he has no right to vote.
Hindu caves and 5 were Jain caves. The proximity of 38. (c) Bagasse is often used as a primary fuel source as it
the caves clearly demonstrates the religious harmony produces sufficient heat energy. Molasses can be used
prevalent at that period of time. for the production of Ethanol. Ethanol is produced by
30. (c) Recombinant DNA technology is the hybridization of the age old technique of fermentation of cereals, grams,
DNA from different sources to achieve desired molasses and other materials with high starch contents.
genotype and phenotype in an organism. The major Molasses is an inexpensive and readily available raw
tools required for rDNA technology are restriction material.
enzymes, cloning vector and competent host. Molasses cannot be used as a synthetic fertilizer as
Restriction enzymes are biological scissors that molasses contain calcium, magnesium and Iron where
recognise and cut DNA at specific points. Eukaryotes as synthetic fertilizers are comprised of NPK i.e.
do not have restriction enzymes. Vectors are cloning
Nitrogen, Phosphorous and Potassium.
vehicles required to tansfer DNA of interest from one
39. (b) Rotation of earth on its axis causes day and night but
organism to another, e.g., bacterial plasmid,
the revolution of earth in an elliptical manner around
bacteriophage, cosmids, YAC and BAC. Competent
the sun causes seasons, equinoxes and solstices.
host is the organism in which desired DNA is introduced
40. (a) The land slopes west ward from central India due to
with the help of vector. Host should have a specific
DNA sequence called origin of repletion (Oric) which deposition of alluvial fan. The rift valley which is
responsible for initiating replication E.g. Yeast, E. coli, occupied by the Narmada is one of the reasons of
plant and animal cells. Narmada flowing to the west. The rift is formed due to
31. (b) The tradesmen had to pay light duties at ferries and complex natural processes leading to either subsidence
barriers. After paying the revenue they could go to and or upliftment of the earth’s surface. The area is
fro to barter their merchandise. The punishment for seismically active and has seen major upheavals. Such
social offences according to the traveller’s account, movements reshape the landscape. There is
were to cut off the nose, or an ear or a hand. Minor readjustment of slopes and rivers either start bringing
offences were dealt with fines. And to determine guilt more material or change course.
or innocence, ordeals by fire, water and poison were 41. (c) The percentage of freshwater that exists on earth is 3%
the instruments which were used. out of which almost 70% is icecaps and glaciers. And
32. (a) Star tortoise is found in India in the dry and scrub the remaining 30% is the ground water. Lakes, rivers
forests. Pygmy Hog is an endangered species found in and swamps are approximately 10% of that 30% of
Assam. Only 150 animals are left. Monitor Lizard is ground water.
found in India, Sri Lanka and Pakistan. Spider Monkey 42. (a) Nokrek Biosphere Reserve is situated in Garo Hills in
is the inhabitant of tropical forests of Central and South Meghalaya. Logtak Lake is in Manipur. Barail Range is
America. in Assam. Though Namdapha National Park and Dafla
33. (a) Drinking water in some parts of India has contaminants Hill both are in Arunachal Pradesh, the two are separate
like Arsenic, Fluoride other than many other entities.
contaminants. The sources of Arsenic are run off from 43. (d) From electromagnetic radiation to revolution of the earth,
orchards. The sources of fluoride are erosion of natural
everything is responsible for bringing dynamic changes
deposits, discharge from fertilizers and aluminum
on the surface of the earth. For example: Electromagnetic
factories.
34. (c) The member of the constituent assembly were elected radiation brings changes in the field of microwaves,
indirectly by the members of the individual provincial wavelengths of radio, UV rays, infra red rays, X rays
legislative assemblies. and gamma rays. Geothermal energy is the heat received
35. (b) Sea cow is a mammal. It is a herbivore and it grows up from the earth’s core. This heat continuously flows
to 9 meters. It looks like a large seal. Sea horse is a fish outward. It transfers to the surrounding layers of rock,
not a mammal. Sea lion is a mammal. It is a carnivore. It the mantle. When temperature and pressure becomes
is also known as eared seal. very high some mantle rocks melt becoming magma. It
36. (d) An amendment to the constitution of India is introduced then either comes out as lava or heat up the nearby
as a bill in the Parliament. It then must be approved by
rocks and water which comes out as hot springs or
both the houses of Parliament. The amendments must
geysers. Gravitational force is constantly working on
then be ratified by the legislatures of at least one half of
the states (not all the states). Once all these stages are all physical bodies. It is giving weights to objects with
complete the amendment is bound to receive the assent mass and causes them to fall to the ground when
of the President of India. dropped. Plate movement is a dynamic change on the
surface of the earth. It explains many aspects of the 54. (a) The pose ‘Tribhanga’ is the favourite posture of Lord
interrelationship of volcanoes, earthquakes, climate Krishna. We have often seen Lord Krishna standing in
change, and the evolution of life itself. Everything about tribhanga posture before his cow ‘Kamdhenu’ or
our planet is related either directly or indirectly to plate whenever he is playing his flute. He is often called
tectonic. Rotation causes day and night. Revolution Tribhana Murari.
causes seasons, change in the length of day and night. 55. (c) Annie Besant had formed and led the Home Rule
44. (d) National Development council is not a constitutional Movement as she was influenced by the Irish Home
body. It is an extra-constitutional body. Planning Rule League. She also became the President of Indian
Commission is a non-constitutional and non-statutory National Congress in 1917. But she was not the founder
body. It was created by the Govt. of India in 1950 by a of the Theosophical Society; rather she was the second
resolution. Zonal councils have been recognized in the President of the society. The founder was Henry Steel
74th Constitutional Amendment Act of 1992. Olcott.
45. (a) The Tebhaga movement is a peasant movement in the 56. (c) Ilbert bill exempted British subjects from trial by Indian
history of Bengal and India. It was a movement of the magistrates and in cases involving death or
peasants who demanded two-third share of their transportation they could only be tried by a high court.
produce for themselves and one-third share to the This proposal provoked furious protests by the Indians.
landlord. 57. (d) General Price rise may be caused by an increase in the
46. (d) Parliament has exclusive power to make law with respect money supply as the real value of the money reduces.
to any of the matters enumerated with the Union List. The higher aggregate price level will reduce the
According to entry no 14 in the Union List it reads- purchasing power subsequently reducing the consumer
‘entering into treaties and agreements with foreign spending. Effective demand also increases when there
contries and implementing of treaties, agreement and is less purchasing power due to inflation.
convention with foreign countries’. 58. (b) In India’s foreign exchange reserve, it includes foreign
47. (c) The trees cannot replace the grasses as a part of an currency deposits, bonds, gold reserves, SDRs and IMF
ecological succession because of water limits and fire. reserve position. Foreign exchange reserves are an
48. (d) The plant life in an ecosystem support the important part of the international investment position
animal life and vice versa. So the sequence of of a country.
ecosystems in the order of decreasing productivity is 59. (d) Extremely high rates of inflation are generally associated
Ocean>mangroves>lakes>grasslands. Ocean or marine with high rates of money growth. It is often the result
ecosystems usually have a large biodiversity. of financing large deficits by printing money.
Mangrove is a part of that ecosystem. So are lakes. 60. (c) The quantity of money demanded is inversely related
Grasslands are areas where vegetation is dominated by to the interest rate.
grasses hence biodiversity is in the decline. 61. (c) Fruits respire. They give out ethylene which helps in
49. (b) Contour Bunding is one of the simple method of soil ripening. When fruits are kept in cold storage the rate
and water conservation. This technique is used at of ethylene production can be controlled and shelf life
places where the land is sloppy. Due to slope, soil and of fruits can be increased.
nutrients erode fast which makes agriculture on this 62. (c) Gharial is critically endangered according to IUCN.
land very uneconomical. To adopt this technique the Overhunting for skin and trophies, habitat loss due to
agriculture fields contours are marked and then the construction of dams and barrages has been the reason
bunds are taken along the contours. for their decline. Leather back turtles are endangered
50. (d) PESA Act does not identify the freedom of tribal people due to human carelessness. Swamp deer occupies a
from exploitation as its objectives, but it automatically place in the list of the endangered species of the world.
becomes a byproduct of its objectives. Deforestation, draining of swamps and marshes for
51. (d) Priority sector lending constitutes the lending to farming has led to the destruction of their natural habitat.
–agriculture, micro and small enterprises, micro credit, 63. (d) The function of ball bearing in any machinery is to
education, housing and weaker sections. reduce the friction. It has nothing to do with contact
52. (d) Thermal power plants use water to turn into steam area.
(heat energy) to generate electricity(electrical energy). 64. (c) Size of the Sun at dusk is an optical illusion because of
53. (a) To reap the benefits of demographic dividend, skills atmospheric refraction. Colour of the sun appears
have to be developed because a relatively larger portion yellow due to scattering whereas pure sunlight is white
of population fall under the category of productive in colour. Twinkling of stars is an optical illusion. Air
labour force when there is a demographic dividend. whirlpools make the stars twinkle.
65. (a) Water droplets serve as prism and refraction of light main purpose of RRB’s is to mobilize financial resources
takes place. The refraction of light at two boundaries of from rural / semi-urban areas and grant loans and
a raindrop results in dispersion of light into a spectrum advances mostly to small and marginal farmers,
of colours. agricultural laborers and rural artisans.
66. (c) During a transplant the fine roots are destroyed due to 76. (b) The committee consists of 15 members of Lok sabha
the jostling during the uprooting process. These root not 25 members. The function of the committee is to
hairs come under a shock. The plant which has been examine the accounts showing the appropriation of the
transplanted either takes time to readjust to the new sums granted by Parliament to meet the expenditure of
soil or dries up and dies due to the root hair loss. the government of India and such other accounts laid
67. (c) A country’s economic growth is reflected through before the House as the committee may think fit.
capital formation, which in turn encourages private Apart from the Reports of the Comptroller and
enterprises in enhancing the growth of a country’s Auditor General of India on Appropriation Accounts
economy. of the Union Government, the Committee also examines
68. (a) Viruses have no metabolic enzymes and cannot generate the various Audit Reports of the Comptroller and
their own energy. Viruses require a growth medium Auditor General on revenue receipts, expenditure by
containing living cell therefore cannot be cultured in various Ministries/Departments of Government and
any synthetic medium. Viruses are transmitted from one accounts of autonomous bodies.
organism to another not only by biological vectors but 77. (b) Guru Nanak Dev was born in 1469. Babur defeated
also by mechanical vectors. Mechanical vector like an Ibrahim Lodi in 1526. When Babur took over and
arthropod vecor which transmit an infective organism established the Mughal dynasty, Nanak was travelling
from one host to another but it is not essential to the to different parts of India and preaching Sikhism. Babur
life of cycle of the parasite. once met Guru Nanak during one of his travel.
69. (d) To inhibit water loss or to conserve water most of the 78. (b) Bacteria and Fungi both are known as decomposer
desert species have waxy leaves that keep them water organisms. Bacteria are important decomposers; they
proof when stomata are closed. Water is further can break down any kind of organic matters. Fungi are
conserved by reducing surface area so most succulents primary decomposers, they not only decompose the
have few leaves or no leaves. Some desert plants have surface organisms but they can also penetrate deep
thorns instead of leaves. Thorns do not let the water into the organic matters.
go out. 79. (c) The temperature is just right for them to survive.
70. (a) Gravity is the feeble force among the four. 80. (d) The canopy is the primary layer of the forest forming a
71. (a) The discovery of Higgs boson is important because it roof over the two remaining layers. The densest of the
explains why sub atomic particles have mass. biodiversity is found here along with a large variety of
According to the scientists, Higgs boson is the only epiphytes.
particle which explains how the basic building blocks 81. (b) Capital Account comprises of Foreign loans, Foreign
of matter interact. Direct Investment and Portfolio Investment. Capital
72. (d) Mycorrhizal associations play vital role in plant Account is the net result of public and private
nutrition. They greatly increase the efficiency of nutrient international investments flowing in and out of a
and water uptake; enhance resistance to pathogens, country. Portfolio investment is the buying of shares
and buffer plant species against several environmental and bonds. FDI is the investments by foreigners in a
stresses and drought resistance. Mycorrhizal also country or the citizens investing in foreign countries.
improve plant growth and survival in soils contaminated 82. (b) Ajanta Caves have mural paintings in caves 1, 2, 16 and
by heavy metals. 17. Some of the paintings were commissioned by
73. (b) The National Development Council includes the Prime Harisena of Vakataka dynasty. The theme of the
Minister, Union ministers, Chief ministers of all states, paintings was Jataka tales. Lepakshi Temple is
administrative heads of the Union Territories and renowned for being one of the best repositories of mural
members of the planning commission . paintings of the Vijaynagar kings. Sanchi Stupa has
74. (d) National Income is the money value of all the final goods many beautiful sculptures but not mural paintings.
and services produced by a country during a period of 83. (c) Sankhya yoga believes that self –knowledge leads to
one year. National Income consists of a collection of liberation. Almost all the rest of philosophies of India
different types of goods and services of different types. adopted Sankhya as its main base. Sankhya yoga also
75. (c) Land development bank started financing long term believes in rebirth. According to Sankhya darshan each
loan for more significant rural development activities purusha experiences bhoga, apavarga and takes birth
like rural and cottage industries, rural artisans etc. The repeatedly until kaivalya.(moksa).
84. (a) Minister/ministers can be removed by issuing no 94. (b) Out of 52 items on the concurrent list, Economic and
confidence motion in the parliament. All cabinet Social Planning is one Article 40 of the Constitution
members are mandated by the constitution to be the which enshrines one of the Directive Principles of
member of either house of the parliament of India. State Policy lays down that the State shall take steps
Cabinet is headed by the cabinet secretary not by the to organize village panchayats and endow them with
Head of the State. such powers and authority as may be necessary to
85. (a) The first statement is correct as we all know. One major enable th em to function as units of self
factor affecting the distribution of the temperature of government. Planning for economic development and
Earth is distribution of Land and Oceans. Since there
social justice is one such power given to village
is more land in Northern Hemisphere and more
panchayats.
waters in Southern hemisphere and there is a big
difference between the specific heat of land and 95. (b) The nominated members of the Rajya Sabha have the
water; the loss of heat from the continents is bigger right to vote in the election of the Vice President so far
than the oceans. The continents get heated faster none from them has been inducted in the Council of
and get cooled faster in comparison to the Oceans. Ministers.
This is the reason that the temperatures of the Oceans 96. (c) The National Legal Services Authority (NALSA) has
are moderate while that of continents is extreme. The been constituted under the Legal Services Authorities
moderating effect on temperature of the land due to Act, 1987 to provide free Legal Services to the weaker
proximity of the seas is called Maritime influence. The sections of the society and to organize Lok Adalats for
increasing effect on temperature of the land at interior amicable settlement of disputes. In every state, State
of the continents is called Continental Influence. Legal Services Authority has been constituted to give
86. (a) Indian coal has high ash content and low calorific value. effect to the policies and directions of the NALSA and
It has low sulphur and low phosphorous content but to give free legal services to the people and conduct
high ash fusion temperature. Lok Adalats in the State.
87. (c) Laterite soil is rusty red in colour due to iron oxide
97. (d) Thunderstorms result from the rapid upward
present in it. In the lateritic soil cashews and tapiocas
movement of warm, moist air. They can occur inside
can be grown.
warm, moist air masses and at fronts. As the warm,
88. (a) Dharwar rocks are non fossilliferous rather they are
metalliferous. They bear out gold, iron ore, manganese moist air moves upward, it cools, condenses, and forms
mica, cobalt, chromium, copper, tungsten, lead, nickel, cumulonimbus clouds that can reach heights of over
precious stones and budding stones. Kodarma is a store 20 km (12.45 miles). The thunderstorms are associated
house of mica and Gondwana beds have natural gases. with the cumulonimbus clouds. These clouds normally
89. (c) Rice is the main kharif crop and groundnut and cotton form on warm sunny days but they can also be found
are also the kharif crops. on cold front. But this question is asking about the
90. (b) The central Asian steppes run through Kazakhstan, Thunder i.e. the sound produced. The lightning
Turkmenistan, Uzbekistan and Mongolia. The climate generates between 100 million and 1 billion volts of
here is harsh with dust storms, little to no rainfall and electricity and can heat the air to around 50K°F. The
temperature ranging from – 4 to 50 degree Celsius rapid expansion causes the shock waves. Thunder
91. ( a) Those who benefit from higher inflation are debtors happens because the lightning would heat the air at
and those who suffer from it are creditors. If one has huge temperatures and the air expands so fast that it
substantial debt, each rupee one has to repay would be make a loud clap of thunder.
worth less than when it was borrowed. In this way, one 98. (a) Limbu tribe is from Sikkim and Dongaria Kondh tribe is
pays back less in real terms.
from Odisha,.( Karbi is from Assam and Bonda is in
92. (c) Disguised unemployment is a situation when people
Odisha.)
do not have productive full-time employment, but are
not counted in the official unemployment statistics. 99. (d) Currency/cash is the most liquid ,then the demand
93. (c) The Council of Ministers shall be collectively deposits (current accounts), then the saving deposits
responsible to the parliament; the Prime minister shall with bank and finally the least liquid is the time deposits
communicate to the president about the proposals for with the bank (fixed deposits).
legislation but the union. If a President were to dismiss 100. (c) It is an activity by a central bank(RBI) to buy or sell
the Council of Ministers on his or her own initiative, it government securities. The aim of open market
might trigger a constitutional crisis. Thus, in practice, operations is to manipulate the short term interest
the Council of Ministers cannot be dismissed as long rate and the supply of base money in an economy, and
as it commands the support of a majority in the Lok indirectly control the total money supply.
Sabha.
INSTRUCTIONS
1. This Test Booklet contains 80 items (questions). Each item comprises four responses (answers). You will select
the response which you want to mark on the Answer sheet. In case, you feel that there is more than one correct
response, mark the response which you consider the best. In any case, choose ONLY ONE response for each
item.
2. All items carry equal marks.
3. Penalty for wrong answers:
THERE WILL BE PENALTY FOR WRONG ANSWERS MARKED BY THE CANDIDATE EXCEPT FOR
QUESTIONS FROM 75 TO 80, WHICH DO NOT CARRY ANY PENALTY FOR WRONG ANSWER.
(i) There are four alternatives for the answer to every question. For each question which has a penalty for which a
wrong answer has been given by the candidate, one-third of the marks assigned to that question will be
deducted as penalty.
(ii) If a candidate gives more than one answer, it will be treated as a wrong answer even if one of the given answers
happens to be correct and there will be same penalty as above to that question, if it has a penalty.
(iii) If a question is left blank, i.e., no answer is given by the candidate, there will be no penalty for that question.

Directions for the following 2 (two) items: 3. In a rare coin collection, there is one gold coin for every three
Read the following passage and answer the two items that follow. non-gold coins. 10 more gold coins are added to the collection
Your answers to these items should be based on the passage and the ratio of gold coins to non-gold coins would be 1 : 2.
only. Based on the information, the total number of coins in the
collection now becomes
PASSAGE (a) 90 (b) 80
Ecological research over the last quarter of the century has (c) 60 (d) 50
established the deleterious effects of habitat fragmentation due to 4. A gardener has 1000 plants. He wants to plant them in such
mining, highways and such other intrusions on forests. When a a way that the number of rows and the number of columns
large block of forests gets fragmented into smaller bits, the edges remains the same. What is the minimum number of plants
of all these bits come into contact with human activities resulting that he needs more for this purpose?
in the degradation of the entire forests. Continuity of forested (a) 14 (b) 24
landscapes and corridors gets disrupted affecting several (c) 32 (d) 34
extinction-prone species of wildlife. Habitat fragmentation is 5. A sum of ` 700 has to be used to give seven cash prizes to
therefore considered as the most serious threat to biodiversity the students of a school for their overall academic
conservation. Ad hoc grants of forest lands to mining companies performance. If each prize is ` 20 less than its preceding
coupled with rampant illegal mining is aggravating this threat. prize, what is the least value of the prize?
1. What is the central focus of this passage ? (a) ` 30 (b) ` 40
(a) Illegal mining in forests (c) ` 60 (d) ` 80
(b) Extinction of wildlife 6. Out of 120 applications for a post, 70 are male and 80 have a
(c) Conservation of nature driver’s license. What is the ratio between the minimum to
(d) Disruption of habitat maximum number of males having driver’s license?
2. What is the purpose of maintaining the continuity of forested (a) 1 to 2 (b) 2 to 3
landscapes and corridors? (c) 3 to 7 (d) 5 to 7
1. Preservation of biodiversity. 7. In a garrison, there was food for 1000 soldiers for one month.
2. Management of mineral resources. After 10 days, 1000 more soldiers joined the garrison. How
3. Grant of forest lands for human activities. long would the soldiers be able to carry on with the remaining
Select the correct answer using the codes given below. food?
(a) 1 only (b) 1 and 2 (a) 25 days (b) 20 days
(c) 2 and 3 (d) 1, 2 and 3 (c) 15 days (d) 10 days
8. The tank-full pertrol in Arun’s motor-cycle lasts for 10 days. 10. According to the passage, why should the discharge of
If he starts using 25% more everyday, how many days will agricultural slurry into watercourses be restricted?
the tank-full pertrol last? 1. Losing nutrients in this way is not a good practice
(a) 5 (b) 6 economically.
(c) 7 (d) 8 2. Watercourses do not contain the microorganisms that
9. A person can walk a certain distance and drive back in six can decompose organic components of agricultural
slurry.
hours. He can also walk both ways in 10 hours. How much
3. The discharge may lead to the eutrophication of water
time will he take to drive both ways?
bodies.
(a) Two hours Select the correct answer using the codes given below:
(b) Two and a half hours (a) 1 only (b) 2 and 3 only
(c) Five and a half hours (c) 1 and 3 only (d) 1, 2 and 3
(d) Four hours 11. The passage refers to the conversion of “pollutant to
Directions for the following 7 (seven) items: fertilizer”. What is pollutant and what is fertilizer in this
Read the following two passages and answer the items that follow context?
each passage. Your answers to these items should be based on (a) Decomposed organic component of slurry is pollutant
the passage only. and microorganisms in soil constitute fertilizer.
(b) Discharged agricultural slurry is pollutant and
PASSAGE-1 decomposed slurry in soil is fertilizer.
The law in many parts of the world increasingly restricts the (c) Sprayed slurry is pollutant and watercourses is fertilizer.
discharge of agricultural slurry into watercourses. The simplest (d) None of the above expressions is correct in this context.
and often the most economically sound practice returns the material 12. According to the passage, what are the effects of
to the land as semisolid manure or as sprayed slurry. This dilutes indiscriminate use of fertilizers?
its concentration in the environment to what might have occurred 1. Addition of pollutants to the soil and water.
in a more primitive and sustainable types of agriculture and 2. Destruction of decomposer microorganism in soil.
converts pollutant into fertilizer. Soil microoganisms decompose 3. Nutrient enrichment of water bodies.
the organic components of sewage and slurry and most of the 4. Creation of algal blooms.
mineral nutrients become available to be absorbed again by the Select the correct answer from the codes given below:
vegetation. (a) 1, 2 and 3 only (b) 1 , 3 and 4 only
The excess input of nutrients, both nitrogen and phosphorus (c) 2 and 4 only (d) 1, 2, 3 and 4
– based, agricultural runoff (and human sewage) has caused many 13. What is/are the characteristics of a water body with cultural
eutrophication?
‘healthy’ oligotrophic lakes (low nutrient concentrations, low
1. Loss of ecosystem services
plant productivity with abundant water weeds, and clear water) to
2. Loss of flora and fauna
change to eutrophic condition where high nutrient inputs lead to 3. Loss of mineral nutrients
high phytoplankton productivity (sometimes dominated by bloom- Select the correct answer using the code given below:
forming toxic species). This makes the water turbid, eliminates (a) 1 only (b) 1 and 2 only
large plants and, in the worst situations, leads to anoxia and fish (c) 2 and 3 only (d) 1, 2 and 3
kills; so called cultural eutrophication. Thus, important ecosystem 14. What is the central theme of this passage?
services are lost, including the provisioning service of wild-caught (a) Appropriate legislation is essential to protect the
fish and the cultural services associated with recreation. environment.
The process of cultural eutrophication of lakes has been (b) Modern agriculture is responsible for the destruction
understood for some time. But only recently did scientists notice of environment.
huge ‘dead zones’ in the oceans near river outlets., particularly (c) Improper waste disposal from agriculture can destroy
those draining large catchment areas such as the Mississippi in the aquatic ecosystems.
North America and the Yangtze in China. The nutrient-enriched (d) Use of chemical fertilizers is indesirable in agriculture.
water flows through streams, rivers and lakes, and eventually to PASSAGE-2
the estuary and ocean where the ecological impact may be huge, The miseries of the world cannot be cured by physical help
killing virtually all invertebrates and fish in areas up to 70,000 km2 only. Until man’s nature changes, his physical needs will always
in extent. More than 150 sea areas worldwide are now regularly arise, and miseries will always be felt, and no amount of physical
starved of oxygen as a result of decomposition of algal blooms, help will remove them completely. The only solution of the problem
is to make mankind pure. Ignorance is the mother of evil and of all
fuelled particularly by nitrogen from agricultural runoff of fertilizers
the misery we see. Let men have light, let them be pure and
and sewage from large cities. Oceanic dead zones are typically
spiritually strong and educated; then alone will misery cease in
associated with industrialized nations and usually lie off countries the world. We may convert every house in the country into a
that subsidize their agriculture, encouraging farmers to increases charitable asylum, we may fill the land with hospitals, but human
productivity and use more fertilizer. misery will continue until man’s character changes.
15. According to the passage, which of the following statements 19. Problem figures:
is most likely to be true as the reason for man’s miseries?
(a) The poor economic and social conditions prevailing in
society.
(b) The refusal on the part of man to change his character.
(c) The absence of physical and material help from his
society.
(d) Ever increasing physical needs due to changing social Answer figures:
structure.
16. With reference to the passage, the following assumptions
have been made”
1. The author gives primary importance to physical and
material help in eradicating human misery.
2. Charitable homes, hospitals, etc. can remove human (a) (b) (c) (d)
misery to a great extent.
Which of the assumption is/are valid? 20. Consider the following diagrams:
x men, working at constant speed, do a certain job in y days.
(a) 1 only (b) 2 only
Which one of these diagrams shows the relation between
(c) Both 1 and 2 (d) Neither 1 nor 2
x and y?
17. Consider the following figures 1, 2, 3 and 4: y

Days

1 2 3 4
In the figures from 1 to 4 above, two symbols are shown to 0 x
Men
change their position in a regular direction. Following the
diagram I
same sequence, which one of the following will appear at the y
fifth stage?
Days

(a) (b)

0 x
Men
(c) (d) diagram II
y

Directions for the following 2 (two) items:


Days

In each item, there are two sets of figures, first four figures named
Problem figure sand next four figures named Answer figures
indicated as (a), (b), (c) and (d). The problem figures follow a
particular sequence. In accordance with the same, which one of 0 x
the four answer figures should appear as the fifth figure? Men
diagram III
18. Problem figures: y
Days

Answer figures:
0 x
Men

diagram IV
(a) diagram I (b) diagram II
(c) diagram III (d) diagram IV
(a) (b) (c) (d)
21. Consider the following matrix: Direction for the following 5 (five) items:
Study the two figures given below and answer the five items that
3 370 7
follow:
2 224 6
36
1 730 X 32
28
What is the number at ‘X’ in the above matrix?
24
(a) 5 (b) 8 20 Male
(c) 9 (d) 11 16 Female
22. Four cars are hired at the rate of ` 6 per km plus the cost of 12
diesel at ` 40 a litre. In this context, consider the details given 8
in the following table: 4
0
Mileage Total

GY
S

NY

S
IC
Car Hours

IC
TR
IC

LO
Payment( `)

YS

TA

OM
(km / l )

IS
AT

HO
PH

BO
EM

ON
M

YC
HE

CH
A 8 20 2120

EC
PS
AT
M
B 10 25 1950
Figure 1: Number of Professors in selected disciplines in a
C 9 24 2064 University by sex
D 11 22 1812
20%
Which car maintained the maximum average speed?
(a) Car A (b) Car B 40% 35 - 44
(c) Car C (d) Car D
10% 25 - 34
23. Examine the following three figures in which the numbers
follow a specific pattern: 60 - 65
45 - 59
84 81
30%
14 12 18 9

Figure 2: Age of Physics Professors


88 25. How many Physics professors belong to the age group
35 – 44?
? 11 (a) 18 (b) 16
(c) 14 (d) 12
The missing number (?) in the third figure above is
(a) 7 (b) 16 26. Which one of the following disciplines has the highest ratio
(c) 21 (d) 28 of males to females?
24. A cube has six numbers marked 1, 2, 3, 4, 5 and 6 on its faces. (a) Physics (b) Mathematics
Three views of the cube are shown below: (c) Chemistry (d) Economics
27. What percentage of all Psychology professors are females?
1 3 3
(a) 40% (b) 50%
4 2 6
6 1 5 (c) 60% (d) 70%
28. If the number of female Physics professors in the age group
25 – 34 equals 25% of all the Physics professors in that age
What possible numbers can exist on the two faces marked group, then what is the number of male Physics professors in
A and B , respectively on the cube? the age group 25 – 34?
(a) 9 (b) 6
B (c) 3 (d) 2
5 29. If the Psychology professors in the University constitute
A 2% of all the professors in the University, then what is the
number of professors in the University?
(a) 2 and 3 (b) 6 and 1 (a) 400 (b) 500
(c) 1 and 4 (d) 3 and 1 (c) 600 (d) 700
30. Consider the following figures: It cannot, of course, be doubted that the institutional structure
of the contemporary practice of democracy is largely the product
of European and American experience over the last few centuries.
This is extremely important to recognize since these development
in institutional formats were immensely innovative and ultimately
effective. There can be little doubt that there is a major ‘Western’
achievement here.
(1) (2) (3) (4) 31. Which of the following is closest to the view of democracy
as mentioned in the above passage?
(a) The subject of democracy is a muddle due to a desire to

(5) (6)
? (7)
portray it as a Western concept, ‘alien’ to non-Western
countries.
(b) The language of imposition of democracy is
inappropriate. There is, however, a need to consider
this concept in the backdrop of culture of ‘own ways’
Which one of the following figures would logically come in of non-Western society.
the 7th position indicated above by a question mark? (c) While democracy is not essentially a Western idea
belonging exclusively to the West, the institutional
structure of current democratic practices has been their
contribution.
(a) (b) (d) None of the statements (a), (b) and (c) given above is
correct.
32. With reference to the passage, the following assumption have
been made:
1. Many of the non-Western countries are unable to have
(c) (d) democracy because they take democracy to be a
specialized cultural product of the West.
2. Western countries are always trying to impose
Directions for the following 8 (eight) items: democracy on non-Western countries.
Read the following four passages and answer the items that follow Which of the above is/are valid assumption/assumptions?
each passage. Your answer to these items should be based on the (a) 1 only (b) 1 only
passage only. (c) Both 1 and 2 (d) Neither 1 nor 2
PASSAGE-1
The subject of democracy has become severely muddled PASSAGE-2
because of the way the rhetoric surrounding it has been used in Corporate governance is based on principles such as
recent years. There is, increasingly, an oddly confused dichotomy conducting the business with all integrity and fairness, being
between those who want to ‘impose’ democracy on countries in transparent with regard to all transactions, making all the necessary
the non-Western world (in these countries’ ‘own interest’, of disclosures and decisions, complying with all the laws of the land,
course) and those who are opposed to such ‘imposition’ (because accountability and responsibility towards the stakeholders and
of the respect for the countries’ ‘own ways’). But the entire commitment to conducting business in an ethical manner. Another
language of ‘imposition’, used by both sides, is extraordinarily point which is highlighted on corporate governance is the need
inappropriate since it makes the implicit assumption that democracy for those in control to be able to distinguish between what are
belongs exclusively to the West, taking it to be a quintessentialy personal and corporate funds while managing a company.
‘Wester’ idea which has originated and flourished only in the Fundamentally, there is a level of confidence that is associated
West. with a company that is known to have good corporate governance.
But the thesis and the pessimism it generates about the possibility The presence of an active group of independent directors on the
of democratic practice in the world would be extremely hard to board contributes a great deal towards ensuring confidence in the
justify. There were several experiments in local democracy in market. Corporate governance is known to be one of the criteria
ancient India. Indeed, in understanding the roots of democracy in that foreign institutional investors are increasingly depending on
the world, we have to take an interest in the history of people when deciding on which companies to invest in. It is also known
participation and public reasoning in different parts of the world. to have a positive influence on the share price of the company.
We have to look beyond thinking of democracy only in terms of Having a clean image on the corporate governance front could
European and American evolution. We would fail to understand also make it easier for companies to source capital at more
the pervasive demands for participatory living, on which Aristotle reasonable costs. Unfortunately, corporate governance often
spoke with far-reaching insight, if we take democracy to be a kind becomes the centre of discussion only after the exposure of a
of a specialized cultural product of the West. large scam.
33. According to the passage, which of the following should be input use designed to reduce risk rather than to maximize income.
the practice/practices in good corporate governance? Farmers adopt a number of strategies to manage and cope with
1. Companies should always comply with labour and tax agricultural risks. These include practices like crop and field
laws of the land. diversification, non-farm ‘employment storage of stocks and
2. Every company in the country should have a strategic migration of family members. There are also institutions
government representative as one of the independent ranging from share tenancy to kinship, extended family and informal
directors on the board to ensure transparency. credit agencies. One major obstacle to risk sharing by farmers is
3. The manager of a company should never invest his that the same type of risks can affect a large number of farmers in
personal funds in the company. the region. Empirical studies show that the traditional methods are
Select the correct answer using the codes given below: not adequate. Hence there is a need for policy interventions,
(a) 1 only (b) 2 and 3 only
especially measures that cut across geographical regions.
(c) 1 and 3 only (d) 1, 2 and 3
Polices may aim at tackling agricultural risks directly or indirectly.
34. According to the passage, which of the following is/are the
Examples of risk-specific policies are crop insurance, price
major benefit/benefits of good corporate governance?
stabilization and the development of varieties resistant to pests
1. Good corporate governance leads to increase in share
price of the company. and diseases. Policies which affect risk indirectly are irrigation,
2. A company with good corporate governance always subsidized credit and access to information. No single risk-specific
increases its business turnover rapidly. policy is sufficient to reduce risk and is without side-effects,
3. Good corporate governance is the main criterion for whereas policies not specific to risk influence the general situation
foreign institutional investors when they decide to buy and affect risks only indirectly. Crop insurance, as a policy measure
a company. to tackle agricultural risk directly, deserves careful consideration
Select the correct answer using the codes given below: in the Indian context and in many other developing countries –
(a) 1 only (b) 2 and 3 only because the majority of farmers depend on rain-fed agriculture
(c) 1 and 3 only (d) 1, 2 and 3 and in many areas yield variability is the predominant cause of
their income instability.
PASSAGE-3 37. The need for policy intervention mitigate risks in agriculture
Malnutrition most commonly occurs between the ages of six is because
months and two years. This happens despite the child’s food (a) farmers are extremely risk-averse.
requirements being less than that of an older child. Malnutrition (b) farmers do not know how to mitigate risks.
is often attributed to poverty, but it has been found that even in (c) the methods adopted by farmers and existing risk
households where adults eat adequate quantities of food, more sharing institutions are not adequate.
than 50 per cent of children-under-five do not consume enough (d) majority of farmers depend on rain-fed agriculture.
food. The child’s dependence on someone else to feed him/her is 38. Which of the following observations emerges from the above
primarily responsible for the malnutrition. Very often the mother is passage?
working and the responsibility of feeding the young child is left to (a) One can identify a single policy that can reduce risk
an older sibling. It is therefore crucial to increase awareness without any side-effect.
regarding the child’s food needs and how to satisfy them. (b) No single task-specific policy is sufficient to reduce
35. According to the passage, malnutrition in children can be agricultural risk.
reduced. (c) Policies which affect risk indirectly can eliminate it.
(a) if the children have regular intake of food (d) Government’s policy intervention can mitigate
(b) after they cross the age of five.
agricultural risk completely.
(c) if the food needs of younger children are known.
39. Consider the following statements:
(d) if the responsibility of feeding younger children is given
(i) A primary group is relatively smaller in size.
to adults.
(ii) Intimacy is an essential characteristic of a primary group.
36. According to the author, poverty is not the main cause of
malnutrition, but the fact that (iii) A family may be an example of a primary group.
1. taking care of younger ones is not a priority for working In the light of the above statements, which one of the following
mothers. is true?
2. awareness of nutritional needs is not propagated by (a) All families are primary groups.
the Public Health authorities. (b) All primary groups are families.
Select the correct answer using the codes given below: (c) A group of smaller size is always a primary group.
(a) 1 only (b) 2 only (d) Members of a primary group know each other intimately.
(c) Both 1 and 2 (d) Neither 1 nor 2 40. Four friends, A, B, C and D distributed some money among
themselves in such a manner that A gets one less than B, C
PASSAGE-4 gets 5 more than D, D gets 3 more than B. Who gets the
A number of empirical studies find that farmers are risk-averse, smallest amount?
though only moderately in many cases. There is also evidence to (a) A (b) B
show that farmers’ risk aversion results in cropping patterns and (c) C (d) D
Directions for the following 4 (four) items: Which of the following is the most acceptable group of people
Read the following statements and answer the four items that that can be selected by the music director?
follow: (a) Rohit, Shobha, Kunal and Kaushal
(b) Tanya, Kaushal, Shobha and Rohit
Five cities P, Q, R, S and T are connected by different modes (c) Tanya, Mukesh, Kunal and Jaswant
of transport as follows: (d) Shobha, Tanya, Rohit and Mukesh
P and Q are connected by boat as well as rail. 49. Five people A, B, C, D and E are seated about a round table.
S are R connected by bus and boat. Every chair is spaced equidistant from adjacent chairs.
Q and T are connected by air only. (i) C is seated next to A.
P and R are connected by boat only. (ii) A is seated two seats from D.
T and R are connected by rail and bus. (iii) B is not seated next to A.
41. Which mode of transport would help one to reach R starting Which of the following must be true?
from Q, but without changing the mode of transport? (I) D is seated next to B.
(a) Boat (b) Rail (II) E is seated next to A.
(c) Bus (d) Air Select the correct answer from the codes given below:
42. If a person visits each of the places starting from P and gets (a) I only (b) II only
back to P, which of the following places must he visit twice? (c) Both I and II (d) Neither I nor II
(a) Q (b) R Directions for the following 3 (three) items:
(c) S (d) T Examine carefully the following statements and answer the three
43. Which one of the following pairs of cities is connected by items that follow:
any of the routes directly without going to any other city? Out of four friends A, B, C and D.
(a) P and T (b) T and S A and B play football and cricket.
(c) Q and R (d) None of these B and C play cricket and hockey.
44. Between which two cities among the pairs of cities given A and D play basketball and football.
below are there maximum travel options available? C and D play hockey and basketball.
(a) Q and S (b) P are R 50. Who does not play hockey?
(c) P and T (d) Q and R (a) D (b) C
(c) B (d) A
Directions for the following 3 (three) items: 51. Who plays football, basketball and hockey?
Read the following passage and answer the three items that follow: (a) D (b) C
A tennis coach is trying to put together a team of four (c) B (d) A
players for the forthcoming tournament. For this 7 players are 52. Which game do B, C and D play?
(a) Basketball (b) Hockey
available : males A, B and C: and females W, X, Y and Z. All
(c) Cricket (d) Football
players have equal capability and at least 2 males will be there in
53. Geeta is older than her cousin Meena. Meena’s brother Bipin
the team. For a team of four, all players must be able to play with’
is older than Geeta. When Meena and Bipin visit Geeta, they
each other. But, B cannot play with W, C cannot play with Z and
like to play chess. Meena wins the game more often than
W cannot play with Y.
Geeta.
45. If Y is selected and B is rejected, the team will consist of Based on the above information, four conclusions, as given
which one of the following groups? below, have been made. Which one of these logically follows
(a) A, C, W and Y (b) A, C, X and Y from the informaton given above?
(c) A, C, Y and Z (d) A, W, Y and Z (a) While playing chess with Geeta and Meena, Bipin often
46. If B is selected and Y is rejected, team will consist of which loses.
one of the following groups? (b) Geeta is the oldest among the three.
(a) A, B, C and W (b) A, B, C and Z (c) Geeta hates to lose the game.
(c) A, B, C and X (d) A, W, Y and Z (d) Meena is the youngest of the three.
47. If all the three males are selected, then how many combination Directions for the following 4 (four) items:
of four member teams are possible? Read the following passage and answer the four items that follow.
(a) 1 (b) 2 Your answers to these items should be based on the passage
(c) 3 (d) 4 only.
48. The music director of film wants to select four persons to
PASSAGE
work on different aspects of the composition of a piece of
Financial markets in India have acquired greater depth and liquidity
music. Seven persons are available for this work: they are
over the years. Steady reforms since 1991 have led to growing
Rohit, Tanya, Shobha, Kaushal, Kunal, Mukesh and Jaswant.
linkages and integration of the Indian economy and its financial
Rohit and Tanya will not work together. Kunal and Shobha system with the global economy. Weak global economic prospects
will not work together. Mukesh and Kunal want to work and continuing uncertainties in the international financial markets
together.
therefore, have had their impact on the emerging market economies. group meets every sixth day. How many times do all the five
Sovereign risk concerns, particularly in the Euro area, affected groups meet on the same day within 180 days?
financial markets for the greater part of the year, with the contagion (a) 3 (b) 5
of Greece’s soveregin debt problem spreading to India and other (c) 10 (d) 18
economies by way of higher-than-normal levels of volatility. 59. A, B, C, D and E belong to five different cities P, Q, R, S and
The funding constraints in international financial markets could T (not necessarily in that order). Each one of them comes
impact both the availability and cost of foreign funding for banks from a different city. Further it is given that
and corporates. Since the Indian financial system is bank 1. B and C do not belong to Q.
dominated, banks’ ability to withstand stress is critical to overall 2. B and E do not belong to P and R.
financial stability. Indian banks, however, remain robust, 3. A and C do not belong to R, S and T.
notwithstanding a decline in capital to risk-weighted assets ratio 4. D and E do not belong to Q and T.
and a rise in non-performing asset levels in the recent past. Capital Which one of the following statements is not correct?
adequacy levels remain above the regulatory requirements. The (a) C belongs to P (b) D belongs to R
financial market infrastructure continues to function without any (c) A belongs to Q (d) B belongs to S
major disruption. With further globalization, consolidation, 60. Seven men, A, B, C, D, E, F and G are standing in a queue in
deregulation, and diversification of the financial system, the that order. Each one is wearing a cap of different colour like
banking business may become more complex and riskier. Issue violet, indigo, blue green, yellow, orange and red. D is able to
like risk and liquidity management and enhancing skill therefore see in front of him green and blue, but not violet. E can see
assume greater significance. violet and yellow, but not red. G can see caps of all colours
54. According to the passage, the financial markets in the other than orange. If E is wearing an indigo coloured cap,
emerging market economies including India had the adverse then the colour of the cap worn by F is
impact in recent years due to (a) Blue (b) Violet
1. weak global economic prospects. (c) Red (d) Orange
2. uncertainties in the international financial markets. 61. There are some balls of red, green and yellow colour lying on
3. sovereign risk concerns in the Euro area. a table. There are as many red balls as there are yellow balls.
4. bad monsoons and the resultant crop loss. There are twice as many yellow balls as there are green ones.
Select the correct answer using the code given below: The number of red balls.
(a) 1 and 2 only (b) 1, 2 and 3 (a) is equal to the sum of yellow and green balls
(c) 2 and 3 only (d) 2, 3 and 4 (b) is double the number of green balls.
55. The Indian financial markets are affected by global changes (c) is equal to yellow balls minus green balls.
mainly due to the (d) cannot be ascertained.
(a) increased inflow of remittances from abroad
(b) enormous increases in the foreign exchange reserves. Directions for the following 2 (two) items:
(c) growing global linkages and integration of the Indian Read the following passage and answer the two items that follow.
financial markets. Your answer to these items should be based on the passage only.
(d) contagion of Greece’s sovereign debt problem. PASSAGE
56. According to the passage, in the Indian financial system, Crude mineral oil comes out of the earth as a thick brown or black
bank’s ability to withstand stress is critical to ensure overall liquid with a strong smell. It is a complex mixture of many different
financial stability because Indian financial system is substances, each with its own individual qualities. Most of them
(a) controlled by the Government of India are combinations of hydrogen and carbon in varying proportions.
(b) less integrated with banks. Such hydrocarbons are also found in other forms such as bitumen,
(c) controlled by the Reserve of Bank of India. asphalt and natural gas. Mineral oil originates from the carcasses
(d) dominated by Banks. of tiny animals and from plants that live in the sea. Over millions of
57. Risk and liquidity management assumes more importance in years, these dead creatures form large deposits under the sea-
the Indian banking system in future due to bed; and ocean currents cover them with a blanket of sand and
1. further globalization. silt. As this mineral hardens, it becomes sedimentary rock and
2. more consolidation and deregulation of financial system effectively shuts out the oxygen, so preventing the complete
3. further diversification of the financial system. decomposition of the marine deposits underneath. The layers of
4. more financial inclusion in the economy. sedimentary rock become thicker and heavier. Their pressure
Select the correct answer using the code given below: produces heat, which transforms the tiny carcasses into crude oil
(a) 1, 2 and 3 (b) 2, 3 and 4 in a process that is still going on today.
(c) 1 and 2 only (d) 3 and 4 only 62. Mineral oil deposits under the sea do not get completely
58. There are five hobby clubs in a college viz, photography decomposed because they
yachting, chess, electronics and gardening. The gardening (a) are constantly washed by the ocean currents.
group meets every second day, the electronics group meets (b) become rock and prevent oxygen from entering them.
every third day, the chess group meets every fourth day, the (c) contain a mixture of hydrogen and carbon.
yachting group meets every fifth day and the photography (d) are carcasses of organisms lying in saline conditions.
63. Sedimentary rock leads to the formation of oil deposits ENGLISH PASSAGE – 2
because It was already late when we set out for the next town, which
(a) there are no saline conditions below it. according to the map was about fifteen kilometers away on the
(b) it allows some dissolved oxygen to enter the dead other side of the hills. There we felt that we would find a bed for
organic matter below it. the night. Darkness fell soon after we left the village, but luckily
(c) weight of overlying sediment layers causes the we met no one as we drove swiftly along the narrow winding road
production of heat. that led to the hills. As we climbed higher, it became colder and
rain began to fall, making it difficult at times to see the road. I
(d) it contains the substances that catalyze the chemical
asked John, my companion, to drive more slowly. After we had
reactions required to change dead organisms into oil.
travelled for about twenty kilometers, there was still no sign of the
64. In a class of 45 students, a boy is ranked 20th. When two town which was marked on the map. We were beginning to get
boys joined, his rank was dropped by one. What is his new worried. Then without warning, the car stopped and we found we
rank from the end? had run out of pertrol.
(a) 25th (b) 26th 69. The author asked John to drive more slowly because
(c) 27 th (d) 28th (a) the road led to the hills.
65. A thief running at 8 km/hr is chased by a policeman whose (b) John was an inexperienced driver.
speed is 10 km/hr. If the thief is 100 m ahead of the policeman, (c) the road was not clearly visible.
then the time required for the policeman to catch the thief will (d) they were in wilderness.
be 70. The travellers set out for the town although it was getting
(a) 2 min (b) 3 min dark because
(c) 4 min (d) 6 min (a) they were in a hurry.
66. A train travels at a certain average speed for a distance of 63 (b) the next town was a short distance away and was a hill-
km and then travels a distance of 72 km at an average speed resort.
of 6 km/hr more than its original speed. If it takes 3 hours to (c) they were in wilderness.
complete the total journey, what is the original speed of the (d) the next town was a short distance away and promised
a good rest for the night.
train in km/hr?
71. The travellers were worried after twenty kilometers because
(a) 24 (b) 33
(a) it was a lonely countryside.
(c) 42 (d) 66 (b) they probably feared of having lost their way.
Directions for the following 8 (eight) items: (c) the rain began to fall.
(d) it was getting colder as they drove.
The following eight items (items 67 to 74) are based on three
passages in English to test the comprehension of English ENGLISH PASSAGE – 3
language and therefore these items do not have Hindi version. A stout old lady was walking with her basket down the middle of
Read each passage and answer the items that follow. a street in Petrograd to the great confusion of the traffic and no
small peril to herself. It was pointed out to her that the pavement
ENGLISH PASSAGE – 1
was the place for foot-passengers, but she replied “I’m going to
Seven-year-old Jim came home from the park without his new
walk where I like. We’ve got liberty now.” It did not occur to the
bicycle. “An old man and a little boy borrowed it,” he explained.
dear lady that if liberty entitled the foot-passenger to walk down
“They are going to bring it back at four o’clock”. His parents were the middle of the road it also entitled the taxi-driver to drive on the
upset that he had given his expensive new bicycle, but were secretly pavement, and that the end of such liberty would be universal
proud of his kindness and faith. Came four o’clock, no bicycle. chaos. Everything would be getting in everybody else’s way and
The parents were anxious. But at 4:30, the door bell rang, and there nobody would get anywhere. Individual liberty would have
stood a happy man and a boy, with the bicycle and a box of become social anarchy.
chocolates. Jim suddenly disappeared into his bedroom, and then 72. It was pointed out to the lady that she should walk on the
came running out. “All right,” he said, after examining the bicycle. pavement because she was
“You can have your watch back!” (a) a pedestrian (b) carrying a basket
67. When Jim came home without his bicycle his parents (c) stout (d) an old lady
(a) were angry with him 73. The lady refused to move from the middle of the street
(b) were worried because
(c) did not feel concerned (a) she was not afraid of being killed.
(d) were eager to meet the old man and the little boy. (b) she felt that she is entitled to do whatever she liked.
68. Jim returned the watch to the old man and the little boy (c) she did not like walking on the payment.
(d) she was confused.
because
74. The old lady failed to realise that
(a) they gave him chocolates.
(a) she was not really free.
(b) his father was proud of him. (b) her liberty was not unlimited.
(c) he was satisfied with the condition of his bicycle (c) she was an old person.
(d) they were late only by 30 minutes. (d) roads are made for motor vehicles only.
Directions for the following 6 (six) items: (c) Call one of your senior subordinate officers and ask
Given below are six items. Each items describes a situation and him to solve the villager’s problem.
is followed by four possible responses. Indicate the response you (d) Quickly take the application from him, ask him a few
find most appropriate. Choose only one response for each item. relevant questions regarding his problem and then
The responses will be evaluated based on the level of proceed to the meeting.
appropriateness for the given situation. 78. There is a shortage of sugar in your District where you are
Please attempt all the items. There is no penalty for wrong the District Magistrate. The Government has ordered that
answers for these six items. only a maximum amount of 30 kg sugar is to be released for
wedding celebrations. A son of your close friend is getting
75. You are the head of your office. There are certain houses
married and your friend requests you to release at least 50 kg
reserved for the allotment to the office staff and you have
sugar for his son’s wedding. He expresses annoyance when
been given the discretion to do so. A set of rules for the
you tell him about the Government’s restrictions on this
allotment of the houses has been laid down by you and has
matter. He feels that since you are the District Magistrate
been made public. Your personal secretary, who is very close
you can release any amount./ You do not want to spoil your
to you, comes to you and pleads that as his father is seriously
friendship with him. In such circumstances, how would you
ill, he should be given priority in allotment of a house. The
deal with the situation?
office secretariat that examined the request as per the rules
(a) Releases the extra amount of sugar which your friend
turns down the request and recommends the procedure to be
has requested for.
followed according to the rules. You do not want to annoy
(b) Refuse your friend the extra amount and strictly follow
your personal secretary. In such circumstances, what would
the rules.
you do?
(c) Show your friend the copy of the Government
(a) Call him over to your room and personally explain why
instructions and then persuade him to accept the lower
the allotment cannot be done.
amount as prescribed in the rules.
(b) Allot the house to him to win his loyalty.
(d) Advise him to directly apply to the allotting authority
(c) Agree with the office note to show that you are not
and inform him that you do not interfere in this matter.
biased and that you do not indulge in favouritism.
79. You are in-charge of implementing the Family Planning
(d) Keep the file with you and not pass any orders.
programme in an area where there is a strong opposition to
76. While travelling in a Delhi-registered commercial taxi from
the present policy. You want to convince the residents of the
Delhi to an adjacent city (another State), your taxi driver
need for keeping small families. What would be the best way
informs you that as he has no permit for running the taxi in
of communicating this message?
that city, he will stop at its Transport Office and pay the
(a) By logically explaining to the residents the need for
prescribed fee of ` forty for a day. While paying the fee at
family planning to improve the health and living
the counter you find that the transport clerk is taking an extra
standards.
fifty rupees for which no receipt is being given. You are in a
(b) By encouraging late marriages and proper spacing of
hurry for your meeting. In such circumstances, what would
children.
you do?
(c) By offering incentives for adopting family planning
(a) Go up to the counter and ask the clerk to give back the
devices.
money which he has illegally taken.
(d) By asking people who have been sterilized or are using
(b) Do not interfere at all as this is a matter between the taxi
contraceptives to directly talk to the residents.
driver and the tax authorities.
80. You are a teacher in a University and are setting a question
(c) Take note of the incident and subsequently report the
paper on a particular subject. One of your colleagues, whose
matter to the concerned authorities.
son is preparing for the examination on that subject, comes
(d) Treat it as a normal affair and simply forget about it.
to you and informs you that it is his son’s last chance to pass
77. A person lives in a fair off village which is almost two hours
that examination and whether you could help him by
by bus. The villager’s neighbour is a very powerful landlord
indicating what questions are going to be in the examination.
who is trying to occupy the poor villager’s land by force.
In the past, your colleague had helped you in another matter.
You are the District Magistrate and busy in a meeting called
Your colleague informs you that his son will suffer from
by a local Minister. The villager has come all the way, by bus
depression if he fails in this examination. In such
and on foot, to see you and give an application seeking
circumstances, what would you do?
protection from the powerful landlord. The villager keeps on
(a) In view of the help he had given you, extend your help
waiting outside the meeting hall for an hour. You come out of
to him.
the meeting and are rushing to another meeting. The villager
(b) Regret that you cannot be of any help to him.
follows you to submit his application. What would you do?
(c) Explain to your colleague that this would be violating
(a) Tell him to wait for another two hours till you come
the trust of the University authorities and you are not
back from your next meeting.
in a position to help him.
(b) Tell him that the matter is actually to be dealt by a
(d) Report the conduct of your colleague to the higher
junior officer and that he should give the application to
authorities.
him.
ANSWERS AND EXPLANATIONS
1. (d) The passage revolves around the problem of habitat \ 2000 : 1000 : : 20 : x
fragmentation which poses a serious threat to
2000 20
biodiversity conservation. Þ = or x = 10 days
2. (a) When forest lands get fragmented human activities start 1000 x
on the edges of these fragmented lands which results 8. (d) Let ‘x’ be the initial petrol consumption. Let ‘y’ be the
in degradation of entire forests. Therefore continuity number of days petrol lasts for. According to question
of the forested landscapes and corridors should be
maintained. Petrol consumption Days
3. (a) Let gold coin be represented by ‘G’. Initial x 10
Let non-gold coin be represented by ‘N’.
125
G 1 Final x y
Initial ratio = = ...(1) 100
N 3
After adding 10 gold coins, the new ratio will be Hence, it’s a case of indirect variation.
G + 10 1
= ...(2) 125 1000
N 2 \ x × 10 = x´ y Þ y = = 8 days
100 125
N
Putting the value of G = in equation (2). 9. (a) 2-way walk = 10 hrs
3
\ 1-way walk = 5 hrs
N
+ 10 1-way walk + 1 way drive = 6 hrs
3 1
= \ 1 way drive = 6 – 5 = 1 hr
N 2
On solving, we get N = 60 Hence, two way drive take 2 hrs.
and G = 30 10. (d) The slurry cannot be decomposed by the watercourses
Presently, the total number of coins in the collection as they do not have microorganisms therefore this may
= 10 + 20 + 60 = 90 lead to eutrophication of water bodies.
4. (b) If the number of rows and columns are to be equal, then 11. (b)
the total number of trees would represent a perfect 12. (b) The pollutants are added to both the soil and water.
square. Since, 1000 is not a perfect square, we need to Soil doesn't get polluted as microorganisms convert
check for a perfect square above and nearest to 1000. pollutant into fertilizer. Water is enriched with the
It’s 1024, which is square of 32. So he needs 24 more nutrient which flows through streams, rivers and lakes.
trees to get 1024. These nutrients are responsible for eutrophication
5. (b) Let the least value be x. Then the next value is x + 20 which results in decomposition of algal blooms.
and the next value is x + 40 and so on. 13. (b) Cultural eutrophication will result in loss of flora and
According to question fauna as eutrophication of water bodies do not allow
(x) + (x + 20) + (x + 40) + (x + 60) + (x + 80) + (x + 100) oxygen to pass through it.
+ (x + 120) = 700 14. (c) The theme of the passage is based on how the
Þ 7x + (20 + 40 + 60 + 80 + 100 + 120) = 700 agricultural slurry when discharged in water causes
Þ 7x + 20 (1 + 2 + 3 + 4 + 5 + 6) = 700 cultural eutrophication resulting ecosystem service
Þ 7x + 20 × 21 = 700 loss. The magnitude of the problem has been so much
Þ 7(x + 60) = 7(100) Þ x = 40 that the scientists have noticed huge 'dead zones' near
6. (c) Out of 120, there are 70 males. big water bodies which means destruction of aquatic
\ 120 – 70 = 50 females life.
For a maximum, all 70 male shall have driver’s license.
15. (b) Man's physical need is never going to cease as long as
For a minimum, if all 50 females posses driver’s license,
man is craving for it. Desire brings along miseries. If
then the remaining 30 licenses, shall be possessed by
man doesn't realize this and keeps on doing what he is
males.
doing then no physical help can reduce his miseries. It
So minimum male driver’s licenses = 30.
is man's nature which is responsible for all the miseries.
Min. to max. = 30 to 70 = 3 to 7
The only solution to this is man's spiritual growth and
7. (d) Let the remaining food last for x days.
bringing change in his character.
1000 soldiers had provisions for (30 – 10) = 20 days
16. (d) The author gives primary importance to spiritual growth
(1000 + 1000) men had provisions for x days.
in man in eradicating human misery.
More men, less days (indirect proportion)
17. (b) Below is the pattern followed by symbols. c 9 ´1032
Hence averge speed by ‘C’ = = = 8.23
Fig. (1) to (2) Fig. (2) to (3)
24 47 ´ 24
For D, total payment for car D
d 11 ´ 906
= 6 ´ d + 40 ´ = 1812 or d =
11 53
d 453
Hence, the average speed of D = = = 8.5
Fig. (3) to (4) Answer figure 22 53
All speeds except A are less than 9. Hence it’s greatest
value.
23. (b) Observe the first two figures carefully :
2 ´ 84 2 ´ 81
= 14 and = 18
12 9
18. (c) In each step arrow [ ] symbol is inverting 180° and 2 ´ 88
Hence for the third figure, = x Þ x = 16
cross circle symbol in each step touches one vertex 11
of triangle in clockwise direction. 24. (a) By going options.
Hence, option (c) is the correct answer. The numbers adjacent to ‘1’ are ‘4’ and ‘6’ in fig. (1).
19. (b) From figure (1) to (2), 2nd and 3rd design makes a pair The numbers adjacent to ‘1’ are ‘2’ and ‘3’ in fig. (2).
and comes forward while 1st design gets 180° inverted. From these two statements, we can’t deduce that the
Same from figure (2) to (3) and figure (3) to (4). number opposite to ‘1’ is ‘5’.
Hence option (b) is correct answer. \ Possible value of (A) and (B) can be 2 and 3.
20. (d) Time taken to finish a work is inversely proportional to 25. (b) Number of professors in physics = 40
the number of persons working at it. Percentage of professors in age group 35 – 44 = 40%
\ More persons then less is the time taken to finish a job. \ Physics professors belong to age group
Hence, diagram IV represents the relation between x 35 – 44 = 40% of 40 = 16
and y. 26. (a) By looking to graph I. We can deduce that physics
21. (c) Consider 2nd element in each row. Sum up the digits of discipline has the highest ratio of males to females.
each number. 27. (c) No. of females = 6
3 + 7 + 0 = 10 No. of males = 4
2+ 2 + 4 =8
No. of females
7 + 3 + 0 = 10 Required percentage = ´ 100
3 10 7 Males + Females
Now redraw the table agains: 6
2 8 6 = ´ 100 = 60%
\ 1 + x = 10 Þ x = 9 1 10 x 10
22. (a) Let the distances travelled by cars A, B, C and D be a, 28. (a) Number of all physics professors in age group
b, c and d respectively. 30
Then, the total payment for car A 25 – 34 = 30% of 40 = ´ 40 = 12
100
a
= 6 ´ a + 40 ´ = 11a Number of female physics professors in the age group
8
25
2120 25 – 34 = 25% of 12 = ´ 12 = 3
Þ 11a = 2120 or a = 100
11 \ Number of male physics professors in the age group 25
a 2120
Hence, average speed of ‘A’ = = = 9.63 – 34 = 12 – 3 = 9
20 11 ´ 20 29. (b) Let the number of professors in the university = x
b According to question
For B, total payment for car B = 6 ´ b + 40 ´
10 10 ´100
Þ 10b = 1950 \ b = 195 2% of x = 10 Þ x = = 500
2
b
Hence, average speed of ‘B’ = 30. (d) From fig. (1) to (2), all arrows are moving clockwise
25 directions. From fig. (2) to (3), all arrows are moving
195 anticlockwise direction. From (3) to (4), one arrow is in
\ = 7.8 reserved direction and other one moving clockwise.
25
c same pattern is followed for rest.
For C, total payment for car C = 6 ´ c + 40 ´ 31. (c) The democracies in the west have evolved over a period
9
of time because of people’s participation in the
9 ´ 1032
Þ c= democratic process. The western democracies have
47
institutionalized the various instrument of governance. 41. (a) Looking at the above diagram, we can see that we have
As a result their democracies are far more robust than only one route i.e. Q P R which has a single mode
the newly formed democracies. of transportation i.e. boat.
32. (a) Many non- western countries have not seen economic 42. (b) With the given options, we have the following the
growth the way the western democracies have seen. possibilities :
The existing rulers of those countries fear, peoples P Q T R S R P
participation in democratic process may question the P R S R T Q P
established values. In both the cases, R should be visited twice as S is
33. (c) Compliance with labour and tax laws of land shows connected only to R.
transparency and ethics on the part of a company. On 43. (d) None of the cities in the given options directly
the part of a controlling head, being able to distinguish connected.
between personal and corporate funds while managing Therefore, option (d) is correct answer.
a company is the cornerstone of good corporate 44. (a) Going by options:
governance. Q and S:
34. (c) A Company’s good governance is a pointer of its Air Bus/Rail Boat/Bus
Q T R S
integrity, ethics and discipline. If these values are the
Hence, all the options are available. Therefore, option
foundation of a company, the company’s growth index
(a) is correct answer.
will go up which will build up confidence in the market,
45. (b) If Y is selected, W shall not be selected. So options (a)
influence their share value positively by attracting
and (d) are out. As C cannot play with Z, option (c) is
foreign investors.
also out. Hence (b) is correct answer.
35. (c) Malnutrition occurs due to not eating adequate quantity
46. (c) If B is selected, W shall not be selected. So, options (a)
of food. The knowledge of the food needs of younger
and (d) are out. As C cannot play with Z, option (b) is
children can curb this problem.
also out. Hence, (c) is correct answer.
36. (a) Working mothers proceed to work leaving behind their
47. (b) If males A, B and C are selected, we need to select only
kids in the hands of either maids or older siblings. The
one more person to make up a four member team. As B
priority of a working mother is her career not her child
is selected, W cannot be selected and as C is selected,
therefore the neglect happens to the child.
Z cannot be selected. So the choice is between X and Y.
37. (c) Policy intervention can save the farmers from
Then we have the following possibilities :
agricultural risk. Because Indian farmers are risk averse.
(A, B, C, X) and (A, B, C, Y)
So they design inputs and cropping patterns to reduce
So, we have two options of team.
risk not to maximize their profit.
48. (c) As, Rohit and Tanya will not work together.
38. (b) Agricultural risks can be tackled with many strategies
Therefore,option (b) and (d) are out.
like crop insurance, price stabilization, development of
As, Kunal and Shobha will not work together.
varieties resistant to pests and diseases. So no single
Therefore, option (a) is also out.
risk specific policy is sufficient to reduce it.
Hence, option (c) is correct answer.
39. (d) As intimacy is an essential characteristic of a primary
49. (c) From the given conditions. two cases are possible as
group, members of a primary group know each other
shown below :
intimately.
C C
40. (a) According to question
A=B–1
Þ B=A+1
C= D+5 D A A D
D=B+3 or
If we see these equations, we can find that every one B
E
except A has got something more than some other. So, B E
answer must not be B, C or D. So, A is the choice.
For questions 41 to 44 : Hence, In any case, Both the conditions are true.
For Questions 50 to 52 :
Players Football Cricket Hockey Basket
ball
A ×
B ×
C ×
D ×
50. (d) 51. (a) 52. (b)
53. (d) From the given information, we conclude that ; Bipin is
older than Geeta and Geeta is older than Meena.
Hence, Meena is the youngest of the three.
54. (b) 55. (c) 56. (d) 57. (a) Þ (x) (x + 6) = 21(x + 6) + 24(x)
58. (a) Take the LCM. of 2, 3, 4, 5, 6 = 60 Þ x2 – 39x – 126 = 0 Þ x2 – 42x + 3x = 126 = 0
So every 60th day all five hobby clubs meet. So in a Þ (x – 42) (x + 3) = 0
period of 180 days they meet thrice i.e. on 60th, 120th x = 42, x = – 3
and180th day. x = – 3 is rejected.
Hence original speed was 42 km/hr.
Cities 67. (b) His parents were worried because the cycle was new.
59. (d) P Q R S T
Person They probably thought he has lost it.
A × × × × 68. (c) As a security of his cycle he had kept the watch from
B × × × × the old man and the little boy
C × × × × 69. (c) As it was raining. Moreover it was dark so visibility
D × × × × was a problem.
E × × × × 70. (d) They expected to reach the town within a short time as
\ B belongs to S is not correct. the distance was only 15 km.
60. (c) The key point is in this question is “A person can see 71. (b) They were supposed to reach their destination after a
but caps of all persons standing in front of him can’t run of 50 km according to the map. After going 20 km
see his own cap and caps of persons standing behind and still not finding their destination they became
him. worried as it was dark and rain had begun to fall.
From the given data he can summarise all the information 72. (a) If she did not walk on the pedestrians way she might
in a table : face accident.
73. (b) She felt so because she was arrogant and newly
Person Cap achieved liberty had gone into her head in a wrong
A/B/C Green/blue/yellow way.
D Violet 74. (d) This is because she was confused and arrogant at the
same time.
E Indigo
75. (a) When it comes to declining a request it is better done in
F Red private and then explanation for not being able to do a
G Orange favour does not make the other person feel so bad
especially when the other person is very close to you.
61. (b) According to question
76. (c) One should not let go the wrong doer because that will
R = y and y = 2G
set a wrong precedence. So if the spur of the moment is
\ R = 2G not rife to act, one must restrain and make a mental note
Hence, the number of red balls is double the number of instead. Complaint can be lodged later but an urgent
green balls. meeting can be ignored or spoilt just because of some
62. (b) When Oxygen is shut from entering the decomposition unscrupulous people. They can be taken to task later.
doesn't take place completely. The Carcasses inside 77. (d) The next meeting can be put on hold for a few minutes,
the sedimentary rocks get transformed into crude oil but the person who has already made lot of effort to
due to heat and pressure. reach the district magistrates office in anticipation of
63. (c) The layers of Sedimentary rocks become thicker and help should be heard first. This humane attitude towards
heavier due to deposition and create pressure on the common people and their problems will enhance the
marine creatures under it, which in turn change the respect of the chair, the person and the district
carcasses into crude oil. administration.
64. (c) Initial strength of class = 45 78. (d) This will make it very clear to him that doing small
Now, new strength = 47 favours are not part of job. You are far more responsible
The boy’s rank is dropped by one Now, his rank for the position. The advise will show that you care for
becomes 21. him and a direct approach approach to the allotting
Total number of boys in class = authority may work for him. This gives a lasting
Boy’s rank from starting + Boy’s rank from end – 1 impression that you follow the rules.
\ Boy’s rank from end = 47 – 21 + 1 = 27 79. (a) A place where there is a strong opposition for family
65. (b) If we consider the difference of speeds, policeman is 2 planning, it is not easy to implement any programme
km/hr leading speed and he can catch the thief at 100 m quickly or hastily. But persuasion and explanation can
surely make a difference. So by logically explaining the
100 m 100 pros and cons of the progamme people can be
ahead by = ´ 60 = 3 min.
2 kmph 1000 ´ 2 persuaded better though it may take longer.
80. (c) Extending a helping hand to someone for personal
66. (c) Let the original speed = x reasons while sitting in a responsible position
Total time taken = 3 hr Is unethical and does not do justice to the trust and
63 72 responsibility of the authority conferred on
Þ 3= + You by the authority. Your position and responsibility
x x+6
is far more greater than personal obligations.
INSTRUCTIONS
1. This Test Booklet contains 100 items (questions). Each item comprises four responses (answers). You will select
the response which you want to mark on the Answer sheet. In case, you feel that there is more than one correct
response, mark the response which you consider the best. In any case, choose ONLY ONE response for each
item.
2. All items carry equal marks.
3. Penalty for wrong answers:
THERE WILL BE PENALTY FOR WRONG ANSWERS MARKED BY THE CANDIDATE FOR WRONG
ANSWER.
(i) There are four alternatives for the answer to every question. For each question which has a penalty for which a
wrong answer has been given by the candidate, one-third of the marks assigned to that question will be
deducted as penalty.
(ii) If a candidate gives more than one answer, it will be treated as a wrong answer even if one of the given answers
happens to be correct and there will be same penalty as above to that question.
(iii) If a question is left blank, i.e., no answer is given by the candidate, there will be no penalty for that question.

1. In the areas covered under the Panchayat (Extension to the Which of the statements given above is /are correct?
Scheduled Areas) Act, 1996, what is the role/power of Gram (a) 1only (b) 2 and 3 only
Sabha? (c) 1 and 3 only (d) 1, 2 and 3
1. Gram Sabha has the power to prevent alienation of land 4. The National Green Tribunal Act, 2010 was enacted in
in the Scheduled Areas. consonance with which of the following provisions of the
2. Gram Sabha has the ownership of minor forest produce. Constitution of India?
3. Recommendation of Gram Sabha is required for granting 1. Right to healthy environment, construed as a part of Right
prospecting licence or mining lease for any mineral in to life under Article 21
the Scheduled Areas. 2. Provision of grants for raising the level of administration in
Which of the statements given above is/are correct? the Scheduled Areas for the welfare of Scheduled Tribes
under Article 275(1)
(a) 1 only (b) 1 and 2 only
3. Powers and functions of Gram Sabha as mentioned under
(c) 2 and 3 only (d) 1, 2 and 3
Article 243 (A)
2. In the Parliament of India, the purpose of an adjournment
Select the correct answer using the codes given below :
motion is (a) 1 only (b) 2 and 3 only
(a) to allow a discussion on a definite matter of urgent (c) 1 and 3 only (d) 1, 2 and 3
public importance 5. If National Water Mission is properly and completely
(b) to let opposition members collect information from the implemented, how will it impact the country?
ministers 1. Part of the water needs of urban areas will be met through
(c) to allow a reduction of specific amount in demand for recycling of wastewater.
grant 2. The water requirements of coastal cities with
(d) to postpone the proceedings to check the inappropriate inadequate alternative sources of water will be met by
or violent behaviour on the part of some members adopting appropriate technologies that allow for the
3. How does National Biodiversity Authority (NBA) help in use of ocean water.
protecting the Indian agriculture? 3. All the rivers of Himalayan origin will be linked to the
1. NBA checks the biopiracy and protects the indigenous rivers of peninsular India,
and traditional genetic resources. 4. The expenses incurred by farmers for digging bore-
2. NBA directly monitors and supervises the scientific wells and for installing motors and pump-sets to draw
research on genetic modification of crop plants. groundwater will be completely reimbursed by the
3. Application for Intellectual Property Rights related to Government.
genetic/biological resources cannot be made without Select the correct answer using the codes given below :
the approval of NBA. (a) 1 only (b) 1 and 2 only
(c) 3 and 4 only (d) 1, 2, 3 and 4
6. Consider the following provisions under the Directive 11. Consider the following kinds of organisms :
Principles of State Policy as enshrined in the Constitution of 1. Bacteria 2. Fungi
India : 3. Flowering plants
1. Securing for citizens of India a uniform civil code Some species of which of the above kinds of organisms are
2. Organizing village Panchayats employed as biopesticides?
3. Promoting cottage industries in rural areas (a) 1 only (b) 2 and 3 only
4. Securing for all the workers reasonable leisure and (c) 1 and 3 only (d) 1, 2 and 3
cultural opportunities 12. Biomass gasification is considered to be one of the
Which of the above are the Gandhian Principles that are sustainable solutions to the power crisis in India. In this
reflected in the Directive Principles of State Policy? context, which of the following statements is/are correct?
(a) 1, 2 and 4 only (b) 2 and 3 only
(c) 1, 3 and 4 only (d) 1, 2, 3 and 4 1. Coconut shells, groundnut shells and rice husk can be
7. Consider the following statements: used in biomass gasification.
1. Union Territories are not represented in the Rajya Sabha. 2. The combustible gases generated from biomass
2. It is within the purview of the Chief Election gasification consist of hydrogen and carbon dioxide
Commissioner to adjudicate the election disputes. only.
3. According to the Constitution of India, the Parliament 3. The combustible gases generated from biomass
consists of the Lok Sabha and the Rajya Sabha only. gasification can be used for direct heat generation but
Which of the statements given above is/are correct? not in internal combustion engines.
(a) 1 only (b) 2 and 3 Select the correct answer using the codes given below :
(c) 1 and 3 (d) None (a) 1 only (b) 2 and 3 only
8. With reference to consumers’ rights/privileges under the (c) 1 and 3 only (d) 1, 2 and 3
provisions of law in India, which of the following statements 13. What is the role of ultraviolet (UV) radiation in the water
is/are correct? purification systems?
1. Consumers are empowered to take samples for food 1. It inactivates /kills the harmful microorganisms in water.
testing. 2. It removes all the undesirable odours from the water.
2. When a consumer files a complaint in any consumer 3. It quickens the sedimentation of solid particles, removes
forum, no fee is required to be paid.
turbidity and improves the clarity of water.
3. In case of death of a consumer, his/her legal heir can
Which of the statements given above is/are correct?
file a complaint in the consumer forum on his/her behalf.
(a) 1 only (b) 2 and 3 only
Select the correct answer using the codes given below:
(c) 1 and 3 only (d) 1, 2 and 3
(a) 1 only (b) 2 and 3 only
14. Graphene is frequently in news recently. What is its
(c) 1 and 3 only (d) 1, 2 and 3
importance?
9. Regarding the office of the Lok Sabha Speaker, consider the
1. It is a two-dimensional material and has good electrical
following statements:
conductivity.
1. He/She holds the office during the pleasure of the
President. 2. It is one of the thinnest but strongest materials tested
2. He/She need not be a member of the House at the time so far.
of his/her election but has to become a member of the 3. It is entirely made of silicon and has high optical
House within six months from the date of his/her transparency.
election. 4. It can be used as ‘conducting electrodes’ required for
3. If he/she intends to resign, the letter of his/her touch screens, LCDs and organic LEDs.
resignation has to be addressed to the Deputy Speaker. Which of the statements given above are correct?
Which of the statements given above is /are correct? (a) 1 and 2 only (b) 3 and 4 only
(a) 1 and 2 only (b) 3 only (c) 1, 2 and 4 only (d) 1, 2, 3 and 4
(c) 1, 2 and 3 (d) None 15. Lead, ingested or inhaled, is a health hazard. After the
10. Which of the following are included in the original jurisdiction addition of lead to petrol has been banned, what still are the
of the Supreme Court? sources of lead poisoning ?
1. A dispute between the Government of India and one or 1. Smelting units 2. Pens and pencils
more States 3. Paints 4. Hair oils and cosmetics
2. A dispute regarding elections to either House of the Select the correct answer using the codes given below :
Parliament or that of Legislature of a State (a) 1, 2 and 3 only (b) 1 and 3 only
3. A dispute between the Government of India and a Union (c) 2 and 4 only (d) 1, 2, 3 and 4
Territory 16. With reference to ‘stem cells’, frequently in the news, which
4. A dispute between two or more States of the following statements is/are correct?
Select the correct answer using the codes given below : 1. Stem cells can be derived from mammals only.
(a) 1 and 2 (b) 2 and 3 2. Stem cells can be used for screening new drugs.
(c) 1 and 4 (d) 3 and 4 3. Stem cells can be used for medical therapies.
Select the correct answer using the codes given below : Select the correct answer using the codes given below :
(a) 1 and 2 only (b) 2 and 3 only (a) 1 and 2 only (b) 3 and 4 only
(c) 3 only (d) 1, 2 and 3 (c) 1, 3 and 4 only (d) 1, 2, 3 and 4
17. Consider the following statements : 22. With reference to the history of ancient India, which of the
Chlorofluorocarbons, known as ozone-depletin g following was/were common to both Buddhism and Jainism?
substances, are used
1. In the production of plastic foams 1. Avoidance of extremities of penance and enjoyment
2. In the production of tubeless tyres 2. Indifference to the authority of the Vedas
3. In cleaning certain electronic components 3. Denial of efficacy of rituals
4. As pressurizing agents in aerosol cans Select the correct answer using the codes given below :
Which of the statements given above is/are correct? (a) 1 only (b) 2 and 3 only
(a) 1, 2 and 3 only (b) 4 only (c) 1 and 3 only (d) 1, 2 and 3
(c) 1, 3 and 4 only (d) 1, 2, 3 and 4 23. Which of the following can be said to be essentially the
18. A team of scientists at Brookhaven National Laboratory parts of ‘Inclusive Governance’?
including those from India created the heaviest anti-matter 1. Permitting the Non-Banking Financial Companies to do
(anti-helium nucleus). What is/are the implication/ banking
implications of the creation of anti-matter? 2. Establishing effective District Planning Committees in
all the districts
1. It will make mineral prospecting and oil exploration
3. Increasing the government spending on public health
easier and cheaper.
4. Strengthening the Mid-day Meal Scheme
2. It will help probe the possibility of the existence of
Select the correct answer using the codes given below :
stars and galaxies made of anti-matter.
(a) 1 and 2 only (b) 3 and 4 only
3. It will help understand the evolution of the universe.
(c) 2, 3 and 4 only (d) 1, 2, 3 and 4
Select the correct answer using the codes given below :
24. The Nagara, the Dravida and the Vesara are the
(a) 1 only (b) 2 and 3 only (a) three main racial groups of the Indian subcontinent
(c) 3 only (d) 1, 2 and 3 (b) three main linguistic divisions into which the languages
19. Which of the following is/are cited by the scientists as of India can be classified
evidence/evidences for the continued expansion of (c) three main styles of Indian temple architecture
universe? (d) three main musical Gharanas prevalent in India
1. Detection of microwaves in space 25. The Congress ministries resigned in the seven provinces in
2. Observation of redshift phenomenon in space 1939, because
3. Movement of asteroids in space (a) the Congress could not form ministries in the other
4. Occurrence of supernova explosions in space four provinces
Select the correct answer using the codes given below : (b) emergence of a ‘left wing’ in the Congress made the
(a) 1 and 2 (b) 2 only working of the ministries impossible
(c) 1, 3 and 4 (c) there were widespread communal disturbances in their
(d) None of the above can be cited as evidence provinces
20. Electrically charged particles from space travelling at speeds (d) None of the statements (a), (b) and (c) given above is
of several hundred km/sec can severely harm living beings correct
if they reach the surface of the Earth. What prevents them 26. With reference to National Rural Health Mission, which of
from reaching the surface of the Earth? the following are the jobs of ‘ASHA’, a trained community
(a) The Earth’s magnetic field diverts them towards its poles health worker?
(b) Ozone layer around the Earth reflects them back to outer 1. Accompanying women to the health facility for
space antenatal care checkup
(c) Moisture in the upper layers of atmosphere prevents 2. Using pregnancy test kits for early detection of
them from reaching the surface of the Earth pregnancy
(d) None of the statements (a), (b) and (c) given above is 3. Providing information on nutrition and immunization
correct 4. Conducting the delivery of baby
21. With reference to the scientific progress of ancient India, which Select the correct answer using the codes given below:
of the statements given below are correct? (a) 1, 2 and 3 only (b) 2 and 4 only
1. Different kinds of specialized surgical instruments were (c) 1 and 3 only (d) 1, 2, 3 and 4
in common use by 1st century AD. 27. Which of the following is/are the principal feature(s) of the
2. Transplant of internal organs in the human body had Government of India Act, 1919 ?
begun by the beginning of 3rd century AD. 1. Introduction of dyarchy in the executive government
3. The concept of sine of an angle was known in 5th of the provinces
century AD. 2. Introduction of separate communal electorates for
4. The concept of cyclic quadrilaterals was known in 7th Muslims
3. Devolution of legislative authority by the centre to the
century AD.
provinces
Select the correct answer using the codes given below : Select the correct answer using the codes given below :
(a) 1 only (b) 2 and 3 only (a) 1 only (b) 2 and 3 only
(c) 1 and 3 only (d) 1, 2 and 3 (c) 1 and 3 only (d) 1, 2 and 3
28. During Indian freedom struggle, the National Social 33. Which of the following is /are among the noticeable features
Conference was formed. What was the reason for its of the recommendations of the Thirteenth Finance
formation? Commission?
(a) Different social reform groups or organizations of 1. A design for the Goods and Services Tax, and a
Bengal region united to form a single body to discuss compensation package linked to adherence to the
the issues of larger interest and to prepare appropriate proposed design
petitions/representations to the government 2. A design for the creation of lakhs of jobs in the next ten
(b) Indian National Congress did not want to include social years in consonance with India’s demographic dividend
reforms in its deliberations and decided to form a 3. Devolution of a specified share of central taxes to local
separate body for such a purpose bodies as grants
(c) Behramji Malabari and M. G. Ranade decided to bring Select the correct answer using the codes given below :
together all the social reform groups of the country (a) 1 only (b) 2 and 3 only
under one organization (c) 1 and 3 only (d) 1, 2 and 3
(d) None of the statements (a), (b) and (c) given above is 34. What is/are the recent policy initiative(s) of Government of
correct in this context India to promote the growth of manufacturing sector?
29. Which of the following parties were established by Dr. B. R.
Ambedkar? 1. Setting up of National Investment and Manufacturing
1. The Peasants and Workers Party of India Zones
2. All India Scheduled Castes Federation 2. Providing the benefit of ‘single window clearance’
3. The Independent Labour Party 3. Establishing the Technology Acquisition and
Select the correct answer using the codes given below : Development Fund
(a) 1 and 2 only (b) 2 and 3 only Select the correct answer using the codes given below :
(c) 1 and 3 only (d) 1, 2 and 3 (a) 1 only (b) 2 and 3 only
(c) 1 and 3 only (d) 1, 2 and 3
30. Which of the following special powers have been conferred on
35. Which of the following are the methods of Parliamentary
the Rajya Sabha by the Constitution of India?
control over public finance in India?
(a) To change the existing territory of a State and to change
1. Placing Annual Financial Statement before the
the name of a State
Parliament
(b) To pass a resolution empowering the Parliament to make
2. Withdrawal of moneys from Consolidated Fund of
laws in the State List and to create one or more All India
India only after passing the Appropriation Bill
Services
3. Provisions of supplementary grants and vote-on-
(c) To amend the election procedure of the President and
account
to determine the pension of the President after his/her 4. A periodic or at least a mid-year review of programme
retirement of the Government against macroeconomic forecasts
(d) To determine the functions of the Election Commission and expenditure by a Parliamentary Budget Office
and to determine the number of Election Commissioners 5. Introducing Finance Bill in the Parliament
31. How does the National Rural Livelihood Mission seek to Select the correct answer using the codes given below :
improve livelihood options of rural poor? (a) 1, 2, 3 and 5 only (b) 1, 2 and 4 only
1. By setting up a large number of new manufacturing (c) 3, 4 and 5 only (d) 1, 2, 3, 4 and 5
industries and agribusiness centres in rural areas 36. Mahatma Gandhi undertook fast unto death in 1932, mainly
2. By strengthening ‘self-help groups’ and providing skill because
development (a) Round Table Conference failed to satisfy Indian political
3. By supplying seeds, fertilizers, diesel pump-sets and aspirations
micro-irrigation equipment free of cost to farmers (b) Congress and Muslim League had differences of
Select the correct answer using the codes given below : opinion
(a) 1 and 2 only (b) 2 only (c) Ramsay Macdonald announced the Communal Award
(c) 1 and 3 only (d) 1, 2 and 3 (d) None of the statements (a), (b) and (c) given above is
32. The Multi-dimensional Poverty Index developed by Oxford correct in this context
Poverty and Human Development Initiative with UNDP 37. With reference to Ryotwari Settlement, consider the following
support covers which of the following? statements :
1. Deprivation of education, health, assets and services 1. The rent was paid directly by the peasants to the
at household level Government.
2. Purchasing power parity at national level 2. The Government gave Pattas to the Ryots.
3. Extent of budget deficit and GDP growth rate at national 3. The lands were surveyed and assessed before being
level taxed.
Which of the statements given above is/are correct? Which one among the following States has all the above
(a) 1 only (b) 1 and 2 only characteristics?
(c) 1, 2 and 3 (d) None (a) Arunachal Pradesh
38 Consider the following specific stages of demographic (b) Assam
transition associated with economic development : (c) Himachal Pradesh
(d) Uttarakhand
1. Low birthrate with low death rate 44. Consider the following crops of India :
2. High birthrate with high death rate 1. Cowpea 2. Green gram
3. High birthrate with low death rate 3. Pigeon pea
Select the correct order of the above stages using the codes Which of the above is/are used as pulse, fodder and green
given below : manure?
(a) 1, 2, 3 (b) 2, 1, 3 (a) 1 and 2 only (b) 2 only
(c) 2, 3, 1 (d) 3, 2, 1 (c) 1 and 3 only (d) 1, 2 and 3
39. In India, in the overall Index of Industrial Production, the 45. Consider the following factors:
Indices of Eight Core Industries have a combined weight of 1. Rotation of the Earth
37.90%. Which of the following are among those Eight Core 2. Air pressure and wind
Industries? 3. Density of ocean water
1. Cement 2. Fertilizers 4. Revolution of the Earth
3. Natural gas 4. Refinery products Which of the above factors influence the ocean currents?
5. Textiles
(a) 1 and 2 only (b) 1, 2 and 3
Select the correct answer using the codes given below :
(c) 1 and 4 (d) 2, 3 and 4
(a) 1 and 5 only (b) 2, 3 and 4 only
46. With reference to the wetlands of India, consider the
(c) 1, 2, 3 and 4 only (d) 1, 2, 3, 4 and 5
following statements :
40. Which of the following provisions of the Constitution of
1. The country’s total geographical area under the
India have a bearing on Education?
category of wetlands is recorded more in Gujarat as
1. Directive Principles of State Policy
compared to other States.
2. Rural and Urban Local Bodies
2. In India, the total geographical area of coastal wetlands
3. Fifth Schedule
is larger than that of inland wetlands.
4. Sixth Schedule
Which of the statements given above is/are correct?
5. Seventh Schedule
Select the correct answer using the codes given below : (a) 1 only (b) 2 only
(a) 1 and 2 only (b) 3, 4 and 5 only (c) Both 1 and 2 (d) Neither 1 nor 2
(c) 1, 2 and 5 only (d) 1, 2, 3, 4 and 5 47. Consider the following crops of India :
41. Government of India encourages the cultivation of ‘sea 1. Groundnut 2. Sesamum
buckthorn’. What is the importance of this plant ? 3. Pearl millet
1. It helps in controlling soil erosion and in preventing Which of the above is/are predominantly rainfed crop/crops?
desertification. (a) 1 and 2 only (b) 2 and 3 only
2. It is a rich source of biodiesel. (c) 3 only (d) 1, 2 and 3
3. It has nutritional value and is well-adapted to live in 48. When you travel in Himalayas, you will see the following :
cold areas of high altitudes.
4. Its timber is of great commercial value. 1. Deep gorges
Which of the statements given above is/are correct? 2. U-turn river courses
(a) 1 only (b) 2, 3 and 4 only 3. Parallel mountain ranges
(c) 1 and 3 only (d) 1, 2, 3 and 4 4. Steep gradients causing land-sliding
42. Which of the following is the chief characteristic of ‘mixed Which of the above can be said to be the evidences for
farming’? Himalayas being young fold mountains?
(a) Cultivation of both cash crops and food crops (a) 1 and 2 only (b) 1, 2 and 4 only
(b) Cultivation of two or more crops in the same field (c) 3 and 4 only (d) 1, 2, 3 and 4
(c) Rearing of animals and cultivation of crops together 49. Normally, the temperature decreases with the increase in
(d) None of the above height from the Earth’s surface, because
43. A particular State in India has the following characteristics : 1. the atmosphere can be heated upwards only from the
Earth's surface
1. It is located on the same latitude which passes through 2. there is more moisture in the upper atmosphere
northern Rajasthan. 3. the air is less dense in the upper atmosphere
2. It has over 80% of its area under forest cover. Select the correct answer using the codes given below :
3. Over 12% of forest cover constitutes Protected Area (a) 1 only (b) 2 and 3 only
Network in this State. (c) 1 and 3 only (d) 1, 2 and 3
50. The acidification of oceans is increasing. Why is this 54. With reference to the Delimitation Commission, consider the
phenomenon a cause of concern? following statements :
1. The growth and survival of calcareous phytoplankton 1. The orders of the Delimitation Commission cannot be
will be adversely affected. challenged in a Court of Law.
2. The growth and survival of coral reefs will be adversely 2. When the orders of the Delimitation Commission are
affected. laid before the Lok Sabha or State Legislative Assembly,
they cannot effect any modifications in the orders.
3. The survival of some animals that have phytoplanktonic
Which of the statements given above is/are correct?
larvae will be adversely affected.
(a) 1 only (b) 2 only
4. The cloud seeding and formation of clouds will be
(c) Both 1 and 2 (d) Neither 1 nor 2
adversely affected.
55. Consider the following:
Which of the statements given above is /are correct? 1. Hotels and restaurants
(a) 1, 2 and 3 only (b) 2 only 2. Motor transport undertakings
(c) 1 and 3 only (d) 1, 2, 3 and 4 3. Newspaper establishments
51. In India, other than ensuring that public funds are used 4. Private medical institutions
efficiently and for intended purpose, what is the importance The employees of which of the above can have the ‘Social
of the office of the Comptroller and Auditor General (CAG)? Security’ coverage under Employees’ State Insurance
Scheme?
1. CAG exercises exchequer control on behalf of the (a) 1, 2 and 3 only (b) 4 only
Parliament when the President of India declares national (c) 1, 3 and 4 only (d) 1, 2, 3 and 4
emergency/financial emergency. 56. According to the Constitution of India, it is the duty of
2. CAG r epor ts on the execution of projects or the President of India to cause to be laid before the Parliament
programmes by the ministries are discussed by the which of the following?
Public Accounts Committee. 1. The Recommendations of the Union Finance
Commission
3. Information from CAG reports can be used by
2. The Report of the Public Accounts Committee
investigating agencies to press charges against those
who have violated the law while managing public 3. The Report of the Comptroller and Auditor General
finances. 4. The Report of the National Commission for Scheduled
Castes
4. While dealing with the audit and accounting of
Select the correct answer using the codes given below :
government companies, CAG has certain judicial powers
(a) 1 only (b) 2 and 4 only
for prosecuting those who violate the law.
(c) 1, 3 and 4 only (d) 1, 2, 3 and 4
Which of the statements given above is/are correct? 57. A deadlock between the Lok Sabha and the Rajya Sabha
(a) 1, 3 and 4 only (b) 2 only calls for a joint sitting of the Parliament during the
(c) 2 and 3 only (d) 1, 2, 3 and 4 passage of
52. The endeavour of ‘Janani Suraksha Yojana’ Programme is 1. Ordinary Legislation
2. Money Bill
1. to promote institutional deliveries 3. Constitution Amendment Bill
2. to provide monetary assistance to the mother to meet Select the correct answer using the codes given below :
the cost of delivery (a) 1 only (b) 2 and 3 only
(c) 1 and 3 only (d) 1, 2 and 3
3. to provide for wage loss due to pregnancy and
confinement 58. How do District Rural Development Agencies (DRDAs) help
in the reduction of rural poverty in India?
Which of the statements given above is/are correct?
1. DRDAs act as Panchayati Raj Institutions in certain
(a) 1 and 2 only (b) 2 only specified backward regions of the country.
(c) 3 only (d) 1, 2 and 3 2. DRDAs undertake area-specific scientific study of the
53. The Prime Minister of India, at the time of his/her causes of poverty and malnutrition and prepare detailed
appointment remedial measures.
(a) need not necessarily be a member of one of the Houses 3. DRD As secure inter-sectoral and inter-departmental
of the Parliament but must become a member of one of coordination and cooperation for effective implementation
the Houses within six months of anti-poverty programmes.
(b) need not necessarily be a member of one of the Houses 4. DRDAs watch over and ensure effective utilization of
of the Parliament but must become a member of the Lok the funds intended for anti-poverty programmes.
Sabha within six months Which of the statements given above is/are correct?
(c) must be a member of one of the Houses of the Parliament (a) 1, 2 and 3 only (b) 3 and 4 only
(d) must be a member of the Lok Sabha (c) 4 only (d) 1, 2, 3 and 4
59. Which of the following is/are among the Fundamental Duties 3. There is an apprehension that the consumption of Bt
of citizens laid down in the Indian Constitution? brinjal may have adverse impact on health.
1. To preserve the rich heritage of our composite culture 4. There is some concern that the introduction of Bt brinjal
2. To protect the weaker sections from social injustice may have adverse effect on the biodiversity.
3. To develop the scientific temper and spirit of inquiry Select the correct answer using the codes given below :
4. To strive towards excellence in all spheres of individual (a) 1, 2 and 3 only (b) 2 and 3 only
and collective activity (c) 3 and 4 only (d) 1, 2, 3 and 4
Select the correct answer using the codes given below : 65. Other than resistance to pests, what are the prospects for
(a) 1 and 2 only (b) 2 only which genetically engineered plants have been created?
(c) 1, 3 and 4 only (d) 1, 2, 3 and 4
60. What is the provision to safeguard the autonomy of the 1. To enable them to withstand drought
Supreme Court of India? 2. To increase the nutritive value of the produce
1. While appointing the Supreme Court Judges, the 3. To enable them to grow and do photosynthesis in
President of India has to consult the Chief Justice of spaceships and space stations
India. 4. To increase their shelf life
2. The Supreme Court Judges can be removed by the Chief Select the correct answer using the codes given below :
Justice of India only. (a) 1 and 2 only (b) 3 and 4 only
3. The salaries of the Judges are charged on the (c) 1, 2 and 4 only (d) 1, 2, 3 and 4
Consolidated Fund of India to which the legislature 66. Consider the following statements :
does not have to vote. The most effective contribution made by Dadabhai Naoroji
4. All appointments of officers and staffs of the Supreme to the cause of Indian National Movement was that he
Court of India are made by the Government only after
consulting the Chief Justice of India. 1. exposed the economic exploitation of India by the
Which of the statements given above is/are correct? British
(a) 1 and 3 only (b) 3 and 4 only 2. interpreted the ancient Indian texts and restored the
(c) 4 only (d) 1, 2, 3 and 4 self-confidence of Indians
61. To meet its rapidly growing energy demand, some opine that 3. stressed the need for eradication of all the social evils
India should pursue research and development on thorium as before anything else
the future fuel of nuclear energy. In this context, what Which of the statements given above is/are correct?
advantage does thorium hold over uranium? (a) 1 only (b) 2 and 3 only
1. Thorium is far more abundant in nature than uranium. (c) 1 and 3 only (d) 1, 2 and 3
2. On the basis of per unit mass of mined mineral, thorium 67. With reference to Dhrupad, one of the major traditions of
can generate more energy compared to natural uranium. India that has been kept alive for centuries, which of the
3. Thorium produces less harmful waste compared to following statements are correct?
uranium. 1. Dhrupad originated and developed in the Rajput
Which of the statements given above is/are correct? kingdoms during the Mughal period.
(a) 1 only (b) 2 and 3 only 2. Dhrupad is primarily a devotional and spiritual music.
(c) 1 and 3 only (d) 1, 2 and 3 3. Dhrupad Alap uses Sanskrit syllables from Mantras.
62. The increasing amount of carbon dioxide in the air is slowly Select the correct answer using the codes given below :
raising the temperature of the atmosphere, because it absorbs (a) 1 and 2 only (b) 2 and 3 only
(c) 1, 2 and 3 (d) None of the above is correct
(a) the water vapour of the air and retains its heat 68. How do you distinguish between Kuchipudi and
(b) the ultraviolet part of the solar radiation Bharatanatyam dances?
(c) all the solar radiations 1. Dancers occasionally speaking dialogues is found in
(d) the infrared part of the solar radiation Kuchipudi dance but not in Bharatanatyam.
63. Which one of the following sets of elements was primarily 2. Dancing on the brass plate by keeping the feet on its
responsible for the origin of life on the Earth? edges is a feature of Bharatanatyam but Kuchipudi
(a) Hydrogen, Oxygen, Sodium dance does not have such a form of movements.
(b) Carbon, Hydrogen, Nitrogen Which of the statements given above is/are correct?
(c) Oxygen, Calcium, Phosphorus (a) 1 only (b) 2 only
(d) Carbon, Hydrogen, Potassium (c) Both 1 and 2 (d) Neither 1 nor 2
64. What are the reasons for the people’s resistance to the 69. With reference to the religious history of medieval India, the
introduction of Bt brinjal in India? Sufi mystics were known to pursue which of the following
1. Bt brinjal has been created by inserting a gene from a practices?
soil fungus into its genome. 1. Meditation and control of breath
2. The seeds of Bt brinjal are terminator seeds and 2. Severe ascetic exercises in a lonely place
therefore, the farmers have to buy the seeds before 3. Recitation of holy songs to arouse a state of ecstasy in
every season from the seed companies. their audience
Select the correct answer using the codes given below : 76. Under which of the following circumstances may ‘capital
(a) 1 and 2 only (b) 2 and 3 only gains’ arise?
(c) 3 only (d) 1, 2 and 3 1. When there is an increase in the sales of a product
70. The Rowlatt Act aimed at 2. When there is a natural increase in the value of the
(a) compulsory economic support to war efforts property owned
(b) imprisonment without trial and summary procedures 3. When you purchase a painting and there is a growth in
for trial its value due to increase in its popularity
(c) suppression of the Khilafat Movement Select the correct answer using the codes given below :
(d) imposition of restrictions on freedom of the press (a) 1 only (b) 2 and 3 only
71. The Lahore Session of the Indian National Congress (1929) (c) 2 only (d) 1, 2 and 3
is very important in history, because 77. Which of the following measures would result in an increase
1. the Congress passed a resolution demanding complete in the money supply in the economy?
independence 1. Purchase of government securities from the public by
2. the rift between the extremists and moderates was the Central Bank
resolved in that Session 2. Deposit of currency in commercial banks by the public
3. a resolution was passed rejecting the two-nation theory 3. Borrowing by the government from the Central Bank
in that Session 4. Sale of government securities to the public by the
Central Bank
Which of the statements given above is/are correct?
Select the correct answer using the codes given below :
(a) 1 only (b) 2 and 3
(a) 1 only (b) 2 and 4 only
(c) 1 and 3 (d) None of the above
(c) 1 and 3 (d) 2, 3 and 4
72. Lord Buddha’s image is sometimes shown with the hand
78. Which of the following would include Foreign Direct
gesture called ‘Bhumisparsha Mudra’. It symbolizes
Investment in India?
1. Subsidiaries of foreign companies in India
(a) Buddha’s calling of the Earth to watch over Mara and 2. Majority foreign equity holding in Indian companies
to prevent Mara from disturbing his meditation 3. Companies exclusively financed by foreign companies
(b) Buddha’s calling of the Earth to witness his purity and 4. Portfolio investment
chastity despite the temptations of Mara Select the correct answer using the codes given below :
(c) Buddha’s reminder to his followers that they all arise (a) 1, 2, 3 and 4 (b) 2 and 4 only
from the Earth and finally dissolve into the Earth, and (c) 1 and 3 only (d) 1, 2 and 3 only
thus this life is transitory 79. Consider the following statements:
(d) Both the statements (a) and (b) are correct in this context The price of any currency in international market is decided
73. The religion of early Vedic Aryans was primarily of by the
1. World Bank
(a) Bhakti 2. demand for goods/services provided by the country
(b) image worship and Yajnas concerned
(c) worship of nature and Yajnas 3. stability of the government of the concerned country
(d) worship of nature and Bhakti 4. economic potential of the country in question
74. Which of the following statements is/are correct regarding Which of the statements given above are correct?
Brahmo Samaj? (a) 1, 2, 3 and 4 (b) 2 and 3 only
1. It opposed idolatry. (c) 3 and 4 only (d) 1 and 4 only
2. It denied the need for a priestly class for interpreting 80. The basic aim of Lead Bank Scheme is that
the religious texts. (a) big banks should try to open offices in each district
3. It popularized the doctrine that the Vedas are infallible. (b) there should be stiff competition among the various
Select the correct answer using the codes given below : nationalized banks
(a) 1 only (b) 1 and 2 only (c) individual banks should adopt particular districts for
(c) 3 only (d) 1, 2 and 3 intensive development
75. The Reserve Bank of India (RBI) acts as a bankers’ bank. (d) all the banks should make intensive efforts to mobilize
This would imply which of the following? deposits
1. Other banks retain their deposits with the RBI. 81. Consider the following :
2. The RBI lends funds to the commercial banks in times 1. Assessment of land revenue on the basis of nature of
of need. the soil and the quality of crops
3. The RBI advises the commercial banks on monetary 2. Use of mobile cannons in warfare
matters. 3. Cultivation of tobacco and red chillies
Select the correct answer using the codes given below : Which of the above was/were introduced into India by the
(a) 2 and 3 only (b) 1 and 2 only English?
(a) 1 only (b) 1 and 2
(c) 1 and 3 only (d) 1, 2 and 3
(c) 2 and 3 (d) None
82. With reference to the guilds (Shreni) of ancient India that Which of the above are declared Tiger Reserves?
played a very important role in the country's economy, which (a) 1 and 2 only (b) 1, 3 and 4 only
of the following statements is/are correct? (c) 2, 3 and 4 only (d) l, 2, 3 and 4
1. Every guild was registered with the central authority of 88. Consider the following statements :
the State and the king was the chief administrative 1. The duration of the monsoon decreases from southern
authority on them. India to northern India.
2. The wages, rules of work, standards and prices were 2. The amount of annual rainfall in the northern plains of
fixed by the guild. India decreases from east to west.
3. The guild had judicial powers over its own members. Which of the statements given above is/are correct?
Select the correct answer using the codes given below : (a) 1 only (b) 2 only
(a) 1 and 2 only (b) 3 only (c) Both 1 and 2 (d) Neither 1 nor 2
(c) 2 and 3 only (d) 1, 2 and 3 89. Which one of the following is the characteristic climate of
83. The distribution of powers between the Centre and the the Tropical Savannah Region?
States in the Indian Constitution is based on the scheme (a) Rainfall throughout the year
provided in the (b) Rainfall in winter only
(a) Morley-Minto Reforms, 1909 (c) An extremely short dry season
(b) Montagu-Chelmsford Act, 1919 (d) A definite dry and wet season
(c) Government of India Act, 1935 90. In which one among the following categories of protected
(d) Indian Independence Act, 1947 areas in India are local people not allowed to collect and use
84. Despite having large reserves of coal, why does India import the biomass?
millions of tonnes of coal? (a) Biosphere Reserves
1. It is the policy of India to save its own coal reserves for (b) National Parks
future, and import it from other countries for the present
(c) Wetlands declared under Ramsar Convention
use.
(d) Wildlife Sanctuaries
2. Most of the power plants in India are coal-based and
91. Consider the following kinds of organisms :
they are not able to get sufficient supplies of coal from
1. Bat 2. Bee
within the country.
3. Bird
3. Steel companies need large quantity of coking coal
Which of the above is/are pollinating agent/agents?
which has to be imported.
Which of the statements given above is/are correct? (a) 1 and 2 only (b) 2 only
(a) 1 only (b) 2 and 3 only (c) 1 and 3 only (d) 1, 2 and 3
(c) 1 and 3 only (d) 1, 2 and 3 92. Which one of the following groups of animals belongs to
85. A person stood alone in a desert on a dark night and wanted the category of endangered species?
to reach his village which was situated 5 km east of the point (a) Great Indian Bustard, Musk Deer, Red Panda and Asiatic
where he was standing. He had no instruments to find the Wild Ass
direction but he located the polestar. The most convenient (b) Kashmir Stag, Cheetal, Blue Bull and Great Indian
way now to reach his village is to walk in the Bustard
(a) direction facing the polestar (c) Snow Leopard, Swamp Deer, Rhesus Monkey and Saras
(b) direction opposite to the polestar (Crane)
(c) direction keeping the polestar to his left (d) Lion-tailed Macaque, Blue Bull, Hanuman Langur and
(d) direction keeping the polestar to his right Cheetal
86. Recently, there has been a concern over the short supply of 93. Consider the following statements :
a group of elements called ‘rare earth metals’. Why? If there were no phenomenon of capillarity
1. it would be difficult to use a kerosene lamp
1. China, which is the largest producer of these elements, 2. one would not be able to use a straw to consume a soft
has imposed some restrictions on their export. drink
2. Other than China, Australia, Canada and Chile, these 3. the blotting paper would fail to function
elements are not found in any country. 4. the big trees that we see around would not have grown
3. Rare earth metals are essential for the manufacture of on the Earth
various kinds of electronic items and there is a growing Which of the statements given above are correct?
demand for these elements. (a) 1, 2 and 3 only (b) 1, 3 and 4 only
Which of the statements given above is/are correct? (c) 2 and 4 only (d) 1, 2, 3 and 4
(a) 1 only (b) 2 and 3 only 94. The Millennium Ecosystem Assessment describes the
(c) 1 and 3 only (d) 1, 2 and 3 following major categories of ecosystem services-
87. Consider the following protected areas: provisioning, supporting, regulating, preserving and cultural.
1. Bandipur 2. Bhitarkanika Which one of the following is supporting service?
3. Manas 4. Sunderbans
(a) Production of food and water 98. Consider the following agricultural practices :
(b) Control of climate and disease
(c) Nutrient cycling and crop pollination 1. Contour bunding 2. Relay cropping
(d) Maintenance of diversity 3. Zero tillage
95. What is the difference between the antelopes Oryx and Chiru?
In the context of global climate change, which of the above
helps/help in carbon sequestration/storage in the soil?
(a) Oryx is adapted to live in hot and arid areas whereas
Chiru is adapted to live in steppes and semi-desert areas (a) 1 and 2 only (b) 3 only
of cold high mountains (c) 1, 2 and 3 (d) None of them
(b) Oryx is poached for its antlers whereas Chiru is poached 99. What would happen if phytoplankton of an ocean is
for its musk completely destroyed for some reason?
(c) Oryx exists in western India only whereas Chiru exists 1. The ocean as a carbon sink would be adversely
in north-east India only affected.
(d) None of the statements (a), (b) and (c) given above is 2. The food chains in the ocean would be adversely
correct affected.
96. Which of the following can be threats to the biodiversity of 3. The density of ocean water would drastically decrease.
a geographical area?
Select the correct answer using the codes given below :
1. Global warming
2. Fragmentation of habitat (a) 1 and 2 only (b) 2 only
3. Invasion of alien species (c) 3 only (d) 1, 2 and 3
4. Promotion of vegetarianism 100. Vultures which used to be very common in Indian countryside
Select the correct answer using the codes given below : some years ago are rarely seen nowadays. This is attributed
(a) 1, 2 and 3 only (b) 2 and 3 only to
(c) 1 and 4 only (d) 1, 2, 3 and 4 (a) the destruction of their nesting sites by new invasive
97. Consider the following : species
1. Black-necked crane (b) a drug used by cattle owners for treating their diseased
2. Cheetah cattle
3. Flying squirrel (c) scarcity of food available to them
4. Snow leopard
(d) a widespread, persistent and fatal disease among them
Which of the above are naturally found in India?
(a) 1, 2 and 3 only (b) 1, 3 and 4 only
(c) 2 and 4 only (d) 1, 2, 3 and 4

ANSWERS AND EXPLANATIONS


1 (d) According to Panchayat Extension to the Scheduled 3 (d) National Biodiversity Authority (NBA) checks the
Areas) Act 1996, Gram Sabha has the power to prevent biopiracy and protects the indigeneous and traditional
alienation of land in the Scheduled Areas, has the genetic resources. It directly monitors and supervises
owner ship of minor forest produce and the the scientific research on genetic modification of crop
recommendation of Gram Sabha is required for granting plants application for intellectual property Rights related
prospecting licence or mining lease for any mineral in
to genetic biological resources cannot be made without
the Scheduled Areas.
2. (a) Adjournment motion : the approval of NBA.
(i) It is introduced in the Parliament to draw attention of 4 (a) The National Green Tribunal Act, 2010 was enacted in
the house to a definite matter of urgent public consonance with the Right to healthy environment,
importance and needed to support of 50 members to be construed as a part of Right to life under Art. 21.
admitted. 5. (b) If National Water Mission is properly and completely
(ii) It is regarded as an extraordinary device, because it implemented, then the part of the water needs of urban
interrupts the normal business of the house. areas will be met through recycling of waste water and
(iii) It involves an element of censure against the
the water requirements of coastal cities with inadequate
government and Rajya Sabha is not permitted to make
use this device. alternative sources of water will be met by adopting
(iv) The discussion an adjournment motion should last for appropriate technologies that allow for the use of ocean
not less than two hours and thirty minutes. water.
6. (b) Organizing village Panchayats and promoting cottage 28. (b) As Indian National Congress did not want to include
industries in rural areas are the Gandhian principles social reforms in its deliberations, it decided to form a
that are reflected in the Directive Principles of State separate body for this purpose.
Policy. 29. (b) The parties established by Dr. B. R. Ambedkar were -
7. (d) None of the above statements are correct All India Scheduled Castes Federation and Independent
(i) Union Territories (Delhi and Pondicherry) are Labour Party.
represented in the Rajya Sabha. 30. (b) The Constitution of India empowering the Parliament
(ii) It is not within the purview of the Chief Election to make laws in the State List and to create one or more
Commissioner to adjudicate election disputes. It is the All India Services.
Supreme Court and High Court which look into the 31. (b) 1st and 2nd are correct.
disputes 32. (a) Deprivation of education, health, assets and services
(iii) According to the Constitution of India, the Parliament at household level
consists of the Lok Sabha the Rajya Sabha and the 33. (a) A design for the Goods and Services Tax, and a
President of India. compensation package linked to adherence to the
8. (c) Consumers are empowered to take samples for food proposed design
testing. In case of death of a consumer his/her legal 34. (c) 1. Setting up of National Investment and
heir can file a complaint in the consumer forum on his / Manufacturing Zones
her behalf. 2. Establishing the Technology Acquisition and
9. (b) If the Lok Sabha Speaker wants to resign, the letter of Development Fund
his / her resignation has to be addressed to the Deputy 35. (a) 1. Placing Annual Financial Statement before the
Speaker. Parliament
10 (c) The Original Jurisdiction of the Supreme Court includes 2. Withdrawal of moneys from Consolidated Fund
dispute between the Government of India and one or of India only after passing the Appropriation Bill
more States, and dispute between two and more States. 3. Provisions of supplementary grants and vote-on-
11. (d) All of them are employed as Biopesticides. account
12. (c) Coconut shells, groundnut shells and rice husk can be 5. Introducing Finance Bill in the Parliament
used in biomass gassfication.
36. (c) Mahatma Gandhi undertook a fast unto death in 1932,
Combustible gases generated from biomass gasification against Ramsay Macdonald Announcement of the
can be used for direct heat generation but not in internal Communal Award.
combustion engines.
37. (b) Ryotwari settlement, the rent was paid directly by the
13. (a) Ultraviolet radiation inactivates / kills harmful micro peasants to the Government and the Government gave
organisms in water only. pattas to the Ryots.
14. (c) Graphene is a two-dimensional material and has good
38. (c) 2, 3, 1, i.e. High birthrate with high death rate, High
electrical conduction. It is one of the thinnest but
birthrate with low death rate, Low birth rate with low
strongest materials tested so far. It can be used for
conducting electrodes required for touch screens, LCDs death rate.
and organic LEDs. 39. (c) Except textiles, all are core industries.
15. (a) Smelting units, pens and pencils and paints. 40. (c) 1, 2 & 5 are correct.
16. (c) Stem cells can be used for medical therapies. 41. (d) All are correct
17. (c) Chlorofluorocarbons are used in the production of 42. (c) Mixed farming refer to rearing of animals and cultivation
plastic foams, in cleaning electronic components and of crops together.
as pressurizing agents in aerosol cans. 43. (a) Arunanchal Pradesh
18. (b) It will help probe the possibility of the existence of 44. (d) All are used as pulse, folder and green manure ?
stars and galaxies made of anti-mater. It will also help in 45. (b) 1. Rotation of the Earth
understanding the evolution of the universe 2. Air pressure and wind
19. (a) Only 1st and 2nd are correct. 3. Density of ocean water
20. (a) Earth's magnetic field diverts them torwards its poles. 46. (a) The countries total geographical area under the
21. (c) 1st, 2nd and 3rd are correct. category of wetlands is more in Gujarat as compared to
22. (b) Both Buddhism and Jainism denied the authority of the other states.
Vedas the efficacy of Rituals. 47. (d) All of them are predominantly rain fed crops
23. (c) 2nd, 3rd and 4th are correct. 48. (d) All are correct.
24. (c) Nagara, Dravida and Vesara are three main styles of 49. (c) 1. the atmosphere can be heated upwards only from
Indian temple architecture. the Earth's surface
25. (d) d is correct option. 2. there is more moisture in the upper atmosphere
26. (d) All are correct. 3. the air is less dense in the upper atmosphere
27. (c) 1st and 3rd are correct. 50. (a) 1, 2 and 3 only are correct.
51. (c) Only 2nd and 3rd are correct statements. 78. (d) All statements except 4th regarding F.D.I in India are
(2) CAG reports on execution of projects or programmes correct.
by the ministries are discussed by the Public Accounts 79. (b) Price of any currency in international market is
Committee. determined by the demand for goods/services produced
(3) Information from CAG reports can be used by by the country. If the demand for the product is high
investigating agencies to press charges against those then the currency would be strong. The current
who have violated the law while managing public depreciation of the Indian Rupee is due to rising oil
finance. prices. The stability of the government of the concerned
52. (a) The endeavour of Janani Suraksha Yojana programme country has no role to play in determining the prices of
are : (i) to promote institutional deliveries (ii) to provide a currency.
monetary assistance to the mother to meet the cost of 80. (c) The basic aim of Lead Bank scheme is that the bank
delivery. should adopt particular districts for intensive
53. (a) Option a is correct. development by offering loans and banking services.
54. (c) Both are correct. Hence the option c is right. 81. (d) None of the above
55. (a) Private medical institutions do not come in this category. 82. (c) 2. The wages, rules of work, standards and prices
56. (c) It is not the duty of the President of India to cause to be were fixed by the guild.
laid report of public Accounts Committee before the 3. The guild had judicial powers over its own
Parliament. members.
57. (a) Only 3rd and 4th are correct, thus the option (b) is 83. (c) Distribution of power between the Centre and the States
right. in the Indian Constitution is based on the Government
58. (b) Only 3rd and 4th are correct thus the option (b) is right. of India Act. 1935.
59. (c) All the statements except 2 regarding the Fundamental 84. (b) 2nd and 3rd are correct statements.
Duties of citizens are correct. 85. (c)
60. (a) 1st and 3rd are correct statements thus option (a) is 86. (c) The statements, 1st and 2nd are correct. Hence the
right. option (C) is correct.
61. (a) Thorium is for more abundant in nature than uranium.
87. (b) Bandipur, Manas and Sunderbans are declared Tiger
62. (d) Carton dioxide absorbs the Infra Red part of Solar
Reserves
Radiation
88. (c) Both statements are correct.
63. (b) Carbon, Hydrogen, Nitrogen.
89. (d) The characteristic of the Tropical Savannah Region is
64. (b) 2nd and 3rd statements are correct hence the option
a definite dry and wet season.
(b) is right.
90. (b) In national parks, local people are not allowed to collect
65. (c) Except 3rd, all the statements are correct
and use the biomass available in the areas.
66. (a) He exposed the economic exploitation of India by the
British. 91. (d) All of them are pollinating agents.
67. (b) Dhrupad is a primarily a spiritual and devotional music 92. (a) Red Panda and Asiatic Wild Ass, are endangered
using Sanskrit syllables from mantras. species.
68. (a) Only statement 1, is correct 93. (b)
69. (d) All statements are correct, hence the option (d) is right. 1. it would be difficult to use a kerosene lamp
70. (b) The act had provisions of imprisonment without trial 3. the blotting paper would fail to function
and summary procedures for trial. 4. the big trees that we see around would not have grown
71. (a) Only 1st statement is correct. on the Earth
72. (d) Bhumisparsha Mudra of Lord Buddha, symbolizes that 94. (d) Maintenance of diversity.
Buddha is calling the earth to watch over Mara and to 95. (a) Oryx has adapted to live in hot and arid areas whereas
present Mara from disturbing his meditation. This image Chiru is adapted to live in steppes and semi-desert areas
of Buddha also symbolizes Buddha's calling the earth of cold high mountains.
to witness his purity and chastity despite the 96. (a) Except promotion of vegetarianism all other acts are
temptations of Mara threats to the biodiversity of a geographical area.
73. (c) The early Vedic Aryans indulged in Image Worship and 97. (b) Cheetah is not naturally found in India.
Yajnas. 98. (c) All the given agricultural practices help in carbon
74 (b) Statements 1 & 2 are correct. sequestration.
75. (d) All the statements are correct. 99. (a) If phytoplankton of an ocean is completely destroyed
76. (b) 2. When there is a natural increase in the value of for some reason then
the property owned (i) The ocean as a carbon sink would be adversely
3. When you purchase a painting and there is a affected.
growth in its value due to increase in its popularity (ii) The food chains in the ocean would be adversely
77. (c) 1. Purchase of government securities from the public affected.
by the Central Bank 100. (b) A drug used by cattle owners for treating their diseased
3. Borrowing by the government from the Central cattle.
Bank
INSTRUCTIONS
1. This Test Booklet contains 80 items (questions). Each item comprises four responses (answers). You will select
the response which you want to mark on the Answer sheet. In case, you feel that there is more than one correct
response, mark the response which you consider the best. In any case, choose ONLY ONE response for each
item.
2. All items carry equal marks.
3. Penalty for wrong answers:
THERE WILL BE PENALTY FOR WRONG ANSWERS MARKED BY THE CANDIDATE EXCEPT FOR
QUESTIONS FROM 73 TO 80, WHICH DO NOT CARRY ANY PENALTY FOR WRONG ANSWER.
(i) There are four alternatives for the answer to every question. For each question which has a penalty for which a
wrong answer has been given by the candidate, one-third of the marks assigned to that question will be
deducted as penalty.
(ii) If a candidate gives more than one answer, it will be treated as a wrong answer even if one of the given answers
happens to be correct and there will be same penalty as above to that question, if it has a penalty.
(iii) If a question is left blank, i.e., no answer is given by the candidate, there will be no penalty for that question.

Directions for the following 15 (fifteen) items : 2. According to the passage, education must be respected in
itself because
Read the following three passages and answer the items that
(a) it helps to acquire qualifications for employment
follow each passage. Your answers to these items should be based
(b) it helps in upward mobility and acquiring social status
on the passages only.
(c) it is an inner process of moral and intellectual
PASSAGE-1 development
Education, without a doubt, has an important functional, (d) All the (a), (b) and (c) given above are correct in this
instrumental and utilitarian dimension. This is revealed when one context.
asks questions such as ‘what is the purpose of education?’. The 3. Education is a process in which
answers, too often, are ‘to acquire qualifications for employment/ (a) students are converted into trained professionals.
upward mobility’, ‘wider/higher (in terms of income) opportunities’, (b) opportunities for higher income are generated.
and ‘to meet the needs for trained human power in diverse fields
(c) individuals develop self-critical awareness and
for national development’. But in its deepest sense education is
independence of thought.
not instrumentalist. That is to say, it is not to be justified outside
(d) qualifications for upward mobility are acquired.
of itself because it leads to the acquisition of formal skills or of
certain desired psychological – social attributes. It must be PASSAGE-2
respected in itself. Education is thus not a commodity to be acquired Chemical pesticides lose their role in sustainable agriculture
or possessed and then used, but a process of inestimable if the pests evolve resistance. The evolution of pesticide resistance
importance to individuals and society, although it can and does is simply natural selection in action. It is almost certain to occur
have enormous use value. Education then, is a process of expansion when vast numbers of a genetically variable population are killed.
and conversion, not in the sense of converting or turning students One or a few individuals may be unusually resistant (perhaps
into doctors or engineers, but the widening and turning out of the because they possess an enzyme that can detoxify the pesticide).
mind—the creation, sustenance and development of self-critical If the pesticide is applied repeatedly, each successive generation
awareness and independence of thought. It is an inner process of of the pest will contain a larger proportion of resistant individuals.
moral-intellectual development. Pests typically have a high intrinsic rate of reproduction, and so a
1. What do you understand by the ‘instrumentalist’ view of few individuals in one generation may give rise to hundreds or
education? thousands in the next, and resistance spreads very rapidly in a
(a) Education is functional and utilitarian in its purposes. population.
(b) Education is meant to fulfil human needs. This problem was often ignored in the past, even though
(c) The purpose of education is to train the human intellect. the first case of DDT (dichlorodiphenyltrichloroethane) resistance
(d) Education is meant to achieve moral development. was reported early as 1946. There is exponential increase in the
numbers of invertebrates that have evolved resistance and in the 7. How do pesticides act as agents for the selection of resistant
number of pesticides against which resistance has evolved. individuals in any pest population?
Resistance has been recorded in every family of arthropod pests 1. It is possible that in a pest population the individuals
(including dipterans such as mosquitoes and house flies, as well will behave differently due to their genetic makeup.
as beetles, moths, wasps, fleas, lice and mites) as well as in weeds 2. Pests do possess the ability to detoxify the pesticides.
and plant pathogens. Take the Alabama leaf-worm, a moth pest of 3. Evolution of pesticide resistance is equally distributed
cotton, as an example. It has developed resistance in one or more in pest population.
regions of the world to aldrin, DDT, dieldrin, endrin, lindane and Which of the statements given above is/are correct?
toxaphene. (a) 1 only (b) 1 and 2 only
If chemical pesticides brought nothing but, problems, — if (c) 3 only (d) 1, 2 and 3
their use was intrinsically and acutely unsustainable — then they 8. Why is the use of chemical pesticides generally justified by
would already have fallen out of widespread use. This has not giving the examples of poor and developing countries?
happened. Instead, their rate of production has increased rapidly. 1. Development countries can afford to do away with use
The ratio of cost to benefit for the individual agricultural producer of pesticides by adapting to organic farming, but it is
has remained in favour of pesticide use. In the USA, insecticides imperative for poor and developing countries to use
have been estimated to benefit the agricultural products to the chemical pesticides.
tune of around $5 for every $1 spent. 2. In poor and developing countries, the pesticide
Moreover, in many poorer countries, the prospect of imminent addresses the problem of epidemic diseases of crops
mass starvation, or of an epidemic disease, are so frightening that and eases the food problem.
the social and health costs of using pesticides have to be ignored. 3. The social and health costs of pesticide use are generally
In general the use of pesticides is justified by objective measures ignored in poor and developing countries.
such as ‘lives saved’, ‘economic efficiency of food production’ Which of the statements given above is/are correct?
and ‘total food produced’. In these very fundamental senses, their (a) 1 only (b) 1 and 2 only
use may be described as sustainable. In practice, sustainability (c) 2 only (d) 1, 2 and 3
depends on continually developing new pesticides that keep at 9. What does the passage imply?
least one step ahead of the pests – pesticides that are less
(a) Alternative options to chemical pesticides should be
persistent, biodegradable and more accurately targeted all the
promoted.
pests.
(b) Too much use of chemicals is not good for the
4. “The evolution of pesticide resistance is natural selection in
ecosystem.
action.” What does it actually imply?
(c) There is no scope for the improvement of pesticides
(a) It is very natural for many organisms to have pesticide
and making their use sustainable.
resistance.
(d) Both the statements (a) and (b) above are correct.
(b) Pesticide resistance among organisms is a universal
phenomenon. PASSAGE-3
(c) Some individuals in any given population show Today's developing economices use much less energy per
resistance after the application of pesticides. capita than developed countries such as the United States did at
(d) None of the statements (a), (b) and (c) given above is similar incomes, showing the potential for lower-carbon growth.
correct. Adaptation and mitigation need to be integrated into a climate-
5. With reference to the passage, consider the following smart development strategy that increases resilience, reduces the
statements: threat of further global warming, and improves development
1. Use of chemical pesticides has become imperative in all outcomes. Adaptation and mitigation measures can advance
the poor countries of the world. development, and prosperity can raise incomes and foster better
2. Chemical pesticides should not have any role in institutions. A healthier population living in better-built houses
sustainable agriculture. and with access to bank loans and social security is better
3. One pest can develop resistance to many pesticides. equipped to deal with a changing climate and its consequences.
Which of the statements given above is/are correct? Advancing robust, resilient development policies that promote
(a) 1 and 2 only (b) 3 only adaptation is needed today because changes in the climate, already
(c) 1 and 3 only (d) 1, 2 and 3 begun, will increase even in the short term.
6. Though the problems associated with the use of chemical The spread of economic prosperity has always been
pesticides is known for a long time, their widespread use intertwined with adaptation to changing ecological conditions.
has not waned. Why? But as growth has altered the environment and as environmental
(a) Alternatives to chemical pesticides do not exist at all. change has accelerated, sustaining growth and adaptability
(b) New pesticides are not invented at all. demands greater capacity to understand our environment, generate
(c) Pesticides are biodegradable. new adaptive technologies and practices, and diffuse them widely.
(d) None of the statements (a), (b) and (c) given above is As economic historians have explained, much of humankind's
correct. creative potential has been directed at adapting to the changing
world. But adaptation cannot cope with all the impacts related to Select the correct answer using the codes given below:
climate change, especially as larger changes unfold in the long (a) 1 only (b) 2 only
term. (c) Both 1 and 2 (d) Neither 1 nor 2
Countries cannot grow out of harm's way fast enough to 14. Consider the following assumptions :
match the changing climate. And some growth strategies, whether 1. Sustainable economic growth demands the use of
driven by the government or the market, can also add to creative potential of man.
vulnerability — particularly if they overexploit natural resources. 2. Intensive agriculture can lead to ecological backlash.
Under the Soviet development plan, irrigated cotton cultivation 3. Spread of economic prosperity can adversely affect the
expanded in water-stressed Central Asia and led to the near ecology and environment.
disappearance of the Aral Sea, threatening the livelihoods of With reference to the passage, which of the above
fishermen, herders and farmers. And clearing mangroves — the assumptions is/are valid?
natural coastal buffers against storm surges – to make way for (a) 1 only (b) 2 and 3 only
intensive farming or housing development , increases the physical (c) 1 and 3 only (d) 1, 2 and 3
vulnerability of coastal settlements, whether in Guinea or in 15. Which one of the following statements constitutes the
Louisiana. central theme of this passage?
10. Which of the following conditions of growth can add to (a) Countries with greater economic prosperity are better
vulnerability? equipped to deal with the consequences of climate
1. When the growth occurs due to excessive exploitation change.
of mineral resources and forests. (b) Adaptation and mitigation should be integrated with
2. When the growth brings about a change in humankind's development strategies.
creative potential. (c) Rapid economic growth should not be pursued by both
3. When the growth is envisaged only for providing developed and developing economies.
houses and social security to the people. (d) Some countries resort to overexploitation of natural
4. When the growth occurs due to emphasis on farming resources for the sake of rapid development.
only. 16. Consider the following statements:
Select the correct answer using the codes given below: 1. All artists are whimsical.
(a) 1 only 2. Some artists are drug addicts.
(b) 2, 3 and 4 only 3. Frustrated people are prone to become drug addicts.
(c) 1 and 4 only From the above three statements it may be concluded that
(d) 1, 2, 3 and 4 (a) Artists are frustrated.
11. What does low-carbon growth imply in the present context? (b) Some drug addicts are whimsical.
1. More emphasis on the use of renewable sources of (c) All frustrated people are drug addicts.
energy. (d) Whimsical people are generally frustrated.
2. Less emphasis on manufacturing sector and more 17. Examine the following statements:
emphasis on agriculture sector. 1. Either A and B are of the same age or A is older than B.
3. Switching over from monoculture practices to mixed 2. Either C and D are of the same age or D is older than C.
farming. 3. B is older than C.
4. Less demand for goods and services. Which one of the following conclusions can be drawn from
Select the correct answer using the codes given below: the above statements?
(a) 1 only (b) 2, 3 and 4 only (a) A is older than B (b) B and D are of the same age
(c) 1 and 4 only (c) D is older than C (d) A is older than C
(d) None of the above implies low-carbon growth 18. Examine the following statements:
12. Which of the following conditions is/are necessary for 1. Only those who have a pair of binoculars can become
sustainable economic growth? the members of the birdwatcher’s club.
1. Spreading of economic prosperity more.
2. Some members of the birdwatcher’s club have cameras.
2. Popularising/spreading of adaptive technologies
3. Those members who have cameras can take part in
widely.
photo-contests.
3. Investing on research in adaptation and mitigation
Which of the following conclusions can be drawn from the
technologies.
above statements?
Select the correct answer using the codes given below:
(a) All those who have a pair of binoculars are members of
(a) 1 only (b) 2 and 3 only
the birdwatcher’s club.
(c) 1 and 3 only (d) 1, 2 and 3
(b) All members of the birdwatcher’s club have a pair of
13. Which of the following inferences can be made from the
binoculars.
passage?
1. Rainfed crops should not be cultivated in irrigated areas. (c) All those who take part in photo-contests are members
2. Farming under water-deficient areas should not be a of the birdwatcher’s club.
part of development strategy. (d) No conclusion can be drawn.
19. During the last summer vacation. Ankit went to a summer (a) All expensive things are regarded as luxury.
camp where he took part in hiking, swimming and boating. (b) All essential things for learning are not luxury.
This summer, he is looking forward to a music camp where (c) Television is essential for learning.
he hopes to sing, dance and learn to play the guitar. (d) Television is not a luxury item.
Based on the above information, four conclusions, as given 24. Mr. Kumar drives to work at an average speed of 48 km per
below, have been made. Which one of these logically follows hour. The time taken to cover the first 60% of the distance is
from the information given above? 10 minutes more than the time taken to cover the remaining
(a) Ankit's parents want him to play the guitar. distance. How far is his office?
(b) Ankit prefers music to outdoor activities. (a) 30 km (b) 40 km
(c) Ankit goes to some type of camp every summer. (c) 45 km (d) 48 km
(d) Ankit likes to sing and dance. 25. Gita is prettier than Sita but not as pretty as Rita. Then,
20. Three persons A, B and C wore shirts of black, blue and (a) Sita is not as pretty as Gita.
orange colours (not necessarily in that order) and pants of (b) Sita is prettier than Rita.
green, yellow and orange colours (not necessarily in that (c) Rita is not as pretty as Gita.
order). No person wore shirt and pants of the same colour. (d) Gita is prettier than Rita.
Further, it is given that 26. Given that,
1. A did not wear shirt of black colour. 1. A is the brother of B.
2. B did not wear shirt of blue colour. 2. C is the father of A.
3. C did not wear shirt of orange colour. 3. D is the brother of E.
4. A did not wear pants of green colour. 4. E is the daughter of B.
5. B wore pants of orange colour. Then, the uncle of D is
What were the colours of the pants and shirt worn by C, (a) A (b) B
respectively? (c) C (d) E
(a) Orange and black (b) Green and blue 27. Examine the following statements:
(c) Yellow and blue (d) Yellow and black 1. Rama scored more than Rani.
21. Ten new TV shows started in January — 5 sitcoms, 3 drama 2. Rani scored less than Ratna.
and 2 news magazines. By April, only seven of the new 3. Ratna scored more than Rama.
shows were still on, five of them being sitcoms. Based on 4. Padma scored more than Rama but less than Ratna.
the above information, four conclusions, as given below, Who scored the highest?
have been made. Which one of these logically follows from (a) Rama (b) Padma
the information given above? (c) Rani (d) Ratna
(a) Only one news magazine show is still on.
(b) Only one of the drama shows is still on. Directions for the following 11 (eleven) items:
(c) At least one discontinued show was a drama.
Read the following three passages and answer the items that
(d) Viewers prefer sitcoms over drama.
follow each passage. Your answers to these items should be based
22. Read the passage given below and the two statements that
on the passages only.
follow (given on the basis of the passage):
Four men are waiting at Delhi airport for a Mumbai flight. PASSAGE-1
Two are doctors and the other two are businessmen. Two Invasions of exotic species into new geographic areas
speak Gujarati and two speak Tamil. No two of the same sometimes occur naturally and without human agency. However,
profession speak the same language. Two are Muslims and human actions have increased this trickle to a flood. Human-caused
two are Christians. No two of the same religion are of the introductions may occur either accidentally as a consequence of
same profession, nor do they speak the same language. The human transport, or intentionally but illegally to serve some private
Tamil-speaking doctor is a Christian. purpose or legitimately to procure some hoped-for public benefit
1. The Christian-businessman speaks Gujarati. by bringing a pest under control, producing new agricultural
2. The Gujarati-speaking doctor is a Muslim. products or providing novel recreational opportunities. Many
Which of the above statements is/are correct conclusion/ introduced species are assimilated into communities without much
conclusions? obvious effect. However, some have been responsible for dramatic
(a) 1 only (b) 2 only changes to native species and natural communities. For example,
(c) Both 1 and 2 (d) Neither 1 nor 2 the accidental introduction of the brown tree snake Boiga
23. Consider the following statement: irregularis into Guam, an island in the Pacific, has through nest
“Though quite expensive, television is not a luxury item, as predation reduced 10 endemic forest bird species to the point of
one can learn many things through television.” extinction.
Which one of the following is a valid inference from the One of the major reasons for the world's great biodiversity is
above statement? the occurrence of centers of endemism so that similar habitats in
different parts of the world are occupied by different groups of Which of the statements given above are correct?
species that happen to have evolved there. If every species (a) 1 and 2 (b) 2 and 3
naturally had access to everywhere on the globe, we might expect (c) 1 and 3 (d) 2 and 4
a relatively small number of successful species to become dominant 32. What can be the impact of invasion of exotic species on an
in each biome. The extent to which this homogenization can happen ecosystem?
naturally is restricted by the limited powers of dispersal of most 1. Erosion of endemic species.
species in the face of the physical barriers that exist to dispersal. 2. Change in the species composition of the community
By virtue of the transport opportunities offered by humans, these of the ecosystem.
barriers have been breached by an ever-increasing number of exotic Select the correct answer using the codes given below:
species. The effects of introductions have been to convert a hugely (a) 1 only (b) 2 only
diverse range of local community compositions into something (c) Both 1 and 2 (d) Neither 1 nor 2
much more homogeneous. PASSAGE-2
It would be wrong, however, to conclude that introducing Most champions of democracy have been rather reticent in
species to a region will inevitably cause a decline in species richness suggesting that democracy would itself promote development and
there. For example, there are numerous species of plants, enhancement of social welfare–they have tended to see them as
invertebrates and vertebrates found in continental Europe but good but distinctly separate and largely independent goals. The
absent from the British Isles (many because they have so far failed detractors of democracy, on the other hand, seemed to have been
to recolonize after the last glaciations). Their introduction would quite willing to express their diagnosis of what they see as serious
be likely to augment British biodiversity. The significant detrimental tensions between democracy and development. The theorists of
effect noted above arises where aggressive species provide a novel the practical split — ‘‘Make up your mind : do you want democracy,
challenge to endemic biotas ill-equipped to deal with them. or instead, do you want development ?’’ — often came, at least to
start with, from East Asian countries, and their voice grew in
28. With reference to the passage, which of the following influence as several of these countries were immensely successful
statements is correct? – through the 1970s and 1980s and even later – in promoting
(a) Introduction of exotic species into new geographical economic growth without pursuing democracy.
areas always leads to reduced biodiversity. To deal with these issues we have to pay particular attention
(b) Exotic species introduced by man into new areas have to both the content of what can be called development and to the
always greatly altered the native ecosystems. interpretation of democracy (in particular to the respective roles
(c) Man is the only reason to convert a hugely diverse of voting and of public reasoning). The assessment of
range of local community compositions into more development cannot be divorced from the lives that people can
homogeneous ones. lead and the real freedom that they enjoy. Development can scarcely
(d) None of the statements (a), (b) and (c) is correct in this be seen merely in terms of enhancement of inanimate objects of
context. convenience, such as a rise in the GNP (or in personal incomes), or
29. Why does man introduce exotic species into new industrialization – important as they may be as means to the real
geographical areas? ends. Their value must depend on what they do to the lives and
1. To breed exotic species with local varieties. freedom of the people involved, which must be central to the idea
2. To increase agricultural productivity. of development.
3. For beautification and landscaping. If development is understood in a broader way, with a focus
Which of the above statements is/are correct? on human lives, then it becomes immediately clear that the relation
(a) 1 only (b) 2 and 3 only between development and democracy has to be seen partly in
(c) 1 and 3 only (d) 1, 2 and 3 terms of their constitutive connection, rather than only through
30. How is homogenization prevented under natural conditions? their external links. Even though the question has often been asked
(a) Evolution of groups of species specific to local habitats. whether political freedom is ‘‘conducive to development’’, we must
(b) Presence of oceans and mountain ranges. not miss the crucial recognition that political liberties and
(c) Strong adaptation of groups of species to local physical democratic rights are among the “constituent components” of
and climatic conditions. development. Their relevance for development does not have to
(d) All the statements (a), (b) and (c) given above are be established indirectly through their contribution to be growth
correct in this context. of GNP.
31. How have the human beings influenced the biodiversity? 33. According to the passage, why is a serious tension
1. By smuggling live organisms. perceived between democracy and development by the
2. By building highways. detractors of democracy?
3. By making ecosystems sensitive so that new species (a) Democracy and development are distinct and separate
are not allowed. goals.
4. By ensuring that new species do not have major impact (b) Economic growth can be promoted successfully without
on local species. pursuing a democratic system of governance.
(c) Non-democratic regimes deliver economic growth faster 37. According to the passage, how does a foreign investor
and far more successfully than democratic ones. dominate the relevant domestic market?
(d) All the statements (a), (b) and (c) given above are 1. Multinational companies get accustomed to domestic
correct in this context. laws.
34. According to the passage, what should be the ultimate 2. Foreign companies establish joint ventures with
assessment/aim/view of development? domestic companies.
(a) Rise in the per capita income and industrial growth rates. 3. Affiliates in a particular market/sector lose their
(b) Improvement in the Human Development Index and independence as their parent companies overseas
GNP. merge.
(c) Rise in the savings and consumption trends. 4. Foreign companies lower the cost of their products as
(d) Extent of real freedom that citizens enjoy. compared to that of products of domestic companies.
35. What does a ‘‘constitutive’’ connection between democracy Which of the statements given above are correct?
and development imply? (a) 1 and 2 only (b) 2 and 3 only
(a) The relation between them has to be seen through (c) 1, 2 and 3 only (d) 1, 2, 3 and 4
external links. 38. What is the inference from this passage?
(b) Political and civil rights only can lead to economic (a) Foreign investors and multinational companies always
development. dominate the domestic market.
(c) Political liberties and democratic rights are essential (b) It is not in the best interests of the domestic economy
elements of development. to allow mergers of companies.
(d) None of the statements (a), (b) and (c) given above is (c) With competition law, it is easy to ensure a level playing
correct in this context. field between domestic and foreign firms.
PASSAGE-3 (d) For countries with open economy, Foreign Direct
The need for Competition Law becomes more evident when Investment is essential for growth.
foreign direct investment (FDI) is liberalized. The impact of FDI is 39. Examine the following statements:
not always pro-competitive. Very often FDI takes the form of a 1. I watch TV only if I am bored.
foreign corporation acquiring a domestic enterprise or establishing 2. I am never bored when I have my brother’s company.
a joint venture with one. By making such an acquisition the foreign 3. Whenever I go to the theatre I take my brother along.
investor may substantially lessen competition and gain a dominant Which one of the following conclusions is valid in the
position in the relevant market, thus charging higher prices. Another context of the above statements?
scenario is where the affiliates of two separate multinational (a) If I am bored, I watch TV.
companies (MNCs) have been established in competition with (b) If I am bored, I seek my brother’s company.
one another in a particular developing economy, following the
(c) If I am not with my brother, then I watch TV.
liberalization of FDI. Subsequently, the parent companies overseas
(d) If I am not bored, I do not watch TV.
merge. With the affiliates no longer remaining independent,
40. Only six roads A, B, C, P, Q and R connect a military camp to
competition in the host country may be virtually eliminated and
the rest of the country. Only one out of A, P and R is open at
the prices of the products may be artificially inflated. Most of
any one time. If B is closed, so is Q. Only one of A and B is
these adverse consequences of mergers and acquisitions by MNCs
open during storms. P is closed during floods. In this context,
can be avoided if an effective competition law is in place. Also, an which one of the following statements is correct?
economy that has implemented an effective competition law is in a
better position to attract FDI than one that has not. This is not just
(a) Under normal conditions only three roads are open.
because most MNCs are expected to be accustomed to the
(b) During storms at least one road is open.
operation of such a law in their home countries and know how to
(c) During floods only three roads are open.
deal with such concerns but also that MNCs expect competition
(d) During calamities all roads are closed.
authorities to ensure a level playing field between domestic and
41. Examine the following statements:
foreign firms.
1. None but students are members of the club.
36. With reference to the passage, consider the following
statements: 2. Some members of the club are married persons.
1. It is desirable that the impact of Foreign Direct 3. All married persons are invited for dance.
Investment should be pro-competitive. Which one of the following conclusions can be drawn from
2. The entry of foreign investors invariably leads to the the above statements?
inflated prices in domestic markets. (a) All students are invited for dance.
Which of the statements given above is/are correct? (b) All married students of the club are invited for dance.
(a) 1 only (b) 2 only (c) All members of the club are married persons.
(c) Both 1 and 2 (d) Neither 1 nor 2 (d) None of the above conclusions can be drawn.
42. Four political parties W, X, Y and Z decided to set up a joint 46. Consider the following information regarding the
candidate for the coming parliamentary elections. The performance of a class of 1000 students in four different
formula agreed by them was the acceptance of a candidate tests:
by most of the parties. Four aspiring candidates, A, B, C and
D approached the parties for their tickets. Tes ts I II III IV
A was acceptable to W but not to Z. A verage marks 60 60 70 80
B was acceptable to Y but not to X. Range of marks 30 45 20 0
C was acceptable to W and Y. to to to to
D was acceptable to W and X. 90 75 100 100
When candidate B was preferred by W and Z, candidate C
was preferred by X and Z, and candidate A was acceptable If a student scores 74 marks in each of the four tests, in
to X but not to Y; who got the ticket? which one of the following tests is her performance the best
(a) A (b) B comparatively?
(c) C (d) D (a) Test I (b) Test II
43. Consider the following statements:
(c) Test III (d) Test IV
1. All X-brand cars parked here are white.
2. Some of them have radial tyres. 47. Six squares are coloured, front and back, red (R), blue (B),
3. All X-brand cars manufactured after 1986 have radial yellow (Y), green (G), white (W) and orange (O) and are
tyres. hinged together as shown in the figure given below. If they
4. All cars are not X-brand. are folded to form a cube, what would be the face opposite
Which one of the following conclusions can be drawn from the white face?
the above statements?
R B
(a) Only white cars are parked here.
G Y O
(b) Some white X-brand cars with radial tyres are parked
W
here.
(c) Cars other than X-brand cannot have radial tyres. (a) R (b) G
(d) Most of the X-brand cars are manufactured before 1986. (c) B (d) O
44. Consider the following statement:
The Third World War, if it ever starts, will end very quickly 48.
with the possible end of civilization. It is only the misuse of P Q
nuclear power which will trigger it. 10 11 1
Based on the above statement, which one of the following
inferences is correct? 9 7 2
(a) Nuclear power will be used in the Third World War. 8 6
(b) There will be no civilization left after the Third World
R 4 5 3 S
War.
(c) The growth of nuclear power will destroy civilization in
the long run.
(d) The Third World War will not take place. In the above figure, circle P represents hardworking people,
45. The elements of the problem figures given below are circle Q represents intelligent people, circle R represents
changing with a certain rule as we observe them from left to truthful people, and circle S represents honest people. Which
right: region represents the people who are intelligent, honest and
truthful but not hardworking?
(a) 6 (b) 7
(c) 8 (d) 11
49. Three views of a cube following a particular motion are given
According to this rule, which of the following would be the below:
next figure if the changes were continued with the same
rule? K H B
B K H
A M P
(a) (b)

What is the letter opposite to A?


(a) H (b) P
(c) (d)
(c) B (d) M
52. What does the author imply by ‘‘gender impact’’?
50. (a) Women are doubtful participants in cooperatives.
(b) Family cooperatives may not include women.
(c) Women benefiting from group farming.
(d) Women’s role in transition economies is highly
? 53.
restrictive.
Consider the following assumptions:
1. It is imperative for transition economies to have
agricultural collectivities.
2. Agricultural productivity can be increased by group
approach to farming.
Which one of the figures shown below occupies the blank With reference to the above passage, which of these
space (?) in the matrix given above? assumptions is/are valid?
(a) 1 only (b) 2 only
(c) Both 1 and 2 (d) Neither 1 nor 2
(a) (b) PASSAGE-2
In a typical Western liberal context, deepening of democracy
invariably leads to consolidation of 'liberal values'. In the Indian
(c) (d) context, democratization is translated into greater involvement of
people not as 'individuals' which is a staple to liberal discourse,
but as communities or groups. Individuals are getting involved in
Directions for the following 6 (six) items: the public sphere not as ‘atomized’ individuals but as members of
Read the following two passages and answer the items that follow primordial communities drawn on religious or caste identity.
each passage. Your answers to these items should be based on Community-identity seems to be the governing force. It is not
the passages only. therefore surprising that the so-called peripheral groups continue
to maintain their identities with reference to the social groups
PASSAGE-1
(caste, religion or sect) to which they belong while getting involved
The poor especially in market economies, need the strength
in the political processes despite the fact that their political goals
that collectivities offer for creating more economic, social and
remain more or less identical. By helping to articulate the political
political space for themselves, for enhancing their socio-economic
voice of the marginalized, democracy in India has led to ‘a
well-being and voice, and as a protection against free market
loosening of social strictures’ and empowered the peripherals to
individualism. It has been argued that a group approach to farming,
be confident of their ability to improve the socio-economic
especially in the form of bottom up agricultural production
conditions in which they are placed. This is a significant political
collectivities, offers substantial scope for poverty alleviation and
process that had led to a silent revolution through a meaningful
empowering the poor as well as enhancing agricultural productivity.
transfer of power from the upper caste elites to various subaltern
To realize this potential, however, the groups would need to be
groups within the democratic framework of public governance.
voluntary in nature, small in size, participative in decision making
and equitable in work sharing and benefit distribution. There are
54. According to the passage, what does ‘‘deepening of
many notable examples of such collectivities to be found in varied
democracy’’ mean in the Western context?
contexts, such as in the transition economies. All of them bear
(a) Consolidation of group and class identities.
witness to the possibility of successful cooperation under given
(b) Democratization translated as greater involvement of
conditions. And although the gender impact of the family
people.
cooperatives in the transition economies are uncertain, the Indian
(c) Democratization as greater involvement of 'atomized'
examples of women-only groups farming offer considerable
individuals in the public sphere.
potential for benefiting women.
(d) None of the statements (a), (b) and (c) given above is
51. Agricultural collectivities such as group based farming can
correct in this context.
provide the rural poor
55. Greater democratization in India has not necessarily led to
1. empowerment.
(a) the dilution of caste and communal identities in the
2. increased agricultural productivity.
public sphere.
3. safeguard against exploitative markets.
(b) irrelevance of community identity as a governing force
4. surplus production of agricultural commodities.
in Indian politics.
Select the correct answer using the codes given below:
(c) marginalization of elite groups in society.
(a) 1, 2, 3 and 4 (b) 1, 2 and 3 only
(d) relative unimportance of hereditary identities over class
(c) 2 and 4 only (d) 1, 3 and 4 only
identities.
56. What is the ‘‘silent revolution’’ that has occurred in the 64. Examine the following statements:
Indian democratic process? 1. None but the rich can afford air-travel.
(a) Irrelevance of caste and class hierarchies in political 2. Some of those who travel by air become sick.
processes. 3. Some of those who become sick require treatment.
(b) Loosening of social strictures in voting behaviour and Which one of the following conclusions can be drawn from
patterns. the above statements?
(c) Social change through transfer of power from upper (a) All the rich persons travel by air.
caste elites to subaltern groups. (b) Those who travel by air become sick.
(d) All the statements (a), (b) and (c) given above are (c) All the rich persons become sick.
(d) All those who travel by air are rich.
correct in this context.
65. In five flats, one above the other, live five professionals.
Directions for the following 5 (five) items: The professor has to go up to meet his IAS officer friend.
Examine the information given in the following paragraph and The doctor is equally friendly to all, and has to go up as
answer the items there follow: frequently as go down. The engineer has to go up to meet
his MLA friend above whose flat lives the professor's friend.
Guest lectures on five subjects viz., Economics, History, From the ground floor to the top floor, in what order do the
Statistics, English and Mathematics have to be arranged in a week five professionals live?
from Monday to Friday. Only one lecture can be arranged on each (a) Engineer, Professor, Doctor, IAS officer, MLA
day. Economics cannot be scheduled on Tuesday. Guest faculty (b) Professor, Engineer, Doctor, IAS officer, MLA
for History is available only on Tuesday. Mathematics lecture has (c) IAS officer, Engineer, Doctor, Professor, MLA
to be scheduled immediately after the day of Economics lecture. (d) Professor, Engineer, Doctor, MLA, IAS officer
English lecture has to be scheduled immediately before the day of
Economics lecture. Directions for the following 8 (eight) items:
57. Which lecture is scheduled on Monday? The following eight items (questions 66 to 73) are based
(a) History (b) Economics on three passages in English to test the comprehension of English
(c) Mathematics (d) Statistics language and therefore these items do not have Hindi version.
58. Which lecture is scheduled between Statistics and English? Read each passage and answer the items that follow.
(a) Economics (b) History
PASSAGE-1
(c) Mathematics (d) No lecture
For fourteen and a half months I lived in my little cell or room
59. Which lecture is the last one in the week?
in the Dehradun jail, and I began to feel as if I was almost a part of
(a) History (b) English it. I was familiar with every bit of it, I knew every mark and dent on
(c) Mathematics (d) Economics the whitewashed walls and on the uneven floor and the ceiling
60. Which lecture is scheduled on Wednesday? with its moth-eaten rafters. In the little yard outside I greeted little
(a) Statistics (b) Economics tufts of grass and odd bits of stone as old friends. I was not alone
(c) English (d) History in my cell, for several colonies of wasp and hornets lived there,
61. Which lecture is scheduled before the Mathematics lecture? and many lizards found a home behind the ratters, emerging in the
(a) Economics (b) History evenings in search of prey.
(c) Statistics (d) English 66. Which of the following explains best the sentence in the
62. Two glasses of equal volume are respectively half and three- passage “I was almost a part of it”?
fourths filled with milk. They are then filled to the brim by (a) I was not alone in the cell.
adding water. Their contents are then poured into another (b) I was familiar with every bit of the cell.
vessel. What will be the ratio of milk to water in this vessel? (c) I greeted little tufts of grass like old friends.
(a) 1 : 3 (b) 2 : 3 (d) I felt quite at home in the cell.
(c) 3 : 2 (d) 5 : 3 67. The passage attempts to describe
63. Consider the following statements: (a) the general conditions of the country's jails.
1. All machines consume energy. (b) the prisoner's capacity to notice the minute details of
2. Electricity provides energy. his surroundings.
(c) the prisoner's conscious efforts to overcome the
3. Electrically operated machines are cheap to maintain.
loneliness.
4. Electrically operated machines do not cause pollution.
(d) the prisoner's ability to live happily with other creatures.
Which one of the following inferences can be drawn from
68. The author of the passage seems to suggest that
the above statements? (a) it is possible to adjust oneself to uncongenial
(a) All machines are run by electric energy. surroundings.
(b) There is no form of energy other than electricity. (b) the conditions in Indian prisons are not bad.
(c) Most machines are operated on electric energy. (c) it is not difficult to spend one's time in a prison.
(d) Electrically operated machines are preferable to use. (d) there is a need to improve the conditions in our jails.
PASSAGE-2 Directions for the following 7 (seven) items :
We started pitching the highest camp that has ever been Given below are seven items. Each item describes a situation
made. Everything took five times as long as it would have taken in and is followed by four possible responses. Indicate the response
a place where there was enough air to breathe; but at last we got you find most appropriate. Choose only one response for each
the tent up, and when we crawled in, it was not too bad. There was item. The responses will be evaluated based on the level of
only a light wind, and inside it was not too cold for us to take off appropriateness for the given situation.
our gloves. At night most climbers take off their boots; but I prefer Please attempt all the items. There is no penalty for wrong
to keep them on. Hillary, on the other hand, took his off and laid answers for these seven items.
them next to his sleeping bag.
74. You have differences of opinion regarding the final report
69. What does the expression “pitching the highest camp”
prepared by your subordinate that is to be submitted,
imply?
urgently. The subordinate is justifying the information given
(a) They reached the summit of the highest mountain in
in the report. You would…
the world.
(a) Convince the subordinate that he is wrong.
(b) Those who climbed that far earlier did not pitch any
(b) Tell him to reconsider the results.
camp.
(c) Revise the report on your own.
(c) So far nobody has ever climbed that high.
(d) Tell him not to justify the mistake.
(d) They were too many climbers and needed to pitch a big
75. You are competing with your batch-mate for a prestigious
camp.
award to be decided based on an oral presentation. Ten
70. They took a long time to finish the work because
minutes are allowed for each presentation. You have been
(a) they were very tired. asked by the committee to finish on time. Your friend,
(b) there was not enough air to breathe. however, is allowed more than the stipulated time period.
(c) it was very cold. You would.
(d) it was very dark. (a) Lodge a complaint to the chairperson against the
71. When they crawled into the tent discrimination.
(a) they took off their gloves because it was not very cold. (b) Not listen to any justification from the committee.
(b) they could not take off their gloves because it was very (c) Ask for withdrawal of your name.
cold. (d) Protest and leave the place.
(c) they took off their gloves though it was very cold. 76. You are handling a time-bound project. During the project
(d) they did not take off their gloves though it was not review meeting, you find that the project is likely to get
cold. delayed due to lack of cooperation of the team members.
Passage-3 You would.
A local man, staying on the top floor of an old wooden (a) Warn the team members for their non-cooperation.
house, was awakened at midnight by a fire. Losing his way in a (b) Look into reasons for non-cooperation.
smoke-filled passage, he missed the stairway and went into another (c) Ask for the replacement of team members.
room. He picked up a bundle to protect his face from the fire and (d) Ask for extension of time citing reasons.
immediately fell through the floor below where he managed to 77. You are the chairperson of a state sports committee. You have
escape through a clear doorway. The ‘‘bundle’’ proved to be the received a complaint and later it was found that an athlete in
baby of the Mayor's wife. The ‘‘hero’’ was congratulated by all. the junior age category who has won a medal has crossed the
age criteria by 5 days. You would...
72. The man went into another room because (a) Ask the screening committee for a clarification.
(a) he did not know where exactly the stairway was. (b) Ask the athlete to return the medal.
(b) the passage was full of smoke. (c) Ask the athlete to get an affidavit from the court
(c) he was extremely nervous. declaring his/her age.
(d) he stumbled on a bundle. (d) Ask the members of the sports committee for their
73. The man was called a hero because he views.
(a) expressed his willingness to risk his life to save others. 78. You are handling a priority project and have been meeting all
(b) managed to escape from the fire. the deadlines and are therefore planning your leave during
(c) showed great courage in fighting the fire. the project. Your immediate boss does not grant your leave
(d) saved a life. citing the urgency of the project. You would...
(a) Proceed on leave without waiting for the sanction. (c) Recommend that a fixed monthly charge be levied only
(b) Pretend to be sick and take leave. on the non-BPL families and for BPL families water
(c) Approach higher authority to reconsider the leave should be free.
application. (d) Recommend that the users pay a charge based on the
(d) Tell the boss that it is not justified. consumption of water with differentiated charges for
79. You are involved in setting up a water supply project in a non-BPL and BPL families.
remote area. Full recovery of cost is impossible in any case. 80. As a citizen you have some work with a government
The income levels in the area are low and 25% of the department. The official calls you again and again, and without
population is below poverty line (BPL). When a decision directly asking you, sends out feelers for a bribe. You want to
has to be taken on pricing you would... get your work done. You would...
(a) Recommend that the supply of water be free of charge (a) Give a bribe.
in all respects. (b) Behave as if you have not understood the feelers and
(b) Recommend that the users pay a one time fixed sum for persist with your application.
the installation of taps and the usage of water be free. (c) Go to the higher officer for help verbally complaining
about feelers.
(d) Send in a formal complaint.
ANSWERS AND EXPLANATIONS
1. (a) The passage clearly suggests that education is not 2 is correct. Because of this problem it becomes
instrumentalist in its deepest sense. But the opening imperative to use pesticides.
sentence calls it to be functional, instrumental and 3 is wrong as the social and health costs have to be
utilitarian. Thus the instrumentalist view of education ignored because of the frightening prospects of the
is the functional and utilitarian dimension in its epidemic diseases.
purposes. 9. (b) The passage does not talk about any alternative option
2. (c) The second part of the passage clearly states education to chemical pesticides. It talks about a balance being
is not a commodity but a process of expansion and drawn between sustainability and use of pesticides.
conversion of the mind – the moral-intellectual 10. (a) Only 1 makes sense. According to the 3rd paragraph
development. Acquiring qualifications, upward mobility second line, ‘And some growth ........................... natural
and social status are the basic utility values of resources.’ 2, 3 & 4 are irrelevant statements.
education. 11. (d) The options provided in the question does not imply
3. (c) Again the second part clearly states the answer. a, b & low-carbon growth.
d are the utilitarian dimensions of education. But A number of low-carbon growth options exist for
ultimately education leads to self-critical awareness and reducing our net greenhouse gas emissions, particularly
independence of thought. carbon dioxide, which could be implemented over
4. (d) Natural selection is a key mechanism of evolution. It is different time horizons. These include: (1) improved
the gradual, non-random, process by which biological efficiency in energy use, especially over the short to
traits become either more or less common in a medium term, through technological and behavioral
population as a function of differential reproduction of changes; (2) producing energy which minimizes carbon
their bearers. Variation exists within all populations of dioxide emitted, especially for new power plants, and
organisms. This occurs partly because random realistically over the medium to long term; and (3)
mutations cause changes in the genome of an individual reducing carbon dioxide produced in non-energy
organism, and these mutations can be passed to sectors, such as agriculture and forestry, and industries,
offspring. Throughout the individuals’ lives, their such as cement production. In addition to these,
genomes interact with their environments to cause technologies are being developed to capture and
variations in traits. permanently store greenhouse gases, especially carbon
5. (b) 1 is not correct because the passage does not talk about dioxide.
all the poor countries. 12. (b) 1 is not correct as economic prosperity can raise
2 is not correct because the passage talks about the incomes and foster better institutions but it cannot
role of pesticides in sustainable agriculture especially forster sustainable economic growth. 2 is correct.
in poor countries. Generating adaptive technologies can lead to a
3 is correct as the 2nd para clearly illustrates Alabama sustainable growth as discussed in para 2. 3 is correct.
leaf worm developing resistance to aldrin, DDT, dieldrin, As investing on research in adaptation will help us in
endrin,lindane and toxaphene. better handling of the changing climate.
6. (d) The widespread use of pesticides has not waned 13. (a) 1 is correct because if rainfed crops are grown in irrigated
because the ratio of cost to benefit for the individual areas it would lead to overexploitation of natural
agricultural producer has remained in favour of pesticide resources. 2 is wrong as farming provides employability
use. as well as food resources. The strategy must be not to
7. (a) Natural populations of pests contain, among their vast overexploit and do limited farming.
numbers of individual members, considerable variation 14. (a) 1 is correct as the creative potential of man will provide
in their genetic material, primarily as the result of better adaptability to the changing climate. 2 is wrong
mutations. When exposed to pesticides, most pests as the passage does not talk about intensive agriculture
die quickly, but some may have mutations that make leading to an economic back lash. It is our growth –
them slightly less susceptible. If the exposure to pests industrial and overexploitation of resources – which
is short, these individuals will survive the treatment. has lead to such changes.
This selective elimination of maladapted individuals 3 is wrong as economic prosperity will enable us in
from a population is natural selection. better handling of the environmental changes.
8. (c) 1 is not correct as organic farming is not mentioned in 15. (b) The central theme of the passage is clear that adaptation
the passage. Further the passage justifies the cost and mitigation should be integrated with development
benefit to developed countries like USA. strategies. The author is not against development but
a sustainable development is what he is talking about.
Thus the uncle of D is A
16. (b) Whimsical 27. (d) Ratna > Padma > Rama > Rani
Thus Ratna scored the highest.
28. (c)
Drug (a) There is a single example given of such a case but it is
Artists
addicts
not always true.
(b) This statement is true in some cases but not always so.
(c) It is clearly stated that man is the only reason to convert
local community compositions into more homogeneous
Thus some drug addicts are whimsical. ones.
17. (d) (1) A ³ B (d) The option (c) is correct in this context.
29. (b)
(2) D ³ C
(a) To breed exotic species with local varieties is obviously
(3) B > C
not the purpose.
Then either B & D are of same age or B is older than D
(b) Man intentionally introduces exotic species into new
and either A & D is older than D. Thus A will older than geographical areas for agricultural or recreational
C. purposes. This is clearly stated in the passage.
18. (b) Having a pair of binoculars and be a member of the (c) (1) is ruled out. So this answer is not correct.
birdwatcher's club (d) Again (1) is ruled out making the answer incorrect.
30. (b)
m. of B.C. (a) This is not the correct answer.
Cameras (b) This is the dominant idea in the whole passage. The
P.B.
presence of “physical barriers” have prevented
homogenization.
Can take part in (c) This is not clearly stated in the passage.
photo-contests (d) This idea of “physical barriers” is the most important
19. (d) Ankit's last summer camp’s events in which is took reasons for the others.
part-hiking, swimming boating. 31. (a)
Ankit's this summer camp’s events sing, dance and learn (a) This obviously the correct answer, because it is the
to play the guitar. most practical one. It is humanly possible to smuggle
20. (b) live organisms and to build highways.
(b) It is not easy nor practical to make ecosystems
Pers ons Shirt Pant sensitive. So this option is ruled out.
Black Blue Orange Green Yellow Orange (c) This option is also ruled out for the above said reason.
(d) It is very difficult and quite unnecessary to ensure new
A ü ü
species do not have an impact on local species. So this
B ü ü option is also ruled out.
32. (c) Both the options (1) and (2) are correct.
C ü ü
33. (b)
Colour of C’s Pant-Green (b) The first paragraph in the passage conveys the
and Colour of C’s Shirt-Blue message that the detractors of democracy are quite vocal
about that fact that a number of non-democratic
21. (c)
governments particularly in East Asia have successfully
22. (c)
achieved economic development.
Options (a), (c) and (d) are automatically ruled out.
0.6d 0.4 d 10 34. (d) The second and last paragraphs the fact that freedom
23. (d) – =
48 48 60 and liberty are essential components of development.
35. (c) The “constitutive” connection between democracy and
10× 48 development is political freedom and democrative
0.2 d =
60 rights.
\ d = 40 km 36. (a) The effect of FDI or Foreign Direct Investment should
25. (a) Rita > Gita > Sita be to induce competition because this is ensured in
Father of Brother of most countries worldwide.
26. (a) C A 37. (b)
Father/mother of Sister/Brother of (a) The first option of multinational companies getting
B E D accustomed to domestic laws is not mentioned at all.
So, this is not the correct answer.
(b) Foreign companies may establish joint ventures with
domestic or companies may get stronger as the parent 48. (a) Hardworking Intelligent
companies merge overseas. Both options are mentioned
in the paragraph. So, this is the correct answer.
10 11 1
(c) Since option (1) is included this is not the right answer.
(d) Same as in (c). Moreover option (4) where foreign
companies lower costs finds no mention in the 9 7 2
paragraph. 8 6
38. (c) The message conveyed in the passage is that it is 5
important to have a competition law in the country to Truthful 4 3 Honest
ensure that both domestic and foreign firms have a
level playing flied. The region (6) represents, the people who are intelligent,
39. (d) 40. (b) honest, truthful but not hardworking.
41. (b) 49. (a) B & K can't opposite to A
42. (c) M & K '' '' to H
Political parties B & P '' '' to H
Candidates
W X Y Z From above statements
A ü × H can’t be opposite to B, K, M, P
Thus H will in opposite of A.
B × ü
50. (d) In the first and 2nd row, the dark part of the circle rotates
C ü ü 180° clockwise at every new step.
D ü ü 51. (b) This is the first of the passage. Group farming helps
remove poverty, increases agricultural productivity and
Now B was preferred by W & Z secures the individual from exploitation.
C was preferred by X & Z There is no mention of surplus production. So the other
A was acceptable to X but not to Y. options are ruled out.
Since C was acceptable Y & W and preferred by X & Z. 52. (d) By “gender impact” the author implies that women do
Thus C would got the ticket. not enjoy much power in the transition economies.
43. (b) White This is the most appropriate meaning the others come
close but are not as accurate.
Radial 53. (b)
X-brand tyres (a) There is compulsion on the transition economies to go
in for group farming. Therefore, this is not the correct
answer.
(b) The paragraph is all about the benefits of group
Manufactured approach to farming. This is the right answer.
after 1986 (c) Both options correct included.
(d) Both cannot be negated.
44. (a) 45. (d)
54. (c) In the Western context “deepening of democracy”
46. (b) means the increased participation of the individual in
Tests I II III IV
the democratic process.
Average marks 60 60 70 80
So, the other option (a), (b) and (d) are automatically
Range of marks 30 45 20 0 negated.
90 75 100 100 55. (a) According to the passage Democracy in the Indian
context means the increased participation of
If a student scores 74 marks in each of the four tests, communities based on religious, caste or sectarian
his performance would be the best comparatively in identities. That is why greater democratization in India
test II because there would be less numbers of the does not lead to the dilution of caste and communal
students in this range in test II only. identities of the individual.
47. (a) In the context of this explanation options (b), (c) and
(d) are automatically negated.
1 R
56. (c) The involvement of communities in the democratic
G B
process in India has led to a silent revolution. The upper
W Y 2 castes held power in earlier days. This power is getting
O 3 slowly, silently and surely transferred to the subaltern
G– O groups
R–W In the context of this argument option (a), (b) and (d)
B– Y are ruled out.
For Q. 57 to 61 68. (a) (a) Since the central idea of the passage is loneliness
and the author’s struggle to adjust himself to rather
Subjects Monday Tuesday Wednesday Thursday Friday difficult circumstances option (a) is the right answer.
Economics × ü (b) This obviously not correct.
History ü (c) This is true but it is not the central idea.
Statistics ü (d) This is also true but that is not just what the author
English ü wants to suggest.
Mathematics ü 69. (b) The implication here is that even if anybody else had
57. (d) 58. (b) climbed this far earlier. They had not pitched any camp.
59. (c) 60. (c) (a) There is no reference to the building of any camp in
this option.
61. (a)
(c) It is implied that no camp had been built so high.
1 (d) There is no mention of the manpower required to
62. (d) Milk in 1st Glass = V
2 pitch the camp in the paragraph.
70. (b) They to a long time to finish the work because there
3 was not enough air to breathe at such a high altitude.
Milk in 2nd Glass = V
4 All the other three options (a), (c) and (d) are true, but
they are only implied not clearly stated in the passage
1 as in the case of option (b).
Now the 1st glass = V 71. (a) (a) When they crawled in the tent they took off their
2
gloves since it was not very cold.
1 (b) The narrator says that is was not very cold at such
Now the 2nd glass = V a high altitude. So this option is ruled out.
4
(c) The same argument as in (b) holds true.
When both glasses are mixed then the ratio of milk to (d) The narrator says they took of their gloves. So this
water option is not correct.
72. (b) The man went into another room because the passage
1 3 1 1 2 + 3 2 +1
+ : + = : =5 : 3 was full of smoke. It was an old wooden house. There
2 4 2 4 4 4 was a fire at midnight. The man who was staying on the
63. (d) 64. (d) top floor of the house was caught unawares. He
stumbled out into the smoke filled passage and lost his
65. (d) Professionals Flats way. So the root cause of his going into another room
IAS 5th was the smoke filled passage.
MLA 4th All the other three options (a), (c) and (d) are
Doctor 3rd automatically cancelled in the height of the above
Engineer 2nd context.
Professor 1st 73. (d) (a) In the context of the passage he saved the life of a
baby accidentally. So there is no question of his
66. (b) When the narrator says that he was “almost a part of expressing his willingness to risk his life for others.
it” he means that he was familiar with every corner of (b) He did escape from the fire but that is no reason
why he should be called a hero.
the cell.
(c) He just kept stumbling and falling from one spot to
It is true that he was not alone in the cell; there were another; this does not call for courage.
lots of insects to keep him company. But this is not the (d) The man was called a hero because he saved a life.
meaning conveyed by the quoted text. 74. (c) (a) It is only possible to change oneself not others.
The same goes for options (c) and (d). Convincing the subordinate will not be easy because
67. (c) The impression that comes across through the his ego will not accept that he is wrong.
(b) Here again the same argument will hold true. When
narrator’s description is his untiring efforts to adjust to
he is not sure that he is wrong he may not reconsider
extreme loneliness. It is quite obvious that he was very the results so faithfully.
lonely and that he was making friends with various (c) It would be most advisable to revise the report on
types of insects, flowers, etc. to retain his sanity. your own. It would involve effort but it would be the
(a) The general condition of the country jails is revealed most peaceful and positive means.
in the passage but this is not the central idea of this (d) This would be the most negative way of approaching
the matter. Dealing with the subordinated aggressively
passage.
will only make him rebellions and quarrelsome. It would
The same is true of options (b) and (d). be better to avoid such a situation.
75. (a) (a) This would be the most positive approach to the 78. (d) (a) A very negative approach. It could cost you your
whole situation. Moreover, one has to follow the job.
protocol. So just lodge a complaint since the results are (b) This is also a negative approach. Next time you are
yet awaited. really sick your leave may not be granted for your boss
(b) Taking an aggressive stand against the committee will definitely find it fishy.
who are the deciding authority will only worsen matters. (c) If you approach the higher authorities you will be
(c) By withdrawing your name you are harming yourself. spoiling relations with your immediate boss. Only a
It is a negative approach. good understanding between your boss and you can
(d) This is the most negative approach of all. ensure peace at work.
76. (b) (a) Warning the team - members for their non- (d) This would be the best option. By discussing your
cooperation would not yield much results. It is not so problem directly with your boss you will both be able
easy to change other. to resolve the issue and reach a compromise. This will
(b) If we can look into reasons for their non- ensure a better working relationship. The ice will be
cooperation; we have a chance to rectify the situation. broken.
(c) A negative approach. There is no guarantee that the 79. (d) (a) It would be practically feasable to make the supply
next set of team members will be cooperative. of water free of cost.
(d) This is slightly negative. Extending the time will not (b) A one time fee for taps can be easily afforded by the
change the attitude of the team members. non – BPL families but by not by the BPL families.
77. (b) (a) Asking the screening committee for a clarification (c) This is again not a very feasable solution for it may
will make it a long drawn process. lead to a lopsided usage of water.
(b) Strict action has to be taken so that such incidents (d) An extremely feasable option. This will atleast
do not take place in the future. ensure that the consumption of water is well regulated
and the non – BPL families will not feel exploited.
(c) This decision is based on the assumption that the
athlete is telling the truth. It is not fine to fair. 80. (d) (a) By giving a bribe you are accepting the situation for
what it is. You are also inviting similar trouble in the
(d) Unless quick action is taken your authority as the
future.
chairman of the sports committee can also be
questioned. (b) If you behave as if you are ignorant of the feelers
they will not stop. On the contrary the work will get
postponed indefinitely.
(c) Going to the higher officer will not ensure that the
matter will be looked into the higher officer may himself
be corrupt and may also be involved in the matter.
(d) By sending in a formal complaint you can ensure
that you will be given a hearing. The letter will reach
the concerned authorities and can even be sent to the
press.
UPSC CIVIL SERVICES (PRE) EXAM

General Studies
Paper 1
Practice Set 1
Time : 2 hrs MM : 200

Instructions
1. This set contains 100 questions. Each question comprises four responses (answers). You will select the response which
you want to mark on the Answer Sheet. In case, if you feel that the correct response is more than one, then mark the
response, which you consider the best. In any case, choose only one response for each questions.
2. All questions carry equal marks.
3. Penalty for Wrong Answer
(i) There are four alternatives to answer every question. If a question is marked wrong, one-third of the marks assigned
to that question will be deducted as penalty.
(ii) If a candidate gives more than one answer, it will be treated as wrong answer even, if one of the given answers
happens to be correct and there will be same penalty as to that question, if it has a penalty.
(iii) If a question is left blank i.e. no answer is given by the cnadidate, there wil be no penalty for that question.

1. Which of the following Biosphere reserves in India are part of the World Network of Biosphere
Reserves, based on the UNESCO Man and the Biosphere (MAB) Programme list?
1. Sundarbans Biosphere Reserve 2. Nanda Devi Biosphere Reserve
KNOW the TREND
3. Nokrek Biosphere Reserve 4. Pachmarhi Biosphere Reserve
5. Simlipal Biosphere Reserve 6. Achanakmar-Amarkantak Biosphere Reserve
7. Nilgiri Biosphere Reserve
Select the correct answer using the codes given below.
(a) 2, 3, 4 and 5 (b) 1, 4, 6 and 7
(c) 1, 2, 3, 5 and 7 (d) All of these

2. Which of the following statements regarding CITES are correct?


1. It lists several species of animals and birds in which trade is prohibited.
2. USA is not a party to the convention.
3. It is also known as Washington Convention.
Stage 1

Select the correct answer using the codes given below.


(a) 1 and 2 (b) 2 and 3
(c) 1 and 3 (d) All of these
2 Practice Set 1

3. Consider the following statements regarding 2011 census.


1. Kerala has registered the lowest growth rate of population in 2011 census.
2. The Northern part of the India has recorded high growth rate of population than Southern India.
3. The decadal growth rate of India’s population is recorded higher than previous census.
Which of the statements given above are correct?
(a) 1 and 2 (b) 1 and 3 (c) 2 and 3 (d) All of these
4. Which of the following pairs are incorrectly matched?
1. Hadis : Sayings and doings of the prophet
2. Zawabit : Islamic law
3. Fatwa : Islamic legal decision
4. Farman : Rules and regulations framed by the sultans
5. Shariat : Royal order
Codes
(a) 1, 2 and 3 (b) 2, 3 and 4 (c) 2, 4 and 5 (d) 1, 3 and 5
5. Consider the following statements regarding the Fundamental Rights described in the
Constitution.
1. Laws seeking to give effect to Fundamental Rights can be made by both Parliament and State
Legislatures.
2. Any of the Fundamental Rights given in Part III of the Constitution can be amended by Parliament.
3. All the Fundamental Rights are applicable only against the actions of the state and not those of private
individuals.
4. All Fundamental Rights are negative in nature i.e. they impose restrictions upon the actions of others.
Which of the statement(s) given above is/are correct?
(a) 1, 2 and 4 (b) Only 2 (c) 2 and 3 (d) 3 and 4
6. Which of the following factors have led to the decline in the working of Indian Parliament?
1. Growth of delegated legislation.
2. Low level of attendance in Parliament.
3. Frequent promulgation of ordinances.
4. Frequent amendment of the Constitution.
5. Set block in Parliamentary behaviour and ethics.
Select the correct answer using the codes given below.
(a) 1, 2, 3 and 5 (b) 1, 3, 4 and 5 (c) 2, 3 and 4 (d) 1, 3 and 5
7. Consider the following about Mudiyettu form of traditional theatre.
1. It is the traditional theatre of Tamil Nadu.
2. It enacts the mythological battle between ‘Kali’ and demon Darika.
3. In 2010, Mudiyettu was recognised by UNESCO in the list of intangible cultural heritage.
Which of the statements given above are correct?
(a) 1 and 2 (b) 2 and 3 (c) 1 and 3 (d) All of these
8. Consider the following statements, with reference to Bharat Stage Emission Standards.
1. These are emission standards instituted by the Government of India to regulate the output of air
pollutants from internal combustion engine equipment, including motor vehicles.
2. The stoppage of production of Maruti-800 is because of regulation of emission.
3. At present India has adopted Bharat Stage -V emission norms.
4. Under Bharat Stage - III, two wheelers were allowed to emit upto 1g/km of carbon monoxide.
Which of the statements given above are correct?
(a) 1, 2 and 3 (b) 1, 3 and 4 (c) 1, 2 and 4 (d) All of these
Stage 1 Know the Trend 3

9. Arrange the following mountain ranges of Peninsular India from North to South.
1. Maikala range 2. Balaghat range 3. Kaimur range 4. Ajanta range
Codes
(a) 3, 1, 4, 2 (b) 1, 2, 3, 4 (c) 3, 1, 2, 4 (d) 1, 2, 4, 3
10. Consider the following statements.
1. These are the most widespread forests of India. They are also called the monsoon forests and spread
over the region receiving rainfall between 70 cm and 200 cm.
2. Teak is the most dominant species of this forest, Bamboos, Sal, Shisham, Sandalwood, Khair, Kusum
Arjun, Malberry are other commercially important species.
To which type of forests in India does above description refer to?
(a) Tropical deciduous forests (b) Tropical rain forests
(c) Montane forests (d) Montane forests
11. The ultrasound technique is used frequently in obstetrics instead of X-rays based techniques.
What are the principles behind this?
1. Ultrasound is a non-ionising imaging technique.
2. Ultrasound gives a clear picture of soft tissues, which do not show up in X-rays.
3. Ultrasound causes almost no health problems (for a mother or unborn foetus).
Select the correct answer using the codes given below.
(a) 2 and 3 (b) 1 and 2 (c) 1 and 3 (d) All of these
12. Consider the following statements regarding 21st Law Commission of India.
1. It constituted for a period of 3 years with effect from 1st September, 2015 to 31st August, 2018.
2. It undertake research in law and review of existing laws in India for making reforms therein and
enacting new legislations.
3. The Secretary Department of Legal Affairs is the only ex-officio member of the commission.
4. Its function is to suggest enactment of new legislations to implement the Directive Principles and to
attain the objectives set out in the Preamble of the Constitution as well.
Which of the statements given above are correct?
(a) 1, 2 and 3 (b) 1, 3 and 4 (c) 2, 3 and 4 (d) All of these
13. Consider the following statements, with reference to Liquid Apogee Motor (LAM).
1. It refers to a rocket motor that is regularly employed on artificial satellites destined for a geostationary
orbit.
2. They often employ bipropellant engine.
3. India’s GSAT-SP will use LAM in its mission.
Which of the statement(s) given above is/are correct?
(a) 1 and 3 (b) 1 and 2 (c) Only 1 (d) All of these
14. Which of the following is the reason for considering insider trading as harmful to the domestic
economy and illegal?
(a) It hurts the economic growth of a nation, increases inflation and Current Account Deficit (CAD)
(b) It results in crash of the security markets which are so vital for channeling savings in the domestic
economy
(c) It distorts the level-playing field between investors in the stock market and makes the markets
susceptible to heavy speculation
(d) Speculation by individual investors is illegal and hurts the retail investors
4 Practice Set 1

15. Consider the following statements related to NITI Aayog.


1. NITI Aayog is non-constitutional non-statutory body as Planning Commission was.
2. NITI Aayog will enjoy more financial powers than Planning Commission.
3. All members of the NITI Aayog will be full time members.
Which of the statement(s) given above is/are incorrect?
(a) Only 3 (b) 1 and 2 (c) 2 and 3 (d) 1 and 3
16. Which of the following did not follow in the wake of the extensive tour undertaken by Mahatma
Gandhi during 1933-34 in the cause of the upliftment of the Harijans?
(a) Admission of the untouchables to the Guruvayur, temple in Kerala
(b) Admission of the untouchables to the Golden Temple at Amritsar
(c) Introduction of legislation in the Central Legislative Assembly for the abolition of untouchability
(d) Introduction of legislation in the Madras legislature for the abolition of untouchability
17. Consider the following statements about the gradual increase in rural indebtedness in India under
the British rule was due to
1. fragmentation of landholdings. 2. decline of cottage industries.
3. lack of development of irrigational facilities. 4. introduction of cash crops.
Which of the statements given above are correct?
(a) 1, 2 and 3 (b) 2 and 4 (c) 1, 3 and 4 (d) All of these
18. Which statements regarding Savanna region are correct?
1. Savanna is a tropical grassland.
2. Savanna eco-region is also called world’s paradise of hunters.
3. In Savanna region trees are sparse and chemo-resistance.
4. Savanna eco-regions has maximum biodiversity.
Select the correct answer using the codes given below.
(a) 1 and 2 (b) 2 and 3 (c) 1, 2 and 3 (d) All of these
19. What are the ways, in which the union exercises control over State Legislation?
1. Certain bills passed by the State Legislature have to be compulsorily reserved by the Governor for the
consideration of the President.
2. Governor can also reserve State Bills for the consideration of the President, if he feels that the bills are
against the constitutional provisions, if it is opposed to the Directive Principles of State Policy.
3. Certain category of Money Bills passed by the State Legislature have to be compulsorily reserved for the
consideration of the President.
4. During financial emergency, Money Bills of State Legislatures need prior recommendation of the
President before being introduced.
Select the correct answer using the codes given below.
(a) 1 and 2 (b) 1, 2 and 3 (c) 2 and 4 (d) 1, 3 and 4
20. Which of the following statements is not one of the effects of proclamation of National Emergency
on Fundamental Rights of citizens?
1. Fundamental Rights under Article 19 get automatically suspended under any proclamation of National
Emergency.
2. Fundamental Rights except those under Article 20 and 21 can be suspended by the President under
emergency through his order.
3. Any law made by Parliament under an emergency cannot be declared to be void on the grounds that, it
violates a Fundamental Rights under Article 19.
Select the correct answer using the codes given below.
(a) 1 and 2 (b) 1 and 3 (c) Only 3 (d) All of these
Stage 1 Know the Trend 5

21. Consider the following statements regarding black soils.


1. These are poor in nitrogen, phosphorus, potassium and organic matter and rich in potash and lime.
2. These are porous, friable in nature.
3. These suited for dry farming as it does not require much moisture.
4. These soils expand when wet and become difficult to plough.
Which of the statements given above are correct?
(a) 1, 2 and 3 (b) 2, 3 and 4 (c) 1, 3 and 4 (d) All of these

22. Which one of the following is the correct sequence of dynasties of Delhi Sultanate in the order of
early medieval history to middle medieval history?
(a) Slave dynasty, Khiljis, Tughlaqs (b) Tughlaqs, Mameluks, Khiljis
(c) Khiljis, Slave dynasty, Tughlaqs (d) Tughlaqs, Slave dynasty, Khiljis

23. Which of the following statements is correct regarding the Himalayan Yew?
(a) Bird species known for its long beautiful white feathers
(b) A sub-species of owl which is bigger in size than any other owl species in the world
(c) A medicinal plant known from which world famous anti-cancer drug is produced
(d) None of the above

24. Which of the following statements about Agenda 21 is/are correct?


1. Agenda 21 is a binding action plan of the United Nations with regard to sustainable development.
2. The ‘21’ in Agenda 21 refers to the 21 important points that are need to be implemented by all the
countries.
3. In 2012, at the United Nations Conference on Sustainable Development the attending members
reaffirmed their commitment to Agenda 21 in their outcome document called ‘The Future We Want’.
Select the correct answer using the codes given below.
(a) 1 and 2 (b) 2 and 3
(c) Only 3 (d) All of these

25. One of the key features of many flagship schemes such as MGNREGA and NRLM is that they are
‘Demand Driven’. Which of the following is/are the characteristic(s) of a demand driven
programme?
1. The process is driven from the grassroots of the society.
2. Highly centralised planning.
3. Implementation is largely done by the Central Government.
Select the correct answer using the codes given below.
(a) Only 1 (b) 1 and 2
(c) 2 and 3 (d) All of these

26. Consider the following statements about India Water Week (IWW)-2016.
1. It is based on the theme ‘Water for All : Striving Together’.
2. International Commission on Irrigation and Drainage (ICID) plans to organise first ‘India Irrigation
Forum 2016.’
3. Conceptualised and organised for the first time in 2012, the India Water Week is an annual forum where
the Ministry of Water Resources strategies for conservation, preservation and optimum use of available
water.
Which of the statement(s) given above is/are correct?
(a) 2 and 3 (b) 1 and 3 (c) 1 and 2 (d) All of these
6 Practice Set 1

27. Consider the following statements about honey.


1. It is a food stored by bees in their hives.
2. Honeybees transform nectar into honey by a process of regurgitation and evaporation.
3. Most microorganisms do not grow in honey because of its high water activity.
Which of the statements given above are correct?
(a) 1 and 3 (b) 2 and 3 (c) 1 and 2 (d) All of these
28. Which of the following pair(s) is/are correctly matched?
1. Revenue assignments : Bhoga or Fief
2. Twice born or privileged : Dvija
3. Untouchables : Antyaja
4. Marriage according to norm : Pratilom
Codes
(a) 1 and 2 (b) 1, 2 and 3 (c) Only 3 (d) All of these
29. Consider the following statements regarding Madurai - Coimbatore - Bengaluru region.
1. It is a predominantly cotton and sugarcane growing region.
2. Mettur hydel power project provide energy to this region.
3. The region also support heavy industries such as heavy engineering, locomotive and heavy electricals.
Which of the statements given above are correct?
(a) 1 and 3 (b) 1 and 2 (c) 2 and 3 (d) All of these
30. Which of the following statement(s) is/are correct?
1. As the distance from the sea increases, people experience extreme weather conditions. This condition
is known as Continentality.
2. Ocean currents alongwith onshore winds affect the climate of the coastal areas.
3. The pressure and wind system of any area depend on the latitude and altitude of the place. Thus, it
influences the temperature and rainfall pattern.
Select the correct answer using the codes given below.
(a) Only 1 (b) 1 and 2 (c) 2 and 3 (d) All of these
31. Which of the following pairs is incorrectly matched?
Traditional Puppets Region
(a) Tholu Bommalata : Andhra Pradesh
(b) Putul Nach : Bengal
(c) Togalu Gombeyatta : Kerala
(d) Yampuri : Bihar
32. Consider the following statements about the Government of India Act, 1858.
1. The East India Company’s control came to an end.
2. The Board of Directors was abolished.
3. The basic court of proprietors came to an end.
4. Secretary of state became a member of British Cabinet.
Which of the statements given above are correct?
(a) 2 and 3 (b) 1 and 2 (c) None of these (d) All of these
33. Consider the following statements regarding Biomes.
1. Biomes are classified on the basis of climatic parameters only.
2. Biomes are unaffected by human activities.
3. Each biome type occurs in every continent except Antarctica.
Which of the statement(s) given above is/are incorrect?
(a) Only 2 (b) 1 and 2 (c) 2 and 3 (d) All of these
Stage 1 Know the Trend 7

34. Which of the following climatic factors regulate decomposition in the natural environment?
1. Soil moisture 2. Temperature 3. Wind
Select the correct answer using the codes given below.
(a) 1 and 2 (b) 1 and 3 (c) 2 and 3 (d) All of these

35. Which of the following mission mode projects are exclusively implemented by the State
Governments under National e-Governance Plan?
1. Education 2. Health
3. PDS 4. Agriculture
5. Insurance 6. Banking
7. Road transport
Select the correct answer using the codes given below.
(a) 2, 4 and 5 (b) 1, 2, 3 and 6 (c) 1, 2, 3, 4 and 7 (d) 1, 3 and 4

36. Consider the following statements with reference to the distribution of volcanoes around the
world.
1. The Atlantic costs have many active volcanoes.
2. The Pacific ocean has highest number of active volcanoes in its coasts thanks to folded and faulted land
forms.
Which of the statement(s) given above is/are correct?
(a) Only 1 (b) Only 2
(c) Both 1 and 2 (d) Neither 1 nor 2

37. Consider the following proposals.


1. All parties would be allowed to express their stance on the partition and their views before the
plebiscite.
2. Division, if required, should be by plebiscite by the total population. The participants of plebiscite
should be decided based on education, wealth and social status.
3. The transfer of population, if any would be absolutely on a voluntary basis.
Which of the above proposals were include in C Rajgopalachari formula (1944)?
(a) 1 and 2 (b) 2 and 3
(c) 1 and 3 (d) All of these

38. Consider the following statements.


1. The Godavari is largest peninsular river and also known as Dakshin Ganga.
2. It originates Mahabaleshwar in Shahyadri.
3. Its major part is navigable.
Which of the statement(s) given above is/are correct?
(a) Only 1 (b) 1 and 2
(c) 1 and 3 (d) All of these

39. Consider the following statements regarding the borrowing powers of the Union and the States.
1. The Union Government has unlimited powers to borrow from anywhere and for any amount.
2. State Governments can borrow from outside India, but only with the permission of the centre.
3. State Governments have unlimited powers to borrow from inside India.
Which of the statement(s) given above is/are correct?
(a) Only 1 (b) 1 and 2
(c) 2 and 3 (d) None of these
8 Practice Set 1

40. Which of the following statements regarding the Department Related Standing Committees are
correct?
1. These committees are meant only for scrutinising the budget during the recess of Parliament in the
budget session.
2. The members of these committees are elected based on proportional representation by a single
transferable vote.
3. The report of the committee on budgetary proposals can contain recommendations for cut motions also.
4. All these committees function under the Lok Sabha.
Select the correct answer using the codes given below.
(a) 1 and 2 (b) 3 and 4 (c) All of these (d) None of these

41. Which of the following statement(s) is/are correct regarding Eutrophication?


1. It decreases the Biological Oxygen Demand (BOD).
2. It is caused only due to human activities.
3. It increases the biomass of the phytoplanktons.
Select the correct answer using the codes given below.
(a) 1 and 2 (b) 2 and 3 (c) 1 and 3 (d) Only 3

42. A soil affected by intensive leaching and rich in iron oxide, aluminium compound and potash, but
poor in organic matter, nitrogen, phosphate and calcium, making it unsuitable for cultivation and
is commonly found in which of the following Indian states?
1. Madhya Pradesh 2. Kerala 3. Tamil Nadu 4. Uttar Pradesh
5. Haryana
Select the correct answer using the codes given below.
(a) 4 and 5 (b) 2 and 3 (c) 1, 2 and 3 (d) 1, 2, 3 and 5

43. Consider the following statements about the Odissi dance.


1. Odissi evolved independently apart from the tenets of Natyashastra.
2. Its techniques combine two basic postures of Chowk and Tribhanga.
3. The ‘Chowk’ is a very masculine stance with the weight of body equally balanced.
Which of the statements given above are correct?
(a) 1 and 2 (b) 1 and 3 (c) 2 and 3 (d) All of these

44. Consider the following statements regarding Saline soils.


1. Saline soils also known as Usara soils occur in arid and semi-arid region and in water logged and
swampy areas.
2. Their structure ranges from sandy to loamy. They lack in nitrogen and calcium.
3. In the areas of intensive cultivation with excessive use of irrigation, especially in areas of green
revolution the fertile alluvial soils are becoming saline soils.
Which of the statement(s) given above is/are correct?
(a) Only 1 (b) Only 2 (c) 1 and 3 (d) All of these

45. The main objective of the National Development Council is


1. to promote common economic policies in all vital spheres.
2. to strengthen and mobilise the efforts and resources of the nation in support of the plan.
3. to ensure balanced and rapid development of all parts of country.
4. to secure cooperation of states in execution of the plan.
Which of the statements given above are correct?
(a) 1 and 2 (b) 2 and 3 (c) 1, 2 and 3 (d) All of these
Stage 1 Know the Trend 9

46. Which of the following statements regarding the Right to Equality in the chapter on Fundamental
Rights in the Constitution are incorrect?
1. Equality before the law is an absolute right.
2. Equality of opportunity can be violated in favour of women.
3. Abolition of titles means titles given by foreign countries also.
4. Constitution provides for exceptions to right to equality in favour of backward castes.
Select the correct answer using the codes given below.
(a) 1 and 2 (b) 1, 2 and 3 (c) 1, 2 and 4 (d) 3 and 4

47. Consider the following statements about Bihu festivals of Asom.


1. The Bihu festivals are associated with different farming seasons of Asom.
2. Assamese celebrate three types of Bihus namely Rongaali Bihu, Kaati Bihu and Magh Bihu.
3. These three Bihus mark distinct phases of the wheat crop grown in Asom.
4. Rongaali Bihu marks the starting of Assamese new year.
Which of the statements given above are correct?
(a) 1, 2 and 3 (b) 3 and 4 (c) 1, 2 and 4 (d) All of these

48. Consider the following statements in respect of the Regulating Act of 1773.
1. The tenure of the Council members depended on the pleasure of the Governor-General.
2. The Governor-General in Council was empowered with the provision of casting vote.
3. The Governor-General in Council was vested with civil and military powers.
Which of the statements given above are correct?
(a) 1 and 3 (b) 1 and 2 (c) 2 and 3 (d) All of these

49. Which of the following local winds are warm winds?


1. Mistral 2. Chinook 3. Foehn 4. Sirocco
Select the correct answer using the codes given below.
(a) 2 and 3 (b) 1, 3 and 4 (c) 2 and 4 (d) 2, 3 and 4

50. Which of the following statement(s) is/are correct regarding Block mountains?
1. Block mountains are created when large areas are broken and displaced vertically.
2. The uplifted blocks are termed as graben and the lowered blocks are called horsts.
3. The Aravali range in India is one of the oldest Block mountain systems in the world. The range has
considerably worn down due to the processes of erosion.
Select the correct answer using the codes given below.
(a) Only 1 (b) 1 and 2 (c) 2 and 3 (d) All of these

51. Which of the following statements correctly defines the Phillips curve?
(a) It shows the relationship between tax revenue collection and tax rate
(b) It indicates the variation of productivity of a firm with the change in the capital investment
(c) It shows the relationship between per-capita income and population
(d) It shows the relationship between the rate of inflation and rate of unemployment

52. Consider the following statements.


1. The anti-defection law gives for the first time a clear-cut constitutional recognition to the political
parties.
2. The total number of Minister including the Prime Minister in the Central Council of Minister shall not
exceed 10% of the total strength of the Lok Sabha.
Which of the statement(s) given above is/are incorrect?
(a) Only 1 (b) Only 2 (c) Both 1 and 2 (d) Neither 1 nor 2
10 Practice Set 1

53. Consider the following statements with reference to digestion in grass eating animals.
1. They quickly swallow the grass and store it in a separate part of the stomach called rumen.
2. Partially digested food in the rumen is called cud.
3. In ruminants, the cellulose of the food is digested by the action of certain bacteria which are not present
in humans.
Which of the statements given above are correct?
(a) l and 2 (b) l and 3
(c) 2 and 3 (d) All of these

54. Consider the following statements about Indian Nationalist Army (INA).
1. INA or Azad Hind Fauj was founded by Subhash Chandra Bose with the help of Japanese in 1942 in
Singapore.
2. INA was initially composed of the captured Indian army soldiers at Malaya.
3. Subhash Chandra Bose silently left Calcutta in January 1941 and reached Singapore where he was
given the title of ‘Netaji’.
Which of the statement(s) given above is/are correct?
(a) 1 and 2 (b) Only 2
(c) Only 3 (d) All of these

55. Consider the following about the Amrita Devi Bishnoi Wildlife Protection Award.
1. Tribal people of Rajasthan who protect wildlife in the desert regions.
2. Individuals or communities from rural areas that have shown extraordinary courage and dedication in
protecting wildlife.
3. People who practice Transhumance in the forest and hilly regions who protect traditional knowledge.
Which of the statement(s) given above is/are correct?
(a) 1 and 2 (b) Only 2 (c) All of these (d) None of these

56. Consider the following statements regarding Mission Indradhanush.


1. It is a mission under the Ministry Health and Family Welfare.
2. It is a mission to achieve full immunisation coverage for all children by 2020.
3. Mission aims to cover all those children who are either unvaccinated or are partially vacinated against
seven preventable diseases.
Which of the statements given above are correct?
(a) 1 and 2 (b) 2 and 3 (c) All of these (d) None of these

57. Which of the following methods were included in the nationalist programme during
Non-Cooperation movement?
1. Surrender of titles. 2. Boycott of English education.
3. Boycott of courts of law. 4. Boycott of foreign clothes.
5. Non-payment of taxes.
Select the correct answer using the codes given below.
(a) 2, 3 and 5 (b) 1, 3, 4 and 5 (c) 1 and 4 (d) All of these

58. Consider the statements regarding Gupta Period.


1. During this period, many new towns were raised, all old towns were fortified or atleast strengthened
and no old towns declined in North India.
2. Guptas possessed a large quality of gold and issued purest quality of gold coins.
3. Images of Vishnu, Shiva and other Hindu Gods were fashioned for the first time during this period.
Which of the statement(s) given above is/are correct?
(a) Only 3 (b) 2 and 3 (c) 1 and 2 (d) 1 and 3
Stage 1 Know the Trend 11

59. Which of the following factor(s) is/are responsible for West Bengal being the largest producer of
Jute in India?
1. It experiences high temperature and receives high rainfall.
2. Annual flood-silts provide natural fertiliser.
3. It has the highest concentration of Jute mills.
4. It is located at the seaport.
Select the correct answer using the codes given below.
(a) Only 3 (b) 1 and 2 (c) 1, 2 and 3 (d) All of these

60. How is the independence of the Supreme Court ensured in the Constitution?
1. Retired judges of Supreme Court are prohibited from taking up public office.
2. Expenses of the Supreme Court are charged on the Consolidated Fund of India.
3. Supreme Court has complete freedom to appoint any of its staff without interference.
4. Parliament can extend the jurisdiction of the Supreme Court, but cannot curtail the same.
Which of the statements given above are correct?
(a) 1 and 2 (b) 2 and 4 (c) 2, 3 and 4 (d) 3 and 4

61. The Act of 1935, is one of the most important landmarks in the Constitutional History of India.
Which of the following statements are correct about it?
1. It provided for the establishment of an All-India Federation consisting of provinces and princely states
as units.
2. The federation never came into being as the princely states did not join it.
3. It provided for the adoption of dyarchy at the provinces.
4. It provided for the establishment of a Federal Court, which was set up in 1937.
Select the correct answer using the codes given below.
(a) 1, 2 and 3 (b) 1, 2 and 4 (c) 1, 3 and 4 (d) All of these

62. Which of the following statements about life expectancy are correct?
1. It is the number of years that an average person can expect to live.
2. Women tend to have a higher mortality rate at every age.
3. Life expectancies are also used when determining the value of a life settlement, a life insurance policy
sold for a cash asset.
4. Life expectancy is one of the factors in measuring the Human Development Index (HDI) of each nation.
5. Lindy effect and Glasgow effect are theories related to life expectancy.
Select the correct answer using the codes given below.
(a) 1, 3 and 4 (b) 2, 3 and 5 (c) 1, 2 and 4 (d) 1, 3, 4 and 5

63. Consider the following statements.


1. India ranks third in the production of cereals after Brazil and Ukraine.
2. India ranks second in the production of rice after China.
3. India ranks first in the production of pulses.
Which of the statement(s) given above is/are correct?
(a) Only 1 (b) 2 and 3 (c) Only 3 (d) 1 and 3

64. Which of the following pairs are incorrectly matched?


Mineral Region
1. Coal : Damodar valley
2. Copper : Singhbhum
3. Uranium : Kullu
4. Natural gas : Karnataka
Codes
(a) 1 and 4 (b) 2 and 4 (c) 1, 3 and 4 (d) Only 4
12 Practice Set 1

65. Which of the following statements are correct?


1. The vascular tissue for the transport of water and nutrients in the plant is called the xylem.
2. Veins are the vessels which carry carbon dioxide-rich blood from all parts of the body back to the heart.
3. Arteries carry oxygen-rich blood from the heart to all parts of the body.
4. The arteries have thin walls whereas the veins have thick elastic walls.
5. The food synthesised by leaves has to be transported to all parts of the plant. This is done by the
vascular tissue called the phloem.
Select the correct answer using the codes given below.
(a) 2 and 4 (b) 1, 3 and 4 (c) 1, 3 and 5 (d) 1, 2, 3 and 5

66. Consider the following statements about the policy of Doctrine of lapse.
1. Lord Dalhousie used ‘Doctrine of Lapse’ as a chief instrument for expansion of Birtish territories in
India.
2. Under two ‘Doctrine of Lapse’ the Indian state was not allowed to pass on to an adopted heir, unless
prior approval of British.
3. Apart from Jhansi, Nagpur and Satara were annexed using this doctrine.
Which of the statements given above are correct?
(a) 1 and 2 (b) 2 and 3 (c) 1 and 3 (d) All of these

67. Consider the following statements.


1. The green plants in a terrestrial ecosystem capture about only 1% of the solar energy falling on their
leaves.
2. Only 10% of the energy is transferred from one trophic level to the other via the food chain.
3. The lower trophic levels house greater number of individuals than the higher levels.
Which of the statement(s) given above is/are correct?
(a) 1 and 2 (b) Only 2 (c) 1 and 3 (d) All of these

68. The Association of Southeast Asian Nations, or ASEAN, was established on 8th August, 1967 in
Bangkok, Thailand, with the signing of the ASEAN Declaration (Bangkok Declaration) by the
Founding Fathers of ASEAN, namely Indonesia, Malaysia, Philippines, Singapore and Thailand.
What is its motto?
(a) All For One, One For All (b) One Vision, One Identity, One Community
(c) World Is Our Family (d) Cooperation For World’s Development
69. Consider the following statements regarding Kisan Vikas Patra Scheme.
1. The scheme will provide facility of unlimited investment by way of purchase of certificate from post
office in various denominations.
. 2. The certificates can also be issued in single or joint names and can be transferred from one person to
any other person.
3. The facility of transfer from one post office to another anywhere in India and of nomination will also be
available under the scheme.
Which of the statement(s) given above is/are incorrect?
(a) Only 1 (b) Only 3 (c) 2 and 3 (d) None of these
70. Examine the following statements regarding the administration of scheduled and tribal areas in the
Constitution.
1. Distinction between scheduled and tribal areas is made on the basis of forest and non-forest tribes.
2. Any law made by Parliament can be excluded from application to a scheduled area by the Governor.
3. Governor is free to increase or decrease the size and change the name of autonomous district councils in
tribal areas.
Which of the statement(s) given above is/ are correct?
(a) 1 and 2 (b) 1 and 3 (c) 2 and 3 (d) Only 1
Stage 1 Know the Trend 13

71. Consider the following statements regarding Gram Nyayalayas set up under the Gram Nyayalaya Act.
1. Gram Nyayalayas have powers of both Civil and Criminal courts.
2. Awards of Gram Nyayalayas cannot be appealed in Higher courts.
3. Plea bargaining has also been enabled in Gram Nyayalayas.
4. Gram Nyayalayas are required to follow procedures under Indian Evidence Act.
5. Jurisdiction of Gram Nyayalayas is the same as the District court and Sessions court.
Which of the statements given above are correct?
(a) 1, 2 and 5 (b) 1 and 3 (c) 2 and 4 (d) 3, 4 and 5
72. Consider the following statements about Manipuri dance
1. With the arrival of Vaishnavism, popular Radha Krishan theme of Rasleela evolved in Manipuri.
2. Manipuri dance is accompanied by Kirtan.
3. Thang-Ta is a martial art performed by the male dancers of Manipuri.
Which of the statements given above are correct?
(a) 1 and 2 (b) 2 and 3 (c) 1 and 3 (d) All of these
73. Which of the following pairs are not correctly matched?
Agreement Signed in
1. CITES : Washington, USA
2. UNFCCC : Rio de Janeiro, Brazil
3. Convention on Biological Diversity : Nagoya, Japan
4. Convention on Nuclear Safety : Berlin, Germany

Codes
(a) 1 and 4 (b) 2 and 3 (c) 3 and 4 (d) 1 and 3
74. Which of the following can be a form of economic ‘Protectionism’ by India?
1. Visa restriction for foreign nationals.
2. Preferential market access policies for domestic industries.
3. Discouraging FII.
4. Increasing custom duties on imported goods and services.
5. Increasing custom duties on exported goods and services.
Select the correct answer using the codes given below.
(a) 2, 3 and 5 (b) 1, 2 and 3 (c) 1, 2 and 4 (d) 1, 3 and 4
75. Consider the following types of nuclear reactors.
1. Pressurised Heavy Water Reactor (PHWR)
2. Fast Breeder Reactor (FBR)
3. Advanced Heavy Water Reactor (AHWR)
Arrange the above types of nuclear reactors based in the chronological order as per India’s three
stage nuclear power programme.
(a) 1, 2, 3 (b) 1, 3, 2 (c) 2, 3, 1 (d) 3, 1, 2
76. Consider the following statements regarding Crimean Peninsula.
1. Crimean Peninsula is washed by Black Sea and Sea of Azov.
2. Crimean Peninsula borders with Ukraine in the North and separated from Russia by Kerch strait in the
East.
3. The Black Sea parts of Crimea provide quick access to the Eastern Mediterranean Balkans and Middle
East.
4. Sevastopol port is the base of NATO Navy Black Sea fleet.
Which of the statements given above are correct?
(a) 1 and 3 (b) 1 and 4 (c) 1, 2 and 3 (b) 2, 3 and 4
14 Practice Set 1

77. First Round Table Conference convened in London in November, 1930. Ramsay Macdonald was
its Chairman. Congress was not a participant of it. Who came to Gandhiji to persuade him to meet
Lord Irwin and negotiate a settlement in the name of Congress?
(a) BR Ambedkar (b) Tej Bahadur Sapru
(c) Ghanshayam Das Birla (d) CY Chintamani

78. Consider the following statements regarding the Union Finance Commission.
1. The Chairman of Finance Commission is required to have wide experience or expertise in any one of
public finance, administration, economics etc.
2. Finance Commission recommendations are binding on the government.
3. Terms of reference of the Finance Commission are exhaustively listed in the Constitution.
4. It also has to consider the recommendations made by the various State Finance Commissions.
Which of the statement(s) given above is/are correct?
(a) 1 and 2 (b) 2 and 3
(c) 1, 2 and 4 (d) Only 4

79. Which of the following factors affect surface air temperature?


1. Latitude 2. Altitude
3. Cloud cover 4. Land/Water heating differences
Select the correct answer using the codes given below.
(a) 1 and 3 (b) 1, 2 and 3 (c) 2, 3 and 4 (d) All of these

80. Consider the following soil conservation methods.


1. No-tillage farming is planting without removing the existing plant cover and previous crop
residues.
2. Minimum tillage farming is ploughing only along edges of croplands to reduce soil loss from water
runoff.
3. Gully reclamation is seeding gullies with quick-growing plants and using check dams of manure and
straw to reduce erosion.
Which of the statement(s) given above is/are correct?
(a) 1 and 2 (b) 1 and 3 (c) Only 2 (d) All of these

81. Consider the following statements in the context of Ayyappa Temple.


1. The hill temple of Lord Ayyappa in Shabarimala is Situated in the Eastern ghats.
2. The temple is open to all devotees irrespective of caste, creed, religion or social status.
3. Girls and women between 10 and 50 years of age are not allowed to visit the temple to facilitate strict
observance of celibacy in the temple complex.
Which of the statements given above are correct?
(a) 1 and 3 (b) 1 and 2 (c) 2 and 3 (d) All of these

82. Consider the following statements about the Central Electronics Limited (CEL).
1. It is a public sector enterprise of the department of Scientific and Industrial Research, Ministry of
Science and Technology, Government of India.
2. It is one of the top producers of single crystalline silicon solar cells in the world.
3. Its emphasis is on indigenous technology inducted both from its in-house development and from the
national laboratories.
Which of the statement(s) given above is/are incorrect?
(a) 1 and 3 (b) Only 2 (c) Only 3 (d) None of these
Stage 1 Know the Trend 15

83. Consider the following statements about following musical forms of Carnatic music.
1. Gitam are the scholarly and complex compositions in the Tamil and Telugu languages.
2. There are many intricate variations in the music of Gitam.
3. A notable feature of Gitam is the existence of Matrika Padas.
Which of the statement(s) given above is/are correct?
(a) 1 and 2 (b) Only 2 (c) Only 3 (d) All of these

84. Consider the following statements with reference to eutrophication.


1. It is the natural aging of a lake by biological enrichment of its water.
2. As the lake’s fertility increases with more incoming nutrients, plants and animal life burgeons, and
organic remains begin to be deposited on the lake bottom.
3. Eutrophication increases oxygen and hence contributes to over production of phytoplankton.
Which of the statements given above are correct?
(a) 1 and 3 (b) 1 and 2 (c) 2 and 3 (d) All of these

85. Which of the following statement(s) is/are correct?


1. The Net primary productivity in an ecosystem is calculated by reducing the respiratory loss from the
gross primary productivity.
2. Eutrophication is the process of excessive depletion of nutrients in an aquatic ecosystem.
3. About 90% energy is transferred from the lower tropic level to the higher tropic level.
4. Primary and secondary succession in an ecosystem are always a natural process.
Select the correct answer using the codes given below.
(a) 1, 2 and 4 (b) 1, 3 and 4 (c) Only 1 (d) Only 4

86. Consider the following statements regarding the solar water heater.
1. The copper pipes are used for the solar water heaters.
2. The pipes are used for heating and are in the spiral shape.
3. The pipes are painted with black colour.
4. It works on the principle of greenhouse effect.
Which of the statements given above are correct?
(a) 1, 3 and 4 (b) 1 and 2 (c) 2 and 3 (d) All of these

87. Which of the following pair(s) is/are correctly matched?


Resource Country
1. Largest natural gas reserves : USA
2. Largest oil reserves : Saudi Arabia
3. Largest coal reserves : Russia
Codes
(a) l and 2 (b) Only 2 (c) 2 and 3 (d) All of these

88. Hormones are classified in which of the following chemical groups?


1. Polypeptide 2. Steroid 3. Amino-acid derivative
Select the correct answer using the codes given below.
(a) 2 and 3 (b) 1 and 3 (c) Only 2 (d) All of these

89. Which of the following pairs are correctly matched?


1. Strachey Commission : Famine
2. Hartog Committee : Education
3. Fraser Commission : Police Reforms
Codes
(a) 1 and 2 (b) 1 and 3 (c) 2 and 3 (d) All of these
16 Practice Set 1

90. Consider the following statements regarding delegation of functions between the union and states.
1. States cannot delegate any of their administrative powers to the union.
2. Union can delegate any function on the State Government even without taking the consent of the latter.
3. Functions of Union Government can be delegated upon a State Government even without taking
permission from the Union Parliament.
Which of the statement(s) given above is/are correct?
(a) Only 1 (b) 2 and 3
(c) 1 and 2 (d) 1 and 3

91. Consider the following statements regarding citizenship in India.


1. Provisions relating to termination and renunciation of Indian citizenship are also provided in the
Constitution.
2. Any Indian citizen of full age can renounce his citizenship voluntarily.
3. Citizenship of a person can also be taken away by the Central Government in public interest.
4. Questions relating to determination of citizenship can only be determined by the Supreme Court.
Which of the statement(s) given above is/are correct?
(a) 1 and 2 (b) 1, 2 and 3
(c) Only 3 (d) 3 and 4

92. Consider the following statements about Basel Convention.


1. This international treaty was signed explicitly to prevent transfer of hazardous wastes from the
developed countries to developing and less developed countries.
2. This international treaty talks the nuclear wastes and hazards.
3. As per this convention, a waste is called hazardous waste if it is defined as hazardous in the law of
importing country only.
Which of the statement(s) given above is/are correct?
(a) Only 1 (b) 1 and 2
(c) 2 and 3 (d) All of these

93. Which of the following pairs are correctly matched?


Folk Song Region
1. Kajri : Uttar Pradesh
2. Chhakri : Punjab
3. Mando : Goa
4. Pankhida : Rajasthan
Codes
(a) l and 2 (b) 1, 3 and 4
(c) 2, 3 and 4 (d) None of these

94. Consider the following actions.


1. Increasing interest rates.
2. Introduction of Inflation-Indexed National Saving Securities (IINSSs) for retail investors.
3. Increasing excise duty on gold.
4. Permitting FDI in multi-brand retail trading.
Which of the above are the steps taken by RBI to counter inflation?
(a) l and 4 (b) 1, 3 and 4
(c) 1 and 2 (d) 2, 3 and 4
Stage 1 Know the Trend 17

95. With reference to spectrum management, consider the following statements.


1. The purpose of spectrum management is to mitigate radio spectrum pollution and maximise the benefit
of usable radio spectrum.
2. Spectrum and satellite orbits are scarce natural resources, which are prone to harmful interference,
hence exclusively managed by the governments.
3. Spectrum management is the process of regulating the use of radio frequencies to promote efficient use
and gain a net social benefit.
Which of the statements given above are correct?
(a) 1 and 2 (b) 2 and 3
(c) 1 and 3 (d) All of these

96. Which of the following statements meets the objective of the Zonal Councils.
1. To achieve motional integration of the country.
2. To secure some kind of political equilibrium between different regions of the country.
3. To enable the centre and states to cooperate with each other in social and economic matters.
Which of the statements given above are correct?
(a) 1 and 3 (b) 1 and 2
(c) 2 and 3 (d) All of these

97. Which of the following are narrowly utilitarian arguements for conserving biodiversity?
1. Biodiversity plays a major role in many ecosystem services that nature provides.
2. Humans derive countless direct economic benefits from nature such as food (cereals, pulses, fruits), fire
wood fibre, construction material (tannins), lubricants, dyes, resins, perfumes) and products of
medicinal importance.
3. With increasing resources put into ‘bioprospecting’ nations endowed with rich biodiversity can expect
to reap enormous benefits.
Select the correct answer using the codes given below.
(a) 1 and 2 (b) 2 and 3 (c) 1 and 3 (d) All of these

98. Which of the following are examples of primary pollutants?


1. Sulphur Dioxide 2. Peroxyacetyl Nitrate (PAN)
3. Particulate matters 4. Carbon Monoxide
5. CFC
Select the correct answer using the codes given below.
(a) 1, 2 and 4 (b) 1, 3 and 5 (c) 1, 4 and 5 (d) All of these

99. Which of the following is not a function of the International Monetary Fund (IMF)?
(a) To promote exchange rate stability
(b) To assist in the elimination of foreign exchange restrictions
(c) To provide loans to member nations to boost their export competitiveness
(d) To assist member nations by temporarily providing financial resources to correct Balance of
Payments (BoP) Crisis

100. Which of the following is not goal of 12th Five Year Plan (2012-17) regarding population of India?
1. Reducing Maternal Mortality Rate (MMR) to one per thousand livebirths.
2. Reducing Infant Mortality Rate (IMR) to twenty-five per thousand livebirths.
3. Raising Child Sex Ratio (0-6 years) to 950 by the end of the plan.
4. Reducing Total Fertility Rate (TFR) to 2.1 by the end of the plan.
Select the correct answer using the codes given below.
(a) Only 2 (b) Only 3 (c) 3 and 4 (d) None of these
18 Practice Set 1
Stage 1 Know the Trend 19

PRACTICE SET 1 OVERALL ANALYSIS


Answers with Explanations
1. (d) All the given biosphere reserves of India are part of the 8. Bharat Stage Emission Standards are emission standards
world network of biosphere reserves, based on the UNESCO instituted by the Government of India to regulate the output of
Man and the Biosphere (MAB) programme list. The air pollutants from internal combustion engine equipment,
Achanakmar-Amarkantak biosphere reserve of Chhattisgarh including motor vehicles. The phasing out of 2 stroke engine
was added to the list in the last year. and stopping the production of Maruti-800 was done under
this initiative. The standards and the timeline for
2. (c) CITES also known as the Convention on International Trade
implementation are set by the central pollution control board
in Endangered Species of wild flora and fauna entered into
under the ministry of environment and forests. The standards
force in 1975 and became the only treaty to ensure that
based on European regulations were first introduced in 2000.
international trade in plants and animals does not threaten their
Bharat Stage Norm III have been enforced across the country.
survival in the wild. Currently, 179 countries (called Parties),
In 13 major cities Bharat Stage-IV norms have been in place
including the United States, implement CITES, the convention
since 2010.
on international trade in endangered species of wild flora and
fauna is also known as the Washington Convention. 9. The correct order of given mountain, ranges from North to
South is: Kaimur range, Maikala range, Balaghat range and
3. (a) Kerala has registered the lowest growth rate 9.4 in the
Ajanta range . These are ranges of Central India. Most of them
country during 2011 census. A continuous belt of states from
lies in Madhya Pradesh.
West to East in the North-West, North and North central parts
of the country has relatively high growth rate than the Southern 10. Tropical deciduous forests are the most widespread
states. 2001-2011 has recorded the lower growth rate of forests of India. They are also called the monsoon forests and
population (17.64%) than 1991-2001 which was (21.15%). spread over the region receiving rainfall between 200 cm and
70 cm. Trees of this forest type shed their leaves for about six
4. (c) Hadis were the sayings and doings of prophet, Shariat is to eight weeks in dry summer in order to save moisture, these
the Islamic law, whereas Zawabit was the revenue law. Farman forests are further divided into moist and dry deciduous. The
was the royal order and fatwas were the Islamic legal former is found in areas receiving rainfall between 200 and 100
decisions. cm whereas later are found in areas rainfall receiving from 70
5. (b) Statement 1 is incorrect, because the power to make laws to 150 cm.
to implement Fundamental Rights is available only to the 11. (d) Ultrasound, also called sonography, is another type of 3-D
Parliament, so as to ensure uniformity across the country. computerised imaging. Using brief pulses of ultrahigh
Statement 2 is correct and Parliament can amend any frequency acoustic waves (lasting 0.01 seconds), it can
Fundamental Rights as long as such an amendment does not produce a sonar map of the imaged object. The technique is
violate the ‘basic features’ of the Constitution. Statement 3 is similar to the echolocation used by bats, whales and dolphins.
incorrect, as Article 17 against untouchability is also applicable By measuring the echo waves, it is possible to determine the
to private individuals. Statement 4 is also incorrect. Although size, shape, location and consistency (whether it is solid,
most Fundamental Rights are negative in nature as they fluid-filled or both) of an object.
impose restrictions upon the state to not to encroach upon the
rights of the individual, some of them also give positive powers Ultrasound is a very safe, non-invasive imaging technique.
to individuals like the right to establish minority institutions. Unlike X-rays, sonography does not use ionising radiation to
produce an image. It gives a clear picture of soft tissues,
6. (a) Frequent amendments of the Constitution, that involves, which do not show up in X-rays. Ultrasound causes no health
debates and discussion are considered as deepening of problems (for a mother or unborn factor) and may be repeated
democracy and legislation. as often as necessary. Current evidence indicates that
Whereas, growth of delegated legislation, low attendance, diagnostic ultrasound is safe for the unborn child, unlike
frequent promulgation of ordinances by passing parliamentary radiographs, which employ ionising radiation. However, no
processes and unethical behaviour of parliamentarian have randomised controlled trials have been undertaken to test the
wasted precious time of Parliament, that led to decline in its safety of the technology and thus, ultrasound procedures are
working. So, except Statement 4 all other are correct. generally not done repeatedly unless medically indicated.
7. (b) Mudiyettu is a traditional theatre, dance and drama of 12. (b) On 9th September, 2015, the Union Government has
Kerala. It enacts the battle between goddess ‘Kali’ and demon approved the establishment of 21st Law Commission of India
Darika. The theatre is performed in the temples called for a period of three years i.e. 1st September, 2015 to 31st
‘Bhagwati Kavas’ or the temple of Mother Goddess. In year August, 2018. It consists of 1 chairperson and 11 members of
2010, Mudiyettu was inscribed in the UNESCO’s list of which 2 members i.e. Secretary Department of Legal Affairs
‘Intangible Cultural Heritage of Humanity’. Apart from and Legislative Department are the ex-officio members. It
Mudiyettu, the Sanskrit theatre Koodiyattam is another popular undertake research in law and review of existing laws in India
art form of Kerala. for making reforms therein and enacting new legislation. Its
Stage 1 Know the Trend 21

function is to identify laws which are no longer relevant and 20. (d) When a proclamation of national emergency is made, the
recommending for the repeal of obsolete and unnecessary six Fundamental Rights under Article 19 are automatically
enactments; to suggest enacting new legislations to implement suspended. Any law made by Parliament under an
the DPSP and to attain the objectives set out in the Preamble of emergency cannot be declared to be void on the grounds
the Constitution. that, it violates a Fundamental Rights under Article 19. The
13. (d) ISRO took 8 years to develop the LAM and spent several right to protection in respect of conviction for offences (Article
years to build a high-altitude test facility to validate the rocket 20) and the right to life and personal liberty (Article 21) remain
motor in the airless condition of space. LAM refers to a rocket enforceable even during emergency.
motor that is regularly employed on artificial satellites destined 21. (c) Characteristics of black soils are as follow
for a geostationary orbit.
● Fi ne textured and clayey in nature.
14. (c) Insider trading is the trading of a public company’s stock or ● Has high qualities of lime, iron, magnesium and generally
other securities (such as bonds or stock options) by individuals poor percentage of phosphorus, nitrogen and organic
with access to non-public information about the company. In matter.
various countries, insider trading based on inside information is ● Black in colour as it is formed from weathered lava rocks,
illegal. This is because it is seen as unfair to other investors who ● Soil’s colour also varies from black to chestnut brown.
do not have access to the information.
● Very clayey and therefore highly retentive of water.
15. (c) The government has replaced Planning Commission with a Because of high clay content, these soils expand when wet
new institution named NITI Aayog (National Institution for and become difficult to plough.
Transforming India). It is a non-constitutional, cabinet resolution ● During dry season, black soils shrink and develop big
as Planning Commission was established. cracks which help in air circulation.
NITI Aayog will not have any financial powers. It is merely a ● Soil is very fertile in most of places.
advisory body and financial allocations will be done by Finance
● Poor in nitrogen, phosphorus, potassium and organic
Ministry. matter and rich in potash and lime.
NITI Aayog will comprise of full time as well as part time
members. 22. (a) Slave dynasty ruled (1206-1290).
16. (b) Through Gandhi’s inspiration, Harijan Sevak Sangh was Khiljis ruled (1290-1320).
founded to combat untouchability and a new weekly Harijan. He Tughlaqs ruled (1320-1412).
led movement for temple entry at Guruvayur, inspired legislation 23. (c) The Himalayan Yew (Taxus wallichiana) is a medicinal
in the Central Assembly and Madras Legislature. plant found in various parts of Himachal Pradesh and
So, statements (a), (c) and (d) are correct. Arunachal Pradesh. A chemical compound called ‘taxol’ is
17. (d) The gradual increase in rural indebtedness in India, under extracted from the bark, needles, twigs and roots of this tree
the British rule was due to the fragmentation of landholdings, and it has been successfully used to treat some cancers the
decline of cotton industries, lack of development of irrigational drug is now the biggest selling anti-cancer drug in the world.
facilities and introduction of cash crops. The species is under great threat due to overexploitation. In
the last one decade, thousands of Yew trees have dried up in
18. (c) Characteristics of Savanna region. various parts of Himachal Pradesh and Arunachal Pradesh.
● Savanna is a tropical grassland.
24. (c) Agenda 21 is a non-binding, voluntarily implemented
● Savanna eco-region is the paradise of hunters.
action plan of the United Nations with regard to sustainable
● Savanna region has coarse grass and fire resistant trees. development. It is a product of the UN Conference on
● Maximum biodiversity found in equatorial rainfall region. Environment and Development (UNCED) held in Rio de
19. (a) Statement 1 is correct, since any bill of the State Legislature, Janeiro, Brazil, in 1992. It is an action agenda for the UN,
which endangers the position of the High Court has to be other multilateral organisations and individual governments
reserved by the Governor for the recommendation of President. around the world that can be executed at local, national and
Statement 2 is also correct as Governor can also reserve State global levels. The ‘21’ in Agenda 21 refers to the 21st century.
Bills for the consideration of President for many other reasons It has been affirmed and modified at subsequent UN
like unconstitutional, opposing directive principles of state conferences. In 2012, at the United Nations Conference on
policy etc. sustainable development the attending members reaffirmed
There is no certain category of Money Bills to be compulsorily their commitment to Agenda 21 in their outcome document
reserved for the consideration of the President. Normally, the called ‘The Future We Want’. 180 leaders from nations
Governor gives his assent to a Money Bill as it is introduced in participated.
the state legislature with his prior permission. However he may 25. (a) There are two systems, one is allocation based and
reserve the bill for presidential assent. So, Statement 3 is also another is demand driven. Allocation is also called supply
incorrect. Statement 4 is incorrect because during financial driven. The demand driven implies that the scheme
emergency the reservation of all Money Bills or other Financial implementation is driven from below by the grassroots
Bills for the consideration of the President after they are passed institutions of poor. The planning will be highly decentralised,
by the Legislature of the State. it is driven by the institutions such as PRIs, SHGs etc.
22 Practice Set 1

26. (d) The government has involved all stakeholders including 31. (c) Tholu Bommalata is Andhra Pradesh’s shadow
decision makers, politicians, researcher and entrepreneurs of water puppet. These are large size jointed waist, shoulder,
resources from India as well as abroad to discuss strategies for elbow and knees. The puppet plays are drawn from
managing the demands and supplies in the right manner. The first Ramayana, Mahabharata and Puranas. Putul Nach is a
event was organised in New Delhi 2012 and theme was ‘Water, rod puppet form of West Bengal. The puppets are
Energy and Food Security : Call for Solution’. The Indian Water dressed like the Jatra characters (a theatre form of
Week 2016 to be held between 4-8th April, 2016 in New Delhi and Bengals). The technique of manipulation are highly
theme is ‘Water For All : Striving Together’. theatrical and music has close similarly with Jatra theatre.
27. (c) Honeybees transform nectar into honey by a process of Togalu Gombeyatta is the shadow puppet of Karnataka.
regurgitation and evaporation. They store it as a primary food These puppets are mostly small in size, however different
source in wax honeycombs inside the beehive. Most characters are assigned different size, large size for kings
microorganisms do not grow in honey because of its low water and Gods and small size fore servants and common
activity of 0.6. However, honey sometimes contains dormant people.
endospores of the bacterium Clostridium botulinum, which can be Yampuri is the traditional rod puppet of Bihar. These
dangerous to infants, as the endospores can transform into puppets are made up of wood, these puppets are in one
toxin-producing bacteria in infants immature intestinal tracts, piece and have no joints.
leading to illness and even death.
32. (d) In 1858, Government of India was placed directly
28. (b) Bhoga or Fief an estate of land one held on condition of feudal under the crown through the secretary of State for India
service: a fee Dvija (twice born) is a male member of the 1st three and all matters were to be seen by him.
varnas in Brahminical Hindu Society.
This Act had following parts
Antyaja means ‘the last born untouchables’. The marriage of a man
● The East India Company’s control came to an end.
of higher varna with a girl from lower varna was called Anuloma. It
was allowed by the sacred texts. The marriage of a girl of higher ● Governor-General of India - He was placed under the
Secretaries of State.
varna with a man of a lower varna was called Pratiloma. It was not
allowed by the sacred texts. ● It ended the system of double government by
abolishing the Board of Control and Court of Directors.
29. (b) Madurai - Coimbatore - Bengaluru region is a predominantly ● A new office of Secretary of State for India was created,
cotton and sugarcane growing region and has developed around which was to be a member of British Cabinet.
silk textiles, sugar, chemicals, machine tools and leather goods
industries. The region receives hydel power from Mettur, Sharavati,
33. Biomes are various regions of our planet that can be
best distinguished by their climate, flora and fauna. There
Sivasamudram, Papanasam and Pykaram projects. Various public
are different ways of classifying biomes, but the common
sector enterprises located in this belt include the Hindustan
elements are climate, habitat, animal and plant
Machine Tools, the Vishveshwaraiya Iron and steel works, the
adaptation, biodiversity and human activity. Every biome
BHEL, the Indian Telephone Industry and the Hindustan
type does not occur in all the continents. Biomes are
Aeronautics Limited.
majorly affected by human activities.
30. (d) All the given statements are correct. There are six major controls
of the climate of any place. They are latitude, altitude, pressure and
34. (a) Decomposition (or rotting) is the process by which
organic substance are broken down into simpler forms of
wind system, distance from the sea (continentality), ocean currents
matter. The process is essential for recycling the finite
and relief features. Due to the curvature of the Earth, the amount of
matter that occupies physical space in the biome.
solar energy received varies according to latitude.
Temperature and soil moisture are the most important
As a result, air temperature decreases from the equator towards the climatic factors that regulate decomposition through their
poles. As one goes from the surface of the Earth to higher altitudes, effects on the activities of microbes.
the atmosphere becomes less dense and temperature decreases.
35. (c) Originally, the NeGP comprised of 27 Mission Mode
The hills are therefore cooler during summers. The pressure and
Projects (MMPs) and 8 components. However, in 2011 it
wind system of any area depend on the latitude and altitude of the
was augmented by adding 4 more projects-Health,
place. Thus, it influences the temperature and rainfall pattern. The
Education, PDS and Posts to increase the list of 27 MMPs
sea exerts a moderating influence on climate. As the distance from
to 31 MMPs. The 31 MMPs comprising the NeGP are
the sea increases, its moderating influence decreases and the
further classified as state, central or integrated projects,
people experience extreme weather conditions.
the list of the 3 types of MMPs are as follow
This condition is known as continentality (i.e. very hot during
Central MMPs under NeGP Banking, Central Excise and
summers and very cold during winters). Ocean currents alongwith
Customs, Income Tax (IT), Insurance, MCA21, Passport,
onshore winds affect the climate of the coastal areas, any coastal
Immigration, Visa and Foreigners Registrations and
area with warm or cold currents flowing past it, will be warmed or
Tracking, Pension, e-Office, posts and UID.
cooled if the winds are onshore.
Stage 1 Know the Trend 23

State MMPs under NeGP Agriculture, Commercial Taxes 41. Eutrophication can be caused by human activities as well
e-District, Employment Exchange, Land Records (NLRMP) as naturally. Eutrophication leads to increased Biological
Municipalities, e-Panchayats, Police (CCTNS) Road Transport, Oxygen Demand (BOD) due to the increased consumption of
Treasuries Computerisation, PDS, Education, Health. dissolved oxygen. It increases the biomass of the
Integrated MMPs under NeGP CSC, e-Biz, e-Courts, phytoplanktons.
e-Procurement, EDI for e-Trade, National e-Governance 42. (c) The laterite is affected by intensive leaching and rich in
services Delivery Gateway, India Portal. iron oxide, aluminium compound and potash, but poor in
36. (b) Pacific ocean basin also known as ‘Pacific Ring of fire’ is an organic matter, nitrogen, phosphate and calcium making it
area where large number of earthquakes and volcanoes unsuitable for cultivation. It is commonly found in Karnataka,
eruptions occur. Tamil Nadu, Kerala, Madhya Pradesh and the hilly areas of
Asom and Odisha.
37. (c) CR formula was formulated to solve political deadlock
between Congress and Muslim League on the matter of 43. (c) Odissi closely follows the tenets laid down by the
independence and partition of Bengal. It proposed many Natyashastra. Facial expressions, body and hand
things, in which it said that plebiscite should be conducted on movements are used to suggest a certain feeling, an emotion
all inhabitants irrespective of caste and religion on basis of adult or the nine- rases. The techniques of movement are built
suffrage. It also recommended that terms of the binding will be around the two basic postures of chock and tribanga. Chowk
applicable only in case of full transfer of power by Britain to is a position imitating a square and is a very masculine
Government of India. stance with weight of body equally balanced. The tribhanga is
a very feminine stance where the body is defected at the neck
38. (a) The Godavari is the largest Peninsular river system. It is and waist and the knee. The Nataraja Buddhist sculptures,
also called the Dakshin Ganga. It originates from Nasik district
tantric images of dancing yoginis bear testimony to this
of Maharashtra and discharge its water into the Bay of Bengal. It
dance from 2nd century BC to 10th century AD.
is navigable only in the deltaic stretch.
44. (d) Saline soils are also known as Usara soils. Saline soils
39. (d) Statement 1 is incorrect, since such a power has been given contain a larger proportion of sodium, potassium and
to the Union Government according to Article 292 of the
magnesium. They occur in arid and semi-arid regions and in
Constitution. The limits to this power though can be specified by
water logged and swampy areas. Their structure ranges from
the Parliament.
sandy to loamy. In the areas of intensive cultivation with
Statement 2 is incorrect because the Constitution does not excessive use of irrigation, especially in areas of green
allow State Governments to borrow from outside India under revolution the dry climatic conditions promotes capillary
any circumstances. Even when an international loan is assigned action, which results in the deposition of salt on the top layer
to the states, it is taken in the name of the Union Government. of the soil.
Statement 3 is also incorrect because even the power of 45. (d) The NDC was established with the main objective to
State Governments to borrow from inside India is secure cooperation of states in the execution of the plan, to
restricted under certain circumstances. When the Union strengthen and mobile the efforts and resources of the nation
Government has any loan outstanding from a state then that in support of plan, to promote common economic policies in
state can make any additional borrowing from elsewhere only all vital spheres, and to ensure balanced and rapid
after the permission of the Union Government. The right answer development of all parts of the country. So, correct option is
is thus (d). (d).
40. (d) The committees perform many functions, the major one of 46. (c) Equality before the law is not an absolute right and certain
which is scrutinising the budgetary proposals during the recess persons like President, foreign diplomats etc have special
of the budget session. Apart from this, the committee also provisions, which can be said to violate equality before the
examines the bills of related ministries and considers the annual law. Thus, statement 1 is incorrect. Statement 2 is also
reports of ministries. Thus, Statement 1 is incorrect. Statement 2 incorrect, as the exceptions provided to the right to equality of
is also incorrect, as the members of the committee may be opportunity in the Constitution do not mention women. The
elected, appointed, nominated by the Speaker of Lok Sabha exception include residence as a condition, backward
and Chairman of Rajya Sabha from among their members. classes etc. Statement 3 is correct and both national as well
as foreign titles are prohibited under the Constitution.
Statement 3 is incorrect, as the report of the committee cannot
Statement 4 is also incorrect as the only exception in the
contain any proposal for cut motions on the expenditures
Constitution to certain rights to equality have been provided
concerned. Statement 4 is also incorrect. Out of the 24
to backward classes of citizens. Classes have been
committees, 8 functions under the Rajya Sabha and 16 under interpreted later to mean castes. Thus, the right answer is (c).
the Lok Sabha. Thus, the right answer is (d).
24 Practice Set 1

47. (c) The origin of the word ‘Bihu’ is said to be from the Sanskrit This process is called rumination. These animals are called
word ‘Vishu’. The Assamese celebrate three types of Bihu in a ruminants. The grass is rich in cellulose, a type of
year. carbohydrate. Many animals including humans, cannot
1. Rongaali Bihu is celebrated in the middle of April, marks the digest cellulose. Ruminants have a large sac-like structure
advent of new year and seeding time. between the small intestine and large intestine. The cellulose
of the food is digested here by the action of certain bacteria
2. Kaati Bihu marks the completion of sowing of ‘paddy’ and
which are not present in humans.
transplantation of the saplings. It also marks the Assamese
New Year. 54. (b) Indian Nationalist Army or Azad Hind Fauj was founded
3. Magh Bihu marks the culmination of the harvesting period. by captain Mohan Singh with the help of Japanese in 1942 in
These festivals are also accompanied by Bihu songs and Singapore. In December 1941, the Japanese defeated the
Bihu dances. The entire Assamese community irrespective British at Malaya and captain Mohan Singh was requested by
of caste, creed and religions background, celebrates the Japanese to form an Indian Army comprising the captured
Bihu festival. Indian soldiers. Subhash Chandra Bose who silently left
Calcutta on 17th January, 1941 and arrived in Germany,
48. (a) Provisions of the Regulating Act, 1773. The Governor of formed on Indian Government in exile. Indians in Germany
Bengal became Governor-General of Bengal. He got a council of
gave him the title of Netaji and the slogan of Jai Hind was
4 members and decisions were taken in majority. The
imitiated him during that time.
Governor-General in Council was authorised to regulate all the civil
and military matters of the presidency of Bengal. Supreme Court 55. (b) People’s participation has a long history in India the
was set up in Calcutta with Elijah Impey as its Chief Justice. Bishnoi community is known for its peaceful co-existence with
49. (d) Mistral is a cold and dry strong wind in Southern France that nature. A Bishnoi woman Amrita Devi showed exemplary
blows down from the North along the lower Rhine river valley courage by hugging a tree and daring king’s men to cut her
toward the Mediterranean sea. Chinook, Foehn and Sirocco first before cutting the tree. The Government of India has
are the warm local winds. Chinook (Snow eaters) is a hot wind recently instituted the Amrita Devi Bishnoi Wildlife Protection
which comes in south-westerly direction to prairies near Award for individuals or communities from rural areas that
California. Fohn is a type of dry warm down slope wind in have shown extraordinary courage and dedication in
Switzerland. Sirocco is a hot dry and dusty wind which protecting wildlife.
originates in the Sahara coast. Through, it may occur in any 56. (c) Union Ministry of Health and Family Welfare on 25th
season of the year but is most frequent in spring. December, 2014 launched Mission Indradhanush to
50. (a) There are three types of mountains- Fold mountains, Block achieve full immunisation coverage for all children by 2020. It
mountains and the Volcanic mountains. The Himalayan aims to cover all those children who are either unvaccinated
mountains and the Alps are young Fold mountains with rugged or are partially vaccinated against seven preventable
relief and high conical peaks. The Aravali range in India is one diseases.
of the oldest Fold mountain systems in the world. The range has 57. (b) Non-cooperation movement filled Indians with
considerably worn down due to the processes of erosion. Block enthusiasm and they started to believe that Independence is
mountains are created when large areas are broken and a matter of time. They started declining Britishers at every
displaced vertically. The uplifted blocks are termed as horsts level. In order to it, they surrendered titles, boycotted courts,
and the lowered blocks are called graben. The Rhine valley and foreign clothes and refuged to pay taxes. They also boycotted
the Vosges mountain in Europe are examples of such mountain government affiliated educational institutions but, they did not
systems. boycotted English education.
51. (d) The Phillips curve shows the relationship between the rate of 58. (a) Gupta Period is considered as the golden age of ancient
inflation and rate of unemployment. It is a historical inverse India. This was mainly in the context that use of gold was
relationship between the rate of unemployment and the rate of maximum and art and culture flourished most widely.
inflation in an economy. Lower unemployment in an economy is
In economic field, many towns of North India show their
correlated with a higher rate of inflation.
downfall in this period. So, in economic context term ‘golden
52. (d) The 91st Amendment Act of 2003 has made the following period’ can’t be justified. Guptas possessed a large quantity
provisions to limit the size of Council of Ministers, to debar of gold and issued the largest number of gold coins, but they
defectors from holding public offices and to strengthen the were not the purest coins. Purest coins were issued by
anti-defection law under Article 75. The total number of minister, Kushans. In this period, art was largely inspired by religions.
including the Prime Minister, in the Central Council of Ministers The typical Hindu Gods like Vishnu, Shiva and others got
shall not exceed 15% of the total strength of the Lok Sabha. But place for the first time in images and architecture.
the anti-defection law doesn’t give any constitutional status to
political parties.
59. (d) The high temperature, high rainfall and annual flood-silts
which provide natural fertiliser are the reasons behind West
53. (d) Grass eating animals quickly swallow the grass and store it Bengal being the largest producer of Jute. Highest
in a separate part of the stomach called ‘rumen’. Here, the food concentration of Jute mills is the result of high Jute
gets partially digested and is called cud. But later the cud production and the seaport location is the reason behind
returns to the mouth in small lumps and the animal chews it. development of Jute industry.
Stage 1 Know the Trend 25

60. (c) Statement 1 is incorrect as the only prohibition on retired 66. (d) The chief instrument through which Lord Dalhousie
Supreme Court Judges is on practicing of law and they are free implemented his policy of annexation was the ‘Doctrine of
to hold other public offices, like e.g. Chairman of National Lapse’. Under this doctrine, when the ruler of a protected
Human Right Commission (NHRC). Statement 2 is correct as state died without a natural heir, his state was not to pass to
the expenses of Supreme Court including the salaries and an adopted heir as sanctioned by the old traditions, but it was
administrative expenses of Supreme Court judges are charged to be annexed to British India, unless the adoption has been
on the Consolidated Fund of India. approved by the British. Most of the Indian states, including
Statement 3 is also correct as it is provided in Article 146 of the Satara in 1848, Jhansi in 1853 and Nagpur in 1854 were
Constitution. Statement 4 is also correct and Parliament can annexed by applying this doctrine.
only extend the jurisdiction of the Supreme Court and cannot 67. (d) The green plants in a terrestrial ecosystem capture about
curtail it. Thus, the right answer is (c).
1% of the energy of sunlight that falls on their leaves and
61. (b) Provincial Autonomy by the Act of 1935, replaced dyarchy. convert it into food energy. When green plants are eaten by
Provinces were granted autonomy and separate legal identity. primary consumers, a great deal of energy is lost as heat to
Provinces were freed from the superintendence, direction of the the environment. Some amount goes into digestion and in
Secretary of State and Governor-General. Provinces, doing work and the rest goes towards growth and
henceforth derived their legal authority directly from the British reproduction. An average of 10% of the food eaten is turned
crown. Provinces were given independent financial powers and into its own body and made available for the next level of
resources. Provincial Governments could borrow money on their consumers. There are generally a greater number of
own security. Hence, Statement 3 is incorrect and all other individuals at the lower trophic levels of an ecosystem, the
statements are correct. greatest number is of the producers.
62. (d) All the statements except 2 are correct. Women tend to have 68. (b) The Association of Southeast Asian Nations, or ASEAN,
a lower mortality rate at every age. In the womb, male fetuses was established on 8th August, 1967 in Bangkok, Thailand,
have a higher mortality rate (babies are conceived in a ratio
with the signing of the ASEAN Declaration (Bangkok
estimated to be from 107 males to 100 females, but the ratio at
Declaration) by the Founding Fathers of ASEAN, namely
birth in the United States is only 105 males to 100 females).
Indonesia, Malaysia, Philippines, Singapore and Thailand.
Among the smallest premature babies (those under 2 pounds
or 900 g) females again have a higher survival rate. At the other At present there are 10 members namely, Brunei Darussalam,
extreme, about 90% individuals aged 110 are female. Cambodia, Indonesia, Laos, Malaysia, Myanmar, Philippines,
Life expectancy used when determining value of life Singapore, Thailand and Vietnam. The motto of ASEAN is
settlement, life insurance policy and development. “One Vision, One Identity, One Community.”
Human Development and Human Development Index a 69. (d) The Union Government on 18th November, 2014
composite statistic of life expectancy, education and income relaunched Kisan Vikas Patra Scheme in New Delhi to
index used to rank countries into four tier human development. encourage the habit of small savings among the citizen.The
Lindy effect is a theory of the life expectancy of non-perishable scheme will provide facility of unlimited investment by way of
things that posits for a certain class of non-perishables, like
purchase of certificate from post office in various
technology, idea , every additional day may imply a longer life
denominations.
expectancy.
Glasgow effect refers to the unexplained poor health and low 70. (c) Scheduled areas and tribal areas are mentioned in
life expectancy of Glaswegians compared to the rest of United Schedules 5 and 6 of the Constitution respectively. While, the
Kingdom and Europe. 5th Schedule deals with scheduled areas and scheduled
63. (b) The country produces about 11% cereals of the world and tribes in all parts of India except Asom, Meghalaya, Tripura
ranks third in production after China and US. India contributes and Mizoram, the 6th Schedule deals only with tribal areas in
21.6% of rice production in the world and ranked second after Asom, Meghalaya, Tripura and Mizoram.
China in 2008-09. India is a leading producer of pulses and This distinction is made not on the basis of forest non-forest
accounts for about one-fifth of the total production of the pulses tribes, but for certain other anthropological reasons. Thus,
in the world. Statement 1 is incorrect. Statement 2 is correct and the 5th
64. (d) Natural gas is obtained along with oil in all the oil fields but Schedule empowers the Governor to exclude any law made
exclusive reserves have been located along the Eastern coasts by Parliament from being applied to a scheduled area.
as well as Tripura, Rajasthan and offshore wells in Gujarat and 71. (b) Gram Nyayalayas in India have been set up under the
Maharashtra. Gram Nyayalayas Act, 2008. Under this act, Gram
65. (d) Xylem The water conducting tissue of vascular plants, Nyayalayas have the powers of both Civil and Criminal courts.
which consists of dead hollow cells (like wood) in the form of These are mobile courts and try to deliver justice at the
fibers and sclereids which transport water and nutrients. doorsteps of the citizens. The act also provides for appeal of
Arteries bring oxygenated blood to the tissue (except decisions of the Gram Nyayalayas to Sessions court and
pulmonary arteries) and veins bring deoxygenated blood back District court. Plea bargaining has been enabled in the Gram
to heart (except pulmonary veins). Nyayalayas and persons can make application for this
Veins have thin walls. Since, blood flow is rapid and at a high purpose to the Gram Nyayalayas.
pressure, the arteries have thick and elastic walls.
26 Practice Set 1

72. (d) With the arrival of Vaishnavism, the popular Radha Krishan 77. (b) Tej Bahadur Sapru was a liberal who broke openly with
theme of Rasleela evolved in Manipuri. The main characters of Congress over non-violent civil disobedience. He opposed
Rasleela are Radha Krishan and Gopis. CDM and salt satyagraha. He participated in First Round Table
The Kirtan form of singing accompanies the dances of Conference and also participated in central and provincial
Manipuri. The masculine aspect of the dance is called legislatures set up by British. He mediated between Gandhi
choloms. Also the male dances perform a martial art named and viceroy Irwin for Gandhi Irwin Pact.
Thang– Ta, to represent the abilities of men to defend 78. (d) Statement 1 is incorrect, because the only qualification
themselves from wild animals. prescribed in the Constitution to be appointed as Finance
73. (c) UNFCCC is an lnternational environment treaty negotiated Commission Chairman is to have experience in public affairs.
at the United Nation Conference on Environmental and Statement 2 is also incorrect because Finance Commission
Development (UNCED) known as Earth Summit held in Rio de recommendations are not binding on the government although
Janerio. CITES is a Convention on International Trade in a tradition has developed, whereby the Union Government
Endangered Species of wild flora and fauna. International accepts all Finance Commission recommendations. Statement
agreement between governments held in Washington. The 3 is also incorrect because the terms of reference of Finance
Convention on Biological Diversity (CDB) is formally known as Commission are not exhaustively listed in the Constitution and
Biodiversity convention. Its objective is to develop national these are provided by the Union Government, while
strategies for the conservation and sustainable use of constituting the Finance Commission through the President.
biological diversity. The convention was opened for signature Statement 4 is the only correct statement.
at the Earth summit in Rio de Janeiro on 5th June, 1992. The
convention on Nuclear safety is a 1994 International Atomic
79. (d) Factors affecting surface air temperature are
Energy Agency (IAEA) treaty that governs safety rules at A. Latitude High latitudes receive lower Sun angles, thus,
nuclear power plants in state parties to the convention. they have less total insolation than low latitudes. Less
insolation means lower temperature
The convention was adopted in Vienna, Australia, at an IAEA
diplomatic conference on 17th June, 1994. B. Altitude On average temperature decreases with
increasing altitude called normal lapse rate.
74. (c) Protectionism is the economic policy of restraining trade
● Thin atmosphere emits less long wave, so the mountain
between states (countries) through methods such as tariffs on
receives less.
imported goods, restrictive quotas and a variety of other
● Normal Lapse Rate recall that air temperature drops with
government regulations designed to allow (according to
proponents) fair competition between imports and goods and increasing altitude.
service produced domestically. This policy contrasts with free C. Cloud Cover
trade, where government barriers to trade are kept to a ● Day Clouds Insolation blocked results in lower maximum
minimum. In recent years, it has become closely aligned with temperatures i.e. it doesn’t get as hot as a clear day.
anti-globalisation.The options 3 and 5 are not protectionist ● Night Clouds Clouds radiate increased long wave

measures. toward Earth; results in higher minimum temperatures i.e.


75. (a) India’s three-stage nuclear power programme was it doesn’t get as cold as a clear night.
● Daily Temperature Range Difference between the
formulated by Dr Homi Bhabha in the 1950s to secure the
country’s long-term energy independence, through the use of maximum and minimum is low in a cloudy situation.
uranium and thorium reserves found in the monazite sands of D. Land/Water Heating Differences Water has certain
coastal regions of South India especially on Kerala coast. The properties as compared to land. It heats and cools more
ultimate focus of the programme is on enabling the thorium slowly than land.
reserves of India to be utilised in meeting the country’s energy
80. (b) No-tillage farming is a way of growing crops from year to
requirements.
year without disturbing the soil through tillage. No-till is an
As of 2012, the first stage consisting of the PHWRs is near agricultural technique, which increases the amount of water
completion of its planned goals, the second stage consisting and organic matter (nutrients) in the soil and decreases
of Fast Breeder Reactors (FBRs)is poised to go into operation erosion. Minimum tillage farming is shallow ploughing only to
within 1 years and the third stage consisting of AHWRs (one the depth needed to ensure quick seed germination and
among several technology options) is slated to begin leaving crop residues.
construction, so that its commissioning can be done by 2020.
81. (c) Statement 1 is incorrect, the hill temple of Lord Ayyappa in
76. (c) Crimean Peninsula is washed by Sea of Azov and Black Shabarimala is situated in Western ghats of Kerala. Statement
Sea. Crimean Peninsula borders with Ukraine in the North and 2 is correct, the temple is open to all devotees irrespective of
separated from Russia by Kerch strait in the East. The Black caste, creed, religion or social status. Statement 3 is also
Sea parts of Crimea provide quick access to the Eastern correct, girls and women between 10 and 50 years of age are
Mediterranean Balkans and Middle East. Sevastopal port is not allowed to visit the temple.
the base of Russian Navy Black Sea fleet.
Stage 1 Know the Trend 27

82. (d) CEL holds a unique position among the family of public Union Government. The union on the other hand can
sector enterprises in electronics, with its emphasis on delegate functions on to a state even without the consent of
indigenous technology inducted both from its in-house the concerned state by making such a provision in a law
development and from the national laboratories, for its passed by the Union Parliament. Thus, Statement 2 is
production programme in diverse high-technology areas of correct.
national importance. Statement 3 is also correct since the President can entrust
any union function on a State Government without any
83. (c) Out of various music forms of carnatic music, Gitam is the
sanction from the Union Parliament by taking the consent
simplest type of composition and is taught to the beginners of
from the State Government as provided in Article 258(1).
music. The Gitam is very simple in construction, with an easy
Thus, the right answer is (b).
and melodious flow of music.
91. (c) Statement 1 is incorrect as the Constitution only provides
A notable feature of the Gitam is the existence of Gitalankara
for who is a citizen at the commencement of the Constitution
phrases like a iya, a iyam va iya etc called Matrika Padas,
and leaves other matters relating to citizenship to be
reminiscent of similar syllables occurring in sama gana.
determined by Parliament by law. Statement 2 is also
84. (b) Eutrophication is the natural aging of a lake by biological incorrect as renunciation of citizenship by Indian citizens can
enrichment of its water. In a young lake the water is cold and only be done, after they have acquired the citizenship of
clear, supporting little life. With time, streams draining into the another country. Statement 3 is correct, as such a provision
lake introduce nutrients such as nitrogen and phosphorus has been provided for in the Citizenship Act of 1955. Some
encourage the growth of aquatic organisms. As the lake’s other reasons of taking away of citizenship by the Central
fertility increases, plant and animal life burgeons, and organic Government can be disloyalty to the Constitution, reason
remains begin to be deposited on the lake bottom. during a war etc. Statement 4 is incorrect, as questions
relating to determination of citizenship have been completely
85. (c) Gross primary productivity is the total amount of production
left to the Central Government, under the Citizenship Act of
of chemical energy in organic compounds by primary
1955. Thus, the right answer is (c).
producers i.e. autotrophs. Certain portion of this biomass is
used by the producer for its own survival and only 10% energy is 92. (a) Basel convention does not talk about the nuclear waste.
transferred to the next trophic level. Eutrophication is the The agreement on Transboundary Movements of hazardous
process of excessive nutrition load in an aquatic ecosystem. wastes and their disposal, is known as Basel Convention.
Primary succession is one of two types of biological and 93. (b) Kajri is a folk song sung by women from Uttar Pradesh
ecological succession of plant life, occurring in an environment during rainy seasons. On the 3rd day in the 2nd half of the
in which new substrate devoid of vegetation and usually lacking bhadra, women sing kajri songs all through the night.
soil.
Chhakri is a group song of Kashmir. It is sung to the
86. (d) All the given statements are correct. Solar water heater is accompaniment of the ‘noot’ (Earthen pot) rababs, sarangi
based on the principles of surface absorption and greenhouse and tumbaknari (an Earthen pot with high neck).
effect. The copper pipes are used for heating and are in the
spiral shape. The pipes are painted with black colour to absorb Mando is the finest of goan song and is a slow verse and
maximum heat. refrain composition dealing with love, tragedy and social
injustice. Pankhida is sung by the peasants of Rajasthan
87. (b) Saudi Arabia has the largest oil reserves in the world. While while doing work in the fields, the peasants sing and speak
USA has the largest coal reserves. Russia has the largest while playing algoza and manjira.
natural gas reserves.
94. (c) Increasing interest rates and Introduction of Inflation-
88. (d) Hormones are chemical messengers that are secreted Indexed National Saving Securities (IINSSs) for retail
directly into the blood, which carries them to organs and tissues investors are the steps taken by RBI.
of the body to exert their functions. Hormones are classifed into
RBI on consultation with the government plans to launch a 10
all the above three groups Polypeptide, Steroid and Amino-acid
year savings instrument ‘Inflation-lndexed National Saving
derivative. Hormones are classified based on their chemical
Securities (IINSSs) for retail investors’ that will offer
structures. Peptide hormones are chain of amino-acid. Steroid
inflation-linked returns to small investors as an alternative to
hormones are lipids (fat like molecules) whose structures are
investing in gold.
derived from cholesterol.
IINSS will be linked to the new (combined) Consumer Price
89. (d) Strachey Commission is related to famine, Hartog Index (CPI). The interest on these securities would comprise
Committee is related to education and Fraser Commission is a fixed rate plus inflation. Interest would be compounded
related to police reforms. Strachey Commission was set up in half-yearly and paid cumulatively at redemption. These
1877. Hartog Committee was set up in 1929. Fraser securities will be distributed through banks to reach out to the
Commission was set up in 1902. masses (Eligible investors individuals, Hindu undivided
90. (b) Statement 1 is incorrect, according to Article 258A, the families, trusts and charitable institutions). Increasing or
Governor of a state may give to the Union Government any of decreasing taxes, duties permitting FDI in retail are the steps
the administrative functions of that state with the consent of the taken by the Finance Ministry (not RBI)to curb inflation.
28 Practice Set 1

95. (d) Spectrum management is the process of regulating the These gases combine with few other gases, which leads to a
use of radio frequencies to promote efficient use and gain a reduction of the ozone layer that protects the Earth from the
net social benefit. The term radio spectrum typically refers to harmful ultraviolet rays of the Sun. Carbon dioxide (CO 2 ), a
the full frequency range from 3 KHz to 300 GHz that may be greenhouse gas, emitted from combustion. Suspended
used for wireless communication. Particulate Matter (SPM), include smoke, dust and vapour that
can remain suspended for extended periods and can be
96. (d) The Zonal Councils are the statutory (and not introduced in the environment as primary as well as secondary
constitutional) bodies and are established by States
pollutants. Toxic metals, such as lead, cadmium and copper can
Reorganisation Act of 1956. All of the statements given are
enter in the environment through petrol, hair dye products,
correct, some of the other objectives are to help in arresting
paints, batteries etc. Volatile Organic Compounds (VOC), such
the growth of acute state-consciousness, regionalism,
as hydrocarbon fuel vapours and solvents, ammonia (NH3 )
linguism and particularistic trends.
emitted from agricultural processes. Odours from garbage,
To help in removing the after-effects of separation in some sewage and industrial processes. Radioactive pollutants
cases so that the process of reorganisation, integration and produced by nuclear explosions and war explosives and natural
economic advancement may synchronise. To cooperate processes such as radon.
with each other in the successful and speedy execution of
Secondary Pollutants Particulate matter formed from gaseous
major development projects. So, correct option is (d).
primary pollutants and compounds in photochemical smog,
97. (b) The narrowly utilitarian arguements for conserving such as nitrogen dioxide. Ground level ozone (O 3 ) formed from
biodiversity are obvious; humans derive countless direct NO and VOCs, Peroxyacetyl Nitrate (PAN) similarly formed from
economic benefits from nature — food, firewood, fibre NO 2 and VOCs.
construction material, industrial products and products of
99. (c) For giving assistance to nations to boost. Exports, loans from
medicinal importance with increasing resources put into
any regional development bank can be taken like the Asian
bioprospecting nations endowed with rich biodiversity can
development bank. The IMF does not provide such a service.
expect to reap enormous benefits.
The world bank can provide a loan, but the conditions attached
98. (c) Primary and Secondary Pollutant A primary pollutant is with the loan do not qualify for the case (c) in the question. It is
an air pollutant emitted directly from a source. A secondary for general development work.
pollutant is not directly emitted as such, but forms when
100. (d) The time bound goals in the 12th Five Year Plan for India’s
other pollutants (primary pollutants) react in the atmosphere.
population are as follow
Primary Pollutants Sulphur dioxide (SO 2 ) produced from ● Reducing Maternal Mortality Rate (MMR) to 1 per 1000
burning of coal mainly in thermal power plants. It causes livebirths.
smog and acid rain. Nitrogen Oxide (NO 2 ) causes smog and ● Reducing Infant Mortality Rate (IMR) to 25 per 1000 livebirths.
acid rain. It is produced from burning fuels including petrol,
● Reducing Child Sex Ratio (0-6 years) to 950 by the end of the
diesel and coal.
plan.
Carbon monoxide (CO) is a product by incomplete ● Reducing Total Fertility Rate (TFR) to 2.1 by the end of the
combustion of fuel such as natural gas, coal or wood. plan.
Chlorofluorocarbons (CFCs) released mainly from ● Reducing under-nutrition among children aged 0-3 years to
refrigeration. half of the NFHS-3 level by the end of the plan.
UPSC CIVIL SERVICES (PRE) EXAM

General Studies
Paper 1
Practice Set 2
Time : 2 hrs MM : 200

Instructions
1. This set contains 100 questions. Each question comprises four responses (answers). You will select the response which
you want to mark on the Answer Sheet. In case, if you feel that the correct response is more than one, then mark the
response, which you consider the best. In any case, choose only one response for each questions.
2. All questions carry equal marks.
3. Penalty for Wrong Answer
(i) There are four alternatives to answer every question. If a question is marked wrong, one-third of the marks
assigned to that question will be deducted as penalty.
(ii) If a candidate gives more than one answer, it will be treated as wrong answer even, if one of the given answers
happens to be correct and there will be same penalty as to that question, if it has a penalty.
(iii) If a question is left blank i.e. no answer is given by the cnadidate, there wil be no penalty for that question.

1. The process of converting unconsolidated sediments into sedimentary rocks requires which of
the following?
1. Compaction 2. Recrystallisation 3. Cementation 4. Weathering
KNOW the TREND
Select the correct answer using the codes given below.
(a) 1 and 2 (b) 2 and 3
(c) 1, 2 and 3 (d) All of these

2. Consider the following statements about the Atal Pension Yojana (APY).
1. The Government of India has introduced the APY with effect from 1st June, 2015 on creating a
universal social security system for all Indians, especially the poor, the underprivileged and the
workers in the unorganised sector.
2. It is administered by the PFRDA under the administrative and institutional architecture of the
National Pension System (NPS).
3. The minimum age of joining APY is 21 years and maximum age is 40 years who have a saving bank
Stage 1

account.
Which of the statement(s) given above is/are incorrect?
(a) 1 and 2 (b) 2 and 3 (c) Only 3 (d) None of these
30 Practice Set 2

3. Consider the following statements about Biomedical Technologies.


1. Balloon pump technique is used, when the pumping action of the cardiac ventricle temporarily
become too poor to maintain effective profusion of tissues and organs with blood and need support.
2. Electrocardiography is a valuable non-intensive tool for imaging heart and surrounding structures.
3. Cryopreservation technique involves the use of tissues taken from humans and other animals for the
purpose of reconstructive surgery.
Which of the statements given above are correct?
(a) 1 and 2 (b) 1 and 3 (c) 2 and 3 (d) All of these

4. Consider the following statements with regard to vertical variation of temperature in the
oceanic water.
1. In the tropical areas, there are three distinct layers of water according to temperature characteristics.
2. In the polar region, there is a uniform layer of water.
3. Thermocline layer is present in mid-latitude oceanic water during winters.
Which of the statements given above are correct?
(a) 1 and 2 (b) 2 and 3 (c) 1 and 3 (d) All of these

5. Which one of the following statements does not accurately describes the recommendations
made by various committees on decentralisation?
(a) BR Mehta Committee recommended the three-tier Panchayati Raj set up.
(b) Ashok Mehta Committee recommended allowing of political parties to contest Panchayat elections.
(c) GVK Rao Committee called for Chief Executive Officer of the Zila Parishad to be an elected
member.
(d) LM Singhvi Committee called for giving of constitutional status to Panchayat bodies.

6. Consider the following statements with regard to troposphere.


1. Troposphere is higher at the poles than at the equator.
2. All vital atmospheric process leading to various weather and climatic conditions take place in this
layer.
3. The increase in temperature with increase in height in this layer is termed as temperature inversion.
Which of the statements given above are correct?
(a) 1 and 2 (b) 2 and 3 (c) 1 and 3 (d) All of these

7. How does National Biodiversity Authority (NBA) help in protecting the Indian agriculture?
1. NBA checks the bio-piracy and protects the indigenous and traditional genetic resources.
2. NBA directly monitors and supervises the scientific research on genetic modification of crop plants.
3. Applications for Intellectual Property Rights (IPRs) related to genetic/biological resources cannot be
made without the approval of NBA.
Which of the statement(s) given above is/are correct?
(a) Only 1 (b) 2 and 3 (c) 1 and 3 (d) All of these

8. Consider the following about the National Green Tribunal (NGT).


1. It has been established on 18th October, 2010 under the National Green Tribunal Act 2010 and under
India’s constitutional provision of Article 21.
2. It’s objective is to provide for effective and expeditions disposal of cases relating to environmental
protection and conservation of forests and other natural resources including enforcement of any
legal right relating to environment.
3. Relating to environment, it gives relief and compensation for damages to persons and property and
its first chairperson was Justice Lokeshwar Singh Panta.
4. It shall not be bound by the procedure laid down under the code of Civil Procedure, 1908 but shall be
guided by principles of natural justice.
Which of the above statement(s) is/are incorrect?
(a) Only 1 (b) Only 4 (c) 3 and 4 (d) None of these
Stage 1 Know the Trend 31

9. Consider the following statements.


1. Bioventing involves supplying air and nutrients through wells to contaminated soil to stimulate the
indigenous bacteria.
2. Biosparging involves the injection of air under pressure below the water table to increase
groundwater oxygen concentrations and enhances the rate of biological degradation of
contaminants.
3. Bioaugmentation involves the extraction of specific metals from their ores through the use of living
organisms.
Which of the statements given above are correct?
(a) 1 and 3 (b) 2 and 3 (c) 1 and 2 (d) All of these

10. Vijayanagara society was very well developed and women were honoured in the society. Some
statements regarding socio-economic conditions in Vijayanagara empire are mentioned below.
1. Widow’s life was pitiable and they were not allowed to remarry.
2. Sati custom was prevalent in Vijayanagara. The women performed sati within 2 or 3 months of their
husband’s death.
3. The custom of polyandry was common and the special characteristic was that wife was elder to the
husband.
4. The customs of divorce and remarriages were prevalent in the Vijayanagara society.
Which of the statements given above are correct?
(a) 2, 3 and 4 (b) 1, 3 and 4 (c) 2 and 3 (d) None of these

11. Consider the following statements regarding to boycott of Simon commission.


1. It united different groups and parties in the country with Muslim League and Hindu Mahasabha
supporting Congress.
2. As a gesture of solidarity with the nationalists, the Muslim League even accepted the principle of
Joint electorates, provided seats were reserved for the Muslims.
3. Mahatma Gandhi was not involved in anti-Simon commission movement.
4. Nehru report of 1928 was an alternative scheme of constitutional reforms to that of Simon
commission.
Which of the statements given above are correct?
(a) 1 and 4 (b) 1, 3 and 4 (c) 3 and 4 (d) All of these

12. Consider the following statements regarding Kathak.


1. It is originated from Kerala.
2. The three major schools of Kathak are Jaipur Gharana, Lucknow Gharana and Benaras Gharana.
3. Birju Maharaj, Malabika Mitra and Sitara Devi are noted dancers of Kathak.
Which of the statements given above are correct?
(a) 1 and 2 (b) 2 and 3
(c) 1 and 3 (d) All of these

13. Consider the following statements regarding the appellate jurisdiction of the Supreme Court.
1. Appeals by special leave under Article 136 can lie even against those cases, which are expressly
excluded from Supreme Court jurisdiction by the Constitution.
2. Appeals in cases of interpretation of the Constitution lie automatically with the Supreme Court from
the High Courts.
3. Appeal as a matter of right is available if a High Court reverses the acquittal of person and sentences
him to death or even life imprisonment.
Which of the statement(s) given above is/are correct?
(a) Only 2 (b) 2 and 3
(c) 1 and 3 (d) Only 1
32 Practice Set 2

14. Which of the following statements regarding the administration of Union Territories are correct?
1. Legislation in Union Territories is done through regulations made by the President.
2. Union Territories (except Delhi) cannot have a High Court of their own.
3. Even after, the Union Territories get their own assembly, Parliament can still make laws on items in
the State List for such territories.
Select the correct answer using the codes given below.
(a) Only 1 (b) 1 and 2
(c) Only 3 (d) None of these

15. Consider the following statements with reference to conditions which aggravated the misery of
farmers in Deccan regions in 1860’s.
1. The end of the American Civil War in 1864 brought about an acute increase in cotton exports and an
improvement in prices.
2. In Maharashtra, as part of the Ryotwari system, land revenue was settled directly with the peasants,
who were also recognised as the owner of their land.
3. Agriculturists Relief Act, 1879 was passed to make the plight of farmers more pathetic.
Which of the statement(s) given above is/are correct?
(a) Only 2 (b) Only 1
(c) Only 3 (d) None of these

16. Which of the following weather conditions are associated with Cold Front?
1. The cold airmass is advancing and under cutting the warm airmass.
2. The warm airmass is advancing and overriding the cold airmass.
3. Rapid ascent of warm moist air at the front causes rapid cooling and condensation.
4. Development of nimbus clouds.
Select the correct answer using the codes given below.
(a) 1 and 3 (b) 2 and 4
(c) 1, 3 and 4 (d) 2, 3 and 4

17. Consider the following statements.


1. The Earth’s surface receives most of its energy from sun in long wavelengths.
2. When Earth radiates energy to the atmosphere it is in the shortwave form.
Which of the statement(s) given above is/are correct?
(a) Only 1 (b) Only 2
(c) Both 1 and 2 (d) Neither 1 nor 2

18. Consider the following statements about the Digital India Programme.
1. It was launched on 1st July, 2015 with the vision to transform the country into a digitally empowered
society and knowledge economy.
2. Its three key areas are digital infrastructure, governance and services on demand and digital
empowerment of citizens.
3. It was envisaged by the Department of Electronics and Information Technology (DeitY) and
implemented by the entire government and coordinated by DeitY.
4. The President is the Chairman of Monitoring Committee on Digital India.
5. Kerala is the first digital state in the country.
Which of the statements given above are correct?
(a) 1, 2 and 5 (b) 2, 3, 4 and 5
(c) 1, 2, 3 and 5 (d) All of these
Stage 1 Know the Trend 33

19. Consider the following statements with regard to Eastern and Western coastal plains of India.
1. The former is emergent whereas latter is submergent in nature.
2. The former provides better natural conditions for the development of the ports than the latter.
3. The rivers flowing through the former form delta whereas those flowing through the latter have
estuaries mouths.
Which of the statements given above are correct?
(a) 1 and 2 (b) 2 and 3
(c) 1 and 3 (d) All of these

20. Consider the following statements about Savanna region.


1. Tropical Savanna biome characterised by long dry warm season and short rainy season, water is the
limiting factor.
2. Trees 6-12 m height strongly rooted with flattened crown.
3. Trees in Savanna catch fire and species diversity is high in tropical Savanna.
Which of the statement(s) given above is/are correct?
(a) Only 1
(b) 1 and 2
(c) 2 and 3
(d) All of these

21. Which of the following conditions occur during an EI-Nino?


1. Trade winds are weakened in the Eastern Pacific.
2. Upwelling across the coast of Peru is suppressed and warm surface water remains across the coast.
3. Air pressure rises over large areas of the South-East Pacific, while pressure drops over Western
Pacific.
4. Release of H2S in the water off the coast of Peru.
Select the correct answer using the codes given below.
(a) 1 and 2 (b) 2 and 3
(c) 1, 2 and 4 (d) All of these

22. World Bank released a report titled World development Report 2016; Digital Dividends;
Consider the following statements.
1. The report explored the impact of the internet, mobile phones, and related technologies on economic
development across the world.
2. By overcoming information barriers, augmenting factors, and transforming products, digital
technologies can make development more inclusive, less effective.
Which of the statement(s) given above is/are correct?
(a) Only 1 (b) Only 2
(c) Both 1 and 2 (d) Neither 1 nor 2

23. Consider the following statements comparing National Emergency under Article 352 and
President’s rule in a state under Article 356.
1. Both can be declared upon the failure of constitutional machinery.
2. Certain Fundamental Rights can be restricted under both.
3. National emergency can be declared even before occurrence of the actual events, while State
emergency can be declared only after the occurrence of the events.
Which of the statement(s) given above is/are correct?
(a) Only 1 (b) 1 and 2
(c) 2 and 3 (d) Only 3
34 Practice Set 2

24. Consider the following statements regarding the various types of motions in Parliament.
1. Substantive motion are proposal dealing with impeachment of the President.
2. Substitute motions are taken up after the original motion has failed to pass.
3. Subsidiary motions depend on other motions and have no independent existence.
4. A motion to refer a bill to a Parliamentary Committee is a subsidiary motion.
Which of the statements given above are correct?
(a) 1 and 2 (b) 2 and 3
(c) 1, 3 and 4 (d) All of these

25. Over a longer duration, which among the following factors cause the artificial satellites to
experience a reduction in the altitude (orbital decay)?
1. Atmosphere of Earth 2. Tides
3. Earth’s gravitation
Select the correct answer using the codes given below.
(a) Only 1 (b) 1 and 2
(c) 2 and 3 (d) All of these
26. Consider the following statement in the context of Ballistic missile.
1. A Ballistic missile is a missile that follows a sub-orbital ballistic flight path with the delivering one or
more war heads to a predetermined target.
2. A Ballistic missile trajectory consist of three parts the powered flight portion, the free-flight portion
and re-entry phase.
3. Ballistic missiles cannot be launched from fixed sites or mobile launchers.
Which of the statements given above are correct?
(a) 1 and 2 (b) 2 and 3 (c) 1 and 3 (d) All of these
27. Consider the following statements.
1. Phosgene is a poisonous compound formed from chloroform in the presence of UV-rays and O2 .
2. It is used for the synthesis of precursor for the poly-urethane polymer.
Which of the statement(s) given above is/are correct?
(a) Only 1 (b) Only 2 (c) Both 1 and 2 (d) Neither 1 nor 2
28. Megasthenes states that there was no slavery in India. This statement must have been made
because
(a) there was absence of slavery in India
(b) slavery was not prevalent in India in the extreme form as he had seen in Greece
(c) he moved only with the king and elites in the capital
(d) he wrote this on hear-say
29. Which of the following statements about sea breeze and land breeze is/are correct?
1. During the day, the warm air from the land moves towards the sea which is called the land breeze.
2. During the night, cool air from the sea moves towards the land which is called the sea breeze.
Select the correct answer using the codes given below.
(a) Only 1 (b) Only 2 (c) Both 1 and 2 (d) Neither 1 nor 2
30. Consider the following statements with reference to the Communist Party of India (CPI).
1. Communist Party of India was formed under the auspices of the Communist International in 1920.
2. CPI was formally formed in 1925 at the Kanpur Party conference.
3. The Sino-India War of 1962 lead to the split of CPI.
Which of the statement(s) given above is/are correct?
(a) 1 and 2 (b) 2 and 3 (c) Only 3 (d) l and 3
Stage 1 Know the Trend 35

31. Consider the following statements about National Biotechnology Development Strategy (NBDS).
1. The NBDS 2015-2020 was unveiled by the Government of India on 20th December, 2015 with aim to
establish India as a world-class bio-manufacturing hub.
2. It intends to launch a major mission under the Make in India Programme for the creation of new
bio-tech products.
3. It focuses to build the human capital by creating a Life Sciences and Biotechnology Education
Council (LSBEC).
Which of the statement(s) given above is/are incorrect?
(a) Only 2 (b) Only 3
(c) 2 and 3 (d) None of these
32. Consider the following statements.
1. Calcitonin regulates the metabolism of calcium.
2. Oxytocin stimulates contraction of uterine muscles during birth.
3. Grave’s disease is caused by malfunctioning of adrenal gland.
4. ADH stimulates absorption of water and increase the urine production.
Which of the statements given above are correct?
(a) 1 and 3 (b) 1 and 2
(c) 2 and 4 (d) 1, 2 and 3
33. Consider the following statements.
1. Natural uranium consists of 100% U-238.
2. The fission process by which heat energy is released in a nuclear reactor, takes place mainly in
U-235.
3. Nuclear power plants require fuel with U-235 enriched upto a level of 10%.
Which of the statement(s) given above is/are correct?
(a) 1 and 2 (b) Only 2
(c) 2 and 3 (d) All of these
34. Consider the following statements regarding the alternative methods of dispute resolution.
1. In dispute resolution, mediation is followed by conciliation which is then followed by arbitration.
2. Awards given under arbitration are binding in nature.
3. Mediation and conciliation can be either court driven or community driven.
Which of the statement(s) given above is/are correct?
(a) 1 and 2 (b) Only 2 (c) 2 and 3 (d) Only 3
35. Which of the following metallic minerals are found in Himalayas?
1. Cobalt 2. Nickel 3. Zinc 4. Copper
5. Antimony 6. Bismuth
Select the correct answer using the codes given below.
(a) 2, 3 and 5 (b) 1, 2, 4 and 6
(c) 1, 3, 4 and 5 (d) All of these
36. Which of the following are not included in the adaptation process of climate change?
1. Changes in social and environmental processes.
2. Perceptions of climate risk.
3. Practices and functions to reduce risk.
4. Exploration of new opportunities to cope up with the changed environment.
Select the correct answer using the codes given below.
(a) 1, 2 and 3 (b) 2, 3 and 4
(c) 1, 3 and 4 (d) All of these
36 Practice Set 2

37. Consider the following statements about the Northern plains of India.
1. The alluvial plains can be divided into the Khadar and Bhangar categories.
2. Bhangar is a narrow belt ranging between 8-10 km parallel to the Shiwalik foothills at the break-up of
the slope.
3. The Tarai belts are characterised by marshy and swampy landmass.
Which of the statements given above are correct?
(a) 1 and 2 (b) 2 and 3
(c) 1 and 3 (d) All of these
38. Consider the following statements in the content of BLU-ray Disc (BD).
1. Blu-ray Disc (BD) is an optical disc storage medium designed to supersede the DVD format.
2. The major application of Blu-ray Disc is as a medium for video material such as feature films.
3. Blu-ray Disc is not associated with a set of multimedia formats.
4. These formats allow for the video and audio to be stored with greater definition than on DVD.
Which of the statements given above are correct?
(a) 1, 3 and 4 (b) 2 and 3
(c) 3 and 4 (d) 1, 2 and 4
39. With reference to river capture, consider the following statements.
1. It is a geomorphological phenomenon occurring when a stream or river drainage system or
watershed is diverted from its own bed and flows instead down the bed of a neighbouring stream.
2. It is also called as river beheading.
3. Apart from tectonic Earth movements, erosion also results in river capture.
Which of the statements given above are correct?
(a) 1 and 2 (b) 2 and 3
(c) 1 and 3 (d) All of these

40. Consider the following statements.


1. Arunachal Pradesh has the maximum bamboo bearing area followed by Madhya Pradesh.
2. The forest cover in the tribal dominated areas has an increasing trend.
Which of the statement(s) given above is/are correct?
(a) Only 1 (b) Only 2
(c) Both 1 and 2 (d) Neither 1 nor 2

41. Count de Lally, the French commander failed against the English in India because
1. the home authorities in France did not recognise the importance of securing political power in India.
2. the French were inferior to the English in naval strength.
3. there was discord between the French commanders of land and sea forces.
4. the English heavily bribed some important French officers.
Which of the statements given above are correct?
(a) 1, 2 and 3 (b) 2 and 3
(c) 1 and 4 (d) All of these

42. Consider the following statements regarding Tribunals in India.


1. Under Articles 323A and 323B, both the centre and the states are empowered to form Tribunals.
2. Awards of the Tribunals can be challenged in the Supreme Court only.
3. The Central Administrative Tribunal has original jurisdiction in matters of recruitment and service
matters of all public servants covered under it.
4. Tribunals can also be created on matters of foodstuffs and rent and tenancy rights.
Which of the statements given above are correct?
(a) 1, 2 and 3 (b) 1 and 2 (c) 3 and 4 (d) 2 and 4
Stage 1 Know the Trend 37

43. Which one of the following statements is not correct about Indian Depository Receipts (IDRs)?
(a) An IDRs is a depository receipt denominated in Indian rupees issued by a domestic depository in
India
(b) IDRs are similar to equity shares. IDRs holders have the same rights as shareholders
(c) Both ‘a’ and ‘b’
(d) None of the above

44. Which one of the following is not a feature of Limited Liability Partnership (LLP) firm?
(a) Partners should be less than 20
(b) Partnership and management need not be separate
(c) Internal governance may be decided by mutual agreement among partners
(d) It is corporate body with perpetual succession

45. Which of the following does/do come under India’s Hydrocarbon Vision 2025?
1. To assure energy security through self-reliance by increasing the indigenous production and
investment in equity oil abroad.
2. To enhance quality of life by progressively improving product standards to ensure a cleaner and
greener India.
3. To ensure oil security for the country keeping in view strategic and defense consideration.
Select the correct answer using the codes given below.
(a) 1 and 3 (b) 2 and 3
(c) Only 3 (d) All of these

46. The term development as distinct from growth can be associated with
1. increase in the proportion of population living in urban areas.
2. decline in the proportion of GDP generated in agriculture.
3. increase in the proportion of GDP generated in industry and service sector.
4. improvement in quality of life of the people.
Select the correct answer using the codes given below.
(a) 1, 2 and 3 (b) 2 and 3
(c) 2, 3 and 4 (d) All of these

47. Consider the following statements related to Traditional Knowledge Digital Library.
1. This programme is jointly implemented by Council of Scientific and Industrial Research (CSIR) and
Ministry of Human Resource and Development.
2. It will convert the ancient knowledge into unicode structure, knowledge based interpretation.
3. It will compile in a single database.
4. It will translate the information to Sanskrit, Urdu and Punjabi for easy reference of the examiners.
Which of the statements given above are correct?
(a) 1, 2 and 3 (b) 1, 2 and 4 (c) 2, 3 and 4 (d) 2 and 3

48. Global Warming Potential (GWP) compares the amount of heat trapped by a certain mass of a gas
to the amount of heat trapped by a similar mass of carbon dioxide, calculated over a specific time
interval, commonly 20, 100 or 500 years. The GWP depends on which of the following factors?
1. The absorption of infrared radiation by a given species.
2. The spectral location of its absorbing wavelengths.
3. The atmospheric lifetime of the species.
Select the correct answer using the codes given below.
(a) Only 1 (b) Only 3
(c) 1 and 2 (d) All of these
38 Practice Set 2

49. Consider the following statements related to Basel convention.


1. It aims to set global norms for banking sector for their monetary and financial stability.
2. It aims to control the transboundary movement of hazardous waste and their disposal.
Which of the statement(s) given above is/are correct?
(a) Only 1 (b) Only 2
(c) Both 1 and 2 (d) Neither 1 nor 2

50. Which of the following statement(s) is/are incorrect about the Kelkar Committee on PPP Model
2015?
(a) It defines PPP in infrastructure as the provision of a public asset and service by a private
partner for a specified period on the basis of market determined revenue streams.
(b) It recommended amendment in the Prevention of Corruption Act, 1988 which does not
distinguish between genuine errors in decision-making and acts of corruption.
(c) An Infrastructure PPP Adjudication Tribunal (IPAT) chaired by a judicial member with a
technical and financial member will have the power to pass interim orders in relation to the
project or any of the contracts in relation to the project.
(d) None of the above

51. Consider the following statements about useful Algae.


1. Chlorella and Synechococcus are useful for astronauts as food.
2. Chlorellin is obtained from Chlorella, Cladophora and Lyngbya possess antiviral properties.
3. Agar-agar is obtained from certain red algae such as Gelidium, Gracilaria.
Which of the statements given above are correct?
(a) 2 and 3 (b) 1 and 2
(c) 1 and 3 (d) All of these

52. Consider the following statements regarding the extent of biodiversity.


1. Gamma diversity represents the geographic scale species diversity.
2. Alpha diversity compares the diversity between ecosystems.
3. Beta diversity represents the species richness in a particular ecosystem.
Which of the statement(s) given above is/are correct?
(a) Only 1 (b) 2 and 3 (c) 1 and 3 (d) 1 and 2
53. Which of the following statements in relation to District Planning Committees (DPCs) is
incorrect?
1. DPCs are set up under the provisions of the 73rd Amendment Act.
2. Setting up of DPCs is mandatory for State Governments.
3. DPCs are to take up planning of both urban and rural areas, even when the urban area has a
Municipal Planning Committee.
4. Four-fifths of the members of DPCs are elected from members of both Panchayats and
Municipalities.
Select the correct answer using the codes given below.
(a) Only 1 (b) 2, 3 and 4 (c) 1, 2 and 3 (d) None of these
54. Consider the following statements with respect to India’s standing in negotiation at WTO.
1. India is against Overall Trade Distorting Domestic Subsidies (OTDS) of US, EU in agriculture.
2. India supports the inclusion of intellectual property as a criteria for protectionist measures.
3. India supports the establishment of clear linkages between the TRIPS agreement and the Convention
on Biodiversity, by incorporating specific disclosure norms for patent applications.
Which of the statements given above are correct?
(a) 2 and 3 (b) 1 and 3 (c) 1 and 2 (d) All of these
Stage 1 Know the Trend 39

55. Which of the following phenomena occur during the adiabatic heating of gas?
1. Heat input from the surrounding 2. Change in entropy of the gas
3. Compression of the gas
Select the correct answer using the codes given below.
(a) 1 and 2 (b) 1 and 3
(c) Only 2 (d) Only 3

56. Which of the following statements regarding the removal of Supreme Court Judges are incorrect?
1. Motion for removal of a Supreme Court Judge has to be signed by at least 50 members in Lok Sabha
and 25 members in Rajya Sabha.
2. Motion for removal of a judge can be refused to be admitted by the speaker even if signed by the
requisite support.
3. No Supreme Court judge has been impeached so far.
4. Supreme Court judges can be removed only on grounds of proved misbehaviour or incapacity.
5. Charges against the judge are investigated by a committee of one House of Parliament and supported
by a special majority in the other.
Select the correct answer using the codes given below.
(a) 1, 2 and 3 (b) 1 and 5 (c) 2, 3 and 5 (d) 3, 4 and 5

57. Consider the following statements regarding recently structured NITI Aayog, replacing planning
commission.
1. It will provide a critical directional and strategic input into the development process.
2. It will emerge as a ‘think-tank’ that will provide governments at the central and state levels with
relevant strategic and technical advice across the spectrum of key elements of policy.
3. It will incorporate the significant geo-economic and geo-political strength of the non-resident Indian
community.
Which of the statement(s) given above is/are correct?
(a) Only 1 (b) 1 and 2 (c) Only 2 (d) All of these
58. Consider the following statements.
1. The carrying capacity of an ecosystem remains constant with time and the external environment has
very less impact upon it.
2. The population density in an ecosystem is maximum at the verse of limiting factor.
3. Optimum population density is the condition, where individuals in the population will have an
adequate supply of all essentials for existence.
Which of the statement(s) given above is/are correct?
(a) Only 3 (b) 1 and 2 (c) 2 and 3 (d) All of these

59. Tritium, an isotope of hydrogen is radioactive in nature and finds its use in
1. compasses 2. watch faces 3. glowing gun sights
4. porcelain teeth 5. smoke detector
Select the correct answer using the codes given below.
(a) 1, 2 and 3 (b) 2, 3 and 4
(c) 3, 4 and 5 (d) 2, 4 and 5

60. Consider the following statements regarding the Wetlands.


1. Wetlands are lands that are transitional between terrestrial and aquatic ecosystems.
2. Though the Kolleru Lake provides habitat for a number of resident and migratory birds, including the
vulnerable Grey Pelican yet it is not covered within the Ramsar convention.
3. Chilika lake is a brackish lake, placed on the Montreux Record in 1993 due to siltation, which was
choking the mouth of the lake.
Which of the statements given above are correct?
(a) 1 and 3 (b) 1 and 2 (c) All of these (d) None of these
40 Practice Set 2

61. Which of the following is not a true comparison of the ordinance making powers of the President
of India and the Governor of a State?
1. Both, the President and the Governor do not require any instruction for promulgating ordinances
within the competence of the respective legislatures.
2. Both of them cannot make ordinances on matters, which would require Money Bills.
3. Ordinances can be used not just to create new laws, but also to repeal or modify existing laws.
Select the correct answer using the codes given below.
(a) Only 1 (b) 1 and 2
(c) Only 2 (d) All of these

62. Consider the following statements regarding the different types of questions raised in
Parliament during Question Hour.
1. Unstarred Questions are answered orally in detail, but no supplementaries can be asked on the
answers.
2. Starred Questions are answered orally and supplementaries can be asked, but only by the member,
who put up the original question.
3. Short Notice Questions can be dealt with orally or otherwise depending on the request by the
concerned member.
Which of the statement(s) given above is/are correct?
(a) Only 1 (b) 1 and 2
(c) All of these (d) None of these

63. Consider the following statements regarding Oil zapper.


1. Oil zapper is a technique developed by TERI (The Energy and Resource Institute, Government of
India) to clean the oil spill by using bacteria.
2. The process of oil zapping is not used for bio-remediation process.
3. Oil zapper reacts with hydrocarbon materials present in the oil spill and oil sludge and converts
them into carbon monoxide and water.
Which of the statement(s) given above is/are correct?
(a) Only 1 (b) 1 and 2 (c) 1 and 3 (d) All of these

64. Which of the following is/are true about climate change?


1. It is completely an anthropogenic phenomenon.
2. In short run global warming will help increase agricultural productivity of certain crops in temperate
region.
3. Aurora Borealis is result of global warming.
4. Coral bleaching is one of positive outcomes of climate change.
Which of the statements given above is/are correct?
(a) 1 and 2 (b) Only 2 (c) 1 and 4 (d) 2 and 3

65. Consider the following statements regarding multi-purpose river valley projects.
1. The Bhakra-Nangal Project on Sutlej and Beas river is used both for hydel power production and
irrigation.
2. Hirakud Project in the Mahanadi basin integrates conservation of water with flood control.
3. Damodar Valley Corporation is the first multi-purpose project of India.
Which of the statements given above is/are correct?
(a) Only l (b) 1 and 2 (c) 2 and 3 (d) All of these

66. Consider the following statements.


1. Indirect tax revenue comprised custom and excise before 1999 to 2000. However after 1999 to 2000
another component that of service tax was added to indirect taxes.
2. The tax-GDP ratio of the centre has been consistently lower than the level of 10.3% achieved in 1991 to
1992.
Stage 1 Know the Trend 41

3. In respects of customs, initially the reduction in duty rates were attributable to multilateral trade
agreement induced commitments under the aegis of the World Trade Organisation (WTO).
Which of the statements given above are correct?
(a) 1 and 2 (b) 1 and 3
(c) 2 and 3 (d) All of these

67. Consider the following statements.


1. The Global cooling effect occurred just over a few decades after the occurrence of global warming
phenomena.
2. The Global cooling can be just as detrimental to marine life as global warming.
Which of the following statement(s) given above is/are incorrect?
(a) Only 1 (b) Only 2 (c) Both 1 and 2 (d) Neither 1 nor 2

68. Consider the following statements regarding Kalbelia dance.


1. Famous dance forms of Rajasthan, performed by the Kalbelia snake charmers community.
2. In 2010, the Kalbelia folk songs and dances of Rajasthan were declared a part of its intangible
heritage list by the UNESCO.
Which of the statement(s) given above is/are correct?
(a) Only 1 (b) Only 2 (c) Both 1 and 2 (d) Neither 1 nor 2

69. Consider the following statements regarding various folk songs of India.
1. Ghode Modni dance of Goa is performed with swords.
2. In the Lava dance of Lakshadweep Island, dancers move on fire chambers.
3. Nongkrem is a dance of Toda tribals.
Which of the statement(s) given above is/are correct?
(a) Only 1 (b) 1 and 2 (c) 2 and 3 (d) All of these

70. With reference to Ahmedabad worker’s strike where Gandhiji intervened, consider the
following statements.
1. The workers were demanding the Constitution of ‘Plague bonus’.
2. Gandhiji was soft on mill owners as his ashram was saved from extinction by generous donations of
one of richest mill owners.
Which of the statement(s) given above is/are correct?
(a) Only 1 (b) Only 2 (c) Both 1 and 2 (d) Neither 1 nor 2

71. Which of the following are not features of Mughal painting?


1. Had religious themes 2. Devoid of eroticism
3. Portrayed common people and village life
Select the correct answer using the codes given below.
(a) 1 and 3 (b) 1 and 2 (c) 2 and 3 (d) All of these

72. Which of the following statements regarding species interdependence are true?
1. An association of two species where some are benefitted and other remains unaffected called
mutalism.
2. An interspecific association where both partners derive benefit from each other is called
commensalism
3. A direct food relation between two species of animal, in which one animal kills and feeds on another
is referred as predation.
4. A relation between two species of organism where both the partners are benefitted from each other is
called symbiosis.
Select the correct answer using the codes given below.
(a) 1 and 2 (b) 2 and 3 (c) 3 and 4 (d) 1 and 3
42 Practice Set 2

73. Consider the following statements comparing the veto powers of the President and the
Governor.
1. The President and the Governor only have the options of either assenting or withholding assent to
Money Bills.
2. When the Governor reserves a Bill for the consideration of the President, all further interaction then
takes place between the President and the State Legislature.
3. With regard to State Bills, President has unlimited suspensive veto power, while the Governor can
only use it once.
Which of the statement(s) given above is/are correct?
(a) 1 and 2 (b) 2 and 3
(c) Only 3 (d) None of these

74. Which of the following statements correctly describe the present relationship between the State
of Jammu and Kashmir and Union of India?
1. Parliament cannot alter the name and boundary of Jammu and Kashmir like in the case of other
states.
2. Directive Principles of State Policy are not applicable in Jammu and Kashmir.
3. Jammu and Kashmir is not considered a State of India under the Definition of State in Part VI.
4. President’s rule under Article 356 cannot be imposed in the state and instead there is a provision for
Governor’s rule.
Select the correct answer using the codes given below.
(a) 1 and 2 (b) 1, 2 and 3
(c) 2 and 4 (d) 1, 3 and 4

75. Consider the following statements regarding the subsidy provision by WTO.
1. All subsidies, which are supposed to distort production and trade fall into the Amber box.
2. Green box subsidies are known as Amber box with conditions.
3. Any subsidy that would normally be in the Amber box, is placed in the Blue box if it requires farmers
to go for a certain production level.
Which of the statements given above are correct?
(a) 1 and 3 (b) 2 and 3
(c) 1 and 2 (d) All of these

76. Consider the following statements.


1. The main objective of the National Agricultural Insurance Scheme (NAIS) is to protect the farmers
against losses suffered by them due to crop failure on account of natural calamities.
2. The National Agricultural Insurance Scheme (NAIS) is available to all the farmers, loanees and non-
loanees, irrespective of their size of holding.
Which of the statement(s) given above is/are correct?
(a) Only 1 (b) Only 2
(c) Both 1 and 2 (d) Neither 1 nor 2

77. Consider the following statements related to Hunger and Malnutrition (HUNGAMA) survey
related to child malnutrition. Which of the following are the findings of the survey report?
1. A reduction in the prevalence of child malnutrition is observed.
2. Girls nutrition advantage over boys fades away with time.
3. Hand washing with soap is a common practice among people.
4. Anganwadi centres are widespread but not always efficient.
Select the correct answer using the codes given below.
(a) 2 and 4 (b) 1, 2 and 4
(c) 2, 3 and 4 (d) All of these
Stage 1 Know the Trend 43

78. Consider the following statements.


1. The amount of energy at each trophic level in proportion to the next trophic level is called as primary
productivity.
2. The maximum number of links in any food chain is four or five.
3. Every population thrives in an optimal range of abiotic factors. Beyond this range, one finds less and
less numbers of these organisms. This range is called ecological range.
Which of the statement(s) given above is/are correct?
(a) 1 and 2 (b) 2 and 3
(c) Only 3 (d) Only 2

79. Consider the following statements regarding Intergovernmental Panel on Climate Change
(IPCC).
1. World Meteorological Organisation is one of the parent organisations of the IPCC.
2. Its mission is to provide comprehensive scientific assessments of current scientific, technical and
socio-economic information about the risk of climate change caused by human activity.
3. The IPCC also carries out its own original research regarding climate change.
4. It also carries out the work of monitoring climate or related phenomena itself.
Which of the statements given above are correct?
(a) 3 and 4 (b) l and 2 (c) 2 and 3 (d) All of these

80. With reference to mass movements of soil, consider the following statements.
1. Soil creep is a very slow, continuous movement of soil down hill slopes.
2. Soil flow or soil functions is a flow, gradual movement of soil down hill slopes.
3. In soil creep, soil acts as liquid and hence it creeps fast.
Which of the statement(s) given above is/are incorrect?
(a) Only 1 (b) Only 2 (c) Only 3 (d) All of these

81. Which of the following Indian Species have been included in Red Data Book of IUCN as
endangered species?
1. Blue whale 2. Caribou 3. Bengal tiger 4. Kea parrot
5. Black bear 6. Chinchilla 7. Wild goat
Select the correct answer using the codes given below.
(a) 1, 2, 3 and 6 (b) 1, 3, 5 and 7 (c) 2, 4, 5, 6 and 7 (d) All of these

82. Consider the following statements on the implications of coalition politics in the states.
1. Coalition Government have weakened the position and influence of Chief Ministers in the states.
2. Coalition Government have virtually weakened bureaucracy in state administration.
3. Coalition Government adversely affected political homogeneity of the Council of Ministers.
4. The area of coalition politics expanded the scope of the discretionary powers of the Governors.
Which of the statements given above are correct?
(a) 1 and 2 (b) 2, 3 and 4 (c) 1, 3 and 4 (d) All of these

83. Consider the following statements regarding the constitutional set up of urban local governance
institutions under the provisions of the 74th Constitution Amendment Act.
1. The Constitution provides for a three-tier set up of municipal bodies similar to the set up for
Panchayat bodies.
2. According to the Constitution, election of Mayor has to be a direct election.
3. According to the Constitution, Municipal Corporations can only be set up in urban areas with
population of more than 10 lakhs.
Which of the statement(s) given above is/are correct?
(a) Only 1 (b) 1 and 2 (c) 2 and 3 (d) None of these
44 Practice Set 2

84. A pendulum suspended from the roof of a train has a period T (when the train is at rest). When
the train is accelerating with a uniform acceleration ‘a’, the time period of the pendulum will
(a) increase (b) decrease
(c) remain unaffected (d) become infinite

85. Consider the following statements about Law Commission of India (LCI).
1. It is an executive body established by an order of the Government of India and the first law
commission was established in 1955.
2. It is constituted for a period of five years and its major function is to work on legal reforms.
3. It is a convention that a retired judge of Supreme Court/High Court heads the law commission of
India and the secretary of the department of legal affairs is the ex-officio member of law commission.
Which of the statement(s) given above is/are correct?
(a) 1 and 2 (b) Only 3 (c) 1 and 3 (d) All of these

86. Consider the following statements.


1. The phenomenon where people push up to the tax bracket, due to inflation, resulting in higher taxes
paid and reduced spending by individuals. This is known as fiscal drag.
2. Fiscal drag is associated with Indirect tax.
3. Laffer curve talks about the threshold level of taxes, upto which it can be increased without bringing
counter productive effects.
4. Philips curve deals with tax rate and revenue generation.
Which of the statement(s) given above is/are correct?
(a) Only 1 (b) 1 and 3 (c) 1, 2 and 3 (d) All of these

87. Under which of the following circumstances can the union make laws on items in the State list?
1. When a proclamation of national emergency is in place.
2. For giving effect to any international agreement.
3. When two or more states agree to the same.
4. To safeguard the interests of weaker sections and tribal communities.
Select the correct answer using the codes given below.
(a) 1 and 2 (b) 1, 2 and 3 (c) 3 and 4 (d) 1 and 3

88. Despite low warming potential of carbon dioxide among the major GHGs, still it is one of the
most important Greenhouse Gas. What is/are the basis of it?
1. Rising emission 2. Most abundant GHG 3. High average time of presence in stratosphere
Select the correct answer using the codes given below.
(a) Only 1 (b) Only 3 (c) 1 and 2 (d) All of these
89. Consider the following instruments.
1. Dhol 2. Tabla 3. Saxophone 4. Pakhawas
5. Mridanga 6. Tanpura 7. Jal Tarangs
Which of the above are percussion instruments?
(a) 1, 2, 4 and 7 (b) 3, 5, 6 and 7 (c) 1, 2, 4 and 5 (d) 1, 3, 5 and 6
90. Consider the following statements.
1. Saline soils contain a larger proportion of sodium, potassium and magnesium.
2. Saline soils are known by different names such as reh, usar, thur and kallar etc.
3. Saline soils lack nitrogen and calcium.
4. Alkaline soils occur in arid and semi-arid regions.
Which of the statements given above are correct?
(a) 1 and 2 (b) 2 and 3 (c) 1 and 4 (d) All of these
Stage 1 Know the Trend 45

91. Consider the following statements about Aravalli range.


1. The Aravalli range are the oldest block mountains in India.
2. The highest peak is Guru Shikhar in Mount Abu in Rajasthan.
3. The effectiveness of the barrier is felt only in the Central and Northern parts.
4. The Aravalli range are still growing.
Which of the statement(s) given above is/are correct?
(a) 1 and 2 (b) Only 2
(c) 2, 3 and 4 (d) All of these

92. Which one of the following is more probable regarding the Harappan script?
(a) It is distinctly pro-Dravidian
(b) It is distinctly Sanskritic
(c) It is not alphabetical, but mainly pictographic
(d) The language used by these scripts has definite Sumerian connections
93. Consider the following statements regarding the Bhakti Movement.
1. The saints of Bhakti Movement preached in all the languages like Hindi, Sanskrit, Marathi, Gujarati
etc. As a result, people from all the spheres of society joined and followed the principles advocated
by Bhakti saints.
2. The Bhakti Movement protested against the superiority of priests.
3. The Bhakti Movement was an Egalitarian Movement, which opposed the differences of caste and
creed.
4. The two main aims of Bhakti Movement were, firstly Hinduism should be reformed and second was
to stop the interference of Islamic preachers.
Which of the statements given above are correct?
(a) 1 and 3 (b) 2 and 4 (c) 2 and 3 (d) All of these
94. With reference to groundwater, consider the following statements.
1. The permeable rock in which water is stored is known as aquifer.
2. The water table is the surface where the water pressure head is equal to the atmospheric pressure i.e
the surface of the saturated area.
3. The water table is far below on hill tops.
Which of the statements given above are correct?
(a) 1 and 2 (b) 1 and 3 (c) 2 and 3 (d) All of these

95. Consider the following statements about West Asia.


1. The region is surrounded by seven major seas, e.g. the Agean sea, the Black sea, the Caspian sea, the
Persian Gulf, the Arabian sea, Red sea and Mediterranean sea.
2. West Asia is primarily arid and semi-arid region.
3. Western Asia contains large area of mountanious terrain.
4. The region is one of most developed areas in the world.
Which of the statements given above are correct?
(a) 1 and 2 (b) 2 and 3 (c) 1, 2 and 3 (d) All of these

96. Weathering is the process of disintegration of surface by the action of exogenetic forces.
Consider the following statements about weathering.
1. Rocks of massive character are more likely to have a lesser resistance to weathering than those,
which are bedded.
2. Temperature as an important factor affects both physical and chemical weathering.
3. Lichens on rocks reduce the rate of weathering and help in preserving the surface from denudation.
Which of the statement(s) given above is/are correct?
(a) 1 and 2 (b) 2 and 3 (c) 1 and 3 (d) Only 2
46 Practice Set 2

97. Consider the following statements about inbreeding which is used in animal husbandry.
1. Inbreeding refers to the mating of more closely related individuals within the same breed for 4-6
generations.
2. Inbreeding exposes harmful recessive genes that are eliminated by selection.
3. It helps in accumulation of superior genes and elimination of less desirable genes.
4. Continued inbreeding, especially close inbreeding, usually reduces fertility and even productivity.
This is called inbreeding depression.
Which of the statement(s) given above is/are correct?
(a) 1 and 2 (b) 2 and 4 (c) Only 4 (d) All of these

98. With reference to properties of lava, consider the following statements.


l. Basic lavas are the hottest lavas and are highly fluid.
2. When basic lava flows from a volcano it is very explosive.
3. Acid lavas are highly viscous and cause less explosive volcanoes.
4. Acid lava are also called as felsic lava.
Which of the statements given above are correct?
(a) 1 and 4 (b) 2 and 3 (c) 1, 2 and 4 (d) 3 and 4

99. Consider the following characteristics of a soil.


1. These soils are formed under conditions of high temperature and heavy rainfall with alternate wet
and dry periods.
2. Residual soils formed by leaching in areas of heavy rain.
3. It is of coarse texture, soft and friable.
Which of the characteristic(s) given above is/are of laterite soil?
(a) Only 1 (b) 1 and 2 (c) 2 and 3 (d) All of these

100. Consider the following statements.


1. The Atlas mountains to the North-West of Africa are young mountains like the Himalayas.
2. It separates the Mediterranean sea from the Sahara desert.
3. The Atlas ranges are separated by plateaus. Among the Plateau of Chott is largest.
Which of the statements given above are correct?
(a) 1 and 2 (b) 1 and 3 (c) 2 and 3 (d) All of these
Stage 1 Know the Trend 47
48 Practice Set 2

PRACTICE SET 2 OVERALL ANALYSIS


Stage 1 Know the Trend

Answers with Explanations


1. The process, by which loose sediment is hardened to rock 6. Troposphere is higher at equator because heat is
are collectively called Lithification. Once this happens, continued transported to greater distance by convectional currents.
erosion and re-transport of the sediment become much more The layer contains dust particles and over 90% of water
difficult. The process, which involves in it are compaction, droplets. So, all weather phenomena take place in this layer
recrystallisation, cementation, whereas weathering is the only. All vital atmosphere processes leading to various
breaking down or dissolving of rocks and minerals on Earth’s weather and climatic conditions take place in this layer.
surface. Increase in temperature with increase in height in this layer is
known as temperature inversion.
2. The union government launched social security system i.e.
APY on 9th May, 2015 and came into effect from 1st June, 2015 7. NBA is a statutory autonomous body under the Ministry
for all Indians especially the poor, the underprivileged and the of Environment and Forests (MoEF), established in 2003 to
workers in the unorganised sector. It is administered by the implement the provisions under the National Biological
PFRDA. It is open to all citizens of India who have a savings bank Diversity Act, 2002, after India signed Convention on
account between the age of 18 and 40. Fixed pension for the Biological Diversity (CBD) in 1992.
subscribing is ranging between ` 1000 to 5000 depending on It acts as a facilitating, regulating and advisory body to the
their contributions and this allows an individual to contribute at Government of India, on issues of conservation, sustainable
least 20 years before reaping the benefits of the scheme. use of biological resources and fair and equitable sharing of
benefits arising out of the use of biological resources.
3. Biomedical Technologies bridge the medical and
Additionally, it advises State Governments in identifying the
engineering techniques providing an overall enhancement of
areas of biodiversity important (biodiversity hotspots) as
healthcare. Balloon pump supports the ventricles by increasing
heritage sites.
the blood supply to the heart muscles and making it easier for
the ventricle to pump. The balloon pump is an instrument, which 8. National Green Tribunal Act, 2010 (NGT) enables
is used to assist the heart for hours and days, till the patients own creation of a special tribunal to handle the expeditions
heart remains its normal function. disposal of the cases pertaining to environmental issues. It
Electrocardiograph can be quite helpful in establishing a specific was enacted under India’s constitutional provision of Article
diagnosis and estimating the severity of various cardiac 21, which assures the citizens of India the right to a healthy
diseases. ECG is also used to evaluate cardiac chamber size, environment.
wall thickness etc. In cryopreservation, cornea transplantation NGT was established in 2010. The Tribunals dedicated
has been done with great success. It is for this reason that jurisdiction in environmental matters shall provide speedy
cornea can be preserved easily. Cryopreservation of vital organs environmental justice and help reduce the burden of
like kidney, liver and heart are highly complicated and may evoke litigation in the higher courts. It is not be bound by the
immune rejection. procedure laid down under the code of Civil Procedure,
4. In the tropical areas layer of water has three distinct layers 1908 but shall be guided by principles of natural justice.
according to temperature characteristics and thermocline layer is New Delhi is the Principal place of sitting and Bhopal, Pune,
present in mid-latitude oceanic water during winters. In the polar Kolkata and Chennai shall be the other 4 places of sitting of
region, vertical variation of temperature in the oceanic water is the Tribunal.
nearly absent. Thermocline are layers of water where the
temperature changes rapidly with depth.
9. Bioventing is an in soil remediation technology that uses
indigenous microorganisms to biodegrade organic
5. The committees, mentioned in given statements, and their constituents absorbed to soil in unsaturated zone.
major recommendations are– BR Mehta Committee called for Bioventing provides oxygen to stimulate naturally occurring
three-tier Panchayati Raj set up, Village Panchayats to be directly soil microorganism to degrade compounds in soil.
elected, while the other two to be indirectly elected and District Biosparging involves injecting a gas (usually air) under
Collector to be Chairman of Zila Parishad; pressure into the saturated zone to provide oxygen to
Ashok Mehta Committee called for two-tier Panchayat set up, facilitate microbial degradation of contaminants in
political parties to be allowed in Panchayat Elections, giving groundwater and saturated soil. Bioaugmentation is the
compulsory powers of taxation to Panchayat bodies; GVK Rao introduction of a group of natural microbial strains to treat
Committee called for creation of a new post of District contaminated soil or water.
Development Officer, who will also be the CEO of Zila Parishad,
giving role in planning to Panchayat institutions, regular elections
10. Though widows life was pitiable, they were allowed to
remarry. The custom of polyandry, divorce, remarriage were
to Panchayat bodies; LM Singhvi Committee called for giving
prevalent. Hence, Statement 1 is not correct and 2, 3 and 4
constitutional status to Panchayats, establishment of Nyaya
are correct.
Panchayats.
50 Practice Set 2

11. (d) Simon commission was formed by British Government to 16. In a Cold Front, the cold airmass will be advancing and
discuss further constitutional reforms in India. But this under cutting the warm air. The gradient of the cold front is
commission did not have any Indian member. This was usually much steeper than that of the warm front and the
considered as great insult by Indian nationalist leaders. All party rapid ascent of warm moist air at the front causes rapid
conference was soon formed to protest Simon commission at cooling, condensation and the development of
each and every stage. The anti-Simon commission agitations cumulonimbus clouds.
succeeded in uniting different political parties and communalist 17. The Earth’s surface receives most of its energy from sun
parties like Muslim League and Hindu Mahasabha supported in shortwave form because the hotter body emits energy/
congress. heat in shortwave form and cold body emits heat in longwave
Simon commission was manifestation that British Government forms. Therefore when Earth radiates energy to the
did not think that Indians are fit to decide on issues concerning atmosphere it is in longwave form.
them. So, Indian leaders accepted this challenge and a
18. In order to transform the entire ecosystem of public
commission was formed under chairmanship of Motilal Nehru
services through the use of information technology, the
to draw a alternative scheme of constitutional reform. It came
Government of India has launched the Digital India
out with Nehru Report of 1928.
Programme on 1st July, 2015 with the vision to transform
Mahatma Gandhi was away from political life during anti-Simon India into a digitally empowered society and knowledge
commission agitations. Because he was not convinced that economy. It is an umbrella programme that covers multiple
time has reaped yet for mass movement. government ministries and departments. This programme
12. Kathak is one of eight forms of Indian classical dance. This has been envisaged and coordinated by the Deity. The Prime
dance traces origin in North India and is influenced by the Minister is the Chairman of Monitoring Committee on Digital
Bhakti Movement. India. The vision of Digital India is centred on three key
Three major schools of Kathak are as follow areas— digital infrastructure as a utility to every citizen,
governance and services on demand and digital
● Jaipur Gharana developed in courts of Kachwaha kings of
empowerment of citizens. Digital India aims to provide the
Jaipur.
much needed thrust to the 9 pillars of growth areas, namely
● Lucknow Gharana developed in courts of Nawab of Oudh.
Broadband Highways, Universal Access to Mobile
● Benaras Gharana developed by Janki Prasad.
Connectivity, Public Internet Access Programme,
13. The Supreme Court is primarily a court of appeal and hears e-Governance, e-Kranti, Information for all, Electronics
appeals against the judgements of the lower courts. An appeal Manufacturing, IT for jobs and Early Harvest Programmes.
can be made to Supreme Court against a judgement of a High Kerala becomes India first complete digital state.
Court. If the High Court certifies that the case involves a
substantial question of law that requires the interpretation of the 19. Because of the submergence, the Western coastal plain
Constitution. Thus, Statement 2 is incorrect and Statement 1 is a narrow belt and provides natural conditions for
and 3 are correct. development of ports and harbours. The rivers flowing
through the former form delta whereas those flowing through
14. Legislation in Union Territories is not done by presidential the latter have estuarine mouths. Eastern and Western
regulations, but by laws made by Parliament for cases, where coastal plains of India are surrounded by mountains and soil
such territories do not have Legislative Assembly and by both is very fertile.
Parliament and State Legislature for cases, where the territories
have their own assemblies. Alongwith Delhi, Puducherry have 20. Savannas have warm temperature year round. There are
its own High Court. Union Territories can be given High Courts actually two very different seasons in a savanna; a very long
of their own or they can be included in the jurisdiction of some dry season (winter) and a very wet season (summer). In the
other High Court. Both the Legislative Assemblies of Union dry season there is only an average of about 4 inches of
Territories and the Parliament have powers to make laws in rainfall. Between December and February, no rain will fall at
respect of items in the State List and the Concurrent List for that all. Plants of the Savannas are highly specialised to grow in
territory and in case of a conflict between the two laws the union this environment of long periods of drought.
law prevails. Trees 6-12 m height strongly rooted with flattened crown.
15. During the early 1860s, the American Civil War had led to a They have long tap roots that can reach the deep water table,
rise in cotton exports which had pushed up prices. thick bark to resist annual fires, trunks that can store water
The end of the Civil War in 1864 brought about an acute and leaves that drop out during the winter to conserve
depression in cotton exports and a crash in prices. water.Trees are fire resistant and species diversity is low.
Simultaneously, in 1867, the Government raised land revenue 21. EI-Nino is a band of anomalously warm ocean water
by nearly 50%, so Statement 1 is incorrect, as further decrease temperatures that periodically develop off the Pacific coast of
in prices and increased revenue led severe exploitation of South America. Extreme climate change pattern oscillations
peasants. Statement 2 is correct as Deccan was part of fluctuate weather across the Pacific Ocean, which results in
Ryotwari System. After Deccan riots 1875, British government fluctuating droughts, floods and crop yields in varying regions
passed the Deccan Agricultural Relief Act, 1879 to deal with of the world.
agricultural indebtedness.
Stage 1 Know the Trend 51

The first signs of an EI-Nino are as follow 25. The satellites particularly in the LEO are subject to a
● Rise in surface pressure over the Indian Ocean, Indonesia and drag produced by an atmosphere due to frequent collisions
Australia. between the satellite and surrounding air molecules.
● Fall in air pressure over Tahiti and the rest of the Central and The amount of this drag keeps increasing or decreasing
Eastern Pacific Ocean. depending upon several factors including the solar activity.
● Trade winds in the South Pacific weakened or head East. It increase the heat of the upper atmosphere. This drag in a
● Warm air rises near Peru, causing rain in the Northern Peruvian long duration causes a reduction in the altitude of a
deserts. satellite’s orbit which is called orbital decay. So, the major
● Warm water spreads from the West Pacific and the Indian cause of the orbital decay is Earth’s atmosphere NCL 2 .
Ocean to the East Pacific.
The result of the drag is increased heat and possible
Because EI Nino’s warm pool feeds thunderstorms above, it re-entry of satellite in atmosphere causing it to burn. Lower
creates increased rainfall across the East central and Eastern its altitude drops and the lower the altitude, the faster the
Pacific Ocean, including several portions of the South American decay.
West coast. The effects of EI Nino in South America are direct
Apart from atmosphere, the tides can also cause orbital
and stronger than in North America. An EI Nino is associated
decay, when the orbiting body, is large enough to raise a
with warm and very wet weather months are April—October
significant tidal bulge on the body, it is orbiting and is either
along the coasts of Northern Peru and Ecuador, causing major
in a retrograde orbit or is below the synchronous orbit.
flooding whenever the event is strong or extreme.
Moon Phobos is one of the best examples of this.
The effects during the months of February, March and April may Whenever, two masses orbit each other, the combined
become critical. Along the West coast of South America, EI Nino effect of the space-time curvature of the moving objects
reduces the upwelling of cold, nutrient-rich water that sustains produces gravitational waves which carry away orbital
large fish populations, which in turn sustain abundant sea birds, energy.
whose droppings support the fertiliser industry. The reduction in
For small masses this effect is negligible, but for very
upwelling leads to fish kills of the shore of Peru.
massive objects like black holes and neutron stars the
22. The World Bank on January 2016 released a report title World energy carried away can be rapid enough to cause their
Development Report 2016; Digital Dividends in Washington. orbits to spiral in on each other, eventually merging the two
By overcoming information barrier, augmenting factors, and masses. Thus, gravitation is also a cause behind the orbital
transforming products, digital technologies can make decay.
development more inclusive, effective and innovative. There are 26. Statement 1 is correct. A Ballistic missile is a missile that
also persistent digital divides across gender, geography, age follows a sub-orbital ballistic flight path. Statement 2 is also
and income dimensions with each country. correct whereas Statement 3 is incorrect as Ballistic missiles
23. Failure of constitutional machinery is grounds for declaration can be launched from fixed sites or mobile launchers.
of President’s rule, while National emergency can be declared 27. UV radiation in the presence of oxygen, chloroform
only under war, external aggression or armed rebellion. slowly converted into phosgene by radical reaction.
Fundamental Rights cannot be restricted under President’s rule Phosgene is used in the production of isocynates, which are
while certain Fundamental Rights can be restricted under precursors to the polymer.
National emergency.
28. Megasthenes in his book Indica states that there was no
National emergency can be declared even before the actual
slavery in India. This statement must have been made
occurrence of war, external aggression or armed rebellion if the
because slavery was not prevalent in India in the extreme
President has reasons to believe that such events are imminent.
form as he had seen in Greece.
President’s rule on the other hand, can be declared only after the
failure of constitutional machinery in a state. 29. During the day, the land gets heated faster than the
24. Substitute motions are not moved, when the original motion water. The air over the land becomes hotter and rises up.
has failed, but as an alternative to the original motion . If the The cooler air from the sea rushes in towards the land to
substitute motion is adopted, then the original motion is not put take its place. The warm air from the land moves towards
to vote. So, Statement 1 is correct and Statement 2 is incorrect the sea to complete the cycle. The air from the sea is called
Subsidiary motions are always related to some other motion and the sea breeze. At night, it is exactly the reverse. The water
do not have an independent existence. e.g. the motion to send a cools down more slowly than the land. So, the cool air from
Bill for the consideration of a Parliamentary committee is related the land moves towards the sea. This is called the land
to the motion. So Statement 3 and 4 are correct breeze.
52 Practice Set 2

30. Manabendra Nath Roy formed the Communist Party of opportunities to cope up with the changed environment.
India in Tashkent, under the auspices of the Communist Adaptation and mitigation are complementary to each other e.g.
International in 1920. But, formally the CPI was formed in if mitigation measures are undertaken effectively, lesser will be
1925 at the first Kanpur Party conference. Ideological the impacts to which we will need to adapt. Similarly, if
differences led to the split in the party in 1964, one was CPI adaptation measures (or the degree of preparedness) are
and another was CPI (Marxist). strong, lesser might be the impacts associated with any given
degree of climate change. The IPCC stressed in its Fourth
31. The NBDS 2015-20 was announced and launched by Assessment Report that there is a need to focus on both
the GOI on 20th December, 2015. The strategy aim to adaptation and mitigation, seeing them as interconnected.
establish India as a world-class bio-manufacturing hub. It
37. Northern plains can be divided into three major zones:
intends to launch a major mission under the Make in India
Bhangar, Tarai and Alluvial plains. The alluvial plains can be
Programme backed with significant investment for the
further divided into the Khadar and Bhangar. Bhangar is a
creation of new bio-tech products, create a strong
narrow belt ranging between 8-10 km parallel to the Shiwalik
infrastructure for R and D and commercialisation and
foothills at the break-up of the slope. Tarai belt lies to South of
empower India’s human resources scientifically and
Bhangar, with a width of 10-20 km, having marshy and swampy
technologically. It has launched four major
conditions known as the Tarai.
missions–healthcare, food and nutrition, clean energy and
education. It also focuses to build the human capital by 38. Statement 1 is correct. BLU-ray Disc is an optical disc
creating a LSBEC. storage medium designed to supersede the DVD formate.
32. Calcitonin is a hormone known to participate in calcium Statement 2 is also correct, Statement 3 is incorrect as BLU-ray
and phosphorus metabolism. In mammals, the major source Disc is associated with a set of multimedia formats. Statement 4
of calcitonin is from the parafollicular or C cells in the thyroid is correct in aspect of BLU-ray Disc.
gland, but it is also synthesised in a wide variety of other 39. Stream capture, river capture or stream piracy is a
tissues, including the lung and intestinal tract. In birds, fish geomorphological phenomenon occurring when a stream or
and amphibians, calcitonin is secreted from the river drainage system or watershed is diverted from its own bed
ultimobrachial glands. The two main actions of oxytocin in and flows instead down the bed of a neighbouring stream. This
the body are contraction of the womb (uterus) during child can happen for several reasons like tectonic Earth movements
birth and location. Oxytocin stimulates the uterine muscles to where the slope of the land changes.
contract and also increase production of prostaglandins,
which increases the contractions further.
40. North-East region particularly Arunachal Pradesh has the
maximum bamboo bearing area followed by Madhya Pradesh.
33. Natural uranium consists primarily of two isotopes, Punjab shows the increasing trend of forested areas.
99.3% is U-238 and 0.7% is U-235. Most nuclear power
plants require fuel with U-235 enriched to a level of 3-5%. To 41. French Commander Count de Lally failed against the English
increase the concentration of U-235, uranium must be in India because the home authorities in France did not
enriched. recognise the importance of securing political power in India.
Also, the French were inferior to the English in naval strength and
34. The correct order in a dispute resolution is conciliation
there was discord between the French Commanders of land and
followed by mediation followed by arbitration. Awards given
sea forces.
under arbitration are confidential and binding in nature. In
arbitration, a mutually acceptable third party hears the 42. While under Article 323 A, only the Parliament is empowered
arguments from the two sides and pronounces a judgement. to create administrative tribunals, under 323 B, both the centre
Mechanisms of dispute resolution such as mediation and and the states can create tribunals. According to Supreme Court
conciliation can be court driven or community driven. The judgement in Chandra Kumar case, jurisdiction of High Courts
emphasis is on bringing the two parties to a mutual cannot be excluded from decisions of these tribunals, which had
understanding in an informal atmosphere. been prohibited earlier. The CAT has original jurisdiction in all
matters of recruitment and service of public servants covered by
35. Himalayan region is rich in the following metallic it, while the State Administrative Tribunals exercise such powers
minerals that are classified under the following zones in case of state public servants. Article 323 B itself mentions
Foreland Zone Uranium, Gold matters like foodstuffs, rent and tenancy rights, land reforms,
Foreland Thrust zone Tin, Tungsten, Copper, Lead, Zinc taxation etc on which tribunals can be formed by the Central and
Phosphorites, Limestones, Gypsum, Magnesite State Governments by law and many such tribunals have been
Suture Tin, Tungsten, Uranium, Sulphides, Porphyry formed and are functioning currently.
Copper 43. Indian Depository Receipts are issued by domestic
Hinterland Chromium, Copper, Tin, Tungsten, Gold depository. Much like an equity share, it is an ownership of a
company. Since, foreign companies are not allowed to list on
36. Adaptation occurs in physical, ecological and human
Indian equity markets, IDR is a way to own shares of those
systems. It involves the following, changes in social and
companies. IDRs are listed on Indian Stock Exchanges. IDR
environmental processes, perceptions of climate risk,
holders are eligible to vote on decisions of the company, similar
practices and functions to reduce risk and exploration of new
to equity shareholders.
Stage 1 Know the Trend 53

44. LLP defined as a corporate business vehicle that enables ● Review Model Concession Agreements (MCAs) to ensure
professional expertise and entrepreneurial initiative to combine equitable risk distribution.
and operate in flexible, innovative and efficient manner, ● Discourage PSUs from PPP projects unless strategically
providing benefits of limited liabilities while allowing its necessary.
members the flexibility for organising their internal structure as a 51. Chlorella and Synechococcus are useful for astronauts as
partnership. Minimum of two partners are required to form a food. They multiply rapidly and thus, will synthesise a rich
LLP. However, there is no limit on the maximum number of harvest of food utilising CO2 and liberating sufficient oxygen
partners. as a byproduct for the use of astronauts. Chlorellin
45. India launched Hydrocarbon Vision 2025 to achieve some (antibiotics) is obtained from Chlorella, Cladophora and
of the key objectives. All of the Statements are part of the said Lyngbya a possess antiviral properties.
objectives. Apart from the given objectives, it also includes Chlorellin, contained in Chlorella, acts like a natural antibiotic.
developing hydrocarbon sector as a globally competitive Agar-agar is obtained from certain red algae such as
industry, which could be bench named against best in the Gelidium, Gracilaria, Gigartina etc. Agar- agar is also used in
world, to have a free market and promote healthy competition certain medicine, cosmetics, culture media, leather media,
among players and improve the customer’s service. leather industry, textile industry; baked food, meat industry
and as emulsifier in dairy products.
46. Development refers to the event that brings qualitative
changes. It is distinct from growth as it is quantitative in nature. 52. The extent of biodiversity is measured in the following
Increase in the proportion of population cannot be considered terms
as development as it may be caused due to migration. The Alpha Diversity It refers to the diversity within a particular
decrease of share of agriculture sector in GDP is the indication area or ecosystem. It is usually expressed by the number of
of development. So, all the Statements are correct except species (i.e. species richness) in that ecosystem.
Statement 1.
Beta Diversity It indicates a comparison of diversity between
47. This programme is jointly implemented by Council of ecosystems, usually measured as the amount of species
Scientific and Industrial Research (CSIR), Ministry of Science change between the ecosystems.
and Technology and Department of AYUSH (Ayurveda, Siddha,
Gamma Diversity It is a measure of the overall diversity
Unani, Homeopathy), Ministry of Health and Family Welfare.
within a large region i.e. Geographic scale species diversity.
TKDL intend to convert the ancient knowledge into unicode
structure, knowledge based interpretation; Compile in a single 53. DPCs are set up under Article 243ZD of the Constitution,
database and Translate the information to English, Spanish, which was inserted by the 74th Amendment Act. Article
German, French for easy reference of the examiners; 243ZD makes it mandatory upon State Governments to
Digitised form makes it easy transmission, preservation. constitute district planning committees though it is a different
matter that many states have actually not constituted them.
48. Global Warming Potential (GWP) compares the amount of
DPCs take up planning of both urban and rural areas and if
heat trapped by a certain mass of a gas to the amount of heat
there is a Municipal Planning Committee (MPC) within the
trapped by a similar mass of carbon dioxide, calculated over a
district even then the DPC will make plans for the urban area.
specific time interval. The GWP depends on the absorption
Regarding the member of DPC, Statement 4 is correct such a
of infrared radiation by a given species. The spectral location of
composition has been mentioned in Article 243ZD. As
its absorbing wavelengths. The atmospheric lifetime of the
regards the rest of the seats in the DPC the State Legislature
species also plays a dominate role in global warming potential.
will be competent to make laws in this regard.
49. The first statement is related to basel norms, which are
54. India opposes OTDS and inclusion of IPR as a criteria for
associated with monetary and financial stability of banking
protectionism. So, Statement 1 is correct and Statement 2 is
sector.
incorrect. Also, India supports to preserve generic medicines
The basel convention is a global convention which aims to trade and biodiversity. So, Statement 3 is also correct.
control the transboundary movement of hazardous waste and
their disposal. It was adopted in 1989 and come into force in
55. An adiabatic process is one, in which there is no heat
exchange with the surrounding. There is an increase in
1992.
entropy of the gas during a irreversible adiabatic process.
50. The Government of India has constituted a 10-member However, during a reversible adiabatic process, there is no
committee to review and revitalise PPP model of infrastructure change in entropy. Adiabatic heating involves compression of
development in May 2015 and submitted report in November the gas, while adiabatic cooling involves expansion of the
2015. The main recommendations are gas.
● Encourage banks to issue Zero Coupon Bonds (ZCBs).
56. Motion for removal of a Supreme Court judge actually
● Equity divestment by government from stable projects. requires to be signed by 100 members in Lok Sabha and
● Amend Prevention of Corruption Act to shield officials from 50 members in Rajya Sabha, before presentation to the
errors originating from bonafide intentions. Speaker or the Chairman of Rajya Sabha. Such a motion can
54 Practice Set 2

be refused to be admitted by the Speaker or the Chairman in 60. Wetlands are the link between land and water and are
his discretion. No Supreme Court judge has been impeached some of the most productive ecosystems in the world. Some
so far. The grounds for removal of Supreme Court judges common names for different types of wetlands are swamp,
provided in the Constitution are only proved misbehaviour and marsh and bog. They are the lands that are transitional
incapacity. Under the Judges Inquiry Act proof of between terrestrial and aquatic ecosystems, so the Statement
misbehaviour or incapacity has to be examined by an inquiry 1 is correct. Kolleru lake is a part of Ramsar Convention,
committee consisting of the Chief Justice or any other judge of hence the Statement 2 is incorrect. Chilika lake was added to
the Supreme Court, Chief Justice of a High Court and one the list of Ramsar sites in danger-the Montreux Record in
distinguished jurist. If the committee finds the judge to be 1993, however removed from it in 2002 due to substantial work
guilty, then both Houses of Parliament have to pass the motion done to solve ecological problem by Chilika Development
for the removal of the judge by a special majority. Authority (CDA), so the Statement 3 is correct.
57. NITI (National Institution for Transforming India) Aayog is The Ramsar sites in India are: Ashtamudi Wetland, (Kerala)
restructured from planning commission to provide key inputs Bhitarkanika Mangroves, (Odisha) Bhoj Wetland, (Madhya
on various policy matters. Pradesh) Chandra Taal, (Himachal Pradesh) Chilika Lake,
Formed after a wide range of consultation with the various (Odisha) Deepor Beel, (Asom) East Calcutta Wetlands,
stakeholders including the Chief Ministers, this body will see a (Paschim Banga) Harike Wetland, (Punjab) Hokersar Wetland,
landmark change in the form of inclusion of all state CMs and (Jammu and Kashmir) Kanjli Wetland, (Punjab) Keoladeo
LGs of UTs to foster a spirit of cooperative federalism. So, all of National Park, (Rajasthan) Kolleru Lake, (Andhra Pradesh)
these statements are correct. Loktak Lake, (Manipur) Nalsarovar Bird Sanctuary, (Gujarat)
58. The carrying capacity of an ecosystem will not remain Point Calimere Wildlife and Bird Sanctuary, (Tamil Nadu) Pong
constant with time and the external environment has decided Dam Lake (Himachal Pradesh) Renuka Wetland, (Himachal
impact upon it. The ecosystem is a core concept in Biology Pradesh) Ropar, (Punjab) Rudrasagar Lake (Tripura) Sambhar
and Ecology, serving as the level of biological organisation in Lake (Rajasthan) Sasthamkotta Lake, (Kerala)
which organisms interact simultaneously with each other and Surinsar-Mansar Lakes, (Jammu and Kashmir) Thrissur Kole
with their environment. Wetlands (Kerala) Tsomoriri, (Jammu and Kashmir) Upper
Ganga River (Brijghat to Narora strech), (Uttar Pradesh),
As such, ecosystems are a level above that of the ecological
Vembanand-Kol Wetland’ (Kerala), Wular Lake (Jammu and
community (organisms of different species interacting with
Kashmir).
each other) but are at a level below or equal to, biomes and the
biosphere. Essentially, biomes are regional ecosystems and 61. While the President does not require any external
the biosphere is the largest of all possible ecosystems. permission to make ordinances, the Governor does require
Carrying capacity is the maximum population of any species the approval from the President before making ordinances in
that can survive in a particular area before the ecosystem certain areas like those, which in the form of a Bill would
becomes too degraded. require the previous permission of the President before
Population is affected by factors such as disease, competition introduction into the State Legislature or those, which in the
and predator-prey relationships. Ecosystems are dynamic form of a Bill would have made the Governor reserve them for
entities –invariably, they are subject to periodic disturbances the consideration of the President. Ordinances can be used in
and are in the process of recovering from some past any area that the particular legislature has jurisdiction to make
disturbance. laws, including Money Bills. As explained above, if the
legislature has the right to repeal or modify a law, then it can
When an ecosystem is subject to some sort of perturbation, it
be done so with an ordinance as well.
responds by moving away from its initial state. The tendency of
a system to remain close to its equilibrium state, despite that 62. Unstarred are not answered orally and the written answers
disturbance, is termed as its resistance. On the other hand, the are just placed on the table of the House. Starred questions
speed with which it returns to its initial state after disturbance is are answered orally and supplementaries can be asked by
called its resilience. anyone, including those from the ruling party. Short notice
questions are dealt with orally, just like the starred questions.
59. Tritium is a radioactive isotope of hydrogen. The nucleus
of tritium (sometimes called a triton) contains one proton and 63. The process of oil zapping is used for bio-remediation
two neutrons, whereas the nucleus of protium (by far the most process. Oil zapper reacts with hydrocarbon materials present
abundant hydrogen isotope) contains one proton and no in the oil spill and oil sludge and converts them into carbon
neutrons. Naturally occurring tritium is extremely rare on Earth, monoxide and water.
where trace amounts are formed by the interaction of the 64. Climate change is both natural and man-made
atmosphere with cosmic rays. phenomenon. In short run the agricultural productivity will
Tritium finds use in various fields including increase in the temperate region as the growing season will
● Self-powered lighting ● Nuclear weapons increase due to less severe winters. Aurora Borealis is caused
● Neutron initiator ● Boosting by trapping of solar particles in the magnetic field of Earth.
● Tritium is sometimes used as a radiolabel. Coral bleaching is the negative outcome of climate change.
● Tritium is an important fuel for controlled nuclear fusion. So, only Statement 2 is correct.
Stage 1 Know the Trend 55

65. Bhakra-Nangal Project It is the highest multi-purpose river Lava dance is a very colourful and energetic dance of
valley project of India. It is a joint ventur of Punjab, Haryana and Lakshadweep, in which the dancers are dressed in
Rajasthan. Dams are constructed across the river Sutlej in multi-hued costumes and headgears. They also carry a
Himachal Pradesh at Bhakra and Nangal. Bhakra dam is 226m drum while dancing. In this form of dance, the
high and is the highest dam in Asia. The canals of this project are participants perform to the rhythmic beats of drums and
3402 km long and provide irrigation to 14.6 lakh hectares of land. songs.
Delhi and Himachal Pradesh also get irrigation water and The Nongkrem dance festival is celebrated during
hydro-electric power. autumn at Smit, the cultural centre of the Khasi hills. A five
The Hirakud Project Flood control, irrigation and production of day long religious festival of the Khasi tribes, Ka
hydro-electricity are the main purposes of this project. This project is Pemblang Nongkem dance is popularly known as
constructed across the river Mahanadi at a distance of 10 km from Nongkrem dance.
Sambalpur in Odisha. The Hirakud Project has the longest dam in 70. Statement 1 is correct, whereas Statement 2 is
India with a length of 4801 m. It irrigates almost 2.54 lakh hectares of incorrect. Ambalal Sarabhai was a friend of Gandhiji and
land. The States of Odisha, Bihar and Chhattisgarh are provided with had saved the Sabarmati Ashram from extinction by
irrigation and hydro-electricity. generous donations and was leading mill owner of the
Damodar River Valley Project This was the first multi-purpose town. Gandhiji persuaded the mill owners and the
project constructed after independence. It was a joint venture of workers to agree to arbitration by a tribunal but owners
former Bihar and Paschim Banga States. The objectives of this withdrew agreement as had called off strike. The breach
project were to control floods, provide navigation, hydro-electricity, of agreement was treated by Gandhiji as a very serious
fish culture, recreational facilities afforestation and prevention of soil and he advised the workers to go on strike. Throughout
erosion. It has 2495 km length of canals and 4.5 lakh hectares of the strike workers demanded a Plague bonus.
land under irrigation. Dams are constructed across river Damodar 71. Mughal painting is a particular style of South Asian
and its tributaries at Tilaya, Konar, Maithon and Panchet hill. painting, generally confined to miniatures either as book
Thermal power stations are constructed at Bokaro, Chandrapur and illustrations or as single works to be kept in albums, which
Durgapur. Jharkhand and Paschim Banga benefit from this project. emerged from Persian miniature painting, with Indian
66. The Indian Tax sources, can be divided into direct tax and Hindu, Jain and Buddhist influences and developed
indirect tax. Indirect Revenue Tax comprised customs and excise largely in the court of the Mughal Empire (16th-19th
before 1999 to 2000. However after 1999 to 2000 another centuries) and later spread to other Indian courts, both
component that of service tax, was added to indirect taxes. The tax Muslim and Hindu and later Sikh.
GDP ratio of the centre has been consistently lower than the level of The origin of Mughal style was a result of synthesis of
10.3% achieved in 1991 to 1992. In respect of customs, initially the indigeneous Indian style of painting and the Safavid
reductions in duty rates were attributable to multilateral trade school of persian painting. Mughal paintings were unique
agreement induced commitm- ents under the aegis of the World blend of Indian, Persian and Islamic styles.
Trade Organisation (WTO).
The major features are as follows
67. Many studies have focussed on the effects of global warming.
● Based on close observation of nature.
Now, researchers have shown that a phenomenon called ‘Global
● Fine and delicate drawing along with calligraphic text
Cooling’ that occurred about 116 million years ago was associated
descriptions, generally on border.
with the loss of marine life. Global cooling can be just as
detrimental to marine life, as global warming. It is worthnoting that ● High aesthetic merit and was not devoid of eroticism.
the global cooling effect occurred over a period of a million years ● Primarily aristocratic and did not portray common
while human induced global warming has occurred just over a few people and village life.
decade. ● Mostly secular.
68. Kalbelia or Kabeliya is one of the most sensuous dance forms So, all of the Statements given in this regard are incorrect.
of Rajasthan, performed by a tribe of the same name. Both men 72. The 3 and 4 statements describe the predation and
and women in the tribe participate in this activity to celebrate joyful symbiosis respectively.
occasions. The Kalbelia have a reputation for composing lyrics In commensalism relationship between two organisms,
spontaneously and improvising songs during performances. These one organism benefitted from the other without affecting
songs and dances are part of an oral tradition that is handed down it. In mutualism, both organisms benefitted from each
generations and for which there are neither texts nor training other.
manuals. In 2010, the Kalbelia folk songs and dances of Rajasthan
were declared a part of its intangible heritage list by the UNESCO.
73. While the President can only either assent to or
withhold assent to Money Bills including those coming
69. Ghode Modni Dance, the European influence is very strong in from the State Legislature, the Governor has a third option
folk art and culture of Goa. The performance begins with simple of reserving the Bill for the consideration of the President
steps and the performers wave the swords to represent the brave and, so Statement 1 is incorrect.
deeds and praiseful actions.
56 Practice Set 2

Statement 2 is correct and the role of the Governor ends as Every population thrives in an optimal range of abiotic factors.
soon as he reserves a Bill for the consideration of the This range is called tolerance range. Within the range of
President. The President then can return the Bill for the tolerance in each ecosystem population are able to adjust to
reconsideration of the State Legislature if it is not a Money Bill variations in its physical and chemical environment.
and can keep doing so indefinitely. Thus, the President has an
79. Intergovernmental Panel on Climate Change (IPCC) is a
unlimited Suspensive Veto in case of State Bills. The Governor
scientific Intergovernmental body set up in 1998, by two
on the other hand can exercise his Suspensive Veto only once
organisations, the United Nations Environment Programme
and if the State Legislature again passes the Bill with or without
(UNEP) and the World Meteorological Organisation (WMO).
amendments, the Governor will have no option, but to assent
to the Bill. Thus, the right answer is (b). The IPCC does not carry out its own original research nor does
it do the work of monitoring climate or related phenomena
74. Parliament of India cannot alter the name or boundary of itself. A main activity of the IPCC is publishing special reports
Jammu and Kashmir without the consent of the State on topics relevant to the implementation of the (UNFCCC), an
Assembly, while in the case of the other states, the agreement international treaty that acknowledges the possibility of
of the assembly is not required. Thus, Statement 1 is correct. harmful climate change. The IPCC bases its assessment
While Fundamental Rights are applicable in Jammu and mainly on peer reviewed and published scientific literature.
Kashmir, Directive Principles of State Policy and Fundamental Membership of the IPCC is open to all members of the WMO
Duties are not. Thus, Statement 2 is also correct. Statement 3 and UNEP.
is also correct.
Provision in Part VI providing for Legislative Assembly,
80. Mass wasting, also known as slope movement or mass
movement, is the geomorphic process by which soil, sand,
Governor, High Courts etc which are applicable to states of
regolith and rock move down slope typically as a mass, largely
India but not applicable to Jammu and Kashmir and it has its
under the force of gravity, but frequently affected by matter
provisions in this regard, under its own Constitution. Statement
and water content as in submarine environments and
4 is incorrect, as both President’s rule under Article 356 of the
mudslides. Types of mass wasting include creep, slides,
Indian Constitution and Governor’s rule under Jammu and
flows, topples and falls, each with its own characteristic
Kashmir Constitution are applicable to the state. Thus, the right
features and taking place over time scales from seconds to
answer is (b).
years.
75. It is the blue Box, which is known as Amber box with
conditions, because of the explanation given in Statement 3.
81. The IUCN Red List of Threatened Species (also known
as the IUCN Red List or Red Data List), founded in 1964, is
Generally, Green box subsidies are agricultural subsidies,
the world’s most comprehensive inventory of the global
which cause minimal or no distortions to trade. They must not
conservation status of biological species. The International
involve price support.
Union for the Conservation of Nature (IUCN) is the world’s
76. NAIS is being implemented in the country since Rabi main authority on the conservation status of species. A series
1999-2000, as a part of risk management in agriculture with the of Regional Red Lists are produced by countries or
intention of providing financial support to the farmers in the organisations, which assess the risk of extinction to species
event of failure of crops as a result of natural calamities, pests within a political managements unit. The Red List of 2012 was
and deceases. The scheme envisages coverage of all the food released at the Rio + 20 Earth Summit. It contains 132 species
crops (cereals, millets and pulses), oilseeds and of plants and animals in India listed as critically endangered.
commercial/horticultural crops. It is a yield guarantee scheme
Few of the endangered species are
operating on ‘area approach’ basis.
● Asian black bear ● Asian elephant
77. Statement 1 is correct, as the report observed that child ● Bengal tiger ● Blue whale
malnutrition level has been reduced to 42% from 53%, ● Barasingha ● Clouded leopard
Statement 3 is incorrect, because specifically observed that
● Central Kashmir vole ● Dhole
hand washing with soap is not a common practice. Apart from
● Gee’s golden langur ● Gaur
Statement 1, 2 and 4, the report also found that the birth weight
is an important risk factor for child malnutrition and mother’s ● Marbled cat ● Nilgiri marten
education level determines children’s nutrition etc. ● Nicobar flying fox ● Nilgiri tahr
● Nicobar treeshrew ● Palm rat
78. Primary productivity is the rate, at which an ecosystem’s
● Red panda
producers create biomass. The amount of energy at each
trophic level in proportion to the next trophic level is called as 82. Coalition Government have weakened the political
ecological efficiency. 10 % rule, say that only about 10 % of the executive and have created the conditions in which the
energy available to one trophic level gets transferred to the government have become unstable. On the other and the
next trophic level. Most food chains have no more than four or bureaucracy or the permanent executive have either become
five links. There cannot be too many links in a single food more powerful because of the resources and where withal at
chain, because the animals at the end of the chain would not their disposal alongwith a multifurcated legislature, they
get enough food to stay alive. remain unaffected.
Stage 1 Know the Trend 57

83. Statement 1 is incorrect, Municipalities under the 89. The saxophone (also referred to as the sax) is family of
Constitution do not have a three-tier set up like in case of woodwind instruments. Saxophones are usually made of brass
Panchayats. According to the Constitution, election of Mayor and played with a single-reed mouthpiece similar to that of the
has to be a direct election. So, Statement 2 is incorrect. clarinet. The saxophone family was invented by the Belgian
Statement 3 is also incorrect, Municipal Corporations can not instrument maker Adolphe Sax in 1840.
be set up in urban areas with population of more than 10 Tabla is an Indian percussion instrument, which dates back to
lakhs. the times of Persian Muslims.
84. The time period of pendulum (T) =2 π c
g here ι is The dhol is a double-sided barrel drum played mostly as an
pendulum length and g is acceleration of gravity. accompanying instrument in regional music forms.
When train accelerates, new weight relative to train coach will Pakhawaj is essentially a North Indian version of the
be bigger. So, g will be bigger. This would reduce the period Mridangam and is the most common North Indian
of pendulum. representative of the class of barrel shaped drums known as
mridang.
85. Law Commission of India (LCI) is a non-statutory and
non-constitutional body but is an executive body established The Mridangam is a percussion instrument from India of
by an order of the Union Government. It is generally ancient origin. It is the primary rhythmic accompaniment in a
constituted for three years to work on legal reforms. The first Carnatic music.
law commission was constituted in 1834 but the first law The tambura, tanpura, tamburi (Bengali) is a long-necked
commission of independent India was constituted in 1955 plucked lute (a stringed instrument) found in different forms in
under the chairmanship of MC Setalvad and reconstituted Indian music culture. Hindustani musicians speak of ‘tanpura’
every three years since 1955. It is a convention that a retired whereas Carnatic musicians say ‘tambura’; ‘tamburi’ is a
judge of Supreme Court heads India’s Law Commission. The smaller instrument used for accompanying instrumental
secretary of Department of legal affairs is ex-officio member of soloists. The Jal tarang is an Indian melodic percussion
the commission. LCI is inspired by various parts of instrument. It consists of a set of ceramic or metal bowls tuned
constitutions like FRs, DPSPs particularly Article 39A. The with water. The bowls are played by striking the edge with
20th LCI was constituted in 2012 under the chairmanship by beaters, one in each hand.
Justice AP Shah, former Chief Justice of Delhi High court and 90. Saline soils contain a larger proportion of sodium,
the 21st Law commission was constituted in 2015 for three potassium and magnesium and thus they are infertile. Saline
years. soils are known by different names in different areas. Saline
86. The statement 1 gives the definition of fiscal drag. Fiscal soils lack nitrogen and calcium. Alkaline soil occur in arid and
drag is generally associated with progressivity of taxes, which semi-arid regions and in water logged and swamgy areas.
is a feature of direct taxes. Indirect tax is regressive in nature. 91. The Aravalli range are the oldest fold mountains in India. It’s
Also, Laffer curve deals with tax rate limit and the revenue dates back to pre-lndian subcontinental collision with the
generated. It says that after a particular level, the tax rate mainland Eurasian plate. The highest peak is Guru Shikhar in
increase lowers tax revenue rather than increasing it. Philips Mount Abu in Rajasthan. Rising to 5650 ft (1722 m), it lies near
curve deals about the relationship between the level of the South-Western extremity of the range. The effectiveness of
unemployment and rate of change of wage rates. the barrier is felt only in the Central and Western parts. Old fold
87. During a proclamation of national of emergency, the mountains are characterised by having stopped growing higher
Parliament can make laws on items in the State List. Such due to the cessation of upward thrust caused by the stopping
laws though cease to have effect 6 months after the of movement of the tectonic plates in the Earth’s crust below
proclamation has ended. Thus, Statement 1 is correct. Under them. Numerous rivers rise amidst the ranges including the
Article 253 of the Constitution, Parliament can make laws to Banas river, the Luni river, the Sahib, the Sakhi and the
give effect to any international agreement and such laws will Sabarmati river.
not be subject to the usual division of powers into the three 92. The origin of Harappan script is poorly understood as it is
lists given in the Constitution. Thus, Statement 2 is also still undeciphered. It’s connection with Indian writing system
correct, Statement 3 is correct, as Article 252 provides that if (e.g. Brahmi, Devanagari, Bengali etc scripts) is uncertain, so
two or more State Legislatures pass a resolution asking the the option (a) and (b) are incorrect. The script is the Corpus of
union to make laws for them on any item in the State List then symbols produced during 3500-1900 BC, that consists pictures
Parliament will have the authority to do so. Statement 4 is and figures and not any alphabets. So option (c) is correct.
however incorrect as no such provision exists in the Option (d) is incorrect as there is no sumerian connection
Constitution. Thus, the correct answer is (b). found yet in this regard.
88. Despite being low global warming potential of carbon 93. Bhakti saints preached in simple languages, like
dioxide among other major greenhouse gases it is one of the Hindi, Marathi, Gujarati etc and not in Sanskrit. The two main
most important greenhouse gas, because carbon dioxide has aims of Bhakti Movement were to reform Hinduism and second
rising emission and high average time of presence in was to propagate the Hindu-Muslim unity and friendship. The
stratosphere. It is the most abundant GHG in the atmosphere. Bhakti Movement was against the superiority of priests.
58 Practice Set 2

94. The water table is the surface where the water pressure head is 98. Lava is the molten rock expelled by a volcano during
equal to the atmospheric pressure. It may be conveniently visualised an eruption and the resulting rock after solidification and
as the ‘surface’ of the sub-surfaces materials that are saturated with cooling. The source of the heat that liquifies the rock with
ground water in a given vicinity. However, saturated conditions may in the Earth is geothermal energy. Up to 100000 times as
extend about the water table as surface tension holds water in some viscous as water, lava can flow great distances before
pores below atmospheric pressure. Individual points on the water cooling and solidifying because of its thixotropic and
table are typically measured as the elevation that the water rises to in shear thinking properties.
a well screened in the shallow ground water.
99. Laterite = brick (Latin word)
95. Western Asia is located directly South of Eastern Europe. The These soils are formed under conditions of high
region is surrounded by seven major seas. e.g. the Agean sea the temperature and heavy rainfall with alternate wet and dry
Black sea, the Caspian sea, the Persian Gulf, the Arabian sea, the periods. Thus, its formation takes place strictly under
Red sea and the Mediterranean sea. Western Asia is situated on the monsoon conditions.
Western part of trade winds, so the region is primarily arid and can
Residual soils formed by leaching in areas of heavy
be subject to drought. Western Asia contains large area of
rain. Leaching is a process, in which the nutrients get
mountanious terrain such as Anatolian plateau, Pontis mountain,
percolated down below the soil due to heavy rainfall;
Taurus mountain, Mount Ararat and Zagros mountain. Western Asia
thus leaving the top soil infertile also called desilication.
is one of the high economic growth regions in the world because of
Laterite soil are found in elevated areas which receive
extremely large reservoir of petroleum and natural gas. But, this
very high rainfall.
region is not the most developed or developed region of the world.
As a result, top soil gets washed away. This process is
96. Rocks of massive character are more likely to have a greater
called leaching. The soil, therefore, loses its fertility to a
resistance to weathering than those, which are bedded. The bedded
great extent. It covers an area of about 2.4 lakh sq km.
rocks are primarily sedimentary rocks made of different layers
43ue to the presence of iron oxide which is formed by
separated by the cementing material. Bedded rocks are weathered
leaching.
more easily compared to massive rocks, which are primarily igneous
and have interlocking crystalline structure. The soluble plant foods like potash are removed from the
top soil leaving alumina and iron oxide. It is a porus soil,
Temperature affects both physical and chemical weathering. Range
silica is removed from it by chemical action is poor in
of temperature both diurnal and annual subjects the surface layers
lime and magnesium and deficient in nitrogen.
of exposed rocks to expansion and contraction. Temperature also
increases the rate of chemical weathering. 100. The continents of Africa is a huge plateau made up
of hard old rocks. The Atlas mountains to the North-West
97. All the given statements are correct. When breeding is between
are youngfold mountains like the Himalayas. It separates
animals of the same breed it is called inbreeding, while crosses
the Mediterranean sea from the Sahara desert. Its
between different breeds are called outbreeding. Inbreeding refers
highest point is Mount Toubkal (4165 m). The Atlas
to the mating of more closely related individuals within the same
ranges are separated by many plateaus the largest one
breed for 4-6 generations. The breeding strategy is as follows
is called the Plateau of Chotts because it is studded with
superior males and superior females of the same breed are
shallow salt lakes called Chotts.
identified and mated in pairs.
UPSC CIVIL SERVICES (PRE) EXAM

General Studies
Paper 1
Practice Set 3
Time : 2 hrs MM : 200

Instructions

1. This set contains 100 questions. Each question comprises four responses (answers). You will select the response which
you want to mark on the Answer Sheet. In case, if you feel that the correct response is more than one, then mark the
response, which you consider the best. In any case, choose only one response for each questions.
2. All questions carry equal marks.
3. Penalty for Wrong Answer

IMPROVE YOUR WEAKER AREAS


(i) There are four alternatives to answer every question. If a question is marked wrong, one-third of the marks
assigned to that question will be deducted as penalty.
(ii) If a candidate gives more than one answer, it will be treated as wrong answer even, if one of the given answers
happens to be correct and there will be same penalty as to that question, if it has a penalty.
(iii) If a question is left blank i.e. no answer is given by the cnadidate, there wil be no penalty for that question.

1. Consider the following statements regarding cryogenic engines.


1. Cryogenic engines are capable of running at a very low temperature.
2. The fuel used is liquid hydrogen and liquid oxygen.
3. To make liquid hydrogen, the minimum temperature required is - 253°C and for liquid oxygen is -
186°C.
4. In the renewed agreement with Russia, India will import seven cryogenic engines.
Which of the statements given above are correct?
(a) 1, 2 and 4 (b) 2 and 4 (c) 1, 3 and 4 (d) All of these

2. Which of the pair(s) navarasas is/are incorrectly matched?


1. Karuna : Compassion
2. Veera : Valour
Stage 2

3. Shringar : Ornamentation
4. Adbhuta : Wonder
Codes
(a) 1, 2 and 3 (b) Only 3 (c) 2 and 4 (d) Only 4
60 Practice Set 3

3. Which of the following Centrally Sponsored Schemes (CSS) have been undertaken in the 12th
Five Year Plan for environment forestry and wildlife sector?
1. National River Conservation Plan 2. Conservation of Natural Resources and
Ecosystems
3. Afforestation and Forest Management 4. Taj Protection
5. Wildlife Management 6. Project Elephant
7. Project Tiger
Select the correct answer using the codes given below.
(a) 1, 2 and 5 (b) 1, 2, 3, 5 and 7 (c) 3, 4, 6 and 7 (d) All of these

4. The Convention on Biological Diversity (CBD) is one of the key agreements adopted during the
Earth Summit held at Rio de Janeiro in 1992. Consider the following statements.
1. The convention while reaffirming sovereign rights of nations over their biological resources with
establishing three goals of conservation, sustainable use and fair and equitable sharing of the
benefits of the use of resources.
2. India hosted the conference on CBD in Hyderabad in 2012 that also marked the 40th anniversary of
Stockholm Conference.
3. Indian Government has led the provision of setting of Biodiversity Management Committee at union
level on cognisance of the International Convention on Biodiversity.
4. Cartagena Biosafety Protocol was negotiated under the Aegis of the Convention on Biological
Diversity adopted in 2000 to ensure protection in field and safe transfer of living modified organisms.
Which of the statements about CDB given above are correct?
(a) 1, 2 and 4 (b) 1 and 3 (c) 2, 3 and 4 (d) All of these

5. In which of the following circumstances, Constitution empowers the Parliament to make laws
on any matter enumerated in the State list?
1. When Rajya Sabha passes a resolution. 2. During a national emergency.
3. When states make a request. 4. To implement international agreements.
5. During President’s rule.
Select the correct answer using the codes given below.
(a) 1, 2 and 3 (b) 1, 3 and 4 (c) 2 and 4 (d) All of these

6. Consider the following statements for the purpose of holding direct elections to the Lok Sabha.
1. Each State is divided into territorial constituencies.
2. Each State is allotted a number of seats in the Lok Sabha in such a manner that the ratio between that
number and its population is the same for all states. This provision does not apply to a state having a
population of less than a 6 millions.
3. Each state is allotted a number of seats in the Lok Sabha in such a manner that the ratio between the
population of each constituency and the number of seats allotted to it is the same throughout the
state.
Which of the statements given above are correct?
(a) 1 and 3 (b) 1 and 2 (c) 2 and 3 (d) All of these

7. Which of the following pairs are correctly matched?


Biodiversity Hotspot Location
1. Sundaland : Eastern Africa
2. Wallacea : South-East Asia
3. Succulent Karoo : South Africa
4. Cerrado : South America
Codes
(a) 1, 2 and 3 (b) 2, 3 and 4 (c) 1, 3 and 4 (d) 1, 2 and 4
Stage 2 Improve Your Weaker Areas 61

8. Which of the following is/are the potential consequences of Free Trade Agreement (FTA)
between India and ASEAN?
1. Increased employment opportunities between both.
2. Violation of WTO global trading norms.
3. To enhance the investment through Foreign Direct Investment (FDI) from ASEAN into India.
Select the correct answer using the codes given below.
(a) 1 and 3 (b) 1 and 2
(c) 2 and 3 (d) Only 1

9. Which of the following is/are the resultants of the foreign investment in the National
Manufacturing and Investment Zones (NMIZs)?
1. Increase in India’s Current Account Deficit (CAD).
2. Supplement domestic expertise and resources.
3. De-congestion of nearby urban areas.
Select the correct answer using the codes given below.
(a) 2 and 3 (b) Only 3 (c) 1 and 2 (d) All of these

10. Consider the following statements.


1. Different school of arts like Central Asian, Gandhara and Mathura rose in Kushan empire.
2. Influence of Gandhara art spread till Mathura, which was primarily a centre of indigenous art.
3. Images of Buddha in Gandhara style were made in the Graeco Roman style and his hair was
fashioned in the Graeco Roman style.
Which of the statements given above are correct?
(a) 1 and 2 (b) 2 and 3 (c) 1 and 3 (d) All of these

11. Consider the following statements about Eastern hills or mountains.


1. These parts of the Himalayas mountain system have general alignment in East-West direction.
2. These are low altitude hills inhabited by numerous tribal groups practising Jhum cultivation.
3. The Molassis basin of Mizoram has soft and unconsolidated soil deposits.
Which of the statements given above are correct?
(a) 1 and 2 (b) 2 and 3 (c) 1 and 3 (d) All of these

12. Consider the following statements regarding preventive detention.


1. Even a person under preventive detention does have the right to consult and be defended by a legal
practitioner or a lawyer under Article 22.
2. Being a subject related to the security of the nation, only the Parliament can make laws on preventive
detention, the State Legislatures cannot.
3. India is the only nation to have a preventive detention law in the world as of now.
Which of the statement(s) given above is/are correct?
(a) 2 and 3 (b) 1 and 3
(c) 1 and 2 (d) Only 3

13. Consider the following statements with reference to hybridisation in crops.


1. Only way of incorporating desirable characters into crop varieties is by hybridisation.
2. Hybridisation refers to crossing between genetically dissimilar plants.
3. This crossing may be intervarietal (between different varieties), interspecific (between two different
species of the same genus) or intergeneric (between different genera).
Which of the statement(s) given above is/are incorrect?
(a) Only 1 (b) 2 and 3
(c) l and 2 (d) None of these
62 Practice Set 3

14. Which of the following are the objectives of the Science, Technology and Innovation (STI)
Policy 2013?
1. Positioning India among the top five global scientific powers by 2020.
2. Raising Gross Expenditure in Research and Development (GERD) to 2% from the present 1% of the
GDP in this decade by encouraging enhanced private sector contribution.
3. India has declared 2010-20 as the ‘Decade of Innovation’.
4. A strong and visible Science, Research and Innovation System for High Technology led path for
India (SRISHTI).
Select the correct answer using the codes given below.
(a) 1 and 2 (b) 1, 2 and 3
(c) 3 and 4 (d) All of these

15. The Defence Research and Development Organisation (DRDO) successfully test-fired Nag
anti-tank missile. Which of the following is not a feature of Nag missile?
(a) It is a third generation fire-and-forget missile
(b) It is equipped with top attack capability to nullify the explosive reactive armour of a battle tank
(c) It cannot be fired during night
(d) It can be fired against both moving and fixed targets

16. Consider the following statements with reference to the Indus Water Treaty, 1960.
1. Under the treaty, the waters of Western rivers of the Indus river system are allocated to India.
2. India is under obligation to let flow the waters of the Eastern rivers of the Indus river system except
for the uses such as domestic use, non-consumptive use, agricultural use as specified and generation
of hydro-electric power as specified.
3. India is under no obligation to supply information of its storage and hydro-electric projects as
specified.
4. This treaty was brokered by International Bank for Reconstruction and Development between India
and Pakistan.
Which of the statement(s) given above is/are correct?
(a) 1, 2 and 3 (b) 2, 3 and 4
(c) 3 and 4 (d) Only 4

17. Consider the following statements.


1. WC Bannerjee was a notable absentee in the first session of Congress in 1885.
2. WC Bannerjee founded Indian National Association in 1876. He was busy with it during the first
session of Congress.
3. Dadabhai Naoroji was the first President of the Indian National Congress. He was the person who
gave it the name of Congress from union.
Which of the statements given above are correct?
(a) 1 and 2 (b) 2 and 3
(c) All of these (d) None of these

18. Consider the following statements regarding seismic waves.


1. There are two types of body waves. They are called P and S waves.
2. The P-waves are similar to light waves and S-waves are similar to sound waves.
3. The P-waves are slower than S-waves.
4. P-waves can travel through gaseous, liquid and solid materials while S-waves can travel only
through solid materials.
Which of the statements given above are correct?
(a) 1, 2 and 3 (b) 1, 3 and 4
(c) 1 and 4 (d) All of these
Stage 2 Improve Your Weaker Areas 63

19. Consider the following statements.


1. Indian economy nearly stagnated under colonial rule.
2. Foreign investment was discouraged in the early decades post-independence.
3. Central economic planning in India started only after independence.
Which of the statement(s) given above is/are correct?
(a) 1 and 2 (b) Only 3
(c) 2 and 3 (d) All of these

20. Consider the following statements with reference to the Rajiv Gandhi Panchayat Sashaktikran
Abhiyan (RGPSA) scheme.
1. The schemes of Rashtriya Gram Swaraj Yojana (RGSY), e-Panchayat, Panchayat Empowerment and
Accountability Incentive Scheme (PEAIS) and Panchayat Mahila Evam Yuva Shakti Abhiyan
(PMEYSA) will be subsumed in RGPSA from 2013-14.
2. The funding of RGPSA for state plans is envisaged on a 90:10 sharing basis by the Central and all
State Governments respectively.
3. During the 12th Plan, grants to State Election Commissions are linked to the performance under
RGPSA.
4. Constitution of State Finance Commission (SFC), every 5 years and placement of Action Taken
Report (ATR) on the recommendations of the SFC in the State Assembly is a must to avail funds
under RGPSA.
Which of the statements given above are correct?
(a) 1, 2 and 3 (b) 1, 3 and 4
(c) 2 and 4 (d) All of these

21. Consider the following statements with respect to a lake.


1. It is located on the Eastern side of Srinagar city in Kashmir.
2. It is considered an Oxbow lake of the Jhelum river.
3. It is suffering from the problem of excessive sedimentation.
4. The lake has some interesting flora like lotus flowers, water lilies and water chestnut.
Which of the following lakes rightly fulfil all of the above criteria?
(a) Gumsar lake (b) Tso Moriri lake
(c) Dal lake (d) Pangong TSO

22. Consider the following statements regarding superconductivity.


1. Superconductivity is a phenomenon observed in several metals and ceramic materials.
2. Superconductivity is a phenomenon of exactly zero electrical resistance.
3. It shows expulsion of magnetic fields occurring in certain materials when cooled below a
characteristic critical temperature.
4. It is not characterised by the Meissner effect, in which complete ejection of magnetic field lines from
the interior of the superconductor.
Which of the statements given above are correct?
(a) 1, 3 and 4 (b) 1, 2 and 3 (c) 1 and 2 (d) All of these

23. Storing water in the form of groundwater is often considered to be a better option than storing it
in surface reservoirs. What can be the possible reasons behind it?
1. Unlike surface water, groundwater can always be free of contaminants.
2. Evaporation of surface reservoirs is higher as compared to groundwater.
3. Stagnated surface water provides a breeding ground for disease vectors.
4. Even nearby wells can be recharged and water need not be conveyed to each well.
Select the correct answer using the codes given below.
(a) 1 and 4 (b) 2, 3 and 4 (c) 1, 2 and 3 (d) All of these
64 Practice Set 3

24. Consider the following statements.


1. The hydrogen bomb is based on thermonuclear fission reaction.
2. The fundamental physics of the fission chain reaction in a nuclear weapon is quite different to the
physics of a controlled nuclear reactor. Also, the two types of devices are engineered differently.
3. No industrialised nation has been able to meet even one-fourth of its total demand of energy from
nuclear reactors.
Which of the statement(s) given above is/are correct?
(a) 1 and 2 (b) Only 3 (c) All of these (d) None of these

25. Consider the following statements.


1. The last Mauryan ruler, Brihadratha was assassinated by his commander-in-chief, Pushyamitra
Sunga.
2. The last Sunga king, Devabhuti was assassinated by his Brahmana Minister Vasudeva Kanva who
annened the throne.
3. The last ruler of the Kanva dynasty was deposed by the Andhras.
Which of the statement(s) given above is/are correct?
(a) 1 and 2 (b) Only 2 (c) Only 3 (d) All of these

26. ‘Critically endangered’ is the highest risk category assigned by the IUCN to wildlife species.
Which of the following are correctly the part of criteria used by the IUCN to declare a specific
animal as critically endangered?
1. Population have declined or will decrease, by greater than 80% over the last 10 years or 3
generations.
2. Have a wide geographical range.
3. High probability of extinction in the wild.
4. Small population size of less than 250 individuals and continuing decline at 25% in 3 years or
1 generation.
5. Very small or restricted population of fewer than 5000 mature individuals.
6. It must be found in atleast 2 countries.
Select the correct answer using the codes given below.
(a) 1, 3 and 4 (b) 1, 2, 3 and 4 (c) 2, 3, 5 and 6 (d) All of these

27. Recently, biodiversity hotspots are common talk in ecological discussion. Which of the
following statements regarding hotspots are correct?
1. The concept of biodiversity hotspot was originated by Norman Myers.
2. Biodiversity hotspots are areas with high diversity of locally endemic species.
3. India has 3 biodiversity hotspots i.e. Himalayas, Western Ghats and Eastern Ghats regions.
4. To be a biodiversity hotspot, a region must contain atleast 3% of plants as endemic and it has to have
lost atleast 50% of its primary vegetation.
Select the correct answer using the codes given below.
(a) 1 and 2 (b) 2 and 3
(c) 1 and 4 (d) All of these

28. Consider the following statements with reference to lightning.


1. During storms, negative and positive charges meet in the clouds, producing streaks of bright light
and sound. The process is called an electric discharge.
2. During lightning it is not safe to use mobile phone, but it is safe to use a landline.
3. Carrying umbrella is not a good idea at all during thunderstorms.
Which of the statements given above are correct?
(a) l and 2 (b) 2 and 3
(c) 1 and 3 (d) All of these
Stage 2 Improve Your Weaker Areas 65

29. Consider the following statements regarding Brown label ATMs.


1. Banks outsource the ATM operations to a third party: (Brown label ATMs).
2. Brown label ATMs do not have any bank’s logo.
3. Companies operating Brown lable ATMs have to separately get license/permission from RBI to run
business.
Which of the statement(s) given above is/are correct?
(a) Only 1 (b) 2 and 3
(c) 1 and 3 (d) All of these

30. Which of the following descriptions best describes soil texture?


(a) Arrangement of soil minerals
(b) The way in which individual particles of sand, silt and clay are assembled
(c) Proportion of air, water and soil matter
(d) Relative content of sand, silt and clay in a soil

31. Consider the following statements regarding Wavell Plan (1945).


1. Wavell Plan proposed formation of interim government before proceeding for the work of framing
Constitution.
2. It proposed balanced representation of Muslims and caste Hindus in the interim government.
3. All members of the Central Executive Council barring the Governor-General were to be Indians.
4. Defence of India was to be in the hands of Prime Minister of interim government.
Which of the statements given above are incorrect?
(a) 1, 2 and 4 (b) 2, 3 and 4
(c) 3 and 4 (d) 1, 3 and 4

32. Consider the following statements about the ‘Great Indian Desert’.
1. This region lies in the North-West of the Aravali hills, has arid climate.
2. During the Mesozoic era, this region was under the sea.
3. This was proven by the marine deposits found around Brahmsar, Jaisalmer which date back to 180
million years.
Which of the statements given above are correct?
(a) 1 and 2 (b) 2 and 3
(c) 1 and 3 (d) All of these

33. Which of the following pairs are incorrectly matched?


1. Harshavardhan : Ratnavali, Priyadarshika
2. Banabhatta : Harshacharita, Nagananda
3. Bhartrihari : Vakyapadiya, Kadambari
Codes
(a) 1 and 2 (b) 2 and 3
(c) 1 and 3 (d) All of these

34. Consider the following Hindu Gods.


1. Indra 2. Vishnu
3. Varun 4. Shiv
5. Agni 6. Aditi
7. Usha
Which of the above Gods were worshipped during the Rigvedic Age?
(a) 1, 3 and 5 (b) 1, 2, 4 and 5
(c) 1, 3, 5, 6 and 7 (d) All of these
66 Practice Set 3

35. Consider the following statements regarding the national park and sanctuary in India.
1. Dachigam National Park is natural habitat for Kashmiri stag.
2. Nandankandan National Park in West Bengal serves as the natural habitat of giant salt water
crocodile in India.
3. Bhitarkanika in Odisha is home to nearly extinct Asiatic lions in India.
Which of the statement(s) given above is/are correct?
(a) Only 1 (b) 1 and 2
(c) 2 and 3 (d) 1 and 3

36. Consider the following statements on Weekly Iron Folic acid Supplementation (WIFS)
programme.
1. Iron folic acid tablet can be taken during illness and even during menstruation.
2. The Weekly Iron Folic Acid Supplementation (WIFS) is an evidence based programme for addressing
Iron Deficiency Anaemia.
3. If taken on empty stomach, these tablets can even cause death in many cases of children.
4. This programme is restricted to the children between class 4th to class 12th as children below 4th
standard are vulnerable to side effects.
Which of the statements given above are correct?
(a) 1 and 2 (b) 1 and 3
(c) 1, 2 and 3 (d) All of these

37. Consider the following statements.


1. In recent times, the legislature has lost the initiative in policy making to the executive.
2. The policy laid down by the legislature finds expression through the laws made or resolutions passed
by it.
3. In the developing countries, the executive has been even more influence in policy making than in
developed countries.
4. In the developing countries, pressure groups have little influence or impact over policy making out
due to their lack of coordination.
Which of the statements given above are correct?
(a) 1, 2 and 3 (b) 1 and 4
(c) 2, 3 and 4 (d) All of these

38. The PMO provides secretarial assistance and crucial advice to the Prime Minister. Which of the
following are the functions of PMO?
1. Assisting the Prime Minister in respect of his overall responsibilities as head of the government, like
maintaining coordination with Central Ministers and the State Government.
2. Helping the Prime Minister in respect of his responsibilities as Chairman of the NDC.
3. Providing assistance to the Prime Minister in the examination of cases submitted to him for orders
under prescribed rules.
4. Acting as the think-tank to the Prime Minister.
Select the correct answer using the codes given below.
(a) 1, 2 and 3 (b) 1, 3 and 4 (c) 2 and 4 (d) All of these

39. Consider the following statements about the North-Western Himalayas.


1. The world famous valley of Kashmir and Dal lake lies between the Great Himalayas and the Pir
Panjal range.
2. The Kashmir Himalayas are famous for Karewa formations which are glacial clay deposits.
3. Karewa is useful for the cultivation of local variety of saffron.
Which of the statements given above are correct?
(a) 1 and 2 (b) 2 and 3 (c) 1 and 3 (d) All of these
Stage 2 Improve Your Weaker Areas 67

40. Consider the following statements about Lord William Bentinck.


1. Infanticide was declared illegal by Bengal Regulation XXI. Willim Bentinck was main architect of
this regulation.
2. William Bentinck was also chief architect in abolition of Sati and suppression of Thuggee.
3. The Doctrine of Lapse also known as Annexations of Peace was also implemented by Lord William
Bentinck for the first time.
Which of the statement(s) given above is/are correct?
(a) 1 and 2 (b) 1 and 3 (c) 2 and 3 (d) Only 2

41. There are the four venomeous snake species responsible for causing the most human snake bite
cases in South Asia (mostly in India). Which of the following is not one among the ‘big four’?
(a) Indian Cobra (b) Common Krait (c) Russelly’s Viper (d) Indian Python

42. Which of the following diseases are caused by virus?


1. Tuberculosis 2. Bubonic plague 3. Hepatitis-B 4. Polio
5. Measles 6. Cholera 7. Chicken pox
Select the correct answer using the codes given below
(a) 1, 2, 3 and 4 (b) 1, 4, 5 and 6 (c) 2, 3, 5 and 7 (d) 3, 4, 5 and 7

43. Consider the following statements about folk theatre of Nautanki.


1. The word ‘Nautanki’ comprises two words ‘naw’ meaning ‘nine’ and ‘tank’ referring to the different
‘rasas’ of theatre.
2. It consists of mythological dramas with interludes of folk songs and dances.
3. Nautanki lacks in the participation of female performers.
Which of the statement(s) given above is/are correct?
(a) 1 and 2 (b) Only 2 (c) Only 1 (d) All of these

44. Generally we consider drugs and medicines as same but there is difference between them.
Which of the statements given below are correct regarding drugs and medicines?
1. Drugs are chemicals of high molecular masses.
2. Medicines interact with macromolecular targets and produce a non-therapeutic biological response.
3. If taken in doses higher than those recommended, most of the drugs used as medicines are potential
poisons.
4. Drugs usually interact with biomolecules such as carbohydrates, lipids, proteins and nucleic acids.
Select the correct answer using the codes given below.
(a) 1 and 2 (b) 3 and 4 (c) 1, 2 and 4 (d) All of these

45. Consider the following statements regarding Anglo-Mysore Wars.


1. First Anglo-Mysore War saw Tipu Sultan to maintain his strength against the Britishers.
2. Treaty of Mangalore concluded the Second Anglo-Mysore War.
3. Tipu Sultan died during the Third Anglo-Mysore War and Britisher gained full control over Mysore.
Which of the statement(s) given above is/are correct?
(a) 1 and 2 (b) 1 and 3 (c) 2 and 3 (d) Only 2

46. Which of the following are methods of soil conservation?


1. Mulching 2. Contour Barriers 3. Rock Dam 4. Intercropping
5. Shelter Belts
Select the correct answer using the codes given below.
(a) 1, 2 and 4 (b) 1, 3 and 4
(c) 2, 3 and 5 (d) All of these
68 Practice Set 3

47. Consider the following statements regarding the deforestation.


1. Deforestation is a contributor to global warming and is often cited as one of the major causes of the
enhanced greenhouse effect.
2. Deforestation reduces soil cohesion. So that erosion, flooding and landslides ensue.
3. Deforestation on a human scale results in decline in biodiversity and on a natural global scale is
known to cause the extinction of many species.
4. It also results in groundwater getting lowered.
5. In deforested areas, the land heats up faster and reaches a higher temperature, leading to localised
upward motions that enhance the formation of clouds and ultimately produce more rainfall.
Which of the statements given above are correct?
(a) 2, 3 and 4 (b) 1, 3 and 5
(c) 1, 2, 4 and 5 (d) All of these

48. Which of the following constitute the favourable condition for formation of delta?
1. Considerable erosion in upper course of the river. 2. Presence of tidal current at the mouth of delta.
3. Shallow sea at the mouth of river. 4. Existence of lake in course of the river.
Select the correct answer using the codes given below.
(a) 1 and 2 (b) 1 and 3 (c) 1, 2 and 3 (d) 2 and 3

49. Consider the following statements.


1. Commercialisation of agriculture was one of the root cause of Deccan Riots of 1875.
2. Civil War of America, led to cotton boom in the Deccan region. But the benefits reaped by this turned
negative once wars get over their. Farmers were loaded with loan burden.
Which of the statement(s) given above is/are correct?
(a) Only 1 (b) Only 2 (c) Both 1 and 2 (d) Neither 1 nor 2

50. Consider the following statement regarding laser.


1. A laser consists of a gain medium, a mechanism to supply energy to it.
2. The gain medium is a material with properties that do not allow it to amplify light by stimulated
emission.
3. Most practical laser contain additional elements that affect properties of the emitted light such as the
polarisation, the wavelength and the shape of the beam.
4. Laser has not the properties, namely, high monochromaticity, coherence and directionality.
Which of the statements given above are correct?
(a) 1 and 4 (b) 2 and 3 (c) 1, 2 and 3 (d) 1 and 3

51. Consider the following statements about VD Savarkar.


1. VD Savarkar published ‘the Indian War of Independence’ about the rebellion of 1857.
2. VD Savarkar formed Anushilan Samiti to harm Britishers and clear the path for Indian
Independence.
3. VD Savarkar was sentenced two lifetime imprisonment to Cellular Jail in Andaman and Nicobar
islands.
Which of the statements given above are correct?
(a) 1 and 2 (b) 2 and 3 (c) 1 and 3 (d) All of these

52. What advantages are offered by Fly ash concrete in comparison to concrete made with only
cement?
1. It saves water. 2. It lowers heat of hydration.
3. It reduces the corrosion of steel. 4. The concrete quickly becomes strong.
Select the correct answer using the codes given below.
(a) 1 and 4 (b) 2 and 3 (c) 2, 3 and 4 (d) All of these
Stage 2 Improve Your Weaker Areas 69

53. Consider the following statements.


1. The National Urban Health Mission (NUHM) and the National Rural Health Mission (NRHM)
schemes have been now clubbed under Sarva Swasthya Abhiyaan.
2. The NUHM was proposed for the urban poor with focus on those living in urban slums.
3. One Accredited Social Health Activist (ASHA) (community link worker) for 200 to 500 households
will work under NUHM scheme.
Which of the statement(s) given above is/are correct?
(a) l and 2 (b) 2 and 3 (c) Only 3 (d) Only 2
54. Consider the following statements regarding Rajasthan style of paintings.
1. Rajasthan style of paintings combined Arabian style of paintings with Western Indian schools and
Jain styles.
2. It eliminated the court scenes and hunting scenes prevailing in the painting and replaced it with
mythological themes.
Which of the statement(s) given above is/are correct?
(a) Only 1 (b) Only 2 (c) Both 1 and 2 (d) Neither 1 nor 2
55. From which of the following processes is the energy responsible for generation of endogenic
forces derived?
1. Radioactivity 2. Rotational and tidal friction
3. Primordial heat from the origin of the Earth
Select the correct answer using the codes given below.
(a) Only 1 (b) 1 and 3 (c) 2 and 3 (d) All of these
56. Which of the following statements is correct regarding anaerobic respiration?
(a) Anaerobic respiration plays a major role in the global nitrogen, sulphur and carbon cycles
(b) Including muscles and other tissues, all the cells in human bodies respire only aerobically
(c) Anaerobic respiration is used mainly by eukaryotes that live in environments devoid of oxygen
(d) All of the above
57. The fusion reaction is said to the future source of energy. What are the conditions needed to
achieve fusion reaction?
(a) Extremely high pressure and extremely low temperature
(b) Extremely low pressure and extremely low temperature
(c) Extremely high pressure and extremely high temperature
(d) Extremely low pressure and extremely high temperature
58. Which of the following is/are provisions under ‘E-waste (Management and Handlings) Rules,
2011’?
1. It specifies threshold for use of hazardous material including Pb, Hg and Cd.
2. It introduced the concept of ‘Extended Producer Responsibility’ (EPR), with some exceptions.
3. Under EPR, producer is also entrusted with the responsibility to finance the costs involved in
complying EPR.
Select the correct answer using the codes given below.
(a) Only 1 (b) 1 and 2 (c) Only 2 (d) All of these
59. Which of the following statement(s) is/are correct?
1. In India, the constitutional remedy under Article 32 is available only in case of Fundamental Rights,
not in case of rights which follow from some other provision in the Constitution.
2. Both the Supreme Court and High Court can issue the writs of Habeas Corpus, Mandamus,
Prohibition, Certiorari and Quo warranto, for the purpose of enforcement of Fundamental Rights.
Select the correct answer using the codes given below.
(a) Only 1 (b) Only 2 (c) Both 1 and 2 (d) Neither 1 nor 2
70 Practice Set 3

60. Consider the following statements.


1. The office of whip is mentioned neither in the Constitution nor in the rules of the house nor in a
Parliamentary statute.
2. Every political party, whether ruling or opposition has its own whip in the Parliament.
3. Whip is appointed by the Political Party to serve as an assistant floor leader.
Which of the statements given above are correct?
(a) 1 and 2 (b) 2 and 3
(c) 1 and 3 (d) All of these

61. Which of the following factors have caused recent ballooning of India’s trade deficit with China?
1. Low export of iron ore from India. 2. Indian ban on power equipments from China.
3. Protectionist measures adopted by China. 4. Eurozone economic slowdown.
Select the correct answer using the codes given below.
(a) 3 and 4 (b) 1 and 2 (c) 1 and 3 (d) All of these

62. Which of the following is/are true about Double Tax Avoidance Agreements (DTAAs)?
1. It always lowers the annual gross tax revenue generated in the host country.
2. It is in the interest of investing individuals/companies.
3. May encourage round-tripping of money.
Select the correct answer using the codes given below.
(a) Only 1 (b) 2 and 3 (c) Only 3 (d) All of these

63. Casuarina plants are important xerophytic species. Consider the following statements regarding
casurina plants.
1. These plants are confined to Western India and some semi-arid regions of Central India.
2. Roots are one of the principal organ of primary importance and extensively mechanical roots are
there.
3. Roots are thickly covered on the upper layer of the soil.
4. Casuarina plants are used as a tonic and treatment of stomach complaints.
Which of the statements given above are correct?
(a) 1 and 2 (b) 2 and 4
(c) 1, 2 and 3 (d) All of these

64. Ecological succession culminates in climax species. Consider the following statements about
ecological succession.
1. Clement gives the first idea about climax community.
2. Climax stage is comparatively stable stage of succession that led to make a biome.
3. Climax communities are more complex and more diversified.
4. Climatic condition is a deciding factor in establishing climax succession.
Which of the statements given above are correct?
(a) 2 and 3 (b) 1 and 2
(c) 2, 3 and 4 (d) All of these

65. Consider the following statements about the recently launched Jason-3 mission.
1. It is expected to help the USA in better forecasting of hurricane.
2. Jason-3 satellite will examine the human-induced changes on the ocean.
3. It will not extend the time series of ocean surface topography measurement.
Which of the statement(s) given above is/are correct?
(a) Only 1 (b) 2 and 3
(c) 1 and 3 (d) 1 and 2
Stage 2 Improve Your Weaker Areas 71

66. Consider the following statements.


1. The first hour of every parliamentary sitting is slotted for question hour.
2. The zero hour starts immediately after the question hour and lasts until the agenda for the day is
taken up.
3. The questions included in the question hour are starred, unstarred, short notice and long notice
questions.
4. Zero hour is an Indian innovation in the field of parliamentary proceedings.
Which of the statements given above are correct?
(a) 1, 2 and 4 (b) 1, 3 and 4
(c) 2 and 3 (d) All of these

67. President of India can declare National Emergency on the grounds of


1. war 2. external aggression 3. natural disaster
4. armed rebellion 5. imminent danger
Select the correct answer using the codes given below.
(a) 2, 3 and 4 (b) 1, 3, 4 and 5
(c) 1, 2, 4 and 5 (d) 1, 2, 3 and 5

68. Consider the following statements.


1. Varahamihira proved that the Moon rotates around the Earth and the Earth rotates around the Sun in
his book Varahasamhita.
2. Sushruta described the method of operating cataract, stone disease and several other ailments in his
book Sushrutasamhita.
3. Charaka describes various fevers and diseases in his book over medicine known as Charakasamhita.
Which of the statements given above are correct?
(a) 1 and 2 (b) 2 and 3
(c) 1 and 3 (d) All of these

69. The Gross National Happiness (GNH) index differs or is similar to HDI. Which of the following
respects?
1. HDI focuses mainly on the non-material side of development similar to the GNH.
2. GNH includes a responsive, transparent and accountability government as an important indicator of
development unlike the HDI.
3. HDI values are affected by the levels of ecological degradation and imbalances.
Select the correct answer using the codes given below.
(a) Only 2 (b) 1 and 2
(c) Only 1 (d) All of these

70. The Government of India has come up with a new Consumer Price Index (CPI). With respect to
the new Consumer Price Index, consider the following statements.
1. Consumer Price Index is released every month.
2. The common base year for CPI is 2012.
3. Consumer Price Index represents the wholesale price of goods paid by the households for a basket of
goods and services.
4. More weightage is given to the manufacturing items under CPI.
5. Housing forms one of the five major groups for which Consumer Price Index is available for both
rural and urban areas.
Which of the statement(s) given above is/are correct?
(a) Only 1 (b) 1, 3 and 5
(c) 2, 4 and 5 (d) 1, 2, 3 and 4
72 Practice Set 3

71. Consider the following statements with reference to herbivores.


1. Due to the herbivore’s ability to survive solely on tough and fibrous plant matter, they are termed the
primary consumers in the food cycle (chain).
2. Tamaraw and Giant eland are not considered as herbivores.
3. Herbivores principally eat autotrophs.
Which of the statement(s) given above is/are correct?
(a) 1 and 2 (b) 1 and 3
(c) Only 3 (d) 2 and 3

72. “Series of protected areas linked through a global network, intended to demonstrate the
relationship between conservation and development”. This definition refers to which of the
following?
(a) National Park (b) Biosphere Reserve
(c) Wildlife Sanctuary (d) Natural Heritage Site

73. Which of the following statement(s) is / are correct with respect to cosmic rays?
1. They are made up of only positively charged particles.
2. The major source of cosmic rays in universe is Sun.
3. They cause ozone depletion.
Select the correct answer using the codes given below.
(a) 1 and 2 (b) 2 and 3
(c) Only 3 (d) All of these

74. Who cannot participate in the election of the President?


1. Nominated members of both the houses.
2. Nominated members of the Legislative Assembly.
3. Nominated and elected members of State Legislative Council.
4. Nominated members of the Legislative Assemblies of Delhi and Puducherry.
Select the correct answer using the codes given below.
(a) 1, 2 and 3 (b) 1, 3 and 4
(c) All of these (d) None of these

75. The universal declaration of human rights was adopted by the UN General Assembly in 1948.
This declaration consists of 30 Articles which can be divided into four parts. Consider these
statements about the four parts.
1. The first two articles contain the basic principles underlying all human rights.
2. Articles 3 to 21 consist of civil and political rights.
3. Articles 22 to 27 contain economic, social and cultural rights.
4. The last 3 articles specify the context within which all the human rights are to be enjoyed.
Which of the statements given above are correct?
(a) 1, 2 and 3 (b) 1, 3 and 4
(c) 2 and 4 (d) All of these

76. Arrange the following mountain ranges from North to South.


1. Karakoram ranges 2. Ladakh range
3. Zaskar range 4. Pir Panjal range
Codes
(a) 2, 1, 3, 4 (b) 3, 2, 1, 4
(c) 1, 2, 3, 4 (d) 1, 3, 2, 4
Stage 2 Improve Your Weaker Areas 73

77. Consider the following statements about the Islands the Arabian sea.
1. The islands of the Arabian sea include Lakshadweep and Minicoy.
2. The entire island group is built of coral deposits.
3. The entire group of islands is broadly divided by the 10° channel, North of which is Canannore Island
and to the South is the Amini Island.
4. The islands of this archipelago have storm beaches.
Which of the statements given above are correct?
(a) 1, 2 and 3 (b) 1, 3 and 4 (c) 1, 2 and 4 (d) 2, 3 and 4
78. Which of the following statements about generic drugs are correct?
1. A generic drug is a pharmaceutical product that is manufactured without a license from the
innovator company and marketed after the expiry date of the patent or other exclusive rights.
2. Generic drugs are subject to the same regulations over manufacturing, packaging, testing and quality
standards, as their patented / branded equivalent.
3. Prices of the branded/patented drugs are generally quite high, due to the temporary monopoly over
the product, bestowed by the patent.
4. At ‘Jan Aushadhi stores’, where quality generic medicines, which are equivalent to the expensive
branded drugs, in terms of their potency and efficacy are sold at cheaper prices.
Select the correct answer using the codes given below.
(a) 1 and 2 (b) 2 and 3 (c) 1, 3 and 4 (d) All of these
79. Consider the following statements about the architecture of Medieval Mosques.
1. The pulpit (mimbar) of a mosque is meant for the imam who leads the prayer.
2. The minaret or tower which was built for mere architectural purpose, later was utilised by ‘muazzin’
to call the followers for prayers.
3. The prayer hall pointed towards West of India.
Which of the statements given above are correct?
(a) 1 and 2 (b) 2 and 3 (c) 1 and 3 (d) All of these
80. Which of the following are the techniques adopted to stop soil erosion in dry lands?
1. Terrace farming 2. Contour ploughing 3. Afforestation 4. Shelter belts
5. Plugging gullies
Select the correct answer using the codes given below.
(a) 1, 2 and 3 (b) 3, 4 and 5 (c) 1, 2 and 4 (d) 2, 3 and 4
81. Alluvial fans are formed when streams flowing from higher levels break into foot slope plains of
low gradients.
Which one of the following could be the reason?
(a) Load content of the stream reaches above saturation point
(b) Load that is being carried by stream over the mountain slopes becomes too heavy to carry over
gentler gradients
(c) Due to natural vegetation at mountain foot
(d) None of the above
82. Consider the following statements.
1. A person who is not a member of either House of Parliament can be appointed as the Prime Minister.
2. A person must prove his majority in the Lok Sabha before he is appointed as the Prime Minister.
3. He must be a member of any of two Houses of Parliament.
4. Article 75 says only that the Prime Minister shall be appointed by the President.
Which of the statements given above are incorrect?
(a) 1 and 2 (b) 2 and 4 (c) 1, 3 and 4 (d) 3 and 4
74 Practice Set 3

83. Consider the following statements regarding rights of women enshrined in the Constitution of
India.
1. As per Article 51A(e) it shall be the duty of every citizen of India to renounce the practices derogatory
to the dignity of women.
2. As per Article 39 (e) the state is required to ensure that the health and strength of women workers are
not abused and that they are not forced by economic necessity to enter avocations unsuited to their
strength.
3. Protection of women form Domestic Violence Act (2005) is a comprehensive legislation to protect
women from all forms of domestic violence.
Which of the statements given above are correct?
(a) 1 and 2 (b) 2 and 3 (c) All of these (d) None of these

84. Consider the following statements.


1. As India implements Goods and Services Tax (GST), it may be appropriate to calculate National
Income at current prices.
2. Developed nations generally calculate their National Income at market cost at current prices as they
have a uniform taxation regime.
Which of the statement(s) given above is/are correct?
(a) Only 1 (b) Only 2 (c) Both 1 and 2 (d) Neither 1 nor 2

85. Consider the following statements with reference to the prohibition of employment as manual
scavengers and their Rehabilitation Act, 2013.
1. Offences under the act are non- cognisable and bailable.
2. It seeks to rehabilitate manual scavengers and provide for their alternative employment.
3. The act prohibits the employment of manual scavengers, the manual cleaning of sewers and septic
tanks without protective equipment and the construction of insanitary latrines.
4. The act has a wider scope and higher penalties than the 1993 Act.
Which of the statements given above are correct?
(a) 1 and 3 (b) 2 and 3 (c) 2, 3 and 4 (d) 2 and 4

86. SAFAR [System for Air Quality Forecasting and Research]. It is computerised system developed
by Indian Institute of Tropical Meteorology (IITM), Pune with the help of Indian Meteorological
Department (IMD). Consider the following statements regarding SAFAR.
1. It provides data on Air Quality Index, UV index and dynamic city pollution map.
2. Quality of air can forecast 24-72 hours in advance.
3. Operational in Delhi, Pune and Ahmedabad.
Which of the statements given above are correct?
(a) 2 and 3 (b) 1 and 2 (c) All of these (d) None of these

87. Consider the following statements regarding 3D printing.


1. 3D printing is also called additive manufacturing.
2. It can print only on plastic or polyethylene material.
3. The technology is still in development stage and is not commercially available.
Which of the statement(s) given above is /are correct?
(a) Only 1 (b) 1 and 2 (c) 2 and 3 (d) All of these

88. Which of the following pairs are correctly matched?


1. Tuti Nama : Ziya Nakhshabi
2. Tughlaq Nama : Amir Khusarow
3. Tarikh-i-Firoz Shahi : Hasan
4. Fawaid-ul-Fuad : Barani
Codes
(a) 1 and 2 (b) 3 and 4 (c) 1, 2 and 4 (d) 1, 3 and 4
Stage 2 Improve Your Weaker Areas 75

89. The inner walls of the small intestine have thousands of finger like structures called villi.
Consider the following statements about these villi.
1. The villi decrease the surface area for absorption of the digested food.
2. Each villus has a network of thin and small blood vessels close to its surface. The surface of the villi
absorbs the digested food materials.
3. The absorbed substances are transported via the blood vessels to different organs of the body where
they are used to build complex substances such as proteins.
Which of the statements given above are correct?
(a) 1 and 2 (b) 2 and 3 (c) 1 and 3 (d) All of these

90. Which of the following statements is not true about Ocimum Tenuiflorum (Tulasi run)?
1. It is mentioned in the Charaka Samhita an ancient Ayurvedic Text.
2. It is considered to be an adaptogen, balancing different processes in the body.
3. It also acts as insect repellant.
Select the correct answers using the codes given below.
(a) Only 3 (b) 1 and 2 (c) 2 and 3 (d) None of these

91. Constitution of India provides for which types of responsibility to council of ministers.
1. Legal responsibility 2. Collective responsibility
3. Social responsibility 4. Individual responsibility
5. Economical responsibility
Select the correct answer using the codes given below.
(a) 1 and 4 (b) 1 and 5
(c) 2 and 3 (d) 2 and 4

92. Consider the following statements.


1. The Government of India Act, 1919, introduced separate and discriminatory electrorate.
2. The Indian Council Act, 1909, raised the number of additional members to the maximum at 60.
Which of the statement(s) given above is /are correct?
(a) Only 1 (b) Only 2 (c) Both 1 and 2 (d) Neither 1 nor 2

93. Consider the following statements about The National Heritage City Development and
Augmentation Yojana (HRIDAY).
1. The Union Government launched on 26th January, 2015 with a focus on holistic development of
heritage sites and the National Advisory Committee (NAC) is the apex advisory body for this scheme.
2. It aims to bring urban planning, economic growth and heritage conservation together for heritage
cities.
3. The criterion for selection of the cities is their rich heritage and cultural history.
4. The eleven heritage cities have been identified namely Amravati, Gaya, Dwaraka, Badami, Puri,
Amritsar, Ajmer, Kanchipuram, Warrangal, Mathura and Varanasi.
Which of the statements given above are incorrect?
(a) 1 and 4 (b) 3 and 4 (c) 2 and 3 (d) 1, 3 and 4

94. Consider the following statements regarding the Dolphin - Susu.


1. It is a type of dolphin which can thrive both in freshwater and saltwater.
2. It occurs in rivers of both Ganga and Brahmaputra.
3. It is also called as blind river dolphin.
4. The presence of Susu is an indication of the health of the river.
Which of the statements given above are correct?
(a) 1 and 2 (b) 1, 3 and 4 (c) 3 and 4 (d) 2, 3 and 4
76 Practice Set 3

95. Consider the following statements regarding the election of Vice-President of India.
1. A person should not hold any office of profit under the Union Government or any State Government
or any local authority or any other public authority.
2. A sitting President or Vice President of the Union, the Governor of any state is not deemed to hold
any office of profit and hence, qualified for being a candidate for Vice President.
3. The nomination of a candidate for election to the office of Vice President must be subscribed by at
least 20 electors as proposers and 20 electors as seconders.
4. Every candidate has to make a security deposit of ` 15000 in the RBI.
Which of the statements given above are correct?
(a) 1, 2 and 3 (b) 1, 3 and 4 (c) All of these (d) None of these

96. Consider the following statements about nutrients.


1. Fats give much more energy as compared to the same amount of carbohydrates.
2. Proteins are needed for the growth and repair of our body and they are called as building blocks of
body.
3. Vitamins give much more energy than both fats and carbohydrates combined.
Which of the statement(s) given above is /are correct?
(a) 1 and 3 (b) 2 and 3 (c) Only 2 (d) 1 and 2

97. Consider the pairs of critically endangered species and threat to it in India and find the wrong
pair.
(a) Sociable lapwing : Conversion of habitat to arable land and settlement
(b) Siberian crane : Pesticide pollution in wetland drainage
(c) Namdapha flying squirrel : Destruction of habitat
(d) The Leatherback Foturtle : Destruction of habitat and high sea fishing operation

98. Which of the following statement(s) regarding relative humidity is/are correct?
1. Coastal regions have high relative humidity as compared to desert areas.
2. Relative humidity of 100% indicates that dew point is equal to the current temperature.
3. Relative humidity of a region is always above 50% .
Select the correct answer using the codes given below.
(a) Only 1 (b) 1 and 2 (c) 2 and 3 (d) All of these

99. Which of the following statement(s) is/are correct with respect to the Polar Satellite Launch
Vehicle (PSLV).
1. It is the first operational launch vehicle of ISRO. PSLV is capable of launching 1600 kg satellites in
620 km sun-synchronous polar orbit and 1050 kg satellite in geo-synchronous transfer orbit.
2. PSLV has four stages using solid and liquid propulsion systems alternately. The first stage is one of
the largest solid propellant boosters in the world and carries 139 tonnes of propellant.
3. So,far PSLV has launched 87 satellites.
Select the correct answer using the codes given below.
(a) Only 1 (b) Only 2 (c) All of these (d) None of these

100. For Forest Management in India, certain initiatives have been taken up by the government. Find
the correct ones in this regard?
1. A centrally sponsored intensification of forest management scheme was taken up.
2. All the North-Eastern States Jammu and Kashmir, Himachal Pradesh and Uttarakhand has given 75%
central financial assistance for forest management.
3. Recently incentivised scheme of Joint Forest Management Committees (JFMCs) are set up to involve
local people in protection of forest.
Select the correct answer using the codes given below.
(a) 1 and 3 (b) 1 and 2 (c) 2 and 3 (d) All of these
Stage 1 Know the Trend 77
78 Practice Set 1

PRACTICE SET 3 OVERALL ANALYSIS


Stage 2 Improve Your Weaker Areas 79

Answers with Explanations


1. All these statements are correct, cryogenic engines are (3) When the legislatures of two or more states pass
capable of running at a very low temperature. The fuel used is resolutions requesting the Parliament to enact laws on a
liquid hydrogen and liquid oxygen. matter in the State List, then the Parliament can make
Statement 3 is also correct as, to make liquid hydrogen, the laws for regulating that matter. A law so enacted applies
minimum temperature required is -253°C and for liquid oxygen is only to those states which have passed the resolutions.
-186°C . Statement 4 is also correct. However, any other state may adopt it afterwards by
passing a resolution to that effect in its legislature.
2. A ‘rasa’ denotes an essential mental state and is a dominant
(4) The Parliament can make laws on any matter in the
emotional theme of a work of art. Bharata Muni enunciated eight
State List for implementing the international; treaties,
rasas.
agreements and conventions.
Karuna rasa = Compassion, tragedy
(5) When the President’s rule is imposed in a State, the
Deity = Yama
Parliament becomes empowered to make laws with
Colour = Grey
respect to any matter in the State List in relation to that
Veera rasa = Valour and heroic State.
Deity = Indra
So, all of the above statements are correct.
Colour = Golden
Shringar rasa = Love, attractiveness 6. The Constitution ensures that there is uniformity of
representation in two aspects. One is between the different
Deity = Vishnu
states and another is between the different constituencies in
Colour = Light green
the same state.
Adbhuta rasa = Wonder, amazement
The expression population means the population as
Deity = Brahma
ascertained at the preceding census of which the relevant
Colour = Yellow figures have been published. After every census, a
3. The Ministry of Environment and Forest has rationalised eight readjustment is to be made in allocation of seats in the Lok
Centrally Sponsored Schemes (CSS) of the 11th Five Year Plan to Sabha to the states and division of each state into territorial
5 in the 12th Five Year Plan by suitably clubbing the schemes. Taj constituencies.
Protection Scheme is now integrated with Conservation of 7. ● Sundaland hotspot covers the Western half of the

Natural Resources and Ecosystems and Project Elephant is now Indo-Malayan archipelago in South-East Asia.
integrated with ‘Wildlife Management’ Scheme. ● Wallacea hotspot encompasses the central Islands of

Indonesia East of Java, Bali and Borneo, West of New


4. The Convention on Biological Diversity is one key agreement
Guinea and Timor Leste.
adopted under Earth Summit in 1992. In 2012, the conference of
● Succulent Karoo hotspot is stretching along the
CBD was held in Hyderabad, which was also marked the 10th
Atlantic Coast of Africa, from South Africa into
anniversary of Sustainable Development Summit, 20th
Southern Namibia.
anniversary of Rio Summit and 40th anniversary of Stockholm
● The Cerrado hotspot is located in Brazil, South
Conference. Cartagena Biosafety Protocol was negotiated on the
America.
Aegis of CBD which deals with Living Modified Organisms
(LMOs). India is a party to CBD and has set up National 8. Benefits from the FTA can possibly have a positive
Biodiversity Authority at national level, State Biodiversity boards at impact on India’s growth rate, help to improve
state level and Biodiversity Management Committees (BMC) at manufacturing sector that can lead to employment creation
local level. in both countries. However, a country may enter into a Free
Trade Agreement or customs union granting more
5. Parliament can make law on the subjects of State list under favourable treatment to the participating states than to the
the five extraordinary circumstances as follows other WTO members, if it observes certain conditions
1. When Rajya Sabha passes a resolution that it is necessary in stipulated in the relevant provisions of the WTO
the national interest that Parliament should make laws on a agreements, to ensure the complementarity of the FTA with
matter in the State List, then the Parliament becomes the WTO system.
competent to make laws on that matter. Such a resolution 9. The Foreign Direct Investment (FDI) not come under the
must be supported by two-thirds of members present and current account. It is the capital account includes Foreign
voting. Such a resolution remains in force for one year. Direct Investment (FDI), portfolio and other investments,
2. During a national emergency the Parliament acquires the plus changes in the reserve account. Hence, foreign
power to legislate with respect to matters in the State List. The investments in the National Manufacturing and Investment
laws become in operative on the expiration of six months after Zones (NMIZs) do not affect India’s Current Account Deficit
the emergency has ceased to operate. (CAD).
80 Practice Set 3

10. Kushan empire gave rise to several school of art i.e. ● Linking contributions of Science, Research and Innovation
Central Asian, Gandhara and Mathura. These were also system with the inclusive economic growth agenda and
influenced by Buddhism. combining priorities of excellence and relevance.
In Gandhara period, images and hairstyle of Buddha was ● Creating an environment for enhanced private sector
made in the Graeco Roman style. The influence of this art participation in Research and Development. Aims at 2% of
form spread to Mathura, which was also having its own art GDP in next 5 years from current 1%.
form. ● Enabling conversion of Research and Development output
Mathura produced highly artistic images of Buddha. It is also with societal and commercial applications by replicating
famous for headless erect statue of Kanishka. It also hitherto successful models, as well as establishing of new
produced some stone images of Mahavira. Public Private Partnership (PPP) structures.
● Seeking Science and Technology based high risk innovation
11. The Eastern hills or mountains have their general through new mechanisms.
alignment from the North-South direction. They are known by
● Fostering resource optimised cost-effective innovation
different local names such as Patkai Bum, Naga hills, Mizo
across size and technology domains.
hills or Lushai hills.
● Triggering in the mindset and value systems to recognise
These hills are low and are inhibited by numerous tribal respect and reward performances which create wealth from
groups practising Jhum cultivation. Most of these ranges are Science and Technology derived knowledge.
separated by small rivers. The State of Mizoram which is also
● Creating a robust national innovation system. 2010-20 has
known as the the ‘Molassis basin’ which is made up of soft been declared as ‘‘decade of innovation’’.
unconsolidated deposits. Two rivers of Mizoram and Manipur
are the tributaries of the Barak river, which in turn is the 15. The Defence Research and Development Organisation
tributary of Meghna river. (DRDO) successfully test-fired Nag, anti-tank missile at Mahajan
Field Firing Range in Rajasthan. It is a third generation
12. (d) A person under preventive detention does not have any fire-and-forget missile and can be fired anti-tank missile. It can
such rights which a person in custody normally has. The
be fired during night and day as well. Besides Helina, a
State Legislatures can make laws as preventive detention is
helicopter version, it has land, air-launched and man portable
not only concerned with matters of National Security, but also
version. To carry the land version, DRDO developed NAMICA
state security.
(Nag Missile Carrier) for the army.
13. Desirable characters can be introduced by genetically
16. The Indus Water Treaty is a water sharing treaty between
modifying the crops too. One way of incorporating desirable
Pakistan and India, brokered by the World Bank (then the
characters into crop varieties is by hybridisation. So, the
International Bank for Reconstruction and Development).
Statement 1 is incorrect. Hybridisation refers to crossing
between genetically dissimilar plants. Under the treaty, the water of Eastern rivers (Ravi, Beas and
Sutlej and their tributaries) of the Indus river system are
This crossing may be intervarietal (between different
allocated to India. India is under obligation to let flow the water
varieties), interspecific (between two different species of the
of the Western rivers except for the mentioned uses. Both sides
same genus) or intergeneric (between different genera).
are required to exchange information related to river flows
Another way of improving the crop is by introducing a gene
observed by them, not later than 3 months of their observation
that would provide the desired characteristic. This results in
and to exchange specified information on agricultural use every
genetically modified crops.
year.
14. All the given statements are the objectives of Science, 17. SN Bannerjee was founder of Indian National Association in
Technology and Innovation (STI) Policy 2013. 1876 with Anand Mohan Bose. He had already decided the
The key features of the STI Policy 2013 are as follow dates of session of 1885 of Indian National Association, thats
● Promoting the spread of scientific temper amongst all why, he was unable to attend the first session of Indian National
sections of society. Congress. He merged his party with Congress considering the
● Enhancing skills for applications of science among the common objective of both. WC Bannerjee was first President of
young from all social sectors. Congress in 1885. Dadabhai Naoroji gave the name of
● Making careers in science, research and innovation Congress to Indian National Congress from union. He was the
attractive enough for talented and bright minds. second President of Congress.
● Establishing world class infrastructure for Research and 18. P-stands for Primary waves while S stands for Secondary
Development for gaining global leadership in some select waves. P-waves are like sound waves and longitudinal in nature
frontier areas of science. while S-waves are like light waves and transverse in nature.
● Positioning India among the top five global scientific P-waves reach the Earth prior to S-waves i.e. P-waves have
powers by 2020 (by Increasing the share of global more speed than S-waves. P-waves can travel through all
scientific publications from 3.5% to over 7% and matters i.e. gaseous, liquid and solid while S-waves can travel
quadrupling the number of papers in top 1% journals from only through solid materials.
the current levels).
Stage 2 Improve Your Weaker Areas 81

19. Central economic planning started with the setting up of ● Very small or restricted population of fewer than 50 mature
the National Planning Committee (1938). Foreign individuals.
investment was discouraged because of the colonial ● High probability of extinction in the wild.
experience of drain of wealth and the stagnant performance All these criteria are decided by IUCN which published Red Data
of the economy under the colonial rule. It grew by 0% list of these species.
between 1600 and 1870 and a meagre 0.2% between 1870 27. A biodiversity hotspot is a bio-geographic region with a
to 1947. significant reservoir of biodiversity that is under threat from
20. In order to strengthen the Panchayati Raj System and humans. The concept of biodiversity hotspot was originated by
also to address critical gaps that constrain it, Ministry of Norman Myers in 1990. These are the natural ecosystems with a
Panchayati Raj has formulated the scheme, Rajiv Gandhi high diversity of locally endemic species, which are not found or
Panchayat Sashaktikran Abhiyan (RGPSA) which will be are rarely found outside the region. To qualify as a biodiversity
implemented during the 12th Five Year Plan period. The hotspot, a region must contain atleast 0.5% or 1500 species of
funding of RGPSA for state plans is envisaged on a 75:25 vascular plants as endemic and it has to have lost atleast 70% of
sharing basis by the Central and State Governments its primary vegetation. In India, three regions as Eastern
respectively. For North-Eastern states, the ratio will be Himalayas, Indo-Burma region and Western Ghats are
90:10. Thus, all the other statements except Statement 2 are biodiversity hotspots.
correct.. 28. Lightning is sudden electrostatic discharge during electric
storm between electrically charged region of cloud, between that
21. The famous Dal lake is situated on the Eastern side of
cloud and other cloud or between a cloud and the ground. The
Srinagar. It is considered an Oxbow lake of the Jhelum river.
charge region, within the atmosphere temporarily equalise
Sedimentation is main problem, it is suffering from. Efforts
themselves through a lightning flash. Lightning can strike
are being made to make it sediment free. It is decorated by
telephone cords, electrical wires and metal pipes. During a
lotus flowers, water lilies and water chestnut.
thunderstorm contact with these should be avoided. It is safer to
22. Statement 1, 2 and 3 are correct as, superconductivity is use mobile phones and cordless phones. However, it is not wise
a phenomenon observed in several metals and ceramic to call up a person who is receiving your phone through a wired
materials, also it shows exactly zero electrical resistance. It phone. Except, Statement 2 all the other statements are correct.
also shows expulsion of magnetic fields. Statement 4 is 29. Statement 1 is correct, Brown label ATMs are the banks
incorrect. It is characterised by the Meissner effect, in which outsource the ATM operations to a third party.
complete ejection of magnetic field lines from the interior.
RBI is not involved directly in issuing license/permission. The
23. Groundwater sources cannot always be free of outsourcing companies have contractual obligation with
contaminants. However, it usually contains lesser respective banks.
contaminants. Except Statement 1, all the other statements ATM has the logo of the bank which has outsourced this work.
are correct for example, the groundwater of Ganges Plan is Hence, Statements 2 and 3 are incorrect.
severely polluted at regions with naturally occurring arsenic. 30. Soil texture refers to proportion of soil particles of various
24. The hydrogen bomb is based on thermonuclear fusion sizes is sand, silt and clay. Soil texture in turn influences the ease
reaction. A nuclear bomb based on the fission of uranium or with which soil can be worked, the amount of water and air it
plutonium is placed at the core of the hydrogen bomb. This holds. Soil structure refers to the way in which individual particles
nuclear bomb is embedded in a substance which contains of sand, silt and clay are aggregated and therefore the
deuterium and lithium. When the nuclear bomb (based on arrangement of soil pores between them.
fission) is detonated, the temperature of this substance is 31. Wavell Plan made by the then Viceroy of the India. It tried to
raised to 107K in a few microseconds. The high temperature solve the matter of handing over the power to Indians. His
generates sufficient energy for the light nuclei to fuse and a proposals were not accepted to Muslim league as they
devastating amount of energy is released. considered themselves as the sole representators of Muslims
25. Pushyamitra founded the Sunga dynasty by killing the and Muslim representatives of Congress were not acceptable to
last Mauryan ruler Brihadratha and the last Sunga ruler them. All members of the Central Executive Council barring the
Devabhuti was killed by his Minister Vasudeva Kanva who Governor-General and the Commander-in-Chief were to be
founded Kanva dynasty and last ruler of Kanva dynasty Indians. Defence of Indians was to be in the hands of a British
Susharma was killed by Andhra Simuka who established General till power was transferred to the Indian hands.
Satavahana dynasty. 32. The Great Indian Desert lies to the North-West of the Aravali
26. A taxon is critically endangered when the best available hills. It is a land of undulating topography dotted with longitudinal
evidence indicates that it meets any of the following criteria dunes and barchans. Thar region receives a very low rainfall and
● Population have declined or will decrease, by greater has arid climate. It is believed that during the Mesozoic era, this
than 80% over the last 10 years or 3 generations. region was under the sea. This can be identified by the evidence
● Have a restricted geographical range. available at Wood Fossils park at Aakal and Marine deposits
● Small population size of less than 250 individuals and around Brahmsar near Jaisalmer. The approximate age of the
continuing decline at 25% in 3 years or 1 generation. wooden-fossils is estimated to be 180 million years.
82 Practice Set 3

33. Harshavardhan one of the greatest ruler of post Gupta 39. The Kashmir or North-Western Himalayas comprise of
period, was a noted author on his own merit. He wrote three ranges such as Karakoram, Ladakh Zaskar and Pir Panjal.
Sanskrit plays named as Nagananda, Ratnavali, and The North-Western part of the Kashmir Himalayas is a cold
Priyadarshika. His court poet Banabhatta wrote Harshacharita desert, which lies between the Great Himalayas and
an account of Harsha’s rise to power and Kadambari. His other Karakoram ranges. The world famous valley of Kashmir and
court poets Maurya and Bhartrihari. Maurya wrote famous Dal lake lies between the Great Himalayas and Pir
Mauryashataka and Bhartrihari wrote Vakyapadiya. Panjal range. Kashmir Himalayas are famous for Karewa
34. There were a large number of divinities in the Rigveda. The formations, which is a glacial clay and is useful in cultivation
most important of them was Indra, also known as Purandra. He of zafran, a local variety of saffron.
was considered a warlord. Second most important position was 40. Infanticide was declared illegal by Bengal Regulation XXI
given to Agni who was a fire God. The next most important of 1795 and Regulation III of 1804. But, the practises of
position was given to Varun who personified water. The other infanticide still continued which was suppressed by William
important divinty were Soma, Maruts, Saraswati, Aditi and Bentinck. Bentinck became Governor-General in 1828 and
Usha. This also suggests that women divinities were also was first Governor-General of India. He continued till 1835.
present from that time only. Saraswati was actually devoted to He was chief architect in abolition of Sati and suppression of
river Saraswati. The women divinities were present but not Thugee. The Doctrine of Lapse, also known as Annexations
prominent considering the patriarchal society of the period. of Peace was devised by Dalhousie. Satara was the first
Dachigam National Park is natural habitat for Kashmiri princely state to annexed followed by Jatipur, Sambalpur,
35. ●

stag. Nagpur and Jhansi, Awadh etc.


● Nandankandan National Park in Odisha serves as the 41. Saw-scaled viper is part of big four dangerous snakes of
natural habitat of white tigers. India, and not Indian Python, which is higly arboreal and
● Bhitarkanika in Odisha is a hotspot of biodiversity. It is largest specie. The King Cobra is the most poisonous snakes
home to largest population of giant salt water crocodile in in India and Krait is the most deadly and dangerous snake of
India. India.
36. When iron tablet is taken for the first time, the body may find 42. Tuberculosis, Bubonic plague and Cholera are caused by
it little difficult to digest and symptoms such as stomach ache bacteria. Hepatitis-B, Polio, Measles and Chicken pox are
and nausea may occur. However, if taken after food, the caused by virus.
absorption will be little low but stomach ache and nausea will
43. The word, Nautanki, comprises two words, ‘nau’ meaning
not occur. These side effects will eventually disappear once the
‘nine’ and tank referring to a ‘silver coin weighing four grams’.
tablet is regularly taken for a few weeks as the body adjusts to
Before the advent of cinema in India, it was the most popular
the iron tablets. The Weekly Iron and Folic Acid
form of entertainment prevalent in these areas. Usually, a
Supplementation (WIFS) programme is currently reaching out to
Nautanki consists of mythological dramas with interludes of
13 crore school going girls and boys (Class VI - XII) and
folk songs and dances. In many areas, especially in the
out-of-school adolescent girls in government/ aided and
regions of Uttar Pradesh, the Nautanki is mainly performed by
municipal schools and Anganwadi Centres across all states in
female dance performers.
India.
44. A medicine is any substance that is designed to prevent
37. The power of the legislature to formulate policy is real based
or treat a diseases and drug is designed to produce a
as it is in the Constitution, but has lost out to the executive in
specific reaction inside the body. Statement 1 is incorrect as
recent times in a Parliamentary form of government. In the
the drugs are chemicals of low molecular masses (~100 -
course of approving legislation, the Parliament performs by
500 n). These interact with macromolecular and produce a
deliberating, scrutinising, criticising and publishing government
biological response, so Statement 2 is also incorrect.
policies and their consequences for the public on the floor of
Statement 3 and Statement 4 are correct if drugs taken in
the house. Developing countries lack a strong bureaucratic
base and the executive plays a larger role on policy formulation. doses higher than those recommended are potential poisons
It is because of greater concentration of power in government and interact with biomolecules such as carbohydrates, lipids,
heads coupled with less responsiveness to policies and the proteins and nucleic acids.
legislature. 45. First Anglo-Mysore War saw Hyder Ali to maintain his
38. All of the given statements in the regard of PMO are correct. strength against the Britishers. They suffered and defeat at
Further, the PMO is an extra constitutional body that plays an the hand of Marathas, but with the alliance of Nizam of
important role in the process of policy formulation at the top Hyderabad, were able to maintain their sovereignty against
level in the GOI. The office houses the Prime Minister and few Britishers. Treaty of Mangalore concluded the Second
selected officers of Indian Civil Service who work with him to Anglo-Mysore War. This war saw the death of Hyder Ali and
manage and coordinate government and his office. The Prime coronation of Tipu Sultan. Treaty of Seringapatam was signed
Minister through his office coordinates with all ministers in the between Tipu Sultan and Britishers and Mysore has to
Central Union Cabinet, minister of independent charges and surrender almost half of his territory. The Fourth and last
governors and ministers of State Government, perform the Anglo-Mysore War saw the demise of Tipu Sultan and full
functions. annexation of Mysore by East India Company.
Stage 2 Improve Your Weaker Areas 83

46. All the given options are the methods of soil conservation. 52. Although, Fly ash offers environmental advantages, it also
Mulching The bare ground between plants is covered with a improves the performance and quality of concrete. Fly ash
layer of organic matter like straw. It helps to retain soil affects the plastic properties of concrete by improving
moisture. workability, reducing water demand, reducing segregation and
Contour Barriers Stones, grass, soil are used to build bleeding, and lowering heat of hydration. Fly ash increases
barriers along contours. Trenches are made in front of the strength, reduces permeability, reduces corrosion of reinforcing
barriers to collect water. steel, increases sulphate resistance and reduces
alkali-aggregate reaction. Fly ash reaches its maximum
Rock Dam Rocks are piled up to slow down the flow of water.
strength more slowly than concrete made with only Portland
This prevents gullies and further soil loss.
cement.
Intercropping Different crops are grown in alternate rows and
are sown at different times to protect the soil from rain wash. 53. The 11th Plan document mentions that the Plan will aim for
Shelter Belts In the coastal and dry regions, rows of trees are inclusive growth by introducing National Urban Health Mission
planted to check the wind movement to protect soil cover. (NUHM), which along with National Rural Health Mission
(NRHM) will form the Sarva Swasthya Abhiyaan. The Union
47. Deforestation is the removal of a forest or stand of trees Cabinet gave its approval to launch a National Urban Health
where the land is thereafter converted to a non-forest use. All Mission (NUHM) as a new sub-mission under the over-arching
the given statements regarding deforestation are correct. National Health Mission (NHM). Accredited Social Health
48. Deltas are wetland that form as rivers that empty their Activist (ASHA) are community health workers instituted by
water and sediment into another body of water, such as an Government of India’s Ministry of Health and family welfare as
ocean, lake or another river. Delta formation requires past of National Rural Health Mission. The mission began in
considerable erosion in upper course of the river and shallow 2005.
sea at the mouth of the river. Presence of ocean current may 54. Rajasthan style of paintings combined the Mughal forms
wash out the deposits; hence are not favourable for delta and styles with earlier themes and traditions of Western India or
formation. Existence of lake in course of river too does not Jain style of painting. Now, paintings start to include hunting
contribute in delta formation. and court scenes alongwith mythological themes for example
49. Due to Civil War in America, the factories of England were the Barahmasa or the ragas etc.
lacking in supply of raw material. So, they encourage the 55. The energy responsible for generation of endogenic forces
plantation of cotton in Deccan. This led to commercialisation is mostly generated by radioactivity, rotational and tidal friction
of agriculture. and primordial heat from the origin of the Earth. This energy is
This has very positive impact initially that farmer started to due to geothermal gradients and heat flow from within induces
earn cash. This was help of providing loan to farmers as the diastrophism and volcanism in the lithosphere.
crop needed investment in start. Britisher gave instructions to 56. Anaerobic respiration plays a major role in the global
moneylenders to provide the same to farmers. nitrogen, sulphur and carbon cycles through the reduction of
Now, once war was over, the Britisher again moved to cottons the oxyanions of nitrogen, sulphur and carbon to more reduced
of America due to their superior quality. Farmers were unable compounds. Dissimilatory denitrification is the main route by
to understand this and they continued cash crops. Now, they which biologically fixed nitrogen is returned to the atmosphere
had neither sufficient food nor money. They were also loaded as molecular nitrogen gas.
with loan burden. This situation could only lead to riot, which
57. In nuclear physics, nuclear fusion is a nuclear reaction in
occurred in 1875.
which two or more atomic nuclei collide at a very high speed
50. Statement 1 is correct, a laser consists of a gain medium and join to form a new type of atomic nucleus. During this
to supply energy to it. Statement 2 is incorrect, the gain process mass is not conserved because some of the mass of
medium is a material with properties that allow it to amplify the fusing nuclei is converted to photons (energy). Fusion is the
light by stimulated emission. Statement 3 is correct with the process that powers active or ‘main sequence’ stars. For a
aspect of laser. Statement 4 is incorrect, laser has the fusion reaction to occur, extremely high pressure and extremely
properties like, high monochromaticity, coherence and high temperature is required.
directionality.
58. The E-waste (Management and Handlings) Rules, 2011,
51. ( Vinayak Damodar Savarkar was a freedom fighter who came into effect from April, 2012. Rules aim at reduction in the
considered the War of 1857 as the First War of use of hazardous substances in electrical and electronic
Independence. He was instrumental in the formation of equipment by specifying threshold for use of hazardous
Abinav Bharat. He was also associated with India House. For material including Pb, Hg and Cd. It introduced concept of
his association with Indian house, he was given two lifetime EPR, however with some exceptions such as lead acid
imprisonment of 55 years. He was sent to Cellular Jail in batteries, MSEs and radioactive wastes as covered under the
Andaman and Nicobar islands. provisions of the Atomic Energy Act, 1962.
Anushilan Samiti was formed by Pulin Bihari Das which was So, all of the statements are correct and option(d) is correct
also known as Dhaka Anushilan Samiti. answer.
84 Practice Set 3

59. The Supreme Court under Article 32 and High Courts under 66. Unlike the question hour, the zero hour is not mentioned
Article 226 can issue writs for the enforcement of Fundamental in the rules of procedure. Thus, it is an informal device
Rights under Part III of the Constitution of India. The writs issued available to the members of the Parliament to raise matters
under Article 32 and 226 for the enforcement of Fundamental without any prior notice. It has been in existence in India
Rights are since, 1962. The question hour is used as a device in the
● Habeas Corpus ● Mandamus parliamentary proceedings which includes starred, unstarred
● Prohibition ● Certiorari and short notice questions.
● Quo-warranto 67. Under Article 352, the President can declare a National
60. ( Whip is based on the conventions of the Parliamentary Emergency when the security of India or a part of it is
Government. Whip is charged with the responsibility of ensuring threatened by war, external aggression or armed rebellion.
the attendance of his party members in large numbers and It is possible that the President can declare a National
securing their support in favour of or against a particular issue. He Emergency even before the actual occurrence of war or
regulates and monitors their behaviour in the Parliament. The external aggression or armed rebellion, if he is satisfied that
members are supposed to follow the directives given by the whip, there is an imminent danger.
otherwise, disciplinary action can be taken. 68. Ancient Indians were very much interested in science
61. The low export of iron ore to China is due to Supreme Court’s and technology. In many means, they were ahead of their
ban on iron ore mining in many states. China has not relaxed its contemporary rivals. Some of the important scientists were
protectionist measures in sectors of pharmaceuticals and Aryabhatta, Varahamihira, Sushruta, Charaka etc.
information technology. There is no ban on power equipment Varahamihira wrote Brihatsamhita in which he wrote and
from China and Eurozone economic slowdown has resulted in proved that Moon rotates around the Earth and Earth rotate
increased trade between India and China. around the Sun. Sushruta wrote Sushrutasamhita to
describe the method to deal with operating cataract, stone
62. DTAAS may increase the annual gross tax revenue generated
diseases etc. Charaka’s book Charakasamhita was an
in the host country if the tax in the host country is lesser than the
encyclopedia over Indian medicine. He explained a lot of
other country. It is in the interest of investing
diseases and fever and their cure.
individuals/companies since, it will reduce the double taxation. It
may encourage round-tripping of money to the country with low 69. Human Development Index (HDI) is a composite
taxation. statistic of life expectancy, education and income indices
used to rank countries into four tier of human development.
63. Casuarina plants are one of the important xerophytic species
It was created by Indian economist Amartya Sen and
found in coastal India and Western India as well as semi-arid
Pakistani Economist Muhhub ul Haq in 1990 and was
regions. These are also called coniferous shrubs of semi-arid
published by UNDP.
tropics. Roots are primary organs and go deep into the soil layers.
These are used as fuel wood and for dyeing fishermen’s nets. Gross National Happiness (GNH) was designed in an
These are also useful for tonic and in the treatment of stomach attempt to define an indicator and concept that measures
complaints. quality of life or social progress. GNH is only been officially
used in Bhutan.
64. Ecological succession is an orderly process of community
The HDI has been criticised on a number of grounds
development in any region that starts with primary succession on
including ecological degradation and imbalances.
a bare rock and proceed through seres to the climax stage that is
comparatively stable and species communities are comparatively 70. While WPI (Wholesale Price Index) represents the
more complex and more diversified. This climax concept was first, wholesale prices of goods, CPI indicates the average price
explained by Frederic Clement who gives idea of ‘Mono climax’ paid by households for a basket of goods and services.
and later Arthur Tansley gives idea of ‘Poly climax’. Central Statistics Office (CSO), Ministry of Statistics and
The climax stage is not permanent, it is changed with change in Programme Implementation releases CPI with base year
climate, edaphic factor etc. 2010 for all India and States/UTs separately for rural, urban
and combined (rural+urban) every month with effect from
65. NASA launched Space X Falcon 9 Rocket with Jason-3
January, 2011.
ocean-monitoring satellite. The rocket carried the ocean
monitoring satellite Jason-3 on board and successfully inserted it These indices are available for five major groups namely
in desired orbit. However, the rocket failed a return landing on a food, beverages and tobacco; fuel and light; housing (only
drone platform in the Pacifc ocean. This satellite will examine the for urban; rural areas are not included); clothing, bedding
topography of the ocean floor to help study effects of climate and footwear and miscellaneous. Approximately 50%
change or human-induced changes on the ocean. It will extend weightage is given to food items and total number of
the time series of ocean surface topography measurement. commodities in the index is 200.
Stage 2 Improve Your Weaker Areas 85

71. Tamaraw and Giant eland are herbivores. Herbivory is 77. Lakshadweep and Minicoy are islands of the Arabian sea. The
a form of consumption in which an organism principally entire island group is built of coral deposits. It is eleven degree
eats autotrophs such as plants, algae and channel, North of which is Amini island and to the South is the
photosynthesising bacteria. More generally, organisms Canannore island. The islands of this archipelago have storm
that feed on autotrophs in general are known as primary beaches consisting of unconsolidated pebbles, shingles, cobbles
consumers. and boulders on the Eastern seaboard.
Herbivory usually refers to animals eating plants; fungi, 78. All the given statements are correct. A generic drug is a
bacteria and protists that feed on living plants are usually pharmaceutical product, usually intended to be interchangeable
termed plant pathogens (plant diseases) and microbes with an innovator product, that is manufactured without a license
that feed on dead plants are saprotrophs. Herbivores form from the innovator company and marketed after the expiry date of
an important link in the food chain, because they consume the patent or other exclusive rights. These drugs are subject to the
plants in order to ingest the carbohydrates produced by a same regulations over manufacturing, packaging, testing and
plant with the help of photosynthesis. quality standards, as their patented/branded equivalent. The
Carnivores in turn consume herbivores for the same prices of branded drugs is usually high because of monopoly
reason, while omnivores can obtain their nutrients from acquired through patent. The Government of India is taking fresh
either plants or animals. Due to a herbivore’s ability to measures to bolster the Jan Aushadhi Campaign, as a Public
survive solely on tough and fibrous plant matter, they are Welfare Programme, to supply quality medicines at affordable
termed the primary consumers in the food cycle (chain). prices to the common man through dedicated outlets. A key
Herbivory, carnivory and omnivory can be regarded as initiative under the campaign is opening of ‘Jan Aushadhi Stores’,
special cases of consumer resource systems. where quality generic medicines, which are equivalent to the
expensive branded drugs, in terms of their potency and efficacy,
72. The Biosphere Reserve is series of protected areas are sold at cheaper prices
linked through a global network, intended to demonstrate
the relationship between conservation and development.
79. The large prayer hall of the mosque pointed towards Mecca,
which is to the West of India. In the rear wall of the prayer-hall, the
A National Park is a natural area designated to protect the
centre is occupied by a recess or alcove called ‘mihrab’, and
ecological integrity of one or more ecosystems for present
indicates the direction of prayer (quibla). A pulpit (mimbar) at its
and future generations.
right is meant for the imam who leads the prayer. A tower or
73. Almost 90% of cosmic rays are protons, about 9% are minaret, originally intended for the ‘muazzin’ to call the faithful to
helium nuclei (alpha particles) and nearly 1% are the prayer, later assumed a mere architectural character. A gallery
electrons. This means that cosmic rays are not made up of or compartment of the prayer hall or some other part was screened
only positively charged particles. Cosmic rays may broadly off to accomodate the ladies who observed purdah.
be divided into two categories: primary and secondary. 80. (b) Terrace farming and contour ploughing are the techniques
The cosmic rays that originate from astrophysical sources adopted for restricting soil erosion along the slope i.e. in hilly areas
are primary cosmic rays; these primary cosmic rays and plateau regions. Afforestation is to check soil erosion in almost
interact with interstellar matter creating secondary cosmic all regions. Shelter belts is raised on the margins of the desert and
rays. Sun is not the major source but exactly is a minor dry lands to check the wind erosion. Gully erosion is a major cause
source. of soil erosion in dry lands so, plugging of gullies are a good
measure for prevention of soil erosion. In strip cropping crops are
74. The President is elected not directly by the people, but
grown in alternate strips of land to check the impact of winds.
by members of Electoral College consisting of
● the elected members of both the houses of Parliament.
81. An alluvial fan is a fan or cone shape deposit of sediment
crossed and built up by streams. They are usually created as
● the elected members of the Legislative Assemblies of
flowing water interacts with mountains hills or the steep walls of
the states.
canyons. The load that is being carried by stream over the
● the elected members of the Legislative Assemblies of mountain slopes becomes too heavy to carry over gentler
Delhi and Puducherry.
gradients and hence alluvial fans are formed.
● all of the nominated members at Centre and State
cannot participate. 82. The President is not free to appoint anyone as the Prime
Minister. In accordance with the conventions of the Parliamentary
75. This declaration represents the first international system of government, the President has to appoint the leader of
expression of human rights of which all human beings are the majority party in the Lok Sabha as the Prime Minister.
entitled. It is described as the ‘International Magna Carta’.
In 1980, the Delhi High Court held that the Constitution does not
Human rights are those rights to which all humans are
require that a person must prove his majority in the Lok Sabha
entitled merely by virtue of being humans. They are the
before he is appointed as the Prime Minister. The President may
inalienable and inviolable rights of all human beings. They
first appoint him the Prime Minister and then ask him to prove his
derive from the inherent dignity of human beings. They are
majority in the Lok Sabha within a reasonable period. Article 75
essential for human survival and human development.
says the Prime Minister shall be appointed by the President and
76. The arrangment of mountain ranges from North to the other Ministers shall be appointed by the President on the
South is Karakoram, Ladakh, Zaskar and Pir Panjal range. advice of the Prime Minister.
86 Practice Set 3

83. Article 51 A (e) to renounce the practices derogatory to the 90. All of these statements are true regarding Tulasi. It is
dignity of women. It is one of the Fundamental duty of Indian mentioned in the Charaka Samhita and regarded in
citizen. All these statements are correct, the right available to Ayurveda as a kind of ‘elixir of life’ and believed to promote
woman can be classified into two categories, namely, longevity. For centuries, the dried leaves have been mixed
constitutional rights and legal rights. The constitutional rights are with stored grains to repel insects.
those which are provided in the various provisions of the
91. Article 75 clearly says that the Council of Ministers is
Constitution. So, the Statement 2 is correct as it a part of Directive
collectively responsible to the Lok Sabha. This article also
Principles of State Policy. The legal rights, on the other hand, are
contains the principle of individual responsibility. In India
those which provided in the various laws (acts) of the Parliament
there is no provision in the Constitution for the system of
and the State Legislatures. So, the Statement 3 is correct in this
legal responsibility.
regard.
92. The Indian Council Act, 1909 also known as
84. GST is a value added tax to be implemented in India, the
Morley-Minto reforms introduced seperate and
decision. It will replace the indirect taxation levied on goods and
discriminatory electorate for the first time in India for
services by the Central and State Governments. The aim of GST is
Muslims. Although Britishers implemented this decision,
to make it very comprehensive for most of the goods and services.
but they were also suspicious of its ill effects. And, finally it
Exports will be zero-rated and imports will be levied the same taxes
proved to be the base for partition of India. INC accepted
as domestic goods and services adhering to the destination
separate electorate in its 1916, Lucknow session.
principle.
The Indian Council Act, 1909 also raised the number of
85. Offences under the act shall be cognisable and statement 4 additional members in Central legislative Assembly to 60.
ment explain non-bailable. All the other statements are correct
93. The HRIDAY that seeks to preserve and rejuvenate the
except Statement 1. Prohibition of employment as manual
rich cultural heritage of the country was launched by union
scavengers and their Rehabilitation Act, 2013 came into force on
Government on 21st January, 2015. It aims to bring
6th December, 2013 across India except Jammu and Kashmir. The
together the three elements–urban planning, economic
law prohibits the employment of manual scavengers, the manual
growth and heritage conservation. Rich heritage and
cleaning of sewers and septic tanks without protective equipment
cultural history are the basis on which the cities have to be
and the construction of insanitary latrines.
selected.
86. SAFAR provides data on Air Quality Index, UV index and There are twelve cities have been identified namely
dynamic city pollution map. Air quality forecast in real time and 24 Amravati (Andhra Pradesh), Gaya (Bihar), Dwarka
hours in advance. It has capability to forcast 3 days (72 hrs) in (Gujarat), Badami (Karnataka), Puri (Odisha), Amritsar
advance but 3 day’s advance forecast will be issued when there is (Punjab), Ajmer (Rajasthan), Kanchipuram and Velankanni
specific and extreme event. Currently operational in two cities, (Tamil Nadu), Warrangal (Telangana) and Mathura and
Delhi and Pune. Varanasi (Uttar Pradesh). The scheme broadly focus on
87. Even though 3D printing technology has been around since the four theme areas i.e. physical, institutional, economic and
1980s, the price has dropped in the recent times and it is social infrastructure for reviving and revitalising the soul of
commercially available now. It can ‘print’ on plastic, metal, nylon heritage city. It is a central scheme where 100% funding
and over a hundred other materials. It is also called additive will be provided by the government of India. The duration
manufacturing of this scheme is four years i.e. from December, 2014 to
March, 2018. The National Advisory Committee (NAC) is
88. The Sultanate period was a period of fusion. Although, Arabian,
the apex advisory body for the scheme headed by
Persian and early Hindi maintained its difference, but the writer
secretary, Ministry of Urban Development.
started to write in one or more than one language. They also
started translating the significant writings of other language. 94. The Susu Dolphin thrives only in freshwater. It is also
This lead to cultural amalgamation. The famous writings of the called as the Ganges River dolphin. It is the national
period were Tuti Nama (based on a Sanskrit original) by Ziya aquatic animal of India other names are blind river dolphin.
Nakshabi, Tughlaq Nama, Quran-us-Saadain, Nuh Sipihr etc by As they thrives only in freshwater means they are indicator
Amir Khusarow, Tarikh-i-Firoz Shahi by Barani and Fawaid-ul-Fuad of the health of river.
by Hasan. 95. The Vice President occupies the second highest office
89. The villi increase the surface area for absorption of the in the country. He/she is accorded a rank next to the
digested food. It helps in absorbing more food. The other two President in the official warrant of precedence. Article 66
statements are correct. Each villus has a network of thin and small under Part V of the Constitution deals with the education of
blood vessels close to its surface. The surface of the villi absorbs the Vice-President. Article 66 (3) lays down the
the digested food materials. The absorbed substances are qualifications for the post of the Vice President. So, the
transported via the blood vessels to different organs of the body Statement 1 and 2 prescribes qualifications and the
where they are used to build complex substances such as proteins Statement 3 and 4 are related to election of the Vice
etc. President.
Stage 2 Improve Your Weaker Areas 87

96. The first two statements are correct. Vitamins help in The dew point is the temperature below which the water vapour in
protecting our body against diseases. They also help in air at constant pressure condenses into liquid water. A high
keeping our eyes, bones, teeth and gums healthy. They can relative humidity indicates that dew point is closer to the current
be divided into different categories and are known by air temperature. Dry air has relative humidity less than 50%.
different names. 99. It is the first operational launch vehicle of ISRO. PSLV is
Some of these are Vitamin A, Vitamin C, Vitamin D, Vitamin capable of launching 1600 kg satellites in 620 km sun
E and K. -synchronous polar orbit and 1050 kg satellite in
There is also a group of vitamins called Vitamin B-complex. geo-synchronous transfer orbit.
Our body needs all types of vitamins in small quantities. ● PSLV has four stages using solid and liquid propulsion
Vitamin A keeps our skin and eyes healthy. Vitamin C helps systems alternately. The first stage is one of the largest solid
body to fight against many diseases. Vitamin D helps our propellant boosters in the world and carries 139 tonnes of
body to use calcium for bones and teeth. propellant.
97. Namdapha flying squirrel are found in tropical forest ● ISRO has envisaged a number of variants of PSLV to cater to
and restricted to a single valley in the Namdapha Tiger different mission requirements. There are currently three
Reserve in Arunachal Pradesh. It was threatened as it is operational versions of the PSLV-the standard (PSLV), the
core-alone (PSLV-CA) without the six strap-on booster motors,
being hunted for food. The destruction and conversion of
and the (PSLV-XL) version, which carries more solid fuel in its
habitat to arable land and human settlement threatens the
strap-on motors than the standard version.
Sociable lapwing in to North and North-West of India.
● So far, PSLV has launched 87 satellites.
Siberian crane is a migratory bird found in Keoladeo
National Park in Rajasthan and threatened by pesticide 100. Intensification of forest management scheme is a centrally
pollution to the wetland. sponsored scheme for forest management in India. It was
launched in force from 2009-10 and it was previously as
98. Humidity is a measure of the dampness of the
‘Integrated forest Protection Scheme’ in the 10th Plan period.
atmosphere which varies greatly from place to place at
Center shares 90% of the cost for forest management in all
different times of day. The actual amount of water vapour
North-East states including Sikkim, Himachal Pradesh, Jammu
present in the air is called absolute humidity. While relative
and Kashmir and Uttarakhand and for all other states and UTs
humidity is defined as the ration between the actual amount
center shares 75% of the cost. JFMCs are organised to involve
of vapour and the total amount the air can hold at a given
local people in protection of forest.
temperature, expresses as a percentage.
Rough Work
UPSC CIVIL SERVICES (PRE) EXAM

General Studies
Paper 1
Practice Set 4
Time : 2 hrs MM : 200

Instructions

1. This set contains 100 questions. Each question comprises four responses (answers). You will select the response which
you want to mark on the Answer Sheet. In case, if you feel that the correct response is more than one, then mark the
response, which you consider the best. In any case, choose only one response for each questions.
2. All questions carry equal marks.
3. Penalty for Wrong Answer
(i) There are four alternatives to answer every question. If a question is marked wrong, one-third of the marks assigned

IMPROVE YOUR WEAKER AREAS


to that question will be deducted as penalty.
(ii) If a candidate gives more than one answer, it will be treated as wrong answer even, if one of the given answers
happens to be correct and there will be same penalty as to that question, if it has a penalty.
(iii) If a question is left blank i.e. no answer is given by the cnadidate, there wil be no penalty for that question.

1. The Liquefied Petroleum Gas (LPG) is composed of which of the following gases?
1. Butane 2. Methane
3. Propane 4. Ethane
Select the correct answer using the codes given below.
(a) 1 and 2 (b) 2, 3 and 4
(c) 1 and 3 (d) 1, 3 and 4

2. With reference to the District Mental Health Programme (DMHP), consider the following
statements.
1. It is based on Bellary model developed by National Institute of Mental Health and Neuro Sciences
(NIMHANS).
2. In the 12th plan it has been entended to all the districts of the country.
3. It is a community based programme.
Stage 2

Which of the statements given above are correct?


(a) 1 and 2 (b) 2 and 3
(c) 1 and 3 (d) All of these
90 Practice Set 4

3. Which of the following are true about conservation of elephants in India?


1. Project Elephant was launched in 1993 by Government of India.
2. The project aims at reducing man-animal conflict.
3. The project also provides for migration corridors.
4. India has joined E-a group for protection of elephants.
Which of the statements given above are correct?
(a) 2, 3 and 4 (b) 2 and 3 (c) 1, 2 and 3 (d) 1, 3 and 4

4. Consider the following statements.


1. Terrace farming are made on the steep slopes, so that flat surfaces are available to grow crops. They can
reduce surface run-off and soil erosion.
2. In intercropping, different crops are grown in alternate rows and are sown at different times to protect
the soil from rain wash.
3. Contour barrier is ploughing parallel to the contours of a hill slope to form a natural barrier for water to
flow down the slope.
4. Shelter belts are the rows of trees that planted to check the wind movement to protect soil cover in the
coastal and dry regions.
Which of the statements given above are correct?
(a) 1 and 3 (b) 1, 2 and 3 (c) 2 and 4 (d) 1, 2 and 4

5. Consider the following statements about Deendayal Upadhyaya Gram Jyoti Yojana (DDUGJY).
1. Union Government launched DDUGJY in July 2015 with an aim to provide 24 ´ 7 uninterrupted
electricity supply to each rural household across the country by 2022.
2. The scheme draws its inspiration from the similar pioneering scheme implemented by the Government
of Gujarat.
3. The Rajiv Gandhi Grameen Vidyutikaran Yojana (RJGGVY) is a separate scheme for rural
electrification.
Which of the above statement(s) is/are correct?
(a) 1 and 2 (b) Only 3 (c) 1 and 3 (d) All of these

6. With reference to National Investment and Manufacturing Zones (NIMZs), consider the following
statements.
1. NIMZ aims to enhance the share of manufacturing in the GDP to 25% and to create 100 million jobs over
a decade or so.
2. These industrial townships are proposed to be self-governing and autonomous bodies under
Article-243 (Q-c) of the Constitution.
3. Article-243 (Q-c) of the Constitution deals with Panchayats and their Constitution.
4. These will be green field integrated Industrial townships with state of the art infrastructure.
Which of the statements given above are correct?
(a) 1, 2 and 4 (b) 1, 3 and 4 (c) 2, 3 and 4 (d) All of these

7. With reference to the response of the British against indigo revolt, consider the following
statements.
1. It was ruthlessy suppressed with violent force.
2. The British’s response was restrained because of the support extended to the revolt by the intelligentsia
and missionaries.
3. The government appointed a commission to inquire into the problem of indigo cultivation.
Which of the statement(s) given above is/are correct?
(a) 1 and 3 (b) 2 and 3 (c) Only 3 (d) Only 2
Stage 2 Improve Your Weaker Areas 91

8. The Pancaratra system belongs to which one of the following schools/sects?


(a) Hinayana Buddhist (b) Mahayana Buddhist (c) Nirgranthas (d) Vaishnavites

9. Consider the following statements.


1. Atmospheric nitrogen is converted to simpler forms only by nitrogen fixing bacteria and not by any
other method.
2. Nitrogen is essential in the formation of carbohydrates and fats in human bodies and plants.
Which of the statement(s) given above is/are correct?
(a) Only 1 (b) Only 2 (c) Both 1 and 2 (d) Neither 1 nor 2

10. Which of the following were the features of Bhakti Movement?


1. Oneness of God 2. Brotherhood of all human beings 3. Surrender into God
4. Personal devotion to God 5. Showed path to moksha
Select the correct answer using the codes given below.
(a) 1, 2, 3 and 4 (b) 2, 3, 4 and 5 (c) 1, 4 and 5 (d) 1, 2, 3 and 5

11. Consider the following statements regarding Indus Valley Civilisation.


1. The people relied only on rainfall for agriculture.
2. They did not grow wheat and rice.
3. They did not hunt wild animals.
Which of the statement(s) given above is/are correct?
(a) Only 1 (b) 1 and 3 (c) Only 2 (d) None of these

12. Ozone depletion is a major environmental concern throughout the world. Consider the following
statements regarding ozone depletion.
1. Ozone depletion creates health hazard to the living organisms.
2. Chlorofluorocarbons are the major depleting substances of ozone layer.
3. Ozone at the higher stratospheric layer acts as greenhouse gas.
4. Stockholm Conference is associated with restricting ozone depletion.
Which of the statements given above are correct?
(a) 1 and 2 (b) 1, 2 and 3
(c) 1, 2 and 4 (d) All of these

13. Which of the following elements are found in the e-waste, which are toxic in nature?
1. Lead 2. Beryllium 3. Arsenic 4. Mercury
5. Antimony 6. Cadmium 7. Plutonium 8. Silicon
Select the correct answer using the codes given below.
(a) 2, 3 and 5 (b) 1, 4, 6 and 8
(c) 1, 2, 3, 4, 5 and 6 (d) All of these

14. When a resolution for the removal of the Speaker is under consideration in the Lok Sabha,
1. Speaker cannot preside at the sitting of the House.
2. He/She can speak and take part in the proceeding.
3. He/She cannot vote in any case.
4. Such a resolution can be moved only after giving 1 month advance notice.
Which of the statements given above are correct?
(a) 1 and 2 (b) 1 and 3 (c) 3 and 4 (d) 1, 2 and 4
92 Practice Set 4

15. Consider the following statements.


1. A starred question in Parliament requires an oral answer and hence supplementary questions can
follow.
2. An unstarred question in Parliament requires a written answer and hence, supplementary questions
cannot follow.
3. A short notice question is one that is asked by giving a notice of less than 10 days. It is answered orally.
Which of the statement(s) given above is/are correct?
(a) 1 and 2 (b) Only 3 (c) 2 and 3 (d) All of these

16. Which of the following were the recommendations of Nehru Report (1928)?
1. Dominion status
2. One-third representation of Muslims in Central Legislative Assembly
3. Linguistic Provinces
4. Universal adult suffrage
5. Right to form unions
Select the correct answer using the codes given below.
(a) 1, 2 and 3 (b) 1, 2, 4 and 5 (c) 1, 3, 4 and 5 (d) 3, 4 and 5

17. Consider the following statements regarding environmental issues of India.


1. Gulf of Mannar is one of the biosphere reserves.
2. The Ganga Action Plan, Phase-II has been merged with the National River Conservation Plan.
3. The National Museum of Natural History at New Delhi imparts non-formal education in environment
and conservation.
4. Environmental Information System (ENVIS) acts as a decentralised information network for
environmental information.
Which of the statements given above are correct?
(a) 1 and 4 (b) 2 and 3 (c) 1, 2 and 3 (d) All of these

18. Consider the following statements about the declaration of the first organic state in the country.
1. Entry of chemical inputs for farmland was restricted in the state (Sikkim).
2. It tries to strike a harmonious balance with a complex series of ecosystem.
3. It also help in building the soil health resulting in sustainable inreased crop production.
Which of the statement(s) given above is/are correct?
(a) Only 1 (b) 2 and 3 (c) 1 and 3 (d) All of these

19. With reference to the Navjat Shishu Suraksha Karyakram (NSSK), consider the following
statements.
1. NSSK is a programme aimed to train health personnel in basic newborn care and resuscitation.
2. The training is for 2 days and is expected to reduce neonatal mortality significantly in the country.
3. This programme addresses important interventions of care at birth i.e. prevention of hypothermia,
prevention of infection, early initiation of Breast feeding and basic newborn resuscitation.
Which of the statements given above are correct?
(a) 1 and 2 (b) 2 and 3 (c) 1 and 3 (d) All of these

20. Arrange the following energy sources in the increasing order of consumption in India.
1. Coal 2. Petroleum and natural gas 3. Hydroelectricity
4. Nuclear energy 5. Wind energy
Select the correct answer using codes given the below.
(a) 5, 4, 3, 2, 1 (b) 4, 5, 3, 2, 1 (c) 2, 5, 4, 3, 1 (d) 2, 4, 5, 3, 1
Stage 2 Improve Your Weaker Areas 93

21. Consider the following statements about Andaman and Nicobar Islands.
1. Andaman and Nicobar Islands is separated from mainland by a water body which is called the Ten
Degree Channel.
2. These islands are believed to be the elevation of portion of submerged mountains.
3. There is no evidence of volcanic type of islands in Andaman and Nicobar area.
Which of the statement(s) given above is/are correct?
(a) 1 and 2 (b) Only 2
(c) Only 1 (d) All of these

22. Consider the following statements about the Peninsular river system of India.
1. With respect to Himalayan drainage system; the Peninsular drainage system is of recent origin.
2. Narmada and Tapi flow in trough faults and fill the original cracks with their detritus material.
3. Krishna river is also known as Dakshin Ganga.
Which of the statement(s) given above is/are correct?
(a) 1 and 2 (b) Only 2
(c) 1 and 3 (d) All of these

23. Which one of the following state celebrate the Wangala festival?
(a) Bihar (b) Odisha
(c) Meghalaya (d) Mizoram

24. Western disturbances are the low pressure depressions. Consider the following statements in the
context of this climatic phenomena and choose the correct statements.
1. They originate in the Mediterranean sea and their moisture content augmented from the Arabian sea.
2. They are often in an occluded form when they enter into India.
3. They causing light rain in the Indo-Gangetic plain and snowfall in the Himalaya.
4. The equitorial Easterly jet-stream plays an important role in bringing these disturbances to India.
Which of the statements given above are correct?
(a) 1 and 2 (b) 1 and 4
(c) 2 and 3 (d) 2 and 4

25. Consider the following statements regarding the Jallianwala Bagh massacre.
1. The people had gathered at Jallianwala Bagh to attend the annual Baishakhi fair and there was not any
protest.
2. The British media strongly condemned General Dyer for ordering the killings of the people.
3. The government held an inquiry commission, chaired by Lord Hunter.
4. This commission did not have any Indian member and the report was uncritical of General Dyer.
Which of the statement(s) given above is/are correct?
(a) 1 and 2 - (b) 2 and 3
(c) 2, 3 and 4 (d) Only 3

26. Consider the following statements about the use of zero in the ancient times.
1. It was considered only as a numeral on number line.
2. Its discovery enabled finding the exact distance between Earth and Moon.
3. It opened up the realm of negative numerals.
Which of the statements given above are correct?
(a) 2 and 3 (b) 1 and 2
(c) 1 and 3 (d) All of these
94 Practice Set 4

27. What is/are the correct statement(s) about the Green Climate Fund (GCF).
1. The Green Climate Fund (GCF) is a fund within the framework of the UNFCCC founded as a mechanism
to redistribute money from the developed to the developing world, in order to assist the developing
countries in adaptation and mitigation practices to counter climate change.
2. It was founded as a mechanism to redistribute money from the developed to the developing world, in
order to assist the developing countries in adaptation and mitigation practices to counter climate
change.
3. It was also founded to make a significant and ambitious contribution to the global efforts towards
attaining the goals set by the international community to combat climate change.
(a) Only 1 (b) Only 2 (c) All of these (d) None of these

28. Special Economic Zone (SEZ) is a geographical region that has economic laws different from a
countries ‘typical economic lanes’. Consider the following statements about the objective of SEZs.
1. Promotion of exports of goods and services.
2. Promotion of investment exclusively for domestic sources.
3. Creation of employment opportunities.
4. Development of infrastructure facilities.
Select the correct answer using the codes given below.
(a) 1, 2 and 3 (b) 1, 3 and 4
(c) 2, 3 and 4 (d) All of these

29. Consider the following statements regarding Public Policy.


1. A citizen exercise direct influence on policy-making.
2. In countries, where the courts have the power of judicial review, they have played an important role in
policy formation.
3. In modern societies, political parties generally perform the functions of interest aggregation i.e. they
seeks to convert the particular demands of interest groups into general policy alternatives.
4. Pressure groups or interest groups are organisations with formal structures whose members share some
common interests. They strive to influence the policies of the government without attempting to
occupy political offices.
Which of the statements given above are correct?
(a) 1 and 4 (b) 2, 3 and 4
(c) 2 and 3 (d) All of these

30. Which of the following statements is incorrect about Election Commission?


(a) Since its inception, the Election Commission functioned as a 3 members body
(b) The Chief Election Commissioner cannot be removed, except in same manner and on the same grounds
as a Judge of the Supreme Court
(c) The Constitution has not specified the term of the members of the Election Commission
(d) The Constitution has not debarred the retiring Election Commissioners from any further appointment
by the government

31. Consider the following statements about a particular river.


1. It originates in Rakshastal lake near Mansarovar in Tibet.
2. It is an antecedent river.
3. It is a very important tributary of Indus.
4. It feeds the canal system of the Bhakra Nangal Project.
Identify the river having above characteristics.
(a) Chenab (b) Jhelum (c) Beas (d) Sutlej
Stage 2 Improve Your Weaker Areas 95

32. Consider the following statements about Trans- Himalayas.


1. Most of the part of this Himalayan ranges lies in the Tibet and hence also called Tibetan Himalaya.
2. The Zaskar, the Ladakh, the Kailash and the Karakoram are the main ranges of the Trans-Himalayan
System.
3. The Northernmost range of the Trans-Himalayan ranges in India is the Great Karakoram range also
known as Krishnagiri.
Which of the statements given above are correct?
(a) 1 and 2 (b) 2 and 3 (c) 1 and 3 (d) All of these

33. Which of the following pairs is incorrectly matched?


(a) Adhai Din Ka Jhopra : Ajmer
(b) Quwwat-ul-Islam mosque : Agra
(c) Gol Gumbaz : Bijapur
(d) Humayun’s tomb : Delhi

34. Consider the following statements.


1. The predominant metal used by Satavahanas for minting coins was gold.
2. Coins issued during the reign of Rajaraja Chola have king standing on obverse and king seated on
reverse.
Which of the statement(s) given above is/are correct?
(a) Only 1 (b) Only 2 (c) Both 1 and 2 (d) Neither 1 nor 2

35. Which of the following pairs are correctly matched?


Natural Region Inhabitants
1. Amazon basin in equatorial region : Bambuti
2. Kalahari desert of tropical desert region : Bindibu
3. Temperate grassland region of Central Asia : Kirghiz/Kyrgyz
4. Tundra region in Northern Europe : Eskimo
Codes
(a) 1, 2 and 3 (b) 1 and 4 (c) Only 3 (d) 3 and 4

36. The Non-Cooperation Movement 1920, various social groups participated in this movement.
Consider the following facts.
1. The Programme of ‘‘non-violent non-cooperation’’ included of councils, courts and schools set up by
the British.
2. The Ali Brothers, the Principle leads of the Khilafat, were arrested in September 1921.
3. Lord Reading seemed willing to hold a round table conference with Gandhi and other Indian leaders.
4. Throughout 1921, the tension between the Congress and the Government was steadily mounting.
Which of the statements given above are correct?
(a) 1 and 4 (b) 2 and 3 (c) 1, 2 and 4 (d) All of these

37. GDP of a country as an index of greater well-being of the people of that country may not be correct
because
1. many activities in an economy are not evaluated in monetary terms.
2. non-uniform distribution of GDP.
3. the presence of positive or negative externalities affects the actual estimation of GDP.
Select the correct answer using the codes given below.
(a) 1 and 2 (b) 2 and 3 (c) Only 2 (d) All of these
96 Practice Set 4

38. Consider the following statements.


1. Reserve Deposit Ratio (RDR) is the proportion of the total deposits commercial banks shall keep as reserves.
2. Commercial banks can borrow money from RBI at the bank rate when they run short of reserves.
3. Maintaining a given fraction of their total demand and time deposits in the form of specified liquid
assets by the banks is called Cash Reserve Ratio (CRR).
Which of the statement(s) given above is/are correct?
(a) 1 and 2 (b) 2 and 3 (c) Only 3 (d) 1 and 3

39. Consider the following characteristics of an architecture type.


1. Use of spotted red sandstone 2. Use of Yaksha prototype
3. Promoted by Kushans
Identify the school of architecture based on the above characteristics.
(a) Gandhara school (b) Amaravati school (c) Mathura school (d) Sanchi school

40. Which of the following pairs are correctly matched?


Folk Music Location
1. Lavani : Maharashtra
2. Bhavageethe : Karnataka
3. Pandavani : Chhattisgarh
4. Bihugeet : Bihar
Codes
(a) 1 and 3 (b) 2 and 3 (c) 1, 2 and 4 (d) 1, 2 and 3

41. Which of the following statements are correct about non-confidence motion?
1. It need not state the reasons for its adoption in the Lok Sabha.
2. The motion needs the support of 1/10 members to be admitted.
3. It is passed in the Lok Sabha, the Council of Minister must resign from office.
4. It can be moved against the entire Council of Ministers only.
Select the correct answer using the codes given below.
(a) 1, 2 and 4 (b) 1, 3 and 4 (c) 2 and 3 (d) All of these

42. Consider the following statements regarding the rights of minorities.


1. The wearing and carrying of Kirpans shall be deemed to be included in the profession of the Sikh
religion under the Article 25(2).
2. Any sections of citizens having a distinct language, script or culture of its own has the right to conserve
the same under Article 29(1).
3. No citizen is to be denied admission into any educational institution maintained by the state or aided by
the state on grounds of religion, race, caste or language under Article 30 (2).
4. When the President is satisfied that a substantial proportion of the population of a state desire the use of
any language spoken by them to be recognised by that state, then he may direct that such language
should also be officially recognised in that state under Article 347.
Which of the statements given above are correct?
(a) 1, 2 and 4 (b) 2 and 3 (c) 1 and 4 (d) All of these

43. Consider the following statements regarding Project Tiger.


1. ‘Project tiger’, one of the well-publicised wildlife campaigns in the world, was launched in 1973.
2. Tiger conservation has been viewed not only as an effort to save an endangered species, but with equal
importance as a means of presuming biotypes of sizeable magnitude.
Which of the statement(s) given above is/are correct?
(a) Only 1 (b) Only 2 (c) Both 1 and 2 (d) Neither 1 nor 2
Stage 2 Improve Your Weaker Areas 97

44. What is the aim of the Mangroves For the Future (MFF) initiative in which IUCN is a partner?
(a) To examine the future impact of climate change on mangroves
(b) To find out how mangroves can help coastal communities adapt to climate change
(c) To raise children’s awareness about the importance of mangroves in the fight against climate change
(d) To promote investment in coastal ecosystem for sustainable development

45. Consider the following statements.


1. The United Nations Convention on the Rights of the Child (UNCRC) is a human rights treaty to set out
the civil, political, economic, social and cultural rights of children.
2. The convention knowledges that every child has certain basic rights, including the right to life, his or
her own name and identify, to be raised by his or her parents.
Which of the following statement(s) is/are correct?
(a) Only 1 (b) Only 2 (c) Both 1 and 2 (d) Neither 1 nor 2
46. Which of the following can potentially lead to an increase in employment levels in India?
1. Increasing the Cash Reserve Ratio (CRR) and interest rates.
2. Relaxing stringent labour laws.
3. Providing incentives to the Micro, Small and Medium Enterprises (MSMEs).
4. Relaxing FDI caps in brownfield projects in the manufacturing and services sector.
Select the correct answer using the codes given below.
(a) 2 and 3 (b) 1 and 2 (c) 2, 3 and 4 (d) 1, 3 and 4
47. Consider the following statements.
1. The crocodile is a common inhabitant of the mangrove swamps of the Cumbarjua canal connecting the
Mandovi and Zuari estuaries of Goa.
2. Goan fisherman worship crocodile and it has become an annual feature in Goa.
Which of the following statement(s) given above is/are correct?
(a) Only 1 (b) Only 2 (c) Both 1 and 2 (d) Neither 1 nor 2
48. Which of the following pair(s) is/are incorrectly matched?
1. Beriberi : Weak muscles and very little energy to work
2. Scurvy : Bones become soft and bent
3. Rickets : Bleeding gums, wounds take longer time to heal
Codes
(a) Only 1 (b) 1 and 2 (c) 2 and 3 (d) Only 3
49. Which of the following statement(s) is/are correct about Mughal empire?
1. The eldest son inherited his father’s estate.
2. Afghans were an immediate threat to Mughal authority.
3. Peasant revolts were absent in the Mughal empire.
Select the correct answer using the codes given below.
(a) 1 and 2 (b) Only 2 (c) 1 and 3 (d) Only 1
50. Which of the following statements are correct regarding the spread of religions from India to the world in
ancient times?
1. The Vajrayana Buddhism spread to other parts of the world.
2. During spread of Buddhism, its influence decreased in India.
3. Hinduism and Jainism never spread to other parts of the world.
Select the correct answer using the codes given below.
(a) 1 and 3 (b) 2 and 3 (c) 1 and 2 (d) All of these
98 Practice Set 4

51. Consider the following animals.


1. Gharial 2. Flying squirrel 3. Wild water buffalo
4. Great Indian bustard 5. Jerdon’s courser (a nocturnal bird)
Which of the above is not in IUCN’s list of critically endangered species?
(a) 1 and 4 (b) 4 and 5 (c) 1, 3 and 4 (d) All of these
52. Consider the following statements about Attorney General of India.
1. He must be a person who is qualified to be appointed a Judge of the High Court.
2. The term of office of the Attorney General is not fixed by the Constitution.
3. The Constitution does not contain the procedure and grounds for his removal.
4. Attorney General has the duty to appear in any High Court in any case, in which the Government of
India is concerned.
Which of the statement(s) given above is/are incorrect?
(a) Only 1 (b) 1 and 2 (c) Only 3 (d) 2 and 3

53. Which of the following statements is incorrect with regard to Chemical Weapons Convention
(CWC)?
(a) The Chemical Weapon Convention (CWC) came into force on 29th April, 1997
(b) It provides that after the year 2010, every state party to the convention will never develop produce,
stockpile or retain chemical weapons
(c) 164 countries have signed the convention which was first opened to signature in Paris on 13th
January, 1993
(d) A technical secretariat headquartered at Hague will be responsible for verifying its provisions

54. Which of the following pair(s) is/are not correctly matched?


Country Legislature
1. Norway : Storting
2. Poland : Sejm
3. Spain : National Assembly
4. Sweden : Cortes
Codes
(a) 1 and 2 (b) 3 and 4 (c) Only 1 (d) 2 and 4

55. Consider the following statements with reference to ozone.


1. Unlike the normal diatomic molecule of oxygen, ozone is poisonous.
2. Ozone is a powerful oxidant (far more than dioxygen).This makes ozone a potent respiratory hazard and
pollutant near ground level.
3. Ozone is not stable near the Earth’s surface.
Which of the statement(s) given above is/are correct?
(a) Only 1 (b) 1 and 2 (c) 2 and 3 (d) All of these

56. Consider the following statements with reference to the procedures used in sewage water treatment.
1. The greater the Biological Oxygen Demand (BOD) of wastewater, more is its polluting potential.
2. In the secondary treatment of sewage water, flocs or masses of bacteria is allowed to grow and increase
BOD levels.
3. BOD refers to the amount of the oxygen that would be consumed if all the organic matter in 1 litre of
water were oxidised by bacteria.
Which of the statement(s) given above is/are incorrect?
(a) l and 2 (b) 2 and 3 (c) Only 2 (d) All of these
Stage 2 Improve Your Weaker Areas 99

57. Consider the following statements about the Subsidiary alliance.


1. The rulers of allying Indians state could negotiate the temporary stationing of a British Force within his
territory and to pay a subsidy for its maintenance.
2. Indian ruler could not employ any European in his service without the approval of the British.
3. However, Indian rulers were free to negotiate with any other Indian ruler without consulting the
Governor General.
Which of the statements given above are incorrect?
(a) 1 and 2 (b) 2 and 3
(c) 1 and 3 (d) All of these

58. Consider the following statements regarding Sarojini Naidu.


1. Sarojini Naidu was the first Indian woman to become the President of the Indian National Congress.
2. Sarojini Naidu was the first woman Governor of India.
3. Sarojini Naidu’s birthday is celebrated as ‘Women’s Day’ in India.
4. Sarojini Naidu was one of the framers of Indian Constitution.
Which of the statements given above are correct?
(a) 2 and 3 (b) 1, 3 and 4 (c) 1, 2 and 4 (d) All of these

59. Which of the following are characteristics of Madhubani School of Painting?


1. Paintings are done only on the freshly plastered mud walls mainly by women.
2. Use of geometrical pattern.
3. No space is left empty. Gaps are filled by paintings of flowers, birds and animals.
4. Geographical indication status has been granted already.
Select the correct answer using the codes given below.
(a) 1 and 3 (b) 1, 3 and 4 (c) 2, 3 and 4 (d) 1 and 2

60. Under the Anti-defection law, the speaker of the Lok Sabha takes decisions on the question of
disqualification of a member of the house under which of his following capacities.
(a) Presiding officer (b) Tribunal
(c) Arbitration (d) Administrative head of the Lok Sabha

61. Consider the following statements regarding Bamboo (Dendrocalamus hamiltoni).


1. It is a culturally valued species.
2. It is an ecological valued keystone species conserving potassium in the system.
3. It is a major part of the slash burnt for a 10 to 20 year cycle of Jhum (shifting cultivation) system in
North-East India.
Which of the statement(s) given above is/are correct?
(a) Only 3 (b) 2 and 3 (c) 1 and 2 (d) All of these

62. Consider the following statements with respect to leaching.


1. Leaching refers to loss of soluble substances and colloids from the top layer of soil by percolating
precipitation.
2. The rate of leaching increases with the amount of rainfall, high temperatures and the removal of
protective vegetation.
3. It (leaching) results in the formation of laterite soil.
4. In agriculture, leaching is used as a process to avoid salinity by applying a small amount of excess
irrigation.
Which of the statements given above are correct?
(a) 2 and 3 (b) 1 and 4 (c) 2, 3 and 4 (d) All of these
100 Practice Set 4

63. Consider the following statements about Comptroller and Auditor General of India (CAG).
1. The CAG is appointed by the President of India.
2. He hold his office till the pleasure of the President.
3. He is not eligible for further office under any Government in India.
4. His salary is equal to that of a Judge of the Supreme Court.
Which of the statement(s) given above is/are incorrect?
(a) 3 and 4 (b) Only 2
(c) Only 3 (d) 2 and 4

64. Consider the following statements regarding Mughal rule.


1. ‘Petticoat government’ refers to the early era when Bairun Khan ruled on Akbar’s behalf.
2. Jizya was lowered or raised at different times, but never abolished during Mughal rule.
3. Akbar made clear distinction between positions of religious head and king.
Which of the statement(s) given above is/are correct?
(a) 1 and 3 (b) Only 3 (c) 1 and 2 (d) None of these

65. Consider the following statements about Vijayanagara empire.


1. Portugese played important role in Vijayanagara politics.
2. Trade’s contribution to Vijayanagara revenue was marginal.
3. Horses were imported for military purposes.
Which of the statements given above are correct?
(a) 1 and 2 (b) 2 and 3 (c) 1 and 3 (d) None of these

66. Which of the following statements is correct regarding water resources.


(a) Atmosphere has more freshwater than all surface rivers on the Earth.
(b) River freshwater is in more quantity than the groundwater.
(c) The combined quantity of freshwater in lakes, rivers and groundwater is more than that is stored in
polar ice caps.
(d) The average salinity of the oceans is 40 parts per thousand.

67. The steel industry at Pittsburgh enjoys locational advantages. Of these is one of the world’s best
routes for shipping ore cheaply between it and iron ore mines. That route includes
(a) Mississippi waterway (b) USA - Canada railway network
(c) Great Lakes waterway (d) Panama canal

68. With reference to DTH services, consider the following statements.


1. Only Doordarshan DD Direct Plus (DTH) is providing DTH services free of cost of the consumers.
2. DTH transmission is most preferred in KU-Band so as to avoid the need to larger dish sizes for suitably
receiving the DTH signals.
3. The DTH signals cannot be received in hilly and remote terrains.
Which of the statement(s) given above is/are incorrect
(a) 1 and 2 (b) 2 and 3 (c) 1 and 3 (d) Only 3

69. Consider the following statements about the Departmental Standing Committees of Parliament.
1. It consists of 31 members (21 from Lok Sabha and 10 from Rajya Sabha).
2. A minister is not eligible to be a member of any of the Standing Committee.
3. It considers the demands for grants of the concerned ministries before they are voted in the Lok Sabha.
Which of the statement(s) given above is/are correct?
(a) Only 1 (b) 2 and 3 (c) Only 3 (d) All of these
Stage 2 Improve Your Weaker Areas 101

70. Which of the following are major environmental awareness programmes initiated by the
government?
1. Ecomark Scheme is introduced since 2000 AD with sunrise in the valley.
2. To disseminate environmental awareness among children Eco-Club Scheme is introduced.
3. Eco-Task Force is an pro-environmental feature associated with army men.
4. In 1991, the Supreme Court of India issued directives to make all curricula environment-oriented.
Select the correct answer using the codes given below.
(a) 1 and 4 (b) 2 and 3 (c) 1, 2 and 3 (d) All of these

71. Which of the following options is not correct regarding Sea Buckthorns (Hippophae rhamnoides)?
(a) It is medicinal plant used for a variety of ailments of humans and castle
(b) It can be used to make pies, jams, lotions, fruit wines and liquors
(c) DRDO, India established a factory in Leh to manufacture a multi-vitamin herbal, beverage based
on seabuck thorn juice to support military establishments
(d) Its species are found only in Asia
72. Consider the following statements with reference to spread of cancer in human body.
1. Normal cells show a property called contact inhibition by virtue of which contact with other cells
inhibits their uncontrolled growth.
2. Cancer cells will not have the property of contact inhibition. As a result of this, cancerous cells just
continue to divide giving rise to masses of cells called tumours.
3. Metastasis, the spread of a cancer from one organ or part to another non-adjacent organ or part, is the
most feared property of malignant tumours.
4. Cancer causing agents are called as carcinogens.
Which of the statements given above are correct?
(a) 1 and 2 (b) 1, 2 and 3
(c) 3 and 4 (d) All of these
73. Salt had been used as a preservative since ancient times, to protect food against bacteria, mold and
spoiling. How does salt act as food preservative?
(a) It keeps the temperature of the food low and inhibits bacterial and fungal activity
(b) Salt absorbs water from foods, making the environment too dry to support harmful bacteria or fungi
(c) It releases anti-bacterial and fungal agents
(d) It inhibits aerobic respiration of bacteria and other microorganisms
74. Consider the following statements regarding Khejri tree (Prosopis cineratia).
1. It is Rajasthan’s state tree and covers about two-thirds of the total geographical area of the state.
2. It is also known as ‘The Desert Tree’ and is lifeline of the people in Western Rajasthan, by providing
food and firewood, to support rural economy.
Which of the statement(s) given above is/are correct?
(a) Only 1 (b) Only 2
(c) Both 1 and 2 (d) Neither 1 nor 2
75. Consider the following statements about the Inter Tropical Convergence Zone (ITCZ).
1. It is a high pressure zone located near the equator.
2. During July, ITCZ shifts to about 20°N - 25°N latitudes over Gangetic plain.
3. The shift of ITCZ in summer encourages the South-West monsoon in India.
Which of the statements given above are correct?
(a) 1 and 2 (b) 2 and 3
(c) 1 and 3 (d) All of these
102 Practice Set 4

76. Consider the following statements with reference to Tissue Culture.


1. Micropropagation under tissue culture refers to the transfer of desired genes into plant saplings to
produce identical plants.
2. Explants refer to any part of a plant taken out and grown in a test tube, under sterile conditions in
special nutrient media.
3. Capacity to generate a whole plant from any cell/explant is called totipotency.
4. Each of the plants produced by tissue culture will be genetically identical to the original plant from
which they were grown.
Which of the statements given above are correct?
(a) 1, 2 and 3 (b) 2, 3 and 4
(c) 1 and 4 (d) All of these
77. Consider the following statements with reference to commonly used drugs by addicts.
1. The drugs, which are commonly abused, such as Opioids, Cannabinoids and Coca alkaloids, are
obtained from flowering plants.
2. Cocaine has a potent stimulating action on central nervous system, producing a sense of euphoria and
increased energy.
3. Morphine is used to relieve the pain for patients suffering from terminal illnesses such as cancer, HIV
and thalassemia.
Which of the statements given above are correct?
(a) 1 and 3 (b) 2 and 3
(c) 1 and 2 (d) All of these

78. Which of the following statements with regard to The Prime Minister’s Office (PMO) are correct?
1. It has the status of a department under the Government of India allocation of business rules.
2. It is headed by the Personal secretary to the Prime Minister.
3. The jurisdiction of PMO extends over all such subjects and activities as are not specially allotted to any
individual department.
Select the correct answer using the codes given below.
(a) 1 and 2 (b) 2 and 3
(c) 1 and 3 (d) All of these

79. The shaggy horned wild ox, the bharal (blue sheep), wild sheep and the Kiang are mainly found in
which of the following regions?
1. Arunachal Pradesh 2. Sikkim 3. Ladakh 4. Asom
Select the correct answer using the codes given below.
(a) 1 and 2 (b) 1 and 4
(c) Only 3 (d) 2, 3 and 4

80. Which of the following pyramids is always upright in an ecosystem?


(a) Pyramid of numbers (b) Pyramid of biomass
(c) Pyramid of energy (d) All of these

81. Which of the following reasons are correct for British success over French in India?
1. Stronger navy of the British.
2. English Company did not have bureaucratic interference like French.
3. Stronger cooperation amongst British officers compared to French.
4. British had modern arms which French did not.
Select the correct answer using the codes given below.
(a) 1, 2 and 4 (b) 1, 2 and 3
(c) 1, 3 and 4 (d) 2, 3 and 4
Stage 2 Improve Your Weaker Areas 103

82. Consider the following statements.


1. Nizam-ul-Mulk (Qamar-ud-din Khan) was the first Mughal noble who founded the State of Hyderabad
in Deccan, which marked the beginning of the physical break-up of the Mughal empire.
2. The Second Battle of Panipat resulted in the Kohinoor diamond being taken away by Nadir Shah, the
King of Persia.
3. In 1761, Mughal empire ceased to exist in practice as an all India empire and was reduced as the
Kingdom of Delhi.
4. The Mughal monarchy after the Battle of Buxar till 1857, served as the titular head for the English East
India Company, for it symbolised the political unity of the country.
Which of the statements given above are correct?
(a) 1 and 2 (b) 1 and 3
(c) 2, 3 and 4 (d) All of these

83. Every ecosystem regulates and maintains itself and resists the stresses and disturbances upto a
certain limit. This self-regulation or control system is known as
(a) Cybernetics (b) Adaptability
(c) Homeostasis (d) Feedback mechanism

84. In India, Common Property Resources (CPRs) type of land ownership has been advocated since
decades. What are the intentions behind advocating CPR?
1. It provides fodder for the livestock.
2. It provides more revenue to the government.
3. It provides livelihood for the weaker section of the society.
4. Every member can have property rights over the land.
5. Women are benefitted.
Select the correct answer using the codes given below.
(a) 2 and 4 (b) 1, 3 and 5
(c) 1, 2, 3 and 4 (d) All of these

85. Trade deficit is possibly affected by which of the following factors?


1. Domestic savings 2. Fiscal deficit 3. Investment in the economy
Select the correct answer using the codes given below.
(a) 1 and 3 (b) 2 and 3
(c) 1 and 2 (d) All of these

86. Right to vote in Lok Sabha and State Assembly elections is a


(a) Constitutional right (b) Statutory right
(c) Fundamental right (d) None of these

87. Which one of the following statements is incorrect?


(a) El-Nino is a complex weather system that generally appears in every 3 to 7 years
(b) El-Nino is the appearance to warm current off the coast of Peru and affects weather in many places
including India
(c) El-Nino phenomenon reveals that when the surface temperature goes up in the Southern Pacific
Ocean, India receives deficient rainfall
(d) There is one to one correspondence in the occurrence of El-Nino phenomenon and deficient rainfall in
India
104 Practice Set 4

88. Which of the following is not a function to be performed by Accredited Social Health Activist
(ASHA) under MRHM?
(a) To provide primary medical care for minor ailments such as diarrhoea, fever and first aid for minor
injuries
(b) To escort/accompany pregnant women and children requiring treatment/admission to the nearest
pre-identified health facility and to carry on child deliveries in case of emergency situation
(c) To assist the village health and sanitation committee of the Gram Panchayat to develop a
comprehensive village health plan
(d) To act as a depot older for essential provisions being made available to all habitations like Oral
Rehydration Therapy (ORT), Iron Folic Acid Tablet (IFA), chloroquine, Disposable Delivery kits,
oral trills and condoms etc

89. The Indian landmass has soils of great diversity. Consider the following statements in the context
of characteristics of Indian soils.
1. Most of the soils are old and mature.
2. Indian soils are largely deficient in nitrogen, mineral salts, humus and other organic materials.
3. The black soils are highly impermeable and water retentive.
4. The formation of laterite soils takes place under typical monsoonal conditions.
Which of the statements given above are correct?
(a) 1 and 2 (b) 2, 3 and 4
(c) 1, 3 and 4 (d) All of these

90. Consider the following statements about bio-fertilisers.


1. Bio-fertilisers are organisms that enrich the nutrient quality of the soil.
2. Bio-fertilisers do not contain any chemicals which are harmful to the living soil.
3. The main sources of bio-fertilisers are bacteria, fungi and cyanobacteria.
4. Bio-fertilisers add nutrients through the natural processes of nitrogen fixation, solubilising phosphorus
and stimulating plant growth through the synthesis of growth promoting substances.
Which of the statement(s) given above is/are correct?
(a) Only 2 (b) 3 and 4
(c) 2, 3 and 4 (d) All of these

91. Consider the following statements about Mahatma Gandhi’s famous philosophy of Satyagraha.
1. Satyagraha is not physical force. A Satyagrahi does not inflict pain on the adversary; he does not seek
the destruction of the enemy.
2. Mahatma Gandhi first experimented with Satyagraha at Champaran in Bihar.
3. Satyagraha aims at appealing to the conscience of the oppressor.
4. Satyagraha theory influenced Martin Luther King, Jr’s and James Bevel’s campaigns during the Civil
Rights Movement in the United States.
Which of the statements given above are correct?
(a) 1 and 2 (b) 2 and 3
(c) 1, 3 and 4 (d) All of these

92. Perform Achieve Trade (PAT) mechanism is associated with


(a) encouraging the exports performance of the economy by encouraging competition between them
(b) an Infrastructure model which links the ability to determine usage charges on the basis of
performance
(c) an incentive mechanism for the farmers to get higher prices for better quality foodgrains
(d) trading energy efficiency certificates in large energy intensive industries
Stage 2 Improve Your Weaker Areas 105

93. Consider the following statements about vocal learning ability of humans and animals.
1. During vocal development, young birds and human infants struggle with similar difficulties and go
through a similar stepwise process of learning transitions between syllabus.
2. Our closest relatives, apes and monkeys, lack the ability for vocal learning completely.
3. Besides humans and songbirds, vocal learning exists in parrots and humming birds, some marine
mammals such as whales and dolphins too.
Which of the statement(s) given above is/are correct?
(a) 1 and 2 (b) 1 and 3
(c) Only 3 (d) All of these

94. Which of the following pairs are correctly matched?


1. Article 23 (1) : Traffic in human beings and forced labour are prohibited
2. Article 45 : State shall endeavour to provide early childhood care and education for all children,
until they complete the age of 6 years
3. Article 24 : State shall provide free and compulsory education to all children of the age of 6 to 14 years
4. Article 21 (A) : No child below the age of 14 years shall be employed in any factory, mine or any other
hazardous occupation
Codes
(a) 1 and 2 (b) 1 and 3
(c) 1 and 4 (d) None of these

95. Consider the following factors.


1. Heating by solar energy 2. Wind
3. Gravity 4. Coriolis force
Which of the above are primary forces that influence the ocean currents?
(a) 1 and 4 (b) 2, 3 and 4
(c) 1, 3 and 4 (d) All of these

96. Consider the following statements.


1. Prerna is a scheme where Jansankhya Sthirata Kosh, invites private sector gynecologists and vasectomy
surgeons to conduct operations in Public Private Partnership mode.
2. Santusht is a scheme where the strategy recognises and awards couples who have broken the stereotype
of early marriage and early childbirth and helped change mindsets.
Which of the statement(s) given above is/are correct?
(a) Only 1 (b) Only 2
(c) Both 1 and 2 (d) Neither 1 nor 2

97. Consider the following statements about Skill India Programme.


1. It was launched on the occasion of the first ever world youth skill day on 15th July, 2015 by the Union
Government.
2. It includes National Skill Development Mission, National Policy for Development and
Entrepreneurship, Pradhan Mantri Kaushal Vikas Yojana and Skill Loan Scheme.
3. Its motto is Kaushal Bharat, Kushal Bharat (skilled India, successful India).
4. Union Government has created a new ministry for skill development and entrepreneurship in order to
address the challenge of skill deficiency in 2015.
Which of the statement(s) given above is/are incorrect?
(a) 1 and 2 (b) Only 4 (c) 1 and 4 (d) None of these
106 Practice Set 4

98. Consider the following statements about the functioning of ecosystem.


1. Complex foodwebs are more frequent in the ecosystem than simpler food chains.
2. Iron and zinc are essential nutrients in the food chain.
3. There is uni-directional flow of energy in an ecosystem.
4. Nutrients in an ecosystem move in a cyclic manner.
Which of the statements given above are correct?
(a) 1 and 2 (b) 1, 2 and 3
(c) 1, 3 and 4 (d) 3 and 4

99. Which of the following activities would be included in the revenue expenditure of the Central
Government?
1. Expenditure on building roads 2. Interest payments
3. Grants to State Governments
Select the correct answer using the codes given below.
(a) 1 and 3 (b) Only 2
(c) 2 and 3 (d) All of these

100. Consider the following statements about Kathakali dance.


1. Kathakali is a traditional dance form of Karnataka.
2. Kathakali is a blend of dance, music and acting and dramatised stories.
3. Kathakali dance is mainly interpretive, with actors wearing masks and headgears.
Which of the statements given above are correct?
(a) 1 and 2 (b) 2 and 3
(c) 1 and 3 (d) All of these
Stage 2 Improve Your Weaker Areas 107
108 Practice Set 4
Answers with Explanations
1. LPG is a flammable mixture of hydrocarbon gases- 7. The Government’s response to the Revolt was restrained
Propane (C 3H8 ) and Butane (C 4H10 ). Commonly mixture of both and they did not use violent force as in the case of civilian
gases depending on the season-in winter more propane, in rebellions and tribal uprising, that is due to borrowing
summer more butane. It was first produced in 1910 by experience of the santhal uprising and the Revolt of 1857.
Dr Walter Snelling and the first commercial products appeared Also, intelligentsia and missionaries pretended their support,
in 1912. Usually, butane and isobutane are mixed with propane which forced the government to appoint a commission to
in various proportions, depending on the intended use of the inquire into the problem of Indigo cultivation. So, Statement 2
fuel. In case of ethane and methane, the pressures required to and 3 are correct.
liquefy and the amount of metal necessary to hold them under
8. The Pancaratra system belongs to Vaishnavites school/
these pressures makes their handling in commercial quantities
sects. The doctrinal aspect of the cult developed in the early
impractical.
centuries of Christian era, when era of the names given to it
2. DMHP is an approach to decentralise mental healthcare in was Pancaratra. It is Vaishnava concept in which Lord Vishnu
the community using the public health infrastructure and other appear himself in four forms such as Vasudeo, Sankrashana,
resources. This model has been pilot tested in Bellary district of Pradumana and Anirudha.
Karnataka state and fond to be very useful to address the basic
9. Nitrogen gas makes up 78% of our atmosphere and
mental health needs of the population.
nitrogen is also a part of many molecules essential to life like
3. Project Elephant was launched in 1992 by the Ministry of proteins, nucleic acids (DNA and RNA) and some vitamins.
Environment and Forests to provide financial and technical Nitrogen is found in other biologically important compounds
support of wildlife management efforts by states. It aims to free such as alkaloids and urea too.
range of Wild Asian Elephants which provides for reducing During lightning, the high temperatures and pressures
man-animal conflict, migration corridor, awareness generation, created in the air convert nitrogen into oxides of nitrogen.
eco-development of habitat and better veterinary care. It has These oxides dissolve in water to give nitric and nitrous acids
joined E-8 group for conservation of animal i.e. elephants. and fall on land alongwith rain. These are then utilised by
4. Contour ploughing is ploughing parallel to the contours of a life-forms on the Earth.
hill slope to form a natural barrier for water to flow down the 10. The Bhakti Movement was a religious movement of the
slope while contour barriers are obstructions made on the medieval period that promoted the belief that salvation was
slopes with the help of stones, grass, trenches in the soil to attainable by everyone. The Movement is closely related to
collect water. Thus, statement 3 is incorrect. Islamic Sufism. The Bhakti Movement originated in 7th
5. The scheme was launched by Indian Government in July 2015 century Tamil Nadu and spread northwards through India. A
with an aim to provide 24 ´ 7 uninterrupted electricity supply to true devotee does not want heaven or moksha. He only
each rural household across the country by 2022. It replaces the wants to chant the Lord’s name and be born again and again
existing RJGGVY and draws its inspiration from the similar to sing his praise. Therefore, all the other features are
pioneering scheme implemented by Gujarat Government. It characteristic of Bhakti Movement.
focuses on feeder separation (rural households and agricultural) 11. They grew wheat, barley, pulses, rice, linseed, mustard
and strengthening of sub-transmission and distribution etc. They did not receive heavy rainfall, they relied on
infrastructure including metering at all levels in rural areas. It helps irrigation as well for agriculture. They caught fish and hunted
in providing round the clock power to rural households and wild animals as well.
adequate power supply to agricultural consumers.
12. Ozone depletion has been a major environmental
6. The Government of India has announced a national concern since 1980s. Ozone covers the Earth as an umbrella
manufacturing policy with the objective of enhancing the share and protect the biosphere from hazardous UV rays.
of manufacturing in GDP to 25% within a decade and creating
UV rays can lead to skin cancer, skin burn and other
100 million jobs. It also seeks to empower rural youth by
diseases to life including human on Earth.
imparting vocational training and skill development to make
Chlorofluorocarbons and other halogen compounds react
them employable. National Manufacturing Policy (NMP)
with the ozone at the upper atmosphere and deplete the
proposes to establish 4-5 NMIZs as green field integrated
ozone layer. Ozone itself acts as a greenhouse gas in lower
industrial townships with world class infrastructure financed by
atmosphere (troposphere).
the Central Government in partnership with respective State
Government with a competitive regulatory environment for Montreal protocol deals with the restriction of ozone
attractive investments. All of the statements are correct except depleting substances and Vienna Convention deals with the
Statement 3. protection of ozone layer.
110 Practice Set 4

13. Many hazardous toxic elements are found in e-wastes. 19. NSSK is a programme aimed to train health personnel in
Lead used in soldering the circuit board that causes basic newborn care and resuscitation has been launched to
damage of nervous system, beryllium found in battery and address care at birth issue i.e. prevention of hypothermia,
are carcinogenic in nature, arsenic used in semi prevention of infection, early initiation of breast feeding and basic
conductors and is a poisonous element that damage newborn, resuscitation, newborn care and resuscitation is an
digestive tract. important starting point for any neonatal programme and is
Mercury found in batteries and circuit boards and attack the required to ensure the best possible start in life.
nervous system, endocrine system and harm unborn 20. The correct sequence of energy resources in the increasing
foetus. Antimony used in production of diodes and order of consumption is wind energy, nuclear energy,
batteries and as toxic as arsenic. Cadmium is used in hydroelectricity, petroleum and natural gas and coal.
soldering semi conductors and chip resistors that damage
lung kidneys and liver. Though silicon is found in e-waste, 21. Andaman and Nicobar Islands are situated roughly between
but is not a potential toxic and plutonium is not found that is 6°N-14°N latitude and 92 0 E to 940 E longitude. They are divided
radioactive in nature. into two broad categories: Andaman in North and Nicobar in
South. They are separated by a water body which is called the
14. When a resolution for the removal of the Speaker is
Ten Degree Channel. These islands are believed to be an
under consideration of the house he/she cannot preside at
elevated portion of submerged mountains. However, some
the sitting of the House, though he/she may be present.
smaller islands are volcanic in origin. The Barren Island is an
However, he/she can speak and take part in the
active volcano in India and is situated in Nicobar Islands.
proceedings of the House at such a time and vote in the
first instance, though not in the case of an equality of votes. 22. The Peninsular drainage system is older than the Himalayan
Such a resolution can be moved only after giving 14 days system and is characterised by largely graded shallow valleys
advance notice. and the maturity of the rivers. The Narmada and the Tapi flow in
15. The first hour of every parliamentary sitting is slotted for trough faults and fill the original cracks with their detritus
question hour. During this time, members asks questions materials. Therefore, there is a lack of alluvial and deltaic deposits
and ministers give answers. A starred question requires an in these rivers. The Godavari is the largest river of Peninsular river
oral answer and hence supplementary questions can system and is also called Dakshin Ganga.
follow. An unstarred questions require a written answer and 23. Wangala is the most significant post harvest festival of the
hence, supplementary questions cannot follow. A short Garos generally held in the second week of November every year.
notice questions is asked by giving a notice of less than 10 It is a Thanksgiving ceremony to Misi Saljong also known as
days. It is answered orally. Pattigipa Rarongipa (The Great Giver) for having blessed the
16. It was Jinnah who demanded for one-third people with rich harvest of the season. The festival was started in
representation to Muslims in Central Legislative Assembly. the year 1976 at Asanang. The festival has grown over the years
Nehru report recommended joint electorate with reservation under the patronage of the Government of Meghalaya.
for Muslims in proportion to Muslim population. All the 24. Western disturbance causes winter and pre-monsoon season
others are recommended by the Nehru Report. rainfall across North-West India. Winter rainfall in this region has
17. All the statements are correct. ‘Gulf of Mannar’ is one of great importance in agriculture, particulate for the Rabi crops. It
the 14 biosphere reserves of India. Along with Nilgiri and originated in the Mediterranean sea. Moisture content is
Sundarbans, it was reorganised on world network of augmented from the Mediterranean sea, the Caspian sea and the
Biosphere reserve by UNESCO. The Ganga Action Plan Persian gulf. So, the Statement 1 is incorrect. They intensify over
Phase-II was merged with National River Conservation Plan the North-West India and move eastwards causing rain in Punjab
in December 1996. The National Museum of Natural History and Haryana and snowfall in Himalayan belt. So the Statement 2
at New Delhi promotes environmental education, also and 3 are correct. The sub-tropical westerly jet-stream play vital
provides resources such as school loan kits for schools to role in bringing these disturbances to India. So, the Statement 4 is
use their curricula on environmental education. ENVIS incorrect.
always acts as a decentralised information network for
environmental information.
25. On the 13th April, 1919 a large crowd of non-violent
protesters, along with Baishakhi pilgrims, had gathered in
18. Sikkim was declared as the first organic farming state of Jallianwala Bagh in the Amritsar, Punjab to protest the arrest of
India. On this occasion, state also launched three new two leaders (Saifuddin Kitchlew and Dr Satyapal) despite a curfew
orchid species development in Sikkim. which had been recently declared. On orders of Brigadier-General
The new species are Reginald Dyer, the army fired on the crowd for 10 minutes. The
(i) Cymbidium Sardar (after Sardar Patel) dead numbered between 370 and 1000 or possibly more. The
(ii) Cymbidium Deen Dayal (after Deen Dayal Upaddhyay) government held an inquiry, chaired by Lord Hunter. While, it was
(iii) Cymbidium Namo (after Narendra Modi) not uncritical of Dyer, it held back from outright censure and the
Chief Minister Pawan Chamling was handed over with Indian members of his commission issued a more critical
organic certificate. minority report.
Stage 2 Improve Your Weaker Areas 111

26. Aryabhatta considered zero not only as a numeral, but also 33. Adhai Din Ka Jhopra at Ajmer was constructed from the
as a concept and a symbol. The other two statements are material obtained after demolishing Hindu temples. It is laid
correct. The discovery of zero enabled finding the exact distance on similar plan as the Delhi mosque.
between Earth and Moon and it also opened up the realm of The Quwwat-ul-Islam mosque was constructed in Delhi by
negative numerals. Qutub-ud-din Aibak around AD 1197. Temple bell hanging
27. At a recently concluded meeting of the Green Climate Fund’s by chains were utilised to construct this mosque, known as
Board in Zambia, developed countries including the US have the ‘Might of Islam’.
refused to commit to a deadline by which the rest of the money The Gol Gumbaz of Bijapur is the mausoleum of Mohammed
($ 4.37 billion) would be deposited with the Fund. At present, Adil Shah. It is the largest cubicle in the world covering a
Green Climate Fund has only $5.83 billion in its kitty. total interior surface of over 1600 sq m.
● The US informed that it would not be able to provide the The tomb of Humayun at Delhi was built by his widow, Bega
pledged $3 billion by December 2015 to the Green Climate Begum. It is the first distinct example of Persian effect on
Fund. The EU finance ministers have also concluded their Mughal architecture. This tomb provided the prototype for
meeting without committing to a clear road-map for delivery of later Mughal architecture for monuments like Taj Mahal of
their fair share towards the annual $ 100 billion corpus target, Agra.
which is to be delivered by 2020.
The Green Climate Fund (GCF) is a fund within the framework of
34. The Satavahanas coins were mainly made by silver,
copper and lead. In Satavahanas era, gold may have been
the UNFCCC founded as a mechanism to redistribute money
used as bullion. The Satavahanas did not issue gold coins.
from the developed to the developing world, in order to assist
the developing countries in adaptation and mitigation practices They issue coins of lead, besides tin, copper and bronze
to counter climate change. money. Coins issued during the reign of Rajaraja Chola have
● It was founded as a mechanism to redistribute money from the
king standing on obverse and king seated on reverse.
developed to the developing world, in order to assist the 35. Natural Region Inhabitants
developing countries in adaptation and mitigation practices to ● Amazon basin in equatorial region Boro
counter climate change.
● Kalahari desert of tropical desert region Bushman
28. SEZ policy in India first came into inception on 1st April, ● Temperate grassland region of Central Asia Kirghiz
2000. The prime objective was to enhance foreign investment. ● Tundra region in Northern Europe Eskimo
So, Statement 2 is wrong. It also aims to provide an international
competitive and hassle free environment for exports of goods 36. All statements are correct with respect to
and services. All other options are true. Non-Cooperation Movement. In December 1920, Congress
met in the Nagpur session. This time the differenes of CR
29. Public policies have to be consistent with the interests of the Das had melted away. He moved the main resolution of
citizens, as indirectly influence the policy making. Judicial review Non-Cooperation. The Ali,Brothers, the Principle leaders of
is the power of courts to determine the constitutionality of action the Khilafat, were arrested in September 1921. Lord Reading
of the legislature and to declare them void if such actions are seemed willing to hold a round table conference with Gandhi
found to be in conflict with the constitutional provisions. Political and other leader.
parties present programmes, policies or values to the people in
Gandhiji’s ability to rally hundreds of thousands of common
the form of manifestos in order to gain their support. They
express demands and present alternatives for policy action. citizens towards the cause of Indian independence were first
seen on a large scale throughout 1921, which aggrivated
30. Since, its inception in 1950 and till 1989, the Election tension between Congress and the Government.
Commission functioned as a single member body. In 1989, the
President appointed two more Commissioners. However, the 37. The rise in GDP may not ultimately gives the equal
two posts were abolished in 1990. Again in 1993, the President distribution of income because rise in GDP may be
appointed two more Election Commissioners. Since then, concentrated in the hand of very few individual or firms.
Election Commission is functioning as a 3 members body. Non-Monetary Exchanges Many activities in an economy
are not evaluated in monetary terms. e.g. the domestic
31. Sutlej is an important tributary of Indus. It rises from Rakas
lake in Tibet. It is an antecedent river. Bhakra Nangal project is services women perform at home are not paid for.
built on this river. It is an international river which enters India in Externalities It refers to the benefits (or harms), a firm or an
Shipki La. individual causes to one another for which they are not paid
(or penalised). Externalities do not have any market in which
32. The Himalayan ranges immediately to the North of the Great
they can be bought and sold.
Himalayan range are called the Trans-Himalayas. Most of the
part of this Himalayan range lies in the Tibet and hence also 38. RBI requires commercial banks to keep reserves in order
called Tibetan Himalaya. The Zaskar, the Ladakh, the Kailash to ensure that banks have a safe cushion of assets to draw
and the Karakoram are the main ranges of the Trans-Himalayan on when account holders want to be paid. RBI uses various
System. Statement 3 is also correct as the Northernmost range policy instruments to bring forth a healthy RDR in
of the Trans-Himalayan ranges in India is the Great Karakoram commercial banks. The first instrument is the Cash Reserve
range also known as Krishnagiri. Ratio (CRR) which specifies the fraction of their deposits that
112 Practice Set 4

banks must keep with RBI. There is another tool called 46. Increasing the Cash Reserve Ratio (CRR) and interest rates
Statutory Liquidity Ratio (SLR) which requires the banks to squeezes the money out of market and might result in decrease
maintain a given fraction of their total demand and time in investments and employment levels. Relaxing stringent
deposits in the form of specified liquid assets. Apart from labour laws, providing incentives to MSME and relaxing FDI
these ratios, RBI uses a certain interest rate called the bank caps in manufacturing and service sector would bring increase
rate to control the value of RDR. Commercial banks can in employment opportunities.
borrow money from RBI at the bank rate when they run short
of reserves.
47. The popular belief among Goan fisherman is that the
crocodiles prey on the predator fish and thus keep high the
39. Gandhara school used grey stone, Mathura school used yield of the edible fish and prawns from mangroves over the
spotted red sandstone and Amaravati school used white years. This popular belief has assumed a religious cover to the
marble. There is no Sanchi school of architecture. Yaksha extent that the worship of crocodile has became annual feature.
prototype was mainly used in the Mathura school and it was
promoted by Kushans.
48. The correct pairs are
Beriberi : Weak muscles and very little energy to work;
40. Bihugeet is a traditional folk music of Asom, performed
Scurvy : Bleeding gums, wounds take longer time to heal;
through Bihu dance in the festival of Bihu. All the other pairs
Rickets : Bones become soft and bent.
are correctly matched. Lavani is the folk music of
Maharashtra, Bhavageete is the folk music of Karnataka and Deficiency Diseases of Vitamins
Pandavani is prevalent in Chhattisgarh. Vitamin A - Night Blindness
41. Article 75 of the Constitution says that the Council of Vitamin D - Rickets
Ministers shall be collectively responsible to the Lok Sabha. Vitamin B1 - Beriberi
Non-confidence motion needs the support of 50 members to Vitamin B12 - Pernicious Anemia
be admitted. Vitamin C - Scurvy
42. The Constitution contains special provisions to safeguard 49. The Mughals did not believe in the rule of primogeniture,
the social, educational and economic interests of the where the eldest son inherited his father’s estate. Instead they
minorities. Some of these are common to both religious followed the Mughal and Timurid custom of coparcenary
minorities and linguistic minorities while some others are inheritance, or a division of the inheritance amongst all the
meant exclusively for linguistic minorities. So, the Statement 1 sons. Peasant revolts challenged the stability of the Mughal
and 2 are correct. As per Article 29(2), no citizen is be denied empire from the end of the 17th century.
admission into any educational institution maintained by the 50. While Buddhism spread, its influence virtually became
state or aided by the state on grounds of religion, race, caste non-existent in India. Jainism never spread to other parts of the
or language. So, the Statement 3 is incorrect. Article 347 world, but Hinduism did spread to South-East Asia. Infact,
deals with special provision relating to language spoken by a biggest Hindu temple is in Combodia (Angkorwat).
section of the population of a state. So the Statement 4 is
correct in this regard. 51. All of the given species are critically endangered species
and enlisted in the IUCN’s red data list.
43. Tiger is one of the key wildlife species in the faunal web.
Tiger population decreases because of many reasons, such 52. Article 76 of the Constitution of India provides for the office
as poaching for trade, shrinking habitat, depletion of prey of the Attorney General for India. He is the highest law officer in
base species, growing human population etc. In 1973 the country and is appointed by the President. He must be a
Government launched ‘Project Tiger’ to conserve the tiger person, who is qualified to be appointed a Judge of the
population. It has also been viewed as a means of presuming Supreme Court.
biotypes of sizeable magnitude. 53. The Chemical Weapon Convention (CWC) is a multilateral
44. Mangroves For the Future (MFF) is partnership based treaty that bars chemical weapons and requires their
initiative promoting investment in coastal ecosystems for destruction within a specified period of time. The convention
sustainable development. MFF provides a collaborative opened for singular on 13th January, 1993 and entered into
platform to help countries, sectors and agencies in the MFF force on 29th April, 1997. According to this treaty, after the year
region tackle the growing challenges to coastal sustainability. 2007, every state party of the convention will never develop
produce, stockpile or chemical weapons.
45. In November 1989, after nearly a decade of negotiations,
the United Nations General Assembly unanimously adopted 54. The Cortes Generals is the legislature of Spain. It is a
the Convention on the Rights of the Child-the CRC. For the bicameral Parliament, composed of the Congress of Deputies
first time in history, an international treaty recognised that and the Senate. The risks dag is the national legislature and the
children are not possessions, but people who have human supreme decision making body of Sweden.
rights. The convention acknowledge that every child has 55. Elemental oxygen is normally found in the form of a
certain basic rights, including the right to life, his or her own diatomic molecule. However, in the upper reaches of the
name and identify, to be raised by his or her parents. atmosphere, a molecule containing three atoms of oxygen is
Stage 2 Improve Your Weaker Areas 113

found. This would mean a formula of O 3 and this is called 64. Petticoat government refers to the early era when Akbar’s
ozone. Unlike the normal diatomic molecule of oxygen, ozone foster mother ruled on his behalf. Jizya was abolished by
is poisonous and we are lucky that it is not stable nearer to Akbar. Most Mughal kings were against it. Akbar issued the
the Earth’s surface. But it performs an essential function where famous ‘Infallibility Decree’, which made him as the religious
it is found. It absorbs harmful radiations from the Sun. This head as well as the king.
prevents those harmful radiations from reaching the surface of
65. (c) Portugese established military and trading stations. Their
the Earth where they may damage many forms of life.
superior military technology, especially use of muskets
56. The secondary treatment of sewage water allows vigorous enabled them to become important players in politics. Trade
growth of useful aerobic microbes into flocs (masses of was regarded as a status symbol and generated excessive
bacteria associated with fungal filaments to form mesh like revenue for the state. Import of horses from Arabia and
structures). While growing, these microbes consume the major Central Asia was very important for warfare.
part of the organic matter in the effluent. This significantly
66. (a) Atmosphere has more freshwater than all surface rivers on
reduces the BOD (Biochemical Oxygen Demand) of the
the Earth. Groundwater is in more quantity than the river
effluent.
freshwater. Freshwater in polar ice caps are more than that is
57. The rulers of allying Indians state were compelled to accept stored in lakes, rivers and groundwater. The average salinity
the permanent stationing of a British Force within their territory of the oceans is 35 parts per thousands.
and to pay a subsidy for its maintenance. Indian rulers were not
67. It is an important steel city of the United States of America.
free to negotiate with any other Indian ruler without consulting
The steel industry at Pittsburgh enjoys locational advantages.
the Governor General.
Some of the raw material such as coal is available locally,
58. (d) Sarojini Naidu, also known by the sobriquet ‘The while the iron ore comes from the iron mines at Minnesota,
Nightingale of India’. She was a child prodigy, Indian about 1500 km from Pittsburgh.
independence activist and poet. Sarojini Naidu was one of the Between these mines, Pittsburgh is one of the world’s best
framers of the Indian Constitution and the first Governor of the routes for shipping ore cheaply. The famous Great Lakes
United Provinces from 1947 to 1949, the first woman to waterway, trains carry the ore from the Great Lakes to the
become the Governor of an Indian state. Her birthday is Pittsburgh area. The Ohio, the Monongahela and Allegheny
celebrated as Women’s Day all over India. rivers provide adequate water supply.
59. (c) Traditionally, Madhubani painting is done on freshly 68. The DTH (Direct To Home) service is basically a digital
plastered mud walls. However now, they are also done on satellite service that provides satellite television programming
cloth, T-shirt, handmade paper and canvas. directly to subscribers home. DD Direct Plus is India’s only
60. Anti-defection law originally provided that the decision of free DTH service provider.
the presiding officer is final in this regard and cannot be DTH transmission eliminates the intervening role of a local
questioned in any court. However, in Kihoto Hollohan Case cable operator since a user is directly connected to the DTH
(1993), the Supreme Court declared this provision as service, and transmission is most preferred in Ku-Band so as
unconstitutional on the ground that it seeks to take away the to avoid the need of larger dish sizes for suitable receiving the
jurisdiction of the Supreme Court and High Court. It held that DTH signals. The DTH signals can be received anywhere
the presiding officer, while deciding a question under the Tenth across the country irrespective of the terrain conditions
schedule function as a tribunal. Hence, his decision like that of provided the area comes under the footprint of the satellite.
any other tribunal, is subject to judicial review on the grounds of So, the Statement 1 and 2 are correct and 3 is incorrect.
perversity.
69. The main objective of the Standing Committees is to
61. Bamboo is considered as sacred and is culturally valued in secure more accountability of the Executive to the Parliament.
North-East of India. It is also a keystone species conserving There are 24 Standing Committees, each consists of 31
potassium in the system. Primitive agricultural practises in members (21 from Lok Sabha and 10 from Rajya Sabha). A
North-East India includes Jhum cultivation. So, all of the minister is not eligible to be nominated as a member of any of
statements are correct. the standing committee. These committees considers the
62. Leaching is a process by which various nutrients percolate demands for grants of the related Ministeries/Departments,
down under influence of water. It is a mechanism of soil examines bills and if referred by the Chairman or the Speaker,
formation distinct from the soil forming process of eluviation, considers national basic long term policy documents
which is loss of mineral and organic collids. It leads to presented to the Houses.
formation of laterite soil. It is also used for removing salt in 70. Ecomark Scheme was introduced in 1991 with its symbol
saline areas. an ‘earthen pitcher’, which symbolise eco-friendliness and our
63. The Comptroller and Auditor General of India (CAG) is traditional heritage.
mentioned in the Constitution of India under Article 148-151. He Eco-Club Scheme is for students to make awareness among
is head of the Indian Audit and Accounts Department. He is the children whereas Eco-Task Force for army men have also
guardian of the public purse and controls the entire financial been launched by the government. Supreme Court ruling on
system of the country. He is appointed by the President and revision of curricula in 1991 is an awareness initiative, but not
does not hold the office till the pleasure of the President. promoted by Government of India.
114 Practice Set 4

71. All of the statements are true regarding Sea Buckthorn 80. The pyramids of numbers and biomass may be upright
except statement (d). Its seven recognised species are native or inverted depending upon the nature of the food chain in
over a wide area of Europe and Asia. the particular ecosystem, whereas pyramids of energy are
72. Normal cells show a property called contact inhibition by always uprights.
virtue of which contact with other cells inhibits their uncontrolled 81. The important reasons for British success over French
growth. Cancer cells appears to have lost this property. As a were
result of this, cancerous cells just continue to divide giving rise ● Britain was commercially superior.
to masses of cells called tumours. Tumours are of two types:
● British in India were supported by the Home Government.
Benign and Malignant. Benign tumours normally remain
● The naval power of the British was far superior to that of the
confined to their original location and do not spread to other
French.
parts of the body and cause little damage. The malignant
● There was full cooperation among the English officers.
tumours, on the other hand, are a mass of proliferating cells
called neoplastic or tumour cells. ● The British could concentrate on wars as there was peace
in England, while the French were too busy fighting wars in
73. Salt works by drying food. Salt absorbs water from foods, Europe.
making the environment too dry to support harmful mold or
82. The Battle of Karnal resulted in the Kohinoor diamond
bacteria.
being taken away by Nadir Shah, the King of Persia after
74. Both of the statements are correct regarding Khejri tree, defeating Mughal army. In 1761 (Third Battle of Panipat),
which is now dying a slow death, scientists and Mughal empire ceased to exist in practice as an all India
environmentalists have warned so. Excessive lopping, empire and was reduced as the Kingdom of Delhi.
indiscriminate cutting of branches are the reasons behind Nizam-ul-Mulk was the first Mughal noble who founded the
decline in total coverage, to this tree, which once considered as state of Hyderabad declaring independence from Mughal
lifeline of rural economy. empire. This paved way for disintegration of the empire. Other
75. Inter Tropical Convergence Zone (ITCZ) is a low pressure nobles founded Bengal, Awadh; Marathas could not now
zone located at the equator where the trade winds converge. In hold onto their reign. Ahmad Shah Abdali lost Punjab; no
the month of July, the ITCZ is located around 20°N - 25°N contemporary ruler could refill the authority of the Mughals,
latitudes over the Gangetic plains and is often called the this paved the way for total disintegration of Mughal reign and
‘monsoon trough’. Due to this shift of ITCZ, the trade winds of rise of British powers.
the Southern hemisphere cross the equator between 40° E and Mughal monarchs never served as the titular head for the
60° E longitudes and start blowing from South-West to English East India Company and there was no political unity
North-West due to Coriolis force. This causes the South-West till 1857, so to say.
monsoon in India.
83. Cybernetic system is a basic function of every ecosystem.
76. (b) The capacity to generate a whole plant from any cell/explant It is the tendency of every ecosystem to regulate and maintain
is called totipotency. The nutrient medium must provide a itself and resist any stress or disturbances upto a certain limit.
carbon source such as sucrose and also inorganic salts, Adaptability is related to species not to ecology. Homeostasis
vitamins, amino acids and growth regulators like auxins, is the property of an ecosystem to tolerate external
cytokinins etc. By application of these methods, it is possible to disturbance or stress. Feedback mechanism helps in
achieve propagation of a large number of plants in very short functioning of the ecosystem by countering the disturbances.
durations. This method of producing thousands of plants Feedback mechanism may be positive or negative.
through tissue culture is called micropropagation.
84. CPRs can be defined as community’s natural resource,
77. All of the statements are correct. Majority of these drugs are where every member has the right of access and usage with
obtained from flowering plant except very few are obtained from specified obligations, without anybody having property rights
fungi. Cocaine is psychoactive drug affecting the central over them. Community forests, pasture lands, village water
nervous system. Morphine is essential for patients suffering bodies and other public spaces where a group larger than a
from terminal illnesses such as cancer, HIV and thalassemia. household or family unit exercises rights of use and carries
Morphine is one of the cheapest and the best known responsibility of management are examples of CPRs.
pain-relieving drug.
85. When a country runs a trade deficit, it is important to
78. PMO’s role is to assist the PM in dealing with all the look at the right side of equation to see whether there has
references which comes to him under the rules of business. It is been a decrease in saving, increase in investment or an
headed by the principal secretary to the PM.PMO is declared as increase in the budget deficit. There is reason to worry about
a department under the GOI allocation of business rules, 1961. a country’s long-run prospects, if the trade deficit reflects
79. The Himalayas harbour a wide range of animals which smaller saving or a larger budget deficit (when the economy
survive in extreme cold. Ladakh’s freezing high altitude areas has both trade deficit and budget deficit, it is said to be facing
are home to yak, the shaggy horned wild ox blue sheep and twin deficits. The deficit could reflect higher private or
kiang. government consumption.
Stage 2 Improve Your Weaker Areas 115

86. Article 326 in part XV of the Constitution mentions adult 95. Ocean currents are like river flow in oceans. There are
suffrage. It is not mentioned under the Fundamental Rights two types of factors influencing ocean currents. They are
section neither under any law enacted by the Parliament of India. primary and secondary forces. Primary forces are
Therefore, it is only a Constitutional Right. ● Heating by solar energy ● Coriolis force
87. El-Nino phenomenon involves the occurrence of warm water ● Wind ● Gravity
current off the Peruvian coast every 3 to 7 years, replacing cold 96. National Population Stablisation Fund is an autonomous
Peruvian current or Humbolt current. Though, there are many body of the Ministry of Health and Family Welfare,
evidences of occurrence of El-Nino and low rainfall in India, but Government of India has launched PRERNA, a responsible
there is no one to one relationship as such. parenthood strategy in 2008 in seven focus states (Bihar,
88. Empowered with knowledge and a drug kit to deliver first Uttar Pradesh, Madhya Pradesh, Odisha, Chhattisgarh,
contact healthcare, every ASHA is expected to be a foundational Jharkhand and Rajasthan).
head of community participation in public health programmes in The strategy recognises and awards couples who have
her village. ASHA will be the Ist port of call for any health related broken the stereotype of early marriage and early childbirth
demands of deprived difficult to access health services. and helped change mindset ‘Santusht’ is a scheme of
89. The Indian soils belong to different categories and hence, Jansankhya Sthirata Kosh (JSK) for high populated states of
differ greatly in their physical properties, chemical composition India. Under this scheme Jansankhya Sthirata Kosh invites
and fertility level. These are largely old and mature soils and are private sector gynecologists and vasectomy surgeons to
deficient in nitrogen, mineral salts and organic matter. Black soils conduct operations in public private partnership mode.
are highly impermeable and water retentive due to 97. The Indian Government launched the Skill India
predominance of clay. The formation of laterite soil takes place in Programme on the occasion of the first ever world youth
those regions which experience wet and dry weather in skills day on 15th July, 2015 with the aims to train over 40
alternative manner between the tropics. crore people in India in different skills by 2022. It includes
90. All the given statements are correct. A bio-fertiliser is a the National Skill Development Mission, National Policy for
substance which contains living microorganisms which, when Skill Development and Entrepreneurship 2015, Pradhan
applied to seed, plant surfaces or soil, colonises the rhizosphere Mantri Kaushal Vikas Yojana Scheme and the Skill Loan
or the interior of the plant and promotes growth by increasing the Scheme. Its logo with the tagline is skilled India, successful
supply or availability of primary nutrients to the host plant. India. The Union Government has created a ministry for skill
Bio-fertilisers add nutrients through the natural processes of development and entrepreneurship in 2014 in order to
nitrogen fixation, solubilising phosphorus and stimulating plant address the challenge of skills deficiency in the country.
growth through the synthesis of growth promoting substances. 98. Simpler food chains are rarely found in an ecosystem
91. Mahatma Gandhi first experimented with Satyagraha in South and as ecosystem is an open system food chains are
Africa in his fight against the racist regime. Satyagraha theory interlinked and forms complex food webs. Iron and zinc are
influenced Nelson Mandela’s struggle in South Africa under micro nutrients in the food chain whereas major essential
apartheid, Martin Luther King, Jr’s and James Bevel’s campaigns nutrients are carbon, nitrogen, phosphorus, potassium,
during the Civil Rights Movement in the United States and many hydrogen, oxygen and sulphur. Energy flow is always
other social justice and similar movements. uni-directional and cycling of nutrients always take place
between the biotic and abiotic component.
92. Perform, Achieve and Trade (PAT) is a market based
mechanism to enhance cost effectiveness and improvements in 99. Revenue expenditure consists of all those expenditures
energy efficiency in energy intensive large industries and of the Government, which do not result in creation of
facilities, through certification on energy savings that could be physical or financial assets. It relates to those expenses
traded. incurred for the normal functioning of the government
departments and various services, interest payments on
93. Vocal learning is the ability to modify acrostic and syntactic
debt incurred by the government and grants given to State
sounds, acquire new sounds via imitation and produce
Governments and other parties (even though some of the
vocalisations. Vocal learning is a critical substrate for spoken
grants may be meant for creation of assets).
language and has only been detected in eight animal groups
despite the mide arry of vocalising species, these include 100. Kathakali is a popular traditional dance form of Kerala. It
human, bats, cetaceans, pinnipeds elephants and three distantly is a blend of dance music and acting, which are mostly
related birds including songbirds, parrots and humming birds. adapted from the Indian epics. The costumes of Kathakali
are suited as per the tenets laid down in Natyashastra.
94. As per Article 21 (A) state shall provide free and compulsory
Kathakali dance is chiefly interpretative. The faces of artists
education to all children of the age of 6 to 14 years and
are painted and a mask is worn, artists wear large
according to Article 24, no child below the age of 14 years shall
headgears in the performance. Kathakali uses almost the
be employed in any factory, mine or any other hazardous
entire body to express the dance.
occupation.
Rough Work
UPSC CIVIL SERVICES (PRE) EXAM

General Studies
Paper 1
Practice Set 5
Time : 2 hrs MM : 200

Instructions

1. This set contains 100 questions. Each question comprises four responses (answers). You will select the response which
you want to mark on the Answer Sheet. In case, if you feel that the correct response is more than one, then mark the
response, which you consider the best. In any case, choose only one response for each questions.
2. All questions carry equal marks.
3. Penalty for Wrong Answer
(i) There are four alternatives to answer every question. If a question is marked wrong, one-third of the marks

IMPROVE YOUR WEAKER AREAS


assigned to that question will be deducted as penalty.
(ii) If a candidate gives more than one answer, it will be treated as wrong answer even, if one of the given answers
happens to be correct and there will be same penalty as to that question, if it has a penalty.
(iii) If a question is left blank i.e. no answer is given by the cnadidate, there wil be no penalty for that question.

1. If firms find that consumers are purchasing more than expected, which of the following would
you expect?
(a) Aggregate Expenditure (AE) will likely be greater than GDP.
(b) The economy will adjust to macro-economic equilibrium as inventories rise and production and
employment fall.
(c) The economy will adjust to macro-economic equilibrium as inventories fall and production and
employment fall.
(d) Aggregate Expenditure (AE) will likely be less than GDP.

2. Consider the following statements.


1. According to the early Samkhya philosophy, the presence of divine agency is not essential to the
creation of the world. The world owes its creation and evolution more to nature than to God.
2. Sage Kapila is considered as founder of the Samkhya school.
3. According to this school, a person can attain salvation through the acquisition of real knowledge
Stage 2

and his misery can be ended forever. This knowledge can be acquired through perception,
inference and hearing.
Which of the statement(s) given above is/are correct?
(a) Only 1 (b) Only 2 (c) Only 3 (d) All of these
118 Practice Set 5

3. Which of the following statements is/are correct about the Prime Minister’s National Relief Fund
(PMNRF).
1. It was established with public contribution to assist displaced persons from Pakistan.
2. The fund consists entirely of public contributions and does not get any budgetary support.
3. The corpus of the fund is invested with banks in fixed deposits.
4. Disbursements are made with the approval of the Prime Minister.
(a) 1 and 2 (b) 3 and 4 (c) 1 and 3 (d) All of these

4. Consider the following statements.


1. Both electrostatic force and the force exerted by magnets are examples of non-contact forces.
2. The force of gravity acts on all of us all the time.
3. The force of friction always acts on all the moving objects and its direction is always opposite to the
direction of motion.
Which of the statement(s) given above is/are correct?
(a) 1 and 2 (b) 2 and 3 (c) Only 1 (d) All of these

5. Ramsar Convention is an intergovernmental treaty that provides the framework for the
conservation and wise use of wetlands.
1. The Ramsar Convention is the only global environmental treaty that deals with a particular
ecosystem.
2. The convention defines wetlands including lakes, rivers, swamps, mangroves, coral reefs, rice
paddies and salt pans etc.
3. The convention was signed in Ramsar, in Iran in AD 2000 and presently has 188 member parties and
India is a party to this convention.
4. Montreux record is a register of wetland sites on the list of wetlands of international importance
where changes in ecological character have occurred.
5. World Wetland’s Day is celebrated each year on 5th March.
Which of the statements given above are correct?
(a) 1, 2 and 4 (b) 1, 2 and 3 (c) 3 and 5 (d) 2, 3 and 5

6. Which of the following free radicals or ions are capable of destroying ozone layer?
1. Chlorine ion 2. Hydroxyl radical
3. Bromine ion 4. Nitric Oxide radical
Select the correct answer using the codes given below.
(a) 1, 3 and 4 (b) 2 and 3 (c) 1 and 4 (d) All of these

7. Consider the following statements regarding Succus entericus.


1. It breaks carbohydrates into glucose.
2. It breaks protein into amino acids.
3. It is secreted by pancreas.
Which of the statement(s) given above is/are correct?
(a) 1 and 3 (b) Only 2 (c) 2 and 3 (d) 1 and 2

8. Which of the following statements is/are correct with regard to currency swap agreements?
1. It helps in increasing liquidity.
2. They are considered to be loans provided by one country to another.
3. They are risk free.
4. Currency swaps are required to be documented on a company’s balance sheet.
Select the correct answer using the codes given below.
(a) Only 1 (b) 1, 2 and 3 (c) 2, 3 and 4 (d) All of these
Stage 2 Improve Your Weaker Areas 119

9. Consider the following statements in the context of art and architecture during Mauryan period.
1. Megasthenes explained the Mauryan capital palace at Pataliputra as magnificent and of high
technical superiority.
2. In Pataliputra, capital of Ashoka, each pillar is made of single piece of sandstone and sculptures of
tigers or elephant on the top of the pillars.
3. Caves were excavated by Mauryan artisans for monks to live in.
Which of the statements given above are correct?
(a) 1 and 2 (b) 2 and 3
(c) 1 and 3 (d) All of these

10. Consider the following statements in the context of ‘Council Entry Programme’ of the Swarajists.
1. Swarajists won 42 out of 101 elected seats in elections held in 1923.
2. Elections were held under the Act of 1919.
3. Vallabhbhai Patel was elected as the President of the Central Legislative Assembly.
4. Swarajists left the legislature only after the Lahore Congress Resolution in 1930.
Which of the statements given above are correct?
(a) 1 and 2 (b) 1, 2 and 4
(c) 3 and 4 (d) All of these

11. Consider the following statements.


1. A project for breeding crocodiles which started in 1974 has been instrumental in saving the crocodile
from extinction in India.
2. The salt water crocodile is found along the Eastern coast and in the Andaman and Nicobar islands.
Which of the statement(s) given above is/are correct?
(a) Only 1 (b) Only 2
(c) Both 1 and 2 (d) Neither 1 nor 2

12. What is the basic force behind the generation of Ocean Thermal Energy Conversion (OTEC)?
(a) Heat generated by the seabed volcanoes
(b) Energy of waves and currents
(c) Temperature difference between the surface and deeper ocean
(d) Rise and fall in the level of tides

13. Examine the following statements about the role of the Speaker of the Lok Sabha in the
legislative business of the House.
1. Speaker is not final interpreter of the provisions of the Constitution of India.
2. The Speaker does not participate in any business in the committees.
3. The Speaker certifies whether a bill is a Constitution Amendment Bill.
4. Promulgation of ordinances requires prior permission from the Speaker.
Which of the statements given above are incorrect?
(a) 1 and 2 (b) 1, 3 and 4
(c) 3 and 4 (d) All of these

14. Consider the following pairs regarding Land Revenue System in India during British rule.
1. Permanent settlement : Bengal
2. Ryotwari settlement : Bombay, Madras, Asom and Burma
3. Mahalwari settlement : North-Western Province and Punjab
Which of the pair(s) given above is/are correctly matched?
(a) Only 1 (b) 1 and 2
(c) 2 and 3 (d) All of these
120 Practice Set 5

15. According to the newly amended Kyoto Protocol, which among the followings are considered as
Greenhouse Gases?
1. Carbon dioxide (CO2 ) 2. Methane (CH4 )
3. Nitrous oxide (N2O) 4. Hydrofluoro carbons (HFCs)
5. Perfluoro carbons (PFCs) 6. Sulphur hexafluoride (SF6)
7. Nitrogen trifluoride (NF3 ) 8. Sulphur dioxide (SO2 )
Select the correct answer using the codes given below.
(a) 1, 2, 3 and 8 (b) 4, 5, 6 and 8 (c) 1, 2, 3, 4, 5, 6 and 7 (d) All of these

16. The expert committee on Prasar Bharti submitted its recommendations to the government. With
regard to this, which of the following statements is not correct?
(a) The committee was headed by Sam Pitroda
(b) It has recommended for administrative and financial autonomy to Prasar Bharti
(c) It has recommended for setting up of ‘Basar Bharti Connect’ as 3rd arm of the Public Service
Broadcast independent of DD and AIR, in order to provide Tele-education
(d) It has recommended for setting up of a regulatory body to ensure public accountability of the
organisation with respect to all the content broadcast

17. Consider the following statements regarding the legislative process in Parliament.
1. A bill can be referred to a Select/Joint Committee of Parliament at the first, second or third reading.
2. A bill passed by one House, must be passed by the other House within 6 months of its receiving the
bill.
3. At the joint-sitting of the two Houses to pass a Bill, no new amendments can be made to the bill and
only the differences between the Houses are resolved through voting.
Which of the statement(s) given above is/are correct?
(a) Only 1 (b) Only 2 (c) 2 and 3 (d) All of these

18. Consider the following statements with reference to Kudankulam nuclear plant.
1. The Light Water Reactor (LWR) is a type of thermal reactor that uses heavy water as its coolant and
neutron moderator and a solid compound of fissile element as its fuel.
2. The plant’s reactor is the first Pressurised Water Reactor (PWR) belonging to the Light Water Reactor
(LWR) category in India.
3. It is the 21st nuclear power reactor in the country.
Which of the statement(s) given above is/are correct?
(a) Only 1 (b) 2 and 3 (c) Only 2 (d) All of these

19. Consider the following statements about Anglo-Maratha Wars.


1. First Anglo-Maratha War was fought during 1775-82. It was concluded by Treaty of Salbai. It was not
conclusive.
2. Second Anglo-Maratha War established the supremacy of Britishers over Maratha. It was concluded
by Treaty of Bassein.
Which of the statement(s) given above is/are correct?
(a) Only 1 (b) Only 2 (c) All of these (d) None of these

20. Under Gandhi-Irwin Pact (5th March, 1931) the Government of British India agreed to
1. release all political prisoners.
2. return of confiscated lands not yet sold to third parties.
3. withdraw all pending cases against the political workers.
4. permit peaceful picketing of liquor, opium and foreign cloth shops.
Which of the statements given above are correct?
(a) 1 and 4 (b) 1, 2 and 3 (c) 2, 3 and 4 (d) All of these
Stage 2 Improve Your Weaker Areas 121

21. Which of the following characteristics are associated with Persistent Organic Pollutants (POPs)?
1. Biomagnification
2. Bioaccumulation
3. Resistance to environmental degradation.
4. Travel only in the areas which are near to the source of the pollutant.
5. POPs are found all over the world including Antarctica.
Select the correct answer using the codes given below.
(a) 1 and 3 (b) 1, 2, 3 and 4 (c) 2, 4 and 5 (d) 1, 2, 3 and 5

22. Consider the following statements.


1. Montreal Convention corresponds to the protection of ozone layer.
2. Vienna Convention corresponds to substances that deplete ozone layer.
3. India has ratified both Montreal and Vienna Conventions.
Which of the statement(s) given above is/are correct?
(a) Only 1 (b) Only 3 (c) 2 and 3 (d) All of these

23. Consider the following points of differences between a Private Member Bill and a Government Bill.
1. Private Bill has to be drafted by the member himself without any secretarial assistance unlike a
Government Bill.
2. Introduction of a Private Bill requires 1 month notice as compared to the 7 days in case of Government
Bill.
3. Private Bill reflects the stand of the opposition, whereas Government Bill reflects the stand of the
ruling party or coalition.
4. No Private Bill has ever been passed in the Indian Parliament unlike Government Bills.
Which of the statements given above are correct?
(a) 1 and 2 (b) 1, 2 and 3 (c) 2 and 3 (d) 1 and 4

24. Consider the following statements about sulphuric acid.


1. It is produced in the upper atmosphere of Venus.
2. It is mainly used in the wet process for the production of phosphatic fertilisers.
3. Aluminium sulphate is obtained from the reaction of sulphuric acid and aluminum trihydrate.
4. It is present in batteries used in torch and mobiles.
Which of the statements given above are correct?
(a) 1, 2 and 3 (b) 1 and 2 (c) 1 and 3 (d) All of these

25. Consider the following items of expenditures. Which of the following will come under the
category of non-plan expenditure?
1. Capital expenditure on defence.
2. Revenue expenditure on defence.
3. Programmes related to science and technology, environment and social services.
4. Grants to states and union territories and grants to foreign governments.
Select the correct answer using the codes given below.
(a) Only 2 (b) 1 and 3 (c) 2 and 4 (d) All of these

26. Which of the following is/are true about Rio + 20 Summit?


1. It is also known as United Nations Conference on Sustainable Development.
2. It was held in 2011 in Rio de Janeiro, Brazil.
3. Theme of the summit was green economy with equal emphasis on eradication of poverty.
4. The outcome of the summit was a non-binding document titled ‘our common future’.
Select the correct answer using the codes given below.
(a) 1, 2 and 3 (b) 2 and 4 (c) 1, 3 and 4 (d) 1 and 3
122 Practice Set 5

27. Consider the following statements regarding Harappan civilisation.


1. Like Egypt and Mesopotamia, temples were generally made to worship God and its different forms at
Harappan site.
2. Harappan did not know the art of writing and whatever they want to depict, they did it by inscripting
pictures.
3. Harappan pottery was decorated with designs of trees and circles.
Which of the statement(s) given above is/are correct?
(a) 1 and 2 (b) 2 and 3
(c) Only 3 (d) Only 1

28. Consider the following statements.


1. Rigveda gives elaborate details of the king’s administration of justice.
2. Atharvaveda mentions garments (dursa) and goat-skin (ajina) as articles of trade.
Which of the statement(s) given above is/are correct?
(a) Only 1 (b) Only 2 (c) Both 1 and 2 (d ) Neither 1 nor 2

29. Some species are endemic to a region that they do not found or rarely found beyond its natural
habitat. Consider the following pairs and find out the correct pairs.
Mammals Habitat
1. Barasingha : Boggy areas of Terai
2. Indian wild ass : Little Rann of Kutch
3. Hangul deer : Kashmir
4. One-horned rhinoceros : North-East India
Codes
(a) Only 2 (b) 2 and 4 (c) 1, 2 and 3 (d ) All of these

30. Arrange the following events in correct chronological order.


1. CR Formula 2. Cripps Mission 3. Cabinet Mission 4. Wavell Plan
5. Mountbatten Plan 6. August offer
Codes
(a) 6 , 2 , 1, 4 , 3 , 5 (b) 6 , 2 , 1, 3 , 4, 5 (c) 2 , 1 , 6 , 4 , 3 , 5 (d) 2 , 1 , 6 , 3 , 4 , 5

31. With reference to CFL bulbs, consider the following statements.


1. Unlike other bulbs, CFL are consumer friendly and don’t pose health hazard.
2. The luminous efficacy of a typical CFL is 50-70 lumens per watt (1m/w) and that of a typical
incandescent lamp is 10-17 1m/w.
3. The luminous efficacy of lamps is the number of lumens produced for each watt of electrical power
used.
4. CFLs emit light from a mix of phosphorus inside the bulb each emitting one band of colour.
Which of the statements given above are correct?
(a) 1 and 4 (b) 2, 3 and 4 (c) 3 and 4 (d) All of these

32. Consider the following statements about ‘hot money’.


1. This is the fund which is dumped into a country to get the advantage of a favourable interest rate and
hence brings higher returns.
2. This is the fund which is provided by a bank in US at very short notice and at a very high rate of
interest and for a longer period of repayment.
3. This is the fund which is pushed into market through Hawala or some other such illegal methods and
sometimes referred also as black money.
Which of the statement(s) given above is /are correct?
(a) Only 1 (b) 1 and 2 (c) Only 3 (d) 1 and 3
Stage 2 Improve Your Weaker Areas 123

33. The physical topography of the Peninsular plateau is Senile in nature. Consider the following
statements in this regard.
1. The Aravali is a relict mountain range and an important part of the water divide of India.
2. The Deccan lava plateau is an elevated table land and is a consequence of fissure eruption.
3. The Rajmahal Garo Gap separates the Shillong plateau from the rest part of the Peninsular plateau.
Which of the statement(s) given above is/are correct?
(a) Only 1 (b) 1 and 2
(c) 2 and 3 (d) All of these

34. The Cabinet Secretariat has a very important role in policy formulation in Central Government.
Consider some functions given below.
1. It prepares agenda for meetings of cabinet and provides necessary information and material for its
deliberation.
2. It provides secretarial assistance to the cabinet committees.
3. On the permission of the President of India. It prepares and finalise the rules of business of the
government and allots the business of the government among ministries of Union Government.
4. It secure the document of cabinet decisions by the concerned Ministries/Departments and other
executive decisions.
Which of the above functions are of Cabinet Secretariat?
(a) 1, 2 and 3 (b) 1, 2 and 4
(c) 1, 3 and 4 (d) All of these

35. Kyoto Protocol has provided market based mechanisms to meet the carbon emission targets by the
countries party to the protocol. Consider the following statements regarding the market mechanisms.
1. ‘Joint Implementation’ mechanism is the market mechanism for meeting emission targets between
industrialised countries among Annex B parties.
2. Clean Development Mechanism is associated with green projects taken over by industrialised
countries in developing countries.
3. International emission trading now create an institutional space for ‘carbon market’.
4. The CDM is the main source of income for the UNFCCC Adaptation Fund.
Which of the statements given above are correct?
(a) 1 and 2 (b) 1, 2 and 3 (c) 2 and 4 (d) All of these

36. Which of the following statements about the ‘Economic Drain’ of India by the British is
incorrect?
(a) It disturbed the precarious balance between food supply and population in the country by forcing it
to export foodgrains.
(b) It resulted in turning the balance of trade totally against India and in utilisation of Indian resources
for the growth of the colonial economy
(c) It kept down the rate of India’s capital formation and thereby hold up her industrial progress
(d) It cost India an amount of 15 million pounds per annum

37. Which of the following statement is true regarding FSSAI (Food Safety and Standard Authority
of India).
1. It has been established under Food Safety and Standard Act, 2006.
2. Ministry of Health and Family Welfare, Government of India is the Administrative Ministry for the
Implementation of FSSAI.
3. The Chairperson and Chief Executive officer of FSSAI are appointed by Government of India.
4. The Chairperson is appointed by the Home Minister.
Codes
(a) 1 and 2 (b) 1, 2 and 3 (c) 1 and 4 (d) All of these
124 Practice Set 5

38. Consider the following statements.


1. A person who has hold office as a judge of the Supreme Court is prohibited from practising law
before any court.
2. Supreme Court can declare a law void only on substantive grounds of being violation of Constitution
and not on procedural grounds of being unreasonable.
3. Salaries and allowances of Supreme Court Judges cannot varied to his disadvantage during his term
of office, except during a financial emergency.
4. While interpreting the Constitution, the Supreme Court is guided by Doctrine of Progressive
Interpretation.
Which of the statements given above are correct?
(a) 1 and 3 (b) 1, 3 and 4 (c) 2 and 4 (d) All of these

39. Consider the following statements.


1. The High Court issue writs only for the purpose of enforcement of Fundamental Rights whereas the
Supreme Court can issue writs for the purpose of enforcement of Fundamental Rights and also for
any other purpose.
2. High Courts can issue writs of jurisdiction whereas the Supreme Court cannot issue the writ of
jurisdiction.
3. High Courts issue writs only when the party directly approaches it, whereas the Supreme Court can
issue writs necessarily by way of appeal.
Which of the statement(s) given above is/are correct?
(a) Only 3 (b) 1, 2 and 3 (c) 1 and 3 (d) None of these

40. Consider the following statements.


1. A person who has hold office as a Judge of the Supreme Court is prohibited from practising law
before any court.
2. Constitution provides that salaries and allowances of Supreme Court Judges cannot be varied to his
disadvantage during his term of office, but President can override this guarantee during the
proclamation of financial emergency.
Which of the statement(s) given above is/are correct?
(a) Only 1 (b) Only 2 (c) Both 1 and 2 (d) Neither 1 nor 2

41. Which of the following are correct regarding the Humboldt current?
1. It is a cold current and badly affected in the year of EI-Nino.
2. This makes the East Pacific region rice in microscopic marine animals.
3. It caused drier climatic condition in nearby landmass.
Select the correct answer using the codes given below.
(a) 1 and 2 (b) 2 and 3 (c) Only 2 (d) All of these

42. Which one of the following statements is not correct?


(a) The crust beneath the ocean basin is composed of basaltic rock
(b) Acidic waves are highly viscous with high malign point and are light in colour
(c) Predominance of inter montane plateaus are characteristics feature of young fold mountain regions
of the world
(d) ‘The zone of low-velocity’ of seismic waves is found between outer core and inner core

43. In the winter, we use woollen clothes because


1. wool is a good conductor of heat.
2. there is air trapped in between the wool fibers. This air prevents the flow of heat from our body to the
cold surroundings.
Which of the statement(s) given above is/are correct?
(a) Only 1 (b) Only 2 (c) Both 1 and 2 (d) Neither 1 nor 2
Stage 2 Improve Your Weaker Areas 125

44. Which of the following statements regarding concave and convex mirrors is incorrect?
(a) Concave mirrors are used by dentists, doctors and in torches
(b) Concave mirror is also called as fish eye mirror and these are used in vehicle mirrors
(c) The image on a convex mirror is always virtual, diminished (smaller) and upright
(d) Unlike convex mirrors, concave mirrors show different image types depending on the distance
between the object and the mirror

45. Consider the following kinds of organisms.


1. Fungi 2. Bacteria 3. Flowering plants
Which of the above are used as biopesticides?
(a) Only 1 (b) 1 and 2 (c) 2 and 3 (d) All of these

46. Consider the following statements regarding nitrate.


1. Excess nitrate in drinking water can cause a disease called Methemoglobinemia.
2. The maximum concentration of nitrate in drinking water as per WHO norms is 45 ppm.
Which of the statement(s) given above is/are correct?
(a) Only 1 (b) Only 2 (c) Both 1 and 2 (d) Neither 1 nor 2

47. With reference to buffer stock of foodgrains, consider the following statements.
1. Buffer stock is the stock of foodgrains, namely wheat and rice procured by the government through
Food Corporation of India (FCI).
2. Buffer stock are required to stabilise prices during period of production shortfall through open market
sales.
3. The seasonality of production and procurement is a decisive factor in determining the minimum
norm of foodgrains stocks required in a particular quarter of the year.
4. In addition to buffer norms, government has prescribed strategic reserve of wheat and rice.
Which of the statement(s) given above is/are incorrect?
(a) Only 2 (b) Only 3 (c) 2 and 4 (d) None of these

48. Consider the following statements with regard to Non-Banking Finance Companies (NBFCs).
1. NBFCs can also engage in micro finance activities.
2. NBFCs hold banking licence and provide banking services without meeting the legal definition of
bank.
3. NBFCs are allowed to take limited deposit from the public.
Which of the statement(s) given above is/are correct?
(a) 1 and 2 (b) 2 and 3 (c) Only 1 (d) None of these

49. Consider the following statements about Bioleaching.


1. Bioleaching is the extraction of specific metals from their ores through the use of bacteria.
2. Bioleaching is more environmentally friendly than traditional extraction methods.
3. Toxic chemicals are sometimes produced in the process. Heavy ions such as iron, zinc and other
arsenic leak during acid mine drainage.
Which of the statements given above are correct?
(a) 1 and 2 (b) 1 and 3 (c) 2 and 3 (d) All of these

50. Consider the following statements about sound.


1. The loudness of sound depends on its frequency. When the frequency of vibration is large, the sound
produced is loud.
2. The amplitude determines the shrillness or pitch of a sound.
3. In humans, the sound is produced by the larynx.
Which of the statement(s) given above is/are correct?
(a) 1 and 2 (b) Only 3 (c) All of these (d) None of these
126 Practice Set 5

51. Which of the following mechanisms is not associated with chemical weathering?
(a) Solution (b) Abrasion (c) Carbonation (d) Oxidation

52. Consider the following statements.


1. The Chief Justice of India is empowered to appoint ad hoc judges in the Supreme Court from among
judges of High Courts, only after consultation with Chief Justice of High Court concerned.
2. The mode of a removal of a Judge of a High Court in India is same as that of removal of a Judge of the
Supreme Court.
3. After retirement from the office, a permanent judge of a High Court cannot plead or act in any court
or before any authority in India.
4. The Chief Justice of India may invite a retired Judge of the Supreme Court to sit and act as Judge of
the Supreme Court, only with the previous consent of the President.
Which of the statements given above are correct?
(a) 1 and 3 (b) 1, 2, 3 and 4 (c) 2 and 4 (d) 1, 2 and 4

53. Which of the following statements regarding the Pro–Tem Speaker of Lok Sabha is/are correct ?
1. After the Lok Sabha elections, the President administers oath to the Pro–Tem Speaker, which is same
as oath for membership of Lok Sabha.
2. The Pro–Tem Speaker conducts the proceedings of the House, when both the speaker’s and the
Deputy Speakers posts lie vacant due to death, resignation etc.
Select the correct answer using the codes given below.
(a) Only 1 (b) Only 2 (c) Both 1 and 2 (d) Neither 1 nor 2

54. Consider the following statements about the cave paintings of Ajanta, situated near Aurangabad,
Maharashtra.
1. The execution period of these cave paintings extends to about 800 years.
2. The subject matter of these paintings is mostly the Buddhist Jatakas.
3. Jatakas were the sermons and teachings of Buddha during his entire life.
4. These paintings lack in decorative patterns of ceilings etc.
Which of the statements given above are correct?
(a) 1 and 2 (b) 2, 3 and 4 (c) 1, 2 and 3 (d) All of these

55. Jainism aims at attainment of freedom from worldly bonds. For this ‘they believe no ritual is
required for the liberation.’ It can be obtained through famous jewels of Jainism. Which of the
following are jewels of Jainism?
1. Right teacher 2. Right knowledge 3. Right effort 4. Right action
5. Right observation 6. Right faith
Select the correct answer using the codes given below.
(a) 2, 4 and 6 (b) 1, 2, 3 and 4 (c) 2, 4, 5 and 6 (d) 3, 5 and 6

56. Conservation of biodiversity is quite necessary to maintain the ecological balance on the Earth.
What are the correct statements among the following, regarding conservation of biodiversity?
1. In-situ conservations are done through protected areas as national parks, wildlife sanctuaries and
zoos.
2. Ex-situ conservations are the technique of conserving species outside their natural habitat like
botanical garden, breeding centres and conservation of wetlands.
3. IUCN states that it is essential to include atleast 10% of all ecosystems as protected areas of
biodiversity is to be conserved in the long-term.
4. Tiger is the first animal to be protected through a National Programme in 1973.
Select the correct answer using the codes given below.
(a) 1 and 2 (b) 1, 2 and 4 (c) 3 and 4 (d) All of these
Stage 2 Improve Your Weaker Areas 127

57. The endangered species has the largest mass nesting site in Odisha followed by Coast of Mexico
and Coast Rica. Its unique mass nesting site is called Arribada. The nesting site is now protected
as a wildlife sanctuary in 1997. Now they are threatened due to their eggs fall pray to jackles,
dogs, crabs, birds etc. The nesting site lies on the mouth of two famous rivers in Odisha. Name
the species which most suitably explain this.
(a) Olive ridley turtle (b) Leather back sea turtle
(c) Piping Plover (d) Southern Leopard frog

58. Consider the following statements about Rhizobium.


1. Rhizobium lives in the roots of gram, peas, moong beans and other legumes and provides them with
nitrogen.
2. It converts atmospheric nitrogen into plant soluble ammonium.
3. It can make its own food.
Which of the statement(s) given above is/are correct?
(a) 1 and 2 (b) 1 and 3 (c) 2 and 3 (d) Only 3

59. The ultrasound technique is used frequently in obstetrics instead of X-rays based techniques.
What are the principles behind this?
1. Ultrasound is a non-ionising imaging technique.
2. Ultrasound gives a clear picture of soft tissues, which do not show up in X-rays.
3. Ultrasound causes almost no health problems (for a mother or unborn foetus).
Select the correct answer using the codes given below.
(a) 2 and 3 (b) 1 and 2 (c) 1 and 3 (d) All of these

60. Consider the statements in light of landslides in Indian. It is caused due to


1. poor ground conditions.
2. geomorphic phenomena
3. natural physical force
4. heavy spells of rainfall coupled with impeded drainage.
5. vegetation removal and excavation at the toe of stope of mountains.
Which of the statement(s) given above are correct?
(a) 1, 3 and 5 (b) 1, 2 and 3 (c) 1 and 4 (d) All of these

61. Consider the following statements about Bhakti Movement.


1. Nayanars and Alvars spread their movement in North. In start of their movements, they were popular
in South only.
2. Kabir denounced formal worship, even namaz. He neither worshipped idols, nor took holy baths in
river or pilgrimages.
Which of the statement(s) given above is/are incorrect?
(a) Only 1 (b) Only 2 (c) All of these (d) None of these

62. Consider the following statements with reference to National Food for Work Programme.
1. It was launched on 14th November, 2004 in 150 most backward districts of the country with the
objective of intensifying the generation of supplementary wage employment.
2. The programme is open to all rural poor, who are in need of wage employment and desire to do
manual unskilled work.
3. This programme along with NREGA is still run in these 150 districts as a supplementary programme
to benefit the poor.
Which of the statement(s) given above is/are correct?
(a) 1 and 3 (b) 1 and 2 (c) Only 2 (d) All of these
128 Practice Set 5

63. Green Energy Corridors project is to facilitate


(a) the availability of renewable energy into the national grid
(b) the transportation of the fuels and other energy products from one region to another
(c) the construction of environment friendly energy providing plants all over India
(d) the construction of industries based on clean fuel technology in India

64. Consider the following statements regarding the powers of the Governor.
1. Appointment and removal of State Public Service Commission members is done by the Governor.
2. The Governor acts as the Chancellor of all Universities in the state.
3. He causes to be laid before the State Legislature CAG reports relating to the state.
4. The Chief Minister and other ministers hold office during his pleasure.
Which of the statements given above are correct?
(a) 1 and 2 (b) 3 and 4
(c) 2, 3 and 4 (d) All of these

65. Which of the following statements regarding the Jet streams is/are correct?
1. Jet streams are caused by a combination of the Earth’s rotation on its axis and atmospheric heating by
solar radiation.
2. The strongest Jet streams are the polar jets.
3. The major Jet streams of the Earth flow from West to East.
Select the correct answer using the codes given below.
(a) Only 1 (b) 1 and 2
(c) 2 and 3 (d) All of these

66. Which of the following statements regarding tropical moist deciduous forests is/are correct?
1. The annual rainfall in these forests is 100-200 cm and the mean annual temperature of about 27°C.
2. The trees of these forests shed their leaves for about 6-8 weeks, during early summer due to
insufficient moisture availability for the leaves.
3. Sal, teak, mango, bamboo and rosewood are some of the trees of moist deciduous forests.
Select the correct answer using the codes given below.
(a) 1 and 2 (b) 2 and 3
(c) Only 3 (d) All of these

67. Which of the following can be said to be the centralising features of the Indian Constitution in
the context of centre-state relations?
1. Single integrated judiciary 2. All India services
3. Emergency provisions 4. Bicameral legislature at the centre
Select the correct answer using the codes given below.
(a) 1 and 2 (b) 1, 2 and 3
(c) 2 and 3 (d) 1, 3 and 4

68. Consider the following statements about the evolution of Indian music.
1. Haripala’s ‘Sangeeta Sudhakara’ written in 14th century AD are earliest evidence of classification of
Carnatic and Hindustani forms.
2. Today’s Carnatic music is actually said to be originated from the capital city of Deogiri’s of Yadava
kingdom.
3. Carnatic music flourished under the Vijayanagara Empire of king Krishnadevaraya.
Which of the statements given above are correct?
(a) 1 and 2 (b) 2 and 3
(c) 1 and 3 (d) All of these
Stage 2 Improve Your Weaker Areas 129

69. Consider the following statements about the Securities and Exchange Board of India (SEBI).
1. SEBI is a quasi-legislative, quasi-judicial and quasi-executive body.
2. The appeal against SEBI decision can be made to securities appellate tribunal.
3. The second appeal can be made to High Court.
Which of the statements given above are correct?
(a) 1 and 2 (b) l and 3
(c) 2 and 3 (d) All of these

70. Consider the following statements with regard to Mahatma Gandhi Pravasi Suraksha Yojana
(MGPSY).
1. It is pension and life insurance fund scheme for blue-collar Indian workers.
2. It will cover workers in Emigration clearance required countries.
3. Government of the country where the worker is located will contribute a fixed amount of the workers
account.
Which of the statements given above are correct?
(a) 1 and 2 (b) 2 and 3
(c) 1 and 3 (d) All of these

71. Consider the following statements about the Laterite Soils of India.
1. Laterite soils develop in areas with low temperature and high rainfall.
2. These are generally the result of ‘leaching’ due to high tropical rainfall.
3. These soils are rich in organic matter, but lack the oxides of iron and ‘potash’.
Which of the statement(s) given above is/are correct?
(a) Only 3 (b) Only 2 (c) Only 1 (d) All of these

72. Consider the following.


1. Rooted aquatic plants 2. Phytoplankton
3. Free floating stage 4. Reed swamp stage
5. Sedge marsh or meadow stage 6. Woodland stage
7. Climax stage
Which of the above stages are intermediate order of succession in a hydrosere?
(a) 1, 2, 3, and 4 (b) 1, 3, 5 and 6 (c) 2, 4, 5 and 6 (d) All of these

73. Acid rain is an invisible threat to any ecosystem in the world. Which of the following findings
are correct about acid rain?
1. Sulphuric acid and nitric acid are two main acids that dissolve in the water in the atmosphere and
fall to the gound as acid rain.
2. Industrialisation increased the occurrence of acid rain.
3. Acid rains are a great threat to Western European countries and much of the acid rain occurs as
acidic snowfall.
4. Acid rain never be a threat to wetlands as it is diluted in the water, but much threat in land
ecosystem.
Select the correct answer using the codes given below.
(a) 1 and 2 (b) Only 2 (c) 1, 2 and 3 (d) All of these

74. What does ‘Bio-prospecting’ refers to?


(a) Exploring an area to measure its diversity richness
(b) Conservation of biodiversity using cry methods
(c) Extinction of a species due to the extinction of its prey
(d) Exploring molecular, genetic and species level diversity for products of economic importance
130 Practice Set 5

75. Consider the following statements regarding municipal structures other than those specified in
the Constitution.
1. Notified Area Committee can be set up only under an act of State Government.
2. Urban township form of municipal body has no elected members.
3. Cantonment Boards function under the control of Ministry of Defence.
4. Port Trusts are created by an Act of Parliament. It consist of both elected and nominated members.
Which of the statements given above are correct?
(a) 1, 2 and 3 (b) 2, 3 and 4
(c) 1 and 4 (d) 1 and 2

76. Consider the following statements about new Air Quality Index (AQI) in India.
1. Union Government has launched AQI in 2015 for monitoring the quality of air on real time basis and
is a part of the government’s Swachh Bharat Abhiyan.
2. The Index considers seven pollutants namely Particulate Matter-10, nitrogen oxide, sulphur dioxide,
ozone, carbon monoxide, ammonia and lead.
3. The new AQI ues colour-coded table to show air-quality and its effects on health for the first time.
4. It acts as one number-one colour-one description for the common man to judge the air quality. It is
based on recommendations made by the IIT, Kanpur.
Which of the statement(s) given above is/are incorrect?
(a) Only 2 (b) 1 and 2
(c) 3 and 4 (d) 2 and 4

77. Which of the following statements regarding Black soils are correct?
1. The fertility of Black soil is generally low due to intensive leaching.
2. The black colour of the Black soil is due to the presence of Titaniferous Magnetite.
3. Black soil is also called the self ploughed soil.
Select the correct answer using the codes given below.
(a) 1 and 3 (b) 2 and 3
(c) 1 and 2 (d) All of these

78. Consider the following statements regarding Inter-State relations in India.


1. Zonal Councils have been set up in India under Article 263 of the Constitution.
2. President of India is the head of the Inter-State Council set up in 1990.
3. Recommendations of the Inter-State Councils set up under the Constitution on disputes between
states are binding in nature.
4. National Integration Council is a non-constitutional body set up to combat communalism, casteism,
regionalism etc.
Which of the statement(s) given above is/ are correct?
(a) 1, 2 and 4 (b) 2 and 3
(c) Only 4 (d) 1 and 3

79. Which of the following is/are not a provision(s) related to financial emergency declared under
Article 360?
1. Salaries of all classes of persons can be reduced except those, whose salaries are provided under the
Constitution not to be reduced after their appointment.
2. Money Bills and other Financial Bills cannot be passed by State Legislature without prior
recommendation of the President.
3. Proclamation of financial emergency has to be renewed by Parliament every 6 months.
Select the correct answer using the codes given below.
(a) Only 1 (b) 1 and 2
(c) Only 3 (d) All of these
Stage 2 Improve Your Weaker Areas 131

80. Recently, India achieved a significant milestone in nuclear technology when the Pressurised
Water Reactor (PWR) aboard INS Arihant achieved criticality. The PWR aboard INS Arihant
differs from PHWR of India in which of the following ways?
1. PWR uses enriched uranium and PHWR are using natural uranium.
2. PWR uses ordinary water as both moderator and coolant.
3. PWR reactors require much lesser enrichment.
Select the correct answer using the codes given below.
(a) Only 1 (b) 1 and 2
(c) 2 and 3 (d) All of these

81. Which of the following statements regarding the financial relations between the union and the
states is/are not true?
1. The power to tax is broadly similar to the power to make laws as enumerated in the 7th Schedule.
2. Distribution of taxes between the union and the states from the distributive pool is done on the basis
of a fixed formula which has to be specified in the Parent Law.
3. According to the Constitution, union and state properties are exempted from mutual taxation.
Select the incorrect answer using the codes given below.
(a) Only 1 (b) Only 2
(c) 1 and 3 (d) All of these

82. Consider the following statements.


1. The DMIC project presently covers industrial cities which are going to be developed along the
railway corridor with the Japanese assistance.
2. India has been the first recipient of Japanese Official Development Assistance (ODA) fund for
infrastructural development.
3. India and Japan are the Ist countries to strike a high level security and strategic partnership under ‘
2 + 2 Agreement’.
Which of the statement(s) given above is/are correct?
(a) Only 1 (b) 1 and 2
(c) Only 2 (d) All of these

83. In the context of evolution of Earth, what do yo understand by ‘Degassing’?


(a) Injection of gases from other plants into the Earth
(b) Transfer of gases between Earth and other planets
(c) Outpouring of gases from the Earth’s interior
(d) None of the above

84. Which of the following crops are Rabi crops?


1. Wheat 2. Gram 3. Barley 4. Millet
Select the correct answer using the codes given below.
(a) 1, 2 and 3 (b) 2, 3 and 4
(c) 1, 3 and 4 (d) 2 and 3

85. Which of the following statement is not correct regarding the coastal plain of India?
1. The entire coastal plain is aggraditional and emergent in nature.
2. The Backwaters are the special features of the Malabar plain.
3. Most of the Creeks and Lagoons are located on the East coastal plain.
Select the correct answer using the codes given below.
(a) Only 1 (b) 1 and 3
(c) 2 and 3 (d) 1 and 2
132 Practice Set 5

86. Consider the following statements regarding early Islamic architecture in India.
1. The prayer chamber of the mosque was spacious whereas the shrine of the temple was comparatively
small.
2. The structure of mosque was dark and closed where as the temple was open and light.
3. In both the styles of architecture, the ornamental decoration was very vital.
Which of the statements given above are correct?
(a) 1 and 2 (b) 2 and 3
(c) 1 and 3 (d) All of these

87. Consider the following statements regarding Sufi Movement.


1. Chishti and Suharawardi Silsilahs were Sufi orders of ‘Be-shara’ division.
2. Ba-shara division were those who followed the Islamic Law (Sharia).
3. Farid-ud-Din was of Be-shara division, whose some of the verses were quoted in the Adi-Granth of
the Sikhs.
4. Activities of Suharawardi were mainly limited to Punjab and Multan.
Which of the statements given above are incorrect?
(a) 2 and 4 (b) 1, 2 and 4
(c) 1, 3 and 4 (d) 1 and 3

88. Consider the following statements.


1. The energy flow in an ecosystem means transfer of organic molecules.
2. The energy flow in an ecosystem is cyclic.
Which of the statement(s) given above is/are correct?
(a) Only 1 (b) Only 2
(c) Both 1 and 2 (d) Neither 1 nor 2

89. Which one of the following pair is incorrectly matched?


National Park Conserved Wildlife
(a) Great Himalayan National Park : Snow leopard
(b) Kaziranga National Park : One-horned rhinos
(c) Bandipur National Park : Bengal Tiger
(d) Dudhwa National Park : Swamp deer

90. Consider the following statements.


1. Hardness of water is indicated by lack of foam formation by soap.
2. Temporary hardness is due to bicarbonates of magnesium and calcium.
3. Permanent hardness of water is due to sulphate and chloride of calcium and magnesium.
4. Permanent hardness can be removed by boiling.
Which of the statements given above are correct?
(a) 3 and 4 (b) 1, 2 and 3
(c) 2 and 3 (d) All of these

91. Consider the following statements regarding security market.


1. Futures are securities or commodities sold or bought with the assumption of delivery at a later stage.
2. If the Reserve Bank of India sells security in the market, it will result in mopping up of excess
liquidity in the economy.
3. Capital market means financial market dealing in long-term funds.
Which of the statements given above are correct?
(a) 1 and 2 (b) 2 and 3
(c) 1 and 3 (d) All of these
Stage 2 Improve Your Weaker Areas 133

92. Which of the following are the functions of Agriculture Produce Market Committee (APMC)?
1. To ensure complete transparency in pricing system and transactions taking place in market area.
2. Providing market-led extension services to farmers.
3. Promoting agricultural processing including activities for value addition in agricultural produce.
4. Setting up and promoting public-private partnership in the management of agricultural markets.
Select the correct answer using the codes given below.
(a) 1, 2 and 4 (b) 1 and 2
(c) 1 and 4 (d) All of these

93. Consider the following pairs.


Species Type of Vegetation
1. Chir : Thorny Bushes Forests
2. Babool : Tropical Deciduous
3. Sundari : Mangrove Forests
4. Deodar : Coniferous
Which of the pairs given above is/are correctly matched?
(a) 1, 2 and 3 (b) 2, 3 and 4
(c) 1 and 3 (d) 3 and 4

94. Consider the following statements regarding a crop.


1. This crop requires moderate temperature and rainfall during growing season and bright sunshine at
the time of harvest.
2. It thrives best in well-drained loamy soil.
3. In India, it is grown in winter.
Identify the crop based on the above conditions.
(a) Cotton (b) Wheat (c) Maize (d) Groundnut

95. The Sangam literature can be roughly divided into two groups i.e. narrative and didactic. They
are respectively called
(a) 8 major works and 8 minor works (b) Tolkkappiyam and Tirukkural
(c) Silappadikaram and Manimekalai (d) Melkannakku and Kilkanakku

96. Bioremediation is now a popular work in maintenance of nature’s overall ecological balance.
Consider the following statements.
1. Bioremediation is the use of living organism (primarily microorganisms) for removal of a pollutant
from the biosphere.
2. Basic principle of bioremediation is natural attenuation which is a natural process occur in
the environment to reduce toxicity or concentration of contamination without anthropogenic
intervention.
3. Bioremediation is less costly than conventional remediation strategies and no waste accumulated
after elimination of contaminants and contaminants are also not transported from its location.
Which of the statement(s) given above is/are correct?
(a) 1 and 2 (b) 2 and 3 (c) Only 1 (d) All of these

97. Name the biosphere reserve which is the first marine Biosphere Reserve established in India and
has extensive mangrove vegetation and has unique Dugong Dugon Species (Sea Cow), which is a
vulnerable marine animal. Illegal coral mining for cement industries is now the main threat to
the biosphere reserve.
(a) Sunderban Deltaic Swamp (b) Gulf of Kutch
(c) Gulf of Mannar (d) Chilka
134 Practice Set 5

98. Consider the following pairs.


1. Diwan-i-risalat : Dealt with religious matters
2. Diwan-i-insha : Dealt with military department
3. Diwan-i-arz : Dealt with state correspondance
In the context of Sultanate period administration, which of the pairs given above is/are
correctly matched?
(a) 1 and 2 (b) 2 and 3
(c) Only 1 (d) All of these

99. Charles Wood prepared a dispatch in 1854 on Education System in India. Consider the following
statements.
1. Government should assume responsibility of spreading education to masses.
2. Considering the conservative society, he proposed religion based education system.
3. Grants-in-aid to private educational institutions.
4. Stress over female education, teacher training and rotational education.
Which of the above were proposals of Woods Dispatch?
(a) 1, 3 and 4 (b) 1 and 4
(c) 3 and 4 (d) All of these

100. Island groups have varied vegetation and cropping pattern due to their location and soil types.
Consider the following in this regard.
1. Andaman and Nicobar islands have rainforest vegetation.
2. Whereas rice is common to the Andaman and Nicobar island as well as the Lakshadweep, coconut
plantation is found in the latter only.
Which of the statement(s) given above is/are correct?
(a) Only 1 (b) Only 2
(c) Both 1 and 2 (d) Neither 1 nor 2
Stage 1 Know the Trend 135
PRACTICE SET 5 OVERALL ANALYSIS
Stage 2 Improve Your Weaker Areas

Answers with Explanations


1. The Aggregate Expenditure (AE) is defined as the total lake in Manipur are only two Ramsar wetland sites in India
amount that firms and households plan to spend on goods and that are listed under Montreux record. World Wetland’s Day
services at each level of income. When income rises, the is celebrated each year on 2nd February to mark the date of
purchasing power also rises correspondingly. So, when AE the adoption of the convention. It was first celebrated in
rises, it also increase Aggregate Demand (AD). The increase of 1997.
AD would be more than its previous GDP. 6. Many free radicals and ions are capable of destroying
ozone. The important varieties are Chlorine ion (Cl), Hydroxyl
2. Samkhya philosophy was a rational and scientific view
radical (OH), Bromine ion (Br) and Nitric Oxide radical (NO).
initially. At the outset the Samkhya School of Philosophy was
All of these have both natural and man-made sources; at the
materialistic, but later it tended to become spiritualistic. In this
present time, most of the OH and NO in the stratosphere is of
system knowledge can be acquired through Pratyaksha,
natural origin, but human activity has dramatically increased
Anumana and Shabda.
the levels of chlorine and bromine.
3. In pursuance of an appeal by the then Prime Minister, 7. Succus Entericus is secreted by small intestine. Succus
Pt Jawaharlal Nehru in January, 1948 the Prime Minister’s Entericus (Intestinal juice) refers to the clear to pale yellow
National Relief Fund (PMNRF) was established with public watery secretions from the glands lining the small intestine
contributions to assist displaced persons from Pakistan. walls. Secretion is stimulated by the mechanical pressure of
● The resources of the PMNRF are now utilised primarily to partly digested food in the intestine. It helps in the breakdown
render immediate relief to families of those killed in natural of carbohydrates into glucose, protein into amino acids.
calamities like foods, cyclones and earthquakes etc and to Intestinal juice also contains hormones, digestive enzymes,
the victims of the major accidents and riots. mucus, substances to neutralise hydrochloric acid coming
● Assistance from PMNRF is also rendered, to partially defray
from the stomach.
the expenses for medical treatment like heart surgeries,
kidney transplantation, cancer treatment etc. 8. A currency swap is an exchange that occurs between two
● The fund consists entirely of public contributions and does
individuals or entities holding set currencies for a specified
not get any budgetary support. period of time after which, they will be exchanged back. This
● The corpus of the fund is invested with banks in fixed
can occur between two or more parties. Upon the maturity of
deposits. the exchange, each party returns to the other the original
● Disbursements are made with the approval of the Prime
amount of currency traded. Due to laws of international
Minister. accounting, currency swaps are not considered to be loans
● PMNRF has not been constituted by the Parliament. and are thus not required to be documented on a company’s
● The fund is recognised as a Trust under the Income Tax Act balance sheet. Instead of being considered a loan, currency
and the same is managed by Prime Minister or multiple swaps are treated as foreign exchange transactions with the
delegates for national causes. maturity date closed as a forward contract, which is a simple
● PMNRF is exempt under Income Tax Act. agreement ensuring the date on which the amounts will be
● Prime Minister is the Chairman of PMNRF and is assisted by repaid. The first and foremost drawback can be that the other
Officers/Staff on honorary basis. party may fail to meet the obligation during the period of
4. A magnet can exert a force on another magnet without being swap or upon maturity. The parties exposed to the risk either
in contact with it. The force exerted by a magnet is an example one or both have chances to default on principal payment as
of a non-contact force. The force exerted by a charged body on well as interest. The other drawback could be devaluation in
another charged or uncharged body is known as electrostatic the domestic currency, which can happen when huge
foreign debts are acquired. India has currency swap
force. This force comes into play even when the bodies are not
arrangement with Japan.
in contact. The electrostatic force, therefore, is another example
of a non-contact force. The force of friction always acts on all the 9. Mauryan paid remarkable attention to art and architecture
moving objects and its direction is always opposite to the during their rein. Capital of Ashoka—Pataliputra, which has
direction of motion. e.g. a ball rolling along the ground gradually been discovered at Kumharar in Patna was considered by
slows down and finally comes to rest. Megasthenes as splendid as capital of Iran. It approves of
the high technical skills of artisans during that period. Each
5. Ramsar Convention was signed in 1971 in Iran and now 168 pillar in the capital was made of a single piece of sandstone
countries are parties to the convention and India is also a party which was buff-coloured. The top of each pillar was covered
to it. It is the only global environmental treaty that deals with a by sculptures of either lion or bull. They also started
particular ecosystem. Montreux record is a register of wetland excavating caves for monks to live inside it. The most
sites on the list of wetlands of international importance where significant of such caves are Barabar caves near Gaya in
ecological changes are occurred or occurring or is threatened to Bihar. This cave architecture spread, later to Western and
be occurred. Keoladeo National Park in Rajasthan and Loktak Southern part of India following the precedence.
138 Practice Set 5

10. CR Das and Motilal Nehru founded Swaraj Party. They took 17. The first reading is the introduction stage, where the bill is
part in elections held under the Act of 1919. The main purpose introduced to the House. There is no need to refer the bill to a
of ‘Council Entry Programme’ was to enter into the Legislative committee at this stage. The second reading is the detailed
Councils, obstruct their working, expose their real nature and consideration of the bill and it is here that the bill is referred to
arouse public enthusiasm. They won 42 out of 101 seats in the committees.
Central Legislative Assembly and a majority in central The third reading is the final voting and discussion on the bill
provinces. Vallabhbhai Patel, in 1925 was elected as President and here again, there is no need to refer the bill to a
of Central Legislative Assembly. They continued till 1930, when
committee. So, Statement 1 is incorrect. Statement 2 is
they decided to leave the government due to the resolutions
correct further, in two cases new amendments can be
passed in Lahore Congress and beginning of Civil
proposed in joint sitting. Statement 3 is incorrect.
Disobedience Movement.
18. The Light Water Reactor (LWR) is a type of thermal reactor
11. Amongst the crocodiles and gharials, the salt water
that uses normal water, as opposed to heavy water, as its
crocodile is found along the Eastern coast and in the Andaman
coolant and neutron moderator and a solid compound of
and Nicobar islands. A project for breeding crocodiles which
fissile element as its fuel. There are three varieties of light
started in 1974 has been instrumental in saving the crocodile
water reactors : the Pressurised Water Reactor (PWR), the
from extinction.
Boiling Water Reactor (BWR) and (most designs of) the Super
12. Ocean Thermal Energy Conversion (OTEC) is a marine Critical Water Reactor (SCWR). It is 21st nuclear power
renewable energy technology that harnesses the solar energy reactor in India.
absorbed by the oceans to generate electric power. The Sun’s
19. First Anglo-Maratha War was fought between 1775-82. It
heat warms the surface water a lot more than the deep ocean
was concluded by Treaty of Salbai. Both agreed to return the
water, which creates the ocean’s naturally available won areas of their rivals during the war to each other.
temperature gradient or thermal energy.
The Treaty of Bassein which was signed with Baji Rao II, the
13. Statement 1 is incorrect because Speaker is the final Peshwa of Marathas in 1802, gave rise to second
interpreter of the provisions of the Constitution of India. The Anglo-Maratha War (1803-1806). In this war, Britishers took
Speaker does take part in committee business and is in fact the the decisive lead over the Maratha’s in their territory.
Chairman of the Business Advisory Committee, Rules
Committee and General Purpose Committee. Thus, Statement
20. Some of the many demands to which Government of
British India succumbed in front of Gandhiji at the time of
2 is incorrect. The Speaker does not need to certify a Bill as a
Gandhi-Irwin Pact were as follow
Constitution Amendment Bill since the Bill itself mentions that it
intends to amend the Constitution. Thus, Statement 3 is also ● Release of all political prisoners who were not convicted for
incorrect. Statement 4 is also incorrect since there is no such violence.
provision. Thus, the correct answer is (d). ● Permit peaceful picketing of liquor, opium and foreign cloth
shops.
14. Permanent settlement was implemented in Bengal by Lord ● Return of confiscated lands not yet sold to third party.
Cornwallis in 1793. It was for 30 years. ● To withdraw all pending cases against the political workers.
After not realising the expected target of profit and other In return, Gandhiji agreed over.
ill-effects of permanent settlement, Britishers opted for devising ● Immediate withdrawal of the Civil Disobedience Movement.
some other method of land revenue. So, they implemented ● To stop all boycott.
Ryotwari settlement in Bombay, Madras, Assam and Burma.
● To participate in the Second Round Table Conference.
This also disturbed the social fabric of the area.
Then, Britishers implemented Mahalwari settlement in most part 21. POPs are released to the environment have been shown
of modern Uttar Pradesh, North-Western Province and Punjab. to travel vast distances from their original source. Due to their
chemical properties, many POPs are semi-volatile and
15. Greenhouse Gases or GHG in Annex A to the protocol now insoluble. These compounds are therefore unable to
include 7 GHGs in the list. They are CO 2 , CH4 , N2O, HFCs, transport directly through the environment. The indirect routes
PFCs, SF6 and NF3 . Nitrogen trifluoride is the last one to be include attachment to particulate matter and through the
included in the list. It is mostly used in electronics industry foodweb. The chemicals’ semi-volatility allows them to travel
during various processes including plasma etching is a process long distances through the atmosphere before being
that is used to fabricates ICs, cleaning silicon, chips, deposited. Thus POPs can be found all over the world,
manufacture of LCD panel. It is highly effective in trapping including in areas where they have never been used and
atmospheric heat second only to Sulphur hexafluoride (SF6 ). remote regions such as the middle of oceans and Antarctica.
16. The Ministry of Information and Broadcasting has 22. Montreal Convention corresponds to substances that
constituted an expert committee for the purpose of reviewing deplete ozone layer. Vienna Convention on the other hand
the institutional framework of Prasar Bharti. ‘Prasar Bharti deals with protection of ozone layer. India has ratified both of
Connect’ as a 3rd arm of the Public Service Broadcast, aims to them.
manage various social media initiatives of the organisation.
Stage 2 Improve Your Weaker Areas 139

23. Statement 1 is correct, since, being out of the government Hangul deer found only in Kashmir. Among other mammals
means that private members are not able to take the are wild buffalo or gear is endemic to terai region. Lion-tailed
Assistance of Government Departments. Statement 2 is also macaque found only in Southern-Western Ghats and
correct, according to parliamentary procedures. Statement 3 is Annamalai. Rare Golden langur live along the banks of the
correct, Private Bill reflects the stand of opposition party on Manas river in Asom.
public matter, whereas Public Bill reflects the policies of the 30. The road to independence was followed by many talks,
government. Statement 4 is incorrect as just mentioned above. plans, offers and formula’s in last decade before the freedom.
Some of the Private Members’ Bills, which became laws are The one event led to the requirement of another.
Hindu Marriage Bill, Marine Insurance Bill etc. Thus, the correct The events that took place were August offer by Britishers in
answer is (b). 1940, in sought of help from Indian’s in World War II. It was
24. Sulphuric acid is produced in the upper atmosphere Venus followed by Cripps Mission in 1942, whose proposals were
by the Sun’s photochemical action on, carbon dioxide sulphur totally rejected and criticised by Indian leaders. Then C
dioxide and water vapour, present in its atmosphere sulphuric Rajagopalachari gave a formula, supported by Gandhiji, in
acid hydrate is also present on Jupiter’s Moon Europa. It is 1944 which was not acceptable to the Muslim League.
present in batteries used in torch and mobiles. Then Britishers under the Viceroy Wavell, made a plan in
25. Capital expenditure on defence is not a non-plan 1945, which is also known as Shimla Conference, for Indian
expenditure. It is a planned one for activities like R and D, self-government. It was again not accepted by Muslim
DRDO etc. But, revenue expenditure of defence can’t be League.
planned. Examples like salary of defence officers etc. The grant Then came the three membered Cabinet Mission in 1946. It
to states is a major non-plan expenditure and the programmes was followed by direct action day by Muslim League in the
related to social sector like MGNREGA, Lay, Bharat Nirman are same year after 3 months. Finally, Mountbatten Plan in 1947
plan expenditures, as they are included in Five Year Plans. gave the solution and led to Indian Independence Act, 1947.
26. The United Nations Conference on Sustainable 31. CFL contain mercury and is an health hazard. CFL emit
Development (UNCSD), also known as Rio + 20 or Earth light from a mix of phosphorus inside the bulb, each emitting
Summit, 2012 was the third International Conference on one band of colour.
Sustainable Development, hosted by Brazil in Rio de Janeiro Every extra phosphorus added to the coating mix improves
from 13th to 22nd June, 2012. colour rendering, but decreases efficiency and increases
The official discussions had two main themes cost. Compared to a theoretical 100% efficient lamp (680
(i) How to build a green economy to achieve sustainable 1m/w) CFL lamps have lighting efficiency ranges of 7-10%
development and lift people out of poverty, including versus 1.5-2.5% for incandescents.
support for developing countries that will allow them to find 32. Hot money is a term that is most commonly used in
a green path for development. financial markets to refer to the flow of funds (or capital)
(ii) How to improve international coordination for sustainable from one country to another in order to earn a short-term profit
development by building an institutional framework. on interest rate difference and/or anticipated exchange rate
The primary result of the conference was the non-binding shifts.
document, ‘The future we want’.
These speculative capital flows are called ‘hot money’
27. Unlike Egypt and Mesopotamia, no temple was found at because they can move very quickly in and out of markets,
Harappan civilisation. Great Bath of Mohenjo Daro is potentially leading to market instability.
considered as only exception of religious site in Harappan
33. The Aravali is the oldest mountain range in India and, it
civilisation. Harappan had a well-developed script and they
faced long duration of erosional activity. The Deccan lava
were aware of art of writing.
plateau is product of basaltic lava flow caused by fissure
They also used pictures in their inscriptions. But, modern eruption during the cretaceous and subsequent period.
historians are yet to decipher their script to understand what Rocks of the Indian Peninsular Plateau re-appear in
that meant in their writings. Harappan made beautiful potteries Meghalaya and form a rectangular block known as the
which they decorated with designs of trees and circles and also Shillong-Plateau or the Meghalaya Plateau. This plateau has
with images of men over it. been separated from the main block of Indian Plateau by a
28. Rigveda has no details of the king’s administration of wide stretch of alluvial lowland called the Garo-Rajmahal
justice. Atharvaveda mentions garments (dursa) and goat-skin Gap.
(ajina) as articles of trade. 34. The Cabinet Secretariat is responsible for the
29. India is a mega diverse country with many endemic species administration of the Government of India (Transaction of
restricted to specific regions. Barasingha or Swamp deer are Business) Rules, 1961 and the Government of India (Allocation
endemic to boggy areas of Terai at the foothill of Himalaya. of Business) Rules 1961, facilitating smooth transaction of
Indian wild ass is endemic to Little Rann of Kutch. The business in Ministries/ Departments of the Government by
one-horned rhinoceros, once found throughout the Gangetic ensuring adherence to these rules. The Secretariat assists in
plains now restricted only to Asom of North-East India. decision-making in government by ensuring Inter-ministerial
140 Practice Set 5

coordination, ironing out differences amongst Ministries/ ● The Chairperson and Chief Executive Officer of Food Safety
Departments and evolving consensus through the and Standard Authority of India (FSSAI) are appointed by
instrumentality of the Standing/Adhoc Committees of Government of India.
Secretaries. Through this mechanism new policy initiatives are ● The Chairperson is in the rank of Secretary to Government of
also promoted. India.
35. Under the Kyoto Protocol, countries must meet their 38. The retired judges of the SC are prohibited from pleading or
targets primarily through national measures, but there is an acting in any court of law or before any authority within the
additional means to meet their targets by way of three territory of India. This ensures that they do not favour any one in
market- based mechanisms. the hope of future favour. So, Statement 1 is correct.
These are International Emissions Trading, Clean Independence of SC is maintained through mode of
Development Mechanism (CDM) and Joint Implementation appointment, security of tenure, fixed service conditions etc. So
(JI). International Emissions Trading now created ‘Carbon the Statement 3 is correct.
Market’ as a new commodity for sell that is helping in SC is guided by Doctrine of Progressive Interpretation while
meeting emission targets. interpreting the constitution, this doctrine says that a
Joint Implementation makes provision for establishing green Constitution is organic and must be read in a broad and
projects between industrialised countries whereas CDM sets progressive manner so as to adopt it to the changing times, so
provision for establishing green projects in developing the Statement 4 is also correct.
countries by industrialised countries. Judicial review is the power of the Supreme Court to examine
The CDM is the main source of income for the UNFCCC the constitutionality of legislative enactments and executive
Adaptation Fund, which was establish to finance adaptation orders of both the Central and State Governments. On
projects and programmes in developing countries parties to examination, if they are found to be violative of the Constitution
Kyoto Protocol that are particularly vulnerable to the adverse they can be declared as illegal, unconstitutional and void by the
effects of climate change. Supreme Court. So, the Statement 2 is correct.
36. The term ‘economic drain’ refers to a portion of national 39. Statement 1 is incorrect High Court to issue writs for the
product of India which was not available for consumption of enforcement of the Fundamental Rights and ordinar right,
its peoples, but was being drained away to Britain for political whereas the Supreme Court can issue writs for enforcement of
reasons and India was not getting adequate economic or Fundamental Rights only.
material returns for it. The drain theory was put forward by
Statement 2 is also incorrect because High Court as well as
Dadabhai Naoroji in his book ‘Poverty and UN British Rule in
Supreme Court both can issue writs of jurisdiction.
India’. The major components of this drain were salaries and
pensions of civil and military officials, interests on loans taken Statement 3 is also incorrect Supreme Court has original
by the Indian Government from abroad, profits on foreign jurisdiction in the sense that an aggrieved citizen can directly go
investment in India, stores purchased in Britain for civil and to the Supreme Court, not necessarily by way of appeal.
military departments, payments to be made for shipping, 40. According to Article 124 (6) and (7) Constitution prohibits a
banking and insurance services which stunted the growth of person who has held office as Judge of Supreme Court from
Indian enterprise in these services. The drain of wealth practising law before any court in the territory of India. The
checked and retarded capital formation in India while the Constitution by fixing the salaries of the judges and providing
same portion of wealth accelerated the growth of British that thought the allowances, leave and pensions may be
economy. The surplus from British economy re-entered India determined by law made by Parliament, these shall not be
as finance capital, further draining India of its wealth. This varied to the disadvantage of a judge during his team of office.
had immense effect on income and employment potential But, it will be competent for President to override this guarantee
within India. So, the options (a), (b), (c) are correct in this under a proclamation of emergency.
regard and (d) is incorrect because of precise/necessary
data it is difficult to quantify the Economic Drain with any 41. The flow of El-Nino current (waver current) affect the flow of
exactness. the Humboldt or Peruvian current and creates low pressure
zone in the Eastern Pacific region. The cold water chills any wind
37. The Food Safety and Standard Authority of India (FSSAI)
that blows onshore so that the Chilean and Peruvian coasts are
has been established under Food Safety and Standard Act,
practically rainless. The region is rich in microscopic marine
2006 which consolidates various acts and orders that have
plants and animals that attract huge shoals of fish. Since,
hitherto handled food related issues in various Ministries and
Humboldt current is cold current, it causes drier climatic
Departments.
condition in the nearby landmass.
● It was created for laying down science based standards for
articles of food and to regulate their manufacture, storage, 42. The oceans are mostly under laid by dark coloured basalt
distribution, sale and import to ensure availability of safe followed down by a thick greenish and tremendously hot layer of
and wholesome food for human consumption. magma. Acidic waves are highly viscous due to pre-dominance
● Ministry of Health and Family Welfare, Government of India of silica content with high melting point and are light in colour
is the Administrative Ministry for the implementation of (yellowish).
FSSAI.
Stage 2 Improve Your Weaker Areas 141

Inter montane plateaus are found in the regions of young fold 51. Abrasion occurs when some force causes two rocks
mountain as a characteristic feature of these mountain ranges. The to come together causing mechanical wearing or
zone of low-velocity of seismic waves is found between the grinding of their surfaces. While solution refers to the
lithosphere and the asthenosphere, so Statement 4 is incorrect. dissolution of soluble particles and minerals from the
43. Wool is a poor conductor of heat. The air trapped in between the rocks with the help of water in motion. Carbonation is the
reaction of carbonate or bicarbonate ions with minerals.
wool fibers prevents the flow of heat from our body to the cold
The process of carbonation is also known as solution
surroundings. When wool fibers are packed together, they form
wherein atmospheric CO 2 after mixing with water forms
millions of tiny air pockets which trap air, and in turn serves to keep
carbonic acid (H2CO 3 ) which after reacting with
warmth in during winter.
carbonate rocks, say limestones forms calcium
44. Convex mirror is also called as fish eye mirror and these are bicarbonate Ca(HCO 3 )2 which is easily dissolved in
used in vehicle mirrors. The other statements are correct. Concave water. The chemical process of oxidation simply means a
mirrors are used by dentists, doctors and in torches. The image on a reaction of atmospheric oxygen to form oxides.
convex mirror is always virtual, diminished (smaller) and upright. 52. When there is a lack of quorum of the permanent
45. Biopesticides (also known as biological pesticides) are judges to hold or continue any session of the Supreme
pesticides derived from such natural materials as animals, plants, Court, the Chief Justice of India can appoint a judge of a
bacteria, fungi and certain minerals. It is a form of pesticide based High Court as an ad hoc judge of the Supreme Court for
on microorganisms or natural products. So, all of the above options a temporary period, after the Constitution with Chief
given are correct. Justice of High Court concerned. The procedure for the
impeachment of judge of a High Court is the same as
46. Nitrate oxidise the iron atoms in haemoglobin rendering it unable that for a judge of the Supreme Court. At anytime, the
to carry oxygen, this can lead to generalised lack of oxygen in organ Chiefs Justice of India can request a retired Judge of the
tissue and a dangerous condition called Methemoglobinemia (which Supreme Court or High Court (who is duly qualified for
is called blue baby syndrome in instants). WHO prescribes nitrate appointment as a judge of the Supreme Court) to act as
content not more than 45 ppm. So, both the statements are correct a judge of the Supreme Court for a temporary period,
and option (c) is the answer. only with the previous consent of the President. So, the
47. The buffer stocks are required to 1. Feed TPDS and other Statement 1, 2 and 4 are correct. The retired permanent
welfare schemes. 2. Ensure food security during the periods when Judges of a High Court are prohibited from pleading or
production is short of normal demand during bad agricultural years. acting in any court or before any authority in India except
3. Stabilise prices during period of production short fall through the Supreme Court and the other High Courts. This
open market sales. The total annual stock of foodgrains in the ensure that they do not favour anyone in the hope of
cultural pool is distributed over different quarters of the year future favour. So, the Statement 3 is incorrect
depending upon off take and procurement patterns. 53. The President appoints a member of the Lok Sabha
as the Speaker Pro–Tem. Usually, senior most member
48. Non-Banking Finance Companies (NBFCs) are financial
is selected for this and the President himself administers
institutions that provide banking services without meeting the legal
oath. Office of the Speaker Pro–Tem is temporary and
definition of a bank, that is one that does not hold a banking licence.
ceases to exist once Speaker is elected. However,
These institutions are not allowed to take deposits for the public.
Speaker Pro–Tem has all the powers of the Speaker,
NBFCs offer most sorts of banking services, such as loans and
presides over the first meeting of Lok Sabha and main
credit facilities, private education funding, retirement planning,
duty is to administer oath to new members.
trading in money markets, underwriting stocks and shares.
54. Ajanta caves are situated near Aurangabad,
49. Bioleaching is the extraction of specific metals from their ores
Maharashtra. There are 30 caves chiseled out of the rock
through the use of bacteria. This is much cleaner than the traditional
in semicircular fashion. Their execution covers a period
heap leaching using cyanide solution. Bioleaching comes under
of about 8th centuries, the earliest being 2nd century BC.
biohydrometallurgy and several methods are used to recover
copper, zinc, lead, arsenic, antimony, nickel, molybdenum, gold, The subject matter of these paintings are mostly
silver and cobalt. Buddhist Jatakas, collection of stories, recording the
previous births of the Lord Buddha. The composition of
50. The loudness of sound depends on its amplitude. When the these paintings are large in extent, but majority of figures
amplitude of vibration is large, the sound produced is loud. When are smaller than life size.
the amplitude is small, the sound produced is feeble. The frequency
These paintings also exhibit the decorative patterns on
determines the shrillness or pitch of a sound. If the frequency of
the ceilings and pillars of the caves.
vibration is higher we say that the sound is shrill and has a higher
pitch. If the frequency of vibration is lower, we say that the sound 55. The jewels to obtain liberation in Jainism are
has a lower pitch. e.g. a drum vibrates with a low frequency. considered as three jewels or Triratna. They are right
Therefore, it produces a low-pitched sound. On the other hand, a knowledge, right action and right faith. By this, as
whistle has a high frequency and therefore, produces a sound of Jainism believes everyone can attain freedom from
higher pitch. worldly bonds.
142 Practice Set 5

The Triratna of Jains were a little different from eight fold path of 60. Landslides are defined as the Mass Movement of rock,
Buddhists. The eight fold path of Buddhists are known as debris and Earth down a slope where by following, sliding
Ashtangika Marga. It comprised of right observation, right and flowing. The action of gravity is the primary driving force
determination, right speech, right action, right livelihood, right that affects the stability of slope. Other causes includes
effort, right awareness and right concern. ● Groundwater
56. Biodiversity to be conserved for the sustenance of ● Loss or absence of vertical vegetative structure, soil
ecosystem. In-situ or on the site conservation of biodiversity nutrients and soil structure.
alongwith its natural habitat, it is called protected areas. ● Geomorphic phenomenon such as erosion, earthquake
Protected areas are national parks, wildlife sanctuaries, and volcanic eruptions.
biosphere reserves, eco-sensitive zones etc. Ex-situ are the sites ● Weakening of a slope through saturation by snow melt,
of conservation of biodiversity or any species outside its natural glaciers melting or heavy rains.
habitat like zoo, botanical garden, breeding centre, gene banks ● Heavy rainfall, deforestation, cultivation and construction.
etc but conservation of wetland is an act of In-situ conservation. So, all of the Statements given above are correct.
IUCN suggests to maintain 10% of all ecosystem as protected
areas. Tiger was the first species to be protected through a 61. Nayanars and Alvars preached and composed in their
National Programme 1973 followed by crocodile, rhinoceros, local languages. This single factor limited their success in
elephants etc. North while made them succeed in South.

57. The olive ridley sea turtle is also known as the pacific ridley Nayanars were Shaivites and Alvars were Vaishnavite. They
found in warm and tropical waters of Primarily Pacific and Indian disregarded the austerities preached by the Jains and the
Oceans. They are famous for their behaviour of synchronised Buddhists and preached personal devotion to God as a
nesting in mass numbers, termed as Arribadas. The largest means of salvation.
nesting site is in Odisha, Kendrapara district. The sea beach on Kabir emphasised over the unity of God whom he called by
the Mouth of River Rushikulya and Devi is a best nesting site, this various names like Rama, Hari, Govinda etc. He denounced
site is also called ‘Gahirmatha’ now protected as Gahirmatha all kind of formal worship and symbols of it like idol worship,
wildlife sanctuary contiguous to Bhitarkanika wildlife sanctuary. It holy bathing.
takes 50-60 days for the egg to be matured so it easily fall pray to 62. The National Food for Work Programme was launched
jackal, dog, crabs etc and only 1 out of 1000 become mature to on 14th November, 2004 in 150 most backward districts of
bring out offspring. the country, identified by the Planning Commission in
58. Rhizobium cannot make its own food, so it lives symbiotically consultation with the Ministry of Rural Development and the
with legumes. The first two statements are correct. Rhizobia are State Governments. The objective of the programme was to
soil bacteria that fixes nitrogen (diazotrophs) after becoming provide additional resources apart from the resources
established inside root nodules of legumes (Fabaceae). Rhizobia available under the Sampoorna Grameen Rozgar Yojana to
require, a plant host they cannot independently fix nitrogen. 150 most backward districts of the country, so that
Rhizobium can fix nitrogen from the atmospheric, elemental N2 generation of supplementary wage employment and
into a plant-usable form, ammonium, using the enzyme providing food security through creation of need based
nitrogenase. economic social and community assets in these districts
59. Ultrasound, also called sonography, is another type of 3-D are further intensified.
computerised imaging. Using brief pulses of ultra high frequency 63. Working to setting up commercial cooperation in
acoustic waves (lasting 0.01 seconds), it can produce a sonar renewable energy with developed and developing
map of the imaged object. The technique is similar to the countries. The ministry of new and renewable energy has
echolocation used by bats, whales and dolphins. By measuring firmed up international cooperation agreements with
the echo waves, it is possible to determine the size, shape, Germany and United States to establish ‘Green Energy
location and consistency (whether it is solid, fluid-filled or both) of Corridors’ in the country. And also working in renewable
an object. Ultrasound is a very safe, non-invasive imaging energy sector with a view to establish commercial
technique. cooperation for supplies of indigenously manufactured
Unlike X-rays, sonography does not use ionising radiation to renewable energy products, system and devices.
produce an image. It gives a clear picture of soft tissues, which 64. Statement 1 is incorrect. While the Governor does
do not show up in X-rays. Ultrasound causes no health problems appoint the members of the State Public Service
(for a mother or unborn fetus) and may be repeated as often as Commission, their dismissal is done by the President.
necessary. Statement 2 is correct as the Governor acts as the
Current evidence indicates that diagnostic ultrasound is safe for Chancellor of Universities in the state and appoints the
the unborn child unlike radiographs, which employ ionising Vice-Chancellors of these Universities. He also causes to be
radiation. However, no randomised controlled trials have been laid before the State Legislature, the reports of CAG related
undertaken to test the safety of the technology and thus to the state administration, so Statement 3 is also correct.
ultrasound procedures are generally not done repeatedly unless Statement 4 is also correct.
medically indicated.
Stage 2 Improve Your Weaker Areas 143

65. The Jet stream consists of ribbons of very strong winds Humus content of the soil is removed fast by bacteria that
which move weather systems around the globe. Jet streams are thrives well in high temperature. These solids are poor in
found 9-16 km above the surface of the Earth, just below the organic mather. Nitrogen, phosphate and calcium are rich in
tropopause and can reach speeds of 200 mph. All the given iron oxide and potash.
statements are correct. Jet streams are caused by a 72. The order of succession in a hydrosere is
combination of the Earth’s rotation on its axis and atmospheric
Phytoplankton>Rooted aquatic plants>Free floating
heating by solar radiation. The strongest of all the Jet streams
stage>Reed swamp stage> Sedge marsh or meadow
are the Polar jets. The major Jet streams of the Earth flow West
stage> Woodland stage>Climax stage.
to East.
66. Tropical moist deciduous forests are found in the areas of
73. Oxides of Sulphur and Nitrogen when stay longer in
atmosphere, are oxidised into acids. Sulphuric acid is the
annual rainfall of 100-200 cm and mean annual temperature of
predominant acid in the acid rain followed by nitric acid. The
about 27°C. The trees of these forests shed their leaves for
problem of acid rain has dramatically increased due to
about 6-8 weeks, during early summer to conserve the moisture
industrialisation. Scandinavian countries, Britain, Germany
during the dry season.These forests are dominated by sal and
and Canada etc are affected most by acid rain. In these
teak, alongwith mango, bamboo and rosewood.
region acid rain occur as acidic snowfall and turn into a
67. is the right option because Bicameralism is a federal pollution time bomb in the course of time. It also affects
characteristic and single integrated Judiciary is a also a part of wetlands and kills fish species and makes the lakes etc turn
federal structure. into fish graveyards.
68. The evolution of Indian music saw the emergence of two 74. Bio-prospecting is the process of discovery and
different sub systems as Hindustani and Carnatic music. In commercialisation of new products based on biological
Haripala’s ‘Sangeeta Sudhakara’ written in 14th century AD we resources. It also includes ‘Biopiracy’. The exploitative
find the terms ‘Carnatic’ and ‘Hindustani’. It is said that the South appropriation of indigenous forms of knowledge by
Indian Music, as known today, flourished in Deogiri the capital commercial actors. So, option (d) is the correct answer.
city of the Yadavas in middle ages. After Muslim invasion of 75. Notified Area Committee as the name suggests is set up
South India, the entire cultural life of city took shelter in Carnatic through a notification of the State Government and does not
Empire of Vijayanagara under the reign of Krishnadevaraya. require a legislation. Industrial townships are established by
69. SEBI has three functions rolled into one body. large enterprises to provide civic amenities to workers living
Quasi-legislative, quasi-judicial and quasi-executive. It drafts in the colonies in the townships. The townships are headed
regulations in its legislative capacity, it conducts investigation by an administrator, who is assisted by other staff all of
and enforcement action in its executive function and it passes whom are appointed members and not elected. Cantonment
rulings and orders in its judicial capacity. Though, this makes it Boards are established under a Central Legislation and work
very powerful, there is an appeal process to create under the control of Ministry of Defence. These boards have
accountability. There is a securities appellate tribunal, which is a both nominated and appointed members and perform
three-member tribunal and is headed by a former Chief Justice municipal functions for the civilian population in the
of a High Court. A second appeal lies directly to the Supreme cantonment area. Port Trusts are created by an Act of
Court. Parliament and consist of nominated and elected members
and are established in port areas. Thus, we can see that all
70. The contribution is by the Government of India and not the statements are correct except 1 and the right answer is (b).
country where the worker is working. The main attractions of
MGPSY is to contribution of ` 1000 per annum in line with
76. Indian Government has launched new AQI in 2015 as a
part of Swachh Bharat Abhiyan, for measuring of air quality
Swavalamban platform for all MGPSY subscriber, which is same
and pollution on real time basis. It acts as one number-one
between ` 100 to 12000 per year in MPS life.
colour-one description for the common man to judge the air
A special government contribution of ` 900 by MOLA forwards quality within his vicinity. There are six AQI categories and
Return and Resettlement (R & R) of the overseas Indian workers, consider eight pollutants namely PM10, PM 2.5, NO2 , SO2 ,
who same ` 4000 or more per annum. CO, O3 , NH 3 and PB for which short-term (upto 24-hourly
So the Statement 3 is in incorrect An overseas Indian worker averaging period). It is based on the recommendations
enrolled in this scheme would be provided with a free cost life made by the IIT, Kanpur and for the first it uses colour coded
insurance cover and applicable as long as they are working in table to show air quality and its effects on health.
Emigration Clearance Required (ECR) countries. So, the
77. Black soils have high fertility and are capable of retaining
Statement 1 and 2 are correct.
high moisture. The presence of Titaniferous Magnetite is the
71. Laterite has been derived from the Latin word ‘Later’ which reason behind the black colour of the Black soil. Black soil is
means brick. The Laterite soils develop in areas with high also called the self ploughed soil. Black soils are highly
temperature and high rainfall. Laterite soils are the result of retentive of moisture, extremely compact and tenacious
intense leaching due to strong tropical rains. With rain, lime and when wet, considerably contracted developing deep wide
silica are leached away and soils rich in the iron oxide and cracks on drying and self-ploughing. Leaching is not a
aluminium compound are left. phenomenon that Black soil suffer from.
144 Practice Set 5

78. Zonal Councils have been set up in India under the states 84. Rabi and Kharif are the two agricultural seasons in the Indian
reorganisation act and not under Article 263. There are a total sub-continent. The Kharif crops will have the autumn harvest;
of five Zonal Councils in India. Thus, statement 1 is incorrect. but better known as the monsoon crops in Indian sub-continent.
The head of the Inter State Council, which was set up in 1990 These crops are totally dependent on the quantity of rain water
under Article 262 is the Prime Minister of India and not the as well as its timing. The Rabi crop is the spring harvest also
President, so Statement 2 is also incorrect. Statement 3 is known as the ‘winter crop’ in Indian sub-continent. The term
also incorrect. Though, the Inter-State Councils can be set up Rabi means ‘spring’ in Arabic and it coincides with mid April to
to decide on any legal dispute between states, its mid June. Of the given crops Millets are Kharif crops and the
pronouncements are non-binding in nature. Statement 4 is remaining crops are Rabi crops.
correct and a reconstituted National Integration Council was 85. The nature of coastal plain is not only emergent, but
set up in 2005 with the purposes described in the above submergent also, Backwaters of the Malabar plain are known as
statement. Thus, the correct answer is (c). Kayals. Most of the Creeks and Lagoons are located on the
79. Statement 1 is incorrect. Under a financial emergency West coastal plain. A Lagoon is a shallow body of water
salaries of any person in public office can be reduced even separated from a larger body of water by barrier islands or reefs.
when the constitutional provision for that particular post A reef is a rock, sandbar or other feature lying beneath the
provides for not reducing the salaries of that post after surface of the water.
appointment of a person to that post. 86. Indo-Islamic architecture begun or around the close of 12th
Statement 2 is also incorrect because the provision under century AD. The most important factors common to both forms
financial emergency talks of ability to reserve the Money Bills of architecture, especially in respect of mosques and temples,
were that to both styles, ornamental decoration was very vital
and other Financial Bills passed by a State Legislature for the
and that the open court in many cases was surrounded by
consideration of the President after they are passed by the
colonnades. But there were contrasting factors also; the prayer
State Legislature. Proclamation of emergency does not
chamber of the mosque was spacious, where as that of the
require repeated Parliament approval and can be revoked by
temple was dark and closed. A Garbha-Griha, of Hindu temple
the President anytime and thus, statement 3 is also incorrect
and small halls for worshippers were considered adequate, but
and the right answer is (d).
in Islamic form of worship spacious courtyard with a large prayer
80. PWR reactors require much higher enrichment. An hall pointing towards Mecca was required.
identical PWR of the same capacity is already operating in 87. Sufi order were broadly divided into Ba-shara and Be-shara.
Kalpakam. They act as forerunners to the DAE building Ba-shara followed the Islamic law which Be-shara was not
commercial PWRs in India. bound to it. Chishti and Suharawardi Silsilahs were Sufi orders
81. Statement 1 is true because the 7th Schedule also of ‘Ba-shara’ division. Khwaja Muinuddin Chishti and his
provides for the powers of exclusive taxation which one disciples Bhakhtiyar Kaki and Farid-ud-Din Ganj-i-Shakar were
available to the union and the states within the three lists of Chishti order. Farid-ud-Din Ganj-i-Shakar’s outlook was so
(Union, State and Concurrent). Statement 2 is incorrect. The humane that some of his verses were included in Adi-Granth of
distribution of the proceeds of taxation between the union the Sikhs. Suharawardi order entered India at the same time as
and the states is done on the basis of a formula not from Chishtis and confined their activities mainly into Punjab and
parent law, but from the formula prescribed by the finance Multan.
commission, which is constituted every 5 years according to 88. The chemical energy stored is plants and animals in the form
the Constitution. Statement 3 is also true because according of organic molecules. The transfer of energy from one trophic
to Article 285 (1) and 289 (1), union and state properties are level to higher trophic level involves consumption of these
exempted from mutual taxation.The right answer is thus, (b). organic molecules. Cyclic flow of energy means energy is again
82. ‘2 + 2 Agreement’, which is a indication of the growing available for reuse. In an ecosystem, the flow of energy is in
directional and non-cyclic. So, correct option is (c).
proximity between India and Japan. Cooperation in
combating piracy intelligence training, counter terrorism are 89. Great Himalayan National Park in Himachal Pradesh is
main agenda for ‘2 + 2 Agreement’. DMIC is a state famous conservation site for critically endangered snow
sponsored industrial development project of the Government leopard. Kaziranga National Park in Asom is famous for
of India. It would be the biggest infrastructure project India one-horned rhinos, Bandipur National Park in Karnataka is
has ever attempted in its history. The project will see major protection site for elephants where as Dudhwa National Park in
Uttar Pradesh is protecting swamp deer.
expansion of infrastructure and industry including smart
cities, industrial dusters along with rail, road port, air 90. Hardness of water is indicated by lack of foam formation
connectivity. when soap is added to water. Temporary hardness is due to
presence of bicarbonate ions of calcium and magnesium, it is
83. During the cooling of the Earth, gases and water vapour
removed by boiling the water or by adding lime (calcium
were released from the interior solid Earth. This started the
hydroxide). Permanent hardness is due to presence of sulphate
evolution of the present atmosphere. The process through
and chloride of calcium and magnesium. It cannot be removed
which the gases were outpoured from the interior is called by boiling as it requires precipitation or ion exchange column
‘Degassing’. method.
Stage 2 Improve Your Weaker Areas 145

91. Futures are financial contract obligating the buyer to 96. Bioremediation is the use of microorganisms for removal of
purchase an asset, such as a physical commodity or a pollutants from biosphere, a species of pseudomonas is found
financial instrument, at a predetermined future date and price. by Japaneses that can grow in a contaminated condition of
Future contracts detail the quantity and quality of the more than 50% tolceine and degrade toxics. A hybrid variety of
underlying asset. The Reserve Bank of India through Open pseudomonas is developed to degrade oil spill marine
Market Operation buys and sells government securities in pollutions. Basic principle of bioremediation is natural
open market to control the supply of money in banking attenuation that the organisms florish naturally without human
system. A capital market is a market, for securities (debt or intervention. It is less costly than traditional agents and not
equity), where business enterprises (companies) and produce any by product and also not transport or relocate the
governments can raise long-term funds. It is defined as a contaminants. Bioremediation is now also used to produce
market, in which money is provided for periods longer than a biogas from toxic elements and also used for degradation of
year as the raising of short-term funds takes place on other synthetic elements like nylon and polythene.
markets (e.g. the money market). So, all of the above
statements are correct. 97. Gulf of Mannar is the first marine Biosphere Reserve
established in India situated in Tamil Nadu. It is also a marine
92. According to the Preamble of the APMC Act, it provide
national park. About 46 species of mangrove plants are
development for efficient marketing system, promotion of
endemic to this region and among the species Sea Cow or
agri-processing and agricultural exports and to lay down
Dugong Dugon is also endemic. Now illegal coral mining for
procedures and systems for putting-in place an effective
cement industries and indiscriminate collection of sea grass is
infrastructure for the marketing of agricultural produce. Apart
the main threat to the reserve. Now about 65% of the existing
from the above listed, it also needs to publish the data on
arrivals and rate of agricultural produce brought into the coral reefs in the area is dead due to human interference.
market area for sale. 98. Diwan-i-risalat dealt in the religious matters, pious
93. Chir, pine and deodar are important trees of Coniferous foundations and stipends to deserving scholars and men of
forests. Cactus, khair, babool, keekar are important thorny pity. The President of this Department of Chief ‘Sadr’, who was
bushes and are found in states of Rajasthan, Punjab, generally an eminent qazi. Diwan-i-insha dealt with state
Haryana, Eastern slopes of Western Ghats and Gujarat. correspondence. It collected the information of the
Mangroves Sundari is a well-known species of trees in correspondents either formal or confidential. Diwan-i-arz was
mangrove forests after which Sunderbans have been named. second most important department of the state after Wazir. It
Important trees of Tropical Deciduous forests are sal, teak, dealt with military activities. Head of this department was
peepal, neem and shisham. Ariz-i-mamalik.
94. Wheat requires moderate temperature and rainfall during 99. Charles Wood was the President of the Board of the East
growing season and bright sunshine at the time of harvest. It India Company. He sent a despatch to Dalhousie in 1854 for
thrives best in well-drained loamy soil. Wheat is grown spreading education in India. He recommended
extensively in USA, Canada, Argentina, Russia, Ukraine, ● To set an educational department in every province.
Australia and India. In India it is grown in winter. ● Establishment of universities in Bombay, Kolkata and
95. Sangam literature were compiled in 300-600 AD. The Madras on the model of London University.
narrative texts of then were called Melkannakku while the ● Opening of government school in every district.
didactic texts were known as Kilkanakku. Melkannakku are ● Affiliated private schools should be given grants in aid.
also known as 18 major works while Kilkanakku are called as ● Education should be secular in nature.
18 minor works. Tolkkappiyam is other than Sangam text of ● Stress on female education, vocational education and
the period which deals with grammar and poetics while teacher’s training.
Tirukkural is important Tamil text which deals with philosophy
and wise maxims. 100. Andaman and Nicobar islands lies close to equator and
experiences only a brief dry spell thereby providing ideal
Silappadikaram and Manimekalai are twin Tamil epics which
condition for growth of rainforest. Coral islands lack soil cover
were composed in 6th century. These two are considered as
brightest gem of early Tamil literature. for any kind of foodgrain crop. Coconut plantation is the most
suitable crop.
Rough Work
CSAT Paper 2

Practice Set 1

1. There are 80 questions in this paper.


2. All questions carry equal marks.
Penalty for wrong answer
3. There are four alternative answers in every question. When you select a wrong answer, then 1/3rd mark of that question is
deducted in your total marks.
4. If any candidate gives more than one answer and one of them is true but it is treated as a wrong answer and the candidate
is penalised for that and1/3rd marks will be deducted.

Directions (Q. Nos. 1-8) Read the following six passages and answer the questions that follow
each passage. Your answers to these questions should be based on the passages only.

B Passage 1
Towards the beginning of the 20th century, Charles Darwin formulated ‘Darwinism’ which then
followed by Lamarckism, Mendelism. Before that Galileo, Copernicus had dedicated much to the field
KNOW THE TREND
of science. After them, in modern era Einstein, Archimedes, Newton and many brand new scientists
presented us facts, technology, means for convenience and pooh-poohed at our dogmas, preconceived
notions. As science advanced, superstitions faded. Men became modernised, cultured and
sophisticated.

1. The author’s view point can be best summed up in which of the following statements?
1. Superstitions are prevalent because people have no scientific outlook that make them
ignorant about reality and they began to believe whatever comes their way.
2. With the advancement in science and technology men will be modern as it will help in the
reduction of dogmas which ultimately lead to the banishment of old beliefs and the irrelevant
notions.
Stage 1

3. Every scientist has worked with a view to fade away the preconceived notions, superstitions
and dogmas with almost a similar approach and inclination.
(a) Only 1 (b) Only 2 (c) Only 3 (d) Both 1 and 2
2 Practice Set 1

B Passage 2
The basic aim of Science is to look for reality and to find those factors, formally uncommon. A scientist
cannot accept the principles, usually because; they were accepted by others formerly. Great discoveries
are always coming from reasoning.
A lot of determination and observation are required. Civilisation, as it is today, also came from
numerous discoveries that were made in the field of Science. Man is a logical being. It is possible for
everyone to save himself from the odds. Only through this way, he can win and control everyone on the
globe. Only wisdom can help a man to conquer the World. Science without conscience is death of
the soul.

2. Which among the following options is the most logical corollary to the above passage?
(a) Science accepts facts and reasoning on the basis of its veracity and not on the grounds that it
has been a perceived notion earlier that has the elements of acceptance of others.
(b) A lot of dedication and observation is required for the discovery of new ideas and concepts of
science; civilisation also follows the same pathways as that of the scientific discovery.
(c) Science without conscience is useless and dangerous, therefore, new discoveries can be said the
outcome of reasoning and logic that forms the basis of ideas and innovations to it as well as for
a civilisation.
(d) Science and civilisation, both have a common and mutual path for their discoveries which is
based purely on the reasoning and logic of our conscience.

B Passage 3
The mind has some difficulty in accepting the idea that there is something that is not explainable. Mind
has a very mad urge for everything to be explained. Anything that remains a puzzle, a paradox, goes on
troubling your mind. The whole of history of philosophy, religion, science, mathematics, has the same
root, the same mind – the same itch.
You may scratch yourself one way, somebody else may do it differently, but the itch has to be
understood. Mind can work freely when there is no mystery about existence. Ideas doesn’t come where
life is mysterious and gap thus formed cann’t be filled with reality. But it is not possible. Whatever you
do, life is a mystery and is going to remain a mystery.

3. Which contrast does the writer want to establish about the working and functioning of the
human mind?
(a) Human mind does have a mad urge for explanation of the mysteries, which, when answered,
lead to a satisfaction but actually there is no final explanation whatsoever for these mysteries
and it will remain as such.
(b) Human mind believes that nothing is a mystery, but, in actual, life is a mystery despite the
conviction that it is understood and holds good to every aspect of the “itch” of the mind.
(c) The “itch” of the human mind is nothing but the belief that existence is not a mystery and it
could be understood well, but, the stark reality is that it is a mystery and will remain a mystery
forever.
(d) Either (b) or (c).
Stage 1 Know the Trend 3

B Passage 4
As the world sees success, it is the achievement of a social status, completing a goal, reaching an
objective or the achievement of an action in a specified set of time. The way God defines success is so
much different than the way the world does. Success to those in the world means achieving something
that is useful in this life on earth. Success as God sees, it is something that is achieved, that is of infinite,
eternal value in this life and in the life to come. That is a huge difference indeed. Defining success is
difficult and is in constant flux for most people. As you achieve more your goal posts move further away.
As you move towards your next goal, you aim for more and try harder and grow and develop. For this
reason, it’s tough to define success and ultimately impossible to achieve it and reach an end point.

4. Success has been described in many ways in the passage above. Which of the following
options best sums up?
(a) Success is the achievement of an individual’s own goal, social status and the achievement of an
objective that defines the completion of one’s wish.
(b) Success is what we think to accomplish in the time to come and to complete that with flying
colors in the individual’s interest or for the interest of the community.
(c) Success is what is accomplished by an individual with his own effort and it serves the
community purpose without any self involved or without any hidden interest.
(d) Success is what one achieves with diligence and it helps one develop one’s reputation when that
success is used by the other fellow members in the interest of the community.

B Passage 5
Adolescence is the best stage in life. You are finally growing up and reaching an age where you have lots
of choices, freedom and chances. The time your parents nagging on you, holding your hands while
crossing the street is now over. You take care yourself and have lot more chances to get out, make new
friends, joining sports teams, student councils and all other after school activities. Getting involved in
such activities increase your popularity, helps to make new friends. You can go to parties and have fun at.
There are lot other things like responsibilities, handling of peer pressure which increase with
adolescence. This might turn people off from thinking this as best stage in life. The choices makes great
differences in life. A wrong choice can get you into wrong crowd and end up in trouble with law. You
don’t want to get do anything stupid for which you have to regret later on.

5. Adolescence age is a stage of opportunities as well as challenge. According to the passage


which of the following is not correct about adolescence?
(a) While making choice one may sometimes end up doing stupid things.
(b) At this stage one have to handle responsibilities and peer pressure.
(c) For not being end up wrong one makes lots of choices.
(d) Going to parties and having fun helps individual to increase popularity.

B Passage 6
Democratic institutions are devices for reconciling social order with individual freedom and initiative
and for making the immediate power of a country’s rulers subject to the ultimate power of the ruled.
The fact that, in Western Europe and America, these devices have worked, all things considered, not too
badly is proof enough that the eighteenth century optimists were not entirely wrong. Given a fair
chance, I repeat; for the fair chance is an indispensable prerequisite.
4 Practice Set 1

No people that pass abruptly from a state of subservience under the rule of a despot to the completely
unfamiliar state of political independence can be said to have a fair chance of being able to govern itself
democratically. Liberalism flourishes in an atmosphere of prosperity and declines as declining prosperity
makes it necessary for the Government to intervene ever more frequently and drastically in the affairs of
its subjects. Over-population and over-organisation are two conditions which deprive a society of a fair
chance of making democratic institutions work effectively. We see, then that there are certain historical,
economic, demographic and technological conditions which make it very hard for Jefferson’s rational
animals, endowed by nature with inalienable rights and an innate sense of justice, to exercise their
reason, claim their rights and act justly within a democratically organised society. We in the West have
been supremely fortunate in having been given a fair chance of making the great experiment in
self-government. Unfortunately, it now looks as though, owing to recent changes in our circumstances,
this infinitely precious fair chance were being, little by little, taken away from us.

6. According to passage which of the following option checks liberalism?


(a ) Reconciling social order with individual freedom
(b ) Political independence and democracy
(c ) Declining prosperity and increasing government intervention
(d ) Over-population and over-organisation

7. Consider the following statements regarding the primary purpose of the author in the passage
1. Define the conditions for social order.
2. Explain a requirement and introduce a warning about a particular requirement.
3. Credit certain thinkers with foresight.
4. Argue for the limitation of a certain form of Government.
Which of the above statement(s) do not reflect the author’s primary purpose?
(a) 1 and 4 (b) 1, 3 and 4 (c) 3 and 4 (d) Only 2

8. Consider the following statements regarding the ‘infinitely precious fair chance’ mentioned in
the passage
1. Jefferson’s rational animals.
2. An atmosphere of prosperity.
3. Organised democratic institutions.
4. Reconciliation of social order with individual freedom.
Which of the above statements are inconsistent with the idea of fair chance?
(a) 1 and 4 (b) 1, 2, 3 and 4 (c) 3 and 4 (d) None of these

9. A dealer marks his goods so as to make 20% profit. But due to a fire in his warehouse, 5% of
his goods are damaged, also 25% became soiled and hence, they had to be sold at half their
cost. If he sells the remaining at the marked price, his net gain or loss is
(a) 2% gain (b) 4% loss (c) 3% gain (d) 3.5% loss

10. A sailor can row a boat 8 km downstream and return back to the starting point in 1 h 40 min. If
the speed of the stream is 2 km/h, then the speed of the boat in still water is
(a) 5 km/h (b) 10 km/h (c) 15 km/h (d) 20 km/h

11. Three wheels making 60, 36, 24 revolutions in a minute start with a certain point in their
circumference downwards. After how long will they first come all together again in the same
position?
(a ) 5 s (b ) 7 s (c ) 9 s (d) 10 s
Stage 1 Know the Trend 5

12. A class has a strength of 48 students, out of which 18 are boys. The students are to be seated in
rows such that the number of students in all the rows is equal. If none of the rows has both
boys and girls, then what is the maximum number of students sitting in one row?
(a ) 4 (b ) 6 (c ) 9 (d ) 8

13. Calculate the number of triangles in the given figure.

(a) 20 (b) 19 (c) 18 (d) 16

Directions (Q. Nos. 14-15) Examine the information given below and answer the questions that follow.
In the network diagram given below, the figure represents the flow of natural gas through
pipelines between major cities A, B, C, D and E (in suitable units). Assume that supply equals
demand in the network (although not on individual nodes).
200
B

700 125 375

x 675
A C E 1400

200
500 350
D
350

14. What is the number of units demanded in B?


(a) 175 (b) 200 (c) 225 (d) 250

15. If the number of units demanded in C is 225, then what is the value of x?
(a) 975 (b) 75 (c) 775 (d) 950

16. A car during its journey travels 30 min at a speed of 40 km/h, another 45 min at a speed of
60 km/h and 2 h at a speed of 70 km/h. Find its average speed.
(a) 58 km/h (b) 63 km/h (c) 67 km/h (d) 71 km/h

17. I forgot the last digit of a 7-digit telephone number. If I randomly dial the final 3 digits after
correctly dialing the first four, then what is the chance of dialing the correct number?
1 1
(a ) (b )
1001 990
1 1
(c ) (d )
999 1000

18. Madhuri travels 14 km westwards and then turns left and travels 6 km and further turns left
and travels 26 km. How far is Madhuri now from the starting point?
(a) 180 km (b) 80 km
(c) 100 km (d) None of these
6 Practice Set 1

19. The yield versus fertiliser input is shown in the graph.


50 (B Maximum point)

C (Saddle point)

Yield
20
A
8 D

O
1 5 10 15
Fertiliser input

Consider the following statements based on this graph.


I. Yield is zero at B and C.
II. There is no yield with no fertiliser input.
III. The yield is minimum at D.
V. The yield is neither minimum nor maximum at C.
Which of the above statements are correct?
(a) I, II and IV (b) III and IV (c) II and III (d) I, III and IV

20. The average speed of a train in the onward journey is 25% more than that of the return
journey. The train halts for one hour on reaching the destination. The total time taken for the
complete to and fro journey is 16 h covering a distance of 800 km. The speed of the train in the
onward journey, is
(a) 45 km/h (b) 47.06 km/h (c) 50.00 km/h (d) 56.25 km/h

Directions (Q. Nos. 21-28) Read the following seven passages and answer the questions that follow
each passage. Your answers to these questions should be based on the passages only.

B Passage 1
Reservation came with the caveat that its usefulness and validity would be examined every few years.
However, since the inception of class-based quotas, the term has morphed to ‘caste-based’ quotas. At
one time, class may have been synonymous with caste. This is not the case any longer. But it doesn’t suit
any politician, or any government, to acknowledge this, since vote bank politics suggest that it is wise to
keep communities, rather than classes, happy. There are other ways of appealing to the economically
backward, after all. The thought behind reservation was that those who come from families which have
been educated for generations have an unfair advantage over those who are first-generation learners.
There is some credence to this idea. But in exploiting the political mileage that this allowed, our parties
have ensured that the principle is all but lost. When you have economically well-off communities, which
have been educated for generations, fighting in the Supreme Court for the removal of the ‘creamy layer’
rider, you know that there is something very wrong about the approach we have taken to reservation.

21. According to the passage which of the following option is correct about reservation
1. The provision for the validity of reservation is examined every few year.
2. It is also termed as class based quotas.
3. It is perpetuating due to vote bank politics.
4. It is based on educational and economic backwardness
(a) Only 1 (b) 1 and 3 (c) 2 and 4 (d) All of the above
Stage 1 Know the Trend 7

B Passage 2
It’s unsure whether they have heard about the Right to Food Act, but this band of women has managed
to ensure that no one from their village goes to bed hungry. Women self-help groups are one of rural
India’s success stories but this group in Sakdi village of Madhya Pradesh has become an agent of change,
literally. They started off as any self-help group, depositing a small part of their income in a common
pool, so that when someone has a pressing need for money, this can come handy.
But saving money was not enough. They realised that a number of people were going hungry. So, they
decided to do something about it. From pooling in money, they graduated to contributing a fistful of
food grains to a food bank. This, in turn, helps feed the hungry. When they collect more than 50 kg of
grains, they organise an event where they donate sacks of food grains to widows, landless families,
elderly people and those who have no source of income," said one of the residents of Sakdi village.

22. Which of the underlying options captures the essence of the Self Help Group in the most
satisfying way as mentioned in the passage?
(a) To think of betterment of the society and its different aspects and not just the individual
person.
(b) To think of betterment of the persons in need with an inclination to get the favor back when
needed.
(c) To support the persons who need a social identity especially from the backward section of the
society.
(d) To assist the persons in need with a mutual contribution for mutual benefit and keeping the
view in mind that they must be viewed as a part of the society.

B Passage 3
In the age of internet online shopping is emerged as a popular option among masses. Time and cost
effectiveness makes shopping easier and attractive. Lower overhead cost allows online stores to offer
attractive prices by giving huge discount. Buyers also no longer have to drive all the way to shops to buy
things. Online shopping beside providing various options also allows to buy things from any part of the
world irrespective of its availability within the country. One can also compare price before purchasing.
This ease of shopping makes one a compulsive shopper.
Nowadays numerous advertisement has bombarded internet and spending more time on internet may
encourage more buying. It not only leads to debt but also cause security concerns as there is risk of your
financial information getting exploited by hacker.

23. Which one of the following assumptions goes against the author’s view with respect to online
shopping?
(a) Online shopping offers attractive prices which ultimately makes a person compulsive shopper.
(b) Online shopping being popular in nature, gets the attention of many people and hence they are
subjected to the barrage of advertisements which results into buying things.
(c) Online shopping offers cost effective deals and it tempts to make frequent purchases thereby
putting a person under debts.
(d) Despite continued purchases made by the people, they still apprehend of the security concerns.
8 Practice Set 1

B Passage 4
‘Police’ means a system of regulation for the preservation of order and enforcement of law, the internal
government of state. Law is the body of rules recognised by people as binding. Order means prevalence
of constituted authority. No doubt police is essential part of any society. Law and order is the first and
foremost pre-requisite of a civilised society .This objective is attained through policing. Under the
Constitution, ‘law and order’ is the direct responsibility of State Governments .The very first entry in
the state list of functions in the seventh schedule is the ‘public order’, followed immediately by police,
including village and railway police. Administration of justice is the third entry. Police is the
instrumentality for both the public order and administration of justice. ‘Law and order’ is the single
most important function of state on which its reputation depends; many of them are in worst condition.
Every state wants more funds but don’t want to share power rather they want it absolute and
unquestioned.

24. Which of the following arguments put forward strengthen the author’s view about police in a
rational and logical way?
(a) Police itself should see to the law and order of the state with an intervention of the judiciary
whenever required.
(b) For a civilised society the police must be efficient enough within the granted powers to deal with
the oddities of the state so that law and order remains intact.
(c) Police should make it strong enough without the law being in their favor so that the
administration of justice can trust them.
(d) Police must know morality and how the laws are to be enforced in a civilised state for up
keeping the law and order.

B Passage 5
Moral policing, fundamentally, refers to the act of enforcing morality in individuals who engage
themselves in, so called, ‘immoral acts’ and also to make sure others too don’t end up doing such things.
The term morality, in today’s time and age, has different meanings for different people and hence moral
policing has different repercussions for every person. As long as a person, consciously, intends on being
morally policed is fine but when vigilante groups consisting of hypocritical people themselves turn to
free the society from the shackles of immoral behaviour, then the problem begins. There have been
innumerable cases of politicians, especially those belonging to the Right wing, who have publicly made
comments which suggest hostility towards people, who they think are immoral. On one hand they want
the public to aspire for a life which is being lived in the West, but, at the same time they feel that most
part of it is not suited to the Indian lifestyle. Apart from the lawmakers even the police have been
actively involved in upholding Indian values, the sad part being the way they go about doing so. Cases of
incarceration of people they presumed to be involved in immoral acts, fining couples for kissing or
holding hands in public or even attacking foreigners for not paying bribe as they thought they exceeded
the time limit for partying.

25. Which of the given arguments about moral policing does the author think is worst in the
context?
(a) When some of the so called moral people start correcting the so called immoral people.
(b) When a politician makes a public comment about moral policing suggesting hostility even when
he advocates a lifestyle so elegant.
(c) When policemen start getting involved in the correction process despite the fact law is not
allowing them to do so.
(d) When foreigners are attacked because they denied paying bribes to the corrupt policemen on the
ground that they have exceeded their time limit for partying.
Stage 1 Know the Trend 9

B Passage 6
The National River Linking Project (NRLP) formally known as the National Perspective Plan, envisages
the transfer of water from water ‘surplus’ basins where there is flooding to water ‘deficit’ basins
where there is drought/scarcity, through inter-basin water transfer projects. Digging further into the
term ‘surplus’ as per the Government, states that it is the extra water available in a river after it
meets the humans’ requirement of irrigation, domestic consumption and industries thereby
underestimating the need of the water for the river itself.
The term ‘deficit’ has also been viewed in terms of humans only and not from the river’s perspective,
which includes many other factors. The project could also create many water conflicts both at the state
and international level. The country is already reeling due to many inter-state water conflicts like the
Ravi-Beas Water Dispute between Punjab-Haryana-Rajasthan and the Cauvery Water Dispute between
Kerala-Karnataka-Tamil Nadu-Puduchery to name a couple.

26. Which of the following options require the rational attention from the government on a
primary basis with respect to the river linking project in the country?
(a) Government must prepare an outline for the transfer of water from the water surplus area to an
area that is water deficit in order to secure equitable distribution of water resources.
(b) Government should see to it that every state gets the proper amount of water for its agriculture,
industry and its population and then it should decide about the project.
(c) Government must seek help from the state level administration in order to resolve the existing
conflict of water sharing which is the primary concern in this whole issue.
(d) Government should start its project and let the things come in the way and then try to resolve
them without bothering too much about the help from the states.

B Passage 7
Einstein’s concept of the universe as a four-dimensional space time continuum becomes plain and clear,
when what he means by ‘continuum’ becomes clear. A continuum is something that is continuous. A
ruler, e.g. a one-dimensional space continuum. Most rulers are divided into inches and fractions, scaled
down to one-sixteenth of an inch. Will it be possible to conceive a ruler, which is calibrated to a
millionth or billionth of an inch. In theory there is no reason why the steps from point to point should
not be even smaller. What distinguishes a continuum is the fact that the space between any two points
can be subdivided into an infinite number of smaller divisions. A railroad track is a one-dimensional
space continuum, on which position can be described at any time by citing a single coordinate, i.e. a
station or a milestone.
The surface of the sea is a two-dimensional continuum and the two-dimensional continuum are latitude
and longitude. An airplane pilot guides his plane through a three-dimensional continuum. He has to
consider latitude, longitude and height above the ground. The space of our world is a three-dimensional
continuum. Just indicating its position in space is not enough while describing any physical event, which
involves motion. How position changes in time also needs to be mentioned. This can be done either by
means of a time table or a visual chart. Similarly, for the best picturisation of the flight of an airplane
from one location to another, a four-dimensional space time continuum is essential. The latitude,
longitude and altitude will only make sense, if the time coordinate is also mentioned. Therefore, time is
the fourth dimension. If a flight has to be looked at, it should be perceived as a continuous
four-dimensional space time continuum curve.
10 Practice Set 1

27. The significant feature of a continuum, according to the passage, revolves around
1. the divisibility of the interval between any two points.
2. an ordinary ruler’s caliber for marking.
Select the correct answer using the codes given below
(a) Only 1 (b) Only 2 (c) Both 1 and 2 (d) None of these

28. The purpose of this passage is to highlight the point that


(a ) plots and sea captains have something in common
(b ) stock market charts may be helpful to physicists
(c ) the fourth dimension is time
(d ) non-mathematicians are often afraid of the common places

Directions (Q. Nos. 29-33) Study the pie charts given below and answer the following questions.
The break-up of the volume share in Mumbai of different car models sold by Maruti car company
for years 2002 and 2003 is shown in figure I. The break-up of the car market according to market
share by sales possessed by different car manufacturing in Mumbai for 2002 is shown in figure II.
Total volume of car sales in the year 2003 in Mumbai is 4 lakh. In the year 2003, the break-up of
Mumbai car market (volumewise) is the same as the break-up in the year 2002.
Omni Others
10% 10%
HM Maruti
Zen
20% Maruti 800 15% 40%
60%
Daewoo
Esteem 15%
10%
Hyundai 25%
Figure I Figure II

29. If in the year 2002, the total number of cars sold in Mumbai was 2.5 lakh, then how many
Esteems were sold?
(a) 1 lakh (b) 0.1 lakh (c) 1.1 lakh (d) 1.01 lakh

30. If the total number of cars sold in Mumbai in the year 2002 is 4 lakh, then how many cars did
Hyundai and Daewoo sell together?
(a) 1 lakh (b) 1.2 lakh (c) 1.4 lakh (d) 1.6 lakh

31. If the total sales of cars in the year 2002 in India was 24 lakh and Mumbai constituted 5% of
India’s total car sales, then what was the sales of Maruti 800 in Mumbai?
(a) 25000 (b) 27000 (c) 28800 (d) 32500

32. If the total car sales in Mumbai grows by 15% in the year 2004 and if the composition of market
shares remains the same as given in figure II, then what is the number of cars sold by HM in the
year 2004?
(a) 59000 (b) 69000 (c) 49000 (d) 79000

33. In Mumbai, if sales of Maruti went up by 20% in 2004 over its value in the year 2003 while the
sales of all other manufacturer remained same, then what percentage of the total sales would
Maruti have in 2004?
(a) 22.22% (b) 33.33% (c) 44.44% (d) 55.55%
Stage 1 Know the Trend 11

34. Two persons are climbing up on two moving escalators which have 120 steps. The ratio of first
person’s speed to that of 1st escalator is 2:3 (steps). The ratio of second person’s speed to that
of second escalator is 3 : 5 (steps). Find the total number of steps they both have taken
together.
(a) 85 (b) 93 (c) 80 (d) 75

35. Examine the information given below.


˜ All the civil servants are loyal. All the loyal persons are patriotic.
Which of the following is not a valid conclusion regarding the above arguments?
(a) All the civil servants are patriotic (b) All the patriotic are civil servants
(c) Some patriotic are civil servants (d) Some loyal persons are civil servants

36. Out of a total 85 children playing badminton or table tennis or both, total number of girls in
the group is 70% of the total number of boys in the group. The number of boys playing only
badminton is 50% of the number of boys and the total number of boys playing badminton is
60% of the total number of boys. The number of children playing only table tennis is 40% of
the total number of children and a total of 12 children play badminton and table tennis both.
What is the number of girls playing only badminton?
(a) 17 (b) 14 (c) 16 (d) 18

37. Examine the information given below.


˜ Some officers are responsible persons. No responsible person is rich.
Which of the following is a valid conclusion regarding the above arguments?
(a) Some responsible persons are officers (b) Some officers are rich
(c) No responsible person is an officer (d) No officer is rich

38. Read the passage carefully and answer the question given below it.
Many persons have been attracted towards smuggling due to huge profits involved in this
anti-national activity. Some of them became millionaires overnight. India has a vast coastline
both on the East and West Coast. It has been a heaven for smugglers who have been carrying
on their activities with great impunity. There is no doubt that from time to time certain
seizures were made by the enforcement authorities, during raids and ambush but even
allowing these losses, the smugglers made huge profits.
The passage best supports the statement that
(a ) smuggling hampers the economic development of a nation
(b ) smuggling needs to be curbed
(c ) authorities are taking strict measures to curb smuggling
(d ) smuggling is fast increasing in our country owing to the quick profit it entails

39. Examine the information given below.


˜
All the faces of cubes are painted with red colour. The cubes are cut into 64 equal small cubes.
How many small cubes have only one face coloured?
(a ) 8 (b) 16 (c) 24 (d) 32

40. The traffic lights at three different road crossings change after every 48 s, 72 s and 108s,
respectively. If they all change simultaneously at 08 : 20 : 00 h, then they will again change
simultaneously at
(a) 08 : 27 : 12 h (b) 08 : 27 : 24 h (c) 08 : 27 : 36 h (d) 08 : 27 : 48 h
12 Practice Set 1

Directions (Q. Nos. 41-47) Read the following six passages and answer the questions that follow
each passage. Your answers to these questions should be based on the passages only.

B Passage 1
Traditional knowledge mining may seem too tedious nowadays, but it was the best way to mine
knowledge before the start of activities such as micro blogging. It involves a series of steps that will
eventually lead to the correct information being found. When mining knowledge bases for knowledge,
one would have to first of all understand the knowledge base they are looking for. Micro blogging has
become a growing trend, both in business and socially. It however has its benefits and limitations that
one should be aware of when doing it, especially when it’s for knowledge management. Micro blogging
offers an easy, convenient and instant way of communication. It may have its limitations but it is
growing by the minute, which should say something about its benefits.

41. Which of the following statements is the most suitable corollary with respect to significance of
the micro blogging?
(a) Micro blogging is a modern approach of mining knowledge from the persons who are active on
the platform and it is growing tremendously with some limitations.
(b) Micro blogging has its own advantages and limitations like other social platform but it is the
instant way of communication.
(c) The exceptional growth of micro blogging clearly reflects the benefits it has and it has made
communication simpler.
(d) Micro blogging is popular among the professionals and it is likely to gain charm in the time to
come.

B Passage 2
While producing alternative energy, less greenhouse gas is emitted, thereby saving the environment from
climate changes such as global warming. Pollution is becoming a rising issue as greenhouse gas levels rise
in the atmosphere from long-term dependence on fossil fuels. Long-term effects of pollution include a
global increase in temperature which harms wildlife and also increase in sea levels.
This increase in temperature harms many different forms of life such as those of the polar bears. Their
homes are starting to melt away as the higher temperatures are disturbing their normal lifestyle. On the
contrary, renewable energy sources emit close to none when it comes to greenhouse gases. Cutting down
on harmful gas emissions will benefit the environment in the long-run and such adaptations to
renewable energy are the beginning steps to reducing pollution. The need to find new energy sources is
rising, making renewable energy sources a rising option. The future holds many challenges; one of them
is being secure in the supply of energy. With this, new sources for energy must be implemented in order
to replace the fossil fuels that are running out.

42. Which of the given options advocates author’s view of finding the alternative sources of
energy for future?
(a) There is an essence to reduce the use of fossil fuels to make the environment safe for living and
sustenance.
(b) The dependence on the fossil fuel should be reduced so that the abodes of the animals can be
protected.
(c) People should cut down the use of fossil fuels in order to bring the level of greenhouse gas down.
(d) The reserves of the fossil fuels are depleting fast so for the proper supply of energy alternatives
are to be found.
Stage 1 Know the Trend 13

B Passage 3
An important feature of the Indian Judicial System is that it’s a ‘common law system’. In a common law
system, law is developed by the judges through their decisions, orders, or judgements. These are also
referred to as precedents. Unlike the British legal system which is entirely based on the common law
system, where it had originated from, the Indian system incorporates the common law system along
with the statutory law and the regulatory law.
Another important feature of the Indian Judicial system is that our system has been designed on
the pattern of the adversarial system. This is to be expected since courts based on the common
law system tend to follow the adversarial system of conducting proceedings instead of the inquisitorial
system. In an adversarial system, there are two sides in every case and each side presents its
arguments to a neutral judge who would then give an order or a judgement based upon the merits of
the case.

43. What is the most rational assumption that the author has implied regarding the common law
system in the passage?
(a) This system allows the development of law based on the decisions, orders and judgments of the
judges.
(b) The Statutory Law and the Regulatory Law can be incorporated in the common law system
that makes it really sane.
(c) The system allows adversarial system and hence offers an equal chance to the parties concerned
in the case.
(d) It being the oldest system in the country, has an insight despite some limitations to oversee.

B Passage 4
To keep the prices of essential commodities under control, and within reasonable limits, the Indian
Government had constituted the Cabinet Committee on Prices and the Special Committee of
Secretaries on Monitoring Prices. These bodies monitor the prices and supplies of essential commodities
regularly. Apart from these, the Department of Consumer Affairs monitors the prices of essential
commodities on a daily and weekly basis.
Those commodities that are in short supply are imported. Though the government’s steps to check
inflation are laudable, these measures will have a positive impact on the prices only when they are
coupled with a massive drive against hoarders, black marketers and anti-social elements.

44. As per the passage which viewpoint of the author can be inferred from the passage?
(a) Inflation is a man made as well as a natural phenomenon that needs to be addressed by
government and public both especially in the segment of essential commodities.
(b) To combat the problems of inflation not only government action is needed but the public also
needs to raise a voice against those who hoard and black market the essential commodities.
(c) The public must support the government in the weekly or monthly monitoring process so that
the hoarders or black marketers can be booked under law.
(d) A mutual attempt of public and the government would help curb the situation of black
marketing and hoarding which is the only way out possible.
14 Practice Set 1

B Passage 5
Cancer is a group of diseases characterised by an uncontrolled growth of abnormal cells and it is the
result of an uncontrolled cell-cycle. If the spread of these abnormal cells is not controlled by radiation
therapy or other means, then cancer can cause death. Most cancers take the form of tumours, although
not all tumours are cancers. A tumour is simply a mass of new tissue that serves no physiological
purpose. It can be benign, like a wart or malignant, like cancer. Benign tumours are made up of cells
similar to the surrounding normal cells and are enclosed in a membrane that prevents them from
penetrating neighbouring tissues. A malignant tumour or cancer, is capable of invading surrounding
structures, including blood vessels, the lymph system and nerves.
These invading cells interfere with the normal functioning of different body cells and it is because of this
that death of the host tissue usually follows. In addition to spreading to surrounding areas, cancer can also
spread to distant sites by the blood and lymphatic circulation and so can produce invasive tumours in
almost any part of the body. In 2010, approximately 7500000 people in the India were diagnosed with
cancer and approximately 5000000 died of the disease. Early screening for cancer is believed to be able to
drastically reduce the number of deaths due to the disease. Knowing what to look for when detecting
cancer, as well as knowing if you are in a high risk population, are two aspects of early prevention which
have been proven to increase survival rates and lower the length and severity of treatments.

45. Which of the following can be inferred from the passage?


1. Cancer past a certain point always results in death.
2. Women have a higher risk than men of contracting lung cancer.
3. Cancer is characterised by an uncontrolled cell-cycle and a result in the spread of tumours
throughout the body.
Select the correct answer using the codes given below
(a) 1 and 2 (b) 2 and 3 (c) Only 3 (d) All of these

46. Which of the following can be inferred from the passage?


1. Finding of tumours will result into cancer.
2. Cancer is capable of invading the lymph system and nerves.
3. There are two aspects of early prevention of cancer.
(a) 2 and 3 (b) All of these (c) 1 and 2 (d) 1 and 3

B Passage 6
Social security is as old as society itself, but its forms have been changing according to the needs and the
level of social consciousness of the people. Before the industrial revolution, the requirements of social
security were met by institutions like the joint family, church, guilds and caste. The family was the first
line of defense and it constituted the original cell of security. Measures adopted by different societies for
protecting the needy individuals have been manifold. Beginning with individual acts of charity and
philanthropy, these devices progressed to include mutual benefit schemes, both formal and informal.

47. Which one of the following is the most suitable corollary in the context of social security
scheme?
(a) It is the universal approach of helping the persons who are socially unconscious and they do not
know about the measures of doing it.
(b) It is one of the oldest means of keeping the public interest and allowing them to be secure in the
down turns of the time.
(c) It is an old concept that keeps on varying in its forms with the advent of new time and the
social concepts.
(d) It is deep rooted in the civilisation itself and the change is seen when society needs a reform.
Stage 1 Know the Trend 15

48. Examine the following information and answer the question that follows.
Employed people
8
6 Backward people
11 3
17 5 7
Educated people

How many backward uneducated people are employed?


(a ) 3 (b ) 4 (c ) 5 (d ) 6

49. Examine the information given below.


˜ A set S contains the following elements: {7, 11, 15, 19, 23, x}. What is the value of x?
Two statements, labelled 1 and 2, are given below. You have to decide whether the data given
in the statements are sufficient for answering the question. Using the data given in the
statements, you have to choose the correct alternative.
Statements
1. The elements are in arithmetic progression.
2. x is a prime number.
(a) Statement 1 alone is sufficient but statement 2 alone is not sufficient to answer the question
asked
(b) Statement 2 alone is sufficient but statement 1 alone is not sufficient to answer the question
asked
(c) Both statements 1 and 2 together are sufficient to answer the question asked but neither
statement alone is sufficient to answer the question asked
(d) Each statement alone is sufficient to answer the question asked

50. A train overtakes two persons who are walking in the same direction in which the train is
going at the rate of 2 km/h and 4 km/h and passes them completely in 9 s and 10 s,
respectively. The length (in metres) of the train is
(a) 70 (b) 80 (c) 60 (d) 50

51. Mohan has coins of 50 paise, 25 paise and ` 1.50 in the ratio of 1 : 2 : 3. (Mohan stays in a
country where all are valid currency coins. Also, country where ` 1 equals 100 paise). How
many coins of 25 paise does Mohan have, if he got ` 6600 in all?
(a) 2000 (b) 2200 (c) 2400 (d) 2600

Directions (Q. Nos. 52-53) Examine the information given below and answer the questions that
follow.
Nine varsity cricket players — G, H, I, J, K, L, M, N and O are to be honoured at a special
ceremony. Three of these players H, M and O are also varsity football players. Two of them K and
N are also basketball players of the varsity team. In arranging the seats, it was decided that no
athlete in two sports should be seated next to another two sport athletes.

52. Which of the following cannot sit next to M?


(a ) G (b ) J (c) G and J (d ) K

53. Before all athletes are seated, there are two vacant seats on either side of N. Which two athletes
may occupy these seats?
(a) G and K (b) G and L (c) J and H (d) L and O
16 Practice Set 1

54. Examine the following arguments.


˜ Some graduates are postgraduates. ˜ All postgraduates are professor.
˜ No scientist is a professor.
Which of the following conclusion(s) can be drawn from the above arguments?
Conclusions
1. Some scientists are graduates.
2. Some scientists are postgraduates.
3. No scientist is a postgraduate.
(a) Only 1 follows (b) Either 1 or 2 follows
(c) Either 2 or 3 follows (d) Only 3 follows
2
55. In a question, division is of the dividend and 2 times the remainder. If the remainder is 75,
3
then find the dividend.
(a) 85 (b) 145 (c) 225 (d) 65

56. Examine the information given below.


˜ It is 8.00 pm. When can a boy get the next bus for Ranchi from Durgapur?
Two statements, labelled 1 and 2, are given below. You have to decide whether the data given
in the statements are sufficient for answering the question. Using the data given in the
statements, you have to choose the correct alternative.
Statements
1. Buses for Ranchi leave after every 30 min, till 10:00 pm.
2. Fifteen minutes ago, one bus has left for Ranchi.
(a ) Statement 1 alone is sufficient while Statement 2 alone is not sufficient to answer the question
(b ) Statement 2 alone is sufficient while Statement 1 alone is not sufficient to answer the question
(c ) Either Statement 1 or Statement 2 is sufficient to answer the question
(d ) Both Statements 1 and 2 together are sufficient to answer the question

Directions (Q. Nos. 57-58) Examine the information given below and answer the questions that
follow.
˜
There is a group of 6 persons A, B, C, D, E and F seated around a circular table.
˜
There are two females and four males in this group.
˜
Both the females are married to two of the males of the same group. The remaining two males are
bachelors.
˜
A, a female, is sitting opposite to one of the bachelors, F. Her husband is not on either of her sides.
˜
Another female is sitting opposite to her husband, D.
˜
The two females are not sitting adjacently.
˜
On the immediate right of both the females is seated a bachelor.
˜
C has the same marital status as F while B is a father of two sons.

57. Who is the other female in the group?


(a ) E (b ) C
(c ) D (d) Cannot be determined

58. Moving clockwise, how many bachelors are there between A and her husband, if one starts
from the husband?
(a) Zero (b ) 2
(c ) 1 (d) Data inadequate
Stage 1 Know the Trend 17

59. I am facing South. I turn right and walk 20 m. Then, I turn right again and walk 10 m. Then,
I turn left and walk 10 m and then turn right and walk 20 m. Then, I turn right again and walk
60 m. In which direction am I with reference to the starting point?
(a) North (b) North-East (c) North-West (d) East

60. Seven men A, B, C, D, E, F and G are sitting in a row in that order from left to right and each
one is wearing a different coloured shirt among red, blue, yellow, green, violet, pink and black.
The person who wears red coloured shirt sits to the right of D but is not F. The person who
wears green coloured shirt sits to the left of C. G wears pink coloured shirt. F does not wear
yellow coloured shirt. C wears blue coloured shirt. If A wears black coloured shirt, then the
colour of the shirt worn by F is
(a) yellow (b) red (c) green (d) violet

Directions (Q.Nos. 61-67) Read the following four passages and answer the questions that
follow the passages. Your answers to these questions should be based on the passages only.

B Passage 1
“The setting up of the Bharatiya Mahila Bank is a small step towards the economic empowerment of
women. It is also a reflection of our commitment to this cause. We are sure that the bank will fulfil the
objective with which it is being established, namely financial inclusion of women and providing them
equal and easy access of financial services. Empowering the women folk of rural India has much to do
with their economic independence. A source of income for themselves is the first step towards their
liberation. The bank has tied up with Drishtee, an NGO that works with the National Skill
Development Corporation, to run small shops with an initial capital of ` 25,000. It has disbursed loans
to seven small shops in Lucknow. Drishtee meanwhile has started the concept in Bihar with nine shops.

61. On the reading of the above passage which statement weakens the viewpoint of the author
about the setting up of Bhartiya Mahila Bank?
(a) This has been done in order to impart financial inclusion among women so that their
empowerment can be insured.
(b) This has been a dedicated effort to provide the women especially from the rural India an easy
access to all the financial services.
(c) It has been set-up with a view to make the women financially independent which will be the
first step towards their liberalisation.
(d) This is a project that is targeting the women of the poor states in order to strengthen them.

B Passage 2
The gold monetisation schemes will tap household gold stocks and the sovereign bond scheme would
help reduce the physical demand of gold for investment. Customers can open a gold savings account
with a bank and earn tax-free interest on gold assets like bullion or jewellery. The Reserve Bank of India
has issued a circular for implementation of the gold monetisation scheme and banks are now putting in
place the requisite processes for implementation. Over the course of time this is also expected to reduce
the country’s dependence on the import of gold. Interest and principal payments on gold deposits under
the scheme will be denominated in gold. Banks can use the stock of gold they collect as part of their
Statutory Liquidity Ratio requirement, under which they have to make 21.5% of their investments in
government bonds.
18 Practice Set 1

62. The primary purpose of the gold monetisation scheme can be understood in the best way from
which of the following statements?
(a) This scheme is an approach to use the gold that is lying idle with the several household of the
country.
(b) This scheme aims to make Reserve Bank of India more responsible in terms of its function by
assigning the task of gold monetisation.
(c) This scheme targets at the good functioning of the banks and to enrich their capital so that
revenue can be geared.
(d) This scheme is meant for the buying of more government bonds by the common public and by
the financial institutions.

B Passage 3
To accelerate the efforts to achieve universal sanitation coverage and to put focus on sanitation, the
Prime Minister of India launched the Swachh Bharat Mission on 2nd October, 2014. The Mission
Coordinator shall be the Secretary, Ministry of Drinking Water and Sanitation with two Sub-Missions,
the Swachh Bharat Mission (Gramin) and the Swachh Bharat Mission (Urban), which aims to achieve
Swachh Bharat by 2019, as a fitting tribute to the 150th Birth Anniversary of Mahatma Gandhi, which
in rural areas shall mean improving the levels of cleanliness in rural areas. It has become a ‘Jan Andolan’
receiving tremendous support from the people. Citizens too have turned out in large numbers and
pledged for a neat and clean India. People have started to take part in it and are helping spread the
message of ‘Cleanliness is next to Godliness’.

63. What rational and logical inference can be drawn from the reading of the above passage?
(a) People were unaware of the importance of cleanliness earlier and there was a need to make them
aware of it which leads to the mission.
(b) To pay a real tribute to Mahatma Gandhi it was required that a mission like this comes into force.
(c) A kind of Jan Andolan is always a handy tool to make these kinds of projects a huge success.
(d) This mission will act as a catalyst in revolutionising the clean India mission which is an utmost
priority in the country.

B Passage 4
If the expansion of the universe is to stop, there must be enough invisible matter in the universe to exceed
the luminous matter in density by a factor of roughly 70. It can be calculated that the critical density of
matter needed to break the expansion and close the universe is equivalent to three hydrogen atoms per
cubic meter but the density of observable universe in the form of galaxies is only a fraction of this.
It has been known for some time that outside the bright nucleus of a typical spiral galaxy, luminosity
falls off rapidly with distance from the centre. If luminosity were a true indicator of mass, most of the
mass would be concentrated toward the centre. Outside the nucleus the rotational velocity would
decrease geometrically with distance from the centre, in conformity with Kepler’s law. Instead we have
found that the rotational velocity in spiral galaxies either remains constant with increasing distance
from the centre or increases slightly. This indicates that the fall off in luminous mass with distance from
the centre is balanced by an increase in non-luminous mass.
Our findings suggest that as much as 90% of the mass of the universe is not radiating at a wavelength
with enough intensity to be detected on the Earth. Such dark matter could be in the form of extremely
dim stars of low mass, of large planets like Jupiter or of black holes, either small or massive. While it has
not yet been determined whether this mass is sufficient to close the universe, some physicists consider it
significant that estimates are converging on the critical value.
Stage 1 Know the Trend 19

64. The authors propose all of the following as possibly contributing to the ‘missing matter’ in
spiral galaxies except
(a) massive black holes (b) small and dim stars (c) massive stars (d) large planets

65. It can be inferred from information presented in the passage that if the density of the universe
were equivalent to significantly less than three hydrogen atoms per cubic meter, which of the
following would be true as a consequence?
1. Luminosity would be a true indicator of mass.
2. Different regions in spiral galaxies would rotate at the same velocity.
3. The universe would continue to expand indefinitely.
Select the correct answer using the codes given below
(a) Only 1 (b) Only 2 (c) Only 3 (d) None of these

66. The passage suggests that the results of the author’s study, have changed their ideas about
which of the following characteristics of spiral galaxies?
1. The relative luminosity of different regions.
2. The relative rotational velocity of different regions.
3. The relative distribution of matter in different regions.
Select the correct answer using the codes given below
(a) Only 1 (b) Only 2 (c) 2 and 3 (d) All of these

67. The author’s study indicates that, in comparison with the outermost regions of a typical spiral
galaxy, the region just outside the nucleus can be characterised as having
1. similar rotational velocity and higher luminosity.
2. lower rotational velocity and lower luminosity.
3. lower rotational velocity and higher luminosity.
4. higher rotational velocity and higher luminosity.
Select the correct answer using the codes given below
(a) Only 1 (b) 2 and 3 (c) Only 3 (d) 3 and 4

Directions (Q. Nos. 68-70) Study the graph carefully to answer the questions that follow.
Profit (in lakh) made by three companies over the years Profit = Income - Expenditure
Company A Company B p Company C
9 p
p
Profit ( ` in lakh)

8 p
7 p p
6
5 p
4
3
2
1
0
2008 2009 2010 2011 2012 2013
Years

68. Profit made by company A in the year 2009 was what per cent of the total profit made by all
the three companies in that year?
(a) 31.25% (b) 28.24% (c) 21.43% (d) 36.25%

69. If the income of company A in the year 2012 was ` 1354300, then what was its expenditure in
that year?
(a) ` 921600 (b) ` 833500 (c) ` 648200 (d) ` 754300
20 Practice Set 1

70. What is the approximate average profit made by company A in all the years together?
(a) ` 398000 (b) ` 382000 (c) ` 483000 (d) ` 512000

Directions (Q. Nos. 71-72) Read the following information carefully and answer the questions
that follow.
1. A is mother of B. 2. C is sister of A. 3. D is father of C. 4. B is son of E.

71. Which of the following statement(s) is/are required to establish that E is a male?
(a) Only 4 (b) Both 2 and 4 (c) Both 1 and 4 (d) Both 2 and 3

72. How is D related to E?


(a) Father (b) Grandfather (c) Son-in-law (d) Father-in-law

Directions (Q. Nos. 73-76) Examine the information given below and answer the questions that
follow.
Five males A, B, C, D and E and four females P, Q, R and S are to be divided into three debating
teams of three persons each. The teams are designated as the Saviours team, the Warriors team
and the Kings team.
1. C, D and E must be the captains of their teams.
2. A and B cannot be in the same team.
3. P and E cannot be in the same team.
4. R must be on a team with either A or D or both.
5. R must be on the Saviours team.

73. Which of the pairs of students could be on the same team as P?


(a) A and D (b) D and E (c) D and S (d) Q and R

74. Any of the following pairs of students could be on the Saviours team except
(a) A and E (b) C and D (c) D and P (d) D and S

75. If E is the only male in his team, then which of the following pairs could be the other members
of his team?
(a) P and Q (b) R and S (c) R and Q (d) S and Q

76. If A and E are on the Warriors team, then which of the following is not an acceptable
assignment of students for the other two teams?
Saviours team Kings team
(a ) B, D and R C, P and S
(b ) D, P and R B, C and S
(c ) R, D and Q B, C and P
(d ) Q, R and D S, C and B

77. An old person receives pension as ` 1000 in first year and after that 95% of the amount in the
preceding year. If the old man lives forever, how much pension does he get?
(a) ` 15000 (b) ` 10000 (c) ` 20000 (d) ` 1053

78. According to Sud, his age is greater than 25 year but less than 34 year but according to his
brother, his age is greater than 26 year but less than 32 year and according to his mother, his
age is less than 29 year. What is the average of all his probable ages, if all the above estimations
are correct?
(a) 27.5 yr (b) 28 yr (c) 27 yr (d) 28.5 yr
Stage 1 Know the Trend 21

79. Examine the information given below.


˜ Can Rahul retire from office A in January 2010, with full pension benefits?
Two statements, labelled 1 and 2, are given below. You have to decide whether the data given
in the statements are sufficient for answering the question. Using the data given in the
statements, you have to choose the correct alternative.
Statements
1. Rahul will complete 30 year of service in office A in April 2004 and desires to retire.
2. As per office A rules, an employee has to complete minimum 30 year of service and attain age
of 60. Rahul has 3 year to complete age of 60.
(a ) Statement 1 alone is sufficient while statement 2 alone is not sufficient to answer the question
(b ) Statement 2 alone is sufficient while statement 1 alone is not sufficient to answer the question
(c ) Either Statement 1 or Statement 2 is sufficient to answer the question
(d ) Both Statements 1 and 2 are sufficient to answer the question

80. Examine the information given below.


˜ How many New Year’s greeting cards were sold this year in a shop?
Two statements, labelled 1 and 2, are given below. You have to decide whether the data given
in the statements are sufficient for answering the question. Using the data given in the
statements, you have to choose the correct alternative.
Statements
1. Last year 2935 cards were sold in the same shop.
2. The number of cards sold this year was 1.2 times that of last year.
(a ) Statement 1 alone is sufficient while Statement 2 alone is not sufficient to answer the question
(b ) Statement 2 alone is sufficient while Statement 1 alone is not sufficient to answer the question
(c ) Either Statement 1 or Statement 2 is sufficient to answer the question
(d ) Both Statements 1 and 2 are sufficient to answer the question
22 Practice Set 1

Space for Rough Work


Stage 1 Know the Trend

Answer with Explanations


1. (d) Statement 1 and 2 best sum up the view point of the 9. (d) Let total number of goods = 100
author. The passage clearly indicates that superstitions and price per good be ` 100, then
are prevalent because people have no scientific outlook marked price = ` 120
and therefore they can’t decide what reality is and what is
Then, his total investment = v (100 ´ 100) = ` 10000
based on truth. But, with the advancement in science and
Price recovered = (25 ´ 50 + 70 ´ 120) = ` 9650
technology people shall be aware of it and thereby, shall
[Qdamaged 25 at 50% discount, 70 at 20% profit]
be able to decide truth.
So, his net loss = (10000 – 9650) = ` 350
2. (c) We know that science is a good servant but a bad master.
\ Loss percentage = æç ´ 100ö÷ = 3.5%
350
It clearly indicates that science must be used with a è 1000 ø
conscience otherwise it will be useless and dangerous. On
this ground we can say that every new discovery is the 10. (b) Let the speed of the boat in still water be x km/h
outcome of logic and reasoning which also forms the and speed of stream be 2 km/h.
basis of innovations and civilisation both. Then, speed in upstream = (x - 2) km / h
3. (a) The passage suggests that human mind is very inquisitive Speed in downstream = (x + 2) km / h
and it always has a desire for explanation of the mysteries 8 8 40 2 5
\ + =1 =1 =
and when this is served well it gets satisfied with the notion x+2 x- 2 60 3 3
that the explanation was final. But the writer asserts that
Þ 5x 2 - 48x- =
20 0
what is mystery will remain a mystery ever. Therefore the
contrast lies here in this statement. Þ 5x - 50x+ 2 x-
2
=
20 0

4. (c) According to the passage success is what one achieves Þ 5x (x - 10+


) 2 (x- =)
10 0
by the effort of one’s own but this is largely meant for the Þ (5x + 2 ) (x - 10=
) 0
betterment of the society. Also there is no self or no 2
Þ x = 10 or -
interest involved with the success. 5
5. (d) Option (d) is incorrect. Getting involved in making new \ Required answer = 10 km / h
friends, joining sports, student council and after school 11. (a) Time taken for one revolution by each wheel is 60 , 60 and
activities increases one’s popularity. 60 36
60 5 5
6. (c) Declining prosperity leads to more government i.e. 1, , s, respectively.
24 3 2
interventions in the life of people to get the things correct
5 5 5
and in this way checks liberalism. \ LCM of 1, , = = 5 s
3 2 1
7. (b) The ‘requirement’ in Statement 2 is the need for giving
democracy a ‘fair chance’, which is what the author keeps 12. (b) Number of boys in the class = 18
on reiterating.In the last sentence of the passage, he Number of girls in the class = 48 - 18= 30
warns that the chance is being taken away. He is neither HCF of 18 and 30 = 6
arguing to limit democracy (eliminate Statement 4) nor is So, a row can have a maximum number of 6 students.
he defining conditions for social order in general (eliminate
13. (d) B
Statement 1). Statement 3 is too narrow to be the main
point of the whole extract. F
8. (d) Statement 1 is with reference to individuals with rights which E G C
A
have been bestowed upon the individual by nature and I
therefore these are rational animals. Hence, it is consistent
H
with ‘fair chance’ as democracy needs these individuals to
claim right according to the passage. D
Statement 2 is also consistent according to the second Number of triangles formed from single unit = 4 triangles
paragraph. Statement 3 is correct according to 1st line of Number of triangles formed from two units = 8 triangles
the 3rd paragraph as the problem is due to over organised Number of triangles formed from four units = 4 triangles
institutions. It can be inferred that organised institutions \ Total number of triangles = 4 + 8 + 4 = 16 triangles
would be appropriate. Statetment 4 is also consistent as it
is the basic idea of democatic institution acording to the 14. (b) From the figure, it would be 700 - (375 + 125) = 200.
1st line of the passage. Hence, all the statements are 15. (a) From the figure,
consistent.
125 + x - 200- =
675 225
Þ x = 975
26 Practice Set 1

æ 30 ´ 40ö + æ 45 ´ 60ö + (2 ´ 70) Due to vote bank politics politicians are supporting
ç ÷ ç ÷
è ø è 60 ø
16. (b) Average speed = 60 reservation. So, option 3 is also correct. In the passage it
30 45 is mentioned that for 1st generation learner reservation is
+ +2
60 60 needed i.e. educational backwardness. It doesn’t talk
20 + 45 + 140 205 about economic backwardness.
= = ´ 60 km/h = 63.07 » 63 km/h
30 + 45 + 120 195
22. (d) As mentioned in the passage, the SHG is thinking of
60 providing assistance to the persons in need. This help
17. (d) It is given that last 3 digits are randomly dialed. would come out of a mutual contribution and it will fairly be
Then, each of the digit can be selected out of 10 digits in for the mutual benefit of the society as a whole.
10 ways. 23. (d) People shop frequently online and they are quite used to it
3 but still there remains an apprehension regarding the
Required probability = æç
1ö 1
\ ÷ = security concerns as they think that some fraudulent may
è 10 ø 1000
do something with their credentials.
18. (a) The movement of Madhuri is as follows
N 24. (b) As obvious, police must remain within the stipulated power
14 km A (Starting point) but they must be efficient enough so that they can perform
their role in the best way and be able to maintain law and
6 km

6 km W E order in the society which is the essence of the civilised


society.

14 km B 12 km C 25. (d) The argument that the author thinks is worst is when the
26 km S foreign nationals are attacked for denying the payment of
bribe. Rests of the statements are also worst but abashing
In right angled D ABC, by using Pythagoras theorem, a foreign national is highly undesirable.
AC = (BC )2 + ( AB)2 = (12 )2 + (6)2 = 180 = 180 km 26. (c) River linking project has been an ever cherished dream
Hence, Madhuri is 180 km far from the starting point. and it is not a reality till the date because conflict
exists between or among the states over the water
19. (b) Yield rate is not zero at B and C. Despite of no fertilisers distribution. Therefore the government must seek the help
input the yield is 20. So, both I and II statements are wrong from the states to resolve the conflict and then start
while III and IV are correct because yield is minimum at D working on the project.
and the yield is neither minimum nor maximum at point C.
27. (a) The words ‘What distinguishes a continuum is the fact that
20. (d) Let the speed of the train during returning journey be the space between any two points can be sub-divided into
x km/h. an infinite number of smaller divisions’ indicate that one of
25x 5x
Then, speed during onward journey = x + = km/h the significant features of continuum is the divisibility of the
100 4 interval between any two points.
800
Distance covered in onward jouney =
= 400 km 28. (c) The whole passage revolves around the concept of
2
dimensions and it starts as well as ends with the mention
Covered distance
Q Total time taken = of four-dimensions. The last sentences of the passage
Speed clearly indicate that the fourth dimension is time.
400
\ Time taken by train in onward journey = 29. (b) Given, total number of cars sold = 2.5 lakh
5x / 4
\ Sales of Maruti = 40% of 2.5 lakh
400
and time taken in returning journey = Hence, sales of Esteem =10% of sales of Maruti
x = 10% of 40% of 2.5 lakh
According to the question, = 01
. lakh
400 400 320 400
+ = 16 Þ + = 16 30. (d) Given, total number of cars sold = 4 lakh
5x / 4 x x x
Total percentage of Hyundai and Daewoo sold
Þ 16x = 720
= (25 + 15)% = 40%
\ x = 45 km/h
Total number of cars sold together by Hyundai and
5 ´ 45
So, the speed of train in the onward journey = Daewoo = 40% of 4 lakh = 1.6 lakh
4
= 56.25 km/h
31. (c) Total sales of cars in India = 24 lakh
Total sales of cars in Mumbai = 5% of 24 =1.2 lakh
21. (b) Option 1 is mentioned at the beginning of passage.
Total sales of Maruti = 40% of 1.2 lakh = 0.48 lakh
Option 2 is wrong as it is ‘caste based quotas’ not
‘class-based’. \ Total sales of Maruti 800 = 60% of 0.48 lakh
= 60% of 48000 = 28800
Stage 1 Know the Trend 27

32. (b) Total car sales in Mumbai in 2003 = 4 lakh 7


Ratio of boys and girls = x : x = 10 : 7
10
Total car sales in Mumbai in 2004 = 115% of 4 = 4.6 lakh 10
Number of boys = ´ 85= 50
\ Number of cars sold by HM = 15% of 4.6 lakh 17
= 0.69 lakh = 69000 and number of girls =
7
´ 50= 35
10
33. (c) Maruti sales in the year 2003 = 40% of 4 lakh = 16
. lakh
Number of boys (playing only badminton)
Increase in Maruti sale in the year 2004 = 20% of 1.6 lakh
50
= 0.32 lakh = ´ 50= 25
100
Thus, Maruti sales in the year 2004 60
Number of boys (playing badminton) = ´ 50= 30
= 1.6 + 0.32 = 1.92 lakh 100
and total sale = 4 + 0.32 = 4.32 lakh \ Number of boys (playing badminton and table tennis)
\ Percentage share of Maruti = æç
1.92 ö
÷ ´ 100= 44.44% = 30 - 25= 5
è 4.32 ø
Number of children (playing badminton and table tennis)
34. (b) Steps covered by first person = 2 ´ 120 = 2 ´ 120 = 48 = 12
(2 + 3)
5 \ Number of girls (playing badminton and table tennis)
120 = 12 - 5= 7
Steps covered by second person = 3 ´
(3 + 5) 40
So, children (playing only table tennis) = ´ 85= 34
120 100
=3´ = 45
8 Number of boys (playing only table tennis)
\ Total number of steps taken by both the persons = 50 - 30= 20
= (48 + 45) = 93 Number of girls (playing only table tennis) = 34 - 20= 14
35. (b) ‘All the civil servants are loyal’ can be represented as Number of girls (playing only badminton)
= 35 - 7- 14 = 14
Loyal Civil servants 37. (a) ‘Some officers are responsible persons’ can be
represented as
‘All the loyal persons are patriotic’ can be represented as
Officers Responsible
persons
Patriotic Loyal

‘No responsible person is rich’ can be represented as


On combining the above two, we get
Responsible Rich
persons
Civil Loyal
servants
On combining the above two, we get

Patriotic Responsible
Rich
persons
From the above diagram, Conclusions (a), (c) and (d) can
be drawn, but as it can be seen that ‘Some patriotic are Officers
civil servants but not all’, so option (b) is a valid
(i)
conclusion.
36. (b) 5 boys Officers

Badminton Table tennis Responsible


persons Rich
25 boys 20 boys
12 (ii)
14 girls 14 girls
So, option (a), i.e. some responsible persons are officers
is valid in both the figures. Rest options (b), (c) and (d) are
7 girls not valid.
Total children = 85 38. (d) The passage above is based on the increase in smuggling
Let the number of boys be x. activities due to huge profits involved in it. Hence, the
70x 7 x passage best supports the option (d).
\ Number of girls = =
100 10
28 Practice Set 1

39. (c) 47. (c) As the passage suggests that social security is a very old
Red concept but it has kept on changing with the need and
situation as per the requirement. Hence, this is the most
suitable corollary in the context.
Red
48. (c) To find the number of backward uneducated people that
4 cm Red are employed, we have to find the common region of both
4 cm the circles, excluding the triangle. ‘5’ shows that particular
region.
4 cm
49. (c) Statement 1 The elements are in arithmetic progression.
These are 64 small cubes, n = 3 64 = 4
So, x could either be 3 or 27. So, this statement alone is
Number of small cubes having only one face coloured insufficient to answer the question.
= (n - 2 )2´ Number of faces = (4 - 2 )2´ 6 = 24 Statement 2 x is prime i.e. x could be any prime number.
40. (a) Let us first calculate LCM of 48, 72 and 108 This statement alone is also insufficient to answer the
question.
2 48, 72, 108
Combining the two statements, we know that x could take
2 24, 36, 54
only two values from Statement 1, i.e. 3 or 27; from
2 12, 18, 27 Statement 2, we know that x is prime.
2 6, 9, 27 \ 3 is the only value that satisfies both the conditions.
3 3, 9, 27
50. (d) Let us assume the speed of the train be x km/h.
3 1, 3, 9 9 10
Then, ( x - 2 ´) = (- x ´ 4)
3 1, 1, 3 3600 3600
1, 1, 1 Þ 9x - 18= 10x- 40
\ x = 22 km/h
\ LCM of 48 s, 72s and 108 s 1000
Thus, distance travelled = (22 - 4´) = 5 m/s
= 2 ´ 2´ ´ 2 ´ 2´ ´ 3 3 3 = 432s 3600
Thus, the second time the three lights will change after and time =10 s
432 s = 7 min 12 s So, length of train = 5 ´ 10= 50 m
Hence, next time the three lights will change 51. (c) The ratio of the number of coins is 1:2:3 for the 50 paise,
simultaneously at 25 paise and `1.50 coins, respectively. In terms of
08 : 20 : 00 + 00 : 07 : 12 = 08 : 27 : 12 h monetary value, the ratio becomes

41. (a) Micro blogging can be described in the best way by option (1 ´ 0.5) : (2´ 0.25) : ´ (3 1.5)
(a). It is a modern tool of evacuating knowledge from the which equals 0.5 : 0.5 : 4.5 or 1 : 1 : 9
\ æç
user as well as it is growing with a pace. It has its own 1 ö
th÷ of the total value comes from 25 paise coins.
merits and demerits. Hence, statement (a) holds well in è 11 ø
the context. 1
´ 6600= ` 600 is in the form of 25 paise coins.
i.e.
42. (a) The new sources of energy should be found out soon 11
because it will make the environment safe for living and will 600
\ Total number of 25 paise coins = = 2400
cut down the amount of greenhouse gases produced by 0.25
burning of fossil fuels thereby ensuring a balance in Solutions (Q. Nos. 52-53)
ecosystem.
From the given information, it is clear that H, M, O, K and N
43. (c) The most rational assumption is that our law system allows play two games and therefore cannot be seated next to
the adversarial system which offers an equal chance to another.
put their cases in the front.
52. (d) Since, H, M, O, K and N cannot be seated together.
44. (b) One of the major reason for inflation is black marketing Therefore, K cannot sit next to M.
and the hoarding. The Govt. is trying hard to check the
53. (b) Any two cricket players from G, I, J and L, can sit on either
menace but only this will not be enough. So, public
side of N. Hence, G and L can occupy these seats.
awareness must be there to raise a strong voice against it
just in order to check it. 54. (d) Statement two + Conversion of statement three gives the
conclusion ‘no postgraduate is a scientist’. [Q A+E = E],
45. (c) The use of the word ’always’ makes Statement 1 incorrect. which on conversion gives Conclusion 3. So, Conclusion 3
Statement 2 is not given in the passage.
follows and thus Conclusion 2 does not follow. Again,
46. (a) Statement 1 cannot be substantiated from the passage. Statement one + no postgraduate are not scientist.
Statements 2 and 3 are mentioned in the passage. [QI + E = 0]
Hence, Conclusion 1 does not follow.
Stage 1 Know the Trend 29

55. (c) Given, remainder =75 62. (a) The primary purpose of the gold monetisation scheme
2 lies with the fact that it is aimed at using the gold that is
\ Divisor = 2 ´ 75= 150 and of the dividend = Divisor lying idle with the households in the country.
3
3 63. (d) It is well understood that cleanliness is a personal
\ Dividend = ´ 150 = 225
2 discipline and people are aware of it. The truth is that
people are mainly concerned with personal hygiene
56. (d) Statement 2 reveals that the previous bus had left at 7:45 pm.
and not with the community hygiene. Therefore it is
As given in the Statement 1, the next bus would leave after
required that a mission like this should be in force.
30 min i.e. at 8:15 pm. So, both statements are needed to
Hence, (d) concludes better.
answer the question.
64. (c) The authors do not include massive stars in their list of
Solutions (Q. Nos. 57-58) possible explanations for missing matter.
The complete arrangement is as follows
65. (c) An inference is drawn from the stated information. This
Here, Male question refers to the first paragraph, where the authors
A
Female explain that the critical density of matter needed to
C D break the expansion and close the universe is
Bachelors
equivalent to three hydrogen atoms per cubic meter. If
the density is significantly less, then the universe will not
‘close’ but continue to expand indefinitely. So, the
correct answer is Statement 3.
E B 66. (c) The authors did not change ideas about luminosity.
F However, the authors changed their ideas about both
rotational velocity and the distribution of matter. So, the
correct answer is option (c).
57. (a) E is the other female.
67. (a) The region has similar rotational velocity and higher
58. (a) If one starts clockwise from the husband B, then there is no luminosity.
bachelor between A and her husband B.
68. (a) Profit made by the company A in the year 2009 = ` 5
59. (b) According to the given question, the following diagram will be lakh
drawn as Total profit made by all the three companies in the
60 m year 2009 = ` 16 lakh
N
NW NE 5
\ Required per cent = ´ 100 = 31.25%
20 m 16
10 m W E 69. (d) Profit of company A in the year 2012 = ` 600000
\ Required expenditure = Income - Profit
10 m SW SE = 1354300 - 600000
S
20 m = ` 754300
3 + 5 + 4 + 5 + 6 + 6ö
Therefore, I am facing North-East direction. 70. (c) Required average = ` æç ÷ lakh
è 6 ø
60. (d) It is given that A, B, C, D, E, F and G are sitting in a row in that
= ` 4.83 ´ 100000
order from left to right A B C D E F G.
= ` 483000
Given the person who wears red coloured shirt sits to the
right of D but is not F and G wears pink coloured shirt. Solutions (Q. Nos. 71-72)
Hence, E wears red coloured shirt. Given C wears blue From the given information, family tree can be made as
coloured shirt and the person who wears green coloured shirt shown below
sits to the left of C. Hence, either A or B wears green coloured (–)
D
(+) Female
shirt. If A wears black, then B wears green coloured shirt. (+) Male
Father
\ The distribution is as shown below (–) (–) (+)
C Sister
A Couple
E
A B C D E F G Mother
Son
(+)
Black Green Blue Yellow Red Violet Pink B

Hence, option (d) is correct. 71. (c) Only Statements 1 and 4 are required to establish that
61. (d) The Bhartiya Mahila Bank has been set-up to improve the E is a male.
financial condition of the women; be it of a poor state or of a 72. (d) From the family tree, it is clear that D is the father of E’s
rich one. Hence, the last statement weakens the argument. wife. Hence, D is the father-in-law of E.
30 Practice Set 1

Solutions (Q. Nos. 73-76) 77. (c) Pension in first year = ` 1000 and thereafter 95% of the amount
Captain ® C, D and E in the preceding year.
A ¹ B, P ¹ E So, the pension received by him forms an infinite GP with
R = A or D or (A + D), R ® Saviours team é a ù
a = 1000 and r = 0.95 ê QSum of infinite GP, S ¥ =
73. (c) As D, E are captains, so they can’t go in the same ë 1 - r úû
team. So, option (b) can be eliminated. Option (d) can
So, the old man will get = [1000/(1 – 0.95)]
be eliminated on the ground that having no captain.
= ` 20000
The statement 4 states that R must be in a team with
if he remains alive forever.
either A or D or both. So, option (a) can be eliminated.
Hence, (c) is the right option as it does not violate any 78. (a) Let his age be x years. Then,
of the conditions. According to Sud: 25 < x < 34
74. (b) A team consisting of R, C and D in the Saviours team According to his brother: 26 < x < 32
would violate the condition that C and D be the According to his mother: x < 29
captains of their respective teams. a
Q Sum of infinite GP, S ¥ =
75. (d) Combination (a) is not possible as E and P cannot be 1- r
in the same team. Combination (b), E, R and S is not So, if all the above estimates are true, then his probable ages
possible as R must be in the team with either A or D could be 27 or 28 yr and hence, average of all probable ages
or both. Combination (c), E, R and Q is not possible, is 27.5 yr.
as R must be in a team with either A or D or both,
79. (d) Clearly, the facts given in 1 and 2 contain two conditions to be
hence option (d) is the right choice as E, S and Q do
fulfilled to get retirement and also indicate that Rahul fulfils only
not violate any rule.
one condition (first) out of them. So, both statements are
76. (d) In each option, the Saviours team and the Kings team needed to answer the question.
have exactly one captain. In no option, A and B are
80. (d) From both Statements 1 and 2, we find that the number of
together either in the Saviours or the Kings team. In
cards sold this year = (2935 ´ 1.2 =
) 3522. So, both statements
each option, R and D together in the Saviours team.
are needed to answer the question.
Option (d) is not possible as P cannot be assigned to
the Warriors team because E is assigned to that team.
CSAT Paper 2

Practice Set 2

1. There are 80 questions in this paper.


2. The answer of any question you are thinking that more than two answers are true, then you must choose the nearest one.
There is only one answer to be selected by you.
Penalty for wrong answer
3. There are four alternative answers in every question. When you select a wrong answer, then 1/3rd mark of that question is
deducted in your total marks.
4. If any candidate gives more than one answer and one of them is true but it is treated as a wrong answer and the candidate
is penalised for that and1/3rd marks will be deducted.

Directions (Q. Nos. 1-8) Read the following six passages and answer the questions that follow.
Your answers to these questions should be based on the passage only.
KNOW THE TREND
B Passage 1
Illiteracy can bring down even the most powerful nations, so if we are to become a developed nation,
the government should first remove the problem of illiteracy by introducing effective programmes with
proper implementation and budget.
It is ironical that even today; our leaders and people’s representatives give literacy a very low priority.
They fail to perceive literacy as part of the development process, as an endeavour to improve the
quality of life, as the process of building awareness among the weaker sections, as part of the
democratisation of political power, as the arrangement to give their due, to bridge the gap between the
rich and the poor. Neglecting the issue of illiteracy can hurt the development of India very badly.
Stage 1
32 Practice Set 2

1. Which of the following statements is not true in the context of illiteracy of the country as per
the passage?
(a) There is a shortage of a suitable plan and the budget that can accommodate the implementation
of an illiteracy programme.
(b) People’s representatives are working on a suitable measure to curb the menace of illiteracy.
(c) People, especially in the rural India are subjected to severe illiteracy due to lack of proper
infrastructure there.
(d) Removal of illiteracy is required to bridge the gap between the rich and the poor in the country.

B Passage 2
Ninety-seven per cent of the water on the Earth is salt water. Only three per cent is fresh water; slightly
over two thirds of this is frozen in glaciers and polar ice. The remaining unfrozen fresh water is found
mainly as groundwater, with only a small fraction present above ground or in the air. Fresh water is a
renewable resource, yet the world’s supply of clean, fresh water is steadily decreasing. Water demand
already exceeds supply in many parts of the world and as the world population continues to rise, so too
does the water demand. Awareness of the global importance of preserving water for ecosystem service
has only recently emerged as, during the 20th century, more than half the world’s wetland have been
lost along with their valuable environmental services for Water Education.

2. Which among the following is the major contributing factor for the scarcity of water resources
on the surface of the earth as per the passage?
(a) 97% of the water is salt water and only 3% pure water make the equitable distribution of water
difficult over the surface of the earth.
(b) Most part of water is trapped in the form of glaciers and polar ice which makes the fresh water
least available to the fauna over the earth.
(c) Increasing world population is a threat to the proper distribution of water over the different
parts of the world.
(d) The loss of the wetland has caused the acute shortage of fresh water resources across the world.

B Passage 3
The main reason for the high level of unemployment is technological progress. Don’t get me wrong;
progress is good and it makes life easier. But if every year we produce the same amount of goods with
fewer people — in a few years far less working hours are needed to produce all the goods that are
required. The historical trend has been to increase the efficiency of work force. If we do not continue this
trend, the supply of working hours is greater than the demand.
We also know that what is abundant has low value, so an oversupply of working hours means they are
worth less, wages and salaries get reduced. Also many persons are out of work; their working hours are
no longer needed. Those that are out of work have no income and therefore the demand for goods goes
down. With fewer sales, less gets produced, more persons are laid off.
This is a vicious circle that accelerates unemployment and produces crime; because some will turn to
crime to obtain income. By distributing the available work between all persons intend to work the
vicious cycle can be broken.
Stage 1 Know the Trend 33

3. Which of the given inference is most critical in connection to unemployment in India?


(a) The development of technology and the dependence over it has grown manifold causing a lot of
people workless.
(b) The production of same kinds of goods have made the consumption go down and with this job
opportunities have dipped.
(c) Adopting the criminal ways of living have been so lucrative that people don’t bother about
being jobless.
(d) The management has not been able to divide the work available with the firm equally among
the workers and hence causing a mess resulting into unemployment.

B Passage 4
The right to privacy refers to the concept that one’s personal information is protected from public
scrutiny. The right to privacy most often is protected by statutory law. It enforces the right to privacy in
various privacy policies and privacy statements. The right to privacy often must be balanced against the
state’s compelling interests, including the promotion of public safety and improving the quality of life.
Seat-belt laws and motorcycle helmet requirements are examples of such laws. And while many citizens
are quite aware that the government collects personal information, most say that government
surveillance is acceptable. It would doubtless be desirable that the privacy of the individual should
receive the added protection of the criminal law, but for this, legislation would be required. Perhaps it
would be deemed proper to bring the criminal liability within narrower limits; but that the community
has an interest in preventing such invasions of privacy.

4. As per the reading of the passage what rational inference can be drawn in the context of right
of privacy?
(a) The right to privacy is a Fundamental Right of people and it must be protected by the statutory
laws on a compulsory basis despite the situation people are in.
(b) The right to privacy is a Fundamental Right of people and it must be protected by the statutory
laws except some cases when social or national interest is there.
(c) The right to privacy is to be protected from the criminal laws so that the basic human right is
saved from the ill effects of it.
(d) For the criminals there must not be the right to privacy as they are damaging the core idea of
the right.

B Passage 5
Attaining long-term food security requires the raising of incomes and making food affordable. To
ensure food security for the vulnerable section of the society a multiple pronged strategy is to be
evolved. To begin with all the existing social safety net programmes need amalgamation and should
focus on vulnerable and underprivileged regions and groups. The existing anti-poverty programmes may
be made more transparent with better government that minimises leakages and benefits from such
programmes. Simultaneously, agricultural needs to be reformed by improving incentives, increasing
investment etc so that production of traditional and high-value commodities can be increased.
34 Practice Set 2

5. Which one of the following statements is the most suitable corollary with respect to the
achievement of food security for the citizens?
(a) Attaining food security is for long-term needs a threshold income and producing enough food.
(b) A strategy is required for the vulnerable sections of the society for food security but rest of the
sections may be covered with the existing one.
(c) There is a need of a complete overhaul in the government programmes to make the food
available and affordable to ensure food security.
(d) An agricultural reform is needed in order to maintain the food production which will ultimately
pay back in the form of food security.

B Passage 6
History proved beyond doubt that every Empire that evolved and flourished across centuries created its
own grave-diggers. On the same lines, historians have debated the causes of the decline of Mughal
Empire. The causes of the decline of the Mughal Empire were deterioration of land relations, emergence
of regional powers as successor states, selfish struggle of nobles at the court, lack of initiative in modern
weapons, lack of control over the bankers of the state and above all, Aurangzeb's Deccan campaign.
Unlike Emperor Akbar who paid his officials’ salaries directly from the state treasury, Shahjahan and
Aurangzeb opted for Jagirs. There was a constant clash of interest between the nobles at the Emperor’s
court and zamindars. Consequently the main danger to law and order came from zamindars as they
refused to pay the revenue and had to be cowed down.
The politics that emerged upon the collapse of the Mughal Empire was of two kinds. In one class the
‘succession states’ like Hyderabad, Bengal and Awadh, which were really fragments of the Empire. In the
second category were the Maratha Confederacy, the Jats, the Sikhs and the Afghans. Their origins as
polities were independent of Mughal Empire. Mysore under Hyder Ali and Tipu Sultan stood outside
these two categories. At the same time, it was the first state in India to make a beginning towards
modernisation. In Mughal Empire, the nobles found that their careers were not linked to talent and that
loyal and useful service was ‘no security against capricious dismissal and degradation’. Their selfish
struggle led to factionalism. In order to sustain their power in court, these nobles had surreptitious
relations with regional governors, zamindars and other chieftains.
There was an increasing involvement of banking firms in revenue collections at regional and local levels
that brought bankers into positions of political power all over India. The Deccan Campaign became
Aurangzeb’s Waterloo as the Mughals failed to defeat the Marathas. Unlike Emperor Akbar, who
assimilated Rajputs within his kingdom, Aurangzeb was unable to effectively assimilate the regional
warrior chiefs. Failure to sustain imperial officers in the province resulted in intensified disorder and
defiance of imperial authority.

6. Identify the cause that was not responsible for fall of Mughal Empire.
1. Lack of modern weapons.
2. Infighting and factionalism of nobles.
3. Peasant and tribal revolts.
4. Wars with English East India Company.
5. Jagirs awarded by Shahjahan.
Select the answer using the codes given below
(a) 2 and 3 (b) 1, 3 and 4 (c) 3 and 4 (d) 2, 4 and 5
Stage 1 Know the Trend 35

7. The politics that emerged upon the fall of Mughal Empire consisted of
1. Succession states.
2. The Maratha Confederacy, the Jats, the Sikhs and the Afghans.
3. Hyder Ali and Tipu Sultan.
Select the correct answer using the codes given below
(a) 1 and 3 (b) 2 and 3 (c) Only 3 (d) 1 and 2

8. Consider the following statements


1. Banking firms were increasingly involved in revenue collection during Mughal times.
2. The Deccan campaign proved disastrons for Akbar.
3. The Mysore kingdom was first to modernise in India.
Which of the statements given above is/are correct?
(a) Only 3 (b) 2 and 3 (c) 1 and 3 (d) 1 and 2

9. A man buys a bus for ` 500000. If the annual maintenance cost of the bus is 4% and annual tax
is ` 5000, for how much rent per month (in `), should the man rent the bus, so that he gets an
annual return of 20%?
(a) 7500 (b) 9250 (c) 12500 (d) 8750

10. While working alone, Sajid and Wajid can paint a wall in 4 days and 8 days, respectively. Sajid
goes and paints the white portion of the wall to black on the first day. On the second day, Wajid
paints the black portion of the wall to white. They both continue to work in this manner. What is
the maximum number of days after which the whole wall will be painted black?
(a) 12 days (b) 13 days (c) 15 days (d) 16 days

11. The average salary of 100 employees in an office is ` 16000 per month. The management
decided to raise salary of every employee by 10% but stopped the transport allowance of
` 1100 per month which was paid earlier to every employee. What will be the new average
monthly salary? (Assume transport allowance is not included in the salary)
(a) ` 162000 (b) ` 16500 (c) ` 17000 (d) No change in the salary

12. Six bells commence tolling together and toll at intervals of 5, 10, 15, 20, 25 and 30 s,
respectively. In 60 min, how many times do they toll together?
(a) 20 (b) 15 (c) 13 (d) 11

13. Urgently, Sudeep has to travel from Delhi to Lucknow. He travels by a car and notices that, if
he increases his speed by 6 km/h, he will reach Lucknow 4 h early. Also, if he decreases his
speed by 6 km/h, he will take 6 h more. The distance between Delhi and Lucknow (in km) is
(a) 600 (b) 810 (c) 640 (d) 720

14. In a class of 80 students, 25 are studying Commerce, 15 Mathematics and 13 Physics.


3 students are studying Commerce and Mathematics, 4 students are studying Mathematics
and Physics and 2 students are studying Commerce and Physics. 1 student is studying all the
three subjects together. How many students are not studying any of the three subjects?
(a) 40 (b) 35 (c) 20 (d) 15

15. A boy walks 2 km to West and then turns North and walks 10 km. Again, he turns to West and
walks 4 km. After this he turns to South and walks 18 km. Now, how far is he from his starting
point?
(a) 10 km (b) 11 km (c) 12 km (d) 13 km
36 Practice Set 2

16. A leak in the bottom of a tank can empty the full tank in 6 h. An inlet pipe fills water at the rate
of 4 L in a minute. When the tank is full, the inlet is opened and due to the leak the tank is
empty in 8 h. Find the capacity of the tank.
(a) 5000 L (b) 5670 L (c) 5700 L (d) 5760 L

17. 720 sweets were distributed equally amongst children in such a way that number of sweets
received by each chld is 20% of the total number of children. How many sweets did each
receive?
(a) 12 (b) 14 (c) 11 (d) 15

18. A student goes to his school from his house walking at 4 km/h and reaches his school 10 min
late. Next day, starting at the same time he walks as 6 km/h and reaches his school 5 min
earlier than the scheduled time. Find the distance between school and home.
(a) 2 km (b) 3 km (c) 4 km (d) 5 km

19. In a bag containing three balls, a white ball was placed and then one ball was taken out at
random. What is the probability that the extracted ball would turn on to be white, if all
possible hypothesis concerning the colour of the balls that were initially in the bag were
equally possible?
(a) 5/8 (b) 3/4 (c) 1/2 (d) 3/8

20. Of the 200 candidates who were interviewed for a position at a manufacturing unit, 100 had a
two-wheeler, 70 had a credit card and 140 had a mobile phone. 40 of them had both a
two-wheeler and a credit card, 30 had both a credit card and a mobile phone and 60 had both a
two wheeler and mobile phone and 10 had all the three. How many candidates had none of the
three?
(a ) 0 (b) 10 (c) 18 (d) 20

Directions (Q. Nos. 21-28) Read the following seven passages and answer the questions that
follow each passage. Your answers to these questions should be based on the passages only.

B Passage 1
Technical progression at global scale has put more pressure on developing countries to improve their
infrastructure and progress in other important areas for prosperity. In various news report and
documents it is demonstrated that the guidelines to recognise a city as the smart city will be prepared by
the department of industrial policy and promotion. The standards for being recognised as a smart city
must have three of the five infrastructure requirements such as energy management, water management,
transport and travel, safety and security and solid waste management.
21. Which of the following statements has triggered the need of developing the smart cities in the
country?
(a) The lack of basic infrastructure in the country compelled the Govt. to revitalise it and it
becomes the smart city project.
(b) Smart cities are the outcome of planned policies in order to boost the existing infrastructure of
the country.
(c) The need of different aspects of management like energy, water, communication and waste etc.
has forced the Govt. to think of smart cities.
(d) The global technical progression has pressurised the countries with fewer infrastructures to
upgrade it and ultimately it has lead to the concept of smart cities.
Stage 1 Know the Trend 37

B Passage 2
Poverty can be defined as a social phenomenon in which a section of the society is unable to fulfil even
its basic necessities of life. The problem of poverty is considered as the biggest challenge to development
planning in India. High poverty levels are synonymous with poor quality of life, deprivation,
malnutrition, illiteracy and low human resource development. There are two basic pre-requisites of a
poverty eradication programmes. First, reorientation of the agricultural relations so that the ownership
of land is shared by a larger section of the society. Second, no programme of removal of poverty can
succeed in an economy plagued by inflation and spiraling rise of price.

22. What critical inference one can draw from the reading of the above passage?
(a) Poverty alone is the biggest challenge in the country and it needs to be addressed soon by the
government.
(b) Poverty is the other name of the poor quality of life and only human resource development can
mitigate it.
(c) A land restructuring programme can boost the agricultural prospects and ultimately poverty
can be addressed.
(d) A composite of agricultural reform and controlling inflation will help in the mitigation of poverty.

B Passage 3
Article 24 of the Indian Constitution provides for prohibition of child labour. It says ‘No child below the
age of 15 years shall be employed to work in any factory or mine or engaged in any other hazardous
employment.” Despite the above mentioned constitutional provision, the child labour scenario in India
presents a disturbing picture. India tops the list of countries having the highest child labour
population. The main cause of the child labour in India is considered lack of education. The prevalence
of child labour in India is largely due to the failure of the educational system. Due to lack of compulsory
education for which constitutional provisions exist, half of the population of school age children is
either at home or in the labour force.

23. Which of the following statements is the most suitable and rational corollary with respect to
child labour in India?
(a) The lack of strength of our constitution is the main reason for the fostering of the child labour
in our country.
(b) The lack of the manpower in the factories and at the workplace is the major cause of child
labour in India.
(c) The lack of proper education in our country and its poor implementation are the reasons of
surge in the child labour.
(d) Strict policies regarding child labour is not in India and with its less severity it has been a weak
link in the system.

B Passage 4
Panchayati Raj genesis can be seen in ancient period in texts of Panini, Buddhist text and Rig Veda in
respect to administration autonomy. In Manusmiriti, Mahabharat and even in Kautilya’s “Arthshastra”
we get talk about officials which constitute a group of village republics. During Mauryan period, Gupta
period and Sultanate period Panchayats were organised and had performed function of sanitation,
education and judiciary etc. The Britishers had created local self government, as early as 1687, the
municipal corporation of Madras was formed. Then in 1870 a resolution for decentralisation of power
was passed by Bengal Chowkidari Act 1870. It was the first step towards local self government. The
Government of India Act of 1935 also had provision for provincial autonomy and had strengthened
panchayats in the country.
38 Practice Set 2

24. Which of the following options captures the essence of the Panchayati Raj System in the best
way?
(a) The Panchayati Raj System has its roots lying in the very ancient period and therefore it is
necessary to follow.
(b) India is a country of villages and therefore the administration should lie with it to run the
things smoothly.
(c) We have adopted the British laws and therefore it is inevitable to follow the Panchayati Raj
System.
(d) The decentralisation of the administration and to strengthen the villages of the country it is
necessary to adopt the policy of Panchayati Raj System.

B Passage 5
Global temperatures began to rise slowly after 1850, with the beginning of the Modern Warm Period.
There was a vast migration from Europe by land-hungry farmers and others, to which the famine caused
by the Irish potato blight contributed, to North America, Australia, New Zealand, and Southern Africa.
Millions of hectares of forest and woodland fell before the newcomers’ axes between 1850 and 1890, as
intensive European farming methods expanded across the world. The unprecedented land clearance
released vast quantities of carbon dioxide into the atmosphere, triggering for the first time humanly
caused global warming. Temperatures rised more rapidly in the twentieth century as the use of fossil
fuels proliferated and greenhouse gas levels continued to soar. The rise has been even steeper since the
early 1980s. The Little Ice Age has given way to a new climatic regime, marked by prolonged and steady
warming.

25. Which of the following statements can’t be attributed for the rise in Global Warming and that
weakens the argument of the author in the passage?
(a) Excessive usage of fossil fuels has triggered a rise in the greenhouse gases which ultimately
causes a steep rise in the global temperature.
(b) Due to steady and continued rise in the temperature, the Little Ice Age has been gone and in its
place a new climate has taken place.
(c) The excessive land clearance by the Europeans has lead to the fall in the density of the
woodland area and in turn it has affected the global temperature.
(d) The migration, in any form, puts a pressure on the environment and that causes severe stress on
the whole ecosystem.

B Passage 6
Malnutrition does not only arise because of insufficient or too much nutrient intake. The problem may
also occur because of the area where there is a population growth which may lead to insufficient food
supply for some part of the population. But these are not the only ways malnutrition can occur in social
conditions; teenage girls often have a problem with their body and go on diets and become either
anorexic or bulimic or even both. Malnutrition is also a cause of a poor economy where people do not
have the money to buy food and because of that end up having an unbalanced diet. The real problem is
money. Food is available but people can’t afford it. We need real development and income-generating
projects. Poor people live on the basic commodities such as bread, tea, sugar and rice. Vegetables and
meat are too expensive for them.
Stage 1 Know the Trend 39

26. Which of the given assumptions can be a handy tool in the process of alleviating the curse of
malnutrition?
(a) The nutritional chart of every individual should be prepared and it should be supervised
properly.
(b) There must be an opulent supply of the basic food items on affordable cost so that people can
afford it.
(c) Government should produce the food items which are basic such as grains and avoid producing
the high cost items in bulk.
(d) Government must put a check on all the social trends and conditions that make the persons opt
for dieting.

B Passage 7
The miseries of the world cannot be cured by physical help only. Until man’s nature changes, his
physical needs will always arise, and miseries will always be felt, and no amount of physical help will
remove them completely. The only solution of the problem is to make mankind pure. Ignorance is the
mother of evil and of all the misery we see. Let men have light, let them be pure and spiritually strong
and educated; then alone will misery cease in the world. We may convert every house in the country
into a charitable asylum, we may fill lend with hospitals, but human misery will continue until man’s
character changes.

27. According to the passage, which of the following statements is most likely to be true as the
reason for man’s miseries?
(a ) The poor economic and social conditions prevailing in society
(b ) The refusal on the part of man to change his character
(c ) The absence of physical and material help from his society
(d ) Ever increasing physical needs due to changing social structure
28. With reference to the passage, the following assumptions have been made.
1. The author gives primary importance to physical and material help in eradicating human
misery.
2. Charitable homes, hospitals, etc can remove human misery to a great extent.
Which of the assumptions is/are valid?
(a) Only 1 (b) Only 2 (c) Both 1 and 2 (d) Neither 1 nor 2

29. How many triangles are there in this figure.

(a) 35 (b) 40 (c) 37 (d) 32

30. Which number replaces the question mark?


3 12 7
4 15 9
6 11 ?

(a ) 2 (b ) 3 (c ) 4 (d ) 0
40 Practice Set 2

31. A gardener increased the area of his square garden by increasing its side by 25%. The area of
the new garden
(a) has increased by 50% (b) has increased by 56.50%
(c) has increased by 56.25% (d) is exactly the same as the old area

32. In this question, from the given answer figures, select the one in which the question figure is
embedded.
Question figure

Answer figures

(a) (b) (c) (d)

33. In a row of children, Varun is seventh from the left and Neetu is fifth from the right. When
Varun and Neetu exchange their positions, Varun will be sixteenth from the left. Then, how
many students are there in a class?
(a) 18 (b) 19 (c) 20 (d) 21

Directions (Q. Nos. 34-38) Examine the graph carefully and answer the questions that follow.
Number of employees working in different departments in a company
400 375
350
350
Number of employees

300 275
250 225
200
200 175
150
100
50
0
HR Advertising IT Finance Production Marketing
Department
Male-Female Ratio
Department Male Female
HR 9 16
Advertising 3 2
IT 9 31
Finance 2 3
Production 11 4
Marketing 4 3

34. What is the total number of male employees working in all the departments together?
(a) 755 (b) 925 (c) 836 (d) 784
Stage 1 Know the Trend 41

35. What is the total number of female employees in HR department?


(a) 158 (b) 128 (c) 140 (d) 144

36. What is the ratio of the total number of employees working in the production department to
that of the marketing department?
(a) 15 : 14 (b ) 8 : 7 (c) 14 : 15 (d ) 7 : 8

37. In which department, is the number of female employees the least?


(a) Advertising (b) Production (c) HR (d) Finance

38. What is the total number of employees in all the departments of the company?
(a) 1500 (b) 1575 (c) 1582 (d) 1600

39. A dice is thrown four times and its four different positions are given below. Find the number
on the face opposite the face showing number 2.

2 1 3 6

3 4 2 3 5 4 2 4

(a ) 4 (b ) 5 (c ) 6 (d ) 3

40. A scooter before overhauling requires 2/3 h service time every 45 days, while after
overhauling, it requires 2/3 h service time every 60 days. What fraction of pre-overhauling
service time is saved in the latter case?
(a) 4/3 (b) 1/3 (c) 3/4 (d) 1/4

Directions (Q. Nos. 41-47) Read the following six passages and answer the questions that follow
each passage. Your answers to these questions should be based on the passage only.

B Passage 1
Skilling and entrepreneurship complete each other! Skills are fundamental to, but not sufficient for,
gaining decent jobs. Improved productivity through skill development must be complemented by
economic growth and employment opportunities.
They are, collectively, a prerequisite to the government’s pursuit of holistic development of the nation.
Unleashed entrepreneurship, besides employment generation, will also utilise the youth power for
productivity improvement and wealth creation; thus, stimulating equitable development and a
sustainable path for decent livelihood creation for the country. Technology and resources have to be
judiciously harnessed to achieve the same. Furthermore, endeavours need to be aligned to ensure the
outreach of opportunities and services to the bottom of the economic pyramid, facilitated by
enterprises.

41. The author’s view point can be best summed up in which of the following statements —
(a) Skilling and entrepreneurship are complementary to each other.
(b) More emphasis is required on generating employment opportunities.
(c) Emphasis on entrepreneurship will help in employment generation as well as stimulate equitable
development and a sustainable path for decent livelihood.
(d) There is no solution in sight for the problem of providing opportunities to the very poor people
of the nation.
42 Practice Set 2

B Passage 2
India has welcomed the Paris Agreement on Climate Change. An important pact concluded by 196
countries to put the world on the track to sustainable development.
Will it now leave the ‘‘fake horse race’’, as many see it, in which climate responsibility and development
are treated as separate competing entities, and move to a green growth trajectory away from fossil fuels?
That is the central question for national policy posed by the agreement, one of the most anticipated
international pacts in decades, and one that could bring about a fundamental shift in the development
paradigm. Important answers will naturally depend on how willing the West is to pay for clean
development in India and around the world, since everyone, not just the poor, stands to lose from global
warming.

42. What is the most logical, rational and crucial message that is implied in the following passage?
(a) India, would do well to look at the Paris Agreement on climate change through the prism of
opportunity not adversity.
(b) Central question faced by any nation who were involved in this Agreement is ‘how to move to a
green Growth Trajectory away from fossil fuels.’
(c) India’s agreement on the issue of climate change rests primarily on the principle of equity and
common but differentiated responsibilities of developed and developing nation.
(d) Developed nations are not willing to take the whole responsibility of making the world free from
global warming.

B Passage 3
BRIC stands for four emerging nations, Brazil, Russia, India and China. It is a coalition of emerging
superpowers proposed by Russian President Vladimir Putin.
The coalition encompasses over 40 per cent of the world's population and holds a combined GDP of
$12.14 trillion. The idea was proposed because of growing concerns regarding the domination of the US
and the European Union over the world economy and political scene. As all the members have
experienced enormous growth in the past few years and are expected to continue this, the coalition will
likely grow to be a larger threat to developed countries. Unlike other developed nations, BRIC
economies do not want to dominate the world economy. What they want is an end to their poverty and
an end to the global and unilateral leadership of the United States.
The major driving force behind the soaring economy is the GDP growth. China will continue to witness
a GDP growth of over 7 per cent till 2010 and over 4 per cent till 2035. At the same time, India will grow
at a rate of 5-6 per cent. In the case of Russia and Brazil, the growth rate will vary between 3-4 per cent.
Both rising currencies and faster growth will narrow the gap between BRIC and developed economies.

43. Which among the following is the most rational and critical inference that can be made from
the above passage.
(a) U.S. and other European countries will dominate the world economy and political scene in
coming future also.
(b) The BRIC countries (Brazil, Russia, India and China) are going to be a major thereat to the
domination of existing countries over world economy.
(c) Main reason of the BRIC countries going to complete with U.S and other European. economy is
their rising currencies and subsequent faster GDP growth.
(d) Main purpose of BRIC economies is not to dominate the world economy but to make concerted
efforts to end the poverty of their regions. Unilateral leadership of this U.S. will automatically
dissolve due to this purpose.
Stage 1 Know the Trend 43

B Passage 4
Modern economy can be called as ‘Bank economy’. Banking system transact business by means of ever
negotiable bank accounts and help in attainment of domestic security.
Over several decades the banking has been the target of vicious political attack irrespective of any
ideology. In fascist and National socialist countries of yesterdays also banks functioned in spite of their
repeated denunciation of “Interest Slavery”. In communist countries where bank directors were
compared to leaches, the banking is operated.
Unaccounted of the political complexion, racial composition or social philosophy of the government,
banks are indispensable for the administration of an up-to-date economic system.
Banks are the coordinating centres and observation posts of economic activities, transcend national as
well as ideological frontiers in this modern age. At present stage of development, banks alone are capable
of ushering the surplus production into productive investment and thus promote technical progress and
human welfare.

44. Which one of the following is the most logical assumption that can be made from the above
passage?
(a) Bank economy works differently in different political systems.
(b) In spite of continued criticism banking is operational is all sorts of political scenario -
democracy, communism or federalism.
(c) Banks are indispensable for the administration of an update economic system in any part of the
world.
(d) Any nation can make a headway when banks exist there as coordinating centres and observation
post of economic activities.

B Passage 5
Once a conflict gets over, aid agencies sanctioned by the World Bank send study groups instead of
requisite personnel. There is a gap of several years before moving from humanitarian relief to economic
development and hence political stability. By the time such help arrives the war has restarted. It is
possible to restart economic development through targeted ‘quick impact’ initiatives. Most economies
in post-conflict countries are based on agriculture. Providing free packages of seeds, fertilizers and
low-cost agricultural tools will ensure that former soldiers will return to their farms and establish their
livelihood. But the window of opportunity closes quickly and one has to implement these measures
almost immediately.

45. How can economic development be restarted in an impoverished country?


1. Deploying peacekeepers in the country.
2. Restricting number of aid agencies to avoid waste.
3. Long-term studies should be commissioned.
4. Focusing on agricultural initiatives.
Select the correct answer using the codes given below
(a) Only 1 (b) 2 and 4 (c) Only 3 (d) Only 4

46. What is the benefit of ‘Quick Impact’ aid?


1. Providing alternative livelihoods to soldiers before war can restart.
2. Free land is given to soldiers.
3. Soldiers earn income from their farms.
Select the correct answer using the codes given below
44 Practice Set 2

(a) Only 1 (b) 2 and 3 (c) Only 3 (d) 1 and 2

B Passage 6
Water conflicts in India now reach every level; divide every segment of our society-political parties,
states, regions and sub-regions within states, districts, castes and groups and individual farmers. Water
conflicts within and among many developing countries are also taking a serious turn. Fortunately, the
‘water wars’ have not yet materialised. War has taken place, but over oil, not water. Water is radically
altering and affecting political boundaries all over the world, between as well as within countries. In
India, water conflicts are likely to worsen before they begin to be resolved. Till then they pose a
significant threat to economic growth, security and health of the ecosystem and the victims are likely to
be the poorest of the poor as well as the very sources of water rivers, wetlands and aquifers.

47. According to the author, which of the following is/are consequences of water conflicts?
1. Trans-border conflicts between developing countries.
2. Water bodies will remain unused and unaffected till the conflict is resolved.
3. Water conflicts have altered the political boundaries within countries.
Select the correct answer using the codes given below
(a) Only 1 (b) Only 2 (c) Only 3 (d) 1 and 3

48. In the following question, two statements numbered 1 and 2 are given. There may be cause
and effect relationship between the two statements. These two statements may be the effect of
the same cause or independent causes. These statements may be independent causes without
having any relationship. Read both the statements and mark your answer.
Statements
1. The police authority has recently caught a group of house breakers.
2. The citizens group in the locality have started night vigil in the area.
(a ) Statement 1 is the cause and Statement 2 is its effect
(b ) Statement 2 is the cause and Statement 1 is its effect
(c ) Both the Statements 1 and 2 are independent causes
(d ) Both the Statements 1 and 2 are effects of some common cause

49. In the Lok Sabha elections, there are two candidates – A and B contesting from North Delhi
seat. 3/5th of the people promise to vote for A and the rest promise to vote for B. If 10% of the
people, who promised to vote for A did not vote and 20% of the people, who promised to vote
for B did not vote. Also, if B got 6400 votes, then how much was the margin of victory for A?
(a) 5500 (b) 4000
(c) 4400 (d) 8800

Directions (Q. Nos. 50-53) Examine the information given below and answer the questions that
follow.
A railway station, R, located in India has only two platforms, one for trains going from North to
South and other those going from South to North. Details about the six daily trains—Shatabdi,
Simadri, Gautami, Godavari, Rajdhani and Ratnachal at ‘R’ are known. These are the only trains
that stop at station R, while going from South to North. Shatabdi is either the first or the last train
to arrive at station R. No two trains, having their names starting with the same letter, arrive
Stage 1 Know the Trend 45

immediately one after the other. The name of the train that arrives just before Simadri has the same
starting letter as that of the train that arrives immediately after Simadri. Rajdhani Express comes
immediately after Shatabdi Express.

50. Which is the last train to arrive at station R?


(a) Shatabdi (b) Godavari (c) Rajdhani (d) Ratnachal

51. Which of the following trains comes definitely just before Godavari?
(a) Simadri (b) Gautami (c) Rajdhani (d) Shatabdi

52. Gautami may come before which of the following trains?


(a) Rajdhani or Ratnachal (b) Simadri or Ratnachal
(c) Shatabdi or Rajdhani (d) Rajdhani or Simadri

53. If the train that arrives first has 18 compartments and the trains that arrive second, third,
fourth, fifth and sixth have 15, 12, 9, 6, 3 compartments respectively, then which of the
following will have the least number of compartments?
(a ) Trains whose names start with ‘S’
(b ) Trains whose names start with ‘G’ or ‘R’
(c ) Trains whose names start with ‘G’ and ‘R’ both
(d ) Trains whose names start with ‘S’ and ‘G’ both

54. Identify the missing figure to complete the matrix.


Question figure

Answer figures

(a) (b) (c ) (d)

55. Examine the statements given below.


Statements Some carrots are brinjals.
Some brinjals are apples.
All apples are bananas.
Conclusions
1. Some apples are carrots. 2. Some bananas are brinjals.
3. Some bananas are carrots.
Which of the conclusions above is correct?
(a) Only 1 (b) Only 2 (c) Only 3 (d) Either 2 or 3

56. Examine the statement given below.


Statement Local administration made elaborate security arrangements and alerted the local
hospitals to be in readiness during the ensuing festival days.
Assumptions
1. A very large number of devotees may assemble in the city during the festival days.
2. Security personnel may not be able to control the crowd.
46 Practice Set 2

(a ) If only Assumption 1 is implicit


(b ) If only Assumption 2 is implicit
(c ) If either Assumption 1 or 2 is implicit
(d ) If neither Assumption 1 nor 2 is implicit

57. There are 4 villages— A, B, C and D nearby each other. B is to the East of A. Village C is in the
North of B and D is to the right of C. In which direction of D is village A?
(a) South-West (b) North (c) North-East (d) West

Directions (Q. Nos. 58-60) Examine the information carefully and answer the questions that
follow.
A, B, C, D, E, F, G and H are sitting around a circular table facing the centre. D is the neighbour of A
but not of H. B is the neighbour of F and fourth to the left of D. E is the neighbour of H and third to
the right of F. C is neither the neighbour of A nor of G.

58. Which of the following is correct?


(a ) D is to the immediate left of G
(b ) A is between C and D
(c ) F is fourth to the right of D
(d ) None of these

59. Which of the following is wrong?


(a ) B is to the immediate left of H
(b ) H is to the immediate left of E
(c ) D is fourth to the right of F
(d ) C is to the immediate right of E

60. Which of the following groups has the second person sitting between the first and the third?
(a) AFC (b) GAD (c) BEH (d) HFB

Directions (Q. Nos. 61-67) Read the following two passages and answer the questions that follow.
Your answers to these questions should be based on the passages only.

B Passage 1
Recent studies by the Intergovernmental Panel on Climate Change and the International Energy Agency
have suggested that, in order to achieve the international goal of limiting global warming to 2ºC, the
world will need to live within a set carbon budget, and a significant portion of proven global fossil fuel
reserves will need to be left in the ground. The world is currently, however, on a path towards global
warming of 4ºC or more, which the World Bank warned must be avoided in order to prevent
catastrophic climate change impacts.

61. Which among the following is the most logical corollary to the above passage?
(a) The world has to follow certain limitations in order to achieve international goal of limiting
global warming.
(b) The world is currently on a path towards global warming of 4°C or more which must be
avoided.
(c) The restrictions required to contain global warming include within a set carbon budget and
global fossil fuel reserve will need to be left in the ground.
(d) The world has reached on the edge of catastrophic climate change impacts.
Stage 1 Know the Trend 47

B Passage 2
Citizenship today requires individuals be knowledgeable of public problems but, more important, have
the capacity to act together toward their solutions (Morse 1989). Voluntary actions by private citizens
working together to right injustices, change directions and pursue benefits for the common good are
noted throughout.

62. Which one of the following expresses the view implied in the above passage?
(a) Responsible citizens are required to act together to find a solution to a common problem.
(b) There are many examples of good citizens acting voluntarily to fight any injustices and bringing
much required change in the system.
(c) Citizens of today are least bothered about common problems and good of the people.
(d) People all over the world have tried to act together for a common cause and bring justice to the
affected people as they have become more knowledgeable about the existing problems.

B Passage 3
Human rights are the rights relating to life, liberty and equality and dignity of individuals guaranteed by
the Constitution or embodied in international covenants and enforceable by the courts in India.
Human Rights have a projection of universality but as a matter of ground reality they are not the same
for all people and societies. For some, especially the well-advanced West, they are predominantly
matters of civil and political liberty while for others, the developing and under-developed Asian, Africa
and Latin America, predominantly they are also matters of survival. Human Rights are therefore about
being citizen of the earth, being part of an earth family. Human Rights in my view are exercised to their
fullness through participating in earth democracy-the democracy of all life. And as earthlings, our
human duties to protect the earth and all her beings are the ground from Human Rights emerge.
‘‘All humanity is one undivided and indivisible family, and each one of us is responsible for the misdeeds
of all others. I cannot detach myself from the wickedest soul.’’ Mahatma Gandhi -

63. With reference to the above passage which of the following is the most valid assumption?
(a) Human Right are common to all irrespective of nationality or region.
(b) In reality Human Rights in developed countries are related to different freedom but in
Underdeveloped and developing countries the Human Rights pertain to basic survival.
(c) Human rights are the prerogative of all individuals and must be protected everywhere in each
country.
(d) If human beings care about the mother earth, their rights will be automatically safe-guarded.
48 Practice Set 2

B Passage 4
Too much heat may lead to high rate of exhaustion that limits the productiveness of human beings. Low
productivity would in turn lead to lower yield hence, slowing down the economic growth. High
temperatures lead, to a lot of heat being generated that may cause Sun burns in most individuals.
Human beings would also face difficulty in the production of food. The ever changing climatic condition
emanating from global warming serves as a setback in the agricultural economy. The weather condition
would be unpredictable and farmers may suffer a big blow due to climatic variation changes. The
elevated temperatures may lead to cracking of runways and sunken foundations in buildings. This may
act as a negative impact towards the economic growth of a given nation.
In conclusion, global warming serves as a major setback in the development of the economy of a given
nation. Industries are eloping up at a high rate and are posing a great challenge in environment
ecosystem. The amount of pollution from industries is increasing which has left government agencies
with strategic planning to be conducted. The increases in vehicles have also contributed to global
warming and it has posed a challenge on better mitigation measures. The consequences of global
warming may lead to the reduction in progress in the economy of the world, if proper mitigation
strategies are not channelled in faster. Although some of the consequences about global warming seem to
be favourable such as, summer hiking and high productivity due to high carbon dioxide present, the
demerits seem to be severe. This has called for stake holders from various spheres of world to find ways
of dealing with this crisis of global warming.

64. Consider the following statements


1. Excessive heat causes Sun burns in most individuals.
2. Too much heat hampers the prodcutiveness of human beings.
3. Much heat enriches the productivity of environment.
Which of the statement(s) given above is/are correct?
(a) 1 and 2 (b) 2 and 3 (c) Only 3 (d) All of these

65. Consider the following statements


1. Global warming serves as a major setback in the development of the economy of a nation.
2. The increase in vehicles has also contributed to global warming.
Which of the statement(s) given above is/are correct?
(a) Only 1 (b) Only 2 (c) Both 1 and 2 (d) Neither 1 nor 2

66. With reference to the passage, consider the following statements


1. Global warming may cause a threat to agricultural economy.
2. Climatic variation changes are also a crucial factor in agriculture.
3. Government’s policy is falacious in this regard.
Which of the statement(s) given above is/are correct?
(a) 1 and 2 (b) 2 and 3 (c) 1 and 3 (d) None of these

67. The passage is mainly/thematically centered on


(a ) the rising temperature and its consequences
(b ) the government initiative in disaster mitigation
(c ) people’s approaches and involvement in environmental protection
(d ) None of the above
Stage 1 Know the Trend 49

68. Examine the following statement.


˜ Should import duty on all the electronic goods be dispensed with?
Two arguments numbered 1 and 2 are given. You have to decide which of the arguments is a
‘strong’ argument and which is a ‘weak’ argument.
Arguments
1. No, this will considerably reduce the income of the government and will adversely affect the
developmental activities.
2. No, the local manufacturers will not be able to compete with the foreign manufacturers who
are technologically far superior.
(a) Only 1 is strong (b) Only 2 is strong
(c) Either 1 or 2 is strong (d) Neither 1 nor 2 is strong

69. Based on the following statements, find who is the uncle of P.


1. K is the brother of J. 2. M is the sister of K.
3. P is the brother of N. 4. N is the daughter of J.
(a ) K (b ) J (c ) N (d ) M

70. Ajay is lighter than Vipul and Priyanka is heavier than Ajay, while Pramod is lighter than
Priyanka and heavier than Vipul. Then who among them is in the second place from the top
when they are arranged in descending order of heavier weights?
(a) Vipul (b) Priyanka (c) Pramod (d) Ajay

71. In the following figure, which number represents the women doctors who are not employed?
Women
9
6 1 5 Employed
3 2 4

7
Doctors
8
(a ) 7 (b ) 3 (c ) 1 (d ) 8

72. A question followed by information given in three Statements 1, 2 and 3. You have to study
the question along with the information in three statements and decide that the information in
which of the statement(s) is/are necessary to answer the question?
What is the area of the rectangular garden?
1. Perimeter of the garden is 220 m.
2. Length and the breadth of the garden are in the respective ratio of 7 : 4.
3. Length is less than twice the breadth by 10 m.
(a) 1 and 2 (b) 1 and 3
(c) 1 and either 2 or 3 (d) 2 and 3

73. A, B and C started a business with their investments in the ratio of 1 : 2 : 4. After 6 months,
A invested the half amount more as before and B invested the twice the amount as before while
1
C withdrew of their investments. Find the ratio of profits at the end of the year.
4
(a) 5 : 12 : 13 (b) 12 : 5 : 14
(c) 5 : 12 : 14 (d) 5 : 12 : 17
50 Practice Set 2

74. A die is rolled and a coin is tossed. Find the probability that the die shows an even number and
the coin shows a tail.
(a) 3/4 (b) 1/2 (c) 1/4 (d) 1/5

75. A sum is invested as SI at certain interest rate for 2 yr. If the rate of interest had been invested
at 3% higher rate, it would have fetched ` 90 more. The sum (in `) was
(a) 1500 (b) 1200 (c) 2000 (d) 1000

76. Examine the statement given below.


˜ The government has decided not to provide financial support to voluntary organisations
from the next Five Year Plan and has communicated that all such organisations should
raise funds to meet their financial needs.
The statement given above is followed by two courses of action numbered 1 and 2. You have to
assume everything in the statement to be true and on the basis of the information given in the
statement, decide which of the suggested courses of action logically follow(s).
Courses of Action
1. Voluntary organisations should collaborate with foreign agencies.
2. They should explore other sources of financial support.
(a) Only 1 follows (b) Only 2 follows
(c) Either 1 or 2 follows (d) Neither 1 nor 2 follows

77. A tells B “if you give me ` 400, then I shall have 25% more than what you have”. B tells A, “if
you give me ` 200, then I shall have three and a half times as much as you have”. How much
does B have?
(a) ` 1200 (b) ` 900 (c) ` 800 (d) ` 1600

78. A family made a down payment of `750 and borrowed a television set that cost `4000. The
balance with interest was paid in 23 monthly instalments of `160 each and a final payment of
`90. The amount of interest paid was what per cent of the amount borrowed?
(a) 12% (b) 15% (c) 15.5% (d) 16%

79. A producer of tea blends two varieties costing `18 per kg and another `20 per kg in the ratio of
5:3. If he sells the blended variety at `21 per kg, then what is his gain percentage?
(a) 12% (b) 10% (c) 14% (d) 15%

80. A shopkeeper had to arrange a certain number of toys in rows. When he put 5 toys in each row,
he was left with 1 toy. Then, he tried 6 in a row, then 8, then 9, then 12 but always he was left
with 1 toy. Next, he put 13 toys in a row and then he was left with none. What is the smallest
number of toys?
(a) 3491 (b) 3600 (c) 3601 (d) 4601
Stage 1 Know the Trend

Answers with Explanations


1. (b) On reading of the passage we can conclude that the 11. (b) Total salary of 100 employees = 100 ´ 16000
representatives of the public are not very much concerned = ` 1600000
about alleviating illiteracy. Hence, it is not true. Increase in salary = 10% of 16000 = ` 1600
2. (c) The major contributing factor for the scarcity of the water Decrease in salary due to withdrawal of transport
resources is the increasing population of the world. With the allowance = ` 1100
increase in the population the demand for water grows as Net effect = 1600 - 1100 = ` 500
more water is required for agriculture, industry and for So, new salary = ` (16000 + 500) = ` 16500
domestic uses. With the limited resources of water,
population becomes really a contributing factor. 12. (c) LCM of 5, 10, 15, 20, 25 and 30 is 300.
So, the bell will toll together after every 300 s (5 min).
3. (a) The most critical inference in connection to unemployment in 60
India is the growth and development of technology and the Hence, number of times they toll together = + 1 = 13
5
dependence over it which has lead to such a situation where
a firm requires less working hours of the human population. 13. (d) Let the distance between Delhi and Lucknow be d km
and the speed of Sudeep be s km/h, then
4. (b) The passage clearly states that the right to privacy of every
æ 1 1 ö
citizen must be protected by the common law but if there is dç - ÷= 4 …(i)
any social or national interest only then the right may be è s (s + 6)ø
considered to be cut short. æ 1 1ö
and dç - ÷= 6 …(ii)
5. (c) The passage suggests that there is a need of transformation è (s - 6) s ø
in the government programmes in order to achieve food On dividing Eq. (i) by Eq. (ii), we get
security by making the food affordable and available to the d (6)
various sections of the society especially the vulnerable ones.
s (s + 6) 4
Hence, option (c) gives the best corollary in this context. Þ =
d (6) 6
6. (c) Though English East India Company was responsible for (s - 6) s
downfall of Mughal Empire, it has not been mentioned in the (s - 6) 4
passage. Peasant and tribal revolts are also not mentioned in Þ =
(s + 6) 6
the passage.
Þ 6 s - 36= 4 s+ 24
7. (d) The politics that emerged upon the collapse of the Mughal
2 s = 60
Empire was of two kinds. In one class the succession states
like Hyderabad, Bengal and Awadh. In the second category Þ s = 30 km
were the Maratha Confederacy, the Jats, the Sikhs and the On putting the value of s in Eq. (i), we get
d æç
Afghans. 1 1ö
- ÷= 4
è 30 36 ø
8. (c) The Deccan campaign was conducted by the Aurangzeb and
d æç
not Akbar. 6 ö
Þ ÷= 4
è 1080 ø
9. (d) Here, total cost of bus per year
\ d = 720 km
= (` 500000 + 4% of ` 500000 + ` 5000) = ` 525000
For getting a return of 20%, he has to get 20% of ` 525000 14. (b) C M
= ` 105000
2 9 C : Commerce
Hence, rent per month = ` (105000/12) = ` 8750 21
M : Mathematics
10. (b) Wajid paints the black portion painted by Sajid to white on 1 1 3
each day. P : Physics

So, from first day onwards in every 2 days 8


1 1 1
= - = th of the wall gets painted black. P
4 8 8
1 3 Let the sets C, M and P represent the students who are
In 12 days = 6 ´ = th of the wall gets painted black. studying Commerce, Mathematics and Physics,
8 4
1 respectively.
The next day Sajid will paint the rest th part to black.
4 Given, n(C Ç MÇ =P) 1,
Hence, the wall will get painted black in (12 + 1) days n(C ) = 25, n(M ) = 15, n(P) = 13
i.e. 13 days. n(C Ç M=) 3, n(CÇ =P) 2, nÇ (M = P) 4
54 Practice Set 2


Since, n(C È MÈ =P) n(+C ) n(+M ) n- (P) Ç n(C 1+ + + ù
M) 3 1 1
\ Required probability =
- n(CÇ - P) nÇ (M+ P) Ç n(Ç C M P) 4 êë 4 2 4 úû
1 4 + 3 + 2 + 1ù
= é
= 25 + 15 + 13 - 3- - 2 + 4 1= 54 - 9 = 45 5
=
\ Number of students who are not studying any of the 4 êë 4 úû 8
three subjects = 80 - 45 = 35 20. (b) Let A, B and C denote the set of the two wheeler, credit
15. (a) The movement of boy is as follows: card and mobile phone.
P 4 km N \ n( A) = 100, n(B) = 70n(C ) = 140
n( A Ç B=) Ç =C )
40, n(B 30
10 km

N and n( A Ç C=) 60
\ n( A È B
È =C ) n+( A) n+(B) n(C )
18 km

4 km 2 km
A - n( AÇ - B) Ç n(B- CÇ ) +n( A Ç C )Ç n( A B C ))
B M (Starting W E
= 100 + 70 + 140 - 40- 30
- +60 =10 190
8 km

point)
\ n( A Ç B
Ç =C ) -
200 =190 10
S 21. (d) According to the passage, the need of smart cities has
C
been triggered by the global technical progression which is
Starting from the point A, passing through the points M, N the root cause of the development of the smart cities in the
and P, he reaches C. country.
In right angled D ABC, by Pythagoras theorem,
22. (d) It can be inferred from the passage that poverty alleviation
AC = (BC )2 + ( AB)2 = (8)2 + (6)2 = 100 = 10 km programme needs a composite effort of agricultural reform
1 as well as a check on the soaring prices.
16. (d) Efficiency of leak =
6 23. (c) The flourishing of child labour in India is due to the lack of
1 proper education and its failure in the process of
Combined efficiency of leak and inlet pipe =
8 implementation. This is the main reason of an increase n
1 1 4- 3 1 the child labour.
So, the efficiency of inlet pipe = - = =
6 8 24 24 24. (d) The essence of the Panchayati Raj System lies with the
Inlet pipe can fill the tank in 24 h, at the rate of 4 L /min. concept of the decentralisation of the administration that
\ Capacity = 24 ´ 60´ =4 5760 L will strengthen the villages. Hence, it is suitable in the
context.
17. (a) Let total number of the children be x.
25. (d) All the form of migration is not severe to the environment
20 x2
Then, x ´ x= 720 Þ = 720 but those migrations may be severe that aims at the
100 5
downfall of the flora. Loss of flora results into excess
Þ x 2 = 5 ´ 720= 5´ ´ 5 144 carbon emission which ultimately harms the ecosystem.
\ x= 5 ´ 5´ 144
= 60 Hence, (d) is the suitable option.
20
Number of sweets received by each child = ´ 60= 12 26. (b) Most of the people are unable to afford even basic food
100 items because they are so costly. Also, there needs to be
18. (b) Let the distance be x km. an enough supply of it to procure food items to the people.
Distance x So, food items should be affordable and available both to
Then, Time = , 1st day time taken = h alleviate malnutrition.
Speed 4
x æ x - xö h 27. (b) That option (b) is the right answer. It is clear from the
2nd day time taken = h, Difference in time = ç ÷
6 è 4 6ø second sentence of the passage which mentions that
miseries cannot be ended unless man changes his
1
Actual difference between these two times = 15 min = h character. On the other hand the passage makes the point
4
that even if the problems mentioned in the other options
x x 1
According to the question, - = are solved the miseries will still remain.
4 6 4
3x - 2 x 1 28. (d) Option 1 is incorrect. The passage infact mentions that
Þ = Þ x=3 providing physical and material help alone will not help in
12 4
eradicating human misery until man changes his
Hence, the required distance is 3 km.
character. Option 2 is also incorrect. The last sentence of
19. (a) Since, all possible hypothesis regarding the colour of the the passage clearly mentions that opening up of hospitals
balls are equally alike, therefore there could be 3 white and charitable homes will not lead to removal of human
balls, initially in the bag. misery until the change of man’s character.
Stage 1 Know the Trend 55

29. (d) Number of triangles 34. (c) Total number of male employees working in all the
A G B departments together
9 3 9 2
= ´ 225+ ´ +
275 ´ + ´
200 175
J 25 5 40 5
F H
11 4
+ ´ 375+ ´ 350
K I L 15 7
E C
= 81 + 165 + 45 + 70 + 275 + 200 = 836
D
35. (d) Number of female employees in HR department
D ABD D FHD D FGH D FGD 16
= ´ 225= 144
D GHD D GJH D HJD D JDF 25
D GJF D KID D LID D EID 36. (a) Required ratio
D CID D CLD D EKD D AFE =
Total number of employees in production department
D AFG D EFK D AGE D EAK Total number of employees in marketing department
D EAD D EFD D EGC D GHB =
375
= 15 : 14
D BHC D CHL D GCB D BLC 350
D BDC D KLD D HCD D EGI 37. (b) Number of female employees in
D GIC D AGD D BGD 16
HR = ´ 225= 144
25
So, there are 35 triangles in the given figure.
2
Advertising = ´ 275= 110
30. (b) In each row of the diagram, add the left and right hand 5
digits together and add 2 to give the result in centre i.e. 31
IT = ´ 200= 155
In row 1, 3+7+2=12 40
3
In row 2, 4+9+2=15 Finance = ´ 175= 105
5
In row 3, 6+?+2=11
4
\ ?=3 Production = ´ 375= 100
15
31. (c) Let the side of the square be denoted by s. 3
Marketing = ´ 350= 150
New side after 25% increase = 125% of s = 1.25 s 7
Area of square = s 2 So, the number of female employees is least in production
\ New area = (1.25 s )2 = 15625
. s2 department.

Now, required per cent increase in area 38. (d) Total number of employees
.
15625 s2 - s2 0.5625 s 2 = ( 225 + 275 + 200 + 175 + 375 + 350) = 1600
= 2
´ 100 = 2
´ 100
s s 39. (b) The numbers 1, 3, 4 and 6 lie on the faces adjacent to the
= 56.25% number 2. Therefore, the number 5 lies on the face opposite
32. (a) It is clearly seen in the option (a), that question figure is to the number 2.
embedded in it. 40. (d) Time required for service befoer overhauling
2
= h = 40 min in 45 days
3
and time required for service after overhauling
2
33. (c) According to the question, = h = 40 min in 60 days
3
Varun Neetu Now, LCM of 45 and 60 = 180 days
7th 5th \ Time required for service before overhauling in 180 days
L R = 40 ´ 4= 160 min
On exchanging their positions, and time required for service after overhauling in 180 days
= 40 ´ 4= 120 min
Neetu Varun 160 - 120 40 1
16th \ Fraction of time saved = = =
160 160 4
L R 41. (c) Is the appropriate answer. The government wants to focus
Now, total number of students = L + R - 1 more on entrepreneurship as it will expedite employment
= 16 + 5 - 1= 21 - 1= 20 generation. Besides more wealth creation and improvement
in productivity can be achieved by motivating people to be
56 Practice Set 2

entrepreneurs. Skill development is the basic needed to But Godavari and Gautami cannot come immediately after one
get a decent job but unless job opportunities are there. another as both their names start with the same alphabet ‘G’.
Skill development alone will not bring any change in the So, Simadri cannot be the third train.
economic development of the country. Also, Simadri cannot be the fifth train to arrive as then Ratnachal
would be the third train to arrive as fourth and sixth will be either
42. (b) It has been agreed by almost every country to adopt to Godavari or Gautami and both Rajdhani and Ratnachal cannot
clean energies for future development. Use of excessive arrive immediately one after the other.
fossil fuel has been one of the major reasons of global
warming which was adopted by developed countries at So, Simadri is the fourth train to arrive and third and fifth are
either Godavari or Gautami. Ratnachal is the last train to arrive
the time of ‘Industrial Revolution.’ Poor or developing
at station R. So, the order is
countries should be helped financially to adopt to safer
clean energies for the purpose of development. 1. Shatabdi 2. Rajdhani
3. Godavari / Gautami 4. Simadri
43. (d) BRIC countries are experiencing rising currencies and
impressive GDP growth. Their enormous growth will 5. Gautami / Godavari 6. Ratnachal
definitely theatern the domination of existing world 50. (d) Ratnachal is the last train to arrive at the station R.
economies and also end the poverty of their regions.
51. (c) Rajdhani’s arrival is definitely just before Godavari.
44. (c) Banks have been and are an integral part of any
52. (b) Gautami may come before either Simadri or Ratnachal.
nation’s economy. They alone can help any country to
promote technical progress and human welfare. Any 53. (c) Number of compartments in trains whose names start with
area devoid of banking facilities will suffer and will not S = Shatabdi + Simadri = 18+9 = 27 compartments
be able to mobilise the untapped financial resources. R = Rajdhani + Ratnachal = 15+3 = 18 compartments
45. (d) In the 4, 5 and 6 sentences of the passage clearly give the G = Godavari + Gautami = 6+12 = 18 compartments
reason for focusing on agricultural initiatives, i.e. to So, both the trains whose names start with ‘G’ and ‘R’ have
establish the livelihoods of the people. The other minimum total number of compartments.
statements are neither mentioned nor implied in the
passage. 54. (a)
46. (a) In the sixth sentence of the passage bring this out
clearly as the correct choice. The other statements are
not specifically mentioned in the passage.
47. (c) Water conflicts are radically altering and affecting
political boundaries all over the world, between as well
as within countries.
55. (b)
48. (d) Both the statements are clearly backed by a common Carrots
Brinjals Apples
cause, which is clearly an increase in the number of
thefts in the locality. Bananas

49. (c) Let total votes be x Conclusions


Then, people who voted for A
1. O 2. P 3. O
= 60% of x - 10% of 60% of x = 0.54x
56. (a) During the festival days, a large number of devotees may
And people who voted for B
assemble in the city for which the local administration is
= 40% of x - 20% of 40% of x = 0.32 x, making appropriate security arrangement. So, Assumption 1
Margin of victory for A = 0.54 x - 0.32 x= 0.22 x is implicit. Security personnel are able to control the crowd.
6400
Now, 0.32 x = 6400 Þ x= = 20000 So, Assumption 2 is not implicit.
0.32
57. (a) According to the question, following diagram is drawn.
Margin of victory for A = 0.22 ´ 20000= 4400 votes N
Solutions (Q. Nos. 50-53) C D
According to the information 1 and 4, we can deduce that W E
Shatabdi is the first train and Rajdhani is the second train A B
to arrive.
SW
Simadri should be the third or fourth or fifth train to arrive. If S
Simadri is the third train, then Ratnachal should be the
fourth. Fifth and sixth would be either Godavari or Gautami. Hence, A is in the South-West direction of D.
Stage 1 Know the Trend 57

Solutions (Q. Nos. 58-60) 60. (b) A is sitting between G and D.


As it is given that B is the neighbour of F and fourth to the 61. (c) Some restrictions are to be followed. Firstly, a limited
left of D. So, the arrangement is carbon budget has to be followed by many countries and
D D reserves of fossil fuels are to be kept underground to avoid
or prevent catastrophic climate change impacts.
62. (d) It is really a pleasant thing to note that knowledgeable
people have joined hands and tried to bring a change in the
OR system. The concerned person has been able to get some
justice at least.

F F 63. (c) Every Individual’s right of equality and dignity must be


protected irrespective of nationality or soil. If we all follow
B B
the basic rule that any one who lives on this earth, has the
Case I Case II right to lead a dignified life, the problem of human rights will
Now, E is third to the right of F. This information rejects the not show its ugly face. Only when rights of people are
Case I. violated, problems and issues crop up is any society.

Also, it is given that E is the neighbour of H and third to the 64. (a) The passage itself implies the impact of excessive
right of F, so the following two possibilities are there temperature. Too much heat may lead to sun burns and
D D inhibition in productiveness in most individuals. Actually
heat is necessary for existence of life on the Earth, but not
H in excessive amount, but in turn it causes harm to
individuals.

E Or E 65. (c) The passage clearly signifies the increasing heat and its
impacts on living beings. The excessive heat causes global
warming which in turn affects the economy of a nation.
F F H While increasing vehicles have also contributed in the
enhancement of global warming.
B B
Case II (a) Case II (b) 66. (a) The given passage explicitly talks about the impacts of heat
in excessive amount. But in passage, nowhere is
Now, from the information that D is the neighbour of A but mentioned about the government policies.
not of H. Hence, Case II (a) can be rejected. So, the
following two possibilities are there 67. (a) The central theme of the passage is to show the causes of
excessive heat and the rising temperature. The uncontrolled
D D
heat radiation causes the inhibition in productivity, sun burn
G/C A A C/G and lowering down in agricultural output.
68. (b) Abolishing the import duty on electronic goods shall reduce
C/G E Or C/G E the cost of imported goods and adversely affect the sale of
the domestic products, thus giving a setback to the Indian
electronics industry. So, Argument 2 holds strong.
F H F H Argument 1 does not provide a convincing reason.
B B
69. (a) According to the question, blood relation diagram is as
Case III (a) Case III (b)
follows
Also, it is given that, C is neither the neighbour of A nor of G, (+) Brother (+)
K J
so Case III (a) can be rejected.
So
Da

Hence, the final seating arrangement will be as follows


Sister

n
ug

D
hte

A C
r

Brother (+)
(–)
M (–)
N P
G E
Here, (+ ) = Male, (- =
) Female
F H
B (—) = linking line
As, P is the son of J.
58. (d) None of the given options is correct.
Given, K is the brother of J.
59. (c) D is fifth to the right of F.
Therefore, K is the uncle of P.
58 Practice Set 2

70. (c) Q Vipul > Ajay 75. (a) Let the sum be ` x and the interest rate be r % per annum.
2 2
Priyanka > Ajay Then, x ´ (r+ 3´) - ´ x ´ (r ) = 90
100 100
Priyanka > Pramod
= 2 x(r + 3) - 2 xr= 90´ 100 Þ 6x = 9000
Priyanka > Pramod > Vipul > Ajay
\ x = ` 1500
Hence, Pramod is at second place from top, when arranged
in descending order of heavier weights. 76. (b) The problem arising is shortage of funds. So, alternative
sources of financial support need to be worked out first.
71. (a) ‘7’ represents the women doctors who are not employed. Thus, only course 2 follows.
72. (c) From Statement 1, 77. (a) Let the money with A be a
Perimeter of the garden = 220 m and money with B be b.
2(l + b ) = 220 …(i) According to the question,
l 7 7b
From Statement 2, = Þ l= 1.25(b - 400=
) a+ 400
b 4 4
Þ 1.25b - 500= a+ 400
On putting the value of l in Eq. (i), we get
Þ 1.25 b - a= 900 … (i)
2 æç + b ö÷ = 220
7b
è 4 ø Again, 3.5(a - 200=
) b+ 200
7 b + 4b 220 11b Þ 3. 5 a - 700= b+ 200
Þ = Þ = 110
4 2 4 Þ 3.5a - b= 900 … (ii)
110 ´ 4 Multiplying Eq. (ii) by 1.25 and then adding Eqs. (i)
\ b= = 40 m
11 and (ii),
7b 7 ´ 40
and l= = = 70 m we get
4 4 1.25 b – a =900
\ Area = l ´ b = 70 ´ 40 = 2800 sq m – 1.25 b + 4.375 a =1125
From Statement 3, l + 10 = 2b Þ l = 2b - 10
Þ 0.375 a = 1125 - 900
Again putting the value of l in Eq. (i), we get
Þ 0.375 a = 255 Þ a = 600
2 (2 b - 10+ b= ) 220
Putting the value of a in Eq. (i),
220
Þ 3b - 10= we get
2
. b - 600= 900 Þ
125 . b = 1500
125
Þ 3b = 110 + 10 =120
120 Þ b = 1200
Þ b= = 40
3 \ a = 600 and b = 1200
\ b = 40 m and l = 70 m So, B have ` 1200.
Now, area = l ´ b= 70´ 40 = 2800 sq m 78. (d) Down payment made = 750
So, we can easily find the area of the rectangular garden \ Balance = 4000 - 750= 3250
with the help of the Statements 1 and either 2 or 3. Total amount paid in 23 months = 23 ´ 160= 3680
73. (c) Let us assume their initial investments be x, 2x and 4x, Total amount paid = 3680 + 90 = 3770
respectively. \ Extra amount paid = 3770 - 3250= 520
Therefore, ratio of their investments during the whole year 520
\ Required percentage = ´ 100 = 16%
= æç x ´ 6+ x´ 6ö÷ : (2 x ´ 6+ 4x´ 6) : (4x ´ 6+ 3x´ 6)
3 3250
è 2 ø
79. (a) Let the quantities of varieties be 5 kg and 3 kg.
=15x : 36x : 42 x = 5x :12 x :14x \ Total cost = 5 ´ 18+ 3´ 20 = 90 + 60 = 150
\ Ratio of their profits = 5:12 :14 Selling price = 21per kg
74. (c) The sample space S of the experiment described in the \ Revenue = 21 ´ 8= 168
question is as follows 168 - 150 18
\ Proft percentage = ´ 100 = ´ 100 = 12%
S ={(1, H), (2, H), (3, H), (4, H), (5,H), (6, H), (1, T), 150 150
(2,T), (3, T), (4, T), (5, T), (6, T)} 80. (c) LCM (5, 6, 8, 9, 12) = 360
So, n(s ) = 12 So, smallest such number which will leave 1 remainder
Let E be the event that ‘the die shows an even number and in each case = 360 + 1 = 361
the coin shows a tail’. and series = 360 N + 1
So, E = {(2, T), (4, T), (6, T)} Þ n(E ) = 3 After putting N = 10, we get
3 1
\ Required probability, P(E ) = = 360 ´ 10+ 1= 3601
12 4
which is completely divisible by 13.
CSAT
C i v i l S e r v i c e s A p t i t u d e Te s t
Paper 2

Practice Set 3
Time : 2 hrs MM : 200

Instructions

1. There are 80 questions in this paper.


2. The answer of any question you are thinking that more than two answers are true, then you must choose the nearest one.
There is only one answer to be selected by you.
Penalty for wrong answer
3. There are four alternative answers in every question. When you select a wrong answer, then 1/3rd mark of that question is
deducted in your total marks.
4. If any candidate gives more than one answer and one of them is true but it is treated as a wrong answer and the candidate
is penalised for that and1/3rd marks will be deducted.

Directions (Q. Nos. 1-13) Read the following five passages and answer the questions that follow
each passage. Your answers to these questions should be based on the passages only. KNOW THE TREND

B Passage 1
‘‘Rural development implies both the economic betterment of people as well as greater social
transformation. In order to provide the rural people with better prospects for economic development,
increased participation of people in the rural development programmes, decentralisation of planning,
better enforcement of land reforms and greater access to credit are envisaged.’’
Initially, main thrust for development was laid on agriculture, industry, communication, education,
health and allied sectors but later on it was realised that accelerated development can be provided only
if governmental efforts are adequately supplemented by direct and indirect involvement of people at
the grass root level. Keeping in view the needs and aspirations of the local people, Panchayati Raj
Stage 1

Institutions have been involved in the programme implementation and these institutions constitute
the core of decentralised development of planning and its implementations.
60 Practice Set 3

1. In the light of the passage above which one of the following statements is the most convincing
explanation?
(a) Rural development means a larger transformation of people which is possible with the help of
Panchayati Raj Institutions.
(b) India’s villages are already developed they do not need any help.
(c) Rural development includes enforcement of land reforms in an effective manner and easy access
to credit, and involvement of rural people at grass root level in all development activities.
(d) Role of Panchayati Raj Institutions in the overall economic development of rural areas is very
crucial and much needed.

B Passage 2
Deontological pacifism decrees that moral agents have an absolute duty to avoid aggression or waging
war against others. Duties are moral actions that are demanded in all pertinent circumstances. The
first problem for deontological pacifism is the potential collision of duties. What if force is to be used
to halt an aggressor who endangers the pacifist’s life or the life of an innocent? Regarding the pacifist’s
own life, it can be argued that he or she possesses no right of self-defence. Among such adherents are
absolute pacifists. Another example: does the duty to respect others outweigh the duty to respect
oneself? Those pacifists who admit the right to defend the self against a threat can admit the use of
restraining or disabling force and even, if the threat is deadly, the right to kill an assailant.
Deontological pacifists can claim that others’ rights to life are of a higher order duty than the duty to
intervene to save oneself. But that hinges upon a moral evaluation of the self compared to others and it
is not clear why others should be accorded a higher moral evaluation; for after all, the self is in turn one
amongst many others from a different subject’s point of view. If the pacifist argues that his life is his own
to lay down in the face of aggression, the problem intensifies when the life of another is threatened.
The pacifist who claims that he has no duty to intervene in saving others’ affairs treads a precarious
moral path here; the immediate retort is why should the moral life of the pacifist be morally more
important than the life of a threatened innocent? For the sake of his own beliefs, could the pacifist
consistently ignore the violence meted upon others? Yes, from two possible perspectives. The first is that
the ideal of pacifism retains a supremacy over all other ideals. The second is that the life of the pacifist is
morally superior to the life of the threatened innocent. Deontologists argue that certain kinds of moral
actions are good in themselves, hence deontological pacifists claim peace to be a duty to be categorically
upheld.

2. Which of the following, according to the passage, would the deontological pacifist consider
idealistic?
1. Resorting to self defence in the face of mortal danger.
2. Using a tool to save an innocent’s life.
3. Devaluing one’s life in favour of an ethical conduct.
Select the correct answer using the codes given below
(a) 1 and 2 (b) 2 and 3 (c) Only 3 (d) None of these

3. Which of the following is the author unlikely to agree with?


1. It is not incumbent on the pacifist to perform duties in all pertinent circumstances.
2. The notion that there is a potential collision of duties is non-existent.
3. Self also should be given the same moral evaluation as any other.
4. The ideal of pacifism should not gain supremacy over all other ideals.
Select the correct answer using the codes given below
(a) 1 and 2 (b) Only 2 (c) 2 and 4 (d) Only 4
Stage 1 Know The Trend 61

4. What according to the passage is not implied by ‘collision of duties’?


1. Duty to protect others from an assailant or the virtue of pacifism.
2. Duty to forfeit one’s life or the duty to respect another’s life.
3. Duty to accord a higher moral value to the lives of others rather than to oneself.
Select the correct answer using the codes given below
(a) Only 1 (b) Only 2 (c) 1 and 2 (d) Only 3

B Passage 3
Mathematician Edward Lorenz found unexpected behaviour in apparently simple equations
representing atmospheric air flows. Lorenz realised that tiny rounding errors in his analog computer
mushroomed over time, leading to erratic results. His findings marked a seminal moment in the
development of chaos theory, which, despite its name, has little to do with randomness. To understand
how unpredictability can arise from deterministic equations, which do not involve chance outcomes,
consider the non-chaotic system of two poppy seeds, placed in a round bowl. If the bowl is flipped over,
two seeds placed on top will roll away from each other. Such a system, while still not technically chaotic,
enlarges initial differences in position.
Chaotic systems, such as a machine mixing bread dough, are characterised by both attraction and
repulsion. As the dough is stretched, folded and pressed back together, any poppy seeds sprinkled in are
intermixed seemingly at random. In fact, the poppy seeds are captured by ‘strange attractors,’
staggeringly complex pathways, whose tangles appear accidental, but are in fact determined by the
system’s fundamental equations.
During the dough-kneading process, two poppy seeds positioned next to each other eventually go their
separate ways. Any early divergence or measurement error is repeatedly amplified by the mixing until
the position of any seed becomes effectively unpredictable. It is this ‘sensitive dependence on initial
conditions’ and not true randomness that generates unpredictability in chaotic systems, of which one
example may be the Earth’s weather. According to the popular interpretation of the ‘Butterfly Effect,’ a
butterfly flapping its wings causes hurricanes. A better understanding is that the butterfly causes
uncertainty about the precise state of the air.

5. The future development basis of chaos theory was provided by


(a ) the unpredictable atmospheric flow of air
(b ) mathematical equations of atmospheric flows
(c ) random results given by analog computers
(d ) mathematical findings of Edward Lorenz
6. In the example discussed in the passage, what is true about poppy seeds in bread dough, once
the dough has been thoroughly mixed?
1. They have been individually stretched and folded over, like miniature versions of the entire
dough.
2. They are scattered in random clumps throughout the dough.
3. They are accidentally caught in tangled objects called strange attractors.
4. They are in positions dictated by the underlying equations that govern the mixing process.
Select the correct answer using the codes given below
(a) Only 1 (b) Only 2 (c) Only 3 (d) Only 4
62 Practice Set 3

7. The passage mentions each of the following as an example or potential example of a chaotic or
non-chaotic system except
1. a dough-mixing machine. 2. atmospheric weather patterns.
3. fluctuating butterfly flight patterns.
Select the correct answer using the codes given below
(a) Only 1 (b) Only 2 (c) 1 and 2 (d) None of these

8. It can be inferred from the passage that which of the following pairs of items would most likely
follow typical pathways within a chaotic system?
(a ) Two particles ejected in random directions from the same decaying atomic nucleus
(b ) Two stickers affixed to a balloon that expands and contracts over and over again
(c ) Two avalanches sliding down opposite sides of the same mountain
(d ) Two baseballs placed into an active tumble dryer

B Passage 4
There are limits to the population that our environment can sustain in the long-run, particularly in view
of our desire to achieve higher standards of living. There is widespread consensus that population
stabilisation entails a holistic, comprehensive approach towards education and health care. We need to
invest in educating the girl child and improving her health and social status. In a country where credible
social safety nets are absent, it is children who form the main pillar of support to their parents in their
old age. This, coupled with high infant mortality rates, compels families to have more children. There
has to be a concerted campaign to improve the health and educational status of the entire population in
general. Regional imbalances exist.
It is necessary to integrate our population policy with our health and education policy as well as a social
empowerment policy. We have to pursue a more balanced policy of regional development so that
adequate employment opportunities are made available in the more populated regions of the country.
The approach of focused attention on women and girls, a general enhancement of educational and
health levels and balanced regional development must be the core of any population stabilisation
strategy. For far too long has the health sector been addressed through a series of vertical, disease based
programmes. The vertical and horizontal fragmentation of health programmes, coupled with a target
driven approach to population stabilisation has distanced the entire gamut of health services from the
common man. Investing in human capabilities converts human liabilities into human assets. Therefore,
even as our population policy focusses on population stabilisation, it must also focus on altering the skill
profile of our population.

9. Which of the following does the author cite as being responsible for population explosion?
1. Lack of educational and health facilities for the masses especially for women.
2. High infant mortality rate that forces the poor to beget more children out of fear.
3. Regional development policies that failed to achieve their targets.
Select the correct answer using the codes given below
(a) Only 1 (b) Only 2 (c) Only 3 (d) All of these

10. By creating new employment opportunities in less developed regions, our Government aims to
1. bring about all-round progress in areas like health, education, etc.
2. increase the participation of women in work to facilitate a better standard of living.
3. empower the poor to make better informed choices.
Select the correct answer using the codes given below
(a) Only 1 (b) 2 and 3 (c) Only 3 (d) None of these
Stage 1 Know The Trend 63

11. Apart from population stabilisation, the population policy must also focus on
1. empowerment of people so that they become assets and not liabilities.
2. ensuring a decent standard of living to everybody.
3. programmes that help better the rate of achievement of targets.
4. evolving a development strategy that focuses on elimination of poverty.
Select the correct answer using the codes given below
(a) Only 1 (b) 2 and 3 (c) 3 and 4 (d) None of these

B Passage 5
Encompassing an increasing range of economic activities such as retail, travel, tourism, food and
beverages, e-commerce has emerged as India’s new sunrise industry, set to cross in business worth $ 16
billion by the end of 2015. The greater adoption of Internet and smartphones is the biggest driver of
e-commerce in India. Internet penetration is rapidly increasing with around 300 million users in 2014.
The smartphone is steadily growing and consists of 35 per cent of the overall mobile phones market in
the country and success rate of some of the technologies is directly connected to the success of
e-commerce.
The e-commerce companies are concentrating their efforts on increasing the penetration of their mobile
apps for higher growth.
The ‘Future of e-Commerce: Uncovering Innovation’ study reveals that the digital commerce market in
India has grown steadily from $ 4.4 billion in 2010 to $ 13.6 billion in 2014 and likely to touch $ 16
billion by the end of 2015 on the back of growing Internet population and increased online shoppers.

12. With reference to the above passage, consider the following statements.
1. E-commerce has emerged as India’s new sunrise industry, set to cross business worth $ 16
billions by the end of 2015.
2. The biggest driver of e-commerce is the greater adoption of internet and smartphones.
Which of the statements given above is/are correct?
(a) Only 1 (b) Only 2 (c) Both 1 and 2 (d) Neither 1 nor 2

13. What is the logical assumption that can be made from the above passage?
(a ) Government is promoting e-commerce
(b ) E-commerce has great scope in India due to internet and smartphones
(c ) E-commerce companies 50 per cent revenue comes from mobile apps
(d ) None of the above statements is a logical assumption that can be made from the passage

14. Examine the information given below.


˜
No student is lazy. All the lazy persons are unsuccessful.
Which of the following is valid conclusion regarding the above argument?
(a ) No student is unsuccessful
(b ) No unsuccessful persons are students
(c ) Some unsuccessful persons are lazy
(d ) All the unsuccessful persons are lazy

15. In the following question, two statements numbered 1 and 2 are given. There may be cause
and effect relationship between the two statements. These two statements may be the effect of
the same cause or independent causes. These statements may be independent causes without
having any relationship. Read both the statements in each question and mark your answer.
64 Practice Set 3

Statements
1. The Reserve Bank of India has recently put restrictions on a few small banks in the country.
2. The small banks in the private and cooperative sectors in India are not in a position to
withstand the competition of the bigger ones in the public sector.
(a ) Statement 1 is the cause and Statement 2 is its effect
(b ) Statement 2 is the cause and Statement 1 is its effect
(c ) Both the Statements 1 and 2 are independent causes
(d ) Both the Statements 1 and 2 are effects of independent causes
16. Which of the following diagrams indicates the best relation among the Factory, Product and
Machinery?

(a) (b) (c) (d)

17. Examine the following arguments.


˜ Some students are engineers. Some engineers are bureaucrats. Some bureaucrats are
honest. Some honest persons are loyal.
Which of the following conclusions follow(s) the above arguments?
Conclusions
1. Some loyal persons are bureaucrats. 2. Some bureaucrats are students.
3. Some students are loyal.
(a) None (b) Both 1 and 2 (c) Both 2 and 3 (d) Both 1 and 3
18. Six squares are marked A, B, C, D, E and F and are hinged together as shown in the figure given
below.
A B
C D E
F

If they are folded to form a cube, then what would be faces adjacent to face A?
(a) B, C, D and E (b) B, C, D and F (c) C, D, E and F (d) B, C, E and F
19. A one-rupee coin is placed on a plain paper. How many coins of the same size can be placed
round it, so that each one touches the central and adjacent coins?
(a ) 4 (b ) 3 (c ) 7 (d ) 6

20. The charges of a hired car are ` 4 per km for the first 60 km and ` 5 per km for next 60 km and
` 8 for 5 km for further journey. If the balance amount left over with Suman is one-fourth of
what he paid towards the charges of the hired car for travelling 320 km, then how much
money did he have initially with him?
(a) ` 1075 (b) ` 1255 (c) ` 1540 (d) None of these

21. Two chairs and three tables cost ` 1025 and three chairs and two tables cost ` 1100. What is
the difference between the cost of one table and one chair?
(a) ` 75 (b) ` 35 (c) ` 125 (d) None of these

22. Three persons A, B and C are running on a circular track with a speed of 10 m/s, 15 m/s and
20 m/s, respectively. If they start simultaneously from point P in the same direction, after how
much time will they meet for the first time?
(a) 100 s (b) 40 s (c) 66.66 s (d) 200 s
Stage 1 Know The Trend 65

Directions (Q. Nos. 23-24) Examine the following information and answer the questions that
follow.
Five persons Amita, Babu, Chanda, Diwakar and Eshoo are travelling in a train. Amita is the
mother of Chanda who is the wife of Eshoo. Diwakar is the brother of Amita and Babu is the
husband of Amita.

23. How is Babu related to Eshoo?


(a) Father (b) Mother-in-law (c) Brother-in-law (d) Father-in-law

24. How is Amita related to Eshoo?


(a) Sister (b) Mother (c) Mother-in-law (d) Niece

25. A 40 L mixture of milk and water contains milk and water in the ratio of 3 : 2. 10 L of the
mixture is removed and replaced with pure milk and the operation is repeated once more. At
the end of the two operations, what is the ratio of milk and water in the resultant mixture?
(a) 2.5 : 1 (b ) 3 : 1 (c) 31 : 9 (d) 3.5 : 1

26. A shopkeeper offers his customers 10% discount and still makes a profit of 26%. What is the
actual cost to him of an article marked at ` 280?
(a) ` 200 (b) ` 100 (c) ` 150 (d) ` 180

27. A car driver completes a 180 km trip in 4 h. If he averages 50 km/h during the first 3 h of trip,
then what was his speed in the final hour?
(a) 44 km/h (b) 45 km/h (c) 35 km/h (d) 30 km/h

28. Roshan is taller than Hardik, who is shorter than Susheel. Niza is taller than Harry but shorter
than Hardik. Susheel is shorter than Roshan. Who is the tallest?
(a) Roshan (b) Susheel (c) Hardik (d) Harry

29. In a carromboard game competition, m boys and n girls (m > n > 1) of a school participate. In
which every student has to play exactly one game with every other student. Out of the total games
played, it was found that in 221 games, one player was a boy and the other player was a girl.
Consider the following statements.
1. The total number of students that participated in the competition, is 30.
2. The number of games in which both players were girls, is 78.
Which of the statements given above is/are correct?
(a) Only 1 (b) Only 2 (c) Both 1 and 2 (d) Neither 1 nor 2

Directions (Q. Nos. 30-38) Read the following three passages and answer the questions that
follow each passage. Your answers to these questions should be based on the passages only.

B Passage 1
If globalisation’s perils tend to be exaggerated in the ways I just discussed, they are also understated by
many who say, ‘Well, we have always had globalisation and it is no big deal.’ True, rapid integration of
the world economy occurred in the late 19th and early 20th centuries. If multinationals bother you,
then just think of the great East India Company, which virtually paved the way for the British conquest
of India and the Dutch East Indies Company. Trade grew rapidly along with European outward
expansion. But all this misses the fact that there are fundamental differences that give globalisation
today a special and at times, sharp edge.
66 Practice Set 3

First, the earlier integration of the world economy was driven more by technological developments in
transportation and communications than by policy changes. It’s true that British Prime Minister Robert
Peel repealed the Corn Laws in 1846, bringing free trade unilaterally to England in the first dramatic
move away from mercantilism. But none of these policy changes did as much to integrate the world
economy in the latter half of the century as did emerging technological revolutions in transportation by
railways and in the oceans. But today’s most dramatic change is in the degree to which governments
have intervened to reduce obstacles to the flow of trade and investments worldwide.
The novel fear today is that globalisation places limits on the freedom to discharge this critical
responsibility and so the complacent view that there is nothing new about globalisation is simply wrong.
We do need to look at the phenomenon closely, seeking to analyse and address the fears that are novel
and indeed appear to be plausible at first blush.

30. Regarding globalisation, the author is of the opinion that


1. the second wave of globalisation is not as distinct as the first one was.
2. it is nothing new.
3. it is a difficult but foolproof method of achieving prosperity.
4. the apprehensions about this phenomenon may not be entirely justified.
Select the correct answer using the codes given below
(a) 1 and 2 (b) 2 and 3 (c) Only 3 (d) Only 4

31. In the process of globalisation, the feature that is visible more now than in the last century is
1. the rapid technological change and its widespread applications.
2. the emergence of the concept of a welfare state.
3. the proactive approach being adopted by Governments in the light of inevitable globalisation.
4. the type of policy changes that will facilitate rapid desegregation, in order to protect local
markets.
Select the correct answer using the codes given below
(a) 1 and 2 (b) Only 1 (c) Only 3 (d) 2 and 4

32. The case of the Corn Laws illustrates that


1. many European countries followed the example of Great Britain.
2. the policy changes made in one country did not benefit its trading partner.
3. policy changes did have some role in ushering free trade in England.
Select the correct answer using the codes given below
(a) Only 1 (b) Only 2 (c) 1 and 2 (d) Only 3

B Passage 2
According to the reports in Economic Survey, the condition of the macro-economy is positive but it has
not glossed over the challenges. The resilience of our economy is seen in its ability to withstand two shocks
in quick succession. The ripple effects of the global economic crisis that devastated world growth, trade
and finances have persisted in the form of the European fiscal crisis. On the domestic front, the farm
sector that saw a negative growth two years back was further hit by erratic monsoons, severe drought and
unseasonal rains in two successive years. Despite this, the economy is growing at rates seen during the
pre-crisis period. The economy is projected to grow at 9% during the year 2011-12. The services sector, for
long ‘the power house of the economy’, with a more than 60% share of the GDP has started gaining
momentum. Another favourable feature is that India’s demographic dividend is yet to peak. The growing
trend in savings and investment rates should benefit from the gradual withdrawal of stimulus measures by
the government. The survey notes that once the economy operates around full capacity, it is not the saving
and investment rates that will drive growth but skill development and innovation.
Stage 1 Know The Trend 67

33. Which of the following are among the favourable indicators noted by the Economic Survey?
1. The growing trend in saving and investment.
2. India’s favourable population statistics.
3. The gain in the momentum of the service sector.
Select the correct answer using the codes given below
(a) Only 3 (b) 1 and 3 (c) 1 and 2 (d) All of these
34. According to the passage, the Economic Survey can be described most aptly by
(a ) it warns of the challenges the country will face on the economic front
(b ) it points to the positive aspects of the economy
(c ) it is cautiously optimistic, pointing to the favourable aspects without ignoring the challenges
(d ) it emphasizes the important role of foreign investments in the country

35. The strength of the Indian economy is evident from


(a ) not falling prey to the ripple effects of the global economic crisis
(b ) the growth it has posted despite global economic crisis and negative growth in the farm sector
(c ) the service sector being the power house of the economy
(d ) None of the above

B Passage 3
A sanctuary may be defined as a place where man is passive and the rest of nature active. Till quite
recently, nature had her own sanctuaries, where man either did not go at all or only as a tool-using
animal in comparatively small numbers. But now, in this machinery age, there is no place left where man
cannot go with overwhelming forces at his command. He can strangle to death all the nobler wildlife in
the world today. Tomorrow he certainly will have done so, unless he exercises due foresight and
self-control in the meantime.
There is not the slightest doubt that birds and mammals are now being killed off much faster than they
can breed. And it is always the largest and noblest forms of life that suffer most. The whales and
elephants, lions and eagles, go. The rats and flies, and all mean parasites, remain. This is inevitable in
certain cases. But it is wanton killing off that I am speaking of tonight. Civilised man begins by
destroying the very forms of wildlife he learns to appreciate most when he becomes still more civilised.
The obvious remedy is to begin conservation at an earlier stage, when it is easier and better in every way,
by enforcing laws for close seasons, game preserves, the selective protection of certain species and
sanctuaries. I have just defined a sanctuary as a place where man is passive and the rest of nature active.
But this general definition is too absolute for any special case. The mere fact that man has to protect a
sanctuary does away with his purely passive attitude. Then, he can be beneficially active by destroying
pests and parasites, like botflies or mosquitoes and by finding antidotes for diseases like the epidemic
which periodically kills off the rabbits and thus starves many of the carnivora to death. But, except in
cases where experiment has proved his intervention to be beneficial, the less he upsets the balance of
nature the better, even when he tries to be an earthly providence.

36. Consider the following statements with regard to the passage


1. Parasites have an important role to play in the regulation of populations.
2. The elimination of any species can have unpredictable effects on the balance of nature.
3. Botflies and mosquitoes have been introduced to the area by human activities.
4. Elimination of these insects would require the use of insecticides that kill a wide range of
insects.
Which of the above statement(s) will weaken the author’s argument that destroying botflies
and mosquitoes would be a beneficial action?
(a) 1 and 3 (b) 2, 3 and 4 (c) 1, 2 and 4 (d) All of these
68 Practice Set 3

37. What is the purpose of the last paragraph?


1. Sum up the main points of the author’s argument.
2. Qualify the author’s definition of an important term.
With reference to the above question, which of the statement(s) given above is/are correct?
(a) Only 1 (b) Only 2 (c) Both 1 and 2 (d) None of these

38. It can be inferred that the passage is


(a ) part of an article in a scientific journal
(b ) a speech delivered in a court of law
(c ) part of a speech delivered to an educated audience
(d ) extracted from the minutes of a nature club

39. Examine the information given below.


˜ How many employees of Company P opted for Life Insurance?
Two statements, labelled 1 and 2 are given below. You have to decide whether the data given
in the statements are sufficient for answering the item. Using the data given in the statements,
you have to choose the correct alternative.
Statements
1. 18% of the 950 officer cadre employees and 6% of the 1100 other cadre employees opted for
Life Insurance.
2. 28% of the employees in the age group of 51 to 56 and 17% of the employees in all other age
groups opted for Life Insurance.
(a ) 1 alone is sufficient while 2 alone is not sufficient to answer the question
(b ) 2 alone is sufficient while 1 alone is not sufficient to answer the question
(c ) Either 1 or 2 is sufficient to answer the question
(d ) Neither 1 nor 2 is sufficient to answer the question

40. A distance-time graph is shown below. The graph shows three boys A, B and C running for a
100 m race.
100
90 C
Distance (in metres)

80
70
60 A
50
B
40
30
20
10
0
5 10 15 20 25 30 35
Time (in minutes)

1. The race was won by C. 2. A was ahead of B upto 70 m.


3. C is overtaking B at 40 m. 4. B ran very slowly from the beginning.
Which of the above statements are correct?
(a) 1 and 2 (b) 1, 2 and 3 (c) 1, 3 and 4 (d) 2, 3 and 4
Stage 1 Know The Trend 69

41. Examine the information given below.


˜ In the city, over 75% of the people are living in slums and sub-standard houses, which is a
reflection on the housing and urban development policies of the government.
The statement given above is followed by three courses of action numbered 1, 2 and 3. You
have to assume everything in the statement to be true and on the basis of the information given
in the statement, decide which of the suggested courses of action logically follow(s) for
pursuing.
Courses of Action
1. There should be a separate department looking after housing and urban development.
2. The policies in regard to urban housing should be reviewed.
3. The policies regarding rural housing should also be reviewed so that such problems could be
avoided in rural areas.
(a) Only 1 (b) Both 1 and 2 (c) Only 2 (d) Either 2 or 3

42. A group of naughty boys is sitting in 3 rows, all facing South, such that X is in the middle row.
A is just to the right of X in the same row, B is just in front of A and C is behind X. In which
direction of B is C?
(a) North-West (b) North-East (c) South-East (d) South-West

43. A gardener increases the length of the field which is in rectangular shape by 20% and
decreased the breadth by 10%. Then, the percentage change in the area of the field will be
(a) 12% increase (b) 8% increase (c) 8% decrease (d) 10% decrease

44. Examine the following information.


˜
The education of a student at college level, not taking into account maintenance
expenses, costs four hundred rupees a year. College education is thus drawing heavily
upon the national resources of an impoverished community. So, college education should
be restricted to a brilliant few.
Two assumptions numbered 1 and 2 are given. You have to decide which of the assumption(s)
is/are implicit in the statement.
Assumptions
1. Our resources are very limited.
2. Only a few students should be admitted to the colleges.
(a) Only 1 (b) Only 2
(c) Either 1 or 2 (d) Both 1 and 2

45. Consider the following figure and answer the item that follows.

What is the total number of triangles in the given figure?


(a) 20 (b) 24 (c) 26 (d) 28
70 Practice Set 3

46. The following table shows the total number of students and percentage of boys in different
colleges.
College Percentage of Total number of
boys students
A 64 550
B 54 600
C 25 672

25% of the girls from college C left the college and took admission in college A. Then, what
will be the difference between the number of girls in colleges A and B now?
(a) 40 (b) 44 (c) 48 (d) Cannot be determined

47. Consider the following figures and examine the situation.

When number 1 is at the top, which number will be at the bottom?


(a ) 1 (b ) 2 (c ) 3 (d ) 6

48. A part of monthly expenditure of Mr X is fixed and remaining varies with the distance
travelled by Mr X. If he travels 200 km in a month, his total expenditure is ` 3300 and if he
travels 500 km in a month, his total expenditure is ` 3900. What will be the total expenditure,
if he travels 800 km in a month?
(a) ` 4200 (b) ` 4500 (c) ` 4400 (d) None of these

49. Average score of Rahul, Suman and Sanjeev is 63. Rahul’s score is 15 less than Pandey and
10 more than Suman. If Pandey scored 30 marks more than the average score of Rahul, Suman
and Sanjeev. Then, what is the sum of Suman and Sanjeev?
(a) 120 (b) 111
(c) 117 (d) Cannot be determined

50. In an office, 30% of employees are men and remains are women. Also, 60% of men are married
and 66% of the employees are married. What percentage of women are not married?
1 3
(a) 30% (b) 17 % (c) 31 % (d) 20%
7 7

Directions (Q. Nos. 51-56) Read the following two passages and answer the questions that follow
each passage. Your answers to these questions should be based on the passages only.

B Passage 1
One of the most fateful errors of our age is the belief that ‘the problem of production’ has been solved.
Things are not going as well as they ought to be going must be due to human wickedness. We must
therefore construct a political system so perfect that human wickedness disappears and everybody
behaves well. In fact, it is widely held that everybody is born good; if one turns into a criminal or an
exploiter, this is the fault of ‘the system’. Modern man talks of a battle with nature, forgetting that, if he
won the battle, he would find himself on the losing side. The illusion of unlimited powers, nourished by
technological achievements, has produced the concurrent illusion of having solved the problem of
production.
Stage 1 Know The Trend 71

The later illusion is based on the failure to distinguish between income and capital where this
distinction matters most. Every economist and businessman is familiar with the distinction and applies
it conscientiously to all economic affairs except the irreplaceable capital which man has not made but
simply found and without which he can do nothing. For larger is the capital provided by nature and not
by man and we do not even recognise it as such. This larger part is now being used up at an alarming rate
and that is why it is an absurd and suicidal error to believe and act on the belief that the problem of
production has been solved.

51. According to the author, the problem of production has not yet been solved because
1. inspite of the huge and cry the world over, man is still exploiting his resource base (i.e. nature),
to reduce the level of exploitation.
2. of man’s failure to realise that his economic activities are adversely affecting nature.
3. technology is yet to be transferred to developing countries.
4. the detrimental effect that the new technologies have when used for achieving economic goals.
Select the correct answer using the codes given below
(a) 2 and 3 (b) Only 3 (c) Only 1 (d) Only 4

52. Identify the statement that cannot be attributed to the author with regard to the control of the
systems of society.
1. Faulty systems tend to breed wickedness.
2. A perfect political system would have all good citizens.
Select the correct answer using the codes given below
(a) Only 1 (b) Only 2 (c) Both 1 and 2 (d) None of these

53. Why would man find himself on the losing side, if he were to conquer nature for economic
progress?
(a ) He is not destined to win this one-sided battle
(b ) The benefits of scientific and technological development are merely illusory
(c ) In conquering nature, he would be killing the goose that lays the golden eggs
(d ) He is handicapped by a faulty understanding of the issues related to income and capital

B Passage 2
A study based on a year-long analysis of data from an extensive mobile phone network has produced
interesting information that might be of use to epidemiologists and social scientists. The data, according
to the researchers, might shed light e.g. how diseases and information (or rumours) are transmitted
through social networks. Researchers ramped the like between a pair of phone were on the basis of the
total time spent talking to each other. ‘Strong’ links exist between members of a close social group.
‘Weak’ links tend to be more long range and join individuals from different social groups.
The researchers observed a dramatically different effect when they removed links in the network in rank
order, depending on whether they removed links starting with the strongest or with the weakest. To their
surprise, removing the strong links first had little effect on the overall structure of the network. But removing
weak links first split the network into a series of unconnected islands, with individual users linked to a small
group of other phone users.
Thus, the researchers have hypothesised that the weak links (the more tenuous connections between
individuals from different social groups) might be very important in maintaining wider social cohesion.
If you lose contact with casual acquaintances you may fragment your social circle, but if you stop talking
to your brother there might be less visible impact on the structure of your social network.
72 Practice Set 3

54. Consider the following statements with regard to the passage


1. ‘Weak’ links are more important than ‘strong’ links.
2. Some people believe that phone-network patterns could be useful to social scientists.
3. Links between family members would be disrupted by terminating ‘weak’ links.
4. The ‘strong’ links are between geographically close individuals.
The passage supports which of the following conclusion(s)?
(a) 2 and 4 (b) Only 2 (c) Only 3 (d) 1 and 2
55. Which of the following can be inferred from the passage?
(a) Information transmission through phone networks is essentially the same as information
transmission through face-to-face contact
(b) The phone-network studied had the same number of users throughout the study
(c) The phone users were unaware of the study
(d) The researchers had not anticipated the specific effects of removing weak links

56. Consider the following statements regarding the purpose of the last sentence of the passage
1. To make the ideas more specific to improve the reader’s understanding.
2. To provide a practical illustration of the meaning of a strong link.
3. To generalise the argument to make it more appealing.
4. To concretise an argument about the usefulness of the current research.
Which of the above statement(s) convey the author’s intentions?
(a) Only 1 (b) 2 and 3 (c) 1, 2 and 3 (d) 1 and 2

Directions (Q. Nos. 57-60) Examine the information and answer the questions that follow.
Eight persons— A, B, C, D, E, F, G and H are seated in two rows with four persons in each row.
Each person in any row faces exactly one person in the other row. Also,
Â
A is to the immediate left of the person, who is opposite to F.
Â
E is in the same row as B and one of them is opposite to G, who is not at the end.
Â
There is only one person to the left of H, who is not in the same row as C.
Â
D is not opposite to E.
Â
E is between C and A.

57. Who are the other three persons in the same row as H?
(a) A, G and F (b) D, G and F
(c) E, D and G (d) D, A and C

58. Who among the following are diagonally opposite to each other?
(a) A and H (b) F and B
(c) D and B (d) E and G

59. Who is the person between H and D?


(a ) E (b ) F (c ) A (d ) G

60. If the positions of E and B are interchanged, then who is opposite to E?


(a ) A (b ) G (c ) F (d ) B

61. After two successive discounts, a jeans, whose marked price is ` 300 is sold for ` 216. If the
second discount is of 10%, then the first discount (in %) is
(a) 25 (b) 15 (c) 30 (d) 20
Stage 1 Know The Trend 73

62. In the following question, there is some relationship based on a certain rule between the
numerals given in each figure. Find the rule in each case and the correct alternative from
among the four alternatives given under it to fill in the vacant place.
5 3 6
2 3 7

4 548 2 2 694 2 4 ? 2

2 3 1
1 2 1
Fig (i) Fig (ii) Fig (iii)
(a) 678 (b) 769 (c) 824 (d) 937

63. Examine the following arguments.


˜ Some students are laborious. Some engineers are students. No engineer is an officer.
Which of the following conclusions follow(s) the above arguments?
Conclusions
1. Some laborious are engineers.
2. Some students are officers.
3. Some officers are not students.
(a) Both 1 and 4 (b) Both 2 and 3
(c) Only 3 (d) None of these

64. Examine the information given below.


˜ A studies in which of the schools B, C, D, E and F?
Two statements, labelled 1 and 2 are given below. You have to decide whether the data given
in the statements are sufficient for answering the item. Using the data given in the statements,
you have to choose the correct alternative.
Statements
1. A does not study in the same school as either R or J.
2. R and J study in schools D and F, respectively.
(a ) 1 alone is sufficient while 2 alone is not sufficient to answer the question
(b ) 2 alone is sufficient while 1 alone is not sufficient to answer the question
(c ) Either 1 or 2 is sufficient to answer the question
(d ) Neither 1 nor 2 is sufficient to answer the question

65. If Neha says, ‘Amruta’s father Raj is the only son of father-in-law Mahesh’, then how Bindu,
who is the sister of Amruta is related to Mahesh?
(a) Daughter (b) Wife (c) Daughter-in-law (d) Granddaughter

Directions (Q. Nos. 66-69) The following questions are based on the pie-charts given below.
Percentagewise distribution of students studying in Arts and Commerce in seven different
institutions (A, B, C, D, E, F and G).
Total member of students studying Arts = 3800

G A
12% 15% B
F
13% 8%
E C
14% D 17%
21%
74 Practice Set 3

Total number of students studying Commerce = 4200


11% 12%
F G A B
13% 17%
E C
18% D 15%
14%

66. What is the total number of students studying Arts in institutions A and G together?
(a) 1026 (b) 1126 (c) 1226 (d) 1206

67. How many students from institute B study Arts and Commerce?
(a) 1180 (b) 1108 (c) 1018 (d) 1208

68. The ratio of the number of students studying Arts in institute E to that studying Commerce in
institute D is
(a) 12:17 (b) 12:7 (c) 19:21 (d) 17:19

69. The ratio of the number of students studying Arts to that studying Commerce in institute E is
(a) 27:14 (b) 19:27 (c) 19:16 (d) 19:28

70. A box contains 2 white balls, 3 black balls and 4 red balls. In how many ways 3 ball be drawn
from the box, if atleast one black ball is to be included in the drawn?
(a) 32 (b) 48 (c) 64 (d) 96

71. From GBPS, a renowned public school of Delhi, 300 students wrote the board examinations,
out of which, 45% got distinction. If 40% of the girls got distinction, the number of boys, who
wrote the examination, is (here, the number of boys, who got distinction is 35 more than the
number of girls who got distinction)
(a) 120 (b) 150 (c) 175 (d) 125

Directions (Q. Nos. 72) Read the following information carefully and answer the question
follows.
Four young men Anil, Mukesh, Piyush and Yogesh are lovingly called Munna, Babu, Prince and
Papu by everyone. They are married to Madhu, Isha, Jyoti and Arti.
(i) Arti and Madhu are not married to Piyush or Anil nor is their husband called Babu.
(ii) Babu is not married to Isha and his name is not Piyush.
(iii) Isha is not married to Munna.
(iv) Mukesh is neither Munna nor Prince nor is married to Madhu.

72. Which of the following pairs of husband-wife is not correct?


(a) Munna-Madhu (b) Babu-Jyoti
(c) Papu-Jyoti (d) None of these

73. 40% of the students in a college play basketball, 34% of the students play tennis and the
number of students who play both the games is 234. The number of students who neither play
basketball nor tennis is 52%. Find the student population in the college.
(a) 800 (b) 1050
(c) 900 (d) 850
Stage 1 Know The Trend 75

Directions (Q. Nos. 74-76) The following questions are based on two passages in English to test the
comprehension of English language and therefore, these questions do not have Hindi version. Read
each passage and answer the questions that follow.

B Passage 1
Surveillance by law enforcement agencies constitutes a breach of a citizen’s Fundamental Rights of
privacy and the Freedom of Speech and Expression. It must, therefore be justified against compelling
arguments against violations of civil rights. ‘Right to Privacy’ in India has long been considered too
broad and inovalistic to be defined judicially. The judiciary, though, has been careful enough not to
assign an unbound interpretation to it. It has recognised that the breach of privacy has to be balanced
against a compelling public interest and has to be decided judiciously.

74. Which among the following is the most rational and critical inference that can be made from
the above passage?
(a) The Right to Privacy is a natural right duty supported by statutes.
(b) There have been instances of surveillance by law enforcing agencies resulting in breach of a
citizens fundamental rights.
(c) The judiciary has come to individual as well as government’s rescue by pronouncing a very
judicial use of this Right to Privacy. An individual has all the Right to Privacy but at the same
time, for the larger interest of nation the government has to adhere to surveillance of certain
suspect individuals. A balanced and judicious use of this right is the only solution.
(d) The comprehensive idea of national interest implies the breach of individual’s right to freedom
of speech and expression and violation of ‘Right to Privacy’ is justified.

B Passage 2
In India, extra-judicial killings by the police or the security forces are called ‘encounter killings’, meaning
that the killing occurred during an armed encounter between the police or security forces and the victim.
The killing by the state forces is most often declared to be defensive, cases of attempted murder and
other related offences are registered against the victims and the cases closed without further
investigation since criminal cases come to an end upon the death of the accused. Despite being
‘unnatural deaths’ and the victim having being killed, no investigation ensues to determine whether the
death was in fact in an actual encounter, nor whether the use and the extent of use, of force was justified.
This is an acknowledged strategy of the state for eliminating certain kinds of opposition to the state and
the established order. In Andhra Pradesh, for instance, the naxalites have been the targets; in Punjab, it
was the militant; in Mumbai, it is those who are alleged to be part of the underworld.

75. Accrding to the passage,


1. the armed and security forces are rarely held accountable for the commission of extrajudicial
killings.
2. extra-judicial killings are also known as encounter killings in India.
3. encounter killings have been particularly prevalent in volatile regions such as Jammu and
Kashmir, Panjab and crime-infested areas of Mumbai.
Which of the statements given above is/are in correct?
(a) Only 1 (b) 1 and 2 (c) 1, 2 and 3 (d) All of these
76 Practice Set 3

76. Which of the following explains the status of extra judicial killings in India?
1. Encounter killings also have been employed in efforts by Indian police and security forces to
quash the Maoist/Naxalite armed rebellion.
2. The fabrication of evidence appears to be a regular feature of fake encounter killings.
Select the correct answer using the codes given below.
(a) Only 1 (b) Only 2 (c) Both 1 and 2 (d) Neither 1 or 2

Directions (Q. Nos. 77-80) Given below are four questions. Each question describes a situation
and is followed by four possible responses. Indicate the response you find most appropriate. Choose
only one response for each question. The responses will be evaluated based on the level of
appropriateness for the given situation. Please attempt all questions. There is no penalty for wrong
answers for these four questions.
77. You are a high-ranking official in a government department. One of your colleagues has been
illegally using his official vehicle for personal purposes though the vehicle has been allotted to
him for official work only. You also come to know that he regularly submits fake bills to the
office in order to earn some extra money. What would you do?
(a) Talk to your colleague and advise him to mend his ways
(b) Report the issues to his immediate boss
(c) Share the matter with your colleagues and ask them to spread the word about your colleague’s
improper activities
(d) Pay no attention to the matter since it does not affect you in any manner

78. You are the General Manager of District Industry Centre. A person has decided to start a new
business and he has come to meet you with a proposal in the same context. He has already met
your subordinate and enquired about the existing scenario before designing the proposal. As a
GM, what will you do?
(a) Know the financial capability of the person to meet the initial expenditure
(b) First see his proposal and its viability
(c) Ask him to visit some industrial units similar to one that he has proposed
(d) Advise him to fill up the forms and submit the proposal and confirmation from a bank by
which he intends to finance his project
79. You are an Income Tax Commissioner. You come to know about tax evasion by a very reputed
firm. You got this information from one of your very reliable officers. You know that the firm
has a very clean image but the officer is sure about the tax evasion. He is a very honest officer.
You will
(a) Conduct a raid immediately
(b) Consult your predecessor who worked in the area for long as his advice could be valuable in
implementing an effective decision
(c) Send a notice to the firm
(d) Call a meeting of some reliable officers and gather all the inputs
80. You are a traffic police officer and are on night duty. You have instructed a car to stop as it was
passing a red light. As you step over to the car, you recognise the driver to be an MLA. When
you approach him, he tells you that he misjudged the light and he sincerely apologises for it.
What would you do?
(a) Tell the MLA that you know who he is but you are just doing your duty
(b) Tell the MLA that you will excuse him from this traffic violation but that he should be careful
in future
(c) Tell him that he can go as he is the MLA of the city
(d) Salute the MLA and let him go
Stage 1 Know The Trend

Answer with Explanations


1. (d) Rural development means lives of rural people get 9. (d) Options (a) and (b) have been talked about in the
changed due to land reforms, more educational facilities, passage. It also talks of regional imbalances and the need
credit availability at easy terms, health and sanitation. All for a more balanced policy of regional development.
this is possible through decentralisation of powers and in Hence, we can summarise that regional development
rural areas Panchayati Raj System is the best example of policies have failed to achieve their targets. Thus, all of the
an instrument responsible for economic development of choices are responsible for population explosion.
rural masses.
10. (a) The passage talks of integrating population policy with
2. (c) Deontological pacifism stands against violence or health and education policies. Option (a) is the primary
aggression of any kind and supports non-intervention in aim of the Government.
the face of even a mortal danger irrespective of who the
11. (a) The last few lines of the passage indicate that the
victim is. So, Statement 1 is ruled out as a covert violent
population policy should also focus on empowerment of
approach is against its principles. Statement 2 too is ruled
people so that they become assets and not liabilities.
out for the same reason. Statement 3 advocates peace in
the face of aggression and hence is in concord with its 12. (c) Option (c) is correct as both statements 1 and 2 are true.
idealism. 13. (b) India is a vast country. Penetration of internet and
3. (b) The author is not in agreement with the ideas of pacifism smartphones has really helped e-commerce industry in
since he foresees problems if these are followed. The India to register high growth.
author agrees with Statement 1 which implies that there 14. (c) ‘No student is lazy’ can be represented as
could be situations where use of force becomes pertinent
and which go against the pacifist’s ideals. According to
Statement 2, these ideals cause no collision of duties. The Student Lazy
passage cites various examples to illustrate otherwise.
Hence, the author is unlikely to agree with it. Statements 3
and 4 are situations that go against pacifism. ‘All the lazy persons are unsuccessful’ can be
represented as
4. (d) While Statements 1 and 2 are supported by the passage,
but Statement 3 is not.
5. (d) The first paragraph of the passage makes it specifically Lazy Unsuccessful
clear that Edward Lorenz’s findings led to the
developmont of chaos theory.
6. (c) Only 3, from second passage it is clear that the poppy On combining the two, we get
seeds are captured by ‘strange attractors’ staggeringly
complex pathways, whose tangle appear accidental.
Student
7. (d) In the last paragraph, the passage states that the Earth’s
weather may be an example of a chaotic system. So,
Statement 2 can be left out. Poppy seeds placed on an Unsuccessful
upside-down bowl are described in the second paragraph Lazy
as an example of a non-chaotic system that creates (i)
divergence. So, Statement 3 is not the correct answer. Or
Butterfly flight patterns are nowhere mentioned as a
system. According to the last paragraph, the ‘Butterfly
Effect’ is caused by the flapping of a single butterfly’s Student
wings to potentially affect atmospheric systems. So,
option (d) is the right answer.
Unsuccessful
8. (d) Option (d) two baseballs placed into an active tumble Lazy
dryer are analogous to two poppy seeds placed in ‘bread (ii)
dough being mixed by a machine’: parts of the system are
separated, intermingled and brought back together again In Fig. (ii), ‘some students are unsuccessful’ and ‘some
in perfectly regular, though complex, ways. unsuccessful persons are students’. So, options (a) and
(b) are not valid. Also, only ‘some unsuccessful persons
The pathways of the two baseballs will diverge and
are lazy’ and not ‘all’. So, option (c) is valid and option (d)
converge repeatedly, as in any other chaotic system.
is not valid.
80 Practice Set 3

15. (b) The inability of the small banks to compete with the bigger Now, put the value of y in Eq. (i), we get
ones shall not ensure security and good service to the 2 x + 3 ´ 175= 1025
customers, which is an essential concomitant that has to be Þ 2 x = 1025 - 525
looked into by the Reserve Bank. Statement 1 seems to be a
Þ 2 x = 500 Þ x = 250
remedial step for the same.
Then, x - y= 75
16. (d) Option (d) explains the relation among factory, product and Hence, difference between cost of one table and one
machinery. chair = `75
22. (d) Let t denotes the time.
So, t = LCM (t AB , t BC ) [relative speed]
Product 1000 1000
\ t AB = = 200 s, t BC = = 200 s
Machinery 15 - 10 20 - 15
Factory So, after 200 s they will meet first time.

17. (a) Here, A® All Solutions (Q. Nos. 23-24)


I® Some From the given information the blood relation is as follows.
E® No Amita(–) Brother
Babu(+) Diwakar(+)
Statement 1 of argument + Statement 2 of argument gives no
Mother
conclusion [ QI + I = no conclusion].
(+) Eshoo Chanda(–)
Similarly, Statement 2 + Statement 3 gives no conclusion. Husband
[Q I + I = no conclusion]. Also, Statement 3 + Statement 4
Here, (+ ) = male, (- =
) female
gives no conclusion due to same reason. Hence, no
conclusion follows. and (« =) married couples

18. (d) According to the question, 23. (d) From above diagram, it is clear that Babu is
C Û E Opposite face father-in-law of Eshoo.
A Û D Opposite face 24. (c) From above diagram, it is clear that Amita is
B Û F Opposite face mother-in-law of Eshoo.
Hence, B, C, E and F would be the faces adjacent to face A. 25. (c) Original quantity of milk in the mixture = 40 ´ 3/5 = 24 L
19. (d) Original quantity of water in the mixture = 40 ´ 2/5
2
1 3 =16 L
After first operation, milk = (24 – 6+ 10) = 28 L
6 4
5 water = (16 – 4) = 12 L
[Q during first operation, (10 ´ 3/ 5= ) 6 L milk and
From above figure, it is clear that we can place only six coins (10 ´ 2 / 5= ) 4 L water is removed and 10 L milk is
where one coin is in the centre as shown in the above added)
diagram. After second operation, Milk = (28 – 7+ 10) = 31 L
8 Water = (12 – 3) = 9 L
20. (a) Hire charges = (4 ´ 60+) (5´ 60 +) ´ -
(320 120)
5 [during second operation, 7 L milk and 3 L water is
= 240 + 300 + 320 = ` 860 removed and 10 L milk is added]
Let Suman had initially be ` x. Hence, final ratio of milk and water = 31: 9
1
Then, x - 860= ´ 860 Þ x = ` 1075 26. (a) Marked price of an article = ` 280
4
Marked price ´ Discount percentage
Discount =
21. (a) Let cost of each chair be ` x 100
280 ´ 10
and cost of each table be ` y. = = ` 28
According to the question, 100
2 x + 3 y = 1025 ...(i) \ Salling price = Marked price - Discount
3x + 2 y = 1100 ...(ii) = 280 - 28 = ` 252
Multiplying Eq. (i) by 3, Eq. (ii) by 2 and then subtracting Eq. Now, SP = ` 252 and profit = 26%
100
(ii) from Eq. (i), we get \ CP = ´ SP
100 + Profit %
6x + 9 y = 3075
100 100
- 6x ± 4 y = - 2200 = ´ 252 = ´ 252 = `200
100 + 26 126
5 y = 875
Þ y = 175 Hence, the actual cost of the article is ` 200.
Stage 1 Know The Trend 81

27. (d) Given, total distance = 180 km 39. (a) From Statement 1,
Distance covered in first 3 h = 50 ´ 3 = 150 km Total number of employees who opted for Life Insurance
Distance covered in last hour = 180 - 150 = 30 km = (18% of 950 officer employees) + (6% of the 1100 other
Distance covered in last hour cadre employees)
\ Speed in last hour = 18 6 17100
Time = ´ 950+ ´ 1100 = + 66 = 171 + 66 = 237
100 100 100
30
= = 30 km/h
1 From Statement 2,
Number of employees who opted for Life Insurance =( 28% of
28. (a) As per the given information, final ranking of height in employees in the age group of 51 to 56) + (17% of employees
descending order is in all other age groups opted for Life Insurance).
Roshan > Susheel > Hardik > Niza > Harry It is clear from the Statement 2 that we cannot get the required
Therefore, the tallest person is Roshan. answer until and unless the number of employees in age
29. (c) From Statements 1 and 2, group of 51 to 56 and other age group is known.
mn = 221 = 17 ´ 13 So, Statement 1 alone is sufficient while Statement 2 alone is
not sufficient to answer the question.
\ m = 17 and n = 13
13 ´ 12´ 11! 40. (b) Boy C had covered the maximum distance in minimum time.
\ Total number of games = 13
C2 = = 78
11! 2 ! Also, he reached the 100 m mark in the shortest time. So, race
was won by C. A was ahead of B upto 70 m and then B
Hence, the total number of students that participated
overtake A at 70 m. Also, C is overtaking B at 40 m, where boy
in the competition is 30 i.e. (17 + 13) and the number
C crosses the boy B. So, Statements 1, 2 and 3 are correct.
of games in which both players were girls, is 78.
Boy B was running faster than C and he covered more
30. (d) The opening sentence of the passage says ‘the perils distance than C, upto 40 m. So, it is not true that B ran very
tend to be exaggerated’, which means that regarding slowly from the beginning. Hence, the answer is option (b).
globalisation, the author is of the opinion that the
apprehensions about this phenomenon may not be 41. (b) The statement talks of housing conditions in urban areas only.
entirely justified. So, 3 does not follow. Also, to improve the deteriorating housing
conditions, the urban housing policies need to be studied and
31. (c) Refer to the lines ‘…..most dramatic change is in the lacunae removed by a team of efficient personnel deployed
degree to which governments have intervened to for the same. So, both 1 and 2 follow.
reduce obstacles to the flow of trade…..’
42. (b) According to the given information, sitting arrangement of the
32. (d) Refer to the lines ‘…….. repealed the Corn laws ….. boys will be as follow.
bringing free trade …..’.
N
C
33. (d) All the three statements are mentioned in eighth, ninth NW NE
and tenth sentences of the passage.
A
Facing W
34. (c) This option is clearly mentioned in the first sentence of E
South X
the passage according to the passage, the Economic
Survey can be described as optimistic, pointing to the SW SE
favourable aspects without ignoring the challenges. B S
35. (a) The second, third and fourth sentences of the From above figure, it is clear that C is in the North-East
passage bring out this option as the correct one. direction of B.

36. (c) We have to find the statements that weaken the idea 43. (b) Let length of field be x and breadth be y.
that destroying botflies and mosquitoes is beneficial. Area = xy
Statements 1, 2 and 4 give reasons why destroying 120 6x
Now, length after 20% increase = x ´ =
these insects might create more problems. 100 5
90 9y
37. (b) The last paragraph is giving details of the modifying or Breadth after 10% decrease = y ´ =
qualifying that the author has done in the third 100 10
54xy
paragraph of the earlier definition of a sanctuary. So, New area =
only Statement 2 is correct. 50
Now, increase in area = xy æç - 1ö÷ =
54 4xy
38. (c) From the words ‘I am speaking of tonight’ (middle of è 50 ø 50
the second paragraph) we can infer that the words
4xy
were delivered orally and not during the daytime. So,
the only possible answer is option (c). \ Percentage change = 50 ´ 100 = 8%
xy
82 Practice Set 3

44. (d) The use of the words ‘impoverished community’ in the 49. (b) Total score of Rahul, Suman and Sanjeev = 63 ´ 3= 189
statement makes 1 implicit while the phrase ‘college
Pandey’s score = 63 + 30 = 93
education should be restricted to a brilliant few’ makes 2
implicit. Rahul’s score is 15 less than Pandey’s score.
\ Rahul’s score = 93 - 15= 78
45. (d) B C
A Hence, sum of Suman and Sanjeev = 189 - 78= 111
K O
U S 50. (c) Let total number of employees be 100.
I
L P
D N Men = 30% = 30
J R
H T Married men = 30 ´ 60%= 18
M Q
E Women employees = 70
G F
Married women = 66 - 18= 48
Total number of triangles = AKU, AIR, AGR, ABR, ACR
Unmarried women = 70 - 48= 22
ACE, AGE, BCR, COS, CDR, CER, DER, EQT, EFR, EGR,
22 3
FGR, GHM, GIR, HJU, JKM, KLR, KMR, LMR, NST, NOQ, \ Percentage of unmarried women = ´ 100 = 31 %
70 7
OQR, OPR, PQR = 28
(100 - 64) Alternate Method
46. (c) Number of girls in college A = ´ 550 100
100
36 ´ 55
= = 198 M W
10
(100 - 54)
Number of girls in college B = ´ 600
100 30 70
= 46 ´ 6 = 276 Unmarried Married
(100 - 25) Unmarried Married
Number of girls in college C = ´ 672
100 12 18 22 48
75
= ´ 672 = 504 22 3
100 \ Required percentage = ´ 100= 31 %
70 7
25 504
Now, 25% of girls of college C = 504 ´ = = 126 Hence, option (c) is correct.
100 4
So, 126 girls took admission in college A. 51. (d) According to the author,the problem of production has not
yet been solved because of illusion of unlimited
\ Total number of girls in college A = 198 + 126 = 324
powers,nourished by technological achievements. It has
Now, difference between the number of girls in college A produced the concurrent illusion of having solved the
and college B = 324 - 276 = 48 problem.
47. (d) From the given condition of dice, we can say that number 52. (a) Refer to line of the passage ‘human wickedness
2 is common in both. disappears and everybody behaves well. In fact, it is
\ Opposite side of number 2 = Number 4 widely held that everybody is born good’, so, option (a)
Opposite side of number 5 = Number 3 can be attributed to a ‘widely held’ belief not to the author.
Opposite side of number 1= Number 6 53. (c) Refer to passage, we are using up the ‘irreplaceable
Hence, when number 1 is at the top, number 6 will be at capital’ and option (c) shows an irreplaceable loss.
the bottom.
54. (b) Statement 2 is clearly implied in the first paragraph.
48. (b) Let fixed expenditure be ` x Statement 3 can be eliminated because weak links are
and variable be y per km. less likely to be between family members. Also, since
geographical distances are not mentioned anywhere in
According to the question,
the passage, Statement 4 is incorrect.
x + 200 y = 3300 ...(i)
x + 500 y = 3900 ...(ii) 55. (d) There is no mention anywhere in the passage that the
number of users was constant or that the users were
On solving Eqs. (i) and (ii), we get
aware that their network was being monitored; hence
300 y = 600 options (b) and (c) are incorrect. Also, we can eliminate
Þ y=2 option (a) because the words ‘essentially the same’ in it
Now, x = 3300 - 400 cannot be justified.
= 2900 56. (d) The author is making a point clear for the reader, so
\ Total expenditure on travelled 800 km Statement 1 is the correct choice. Statement 2 is too
= 2900 + 2 ´ 800 narrow as the author is doing more than just giving an
= ` 4500 illustration of a ‘strong’ link.
Stage 1 Know The Trend 83

Solutions (Q. Nos. 57-60) 66. (a) Total number of students studying Arts in institutions A
From the given information, arrangement can be drawn as and G together
given below. (15 + 12 )
= 3800 ´ = 38 ´ 27= 1026
D G H F 100
· · · ·
· · · · 67. (c) Number of students studying Arts in institute B
C E A B 8
= 3800 ´ = 304
57. (b) The other three persons in the same row as H are D, G and 100
F. Number of students studying Commerce in institute B
17
58. (c) D and B are diagonally opposite to each other. = 4200 ´ = 714
100
59. (d) The person between H and D is G. \ Total number of students = 304 + 714 = 1018
60. (c) If the positions of E and B are interchanged, then the 68. (c) Number of students studying Arts in institute E
person opposite to E is F. 14
= ´ 3800 = 532
61. (d) Let the first discount be of x %. 100
Discount = 300 - 216= ` 84 Number of students studying Commerce in institute D
14
Now, 10% of 300 + x% of (300 - 10% of 300) = 84 = 4200 ´ = 588
100
x
Þ 30 + ´ 270= 84Þ = x 20% 532
100 \ Required ratio = = 19 : 21
588
62. (a) In fig (i), 5 ´ 1= 5
69. (b) Number of students studying Arts in institute E
2 ´ 2= 4 Þ (548)
14
4 ´ 2= 8 = 3800 ´ = 532
100
In fig (ii), 3 ´ 2= 6 Number of students studying Commerce in institute E
3 ´ 3= 9 Þ (694) 18
= 4200 ´ = 756
2 ´ 2= 4 100
Similarly, in fig (iii) 6 ´ 1= 6 \ Required ratio = 532 : 756 = 19 : 27
7 ´ 1= 7 Þ (678) 70. (c) We have the various combinations as given below.
4 ´ 2= 8 (1 black and 2 non-black) Or (2 black and 1 non-black) Or
63. (d) Statement 2 of the argument can be converted to ‘Some (3 black)
students are engineers.’ \ Required number of ways
This plus Statement 3 gives no conclusion ( I + E = 0). No, = ( 3 C1 ´ C2 +
6
) ( 3 C 2´ C+1 )
6
(3 C 3 )
other conclusion is possible. = 45 + 18 + 1= 64
64. (d) As given in Statements 1 and 2, R studies in school D and 71. (c) QNumber of students who got distinction
J studies in school F. So, A does not study in school D or
= 45% of 300
school F. Thus, A studies in anyone of the schools B, C or
E. So, neither Statement 1 nor Statement 2 is sufficient to = 135
answer the question. Let the number of girls who got distinction be x.
Then, the number of boys who got distinction
65. (d) According to the given information,
= (x + 35)
(+) Mahesh
\ x + (x + 35) = 135
Father-in-law
Father Þ 2 x = 100
(+) Raj Neha (–) Granddaughter Þ x = 50
Couple
Again, let the number of girls who wrote the exam be n,
Father
then
n = æç
(–) Amruta Bindu (–) 50 ö
÷ ´ 100= 125
Sister è 40 ø
From above relation tree, we can say that Bindu is the So, the number of boys who wrote the exam
granddaughter of Mahesh. = 300 - 125= 175
84 Practice Set 3

72. (c) On the basis of given information, we can summarise the data in 77. (a) Since the person involved is your colleague, it is
tabular form as given below. reasonable for you to talk to him and give
meaningful advice. If he further violates the rules,
Nickname Wife
Name you should report the issues to his immediate
Munna Babu Prince Papu Madhu Isha Jyoti Arti
boss. Option (c) may give your colleague a bad
Anil û ü û û û û ü û name in the office. You should follow proper
Mukesh û û û ü û û û ü channels instead of randomly spreading negative
Piyush û û ü û û ü û û information about someone. Option (d) does not
Yogesh ü û û û ü û û û take care of the issue at all shows your
irresponsible attitude.
Hence, it is clear from the above table that, Pappoo (Mukesh) is
married to Arti. 78. (d) As the person has already met the subordinate and
designed the proposal after considering all the
73. (c) Let the total number of students be N. scenarios, so he is now in need of guidance and
40N 34N help. As a GM, you should ask him to fulfil the
Again, let n( A) = , n(B) =
100 100 procedural formalities and help him in submitting
52 N the form, so that he can start his business as soon
and n( A È B=)
100 as possible.
Q n( A È B=) - 1 nÈ ( A B) 79. (d) Arranging a meeting will give you a chance to
52 N 48N
= 1- = know about all facts and information about the
100 100 firm as well as about tax evasion. The various
Q n( A Ç B=) n( A+) n(B- ) nÈ ( A B) officers who work in this field will suggest some
40N 34N 48N points about the issue.
Þ 234 = + -
100 100 100 80. (a) As an officer, it is unethical to give special favours
Þ 234 ´ 100= 26N to anyone (whether MLA or MP) who violates the
Þ N = 900 law. If you let him go without proper investigation,
then it is a form of misconduct. So, option (a) is the
74. (c) The judiciary has been wise enough to state that in the compelling
right choice.
public interest, ‘Right to Privacy’ can be modified. But judiciary
has recognised the importance of ‘Right to Privacy’ as a Option (b) shows that you are letting the MLA go
fundamental right of any individual. after giving a warning because he is an MLA. You
should always remember that any person who
75. (d) All the 3 statements can be identified indirectly in the passage violates the law is just a culprit and you should
from lines 3 to 5 and last 2 lines. treat every person equally before law.
76. (c) Last 3 lines of the passage explains where and why extra judicial Option (c) is unethical and against the
killings are operated in India and lines 8, 9 explain, there are responsibilities of your duty as a traffic police
occurrence of fake encounters or encounters that are subjected to officer. Option (d) shows your cowardice and
doubt in India.. selfish attitude.
CSAT
C i v i l S e r v i c e s A p t i t u d e Te s t
Paper 2

Practice Set 4
Time : 2 hrs MM : 200

Instructions
1. There are 80 questions in this paper.
2. The answer of any question you are thinking that more than two answers are true, then you must choose the nearest one.
There is only one answer to be selected by you.
Penalty for wrong answer
3. There are four alternative answers in every question. When you select a wrong answer, then 1/3rd mark of that question is
deducted in your total marks.
4. If any candidate gives more than one answer and one of them is true but it is treated as a wrong answer and the candidate
is penalised for that and1/3rd marks will be deducted.

Directions (Q. Nos. 1-10) Read the following five passages and answer the questions that
follow each passage. Your answers to these questions should be based on the passages only.

TREND
B Passage 1
In the approach to the Smart Cities Mission, the objective is to promote cities that provide core
infrastructure and give a decent quality of life to its citizens, a clean and sustainable environment and
application of ‘Smart’ solutions. The focus is on sustainable and inclusive development and the idea is
THE

to look at compact areas, create a replicable model which will act like a light house to other aspiring
cities. The Smart Cities Mission of the government is a bold, new initiative. It is meant to set examples
that can be replicated both within and outside the Smart City, catalysing the creation of similar Smart
KNOW

Cities in various regions and parts of the country.


The purpose of the Smart Cities Mission is to drive economic growth and improve the quality of life of
people by enabling local area development and harnessing technology, especially technology that leads
to Smart outcomes. Area-based development will transform existing areas (retrofit and redevelop),
Stage 1

including slums, into better planned ones, thereby improving live ability of the whole city.
86 Practice Set 4

New areas (greenfield) will be developed around cities in order to accommodate the expanding
population in urban areas. Application of Smart Solutions will enable cities to use technology,
information and data to improve infrastructure and services. Comprehensive development in this way
will improve quality of life, create employment and enhance incomes for all, especially the poor and the
disadvantaged, leading to inclusive cities.

1. According to the passage which of the following statements is/are true?


1. There is a great and urgent need of a comprehensible development of physical, institutional,
social and economic infrastructure. Development of Smart City is a step in that direction.
2. The focus of Smart City project is on sustainable and inclusive development which will
transform existing areas (retrofit and redevelop, including slums into better planned cities).
3. Smart City Mission is very ambitious and not likely to succeed in India.
Select the correct answer using the codes given below.
(a) Only 1 (b) Only 3 (c) Both 1 and 2 (d) None of these

B Passage 2
The median legal age for drinking in India is 21 years. For driving and voting, it is 18 years. The Juvenile
Justice Act (JJA), 2015, passed by both (otherwise unproductive) Houses of Parliament lowers the age
for Juveniles to be tried as adults for some ‘‘serious and heinous’’ crimes to 16 years. Is that wise?
As one UK penal reform note has it, ‘‘depriving children of their liberty can lead to long-term and costly
psychological and physical damage, whilst overcrowding and poor detention conditions threaten their
development, health and well-being. The removal of children from networks as well as from educational
or vocational opportunities at critical and formative periods in their lives can compound social and
economic disadvantage and marginalisation. Exposure to criminal influences and violent behaviour in
detention and in the worst instances, exposure to adult offenders, is likely to encourage repeat offending.’’
Neither retribution nor rehabilitation will likely be served.
In any case, the real issue with Juvenile law in India is less about the law and more about its
implementation.

2. Which is the critical inference of the given passage?


(a) Recently hastily passed Juvenile Justice Act, is not a wise decision.
(b) In India, the real issue with Juvenile law is more about its correct implementation.
(c) A lot of Indians are emotional and impulsive and not enough reasoning is behind passing of JJA
(d) A Juvenile criminal needs a different type of atmosphere which is not available in India’s reform
homes.

B Passage 3
We can define a Payment Bank in India as a type of bank which is a non-full service niche bank. A bank
licensed as a payment bank can only receive deposits and provide remittances. It cannot carry out
lending activities. Thus, payment banks can issue ATM/debit cards, but cannot issue credit cards as they
are not empowered to carry out lending activities these banks have been created to help India reach its
financial inclusion targets. This type of bank can be highly useful for migrant labourers, low income
households, small businesses, and other unorganised sector entities.
What kind of impact will payment banks and small finance banks will have on existing banking
customers? ‘‘We expect the existing digital banking customers to try services of payments banks that
will kick off operations. Some of these banks, which are backed by deep pockets, can create disruptive
offerings in the Indian market. We look forward to innovation and high competition.
Stage 1 Know The Trend 87

3. Which among the following is the most logical corollary to the above passage?
(a) Government’s decision to introduce new type of payment bank is going to help large number of
people belonging to unorganised sectors.
(b) Introduction of payment banks will not help in achieving the targets of financial inclusion of
larger number marginalised people.
(c) The payment banks will create disruptive offerings in the Indian markets.
(d) Introduction of payment bank will help the rich investors only.

B Passage 4
New discoveries frequently undermine accepted findings and give rise to new theories in Archaeology, like
Physical Sciences. This trend can be seen in the reaction to the recent discovery of a set of 3.3-million year old
fossils in Ethiopia, the remains of the earliest well-preserved child ever found. The fossilised child was
estimated to be about 3 years old at death, female and a member of the australopithecus afarensis species. The
afarensis species, a major human ancestor, lived in Africa from earlier than 3.7 million to 3 million years ago.
‘Her completeness, antiquity and age at death make this find of unprecedented importance in the history of
paleo-anthropology’, said Zeresenay Alemseged, a noted paleo-anthropologist.
Prior to this discovery, it had been thought that the afarensis species had abandoned the arboreal habitat of
their ape cousins. However, while the lower limbs of this fossil supported findings that afarensis walked
upright, its gorilla-like arms and shoulders suggested that it retained the ability to swing through trees. This
has initiated a re-examination of many accepted theories of early human development. Also, the presence of
a hyoid bone, a rarely preserved bone in the larynx that supports muscles of the throat, has had a
tremendous impact on theories about the origins of speech. The fossil bone is primitive and more similar to
that of apes than to that of humans, but it is the first hyoid found in such an early human related species.

4. The passage quotes Zeresenay Alemseged in order to


1. provide evidence to qualify the main idea of the first paragraph.
2. question the claims of other scientists.
3. provide evidence to support the linguistic abilities of the afarensis species.
4. provide evidence that supports the significance of the find.
Which of the statements given above is/are correct?
(a) Only 1 (b) Only 2 (c) 1 and 3 (d) Only 4

5. It can be inferred from the passage’s description of the discovered fossil hyoid bone that
1. Australopithecus afarensis were capable of speech.
2. the discovered hyoid bone is less primitive than the hyoid bone of apes.
3. the discovery of the hyoid bone necessitated the re-examination of prior theories.
Which of the statements given above is/are correct?
(a) Only 1 (b) 2 and 3 (c) 1 and 3 (d) Only 3

6. According to the passage, the impact of the discovery of the hyoid bone in the field of
Archaeology could best be compared to which one of the following examples in another field?
1. The discovery and analysis of cosmic rays lend support to a widely accepted theory of the
origin of the universe.
2. The original manuscript of a deceased 19th century author confirms ideas of the development
of an important work of literature.
3. Newly revealed journal entries by a prominent Civil War era politician lead to a questioning of
certain accepted historical interpretations about the conflict.
Which of the statements given above is/are correct?
(a) Only 1 (b) 1 and 2 (c) Only 3 (d) None of these
88 Practice Set 4

B Passage 5
Sea ice has a bright surface, so much of the sunlight that strikes it is reflected back into space. As a result,
areas covered by sea ice do not absorb much solar energy. Sea ice also affects the movement of ocean
waters.
Water below sea ice has a higher concentration of salt and is denser than surrounding ocean water and
so it sinks. In this way, sea ice contributes to the ocean’s global ‘conveyor-belt’ circulation. Cold, dense,
polar water sinks and moves along the ocean bottom towards the equator, while warm water from
mid-depth to the surface travels from the equator towards the poles. Changes in the amount of sea ice
can disrupt normal ocean circulation, thereby leading to changes in global climate.
‘Satellites can provide detailed measures of how much ice is covering the pole right now, but sediment
cores are like fossils of the ocean’s history’, said Mr Gregery. Sediment cores are essentially a record of
sediments that settled at the sea floor, layer by layer and they record the conditions of the ocean system
during the time they settled.
Scientists can search for a bio-chemical marker that is tied to certain species of algae that live only in ice.
Polar bears, whales, walrus and seals are changing their feeding and migration patterns. Both the
atmospheric currents and the ocean currents can be expected to change.

7. According to the passage, what helps Mr Gregery study the ocean’s history?
1. Sediment cores.
2. Bio-chemical markers.
Select the correct answer using the codes given below.
(a) Only 1 (b) Only 2
(c) Both 1 and 2 (d) None of these

8. Which of the following statements is supported by the author about the usage of bio-chemical
markers?
1. To find the history of ocean structure.
2. To find the existence of ice at that time.
3. To find the changes in total volume of ice.
4. To find the changes in surface area.
Which of the statements given above is/are correct?
(a) 1 and 2 (b) Only 2
(c) 3 and 4 (d) Only 3

9. The second paragraph in this passage explains


1. the formation of sea ice.
2. rise of temperature on Earth due to sea ice.
3. how the ocean currents are formed.
Select the correct answer using the codes given below.
(a) Only 1 (b) Only 2
(c) 1 and 2 (d) Only 3
10. All of the following are possible side effects of sea ice melting except
(a ) change in ocean current
(b ) changes in migration patterns of polar bears
(c ) increase in temperature
(d ) increase in the density of Arctic sea water
Stage 1 Know The Trend 89

11. A dishonest milkman professes to sell his milk at cost price, but he mixes with water and there
by gains 25%. The percentage of water in the mixture is
1
(a) 25% (b) 20% (c) 8% (d) 10 %
2

12. In the firing range, 4 shooters are firing at their respective targets. The first, the second, the
third and the fourth shooter hits the target once every 5 s, 6 s, 7 s and 8 s, respectively. If all of
them hit their target at 10 : 00 am, when till they hit their target together again?
(a) 10 : 14 am (b) 10 : 28 am (c) 10 : 30 am (d) 10 : 31 am

13. All graduates own bikes. Does Akash, an engineer, own a bike?
1. All graduates are engineers. 2. All engineers are graduates.
Mark your answer as
(a) if 1 along is sufficient to answer the question
(b) if 2 alone is sufficient to answer the question
(c) if 1 and 2 together are sufficient to answer the question
(d) if 1 and 2 together are not sufficient to answer the question
14. In an election, there were only 2 candidates Nishant and Arvind. The total voters were 10000
and all the voters cast their votes, out of which 20% were declared invalid. If Nishant got 70%
of the valid votes, then how much did Arvind got?
(a) 2400 (b) 4000 (c) 3000 (d) 5600
15. How many dots are there on the face opposite the face with two dots?

(a ) 1 (b ) 5 (c ) 4 (d ) 6

Directions (Q. Nos. 16-19) The following questions are to be answered with reference to the
following explanation and table.
Ten judges were asked to judge the relative sweetness of five compounds A, B, C, D and E by the
method of paired comparisons. In judging each of the possible pairs, they were required to state
unequivocally which of the two compounds was sweeter a judgement of equality or of no difference
was not permitted.
The results of their judgements are summarised in the given table below. In studying the table,
note that each cell entry shows the number of comparisons in which the ‘row’ compound was
judged to be sweeter than the ‘column’ compound.

A B C D E
A 5 8 10 2
B 5 3 9 6
C 2 7 7 8
D 0 1 3 4
E 8 4 2 6

16. How many comparisons did each judge make?


(a ) 5 (b) 10 (c) 15 (d) 20

17. Which compound was judged to be least sweet?


(a ) A (b ) B (c ) C (d ) D
90 Practice Set 4

18. Which of the following statements is most nearly correct?


(a) There was almost perfect agreement among the ten judges
(b) The nearest discrimination was between compounds B and C
(c) The judges were not experts in discriminating sweetness
(d) Compound D was most clearly discriminated from the other four compounds

19. Between which two compounds was the discrimination least consistent?
(a) A and D (b) B and E (c) C and E (d) A and B

Directions (Q. Nos. 20-26) Read the following two passages and answer the questions that
follow each passage. Your answers to these questions should be based on the passages only.

B Passage 1
The standard methods of science proceed from observations to hypotheses to test these hypotheses in
controlled experiments. However, it would be a mistake to suppose that every hypothesis that comes out
of observation lends itself to rigorous scientific scrutiny. There are, infact, many questions that can be
asked of science that science is not in a position, for one reason or another, to answer. The recent debate
over melanoma (skin cancer) screening provides an interesting example of this area of ‘science that is not
scientific’ or ‘trans science’ as a few eminent thinkers have termed it. Let’s start with the observations.
There has been an increase in the number of early stage melanoma cases over the last twenty years. As a
result of the reported numbers, some physicians recommend screening for melanoma.
The ‘hypothesis’ that is implied here is that screening for melanoma will decrease the death rate from the
disease. But how do we test it? The conventional way to evaluate the effectiveness of a medical
technique is the double blind trial. In this case we would have to assign some people to receive screening
and some control people would not be screened. Then, we would look at the death rate for melanoma in
the two groups. The problems are logistic and ethical. If the answers are to reach statistical significance
we need very large numbers and we need to follow people over whole lifetimes neither of which is
practical. And how do we decide who is to receive what might be a life saving screening and who will be
denied its potential benefits?

20. The author would apparently agree with which of the following?
(a ) The effectiveness of screening for melanoma is not proven
(b ) Double blind trials are the best methods to evaluate
(c ) The death rate from melanoma is rising rapidly
(d ) None of the above

21. The word ‘hypothesis’ is placed in inverted commas to


(a ) suggest that the contention in the same sentence cannot be tested scientifically
(b ) emphasise the importance of framing hypotheses correctly
(c ) draw attention to the main word in the sentence
(d ) indicate that the author is using someone else’s view

22. Which of the following does the author mention as an example/examples of the ‘reasons’
mentioned in the sentence? ‘There are, in fact, many questions that can be asked of science
that science is not in a position, for one reason or another, to answer’?
1. Insufficiency of sample size 2. Ethical considerations
3. Ambiguous data
Which of the statements given above is/are correct?
(a) Only 1 (b) 1 and 2 (c) 1, 2 and 3 (d) Only 3
Stage 1 Know The Trend 91

B Passage 2
In the long run a government will always encroach upon freedom to the extent to which it has the power
to do so, this is almost a natural law of politics, since, whatever the intention of the people who exercise
political power, the sheer momentum of government leads to a constant pressure upon the liberties of
the citizen. But in many countries, society has responded by throwing up its own defenses in the shape
of social classes or organised corporations, which enjoining economic power and popular support, have
been able to set limits to the scope of action of the executive.
The fascist dictatorships of today are the first truly tyrannical governments, which Western Europe has
known for centuries and they have been rendered possible only because on coming to power they
destroyed all forms of social organisations, which were in any way rivals to the state.

23. The passage can most accurately be described as a discussion of the


(a) safeguards of individual liberty (b) functions of trade union
(c) limited power of monarchies (d) ideal of liberal government

24. Fascist dictatorship differs from monarchies of recent times in


(a) eradicating people’s organisations (b) setting limits to their scope of action
(c) exerting constant pressure on liberties (d) rivalling the state in power

25. The natural relationship between government and individual liberty is one of
(a) moderate complicity (b) inherent opposition
(c) marked indifference (d) fundamental interdependence

26. The passage suggests which of the following about fascist dictatorships?
(a ) They maintain their dominance by rechanneling opposing forces in new directions
(b ) They represent a more efficient form of the executive
(c ) Their rise to power came about through an accident of history
(d ) They mark a regression to earlier despotic forms of government

Directions (Q. Nos. 27-31) Study the following graph carefully and answer the questions given
below.
Number of employees working in various departments of two different companies

9
Number of employees (in hundred)

8 Company A
7 Company B
6
5
4
3
2
1
0
IT HR Marketing Finance Admin
Department

27. The number of employees working in the Marketing department of company B is


approximately what per cent of the total number of employees working in the company B?
(a) 30% (b) 28% (c) 23% (d) 32%
92 Practice Set 4

28. What is the ratio of the total number of employees working in the Admin department of both
the companies together and the total number of employees working in the Finance department
of both the companies together?
(a ) 2 : 3 (b ) 4 : 3 (c ) 3 : 2 (d ) 3 : 4

29. The number of employees working in the HR department of company A is approximately what
per cent of the number of employees working in the Finance department of company B?
(a) 44% (b) 207% (c) 53% (d) 267%

30. If the number of employees working in the IT department of company B is increased by 20%,
what would be the difference between the number of employees working in the IT department
of company B and the Admin department of company A?
(a) 560 (b) 350 (c) 700 (d) 400

31. What is the average number of employees working in all the departments together in company A?
(a) 600 (b) 585 (c) 620 (d) 610

32. 4 History books, 3 Geography books and 2 Civics books are kept on a shelf randomly. The
probability that books of same type are kept together, is
(a) 1/35 (b) 1/105 (c) 209/210 (d) 1/210

33. If the second half of the following alphabet is written in the reverse order, then which will be
the tenth letter to the left of the ninth letter coming from your right?
ABCDEFGHIJKLMNOPQRSTUVWXYZ
(a ) I (b ) C (c ) J (d ) H

34. A, B, C, D, E, F and G are sitting in a line facing the East. C is immediate right of D. B is at an
extreme end and has E as his neighbour. G is between E and F. D is sitting third from the South
end. Who are the persons sitting at the extreme ends?
(a) A and E (b) A and B (c) F and B (d) C and D

35. In a feast organised in connection with a marriage, some of the participants were vegetarians,
some others were non-vegetarians. Another group was of non-vegetarians not eating meat and
yet another group of people eating meat, but not fish. Which of the following represents this
Statistics?

(a) (b) (c ) (d)

36. Heavier coins are costlier. Ram’s coin is heavier than Mohan’s and costlier than Ramesh’s.
Naresh’s coin is costlier than Ram’s, but lighter than Yogesh’s. Ramesh’s coin is costlier than
Mohan’s. Who is the owner of the costliest coin?
(a) Ram (b) Ramesh (c) Yogesh (d) Naresh

37. Examine the following three figures in which the numbers follow a specific pattern.
27 42 27

4 13 16 13 11 65 8 ? 72

3 7 9

The missing number (?) in the third figure is


(a ) 9 (b) 18 (c) 12 (d ) 6
Stage 1 Know The Trend 93

1 2
38. The chance of Geeta winning a competition is and that of Seema winning it is . Find the
9 15
probability that only one Geeta and Seema wins it.
29 135 29 29
(a ) (b ) (c ) (d )
132 233 135 59

39. Examine the following statement.


˜A State Government suspended two additional District Judges.
Three assumptions numbered 1, 2 and 3 are given.
Assumptions
1. They were negligent in discharging duties.
2. There was a charge of misconduct against them.
3. The government officials were biased against them.
Which of the above assumptions is/are implicit in the statement?
(a) None is implicit (b) Either 1 or 2 is implicit
(c) Any one of the three is implicit (d) 1 and 3 are implicit

40. The total money collected for new year celebrations in a certain building was ` 20500. The
ratio of the amount contributed by the people of the A wing to that contributed by the people
of the B wing was 8 : 5. Also, the ratio of the amount contributed by the people of the B wing to
that contributed by the people of the C wing was 2 : 3. Find the amount contributed by the
people of B wing.
(a) ` 4000 (b) ` 5000 (c) ` 5500 (d) ` 4950
1
41. A garrison had provisions for a certain number of days. After 10 days, of the men desert and
5
it is found that the provisions will now last just as long as before. How long was that?
(a) 15 days (b) 25 days (c) 35 days (d) 50 days

42. Examine the following information.


˜ Monitoring has become an integral part in the planning of social development
programmes. It is recommended that Management Information System (MIS) be
developed for all programmes. This is likely to give a feedback on the performance of the
functionaries and the efficacy with which services are being delivered.
Two conclusions numbered, 1 and 2, are given below. You have to assume everything in the
statement to be true, then consider the two conclusions together and decide which of them
logically follows beyond a reasonable doubt from the information given in the statement.
Conclusions
1. All the social development programmes should be evaluated.
2. There is a need to monitor the performance of workers.
(a ) Only Conclusion 1 follows
(b ) Only Conclusion 2 follows
(c ) Either 1 or 2 follows
(d ) Both 1 and 2 follow
94 Practice Set 4

Directions (Q. Nos. 43-47) Read the following two passages and answer the questions that
follow each passage. Your answers to these questions should be based on the passages only.

B Passage 1
Classical music is termed ‘classical’ because it can be heard over and over again without the listener
tiring of the music. A raga can be heard many times over with the same or even heightened enjoyment
with every subsequent hearing. It is unfortunate that the Compact Disc sales of classical music are
dismal compared to other types of music. Perhaps this is because many people in our generation were
not exposed to classical music at an early age and therefore did not get to know the music.
In contrast to classical music, rock and contemporary music has a high impact on the listener, but
unfortunately is not evergreen. Its enjoyment lasts only as long as there is current interest in the topic or
emotion that the music portrays. This invariably lasts for three months or so until other music replaces
it, especially when another best selling song comes out. The reason why the impact of this music is not
as great when it first comes out is thought to be because technically the intricacy of the music is not high
and not sophisticated, although many critics believe it is because the music elicits a particular emotional
feeling which gradually wears out overtime.
43. According to the passage, most young people do not like classical music because
(a) they buy only the best selling song and music
(b) they do not have the sophistication of a true music-lover
(c) they grow tired of classical music
(d) they did not hear that type of music in their youth

44. The reason the enjoyment of a particular piece of contemporary music may not last as long as a
piece of classical music is
(a) changes in the emotions of a person
(b) high sophistication of the classical music and its technical intricacy
(c) the emergence of new pieces of contemporary music that replaces the ones before
(d) the desire of the youth for something novel

B Passage 2
If the world is going to use wind energy to reduce greenhouse gas emissions, there are going to have to
be an awful lot more installations. The problem is that in our haste to cash in on the obvious
environmental benefits of wind power, we are largely ignoring the ecological damage that turbines can
do. Some ecologists are warning that unless we think carefully about where wind farms are sited, they
could disrupt fragile ecosystems and even contribute to global warming. Worldwide, wind energy still
accounts for little more than 0.5% of total electricity generation. It is not surprising that government
have looked at growing electricity demand and the public’s fears about global warming and seen wind
energy as part of the solution.
But there is a problem, where do you put hundreds, if not thousands, of wind turbines? Lindsay is an
advocate for renewable energy but has become concerned by the scale and number of wind farm
developments on peat bogs in Europe.
‘This is the Cinderella ecosystem’, he says. ‘Peat land is busy performing a series of important functions
for us and we just don’t see it’, Bogs often play a critical role in providing clean drinking water. More
significantly, in the context of renewable energy, they store three times as much carbon as is held, in
tropical rainforests.
Stage 1 Know The Trend 95

Mike Hal from the Cumbria Wildlife Trust in North-West England has developed a formula to give a
wind energy carbon dioxide ‘budget’ that balances the carbon dioxide savings that a project is expected
to provide against the carbon dioxide costs from the manufacture and shipping of the turbines and
construction work at the site. The major CO 2 debt incurred by a wind turbine on a peat-rich site is not
in its manufacture and installation but in the ongoing degradation of peat.
45. The key question regarding wind energy is
1. Is wind energy really eco-friendly?
2. Where do you put the hundreds or thousands of turbines?
3. Have you assessed all the pros and cons of wind energy?
4. Is wind energy more harmful than beneficial, ecologically, commercially and aesthetically?
Select the correct answer using the codes given below
(a) 1 and 2 (b) Only 2 (c) 3 and 4 (d) Only 3

46. Which of the following statements regarding bogs is not true?


1. Prior to 2005, constructing on peat bogs had posed no threat.
2. From an ecological point of view, bogs are best left alone.
3. Bogs are delicate ecosystems that hold the largest quantity of carbon on land.
Select the correct answer using the codes given below.
(a) Only 1 (b) Only 2 (c) Only 3 (d) None of these

47. The author uses the term ‘Cinderella ecosystem’ for wind mills to
1. draw our attention to the fragility of the system.
2. focus on the fact that wind turbines are an eyesore.
3. show that it is a fairy tale to expect returns from wind turbines.
4. emphasise that the peat bogs have been ignored for a long time and deserve greater attention.
Select the correct answer using the codes given below.
(a) Only 1 (b) 2 and 3 (c) Only 3 (d) Only 4

Directions (Q. Nos. 48-50) Examine the information given below and answer the questions that
follow.
Six friends L, M, N, P, Q and R are sitting around a rectangular table facing the centre such that
two friends are seated along the each of the longer sides and one is seated along each of the shorter
sides. It is also known that
1. M is sitting diagonally opposite to L.
2. only R is sitting between M and Q.
3. P is sitting opposite to M.
48. Who is sitting opposite to N?
(a ) Q (b ) R
(c ) L (d) Cannot be determined
49. Who are the two friends sitting along one of the longer sides of the table?
(a) P and N (b) M and R (c) R and P (d) R and Q

50. Which of the following statements is definitely true?


(a) M is sitting to the immediate right of N (b) R is sitting to the immediate left of Q
(c) L is sitting between P and Q (d) None of these
96 Practice Set 4

Directions (Q. Nos. 51-55) Examine the information given below and answer the questions that
follow.
The growth of the aircraft services is driven by the increase in the number of people using aircraft
services and the increase in per person use of the airplanes. In 2010, it was expected that there
will be 200 million aircraft service users in India or about 20% of the population will generate
50 billion in aircraft services revenues. Services revenues should expand from 50 to 150 billion by
2014, while the number of users should grow to over 560 million or to about half the population of
India in the same period.
51. What is the estimated population of India in 2010?
(a) 98 crore (b) 1000 crore
(c) 100 crore (d) 115 crore

52. It is believed that, if 50% of the population of any country can afford aircraft services
use, it is economically developed. Can we say that India will be a developed country
in 2013?
(a) Yes (b) No
(c) Cannot say (d) Data inadequate

53. What will be the simple average growth rate of population of India in the given period
2010-14?
(a) 2% (b) 3% (c) 4% (d) 4.5%

54. What will be the percentage growth of the revenues of the aircraft services in the given
period 2010-14?
(a) 200% (b) 230% (c) 260% (d) 300%

55. If 18 persons can build a wall 140 m long in 42 days, then the number of days that 30 persons
will take to complete a similar wall 100 m long, is
(a) 18 (b) 21 (c) 24 (d) 28
56. Three statements, labelled 1, 2 and 3 are given below. You have to decide whether the data
given in the statements are sufficient for answering the item. Using the data given in the
statements, you have to choose the correct alternative.
Statements
1. Vidhya’s basic salary is ` 100 more than Rajni’s salary who also serves in Vidhya’s company.
2. Other allowances drawn by Rajni besides her basic salary are ` 2000 per month which is ` 50
less than that of Vidhya.
3. Rajni’s basic salary is ` 1550 per month.
What is the total monthly salary of Vidhya?
(a ) Only 2 is sufficient to answer the question
(b ) Both 2 and 3 are sufficient to answer the question
(c ) Both 1 and 2 are sufficient to answer the question
(d ) All 1, 2 and 3 are needed to answer the question
57. Examine the information given below.
˜All the research scholars are psychologists. Some psychologists are scientists.
Which of the following is a valid conclusion regarding the above argument?
(a) All the research scholars are scientists (b) Some research scholars are scientists
(c) All scientists are psychologists (d) Some psychologists are research scholars
Stage 1 Know The Trend 97

58. Select the answer figure in which the question figure is hidden/embedded.
Question figure

Answer figures

(a) (b) (c ) (d)

59. If Deepesh had walked 20 km/h faster he would have saved 1 h in the distance of 600 km, then
what is the usual speed of Deepesh?
(a) 100 km/h (b) 120 km/h (c) 150 km/h (d) None of these
60. Kavita, Babita and Samita started a work 5 days later Samita left the work and Babita left the
work after working 8 days. In how many more days Kavita would have completed the rest
work, if they take 20, 60 and 30 days individually to finish a work?
(a ) 4 (b ) 5 (c ) 6 (d ) 8
61. In the following figure, if the centres of all the circles are joined by horizontal and vertical
lines, then find the number of squares that can be formed.

(a ) 6 (b ) 7 (c ) 8 (d ) 1
62. In a row of children, Harish is eleventh from the left and Mangesh is seventeenth from the
right. When they exchange their places, Harish will be thirteenth from the left, then which of
the following will be the new position of Mangesh from the right?
(a) 11th (b) 12th (c) 19th (d) 29th
63. Amit said to Sushma, “Your father’s son is the only brother of my sister’s daughter.” How is
Sushma related to the son of Amit?
(a) Maternal grandmother (b) Cousin (c) Paternal grandmother (d) Aunt
64. If 15th February, 1933 was Tuesday, then what was the day on 19th February, 1932?
(a) Tuesday (b) Monday (c) Wednesday (d) Thursday
65. Six faces of a cube are painted with six different colours namely blue, brown, green, pink,
red and yellow. Diagrams given below show two difference positions of the painted cube.

Red Pink
Green
Green

Yellow
Blue

Which colour is on the face opposite to brown coloured face?


(a) Blue (b) Yellow (c) Green (d) Red
98 Practice Set 4

66. What is the water image of given question figure?


Question figure

C D O

Answer figures

C D O C C
(a) (b) (c ) D (d) D
C D O O O

Directions (Q. Nos. 67-73) The following questions are based on three passages in English to
test the comprehension of English language and therefore, these questions do not have Hindi version.
Read each passage and answer the questions that follow.

B Passage 1
On 31st October, 1517, Luther nailed a long list of points for disputation—95 thesis—to the door of the
church near the castly in the Saxon capital of Wittenberg. It is a moment that has reverberated in
history, the dead day on which the protestant reformation was born and the middle ages suddenly
dropped. The reality is more prosaic. Some scholars have denied that the thesis were ever posted at all. It
seems recently likely that they were, but this was hardly a world-shattering act. Luther was now a
professor at the founded University of Wittenberg and the conventional method of initiating academic
debate within the theology faculty was to post thesis in advance. Because of its handy location, the door
of the Castle Church served as the university’s bulletin board and Luther’s gesture has been seen as no
more dramatic is pinning up a lecture list in a modern college.

67. When in the popular perception, died the middle age cease to exist?
(a ) After the birth of Luther
(b ) After Luther’s declaration of his thesis in Wittenberg
(c ) After Luther became a professor
(d ) After the foundation of the University of Wittenberg

68. Which of the following explanations reduces the dramatic effect of the nailing of Luther’s
thesis?
1. It was mere professor lecture list.
2. It was posted in a rather unknown church.
3. That the thesis were non-academic in nature.
4. Protestant reformation was not born with nailing act.
Select the correct answer using the codes given below
(a) Only 1 (b) 1 and 3 (c) Only 4 (d) 2 and 4
Stage 1 Know The Trend 99

B Passage 2
The aim of the Platonic philosophy was to exalt man into a God. The aim of the Baconian philosophy
was to provide man with what he requires while he continues to be a man. The aim of Platonic
philosophy was to raise us far above the vulgar wants. The aim of Baconian philosophy was to supply
our vulgar wants. The former aim was noble; but the latter was attainable.
Plato drew a good bow; he aimed at the stars; and therefore, though there was no want of strength or
skill, the shot was thrown away. His arrow was indeed followed by a track of dazzling radiance; but it
struck nothing. Bacon fixed his eye on a mark, which was placed on Earth and hit it in the white. The
philosophy of Plato began with words and ended in words, noble words indeed, words such as were to be
expected from the finest of human intellects exercising boundless dominion over the fittest of languages.

69. The above passage presents Platonic philosophy as


1. giving rise to vulgar wants.
2. too idealistic in terms of a realistic assessment of a man.
Select the correct answer using the codes given below
(a) Only 1 (b) Only 2 (c) Both 1 and 2 (d) None of these

70. Which one of the following best reflects the underlying tone of the passage?
1. All ideas regarding man are couched in noble words.
2. Man when exalted into a God comes to nothing.
3. It is the image of man conceived differently that makes the basic distinction between different
systems.
Select the correct answer using the codes given below
(a) Only 1 (b) Only 2 (c) 2 and 3 (d) Only 3

B Passage 3
Intellectual property law firms are playing a major role in the global IP protection and enforcement.
Many individuals and companies are utilising the services of Indian law firms and lawyers to protect
their respective IP rights. However, techno-legal issues have complicated the traditional IP rights
management. These days information technology is increasingly being used world over that is both
facilitating and infringing the IP right of others. Thus, role of traditional IP law firms has significantly
changed. These days an IP rights can be protected only if the law firm or lawyer is/are good at both IP
and technical aspects. Indian IP law firms and lawyers must adopt the technological revolution that is
changing the entire service sectors world over. As more and more IT is being used to protect and enforce
IP rights at global level, ignoring IT issues is not a wise option.
International treaties and agreements have further extended the otherwise territorial nature of IP
protection and rights. For instance, with the adoption of Madrid Agreement and Madrid Protocol by
India, trademarks protection in India would be given a new meaning. The IP law firms and lawyers of
India must change gears now as even the Indian Government has adopted technological methods to file
and pursue IP rights in India. E-filing of various IP applications alongwith their online dealing is
gradually becoming popular in India.

71. Consider the following statements


1. Intellectual property rights can be protected, if the lawyers are technically sound.
2. Use of IT services world wide, has decreased the role of traditional intellectual property law
firms.
100 Practice Set 4

3. Detailed knowledge of intellectual property laws, is the only way, to protect the interest in case
of infringement of the concerned rights or interest.
4. India adopted Madrid Agreement and Madrid Protocol, for the protection of trademarks.
Which of the statements given above is/are correct?
(a) Only 4 (b) 2 and 4 (c) 3 and 4 (d) All of these

72. Which one of the following, is the best ‘title’ for the given passage?
(a ) Cyber law firms in India
(b ) Intellectual property law firms in India
(c ) Role of information technology in legal sphere
(d ) Role of international agreements in protecting IP rights

73. Consider the following statements


1. Intellectual property law firms, should concentrate on the core law, in order to succeed in any
litigation, resulting from the infringement of any interest.
2. Indian Government is implementing new methods, which are technologically sound to
safeguard the intellectual property rights.
3. Though, Information Technology has influenced each and every sphere of our lives, but there
is no difference between the traditional and contemporary IP rights services.
Which of the statements given above is\are correct?
(a) Only 3 (b) Only 2 (c) 1 and 3 (d) 1, 2 and 3

74. Examine the situation given below


˜ You are the team leader of a team. A new assignment is given to your team, which is to be
completed in two months. Although you have great experience, your team does not have
much experience.
Arrange the following in the order of the most effective to least effective steps in terms of what
you will do.
1. Request higher authorities for change of team members.
2. Arrange training in skills necessary for the assignment.
3. Do all the work on your own.
4. Assign work to the people based on their strengths and weaknesses.
Codes
(a) 2, 4, 1, 3 (b) 2, 4, 3, 1 (c) 4, 2, 1, 3 (d) 4, 2, 3, 1

75. Three statements, labelled 1, 2 and 3, are given below. You have to decide whether the data
given in the statements are sufficient for answering the item. Using the data given in the
statements, you have to choose the correct alternative.
Statements
1. Pointing to the photograph, Prem said, ‘‘She is the mother of my father’s only granddaughter’’.
2. Prem has no siblings.
3. Pointing to the photograph, Prem said, ‘‘She is the only daughter-in-law of my mother.’’
How is the girl in the photograph related to Prem?
(a ) Any two of the three are sufficient to answer the question
(b ) Both 1 and 2 are sufficient to answer the question
(c ) Both 2 and 3 are sufficient to answer the question
(d ) Either only 3 or both 1 and 2 are sufficient to answer the question
Stage 1 Know The Trend 101

Directions (Q. Nos. 76-80) Given below are five questions. Each question describes a situation
and is followed by four possible responses. Indicate the response you find most appropriate. Choose
only one response for each question. The responses will be evaluated based on the level of
appropriateness for the given situation. Please attempt all the questions. There is no penalty for wrong
answers for these five questions.

76. You are the executive director of an upcoming Infotech company which is making a name for
itself in the market. In a short period of time, Mr A has helped in doubling the revenues as well
as creating a high brand equity for the company so much so, that you are thinking of
promoting him. However, you have been receiving information from many corners about his
attitude towards the female colleagues, particularly his habit of making loose comments on
women. What would you do?
(a) Ask him his resignation
(b) Transfer him to another branch office
(c) You yourself go for fact finding
(d) Let it go as he is a valuable worker in your organisation

77. You have been posted as a District Magistrate in a district where construction work is going on
a large scale. One day suddenly you receive a call from someone that many labourers have
been injured in an accident. When you reach there, you find a huge crowd of labourers, their
relatives and media persons. You will
(a) hold a meeting with experts and direct the rescue team for further action
(b) tell the crowd to vacate the place
(c) call the owner of the construction project
(d) as certain how many labourers were on duty and how many were injured to take any further step

78. You led a team, which has recently completed a very important task, assigned by the
government. A felicitation programme was organised by the Mayor of your city. In the
programme, he was showering accolade on you without mentioning about your team. What
would you do?
(a) Thanked him and then said that you did not work alone and it was a team effort
(b) Took the praise and then told your team that you did not know how to explain in front of the
Mayor that the work was a team effort
(c) Accept the accolades and try to steal the spotlight
(d) Keep mum and do nothing

79. You are working as a Sales Manager in Indian Railways and your senior is trying to make you
understand some difficult, but important project. You are not able to understand some parts of
it. What should be your reaction?
(a ) Listen silently without showing any interest
(b ) Remain aloof from all the discussion
(c ) Try to excuse yourself and ask your senior to explain it to you again
(d ) Make others feel that you have understood everything

80. You are working with Delhi Police. A task has been assigned to one of your co-workers, but
you believe that you can do it in a better way because of your knowledge and prior experience
in that case. What would you do?
(a) Inform your senior to assign this task to you, explaining him that you can better deal with the
case
(b) Work with your co-worker in parallel and submit to your senior for his approval
(c) Show your feelings to your co-worker in achieving the task with cooperation
(d) Concentrate on your work
Space for Rough Work
Stage 1 Know The Trend

Answer with Explanations


1. (c) Nearly 31 per cent of India’s current population lives in Statement 1 is wrong because history of ocean structures
urban areas and contributes 63 per cent of India’s GDP. can be found by sediment core. Statements 3 and 4 are
With increasing urbanisation there is a need of sustainable wrong because these are not talked about in the passage.
and comprehensive development of cities. It is expected 9. (d) It explains how cool water moves towards the equator and
that Smart Cities Mission in India will definitely boost up the warm water at equator moves towards the poles (ocean
employment and enhance income for all sections of currents). Statement 3 is chosen because the whole
people leading to a better quality of life. paragraph explains how the ocean currents are formed.
2. (c) In India, playing on the tunes of what media plays, Indians Statement 1 is wrong because the lines give no explanation
are not very rational in approach. What was required was about the formation of sea ice. Statement 2 is wrong
severe sentence for what Juvenile offender did in because melting of sea ice causes the temperature to rise.
Nirbhaya’s case. Lowering of age for severe punishment 10. (d) Option (d) is chosen because melting of sea ice does not
for Juveniles is not a guarantee that long-term interests will increase the density of the Arctic sea. Also, nothing about
be taken care of the JJA is unlikely to stand the test of it is mentioned in the passage. Options (a), (b) and (c) are
time. the possible effects when the sea ice melts.
3. (a) Introduction of payment banks is going to help a large 11. (b) Let CP of 1 L milk be ` 1.
number of people from lower strata like migrant labours,
Then, SP of 1 L mixture = ` 1
low income household, small businesses and a large 100 4
number of people belonging to unorganised sectors. It is CP of 1 L mixture = ´ 1= `
125 5
really a welcome step and will certainly help government’s
decision to wider financial inclusion. By using Alligation rule,

4. (d) The point of this paragraph is to illustrate that Archaeology CP of 1 L of CP of 1 L


is like a physical science in that important factual pure milk of water
discoveries lead to theoretical changes. The quotation `1 0
provides evidence that this discovery is in fact a significant
one. So, the correct answer is option (d). 4
`—
5
5. (d) The passage states that the discovery of the hyoid bone 4– = — 4=—
1
— 4 1– —
has had a tremendous impact on theories about the 5 0 5 5 5
origins of speech. The passage goes on to say that it is 4 1
the first hyoid found in such an early human related \ Milk : Water = : = 4:1
5 5
species, suggesting that the timeline of human verbal
development would be changed by the discovery. Thus, it Hence, percentage of water in the mixture
1
can be inferred that the discovery made the = ´ 100= 20%
re-examination of prior theories necessary. So, the correct 5
answer is option (d). 12. (a) LCM of 5, 6, 7, 8
6. (c) The option (c) correctly describes a discovery that causes 2 5, 6, 7, 8
a re-examination of earlier ideas in another field. In this
5, 3, 7, 4
case, newly uncovered journal entries by a politician spur
a re-evaluation of certain historical ideas regarding an = 2 ´ 5´ ´ 3 ´ 7 4 = 840 s
important conflict. So, the correct answer is option (c). 840
Again, they hit target at = 10 : 00 am + min
7. (a) Deposits on a sea floor help Mr Gregery to study ocean 60
history. Statement 1 is chosen because sediment cores = 10 : 00 am + 14 min =10 :14 am
are essentially a record of sediments that settled at the
sea floor, layer by layer and they record the conditions of
13. (b) Statement 1 alone is not sufficient to as we cannot
conclude that Akash is a graduate. From Statement 2, we
the ocean system during the time they settled.
can say that Akash is a graduate. Thus, Akash owns a
Statement 2 is wrong. A bio-chemical marker is the study
bike.
of the chemical process that occurs within living things.
Hence, Statement 2 alone is sufficient answer the question
8. (b) Existence of the marker in the sediment reveals the
existence of the frozen land. Statement 2 is correct since it 14. (a) Valid votes = 80% of 10000 = 8000
agrees with author’s view. Author says that scientists can Nishant got = 70% of valid votes
search for biochemical marker that is tied to certain When Arvind got 30% of the valid votes, then
species of algae that live only in ice. Therefore, this marker Number of votes in favour of Arvind
is used to find the existence of ice at that time. = 30% of 8000 = 2400
106 Practice Set 4

15. (d) There are 2, 3, 4 and 6 dots respectively on the adjacent 28. (d) Number of employees in Admin department of both
faces of the face having five dots. Therefore, one dot will companies = 6
be opposite the face having five dots. There are 1, 3 and Number of employees in Finance department of both
5 dots respectively on the adjacent faces of the face companies = 8
having two dots. Therefore, either 4 or 6 dots will be on 6
\ Required ratio = = 3 : 4
the face opposite of the face having two dots. 8
Now, 4 and 6 dots cannot be on the opposite faces. 29. (d) Number of employees working in HR department of
Clearly, 6 dots are there on the face opposite to the company A = 8
face with two dots. Number of employees working in Finance department of
16. (b) Each compound is compared with all the other, giving company B = 3
20 comparisons or results but since each comparison has 8
\ Required percentage = ´ 100» 267%
20 3
been counted twice, we have = 10 comparisons.
2
30. (a) Increased number of employees working in IT department
of compnay B = æç 8 + 8 ´
17. (b) B was judged sweeter only 6 times. 20 ö
÷ ´ 100 = 9.6 ´ 100 = 960
è 100 ø
18. (d) The other compounds were judged sweeter 25, 23, 24
and 20 times, D was judged sweeter only 8 times. From the given graph, number of employees working in
Admin department of company A = 400
19. (d) Five judges called A sweeter and five judges called B
sweeter. \ Required difference = 960 - 400 = 560

20. (a) The author would apparently agree with the effectiveness 31. (d) Average number of employees working in all departments
of screening for melanoma is not proven. together in company A
5.5 + 8 + 8.5 + 4.5 + 4 ö
21. (a) The word ‘hypothesis’ means the contention in the same = æç ÷ ´ 100 = ç
æ 30.5 ö ´ 100= 610
÷
è 5 ø è 5 ø
sentence cannot be tested scientifically.
22. (b) According to the author, the meaning of sentence which 32. (d) Here, total books = 4 + 3 + 2 = 9
is mentioned in the question is insufficiency of sample Total number of ways of keeping the books on the
size and ethical considerations. shelf = 9!
Number of ways in which the books can be kept together
23. (a) The author says that all forms of government tend to
= (4! 3! 2!) 3!
become some what dictatorial. He shows how society
protects itself from this tendency. Throughout, he \ Required probability = (4! 3! 2!) 3! / 9! = 1 / 210
demonstrates how people tend to protect or safeguard 33. (d) Rewriting the second half of English alphabets in reverse
their individual liberties. order
24. (a) The last sentence says that the fascist dictatorship ABCDEFGHIJKLMZYXWVUTSRQPON
destroyed all forms of social organisations, which were in \ Ninth letter from right end = V
any way rivals to the state.
Hence, 10th letter to the left of V = H
25. (b) The author says that the tendency for a government to
34. (b) Sitting arrangement of A, B, C, D, E, F and G are as follows:
encroach upon individual liberty to the extent, to which it
B N
has the power to do so, is ‘almost a natural law of
E
politics’. Thus, government and individual liberty are
G
inherently opposite to one another. W E
F
26. (d) If the fascist dictatorships ‘are the first truly tyrannical D
governments, which Western Europe has known for C
A S
centuries’, then it can be inferred that centuries ago there
were tyrannical or despotic governments in Western Hence, A and B are sitting at the extreme ends.
Europe. Thus, the fascist governments represent a
35. (b)
regression to an earlier form of government.
Non-vegetarian
Solutions (Q. Nos. 27-31)
Meat Fish
We are not taking population multiplied by 100 wherever eating eating
not necessary.
27. (b) Number of employees in the Marketing department of Vegetarian
company B = 7
Total number of employees in company B = 25
7
\ Required percentage = ´ 100= 28%
25
Stage 1 Know The Trend 107

36. (c) According to the question, 43. (d) It is clearly mentioned in the passage that many people in
Ram > Mohan ...(i) our generation were not exposed to classical music at an
Ram > Ramesh ...(ii) early age and therefore, did not get to know the music.

Yogesh > Naresh > Ram ...(iii) 44. (c) In the second para of the passage, it is mentioned that
Ramesh > Mohan ...(iv) contemporary music has a high impact on the listener but
unfortunately is not evergreen. Its enjoyment lasts only as
From Eqs. (i),(ii),(iii) and (iv), we get
long as there is current interest in the topic or emotion that
Yogesh > Naresh > Ram> Ramesh >Mohan the music portrays. This invariably lasts for three months
Therefore, owner of the costliest coin is Yogesh. or so until other music replaces it, especially when another
27 16 best selling song comes out.
37. (c) From first figure, = 9 and =4
3 4
45. (b) The author does not question whether wind energy is
Þ 9 + 4 = 13 (middle number) eco-friendly. He only points to the problem in the selection
42 65
From second figure, = 6 and =5 of place for setting up a wind farm. In the passage, he
7 13 focuses on the problem, where do you put hundreds, if not
Þ 6 + 5 = 11 (middle number) thousands, of wind turbines.
27 72
From third figure, = 3 and =9 46. (a) It is not true that prior to 2005 constructing on peat bogs
9 8
had posed non threat.
Þ 3 + 9 = 12 (middle number)
\ ? = 12
47. (d) The word ‘Cinderella’ (from the fairy tale) means
something that has been ignored for a long-time and
38. (c) Probability that only one of Seema or Geeta wins the deserves to receive more attention.
competition = P (Seema wins and Geeta does not win) or P
(Geeta wins and Seema does not win) Solutions (Q. Nos. 48-50)
= P (S ) ´ P
¢ (G+) P (G´ ) ¢ P (S ) The two possible sitting arrangements as per the given
2 8 1 13 16 13 29 information be depicted as shown in the diagrams below.
= ´ + ´ = + =
15 9 9 15 135 135 135 L P P L

39. (b) A person holding an office is generally suspended on


Q N N Q
charges of misconduct or negligence of duty. So, either
1 or 2 is implicit. Assumption 3 seems to be vague and so
it is not implicit. R M M R

40. (b) Since, the amount collected by wing B is common to both 48. (a) Q is sitting opposite to N.
the ratios, it is used to compare the collections of all 49. (b) M and R are the two friends sitting alone one of the longer
3 wings. So, find the LCM of 5 and 2. sides of the table.
\ LCM of 5 and 2 = 10
50. (c) L is sitting between P and Q is definitely true.
\ Ratio of the amounts contributed by the people of all the
three wings = 16 : 10 : 15 51. (c) As 200 million = 20% of population
100
Let the amount contributed by each wing be 16x, 10x and \ Required population = 200 ´ = 1000 million
15x respectively. 20
Then, 16x + 10x + 15x = 20500 Þ 41x = 20500 = 100 crore
\ x = 500 52. (d) By 2014, half or 50% of the Indian population will use
Amount contributed by wing B = 10x = 10 ´ 500 = ` 5000 aircraft services. As we do not have any information about
2013, so the data is inadequate.
41. (d) Let initially then be x men having food for y days after
10 days, x men had food for ( y - 10) days. Also, æç x -
xö 53. (b) Population in 2010 = 1000 million
÷
è 5ø Population in 2014 after 4 yr = 560 ´ 2= 1120 million
men had food for y days. [as half of population (= 560) million]
\ x( y - 10=
4x
´ y Þ 5xy - 50x= 4xy (1120 - 1000)
) \ Average (simple) growth rate = ´ 100
5 1000 ´ 4
Þ xy - 50x= 0 120 12
= ´ 100 = = 3% per annum
Þ x( y - 50=
) 0 1000 ´ 4 4
Þ y = 50 (as x ¹ 0 )
54. (a) Total revenue in 2010 = 50 billion
42. (d) According to the statement, monitoring and evaluation of Total revenue in 2014 = 150 billion
social development programmes their function, (150 - 50)
performance and efficiency is absolutely essential. So, \ Growth in percentage = ´ 100 = 200%
50
both 1 and 2 follow.
108 Practice Set 4

55. (a) Here, M1 = 18, M 2 = 30 and D1 = 42 Work completed by all the three before Samita left
6´ 5 1
Let D2 = x, W1 = 140 and W2 = 100 = =
60 2
By using the formula, 1 1
M1 D1 M 2 D2 Remaining work = 1 - =
= 2 2
W1 W2 So, the remaining work is done by Kavita and Babita.
18 ´ 42 30 ´ x One day’s work of Kavita and Babita
Þ =
140 100 1 1 3+ 1 4
= + = =
Þ 140 ´ 30´ =x 100
´ ´ 18 42 20 60 60 60
100 ´ 18´ 42 4 ´ 3 12 1
\ x= = 18 3 day’s work of Babita and Kavita = = =
140 ´ 30 60 60 5
1 1 3
Hence, the required number of days are 18 . Remaining work to be done by Kavita = - =
2 5 10
56. (d) From Statement 3, Rajni’s basic salary = ` 1550 Remaining work
From Statement 1 and 3 Vidhya’s basic salary Number of days required =
One day¢ s work of Kavita
= ` (1550 + 100) = ` 1650 3 / 10
From statement 2, Rajni’s other allowances = ` 2000 = = 3 ´ 2= 6
1 / 20
and Vidhya’s other allowances = ` 2050
\ Vidhya’s monthly salary = (1650 + 2050) = ` 3700 61. (c)
D J C
So, all three statements are needed to answer the
question.
E K I
57. (d) The above arguments can be represented as
Scientists Scientists L
F H
Psychologists

Or
A B
G
Psychologists
Research Number of squares formed
Research scholars scholars = AFLG, GBHL, LHIK, FLKE, KICJ, KEJD, AEIB, FHCD
From the above diagram only option (d) i.e. some So, total of 8 squares will be formed.
psychologists are research scholars follows.
62. (c) According to the question,
58. (b) It is clear from the given option that, figure (b) has the Harish Mangesh
above figure embedded in it.
11th 17th

Left Right
On exchanging their places, we get
Mangesh Harish
59. (a) Let the speed of Deepesh be x km/h. 13th
Distance = 600 km (given)
According to the question, Left Right

600 æç -
600 600 1 1 ö
- =1 Þ ÷= 1 Q Total number of children = L + R - 1= 13 + 17 - 1= 29
x x + 20 è x x + 20 ø
Mangesh’s position from right = ( 29 - 11+ 1) =19th
Þ 600 ´ 20= x (x+ 20)
Þ x 2 + 20x - 12000= 0
63. (b) It is obvious from the given information that Sushma is
daughter of Amit’s sister. Hence, son of Amit will be cousin
Þ (x - 100)(x+ 120=
) 0 of Sushma.
Þ x = 100, - 120
64. (d) Number of odd days in the year 1932 (a leap year) = 2
So, x = 100 or x = - 120 (not possible)
The day on 15th February, 1932 must have been 2 week
Hence, the speed of Deepesh is 100 km/h. days before the day on 15th February, 1933 i.e. Sunday.
60. (c) Work done by Kavita, Babita and Samita in one day Hence, the day on 19th February, 1932 was Thursday.
1 1 1 65. (c) It can be inferred from the diagrams, the four adjacent faces
= + +
20 60 30 to green face are red, blue, yellow and pink.
3 + 1+ 2 6 Hence, brown coloured face will be opposite to the green
= =
60 60 coloured face of the cube.
Stage 1 Know The Trend 109

66. (b) The water image of given question figure will be formed as As team members do not have much experience, so
arranging training sessions for them can help them in
gaining skills and these skills can be utilised in the
project. So, Statement 2 is the second best response.
C D O As the project is to be completed on time, so requesting
for team members, who are capable of doing the
C D O
project, is the third best response. The work is to be
completed by a team and as an individual you may not
be able to complete the work within the deadline. So,
Statement 3 is the worst response. Hence, the order is
67. (b) From the lines “on 31st October, 1517, Luther nailed a long 4-2-1-3.
list of points for disputation-95- thesis-to the door of the 75. (d) From Statement 1, we conclude that the girl is either
church near the castle in the Saxon capital of Wittenberg. It is Prem’s or his brother’s wife. But, according to the
the moment that has reverberated in history, the dead day on Statement 2, Prem has no siblings.
which the protestant reformation was born and the middle So, from both 1 and 2, we conclude that the girl is Prem’s
ages suddenly dropped”. It can said that the middle age wife. From Statement 3, we find that the girl is the only
cease to exist after Luther declaration of his thesis in daughter-in-law of Prem’s mother, so she is Prem’s
Wittenberg. So, option (b) is valid while option (a), (c) and (d) wife.
are irrelevant.
76. (c) Without knowing overall situation, you should not ask
68. (a) From the line “Luther’s gesture has been seen as no more his resignation. He is valuable worker in you
dramatic as pinning up a lecture list in a modern college.” It organisation and there may be some conflict of interest
can be said that lists of points of disputation (95 thesis) were with other employees there. So, it’s better to go for fact
nothing, but mere professor’s lecture list, which only reduces finding by yourself and then take the decision
the dramatic effect of the nailing of Luthers thesis. Hence, considering his contribution to the upcoming company.
Statement 1 is only authentic. Statements 2, 3 and 4 are Simultaneously, if he is found to guilty as alleged, you
irrelevant from the point of view of the question. Hence, the should take stern action. So, option (c) is right here.
correct answer is option (a).
77. (a) In an emergency situation, the right way to handle the
69. (b) The Platonic philosophy is too idealistic in terms of a realistic situation is to start the rescue work as soon as possible,
assessment of a man. The aim of the Platonic philosophy so as to save maximum persons.
was to exalt man to the status of a God.
78. (a) Option (a) exhibits the integrity and team spirit of the
70. (d) The underlying tone of the passage is that it is the image of person. A team leader should acknowledge and
man conceived differently that makes the basic distinction appreciate the hard work and inputs provided by the
between different systems as mentioned by the distinct team members. If you do not let the other members
philosophers at different times. share praise with you, your professional relations with
71. (d) Statements 2 and 4 are correct, in 2nd paragraph, 3rd line your subordinates will turn bitter.
justifies Statement 2, as it states that the role of traditional IP 79. (c) Option (c) shows your courage to ask questions about
law firms has changed, here the sense is that role of something you have not understood. It also shows your
traditional IP law firms has decreased due to increase in use desire to learn difficult and interesting things. Also, this
of Information Technology. will help you in doing your work effectively. Options (a)
72. (b) The most appropriate title for this passage is “Intellectual and (b) show your indifferent attitude towards the work.
Property Law Firms in India”, the first line of the passage itself Option (d) shows that you are shying away from
supports it. Although, all the other options are also correct, learning concepts important to the pursuit of your
but they are subsidiary functions of intellectual property rights career.
law firms. 80. (c) Your senior has assigned the task to your co-worker as,
73. (b) Statement 2 is correct, as first 2 lines of the 7th paragraph he would have felt that your co-worker might be the
justifies this. Statement 1 is wrong/incorrect because right person to do the task. Hence, you cannot inform
according to 3rd paragraph, the intellectual property rights your senior to assign the task to you. Also it is your
can be protected only if the law firm or lawyer is/are good at feeling that you can do the task in a better way, but your
both IP and technical aspects. Statement 3rd is incorrect, senior may think otherwise.
because according to 3rd paragraph, there is a clear So, option (a) can be eliminated. Option (b) is not right
distinction between traditional and contemporary intellectual because working on the task without prior informing
property rights. your senior is not just. Option (d) is also not right
74. (c) As each and every one in the team has his/her own because if you have prior experience, then you should
strengths, so identifying them and assigning work on this at least try to help your co-worker in achieving the task
basis is the most effective response. with cooperation. So, option (c) is the right answer.09
Space for Rough Work
CSAT
C i v i l S e r v i c e s A p t i t u d e Te s t
Paper 2

Practice Set 5
Time : 2 hrs MM : 200

Instructions
1. There are 80 questions in this paper.
2. The answer of any question you are thinking that more than two answers are true, then you must choose the nearest one.
There is only one answer to be selected by you.

WEAKER AREAS
Penalty for wrong answer
3. There are four alternative answers in every question. When you select a wrong answer, then 1/3rd mark of that question is
deducted in your total marks.
4. If any candidate gives more than one answer and one of them is true but it is treated as a wrong answer and the candidate
is penalised for that and1/3rd marks will be deducted.

Directions (Q. Nos. 1-7) Read the following four passages and answer the questions that
follow. Your answers to these questions should be based on the passage only.

B Passage 1
YOUR

India’s approach to climate change has shifted dramatically in the span of a few years. India has
developed a comprehensive climate change program domestically and has adopted a new stance in the
international negotiations that has earned it the reputation of being a ‘deal maker’.
IMPROVE

The authors argues that this shift in India’s climate change strategy can be understood if seen in the
context of India's economic and development aspirations and the changes occurring in the larger
geopolitical landscape: India's desire to play a strategically important role in a new global order as well
as deal with domestically critical issues like energy security and energy access, all coalesce with the
climate issue. By engaging proactively on climate change, India may be able to advance all of these
objectives at once. However, to succeed, it must demonstrate that action on climate change does not
come at the expense of economic growth or development goals, and that these can go hand in hand.
Stage 2
112 Practice Set 5

1. Which among the following is the critical and the most rational inference that can be made
from the above passage?
(a) India has been termed as ‘deal maker’ as far as International negotiations on climate change are
concerned.
(b) India’s new stand on climate change is based on India’s desire to play a strategically important
role in a new global order as well as deal efficiently with domestically critical issues.
(c) India is serious when it comes to tackling problems arising out of climate change negotiations.
(d) Economic growth and development issues are left behind while dealing with the problem of
climate change.

B Passage 2
Public spaces in India’s cities are often eyesores, full of rotting piles of trash along the streets, in
neighbourhoods, public parks and play-grounds and outside fancy air-conditioned hall and five star
hotels. Trash and litter is everywhere and it does not bother any one in the world, all around the globe,
in taking a note of India’s economic growth but unfortunately Indians have moved very sluggishly in the
field of environmental cleanliness and taking care of the surroundings. You can easily find a well dressed
and educated person trashing and empty coke bottle in the middle of the street. We need to change this
attitude and make litting a socially unacceptable aspect. Having waste and garbage scattered around
should be a matter of concern for everyone.
2. Which among the following is the most logical and rational assumption that can be made from
the above passage?
(a) Our surroundings can be absolutely neat and clean if local Government bodies have adequate
staff with required skills and competencies.
(b) Indians do not take any note of their environmental cleanliness. For them trash lying scattered
on the roads does not bother.
(c) If everyone gets conscious of cleanliness and takes care of throwing garbage at public places, this
problem can be tackled.
(d) Strict laws and fines should be imposed on those who make our surrounding unhealthy and dirty.

B Passage 3
Tourism in India has a strong relevance to economic development, cultural growth and national
integration. As mentioned earlier, India is a vast country of great beauty and diversity and her tourists
potential is equally vast. With her rich cultural heritage as superbly manifest in many of the
architectural wonders (palaces, temples, mosques, forts, etc), caves and prehistoric wall paintings, her
widely varied topography ranging from the monotonous plains to the loftiest mountains of the world,
her large climatic variations ranging from some of the wettest and the driest as well as from the hottest
and the coldest parts of the world, beautiful long beaches on the sea coast, vast stretches of sands,
gregarious tropical forests and above all, the great variety of the life-style, India offers an unending
choice for the tourists.
3. Which among the following is the most logical and rational inference that can be made from
the above passage?
(a) India has a great potential for tourism industry which has a great relevance to economic
development, cultural growth and national integration.
(b) India’s tourists potential is very vast with unparallel beautiful landscapes and diverse cultural
heritage.
(c) Under the present scenario, threat of terrorist’s attacks, regional clashes and corruption at large,
India can not become a tourist destination.
(d) India’s foreign exchange reserve has gone up due to rise in number of tourists.
Stage 2 Improve your Weaker Areas 113

B Passage 4
Some economists believe that the United States can be utilised as a ‘land-bridge’ for the shipment of
containerised cargo between Europe and the ‘Far East’. Under this concept, containerised freight
travelling between Europe and the ‘Far East’ would shipped by ocean carrier to the United States East
Coast unloaded and placed on special railway flat cars and shipped via railroad to a West Coast port. At
this port, the containers would then be loaded on ships bound to a ‘Far East’ port of entry. Thus, a land
transportation system would be substituted for marine transportation during part of the movement of
goods between Europe and the ‘Far East’.
If a land-bridge system of shipment were deemed feasible and competitive with alternative methods, it
would open a completely new market for both United States steamship lines and railroads. At present,
foreign lines carry all ‘Far East’-Europe freight. American carriers get none of this trade and all water
routes exclude the railroads.
Thus, in addition to the land bridge getting this new business for the railroads, it also gives the United
States East Coast Ships an opportunity to compete for this trade. The land-bridge concept has the
potential of offering new job openings for United States railway workers and seamen. In addition, there
would be expansion of labour requirements for people in the ship building and container manufacturing
business. Such a system also has the potential of relieving the United States of a part of the burden it
now bears in the form of subsidies to the ship industry. By making United States rail transportation an
export service, the land bridge system would have a favourable effect on our balance of payment.
4. According to the passage, the land transportation system would be replaced by
(a ) air transportation system
(b ) water transportation system
(c ) freight container services
(d ) None of these
5. According to the passage, if a land bridge system were feasible, it would
1. increase foreign trade.
2. decrease the amount of air freight.
3. create a new market for steamship lines and railroad.
Select the correct answer using the codes given below
(a) 1 and 2 (b) 1 and 3 (c) Only 3 (d) All of these
6. The author implies that which of the following would be provided employment by the
development of a land bridge?
1. Dock workers.
2. US sailors.
3. US railway workers.
Select the correct answer using the codes given below
(a) Only 1 (b) Only 2 (c) 2 and 3 (d) All of these
7. The passage states that a land bridge would improve United States
(a ) balance of payment
(b ) international relations
(c ) railroad industry
(d ) foreign trade
114 Practice Set 5

Directions (Q. Nos. 8-10) Examine the pie chart and graph carefully and answer the questions
that follow.
Portion of wage earners engaged in Yearly income of the city’s
various occupations in a city manufacturing company

Professional 60
50

Income ( ` in crore)
50
20°
40
Personal
services 120° Manufacturing 30 30
100° 30
20
20
10 12
90° 30°
0
Commerce Transportation 1995 2000 2005 2010 2015
Years

8. What was the average annual income of the company from 2000 to 2005?
(a) ` 15 crore (b) ` 16 crore (c) ` 25 crore (d) ` 30 crore
9. If 1980 workers were engaged in Commerce, then how many were engaged in Manufacturing?
(a) 1485 (b) 1782 (c) 2200 (d) 2640
10. The average income of Professional workers was 50% greater than that of the Transportation
workers. If the total income of the Transportation workers was ` 23760000, then what was the
total income of the Professional workers?
(a) ` 10560000 (b) ` 23760000 (c) ` 31680000 (d) ` 35640000
11. Pipes P and Q can fill a tank in 36 min and 90 min, respectively. Pipe R can empty it at 3 L / min.
All pipes were opened simultaneously. They filled the tank in 45 min. Find its capacity (in L).
(a) 150 (b) 270 (c) 225 (d) 180
12. Amit goes from city A to city B by car at 60 km/h and after that his car had some problem. So,
he left his car at his friend’s home in city B and came back by bus at 40 km/h. If the whole
journey took 5 h, then the distance between cities A and B (in km) is
(a) 120 (b) 150 (c) 90 (d) 80
13. Five bells toll at regular intervals of 10, 15, 20, 25 and 30 s, respectively. If they toll together at
8:00 am. Then, at what time will they toll together for the first after 8:00 am?
(a) 8:04 pm (b) 8:08 am (c) 8:05 am (d) 8:07 am
14. Sonu and Ramu can do a job in 60 and 40 days, respectively. They earned ` 450 for completing
work. What is the share of Sonu?
(a) ` 270 (b) ` 225 (c) ` 180 (d) ` 250
15. A water tank has the dimensions of 22 m × 10 m × 7 m. A pipe of radius 7 cm is connected to it.
If water is flowing through the pipe at the rate of 100 m/min, then the tank will be completely
filled by the pipe in
(a) 500 min (b) 1000 min (c) 800 min (d) 1200 min
Stage 2 Improve your Weaker Areas 115

Directions (Q. Nos. 16-22) Read the following two passages and answer the questions that
follow each passage. Your answers to these questions should be based on the passages only.

B Passage 1
Deficiencies in allocation in financial market are ignored by most traditional financial-market analysis
because of analysts’ inherent preferences for the simple model of perfect completion. Conventional
financial analysis pays limited attention to issue of market structures and dynamics, relative costs of
information and problems of income distribution. Market participants are viewed as acting as entirely
independent and homogeneous individuals with perfect foresight about capital-market behaviour.
Also, it is assumed that each individual in the community at large has the same access to the market and
the same opportunity to transact and to express the preference appropriate to his or her individual
interest. Moreover, it is assumed that transaction costs for various types of financial instruments
(stocks, bonds, etc.) are equally known and equally divided among all community members.
16. The passage states that traditional studies of the financial market overlook imbalances in the
allocation of financial resources because
1. an optimum allocation of resources is the final result of competition among participants.
2. those performing the studies choose an over simplified description of the influences on
competition.
3. the analysts who study the market are unwilling to accept criticism of their methods as biased.
4. such imbalances do not appear in the statistics usually compiled to measure the market’s
behaviour.
Select the correct answer using the codes given below
(a) Only 1 (b) Only 2 (c) 3 and 4 (d) Only 4
17. A difference in which of the following would be an example of inequality in transaction costs
as alluded to in the last lines of the paragraph.
(a ) Maximum amounts of loans extended by a bank to businesses in different areas.
(b ) Fees charged to large and small investors for purchasing stocks.
(c ) Prices of similar goods offered in large and small stores in an area.
(d ) Exchange rates in dollars for currencies of different countries.
18. According to the passage, analysts have conventionally tended to view those who participate
in financial markets as
1. basing judgements about events mainly on market structure.
2. varying in market power with respect to one another.
3. having equal opportunities to engage in the transactions.
Select the correct answer using the codes given below
(a) Only 1 (b) 1 and 2 (c) Only 2 (d) Only 3
19. The author’s main point is argued by
1. showing that the view opposite to the author is self-contradictory.
2. criticising the pre-suppositions of a proposed plan.
3. showing that omissions in a theoretical description make it inapplicable in certain cases.
Select the correct answer using the codes given below
(a) 1 and 2 (b) Only 2 (c) 2 and 3 (d) Only 3
116 Practice Set 5

B Passage 2
Political theories have, in fact, very little more to do with musical creation than electronics theories
have. Both merely determine methods of distribution. The exploitation of these methods is subject to
political regulation and is quite rigidly regulated in many countries.
Political party-influence on music is just censorship and performances can be forbidden and composers
disciplined for what they write, but the creative stimulus come from elsewhere. Nothing really ‘inspires’
an author but money or food or love. That persons or parties subventioning musical uses should wish to
retain veto power over the works used is not at all surprising. That our political masters (or our
representative) should exercise a certain negative authority, a censorship, over the exploitation of works
whose content they consider dangerous to public welfare is also in no way novel or surprising. Our
musical civilisation is older than any political party. We can deal on terms of intellectual equality with
engineers, architects, poets, painters and historians. We cannot be expected to take very seriously the
inspirational dictates of persons or of groups who think they can pay us to get emotional about ideas.
They can pay us to get emotional all right. Anybody can.
Nothing is as emotion-producing as money. But emotions are factual; they are not generated by ideas.
On the contrary, ideas are generated by emotions. To have any inspirational quality there must be
present facts or immediate anticipations, not pie-in-the-sky. Now, pie-in-the-sky has its virtues as a
political ideal, I presume. Certainly most men want to work for an eventual common good. It is
notorious that musical description of hell, which is something we can all imagine, are more varied and
vigorous than the placid banalities that even the best composers have used to describe heaven and that
all composers do better on really present matters than on either, matters like love and hatred and
hunting and war and dancing around and around.
20. The author implies that political doctrines usually fail to generate artistic creativity because
they are too
(a) abstract (b) rigidly controlled (c) concrete (d) ambiguous
21. The author is making a statement defending
1. intellectual freedom.
2. emotional honesty.
3. the apolitical stance of most musicians.
Select the correct answer using the codes given below
(a) Only 1 (b) Only 2 (c) 1 and 2 (d) All of these
22. The author’s reaction to political influence on music is one of
(a) deference (b) surprise (c) disbelief (d) resignation
23. Given below is a map to reach G from A via 5 cities. The number against each route denotes
the cost incurred between two cities. e.g. Travel between cities B and E costs ` 4. What would
be the least cost (in `) incurred to reach G from A?
4
B E
2 5
3
6 6
A D G
2
3 C F 4
5
(a ) 8 (b) 11 (c) 12 (d) 14
Stage 2 Improve your Weaker Areas 117

24. Examine the information given below.


˜Some engineers are bureaucrats. No bureaucrat is rich.
Which of the following is a valid conclusion regarding the above arguments?
(a) No engineer is rich (b) No bureaucrat is engineer
(c) Some engineers are rich (d) Some bureaucrats are engineers
25. Read the following passage carefully and answer the question based on it.
It is well known that the world urgently needs adequate distribution of food, so that everyone
gets enough. Adequate distribution of medicine is just as urgent. Medical expertise and
medical supplies need to be redistributed throughout the world so that people in emerging
nations will have proper medical care. This paragraph best supports the statement that
(a ) the majority of the people in the world has never been seen by a doctor
(b ) food production in emerging nations has slowed during the past several years
(c ) many people who live in emerging nations are not receiving proper medical care
(d ) most of the world’s doctors are selfish about giving time and money to the poor
26. A father has three sons. He distributes some sweets and is left with 12 sweets. The number of
sweets each of them had initially was thrice the number of sweets eaten by them individually.
Find the initial number of sweets with the father.
(a) 72 (b) 48 (c) 108 (d) 54
27. In a club of 240 members, 122 people play cricket, 118 people play football and 40 people play
both games. How many people play atmost one of the two games?
(a) 122 (b) 200 (c) 118 (d) 240
28. Two articles are sold for ` 13860 each. There is loss of 10% on one article and a profit of 10%
on the other. What is the overall profit or loss (in `)?
(a ) 0 (b) 138.60 (c) 280 (d) 272.20
29. Examine the following statement.
˜Poverty is increasing because the people, who are deciding how to tackle it, know
absolutely nothing about the poor.
Which of the following courses of action logically follows for pursuing?
(a ) The decision makers should go to the grass root levels
(b ) The decision makers should come from the poorer sections of the society
(c ) A new set of decision makers should replace the existing ones
(d ) All of the above
30. The elements of the problem figures given below are changing with a certain rule as we
observe them from left to right.

According to this rule, which of the following would be the next figure, if the changes were
continued with the same rule?

(a) (b) (c) (d)


118 Practice Set 5

Directions (Q. Nos. 31-34) Read the following passage and answer the questions that follow.
Your answers to these questions should be based on the passage only.

B Passage
Colonialism was hated for its explicit assumption that the civilisations of colonised people were inferior.
This orientation was particularly blatant among the French. In their colonies, business was conducted in
French. Schools used that language and employed curriculum designed for children in France. In the
Metropole, intellectuals discoursed on the weakness of Arabic-Islamic culture. A noted historian accused
Islam of being hostile to science. There was of course an element of truth in the criticism. After all, Arab
reformists had been engaging in self-criticism for decades.
The reaction of colonised was of course to defend their identity and to label colonial policy, ‘cultural
asphyxia’. Throughout North Africa, nationalist made the defence of Arabic-Islamic civilisation a major
objective, a value in whose name they demanded independence, yet the crisis of identity, provoked by
colonial experiences, has not been readily assured and lingers into the post-colonial period. Factors
producing militant and romantic cultural nationalism are anchored in time.
Memories of colonialism are already beginning to fade and which the Maghreb has had a few decades in
which to grow dislocations associated with social change can also be expected to be fewer. A
preoccupation with identity and culture and an affirmation of Arabism and Islam have characterised the
Maghreb, since independence these still remain important elements in North African life.
A second great preoccupation in independent North Africa is the promotion of a modernist social
revolution. The countries of the Maghreb do not pursue development in the same way and there have
been variations in policies within each country but all three spend heavily on development.
A problem, however, is that such advances are not always compatible with objectives flowing from North
African nationalism. In Morocco, for instance when the government decided to give children an ‘‘Arab’’
education, it was forced to limit enrolments because among other things, most Moroccans had been
educated in French and the country consequently had few teachers qualified to teach in Arabic. Two
years later, with literacy rate declining, this part of the Arabisation programme was postponed.
31. The author provides information that would answer which of the following questions?
(a ) When did colonialism end in North Africa?
(b ) Why did Europeans impose their way of life on their colonies?
(c ) Why was colonialism bad?
(d ) Why was colonialism disliked?
32. Which of the following does the author mention as evidence of cultural colonialism?
(a ) Native children in North Africa learned little about local culture
(b ) Science was not taught in the Arabic language
(c ) Colonialists spent little on development
(d ) Colonial policy was determined in France
33. Which of the following titles best describes the content of the passage?
1. Culture and Language.
2. Nationalism in North Africa.
Select the correct answer using the codes given below
(a) Only 1 (b) Only 2
(c) Both 1 and 2 (d) None of these
Stage 2 Improve your Weaker Areas 119

34. Which of the following is not used to present the author’s arguments?
1. Colonialism produced an identity crisis.
2. Decolonisation does not always run smoothly.
3. Colonialist assumed that local cultures were inferior.
4. Cultural nationalism will soon disappear.
Select the correct answer using the codes given below
(a) Only 1 (b) Only 2 (c) 1 and 3 (d) Only 4

Directions (Q. Nos. 35-37) Examine the information given below and answer the questions that
follow.
There are seven students in a class A, B, C, D, E, F and G. They sit on three benches 1, 2 and 3, such
that atleast two students are on each bench and atleast one girl and one boy on each bench. C, who is
a girl student, does not sit with A, E and D. F, the boy student, sits with only B. A sits on the bench 1
with his best friends. G sits on the bench 3. E is the brother of C.
35. How many girls are there, out of these seven students?
(a ) 3 (b) 3 or 4 (c ) 4 (d ) 2
36. Who sits with C?
(a ) B (b ) D (c ) G (d ) E
37. On which bench, there may be two boys?
(a) Bench 2 or 3 (b) Bench 3 (c) Bench 2 (d) Bench 1

Directions (Q. Nos. 38-40) Examine the information carefully and answer the questions that follow.
A company having 450 employees has sent all its employees for training in one or more areas out of
HRM, computer skills and financial skills. The employees are classified into two categories: officers
and clerks, who are in the ratio 4 : 5. 10% of the officers take training only in computer skills, 16% of
the clerks take training only in HRM, which is equal to the number of officers taking training only in
financial skills and 50% of the number of officers taking training in HRM and financial skills both.
6% of the total employees take training in all the three, of which two-thirds are officers. 10% of the
total employees take training in HRM and computer skills both, which is five times the number of
clerks taking training in computer skills and financial skills. 10% of the clerks take training in HRM
and computer skills both. The number of officers taking training only in HRM is 25% of the number
of clerks taking training only in HRM. 20% of the total number of employees take training only in
computer skills. Number of clerks taking training in HRM and financial skills both is 20% of the
total number of clerks.
38. How many clerks take training in computer skills but not in HRM?
(a) 113 (b) 70 (c) 88 (d) 79
39. How many employees take training in financial skills but not in HRM?
(a) 106 (b) 135 (c) 162 (d) 134
40. What per cent of the total number of officers take training in computer skills but not in
financial skills?
(a) 11% (b) 40% (c) 25% (d) 15%
41. Manish has 7 relatives, 3 men and 4 women and his wife also has 7 relatives, 4 men and
3 women. In how many ways can they invite 3 men and 3 women for dinner, such that 3 are
Manish’s relatives and 3 are his wife’s relatives?
(a) 485 (b) 385 (c) 185 (d) 970
120 Practice Set 5

Directions (Q. Nos. 42-47) Read the following two passages and answer the questions that
follow each passage. Your answers to these questions should be based on the passages only.

B Passage 1
While many points are worth making in an evaluation of the single 6 year presidential term, one of the
most telling points against the single term has not been advanced. This kind of constitutional limitation
on elections is generally a product of systems with weak or non-existence political parties. Since there is no
party continuity or corporate party integrity in such systems, there is no basis for putting trust in the
desire for re-election as a safeguard against mismanagement in the executive branch. Better under those
conditions to operate on the basis of negative assumptions against incumbents. I do not know if the
earliest proposal for a single, non-repeatable term was made in the 1820s because that was a period of
severely weak political parties. But I do feel confident that this is a major reason, if not the only reason,
that such a proposal has been popular since the 1940s. Though the association of the non-repeatable
election with weak political parties is not in itself an argument against the limitation, the fallout from this
association does contribute significantly to the negative argument. Single-term limitations are strongly
associated with corruption.
In any weak party system, including the presidential system, the onus of making deals and compromises,
both shady and honourable, rests heavily upon individual candidates. Without some semblance of
corporate integrity in a party, individual candidates have few opportunities to amortise their obligations
across the spectrum of elective and appointive jobs and policy proposals. The deals tend to be
personalised and the payoffs come home to roost accordingly. If that situation is already endemic in
conditions of weak or non-existent parties, adding to it the limitation against re-election means that
candidates and officials, already prevented from amortising their deals across space, are also unable to
amortise their obligations temporally. This makes for a highly beleaguered situation. The single 6 year
term for President is an effort to compensate for the absence of a viable party system, but it is a
compensation ultimately paid for by further weakening the party system itself.
Observers, especially foreign observers, have often noted that one source of weakness in American
political parties is the certainity of election every 2 or 4 years, not only because any artificial limitation
on elections is a violation of democratic principles, but also because when elections are set in a certain
and unchangeable cycle, political parties do not have to remain alert, but can disappear into inactivity
until a known point prior to the next election. To rigidify matters by going beyond the determinacy of
the electoral cycle to add an absolute rule of one term would hang still another milestone around the
neck of already doddering political parties.
42. Consider the following statements
According to the author which is true of a political system with weak political parties?
1. Politicians appoint unqualified people to important posts.
2. Political parties favour frequent election.
3. Political bargains are made by individual candidates.
Which of the statements given above is/are correct?
(a) 1 and 2 (b) 2 and 3 (c) only 3 (d) All of these
43. Suppose that America adopted a single-term political system. Considering the foreign observers
mentioned in the passage. How would they be expected to respond to such a development?
(a) They would endorse it because it further strengths American democracy.
(b) They would condemn it because it further limits American democracy.
(c) They would neither endorse nor condemn it.
(d) They would condemn it because it gives the President too much power.
Stage 2 Improve your Weaker Areas 121

44. Which of the following if true would most weaken the authors claim about single-term
political systems?
1. The discovery that foreign observers like this system.
2. The discovery that most politicians are honest.
3. The discovery that parliamentary systems are more democratic.
Select the correct answer using the codes given below
(a) 1 and 2 (b) 2 and 3 (c) only 2 (d) All of these

B Passage 2
It is true that the working of liberal democracy has almost universally been characterised by a gap
between its promises and its delivery. The promises made to ‘‘we the people’’ in idealistic constitutions
drafted in grand constitutional assemblies, made up of the most highly educated elites, have been denied
to the people in real life. We have now reached a stage where about 80 individuals own half the world’s
wealth and they have acquired it largely in the most advanced democratic set-ups. Yet, the option of
delivering the promises made by the same liberal democratic constitutions is opening up. Several
experiments in Latin American states and societies, in Iceland and most recently, in Greece have
demonstrated the feasibility of looking after the interests of the poor and the downtrodden— and of the
ecology— within the available framework of ‘democracy’.
In other words, the ideology that has guided these experiments is redefining Marxism itself by drawing it
away from the premise of irreconcilable class antagonism. The objective here is not to do away with the
capitalist system but to compel it to fulfil the promises made by the ‘‘bourgeois’’ constitutions. Honesty of
purpose seems to have replaced the ideological commitment to class antagonism. This appears to be the
guiding perspective of the AAP. But, then perspectives do not evolve through deliberations among
intellectuals. They evolve through interactions with the people and a deliberate as well as intuitive
understanding of their problems and aspirations. It would be interesting to investigate how many
epoch-making leaders, like Mahatma Gandhi and Nelson Mandela,developed their perspectives by
reading learned treatises. As recent exciting experiments in direct democracy. In Iceland and earlier
Venezuela, have demonstrated perspectives traverse many diverse and complex terrains as they evolve.
Will this experiment evaporate into thin air? There is no guarantee that it will survive the expected
onslaught of vested interests. But there is no guarantee that it will succumb to threats or temptations
held out by vested interests either. The AAP has always laid stress on the sincerity of intentions and
shown no sign of wavering from them, although it is too early to decide and power has the habit of
corrupting.
45. Consider the following statements
1. Experiments in the recent times in democracy are redefining marxism by drawing it away from
the premise of irreconcilable class antagonism.
2. Advanced democracies have failed to redistribute the wealth among the people.
Which of the statement(s) given above is/are incorrect?
(a) Only 1 (b) Only 2 (c) Both 1 and 2 (d) Neither 1 nor 2
46. Which of the following statements the author is most likely to agree with?
1. Perspectives in direct democracy are traversing across varied spectrum.
2. It is the sincerity of intentions and the strength of conviction that will determine the success or
failure of the AAP.
Select the correct answer using the codes given below
(a) Only 1 (b) Only 2 (c) Both 1 and 2 (d) Neither 1 nor 2
122 Practice Set 5

47. The author’s attitude towards the evolving political paradigm in democracy is,
(a) critical (b) indifferent (c) of cautions optimism (d) None of these
48. How many litres of water should be added to 60 L of acidic solution, so that the acid
concentration reduces from 60% to 40%?
(a) 15 L (b) 20 L (c) 30 L (d) 60 L
49. What digit will come in place of question mark?
5 8

3 28 2 8 42 ?

4 4

(a ) 3 (b ) 9 (c ) 1 (d ) 2

Direction (Q. No. 50) Read the passage carefully and answer the question given below it.
Mathematics allows us to expand our consciousness. Mathematics tells us about economic trends,
patterns of disease and the growth of populations. Mathematics is good at exposing the truth but it
can also perpetuate misunderstandings and untruths. Figures have the power to mislead people.
50. This paragraph best supports the statement that
(a ) the study of Mathematics is dangerous
(b ) words are more truthful than figures
(c ) the study of Mathematics is more important than other disciplines
(d ) figures are sometimes used to deceive people
51. Amit and Sumit start from Delhi to Kanpur at 40 km/h and 60 km/h. If Sumit reaches Kanpur
4 h earlier than Amit, then the distance between Delhi and Kanpur (in km) is
(a) 720 (b) 360 (c) 640 (d) 480
52. A zookeeper counted the heads of the animals in a zoo and found it is 60. When he counted the
legs of the animals he found it is 200. If the zoo had either peacocks or tigers, then how many
tigers were there in the zoo?
(a) 30 (b) 45 (c) 20 (d) 40
53. A group of workers can do a piece of work in 20 days. But due to some reason, 8 of them went
absent and hence it took 20 days longer to complete the work. How many people actually
worked on the job to complete it?
(a) 18 (b) 16 (c) 24 (d) 20
54. If the ratio of difference between simple interest and compound interest at a certain rate of
interest for 2 yr and 3 yr is 16 : 5, then the rate of interest is
(a) 15% (b) 20% (c) 10% (d) 12%
55. A goat is tied to one corner of a field of dimensions 16 m × 10 m with a rope of 7 m long. Find
the area of the field that the goat can graze.
(a) 77 m 2 (b) 38.5 m 2 (c) 49 m 2 (d) 58.5 m 2
56. A box contains 5 brown and 4 white socks. A man takes out two socks. The probability that
they are of the same colour is
(a) 5/108 (b) 1/6 (c) 5/18 (d) 4/9
Stage 2 Improve your Weaker Areas 123

Directions (Q. Nos. 57-60) Read the following passage and answer the questions that follow.
Your answers to these questions should be based on the passages only.

B Passage
Just as no two individuals have the same fingerprint, no two people have identical theories on how to
achieve a goal or to live life from day-to-day. Some people follow religious outlines to aspire to a level of
moral excellence, while others employ other approaches. Towards the end of the 19th century and on
through the mid-20th century, a movement called existentialism was born a kind of ‘philosophical
theory of life’. Existentialism is not a philosophy, but instead has been likened as a label for several
widely different revolts against traditional philosophy. Existentialism rejects traditional ethical
endeavors, which is contrary to the tenants of many philosophers prior to the existentialism movement.
Philosophers, since the time of Aristotle have held that most people aim for a common peak of ethical
achievement.
Aristotle argued for the existence of a divine being, described as the Prime Mover, who is responsible for
the unity and purposefulness of nature. In order for humanity to attain such a climax, everyone must
imitate the Almighty’s perfect profile and strive to mimic his actions. Aristotle’s basic philosophy
deduces that humanity strives for an identical peak of moral excellence, as judged by a higher entity.
Existentialism declares that the individual must choose his way. Most importantly, there is no
predetermination. Since, the universe is meaningless and absurd, people must set their own ethical
standards. The universe does not predetermine moral rules.
Each person strives toward a unique moral perfection. The 19th century Danish philosopher Soren
Kierkegaard, who was the first writer to call himself existential, reacted against tradition by insisting
that the highest good for an individual is to find his uniqueness. Kierkegaard’s journal reads, ‘I must find
a truth that is true for me ... the idea for which I can live or die’. Existentialists believe that morality
depends on the individual, rather than a supreme being. Existentialism asserts that people do not have a
fixed nature, as other animals and plants do. Our choices determine who we are.
The 20th century French philosopher Jean Paul Sartre proclaimed that the most important choices that
we make are those that directly affect ourselves. Each character makes choices that create his nature.
Existence suggests freedom where mankind is open to a future that is determined by choice and action.
Choice is inescapable and central to human existence; the refusal to choose is a choice.
57. The author’s primary purpose of writing this passage is which of the following?
1. To explain a few tenants of the existential movement and to enumerate a few of the most
prominent existentialists.
2. To downplay existentialism and support classical divine philosophy.
3. To describe the shortcomings of Aristotle’s philosophies.
4. To explain existentialism and to discuss the philosophies of several prominent living
existentialists.
Select the correct answer using the codes given below
(a) Only 1 (b) 2 and 3 (c) Only 3 (d) Only 4
58. The author implies which of the following?
(a) Existentialism will not succeed because Aristotle’s philosophies are too deeply rooted into
today’s cultures
(b) Goals are attainable only if a person has a personal philosophy about life
(c) A peak of ethical achievement is attainable
(d) The universe is unpredictable and existentialism permits for humans to find meaning and
purpose in everyday activities
124 Practice Set 5

59. Which of the following is explicitly stated in the passage?


1. Existentialism is a philosophy.
2. The Prime Mover is Aristotle’s embodiment of a supreme being.
3. Soren Kierkegaard was well read and studied Aristotle’s philosophies.
Select the correct answer using the codes given below
(a) Only 1 (b) Only 2 (c) Only 3 (d) None of these
60. What is the purpose of writing the first sentence of the passage?
(a) To explain that we are all unique
(b) To explain that there are no two people who will ever have identical theories on how to set goals
(c) To explain that living life from day-to-day is difficult
(d) To introduce the concept of individualism, which later will help refute the idea that a common
philosophy cannot be applicable to everybody
61. Examine the following statement.
Should people with educational qualification higher than the optimum requirements be
˜

debarred from seeking jobs?


Four arguments numbered 1, 2, 3 and 4 are given. On the basis of the information, answer the
question.
Arguments
1. No. It will further aggravate the problem of educated unemployment.
2. Yes. It creates complexes among employees and affects the work adversely.
3. No. This goes against the basic rights of the individuals.
4. Yes. This will increase productivity.
(a) 1 and 3 are strong (b) All are strong (c) 2 and 4 are strong (d) Only 3 is strong
62. Two men start from a point in opposite directions at speeds of 1.2 m/s and 1.5 m/s
simultaneously. Find the distance between them after 10 s.
(a) 27 m (b) 75 m (c) 45 m (d) 65 m
63. Three dogs, Tom, Dick and Jerry, have 40 loaves of bread amongst them. If it is known that,
Jerry has as many as four times the number of loaves with Tom and also, 5 more than that of
Dick. The number of loaves with Dick is
(a ) 5 (b) 15 (c) 20 (d) 12
64. One fine morning, Rishu started to walk in such a way that his back is towards the Sun (his face
is opposite to the Sun). After covering some distance, he turned to left and then again to the left
and then after covering some distance, again to his left. Now, in which direction, is he facing?
(a) South (b) West (c) East (d) North

Directions (Q. Nos. 65-69) Read the following information carefully to answer the given
questions.
Six films P, Q, R, S, T and U are to be released on consecutive Friday. The schedule of the release is
to be in accordance with the following conditions.
Â
P must be released a week before T.
Â
R must not be released immediately after the first release.
Â
Q must be released on Friday following the Friday on which U is released.
Â
S must be released on fifth Friday and should not be immediately preceded by Q.
Â
T must not be released in the last.
Stage 2 Improve your Weaker Areas 125

65. Which of the following films preceded T?


(a ) P (b ) Q (c ) S (d ) U
66. Which of the following films is released immediately after Q?
(a ) P (b ) R (c ) T (d ) U
67. Film R cannot be released on which of the following Fridays in addition to second Friday?
(a) First (b) Third (c) Fourth (d) Fifth
68. In between which of the two films S is to be released?
(a) Q and T (b) R and T (c) P and T (d) T and U
69. Which of the following films released first?
(a ) P (b ) Q (c ) R (d ) U
70. Examine the information given below.
• ‘A * B’ means ‘A is the sister of B’. • ‘A # B’ means ‘B is the mother of A’.
• ‘A + B’ means ‘A is the father of B’. • ‘A = B’ means ‘B is the brother of A’.
Which of the following means ‘M is the maternal uncle of N’?
(a ) M = P + Q * N (b ) N + P = Q * M
(c ) N * P # Q = M (d ) M + Q = P # N
71. Select the missing numbers/ letters from the given responses.

A
iii K
J
ii i B
K
N M L
L
D C
M

(a) (i—O), (ii—E), (iii—P) (b) (i—N), (ii—E), (iii—O)


(c) (i—M), (ii—E), (iii—N) (d) (i—P), (ii—F), (iii—Q)

Direction (Q. No. 72) Examine the information carefully and answer the following question.
In the following diagram, three classes of population are represented by three figures. The triangle
represents the school teachers, the square represents the married persons and the circle represents
the persons living in joint families.

E
B C
D

72. Married persons living in joint families but not working as school teachers are represented by
(a ) C (b ) F (c ) D (d ) A
126 Practice Set 5

Directions (Q. Nos. 73-77) The following questions are based on two passages in English to test
the comprehension of English language and therefore, these questions do not have Hindi version. Read
each passage and answer the questions that follow.

B Passage 1
Philosophical logic deals with formal descriptions of natural language. Most philosophers assume that
the bulk of ‘normal’ proper reasoning can be captured by logic, if one can find the right method for
translating ordinary language into that logic. Philosophical logic is essentially a continuation of the
traditional discipline that was called ‘logic’ before the invention of mathematical logic. Philosophical
logic has a much greater concern with the connection between natural language and logic.
As a result, philosophical logicians have contributed a great deal to the development of non-standard
logics (e.g. free logics, tense logics) as well as various extensions of classical logics (e.g. modal logics), and
non-standard semantics for such logics. Logic and the philosophy of language are closely related.
Philosophy of language has to do with the study of how our language engages and interacts with our
thinking. Logic has an immediate impact on other areas of study.
Studying logic and the relationship between logic and ordinary speech can help a person better structure
their own arguments and critique the arguments of others. Many popular arguments are filled with
errors because so many people are untrained in logic and unware of how to correctly formulate an
argument.
73. What is the central idea of this passage?
(a ) The significance and application of philosophy in the study of logic
(b ) The history of logic and various types of their applications
(c ) Studying logic and philosophy together to get the maximum out of both subjects
(d ) The evolution of the branch of logic called philosophical logic and its significance
74. Which of the following options best summarises the passage given above?
(a) The philosophy of language should be studied alongside logic to discover logical errors in speech
and argument.
(b) Philosophical logic was invented to create logical patterns in normal speech. The study of
philosophical logic helps one to formulate argument better.
(c) Philosophical logic is the continuation of what was called ‘logic’ before the advent of
mathematical logic and deals with formal description of normal language.
(d) Philosophical logic has contributed a great deal towards development of non-standard logic like
tense logic, free logic, etc.

B Passage 2
India’s under-development was not traditional or inherited from the pre-colonial past. India of the
eighteenth century was undeveloped and not under-developed. On a world scale, it was not less but
perhaps more developed than many other national economies, as most of the world development has
occurred after the eithteenth century and basically after 1850.
The basic feature of colonialism in India in its long history since the 1750s was the appropriation by
Britain of the social surplus produced in India. There were of course, many changes and some of them
were positive for example, the development of the railways-when seen is isolation. But these changes
came within and as part of the process of under-development. Colonialism is best seen as a totality or a
unified structure.
Stage 2 Improve your Weaker Areas 127

75. Which of the following is the basic feature of colonialism?


(a) Interest on public debt (b) Unequal exchange
(c) Appropriation of social surplus (d) Expropriation of the industrial profit
76. Why 18th century India could not be called under-developed?
(a ) World development occurred after 18th century
(b ) India did not lag behind other nations
(c ) Most other nations developed after 1850
(d ) Indian Economy was comparable with contemporary Europe and Japan
77. Why was India’s under-development not traditional?
(a ) India in the 18th century was undeveloped
(b ) India did not inherit under-development from the past
(c ) India in the 18th century was not under-developed
(d ) India was not a poor nation

Directions (Q. Nos. 78-80) Given below are three questions. Each question describes a situation
and is followed by four possible responses. Indicate the response you find most appropriate. Choose
only one response for each question. The responses will be evaluated based on the level of
appropriateness for the given situation. Please attempt all questions. There is no penalty for wrong
answers for these three questions.

78. You are working in a government department. Your boss gives a piece of work and orders you to
follow his instructions to complete the work but the work is difficult to finish within time. You
will
(a ) persuade him to extend the time
(b ) start the work and try to finish it within the time limit, even working on holidays
(c ) start the work, try to finish it and if you cannot, then you ask your boss for time extension
(d ) take the help of other colleagues in finishing the work
79. While on patrol duty, you come across a scene of eve teasing and molestation. When you go
near to the scene, you find that the molester is the son of a local MLA, who is a law abiding and
a very social person. You will
(a) enquire about the situation and carryout further investigations after ensuring the safety of the
girl
(b) ignore the situation as the molester is the son of a law abiding MLA
(c) decide to let him go after giving him a warning as he is the son of a law abiding and dutiful
person
(d) call up your senior officer; narrate the situation and ask him what should be done
80. You are the police commissioner of a state. Honour killing is very common in your state.
Honour killing is the killing of sons or daughters, who get married in other clan due to the fear
of the Panchayats and to save their honour. One or more such cases are reported everyday.
What will you do to control such situation?
(a) Launch a campaign to open the minds of the people and to make them realise that honour
killing is a crime
(b) Order the police to arrest all those persons who are involved in such heinous crimes
(c) Talk to the people directly and warn them not to get involved in honour killing
(d) Give protection to the people who get married in other clan
Space for Rough Work
Stage 2 Improve your Weaker Areas

Answer with Explanations


1. (b) India has emerged as a new key player as far as dealing 9. (d) Let the number of workers engaged in Manufacturing be x.
with climate change issues in concerned. Moreover a new 90°
Then, = 1980 …(i)
era of economic growth and development is sought while 360°
dealing with domestic issues related to climate changes 120°
and =x …(ii)
such as use of biofuels, clean energy sector revival and 360°
capitulation of solar energy sector. On dividing Eq. (i) by Eq. (ii), we get
90° 1980
2. (c) Cleanliness should be everybody’s endeavour. India can =
120° x
be a clean country only when people of all strata join 1980 × 120
together in this cleanliness drive and do not allow anyone ∴ x= = 2640
90
to litter at public places, Joint endeavour and awareness,
both are a must to deal with this problem.
10. (b) From the given pie chart,
30°
Number of Transportation workers = of total workers
3. (a) India is a one of the most favoured ground for tourists all 360°
over the world because of diversity of culture and beautiful 20°
and number of Professional workers = of total workers
but varied land scapes and gregarious tropical forests. 360°
Focus on this industry can certainly speed up economic Thus, it is clear that there are 50% more Transportation
development and bring prosperity to the country. workers than Professional workers.
4. (b) In the last lines of the first paragraph, it is mentioned that a So, if Professional workers = 20
land transportation system would be substituted for Then, Transportation workers = 30
marine transportation during part of the movement of Given, the total income of Transportation workers
goods between Europe and the ‘Far East’. Marine refers to = ` 23760000
sea and hence water transportation is the correct answer. Average income of Transportation workers
23760000
5. (b) 1 and 3 both options are correct. In the second paragraph = = `792000
30
it is mentioned that if a land-bridge system of shipment
were deemed feasible it would open a completely new and average income of Professional workers
150
market for both United States steamship lines and = 792000 ×
100
railroads,which satisfy the statement 3.
= `1188000
In this paragraph it is also mentioned that the land bridge
Hence, total income of Professional workers
will give the US East coast ships an opportunity to
= 1188000 × 20
compete for the trade which American carriers could not
get before and is occupied by foreign lines that carry all = `23760000
Far East -Europe freights which clearly says increase in 11. (d) It is given that, P and Q can fill a tank in 36 min and 90 min.
foreign trade statement 1 is now satisfied but no mention LCM of 36 and 90 = 180
of decrese in the amount of air freight. 180 180
Now, filling rate of P and Q = +
6. (c) This question is developed from an idea implied in the 36 90
passage. Option (c) is correct because it is argued that = 5 + 2 =7 L / min
new jobs will be created for US railway workers and sailors After all pipes are opened, then tank deposits water at the
but not for dock workers. rate = 7 − 3 = 4 L / min

7. (a) In these type of questions, you may be asked about specific It is given that, tank filled up in 45 min.
facts or details the author has stated explicitly in the So, capacity of tank = 45 × 4 = 180 L
passage. It is mentioned in the passage that land-bridge 12. (a) Let the distance between the cities A and B be x km.
system would have a favourable effect on our balance of Then,
x
+
x
=5
payment. 60 40
120
8. (b) Average annual income from 2000 to 2005 ∴ x=5×
5
12 + 20 32
= = = ` 16 crore = 120 km
2 2
132 Practice Set 5

13. (c) LCM of 10, 15, 20, 25 and 30 22. (d) The author does not find censorship surprising, nor does he
take it seriously. He is resigned to attempts at censorship,
3 10, 15, 20, 25, 30
although he does not believe it can inspire creativity.
5 10, 5, 20, 25, 10
23. (b) The possible routes between A and G (along with the
2 2, 1, 4, 5, 2 respective costs in `) are given below
2 1, 1, 2, 5, 1 ABEG : 2 + 4 + 5 = `11
1, 1, 1, 5, 1 ADEG : 6 + 3 + 5 = `14
= 3 × 5 × 2 × 2 × 5 = 300 ADG : 6 + 6 = `12
ADFG : 6 + 2 + 4 = `12
After 300 s, bells toll together.
ACFG : 3 + 5 + 4 = `12
∴ Required time = 8:00 am + 300 s
Thus, the least cost is `11.
= 8:00 am + 5 min = 8:05 am
24. (d) ‘Some engineers are bureaucrats’ can be represented as
14. (c) It is given that, Sonu and Ramu can do a job in 60 and
40 days, respectively.
Engineers Bureaucrats
Thus, the ratio of their wages = 60: 40 = 3:2
It is given that, their total earning is ` 450.
2 ‘No bureaucrat is rich’ can be represented as
∴Share of Sonu = × 450 = `180
(2 + 3)
Bureaucrats Rich
15. (b) Volume of the tank = 22 × 10 × 7 = 1540 m3
Volume of water flowing per minute = Area of cross-section
of the pipe × Rate of flow of water per minute On combining the two, we get
2
22  7  Rich
= ×   × 100 = 1.54 m / min
3
7  100 
So, the tank will be filled in 1540/1.54 = 1000 min Engineers
16. (b) According to the passage, the studies ignore allocation
problems because of analysts’ inherent preferences for the
simple model of perfect completion. So, the correct answer Bureaucrats
is statement 2 i.e. option (b). (i)
Rich
17. (b) It is mentioned that transaction costs are the costs incurred
when buying or selling assets takes place. In this case, the
costs incurred in buying or selling financial instruments
such as stocks. In other words, fees for purchasing stocks Bureaucrats
are transaction costs. Different fees for large and small
investors are an appropriate examples of inequality in
transaction costs. So, the correct answer is option (b). Engineers
Maximum amount of loans, prices of goods and exchange
(ii)
rate are not transaction costs for financial instruments.
So, in fig. (ii), it can be seen that ‘there are some
18. (d) Analysts assume that all market participants have the same engineers who are rich’. So, option (a) is wrong. Also in
access to the market and the same opportunity to engage
both figures, it is clear that ‘there are some bureaucrats,
in transactions. So, the correct answer is statement 3.
who are engineers.’ So, option (b) is invalid. ‘Some
19. (d) The passage mentions that the minority communities are a engineers are rich’ is valid from fig. (ii) but not from fig. (i).
practical exception to the theoretical rule and also explains Hence, option (c) is not a valid conclusion. Option (d) is
why this exception is typically omitted from consideration. valid in both figures.
Option (d) accurately describes the author’s approach.
25. (c) This answer is implied by the statement that redistribution
20. (b) The author implies that political doctrines usually fail to is needed so that people in emerging nations can have
generate artistic creativity because they are too rigidly proper medical care. Options (a) and (d) are not
controlled. Political influence on music is negative like mentioned in the passage. Option (b) is also incorrect as
censoring and disciplining the music which forbids any the passage indicates the distribution of medicine, not its
creativity. production.
21. (c) The author is arguing that musicians will not confirm to any 26. (d) Let the number of sweets initially with the father be x.
control over their creativity. Thus, they want to be x
Then, number of sweets each son initially =
intellectually free and emotionally honest. It does not mean 3
that they could not be active in politics (apolitical).
Stage 2 Improve your Weaker Areas 133

According to the question, Bench 2 → Fb , Bg

= 3  − 12  ⇒ Bench 3 → G
x x x
= x − 36
3 3  3
where, b → boy, g → girl
x 2x
⇒ x − = 36 ⇒ = 36 C is a girl and does not sit with A, E and D. So, she must sit
3 3
on bench 3. Also, E is the brother of C, which means E is a
∴ x = 54 male and he must sit on bench 1. Also, D sits on bench 1 and
27. (b) D should be a girl as atleast one girl and one boy should be
Cricket Football on each bench. So, G should also be a boy. Hence,
Bench 1 → Ab , Eb, Dg
122 40 118 Bench 2 → Fb, Bg
Bench 3 → Gb, Cg
35. (a) The number of girls are 3.
Total number of members = 240 (given) 36. (c) G sits with C.
But 40 members are playing both the games. 37. (d) There may be two boys on bench 1 only.
People who play atmost one of the two games Solutions (Q. Nos. 38-40)
= 240 − 40 = 200
The information given in the question can be represented as
28. (c) Total loss in percentage = x + y − xy Number of officers = 450 ×
4 4
= 450 × = 200
100
(4 + 5) 9
10 × 10
= 10 − 10 − = − 1%
100 Number of clerks = (450 − 200) = 250
Now, SP of two articles = 2 × 13860 = ` 27720 10
Officers in computer skills = × 200 = 20
100 100
and CP of two articles = 27720 × = ` 28000
99 16
Clerks in HRM = × 250 = 40 = Officers in financial skills
∴Loss = 28000 − 27720 = ` 280 100

29. (a) The statement indirectly asserts that the decision makers Clerks in HRM = 50% of officers in HRM and financial skills
can work effectively to eliminate poverty, only if they get to both
100
know the basic problems afflicting the poor people So, officers in HRM and financial skills both = 40 × = 80
through interaction with them. So, only (a) follows. 50

30. (d) Triangle is present in middle and above while circle is Total employees taking training in all three = 6% of 450
changing anti-clockwise. The lines are also changing 6
= × 450 = 27
anti-clockwise. Hence, option (d) is correct. 100
2
31. (d) This theme begins in line one and continues throughout Officers in all three = × 27 = 18
the passage.Colonialism was hated because it assumed 3
the colonised subjects inferior and impose its own culture. So, clerks in all three = ( 27 − 18) = 9
32. (a) The fact that children were taught very little about their own Employees taking training in HRM and computer skills both
culture and history was due to cultural colonialism. It was 10
= 10% of 450 = × 450 = 45
because of the colonised labelled colonial policy, ‘cultural 100
asphyxia’.
Employees in HRM and computer skills both = 5 × Clerks in
33. (b) Clearly, the main subject of the passage is nationalism. computer skills and financial skills both
45
34. (d) Statements 1, 2 and 3 are mentioned in the passage. So, clerks in computer skills and financial skills both = =9
Cultural nationalism is the seed to revive their identity from 5
the clutches of imperial culture. Clerks in HRM and computer skills both = 10% of 250 = 25
Solutions (Q. Nos. 35-37) Officers only in HRM = 25% of clerks only in HRM
25
It is given that, A sits on bench 1 and G sits on bench 3. i.e. = × 40 = 10
100
Bench 1 → A
Clerks in HRM and financial skills both = 20% of 250 = 50
Bench 2 →
Bench 3 → G 38. (b) Number of clerks taking training in computer skills but not
in HRM = 70
F is a boy student and sits with B only. So, they must sit on
bench 2 and B should be a girl. 39. (c) Number of employees taking training in financial skills but
Bench 1 → Ab not in HRM = 74 + 40 + 48 = 162
134 Practice Set 5

40. (a) enard against the onslanght of vested interests which will
be determined by sincerity of intentions.
Computer skills
HRM 47. (c) The author does not criticise the practice of democracy.
2 (O) On the contrary, he takes keen interest in evolving
10 (O) 20 (O)
16 (C)
40 (C) 70 (C) perspectives. Moreover he talks about its various
18 (O) O Officer
Clerks
determinants.
9 (C) C
62 (O) 48 (O) 48. (c)
41 (C) 0 (C) Acidic solution Water
60% 0%

40 (O)
74 (C) 40%

Financial skills
40 20
Required Percentage 40
Officers taking training in computer Ratio = = 2 :1
20
skills but not in financial skills
= × 100 Here, 2 parts = 60 L
Total number of officers
(20 + 2 ) ∴ Amount of water should be added = 1part = 30 L
= × 100 = 11%
200 49. (c) As, (5 × 4) + (3 × 2 ) + 2 = 28
41. (a) 3 women and 3 men can be selected in the following ways Same as, ( 8 × 4 ) + ( 8 × 1) + 2 = 42
Hence, 1 will be come in place of question mark.
Manish’s Manish’s wife’s Number Value
relatives relatives of ways 50. (d) This answer is clearly stated in the last sentence of the
(4 women and (4 men and paragraph. Option (a) can be ruled out because there is
3 men) 3 women) no support to show that studying maths is dangerous.
Words are not mentioned in the passage, which rules out
Case I 3 men 3 women 3
C 3 × 3C 3 1
option (b). Option (d) is a contradiction of the information
Case II 3 women 3 men 4
C3 × C3
4
16 in the passage. There is no support for option (c).
Case III 2 men, 1 woman 2 women, 1 man 4
C 1 × 3C 2 144 51. (d) Distance to be covered by Amit in 4 h = 4 × 40 =160 km
× 3 C 2 × 4C 1 So, Sumit has gained 160 km over Amit at a relative speed
of 20 km/h, so for gaining 160 km, he has travelled for
Case IV 2 women, 1 man 2 men, 1 woman 4
C 2 × 3C 1 324 =160/20 = 8 h
× 4C 2 × 3 C 1 ∴Distance between Delhi and Kanpur = (8 × 60)
= 480 km
∴Total number of ways = 485 52. (d) Let the number of tigers be x and the number of peacocks
42. (c) It was clearly stated in the opening lines of 2nd paragraph be y, then
that in any weak party system including the presidential x + y = 60 ...(i)
system, the onus of making deals and compromises rests and 4x + 2 y = 200 ...(ii)
heavily upon individual candidates. Multiplying Eq. (i) by 2 and then subtracting Eq. (i) from
43. (b) They would condemn it because it will further weaken the Eq. (ii), we get
party system. 4x + 2 y = 200
44. (c) Author’s claim about single-term political system is based − 2 x ± 2 y = − 120
upon the nation that single-term limitations are strongly 2 x = 80
associated with corruption. So if it is discovered that most ⇒ x = 40
politicians are honest, then it will weaken his claim. Now, put the value of x in Eq. (i), we get
45. (d) Experiments in Latin America, Iceland and Greece have 40 + y = 60
been successful in looking after the interests of the poor ⇒ y = 20
and the downtrodden. The last sentence of the first ∴ x = 40 and y = 20
paragraph criticises the advanced democracies for their
failure to redistribute the wealth. 53. (b) Let n people actually worked on the job.
Then, total work = 20n
46. (c) The last sentence of third paragraph talks about the
Also, total work = (n − 8) 40
evolution of different perspectives. The AAP will have to
Stage 2 Improve your Weaker Areas 135

So, (n − 8) 40 = 20 n the performance of the individual that matters and that


⇒ 2 n − 16 = n ⇒ n = 16 makes him more or less wanted and not his educational
qualifications. So, neither 1 nor 2 holds strong.
54. (b) Let the rate of interest be r % per annum and the sum be ` P.
Besides, the needs of a job are laid down in the desired
Then, difference between CI and SI for 2 yr qualifications for the job. So, recruitment of more
 2  Pr 2
=  P  1 +
r  2 qualified people cannot augment productivity.
 − P − P × r × = 4
  100   100 10 Thus, 4 also does not hold strong. However, it is the right
of an individual to get the post for which he fulfills the
Also, difference between CI and SI for 3 yr
eligibility criteria, whatever be his extra merits. Hence,
 3   Pr 2  (300 + r )
=  P  1 +
r  3 Argument 3 holds strong.
 − P − P × r × =  4 
  100   100  10  100
62. (a) Relative speed of two men = 1.5 + 1.2 = 2.7 m/s
 Pr 2  (300 + r )
 4  Distance travelled by two men = 2.7 × 10 = 27 m
 10  100 (300 + r ) 16
Now, = 16 : 5 ⇒ = 63. (b) Let Tom, Dick and Jerry have x, y and z loaves of bread
 Pr 2  100 5
 4  with them respectively, then
 10 
x + y + z = 40, z = 4x and y = z − 5 = 4x − 5
⇒ 1500 + 5 r = 1600 ∴ x + 4x − 5 + 4x = 40
⇒ r = 20% ⇒ 9x = 45 ⇒ x = 5
90 22
55. (b) Area which the goat can graze = × × 7 × 7 = 38.5 m2 So, number of loaves with Dick = 5 × 4 − 5 = 15
360 7
64. (d) N
56. (d) Total number of socks in the box = 5 + 4 = 9
2 socks can be drawn from the box in W E
9
C 2 ways = 36 ways
Number of ways of drawing 2 brown socks = 5C 2 =10 S
Number of ways of drawing 2 white socks = C 2 = 6
4
It is clear from the figure, that he is facing towards North.
∴Probability of drawing 2 socks of the same colour Solution (65-69) Given information can be summarised as
= P( 2 brown socks) + P( 2 white socks) U>Q>P>T>S>R
10 6 16 4 1 2 3 4 5 6
= + = =
36 36 36 9 65. (a) Clearly, the release of P precedes T.
57. (a) The author’s primary purpose is to explain existentialism 66. (a) P is released immediately after Q.
and to support the thoughts of some prominent
existentialists. Statement 2 is against the theme of the 67. (d) R cannot be released on fifth Friday as well because S
passage. Statement 3 cannot be substantiated from the has to be staged on that day.
passage. Statement 4 is partially correct as author has 68. (b) Clearly, S is released between T and R.
discussed the existentialist of nineteenth and twentieth
69. (d) Clearly, U is released first.
century. So, the correct answer is statement 1.
70. (c) N * P means N is the sister of P.
58. (d) Option (a) cannot be concluded from the passage. The use
of the word ‘only’ makes option (b) incorrect. Option (c) is P # Q means Q is the mother of P as well as of N.
not stated in the passage. Author has mentioned that Q = M means Q is the sister of M.
universe is meaningless, and absurd so people must set Hence, M is the maternal uncle of N.
their own ethical standards.
71. (b)
59. (b) Statement 1 is not mentioned in the passage. Statement 3 is ii iii
A
not stated in the passage. The prime mover is Aristotle’s N i J K
divine philosophy. M
D K B
L
60. (d) Author has started with the concept of individualism, which M L
later paved the way for existentialism. C

61. (d) The issue discussed in the statement is nowhere related to


By going through inner circle, it is clear that each letter is
increase in unemployment, as the number of vacancies
increased by its next one.
filled in will remain the same. Also, in a working place, it is
136 Practice Set 5

+1 +1 +1 +1 76. (b) India of 18th century could not be called under-developed,


i.e. J → K → L → M → N
(i) because India did not lag behind other nations, rather it was
Thus, N will come at the place of (i). more developed than many other national economies.
Similarly, in upper circle we see that 77. (a) India’s under-development was not traditional because India
+1 +1 +1 +1 of the 18th century was undeveloped and not
A → B → C → D → E
( ii ) under-developed, it is proved from the fact that India was
more developed than many other economies.
+1 +1 +1 +1
and K → L → M → N → O
( iii ) 78. (c) Asking for extension of time before starting the work will
Thus, (i — N, ii — E, iii — O) prove to be of no use as you cannot know about the time
needed in finishing the work. So, the best way is to start the
72. (c) Married persons living in joint families are represented work and try to finish it within the time limit. In case, you
by the region common to the square and the circle i.e. D cannot complete the work, you can ask for the time
and B. But according to the given condition, the persons extension.
should not be school teachers. So, B is to be excluded.
Hence, the required condition is denoted by region D. 79. (a) As a public servant, it is your duty to enquire into such cases
and take appropriate action, otherwise citizens will stop
73. (d) Option (a) just mentions the significance and application believing you. Option (b) is ethically and morally wrong and
and not what philosophical logic means. Option (b) talks your duty as a police officer does not permit such conduct.
about logic as a whole and not philosophical logic. Option (c) is in violation of your duty and a pretentious reason
Option (c) is a conclusion that can be made after for letting an offender go. Option (d) shows your
reading the passages. So, the options (a), (b) and (c) indecisiveness and inability to handle situation.
are incorrect while only option (d) mentions the central
idea of the passage. Hence, option (d) is correct. 80. (a) The best way to deal with such a situation is to launch a
campaign on a large scale to open the minds and to make
74. (c) The passage summary have two basic points i.e. how them realise that honour killing is a crime. This action will
philosophical logic came into being and what is its have a large scale impact. Changing the mindsets of the
purpose, both of which are explained in the passage. people on a large scale will prose a long-term solution.
Option (a) draws conclusion based on the passage. Options (b) and (c) are not practically feasible to administer
Option (b) mentions both points however the first point absolutely as the situation is rampant. Warning the public
is incorrect while option (d) uses examples that may be felt as going against their traditional social believe
shouldn’t be used in summaries. Option (c) list both which may aggravate the situation and public may not
these points down. Hence, option (c) is correct. cooperate. So, option (a) is the best solution.
75. (c) The basic feature of colonialism in India was the
appropriation by Britain of the social surplus produced
in India.

You might also like